Você está na página 1de 431

Slay the K 2.

0 CFFP

Thanks
Brought to you by:

David
Mark
Anna
Sage
Liam
Anderson

General K Answers

Sokal
The liberal left has lost all meaning in their work it has devolved to pretty
words and flashy philosophical ideas. A physicist was able to get his fake
article into a left leaning journal even though it was pure nonsense.
Sokal 95 Professor of Physics at New York University (Alan, A Physicist Experiments With
Cultural Studies,
http://physics.nyu.edu/faculty/sokal/lingua_franca_v4/lingua_franca_v4.html)//DWB
For some years I've been troubled by an apparent decline in the standards of intellectual rigor in certain precincts of the American
academic humanities. But I'm a mere physicist: if I find myself unable to make head or tail of jouissance and diffrance, perhaps that
just reflects my own inadequacy. So, to test the prevailing intellectual standards, I decided to try

a modest (though admittedly


uncontrolled) experiment: Would a leading North American journal of cultural studies -- whose
editorial collective includes such luminaries as Fredric Jameson and Andrew Ross -- publish an article liberally
salted with nonsense if (a) it sounded good and (b) it flattered the editors' ideological
preconceptions? The answer, unfortunately, is yes. Interested readers can find my article, ``Transgressing
the Boundaries: Toward a Transformative Hermeneutics of Quantum Gravity,'' in the Spring/Summer 1996 issue of Social Text. It
appears in a special number of the magazine devoted to the ``Science Wars.'' What's going on here? Could

the editors
really not have realized that my article was written as a parody? In the first paragraph I deride ``the
dogma imposed by the long post-Enlightenment hegemony over the Western intellectual outlook'': that there exists an
external world, whose properties are independent of any individual human being and
indeed of humanity as a whole; that these properties are encoded in ``eternal'' physical
laws; and that human beings can obtain reliable, albeit imperfect and tentative,
knowledge of these laws by hewing to the ``objective'' procedures and epistemological
strictures prescribed by the (so-called) scientific method. Is it now dogma in Cultural Studies that
there exists no external world? Or that there exists an external world but science obtains no knowledge of it? In the second
paragraph I declare,

without the slightest evidence or argument, that ``physical `reality' [note the

scare quotes] ... is at bottom a social and linguistic construct.'' Not our theories of physical reality, mind you,
but the reality itself. Fair enough: anyone who believes that the laws of physics are mere social conventions is invited to try
transgressing those conventions from the windows of my apartment. (I live on the twenty-first floor.) Throughout the article, I
employ scientific and mathematical concepts in ways that few scientists or mathematicians could possibly take seriously. For
example, I suggest

that the ``morphogenetic field'' -- a bizarre New Age idea due to Rupert Sheldrake -constitutes a cutting-edge theory of quantum gravity. This connection is pure invention; even Sheldrake
makes no such claim. I assert that Lacan's psychoanalytic speculations have been confirmed by
recent work in quantum field theory. Even nonscientist readers might well wonder what in heavens' name quantum
field theory has to do with psychoanalysis; certainly my article gives no reasoned argument to support
such a link. Later in the article I propose that the axiom of equality in mathematical set theory
is somehow analogous to the homonymous concept in feminist politics. In reality, all the axiom of
equality states is that two sets are identical if and only if they have the same elements. Even readers without mathematical training
might well be suspicious of the claim that the axiom of equality reflects set theory's ``nineteenth-century liberal origins.'' In sum, I

intentionally wrote the article so that any competent physicist or mathematician (or
undergraduate physics or math major) would realize that it is a spoof. Evidently the editors of Social Text felt
comfortable publishing an article on quantum physics without bothering to consult anyone knowledgeable in the subject. The
fundamental silliness of my article lies, however, not in its numerous solecisms but in the dubiousness
of its central thesis and of the ``reasoning'' adduced to support it. Basically, I claim that
quantum gravity -- the still-speculative theory of space and time on scales of a millionth of a billionth of a billionth of a

billionth of a centimeter -- has profound political implications (which, of course, are ``progressive''). In support of
this improbable proposition, I proceed as follows: First, I quote some controversial philosophical pronouncements of Heisenberg and
Bohr, and assert

(without argument) that quantum physics is profoundly consonant with


``postmodernist epistemology.'' Next, I assemble a pastiche -- Derrida and general relativity, Lacan and topology,
Irigaray and quantum gravity -- held together by vague rhetoric about ``nonlinearity'', ``flux'' and ``interconnectedness.'' Finally, I

jump (again without argument) to the assertion that ``postmodern science'' has abolished the
concept of objective reality. Nowhere in all of this is there anything resembling a logical sequence of thought; one finds
only citations of authority, plays on words, strained analogies, and bald assertions. In its concluding passages, my article becomes
especially egregious. Having
again without argument) that

abolished reality as a constraint on science, I go on to suggest (once


science, in order to be ``liberatory,'' must be subordinated to

political strategies. I finish the article by observing that ``a liberatory science cannot be complete without a profound
revision of the canon of mathematics.'' We can see hints of an ``emancipatory mathematics,'' I suggest, ``in the multidimensional
and nonlinear logic of fuzzy systems theory; but this approach is still heavily marked by its origins in the crisis of late-capitalist
production relations.'' I add

that ``catastrophe theory, with its dialectical emphases on


smoothness/discontinuity and metamorphosis/unfolding, will indubitably play a major
role in the future mathematics; but much theoretical work remains to be done before this
approach can become a concrete tool of progressive political praxis.'' It's understandable that the
editors of Social Text were unable to evaluate critically the technical aspects of my article (which is exactly why they should have
consulted a scientist). What's more surprising

is how readily they accepted my implication that the


search for truth in science must be subordinated to a political agenda, and how oblivious they
were to the article's overall illogic. Why did I do it? While my method was satirical, my motivation is utterly serious.
What concerns me is the proliferation, not just of nonsense and sloppy thinking per se, but of a
particular kind of nonsense and sloppy thinking: one that denies the existence of
objective realities, or (when challenged) admits their existence but downplays their practical
relevance. At its best, a journal like Social Text raises important questions that no scientist should ignore -- questions, for
example, about how corporate and government funding influence scientific work. Unfortunately, epistemic relativism does little to
further the discussion of these matters. In short, my concern

over the spread of subjectivist thinking is


both intellectual and political. Intellectually, the problem with such doctrines is that they
are false (when not simply meaningless). There is a real world; its properties are not merely social
constructions; facts and evidence do matter. What sane person would contend otherwise? And yet, much
contemporary academic theorizing consists precisely of attempts to blur these obvious truths -- the utter absurdity of it all
being concealed through obscure and pretentious language. Social Text's acceptance of my
article exemplifies the intellectual arrogance of Theory -- meaning postmodernist literary
theory -- carried to its logical extreme. No wonder they didn't bother to consult a physicist. If all is
discourse and ``text,'' then knowledge of the real world is superfluous; even physics
becomes just another branch of Cultural Studies. If, moreover, all is rhetoric and ``language
games,'' then internal logical consistency is superfluous too: a patina of theoretical
sophistication serves equally well. Incomprehensibility becomes a virtue; allusions,
metaphors and puns substitute for evidence and logic. My own article is, if anything, an extremely modest
example of this well-established genre. Politically, I'm angered because most (though not all ) of this silliness
is emanating from the self-proclaimed Left. We're witnessing here a profound historical volte-face. For most of
the past two centuries, the Left has been identified with science and against obscurantism; we have believed that rational thought
and the fearless analysis of objective reality (both natural and social) are incisive tools for combating the mystifications promoted by
The recent turn of many
``progressive'' or ``leftist'' academic humanists and social scientists toward one or
another form of epistemic relativism betrays this worthy heritage and undermines the
already fragile prospects for progressive social critique. Theorizing about ``the social construction of
reality'' won't help us find an effective treatment for AIDS or devise strategies for preventing global warming. Nor can we
combat false ideas in history, sociology, economics and politics if we reject the notions
the powerful -- not to mention being desirable human ends in their own right.

of truth and falsity. The results of my little experiment demonstrate, at the very least, that some fashionable sectors of the
American academic Left have been getting intellectually lazy. The editors of Social Text liked my article because they liked its
conclusion: that ``the content and methodology of postmodern science provide powerful
intellectual support for the progressive political project.'' They apparently felt no need to analyze the
quality of the evidence, the cogency of the arguments, or even the relevance of the arguments to the purported conclusion. Of
course, I'm not oblivious to the ethical issues involved in my rather unorthodox experiment. Professional communities operate
largely on trust; deception undercuts that trust. But it is important to understand exactly what I did. My article is a theoretical essay
based entirely on publicly available sources, all of which I have meticulously footnoted. All works cited are real, and all quotations
are rigorously accurate; none are invented. Now, it's true that the author doesn't believe his own argument. But why should that
matter? The editors' duty as scholars is to judge the validity and interest of ideas, without regard for their provenance. (That is why
many scholarly journals practice blind refereeing.) If the Social Text editors find my arguments convincing, then why should they be
disconcerted simply because I don't? Or are they more deferent to the so-called ``cultural authority of technoscience'' than they
would care to admit? In the end, I

resorted to parody for a simple pragmatic reason. The targets of


my critique have by now become a self-perpetuating academic subculture that typically
ignores (or disdains) reasoned criticism from the outside. In such a situation, a more direct
demonstration of the subculture's intellectual standards was required. But how can one show that the emperor has no clothes?

Satire is by far the best weapon ; and the blow that can't be brushed off is the one that's
self-inflicted. I offered the Social Text editors an opportunity to demonstrate their intellectual rigor. Did they meet the test? I
don't think so. I say this not in glee but in sadness. After all, I'm a leftist too (under the Sandinista government I taught mathematics
at the National University of Nicaragua). On nearly all practical political issues -- including many concerning science and technology

I'm a leftist (and feminist) becauseof evidence and


logic, not in spite of it. Why should the right wing be allowed to monopolize the
intellectual high ground? And why should self-indulgent nonsense -- whatever its professed political orientation -- be
-- I'm on the same side as the Social Texteditors. But

lauded as the height of scholarly achievement?

Radical Alts
Alt cant transition from theory to political change
Gibson 8
(John, Postdoctoral Fellow, University of Kent, The Myth of the Multitude: The Endogenous
Demise of Alter-globalist Politics, Global Society, Vol. 22, No. 2, April, 2008)
Crucially, such assessments view the rhizomatic structures and reflexive capacities of alter-globalist agents as also negating the internal antagonism
and factionalism often associated with social movements. Hardt understands the World Social Forum as fulfilling the promise of anti-hierarchical politics
an infinitesimally open-ended collection of struggles converging around a basic commitment to resist neoliberalism, in which ideological differences

Gill has argued that alter-globalist practices counter longstanding differences between new and old social movements, creating a radically
fluid postmodern Prince in the process.17 Similarly, Jackie Smith has argued that the demand for
democracy, functioning as an open signifier for the desire for greater personal control
over biopolitics, is the sole commonality in otherwise plural alter-globalist spaces.18 Such
cease to produce significant schisms.16 Stephen

claims resemble Laclau and Mouffes model of a Logic of Equivalence, in which a vast collection of particular social movements and political demands
resonate in accordance with one another through the use of a central empty signifier (in this case, democracy) as an open point for infinite reinscriptions, and whose ontological closure is permanently resisted.19 In such interpretations, the pronoun we in alterglobalist contexts is devoid of
any fixed subjective content and is radically democratic. The political moment is defined within such discourses by Kingsnorths one no.

Unity is

structured around a shared rejection of transnational neoliberal capitalism, facilitating crosscultural forms of identification and epistemic exchange, resisting any fixed meaning of the collective subject that undertakes activism. The
central dilemma for activist life becomes whether the many yeses arising in its wake
can exert counter-hegemonic power in a long-term war of position.20 However, although such
accounts appear to present sources of optimism that dominant technocratic and individualist frames of global politics
can be countered successfully, there are reasons to doubt their claims. After reaching an early apogee
of 250,000 protesters at the Genoa G8 meeting in 2001, in the wake of 11 September 2001 summit protests
have declined significantly, both in terms of frequency and attendees. The protest in Miami against the Free
Trade Area of the Americas (FTAA) agreement in November 2003, advertised within movement circles as an event that would exceed the
importance of Seattle, drew a crowd of below 10,000, and failed to generate any significant media
coverage.21 The 2004 G8 summit in Georgia was met by hundreds, rather than thousands of protesters, and planned solidarity actions
elsewhere failed to materialise.22 Likewise, the 2004 London May Day gathering was cancelled after poorly attended planning meetings, and in 2005

direct action
protests conducted around the Gleneagles G8 summit were undertaken by only a few
thousand, and controversial protest tactics, including acts of vandalism in Auchterarder, generated considerable internal schisms.24 Social
Forums have also become beset by difficulties. Intra-movement concerns reflect a growing atrophy ; the absence
fewer than two dozen self-declared anti-capitalists played cricket on the lawn south of the House of Commons.23 The

of a venue for the prospective fifth European Social Forum reflects a scenario in which there is no more cooperation between our forces than there
was before Florence [in 2002].25 To

compound matters, some activists have complained that such


events are becoming dominated by resource-heavy NGOs.26 The World Social Forum has become frequented
by political elites. Gerhard Schroder and Jacques Chirac both attended the 2003 event, transgressing its
Charter of Principles, in which the WSF exists in opposition to a process of
globalisation commanded by the large multinational corporations and by the governments and
international institutions at the service of those corporations interests, with the complicity of national governments.27 Activists often
assert that the current impasse is temporary, that as new ways of pushing forward are
imagined there will be a corresponding resurgence of antineoliberal politics. However, rather
than the current lull representing a temporary period of torpor for alter-globalist politics, a number
of entrenched weaknesses within the dominant myths of alter-globalist networks stymie

its capacity to embody radical democracy . Caution is required against the hubris evident in
the prevailing accounts of a fluid subject, conscious of its counter-hegemonic potential as
a source of the dissolution of the neoliberal world order. Such mythology is ridden with inherent deficiencies
that are rarely addressed by participants. Rather than a global multitude taking shape, in many
respects we see only the phantasm of such a polis , and one that depends upon the
active repression of contradictions and flaws in order to maintain the idealised image of
the multitude presented by such prominent movement commentators as Klein and Kingsnorth.

Framework

Surveillance-Specific

Discuss Surveil k Resist


Discussion of surveillance is key to resist the surveillance state
deliberation through the public can bring about change
Rice 15
(Rebecca, University of Montana, Resisting NSA Surveillance: Glenn Greenwald and the public sphere
debate about privacy, pg online @ http://scholarworks.umt.edu/cgi/viewcontent.cgi?
article=5439&context=etd YDEL)

Public deliberation can be viewed as a form of resistance through Greenwald's rhetoric.


Though many scholars have looked at microresistance to surveillance power, Greenwald asks the
public to resist through deliberation, which he considers an antidote to surveillance. This
solution grapples with the contradiction of the US as surveillance state and the US as a
democracy. The public sphere will continue to discuss these competing values, sharing
in alternative projections of the future (Goodnight, 2012a). Greenwald encourages the public to
create a shared vision of a state with transparency in the public sphere and privacy for
citizens. Implications for Journalism and Whistleblowing Greenwald's rhetoric also has implications for the practices of journalism and whistleblowing. Though he argues that journalists should do
more to question the US surveillance state, he finds himself constrained by journalistic expectations. These lead to a debate over the boundaries of
journalism and how to portray whistleblowers. Greenwald as journalist or advocate. Greenwald's critique of
the media offers another path of resistive action for special actors within the public
sphere. The media serves the public sphere by spreading information, as Habermas (1974) says, in a large public
body this kind of communication requires specific means for transmitting information and influencing those who receive it. Today newspapers and magazines, radio and television are the media of the public

Greenwald's critique of the media calls for the media to uphold their public sphere
function and transmit information, even state secrets, for public deliberation. However, differences
sphere (p. 49).

between Greenwald's Guardian articles and NPTH show that even though Greenwald says journalists should be advocates, he himself is constrained by journalistic expectations. Greenwald's writings in The
Guardian fulfill less of an advocacy role than his writings in NTPH. Greenwald's newspaper articles often: 1) explain information about surveillance programs in common language, 2) quote various
organizations' standpoints and knowledge about the programs and, 3) explain why these revelations are significant (e.g. they demonstrate that the NSA concealed programs, they demonstrate that Americans
are being spied on without a court warrant). Importantly, many of Greenwald's stronger advocacy claims, like the argument that the NSA functions as a panopticon, are not mentioned until NPTH. Other
functions of the book include: 1) demonstrating the negative private effects of surveillance, 2) linking surveillance to the compromise of democratic ideals, and 3) suggesting possible solutions.

Though Greenwald encourages journalists to provide a check on the government and


advocate for change, his claims are tempered in his own newspaper articles. Greenwald discusses this directly as he talks about trouble publishing with The Guardian throughout
NPTH. Greenwald prepared his first story about BOUNDLESSINFORMANT, but was told that The Guardian editors would have to meet with their lawyers before publishing. Publishing US government secrets,

The Guardian agreed to consult with the government to


demonstrate that they did not intend to harm national security. Greenwald waited impatiently in Hong Kong for three days as
he was told, could be considered a crime under the Espionage Act.

his editor conferred with Guardian lawyers. He considered leaving and publishing the documents independently, but admits that doing it alone, without institutional protection, would be far riskier than

Though Greenwald says journalists should be stronger critics of the


US government, he himself struggled with institutional and journalistic constraints as he
reported the NSA stories.
publishing with a large newspaper (2014, p. 69).

The public sphere through discussion can combat the surveillance statethe public has the power to remove NSA secrecy resulting in legal reform
Rice 15
(Rebecca, University of Montana, Resisting NSA Surveillance: Glenn Greenwald and the public sphere
debate about privacy, pg online @ http://scholarworks.umt.edu/cgi/viewcontent.cgi?
article=5439&context=etd YDEL)

the public sphere can be used as resistance to surveillance


power, though this solution may not satisfactorily solve the overreaches of surveillance power. Greenwald links Foucauldian ideas of
surveillance to the NSA, and then poses the solution of public deliberation. The public sphere may be
a strong form of resistance against surveillance because of the incompatibility of the
technical values of surveillance with the public sphere: if, as Greenwald says, the NSA is successful in
retaining its power because it is secret, a public sphere discussion strips away the NSA's power by
removing its secrecy. Central tenets of the public sphere include public access and
openness (Asen and Brouwer, 2001), and central tenets of surveillance power include public
inaccessibility and technical closure. By framing surveillance as at odds with the public sphere, Greenwald
makes the argument that only through the public sphere can we resist surveillance on a
large-scale. Greenwald's rhetoric bridges the theoretical gap between public sphere scholarship and Foucault's idea of surveillance power,
demonstrating how these conflicting values may overlap in ways that are productive for
resistance.
The key finding from Greenwald's case is that

Public discussion acts as resistance to NSA surveillance- only engaging in


deliberation the state can limit surveillance
Rice 15
(Rebecca, University of Montana, Resisting NSA Surveillance: Glenn Greenwald and the public sphere
debate about privacy, pg online @ http://scholarworks.umt.edu/cgi/viewcontent.cgi?
article=5439&context=etd YDEL)

The public sphere as resistance. The resistive subject Greenwald calls into being would
be an active public sphere participant who questions the surveillance state through
public discussion. Though this subject would take small steps to preserve online privacy, Greenwald spends much of NPTH explaining broader solutions to surveillance. NPTH
constitutes an attempt to use the public sphere to resist NSA surveillance. Resistance literature
has often focused on small ways to resist surveillance power. Foucault focuses on personal, transversal struggles against
surveillance power, because surveillance power stems from compartmentalization and control over the
body. The body is approached as an object to be analyzed and separated into its constituent parts, forging the creation of a docile, useful subject (Dreyfus & Rabinow, 1983, p. 153). As a
result, struggles against this control are often anarchistic, immediate, and focused on the individual (Foucault, 1983).
Greenwald acknowledges this effect, and argues that surveillance leads to the internal suppression of dissenting
thoughts as a result. However, Greenwald does not advocate for just a small, personal
solution to this control over the body. Greenwald's appeals are an example of using the public
sphere as a form of resistance to surveillance. Though this idea differs from acts of microresistance, Greenwald's suggestions still fit with
Foucauldian ideas of resistance, and show how the public sphere can play a part in that resistance. Greenwald
argues that citizens can engage in public deliberation to negotiate with the surveillance
state. This echoes Foucault's idea of not being governed quite so much, or critique (1997, p. 45). Foucault (1997) says that critique is based on several anchoring points, including universal rights. The
act of critique asserts that the subject does not want to accept laws because they are unjust. Critique asks What are the limits of the right to govern? (Foucault, 1997, p. 46). People may
engage in critique to negotiate the way they are being governed if they find the rules of
governance to be contrary to natural rights. Greenwald encourages critique through
public deliberation about the limits of the surveillance state. He draws on American values like political freedom and freedom of
expression, thus using American rights as a basis for critiquing surveillance. Greenwald's call to action echoes Foucault's idea of critique.

Rice 58 Greenwald's ideas also run parallel to Habermas's

a medium through which


private needs were communicated to the state. A key demand of this bourgeois public was that they opposed the principle of supervisionthat
functions of the public sphere. Greenwald's solution is similar to the way Habermas (1974) describes the bourgeois public sphere, which was

very principle which demands that proceedings be made public (p. 52). This group was concerned with private autonomy and the restriction of state power to a few areas. Thus, Habermas argues that the

public sphere can be used to negotiate private freedoms. This becomes the link between the public sphere and Foucauldian ideas of
surveillance: using the public sphere, citizens can collectively negotiate to be governed less.

Greenwald demonstrates that the NSA's surveillance is a public problem, calling on the
American public to deliberate about the future of surveillance.

Talking about surveillance helps us change policies


Greenwald 2014
(Glenn Edward Greenwald is an American lawyer, journalist and author. He was a columnist for Guardian US from August 2012 to
October 2013. He was a columnist for Salon.com from 2007 to 2012, and an occasional contributor to The Guardian. ; No Place to
Hide ; May 13, 2014 ; Introduction: 7-8 ; AWEY)

In the fall of 2005, without much in the way of grandiose expectations, I decided to create a political blog. I had little
idea at the time how much this decision would eventually change my life. My principal motive was that I
was becoming increasingly alarmed by the radical and extremist theories of power the US government had adopted in the wake of 9/11, and I hoped that
writing about such issues might allow me to make a broader impact than I could in my
then-career as a constitutional and civil rights lawyer. Just seven weeks after I began blogging, the
New York Times dropped a bombshell: in 2001, it reported, the Bush administration had secretly ordered the
National Security Agency (NSA) to eavesdrop on the electronic had been going on for four years and had targeted at least several thousand Americans. The
subject was a perfect convergence of my passions and my expertise. The government tried to justify
the secret NSA program by invoking exactly the kind of extreme theory of executive power that had motivated me to begin writing: the notion
that the threat of terrorism vested the president with virtually unlimited authority to do
anything to keep the nation safe, including the authority to break the law. The ensuing debate entailed complex questions of constitutional law
and statutory interpretation, which my legal background rendered me well suited to address . I spent the next two years covering every
aspect of the NSA warrantless wiretapping scandal, on my blog and in a bestselling 2006 book. My position
was straightforward: by ordering illegal eavesdropping, the president had committed
crimes and should be held accountable for them. In Americas increasingly jingoistic and oppressive political climate, this proved
to be an intensely controversial stance.

Talking about government policies allow us to become informed


Greenwald 2014
(Glenn Edward Greenwald is an American lawyer, journalist and author. He was a columnist for Guardian US from August 2012 to
October 2013. He was a columnist for Salon.com from 2007 to 2012, and an occasional contributor to The Guardian. ; No Place to
Hide ; May 13, 2014 ; Introduction: 7-8 ; AWEY)

It was this background that prompted Edward Snowden, several years later, to choose me as his first
contact person for revealing NSA wrong-doing on an even more massive scale. He said he
believed I could be counted on to understand the dangers of mass surveillance and extreme state
secrecy, and not to back down in the face of pressure from the government and its many
allies in the media and elsewhere. The remarkable volume of top secret documents that
Snowden passed on to me, along with the high drama surrounding Snowden himself, have generated unprecedented
worldwide interest in the menace of mass electronic surveillance and the value of privacy in the digital age. But the
underlying problems have been festering for years, largely in the dark. There are, to be sure,
many unique aspects to the current NSA controversy. Technology has now enabled a type of
ubiquitous surveillance that had previously been the province of only the most
imaginative science fiction writers. Moreover, the post-9/11 American veneration of security

above all else has created a climate particularly conducive to abuses of power. And thanks to
Snowdens bravery and the relative ease of copying digital NSA story resonate with numerous episodes from the past, stretching back across the centuries. Indeed,

opposition to government invasion of privacy was a major factor in the establishment of


the United States itself, as American colonists protested laws that let British officials ransack at will any home they wished. It was legitimate, the colonists agreed, for the
state to obtain specific, targeted warrants to search individuals when there was evidence to establish probable cause of their wrongdoing. But general warrantsthe practice of
making the entire citizenry subject to indiscriminate searcheswere inherently illegitimate. The Fourth Amendment enshrined this idea in American law. Its language is clear and
succinct: The right of the people to be secure in their persons, houses, papers, and effects, against unreasonable searches and seizures, shall not be violated, and no warrants
shall issue, but upon probable cause, supported by oath or affirmation, and particularly describing the place to be searched, and the persons or things to be seized. It was

clash over
surveillance in the eighteenth century focused on house searches, but as technology
evolved, surveillance evolved with it. In the mid-nineteenth century, as the spread of railways began to allow for cheap and rapid mail
delivery, the British governments surreptitious opening of mail caused a major scandal in the UK. By the early decades of the twentieth
century, the US Bureau of Investigationthe precursor of todays FBIwas using wiretaps, along with mail
monitoring and informants, to clamp down on those opposed to American government policies. No matter the specific techniques involved, historically
mass surveillance has had several constant attributes. Initially, it is always the countrys dissidents and marginalized who
bear the brunt of the surveillance, leading those who support the government or are merely apathetic to mistakenly believe they are immune. And history shows
that the mere existence of a mass surveillance apparatus, regardless of how it is used, is
in itself sufficient to stifle dissent. A citizenry that is aware of always being watched quickly becomes a compliant and FBIs spying shockingly
intended, above all, to abolish forever in America the power of the government to subject its citizens to generalized, suspicionless surveillance. The

found that the agency had labeled half a million US citizens as potential subversives, routinely spying on people based purely on their political beliefs. (The FBIs list of targets
ranged from Martin Luther King to John Lennon, from the womens liberation movement to the anti-Communist John Birch Society.) But the plague of surveillance abuse is

mass surveillance is a universal temptation for any


unscrupulous power. And in every instance, the motive is the same: suppressing dissent and
mandating compliance. Surveillance thus unites governments of otherwise remarkably divergent political creeds. At the turn of the twentieth century, the
hardly unique to American history. On the contrary,

British and French empires both created specialized monitoring departments to deal with the threat of anticolonialist movements. After World War II, the East German Ministry of
State Security, popularly known as the Stasi, became synonymous with government intrusion into personal lives. And more recently, as popular protests during the Arab Spring

Investigations by
Bloomberg News and the Wall Street Journal have shown that as these dictatorships
were overwhelmed by protestors, they literally went shopping for surveillance tools from Western technology companies. Syrias Assad regime
challenged dictators grasp on power, the regimes in Syria, Egypt, and Libya all sought to spy on the Internet use of domestic dissenters.

flew in employees from the Italian surveillance company Area SpA, who were told that the Syrians urgently needed to track people. In Egypt, Mubaraks secret police bought
tools to penetrate Skype encryption and eavesdrop on activists calls. And in Libya, the Journal reported, journalists and rebels who entered a government monitoring center in
2011 found a wall of black refrigerator-size devices from the French surveillance company Amesys. The equipment inspected the Internet traffic of Libyas main Internet
service provider, opening emails, divining passwords, snooping on online chats and mapping connections among various suspects. The ability to eavesdrop on peoples

unless such power is held in check by rigorous oversight


and accountability, it is almost certain to be abused. Expecting its temptations runs counter to every historical example and all
communications vests immense power in those who do it. And

available evidence about human nature.

Talking about what government surveillance policies do is key to change


Greenwald 2014
(Glenn Edward Greenwald is an American lawyer, journalist and author. He was a columnist for Guardian US from August 2012 to
October 2013. He was a columnist for Salon.com from 2007 to 2012, and an occasional contributor to The Guardian. ; No Place to
Hide ; May 13, 2014 ; Chapter 5: The Fourth Estate ; AWEY)

Shortly before he died in 2005, the heralded Vietnam war correspondent David
Halberstam gave a speech to students at the Columbia Journalism School. The proudest moment of
his career, he told them, was when US generals in Vietnam threatened to demand that his editors at the
New York Times remove him from covering the war. He had, Halberstam said, enraged Washington and Saigon by
filing pessimistic dispatches on the war. The generals considered him the enemy since he had also interrupted their press conferences to accuse
them of lying. For

Halberstam, infuriating the government was a source of pride, the true


purpose and calling of journalism. He knew that being a journalist meant taking risks,
confronting rather than submitting to abuses of power. Today, for many in the profession,
praise from the government for responsible reportingfor taking its direction about
what should and should not be publishedis a badge of honor. That this is the case is the true measure of
how far adversarial journalism in the United States has fallen.

Transparency Key
Public scrutiny solves transparency is the only mechanism
Rice 15
(Rebecca, University of Montana, Resisting NSA Surveillance: Glenn Greenwald and the public
sphere debate about privacy, pg online @ http://scholarworks.umt.edu/cgi/viewcontent.cgi?
article=5439&context=etd)//akim
publics discursively, saying they exist by virtue of being addressed (p.
413). Greenwald calls a concerned public into being throughout NPTH, often by discussing his readers as a
collectivewe. Greenwald's created audience is concerned about surveillance, and willing to take publicaction to advocate for reform.
Greenwald emphasizes the choice readers can make with Snowden's leaked NSA
documents. He says that Snowden's leaks can create a new discussion about surveillance , or
they can fade due to public apathy. In the introduction to NPTH, he writes That's what makes Snowden's revelations so stunning and so vitally important. By daring
to expose the NSA's astonishing surveillance capabilities and its even more astounding ambitions, he has made it clear, with these disclosures, that we stand at a historic
crossroads. Will the digital age usher in the individual liberation and political freedoms
that the Internet is uniquely capable of unleashing? Or will it bring about a system of
omnipresent monitoring and control, beyond the dreams of even the greatest tyrants of the past? Right now, either path is
Greenwald's created public. Warner (2002) encourages scholars to frame

possible . Our actions will determine where we end up. (2014, p. 6). Greenwald gives the audience two choices and links their actions to the two potential paths. In this way, he begins the process of
public deliberation, which Goodnight (2012a) describes as a momentary pause in which we examine political paths, both taken and untaken. As deliberation raises expectations that are feared or hoped for,

public argument is a way to share in the construction of the future, he says (Goodnight, 2012a, p. 198).
Greenwald shares his interpretation of the choice the public must make with this information. He projects two alternative futures based on the public's
deliberation about privacy. This shared future is emphasized through his use of the words our, everyone, and we, which link readers together as the American public. Greenwald's projected paths put the

Greenwald addresses his


readers as part of a public sphere. Greenwald also argues that deliberation is an effective way to resist
surveillance and curb surveillance abuses. Greenwald offers an example from his own life. He says he first learned of the power of deliberation when he
decision into the readers' hand, emphasizing the public's ability to act and intervene in technical surveillance. Through invitations to deliberate,

heard from Laura Poitras, another journalist who accompanied him on the trip to Hong Kong. She said that she had been detained in airports dozens of times as a result of her writing and filmmaking. Greenwald
covered the interrogations of Poitras in a Salon article, which received substantial attention. In the months afterward, Poitras was notdetained again. In NPTH, Greenwald writes The lesson for me was clear

national security officials do not like the light. They act abusively and thuggishly only
when they believe they are safe, in the dark. Secrecy is the linchpin of abuse of power, we
its enabling force . Transparency is the only real antidote (2014, p. 12). Greenwald generalizes this example to other
power without deliberation is the ultimate imbalance, permitting the most
dangerous of all human conditions: the exercise of limitless power with no transparency
or accountability (2014, p. 169). Greenwald presents public deliberation as thesolution and
antithesis to surveillance, which he calls for the public to undertake. Afteraddressing readers as members of this public, Greenwald names special actors within the public
discovered,

abuses of power. He says that

sphere who can also help to effect change.

Public Sphere Disc k Ref NSA


Public sphere discussions aid in congressional reforms to tighten its
reigns on the NSA
Rice 15 (Rebecca, University of Montana, Resisting NSA Surveillance: Glenn Greenwald and
the public sphere debate about privacy, pg online @
http://scholarworks.umt.edu/cgi/viewcontent.cgi?article=5439&context=etd)//akim
the government must make changes in order to curb abuses from the
NSA, and that readers should pressure the government to do so. Greenwald says that public
branches of the government do not have enough control over the NSA. Giving examples of reform that occurred
after his reporting, Greenwald says that he and Snowden were pleased by a bipartisan bill introduced to US Congress. This bill proposed
defunding the NSA, which was by far the most aggressive challenge to the national security state to emerge from Congress since the 9/11 attacks (2014, p.
249). The bill did not pass, but only by a small margin, which Greenwald portrays as a hopeful
Government reform. First, Greenwald says

sign of reform . Additionally, Greenwald suggests converting the FISA court into a real judicial system,
rather than the onesided current setup in which only the government gets to state its
case, would be a positive reform (2014, p. 251). Greenwald's suggestions for change go beyond individual
acts to put pressure on government policy reform. By reforming the FISA court, the secrets of the NSA would be
public knowledge. Though Greenwald writes about power from a Foucauldian perspective, he proposes large acts of
resistance to the public problems created by surveillance in addition to small acts to
resist the discipline of individual bodies. These ideas are compatible with Foucault's (1997) idea of critique, however, which he defines
as the art of not being governed quite so much (p. 45). Greenwald asks the audience to resist through public
sphere discussion in order to negotiate the way they are governed. He argues that discussion through the
public sphere can alter power relations between citizens and the US surveillance state.
Though the government is considered completely separate from the public sphere by
many scholars (Habermas, 1974), others push back on this idea (e.g. Asen & Brouwer, 2001). In the case of the NSA, other government
branches are considered members of the public by Greenwald. Greenwald notes that many congressional members were unaware of the tactics used by the NSA,
including spying on Congress itself (2014). For these reasons, Greenwald specifically calls on Congress
to be part of the solution. This should occur through legislative reform spurred by public pressure.
Greenwald's summoned public addresses politicians as well as average citizens.

Debate k Surv Policies


Surveillance policies should be questioned through forms of debate
Rice 15 (Rebecca, University of Montana, Resisting NSA Surveillance: Glenn Greenwald and
the public sphere debate about privacy, pg online @
http://scholarworks.umt.edu/cgi/viewcontent.cgi?article=5439&context=etd)//akim
Emphasis on multiple voices. Greenwald also moves the groundings of the surveillance debate
by making it just that a debate . He writes about the variety of voices discussing surveillance, which demonstrates that surveillance
is more than just a technical talking point. In his early articles for The Guardian, Greenwald does this by quoting many technical
experts, government voices, and special interest groups. In his first article for The Guardian, revealing the program PRISM, Greenwald takes statements from several US
senators, including Ron Wyden, who Greenwald says raised concerns about surveillance for years before the
leaked documents came to light (2013a). Greenwald also takes statements from Jameel Jaffer, the director of the American Civil Liberties Union (ACLU)
Center for Democracy. Jaffer said that the military has been granted unprecedented access to
civilian communications (Greenwald, 2013a, p. 6). Greenwald quotes public figures who state their concern about the effects surveillance could have
on the public. These statements contrast with quotes from government officials, which are left without commentary. In his articles in The Guardian, Greenwald airs a variety of

readers may begin to engage in the debate . Greenwald also moves the debate into
the public sphere by revealing information to which the government and technology
companies must respond. For example, in NPTH Greenwald tells the story of writing an article for The Guardian about PRISM. He says that NSA
documents showed that technology companies cooperated with the NSA, but the
companies denied this when contacted. When deciding what to do, Greenwald chose to write both statements into his story: Let's
not take a position on who's right. Let's just air the disagreement and let them work it
voices so that

out in public ,' I proposed. Our intention was that the story would force an open discussion of what the
Internet industry had agreed to do with their users' communications; if their version clashed with the NSA
documents, they would need to resolve it with the world watching, which is how it should be (2014, p. 76). This move parallels one way that Goodnight (2012a) says that

If a technical argument becomes unreconcilable, both groups may


take to the public forum governing the technical community's business, each contesting
for leadership and control of scarce resources. If one side or the other is dissatisfied with
the verdict, then the boundaries of the special community are in jeopardy, as disgruntled
advocates appeal to a more general public (p. 202). Greenwald forces this eruption of appeals to the public by reporting that
technology companies are cooperating with the NSA. Because these reports damage
technology companies' reputations, they must respond to their consumer base, or the
public. As Goodnight (2012a) says, An arguer can accept the sanctioned, widely used bundle of rules, claims,
procedures and evidence to wage a dispute. Or, the arguer can inveigh against any or all of these 'customs' in
order to bring forth a new variety of understanding... In the variety of argument endeavors, this tension is
expressed by attempts to expand one sphere of argument at the expense of another (p. 200).
Greenwald challenges the sanctioned forms of argument about surveillance. Using common
examples mixed with technical evidence, he expands the argument into the public sphere at expense of the technical. Goodnight (2012b) says that arguments
engage social change when the systems of authority embedded in spheres not only fail
to provide resolution but the expectations themselves (as implicit norms, conventions of propriety, or explicit rules)
become part of the debate (p. 260). Greenwald makes the technical sphere rules part of the debate by
explaining the value of citizen deliberation. He presents the public with options, which serve as the antithesis to the NSA's secrecy, or lack
of options, for public participation in their programs. Using these strategies, Greenwald moves arguments about surveillance
out of the technical sphere and into the public sphere. Greenwald also discusses the effects of surveillance in more detail,
technical arguments become public.

and calls the public to resist surveillance. These appeals to privacy look less at the actors, or the technical sphere, and more at the
surveillance apparatus itself.

Surv State Resist k Democ


Greenwald reifies Foucauldian ideas through resistance from the public
sphere this challenges the surveillance state and democratic ideals
Rice 15 (Rebecca, University of Montana, Resisting NSA Surveillance: Glenn Greenwald and
the public sphere debate about privacy, pg online @
http://scholarworks.umt.edu/cgi/viewcontent.cgi?article=5439&context=etd)//akim
The public sphere as resistance. The resistive subject Greenwald calls into being would be an active public sphere participant who
questions the surveillance state through public discussion. Though this subject would take small steps to preserve online privacy,
Greenwald spends much of NPTH explaining broader solutions to surveillance. NPTH constitutes an attempt to use the public
sphere to resist NSA surveillance. Resistance literature has often focused on small ways to resist surveillance power. Rice 57
Foucault focuses on personal, transversal struggles against surveillance power, because surveillance power stems from
compartmentalization and control over the body. The body is approached as an object to be analyzed and separated into its
constituent parts, forging the creation of a docile, useful subject (Dreyfus & Rabinow, 1983, p. 153). As a result, struggles against
this control are often anarchistic, immediate, and focused on the individual (Foucault, 1983). Greenwald acknowledges this effect,
and argues that surveillance leads to the internal suppression of dissenting thoughts as a result. However, Greenwald does not
advocate for just a small, personal solution to this control over the body.

of using the public sphere as

a form of

Greenwald's appeals are an example


resistance to surveillance. Though this idea differs from acts of

microresistance, Greenwald's

suggestions still fit with Foucauldian ideas of resistance, and


show how the public sphere can play a part in that resistance. Greenwald argues that citizens can
engage in public deliberation to negotiate with the surveillance state. This echoes Foucault's idea of not
being governed quite so much , or critique (1997, p. 45). Foucault (1997) says that critique is based on several
anchoring points, including universal rights. The act of critique asserts that the subject does not want to accept laws because they
are unjust. Critique asks What

are the limits of the right to govern? (Foucault, 1997, p. 46). People may
engage in critique to negotiate the way they are being governed if they find the rules of
governance to be contrary to natural rights. Greenwald encourages critique through public
deliberation about the limits of the surveillance state. He draws on American values like political
freedom and freedom of expression, thus using American rights as a basis for critiquing
surveillance. Greenwald's call to action echoes Foucault's idea of critique. Rice 58 Greenwald's ideas also run parallel to
Habermas's functions of the public sphere. Greenwald's solution is similar to the way Habermas (1974) describes the bourgeois
public sphere, which was a medium through which private needs were communicated to the state. A key demand of this bourgeois
public was that they opposed the principle of supervisionthat very principle which demands that proceedings be made public (p.
52). This group was concerned with private autonomy and the restriction of state power to a few areas. Thus, Habermas

argues that the public sphere can be used to negotiate private freedoms. This becomes the link
between the public sphere and Foucauldian ideas of surveillance: using the public sphere, citizens can
collectively negotiate to be governed less. Greenwald demonstrates that the NSA's surveillance is a
public problem, calling on the American public to deliberate about the future of
surveillance. Greenwald links this solution to personal control, however, by explaining how surveillance impacts our
individual thoughts and actions. In these ways, his solution does fit with some of Foucault's ideas of a transversal struggle, namely
that his solution struggles with state control over the individual and critiques power for its effects (Foucault, 1983). Greenwald
(2014) says that people

radically change their behavior when they know they are being
watched (p. 173) as he demonstrates the effects of surveillance power. He then links these behavioral changes
to the suppression of free speech, saying that mass surveillance kills dissent in a deeper
and more important place as well: in the mind (2014, p. 177-178). Greenwald argues that surveillance
power controls the individual and critiques power for its effects. Though Greenwald is encouraging
a public debate, he claims that this debate will help negotiate surveillance power that creates

control over individual bodies, thus drawing on some Rice 59 of Foucault's ideas of resistance as he talks about
the public sphere. This solution demonstrates the tension between the surveillance state and the democratic
republic, as Greenwald wrestles with the US as both a security state and a democracy. He
ponders this contradiction as he explains the panopticon, saying Democracy requires accountability and
consent of the government, which is only possible if citizens know what is being done in
their name. The presumption is that, with rare exception, they will know everything their political officials are
doing...conversely the presumption is that the government, with rare exception, will not know anything that law-abiding citizens are
doing. That is why we are called private individuals, functioning in our private capacity. Transparency

is for those
who carry out public duties and exercise public power. Privacy is for everyone else (2014,
p. 209). The fact that the NSA knows more about US citizens than citizens know about the
agency poses challenges to this model of government. As a result, Greenwald encourages
the exercise of democratic rights to combat surveillance. The public sphere has the
capability to put pressure on government officials and demand surveillance reform. Greenwald asks us to
resist using the public sphere. This solution arises as a result of the rhetorical situation, which pits privacy against
security. Greenwald encourages the audience to select privacy. He spends time deescalating the permanent emergency of
terrorism to demonstrate that NSA surveillance is abusive and unnecessary, and then appeals to values
which support public deliberation. Public deliberation can be viewed as a form of resistance through Greenwald's rhetoric. Though

many scholars have looked at microresistance to surveillance power, Greenwald asks Rice 60 the
public to resist through deliberation, which he considers an antidote to surveillance. This solution grapples with the
contradiction of the US as surveillance state and the US as a democracy. The public sphere will continue to discuss these
competing values, sharing in alternative projections of the future (Goodnight, 2012a). Greenwald

encourages the
public to create a shared vision of a state with transparency in the public sphere and
privacy for citizens.

Activism k Engage Govt


Political activism is key to engage the public as well as the government.
Discussions in debate are the first step to create real change.
Rice 15 University of Montana (Rebecca, Theses, Dissertations, Professional Papers from
University of Montana Graduate School, Resisting NSA Surveillance: Glenn Greenwald and the
Public Sphere Debate about Privacy, Paper 4480,
http://scholarworks.umt.edu/cgi/viewcontent.cgi?article=5439&context=etd)//DWB
Greenwald's case is itself an example of the groundings of argument shifting between Goodnight's three spheres of argument.

Greenwald employed several tactics to move surveillance toward publicly-based


appeals. These include demonstration of public effects, explanations of technical jargon,
and the inclusion of multiple voices in the surveillance debate. Greenwald drafts proposed solutions
which would engage US citizens, the US government,. and journalists Notably, these solutions
focus on these groups rather than technical communities with the technical capabilities to alter
surveillance practices. Thus, Greenwald refuses to contain surveillance within the technical sphere
and instead encourages the public to deliberate. This example adds to previous case studies of overlap
between the technical and public spheres of argument. Previous scholarship has often focused on the eclipse of the public sphere
by the technical (e.g. Farrell & Goodnight, 1981; Hauser, 1987; Schiappa, 2012; Sovacool, 2009). Greenwald's case is an example
of pushback on this infiltration, and shows that just

as arguments can become insulated within the


technical sphere, they can also be removed and placed back into the public. Journalistic
activism may be an important part of this move, and will be discussed in a subsequent section.

Individual Actions can impact the government on Surveillance


Greenwald 2014
(Glenn Edward Greenwald is an American lawyer, journalist and author. He was a columnist for Guardian US from August 2012 to
October 2013. He was a columnist for Salon.com from 2007 to 2012, and an occasional contributor to The Guardian. ; No Place to
Hide ; May 13, 2014 ; Introduction: 10-14 ; AWEY)

even before Snowdens revelations, it was already becoming clear that treating the
United States as somehow exceptional on the issue of surveillance is a highly naive stance. In
2006, at a congressional hearing titled The Internet in China: A Tool for Freedom or Suppression?, speakers lined up to condemn American
technology companies for helping China suppress dissent on the Internet. Christopher Smith (R-NJ), the
congressman presiding over the hearing, likened Yahoo!s cooperation with Chinese
secret police to handing Anne Frank over to the Nazis. It was a full throated harangue, a
typical performance when American officials speak about a regime not aligned with the
United States. But even the congressional attendees couldnt help noting that the hearing happened to take place just two months after the New York Times
revealed the vast warrantless domestic wiretapping carried out by the Bush
administration. In light of those revelations, denouncing other countries for carrying out
their own domestic surveillance rang rather hollow. Representative Brad Sherman (D-CA), speaking after Representative Smith, noted that
Indeed,

the technology companies being told to resist the Chinese regime should also be careful regarding their own government. Otherwise, he warned prophetically, while those in China may see their privacy violated
in the most heinous ways, we here in the United States may also find that perhaps some future president asserting these very broad interpretations of the Constitution is reading our e-mail, and I would prefer that
not happen without a court order. Over the past decades, the fear of terrorismstoked by consistent exaggerations of the actual threathas been exploited by US leaders to justify a wide array of extremist
policies. It has led to wars of aggression, a worldwide torture regime, and the detention (and even assassination) of both foreign nationals and American citizens without any charges. But the ubiquitous, secretive
system of suspicionless surveillance that it has spawned may very well turn out to be its most enduring legacy. This is so because, despite all the historical parallels, there is also a genuinely new dimension to the

for the younger generation, the Internet is not


some standalone, separate domain where a few of lifes functions are carried out. It is not merely
our post office and our telephone. Rather , it is the epicenter of our world, the place where virtually everything is
done. It is where friends are made, where books and films are chosen, where political
activism is organized, where the most private data is created and stored. It is where we
develop and express our very personality and sense of self. To turn that network into a system of mass surveillance has
implications unlike those of any previous state surveillance programs. All the prior spying systems were by necessity more limited and capable of being evaded. To permit
surveillance to take root on the Internet would mean subjecting virtually all forms of
human interaction, planning, and even thought itself to comprehensive state
examination. From the time that it first began to be widely used, the Internet has been seen
by many as possessing an extraordinary potential: the ability to liberate hundreds of
millions of people by democratizing political discourse and leveling the playing field
between the powerful and the powerless. Internet freedomthe ability to use the network without institutional constraints, social or
state control, and pervasive fearis central to the fulfillment of that promise. Converting the Internet into a system of surveillance thus guts it of its core
potential. Worse, it turns the Internet into a tool of repression, threatening to produce the most extreme and oppressive weapon of state intrusion human history has ever seen. Thats what
makes Snowdens revelations so stunning and so vitally important. By daring to expose
the NSAs astonishing surveillance capabilities and its even more astounding ambitions, he has made it clear, with these disclosures, that we
stand at a historic crossroads. Will the digital age usher in the individual liberation and political freedoms
that the Internet is uniquely capable of unleashing? Or will it bring about a system of
omnipresent monitoring and control, beyond the dreams of even the greatest tyrants of the past? Right now, either path is
possible. Our actions will determine where we end up.
current NSA surveillance scandal: the role now played by the Internet in daily life. Especially

Surveillance Info k Govt Overreach


Becoming informed about surveillance policies allow us to challenge
government overreach
Greenwald 2014
(Glenn Edward Greenwald is an American lawyer, journalist and author. He was a columnist for Guardian US from August 2012 to
October 2013. He was a columnist for Salon.com from 2007 to 2012, and an occasional contributor to The Guardian. ; No Place to
Hide ; May 13, 2014 ; Chapter 5: The Fourth Estate ; AWEY)

One of the principal institutions ostensibly devoted to monitoring and checking abuse of
state power is the political media. The theory of a fourth estate is to ensure
government transparency and provide a check on overreach, of which the secret surveillance of entire populations is
surely among the most radical examples. But that check is only effective if journalists act adversarially to those who wield political power. Instead, the US media
has frequently abdicated this role, being subservient to the governments interests, even amplifying, rather than scrutinizing, its messages and
carrying out its dirty work. In this context, I knew that media hostility toward my reporting on Snowdens disclosures was inevitable. On June 6, the day after the first NSA article

After writing intensely, even obsessively,


for years about government surveillance and prosecution of journalists, Glenn
Greenwald has suddenly put himself directly at the intersection of those two issues, and
perhaps in the crosshairs of federal prosecutors, the paper proclaimed in a profile of me. My NSA reporting, it
added, is expected to attract an investigation from the Justice Department, which has
aggressively pursued leakers. The profile quoted the neoconservative Gabriel Schoenfeld of the Hudson Institute, who has long advocated the
ran in the Guardian, the New York Times introduced the possibility of a criminal investigation.

prosecution of journalists for publishing secret information, calling me a highly professional apologist for any kind of anti-Americanism no matter how extreme. The most
revealing evidence of the Timess intentions came from the journalist Andrew Sullivan, who was quoted in the same profile saying, Once you get into a debate with [Greenwald],
it can be hard to get the last word, and I think he has little grip on what it actually means to govern a country or run a war. Disturbed by the use of his comments out of context,
Andrew later sent me his full exchange with the Times reporter Leslie Kaufman, which included praise for my work that the paper had notably chosen to omit. What was more
telling, however, were the original questions Kaufman had sent him: He obviously had strong opinions, but how is he as a journalist? Reliable? Honest? Quotes you
accurately? Accurately describes your positions? Or is more advocate than journalist? He says you are a friend, is this so? I get the sense that he is something of a loner and
has the kind of uncompromising opinions that makes it hard to keep friends, but could be wrong. The second questionthat Im something of a loner who has trouble keeping
friendswas, in some sense, even more significant than the first. Discrediting the messenger as a misfit to discredit the message is an old ploy when it comes to whistleblowing, and it often works.

The NSA spies solely to collect data than prevent terrorism


Rice 15 (Rebecca, University of Montana, Resisting NSA Surveillance: Glenn Greenwald and
the public sphere debate about privacy, pg online @
http://scholarworks.umt.edu/cgi/viewcontent.cgi?article=5439&context=etd)//akim
the NSA is insulated within the technical sphere, and the agency favors
technical reasoning, shifting control over who makes decisions about surveillance from
the FISA court to the NSA operatives themselves. The vast amount of discretion vested in NSA analysts is also demonstrated by
First, Greenwald says

the training and briefings given to them by the agency, Greenwald says as he reports on the FISA courts to The Guardian (2013d, p. 8). He cites a leaked NSA memo which
instructs analysts to use their own judgment and creativity to determine if a target is inside or outside of the United States. Though the FISA court is supposed to be consulted if

the decisions about who has their emails and telephone


calls intercepted by the NSA is made by the NSA itself, not by the FISA court (2013d, p. 9).
Though there are legal limits which keep the government from examining the content of
these communications, there are no technical limits on the ability of either the agency
or its analysts to do so (2013d, p. 9). The NSA analysts have specialized, technical knowledge. Here, the groundings of the argument are not legal (as
a surveillance target could be within the US, Greenwald says

Greenwald says the FISA court does not review the NSA's decisions), but technical, and insulated within the NSA. Greenwald's description of the NSA parallels Goodnight's

within the technical sphere, technical arguments are


stamped with procedure and rule where state of the art practice is always at issue (p. 260).
Greenwald similarly argues that within the NSA, technical capabilities are favored over ethical arguments against
spying. Greenwald also portrays the NSA and its partners as insulated and secretive, or
separated from the public sphere. Greenwald argues that the surveillance community is a revolving door of government officials and large
defense companies who work together to produce lucrative and powerful results: The post-9/11 era has seen a massive explosion of
resources dedicated to surveillance. Most of those resources were transferred from the
public coffers (i.e., the American taxpayer) into the pockets of private surveillance defense
corporations...Companies like Booz Allen Hamilton and AT&T employ hordes of former top government officials, while hordes of current top defense officials are
past (and likely future) employees of those same corporations. Constantly growing the surveillance state is a way to ensure that
the government funds keep flowing, that the revolving door stays greased (p. 168). Greenwald names the actors in the surveillance state
idea of the technical sphere. Goodnight (2012b) says that

frequently in his articles and book. These actors include high-up government officials and government contractors. He says that the groups work together to control information
and remain insulated. Large technical companies are also complacent in this surveillance, according to Greenwald. In his articles in The Guardian, Greenwald begins
introducing these figures. In his first article, about PRISM, Greenwald names Microsoft, Yahoo, Apple, Facebook, Skype, and AOL as companies that participate in the NSA's

collectively, the companies cover the vast majority of online


email, search, video, and communications networks (2013a, p. 3). In NPTH, Greenwald reflects that Snowden's
documents reveal a slew of secret negotiations between the NSA and Silicon Valley
about providing the agency with unfettered access to the companies' systems (2014, p. 112).
These companies control a large share of online communications, and they cooperate with the NSA. Greenwald links the NSA together
with other technical experts to demonstrate the vast power of the technical elite. These
groups have access to most communications, and can spy on the majority of online
messages. This is a specialized skill unique to the technical community. Greenwald also emphasizes the breadth of surveillance, and
links it to the power of the NSA. Reflecting on the Snowden documents in NPTH, Greenwald says Even as someone who had spent years writing
about the dangers of secrecy US surveillance, I found the sheer vastness of the spying system genuinely shocking, all the more so
because it had clearly been implemented with virtually no accountability, no transparency,
information-sharing programs (2013a). He says that

and no limits. The thousands of discrete surveillance programs described by the archive were never intended by those who implemented them to become public
knowledge (2014, p. 91). Greenwald notes the link between power and the breadth of power. He says that the NSA collects a vast amount of data simply because it can. The

the NSA
uses a collect it all philosophy, and the agency is devoted to one overarching
mission: to prevent the slightest piece of electronic communication from evading its
systemic grasp (2014, p. 94). Not only is the surveillance community isolated, the community is corrupt : the NSA
mere fact that it (the NSA) has the capability to collect those communications has become one rationale for doing so (2014, p. 95). Greenwald says that

collects data not to prevent terrorism but to contain its power . Displaying many internal NSA memos and
presentations, Greenwald demonstrates that the NSA celebrates the gathering of information, and more
information is always considered better. In one presentation, Greenwald says, the NSA writes that XKEYSCORE is valuable because
the program captures 'nearly everything a typical user does on the internet' (2014, p. 153). Greenwald also displays unflattering NSA memos, which demonstrate what he calls

The Role of National Interests, Money,


and Egos, and says that these three factors together...are the primary motives driving the United States to
maintain global surveillance domination (2014, p. Rice 32 167). By discussing the extent of surveillance gathered, Greenwald
the agency's ego. He points to the title of one presentation designed internally for NSA personnel:

characterizes the NSA as greedythe agency grasps at data and hoards it away from the public. Using the collect it all philosophy, Greenwald gives the surveillance

the community is not just isolated because of its


specializations, but because of its interest in power. Greenwald says the NSA has a
contemptuous and boastful spirit of supremacy behind them (2014, p. 94). As Farrell and Goodnight (1981) say,
arguments grounded in the technical sphere preclude practitioners from social responsibility (p. 296). Greenwald argues that the NSA is not interested in
the ethics of surveillance , but solely in its technical capabilities to spy. Greenwald furthers this
characterization by discussing oversight of surveillance. He says that the so-called checks on surveillance are ineffective,
because they, too, are shrouded in secrecy. Commenting on the FISA court, which issues warrants for NSA surveillance, Greenwald
says The uselessness of this institution as a true check on surveillance abuses is obvious
because the FISA court lacks virtually every attribute of what our society generally
community, and in this case the technical sphere, a new connotation:

understands as the minimal elements of a justice system. It meets in complete secrecy; only one party
the governmentis permitted to attend the hearings and make its case; and the court's
rulings are automatically designated 'Top Secret ' (2014, p. 128).

Discourse k Challenge Surv State


Our discourse is important to challenge the surveillance state
Greenwald 2013
(Glenn Edward Greenwald is an American lawyer, journalist and author. He was a columnist for Guardian US from August 2012 to
October 2013. He was a columnist for Salon.com from 2007 to 2012, and an occasional contributor to The Guardian. ;
http://www.theguardian.com/world/2013/jul/08/edward-snowden-surveillance-excess-interview; July 8, 2013 ; Edward Snowden: US
surveillance 'not something I'm willing to live under; AWEY)

Edward Snowden predicted more than a month ago while still in hiding in Hong Kong
that the US government would seek to demonise him, telling the Guardian that he would be accused of aiding America's enemies. In the
second instalment of an interview carried out before he revealed himself as the NSA whistleblower, Snowden insisted that he was a patriot and
that he regards the US as a fundamentally good country. But he said he had chosen to
release the highly classified information because freedoms were being undermined by
intelligence agency "excesses". The interview was conducted on June 6 in a hotel room in Hong Kong. The first part of the interview was released on Sunday June 9, starting a media
frenzy and intensifying US efforts to track him down. Snowden has since fled Hong Kong for Moscow, where he is reportedly marooned while resisting US attempts to extradite him to face charges under the
Espionage Act. In the newly released interview excerpts, he predicted he would be portrayed not as a whistleblower but a spy. "I think they are going to say I have committed grave crimes, I have violated the
Espionage Act. They are going to say I have aided our enemies in making them aware of these systems. But this argument can be made against anyone who reveals information that points out mass surveillance
systems," he said. Asked whether he had sought a career in the intelligence community specifically to become a mole and reveal secrets, Snowden, 30, said he had joined government service very young, first

his
views shifted over the length of his career as he watched the news, which he saw as
propaganda, not truth. "We were actually involved in misleading the public and
misleading all the publics, not just the American public, in order to create certain mindset
in the global consciousness and I was actually a victim of that." He had not fallen out of
love with America, only its government. "America is a fundamentally good country. We have good people with good values who want to do the right thing.
But the structures of power that exist are working to their own ends to extend their
capability at the expense of the freedom of all publics." In the new excerpts, he explained his motivation for revealing the information. "I
don't want to live in a world where everything that I say, everything I do, everyone I talk
to, every expression of creativity or love or friendship is recorded," he said. "And that's not something I'm willing to
support, it's not something I'm willing to build and it's not something I'm willing to live under." Advertisement He also insisted he had continued with his
job while waiting for political leaders to rein in what he decribed as "government excesses". But, he said, " as I've watched I've seen that's not
enlisting in the US army immediately after the invasion of Iraq out of a belief in "the goodness of what we were doing. I believed in the nobility of our intentions to free oppressed people overseas." But

occuring, and in fact we're compounding the excesses of prior governments and making
it worse and more invasive. And no one is really standing to stop it."

Snowden has been attacked by his critics for

first going to Hong Kong, which is part of China, even though it enjoys freedoms not available on the mainland, and to Russia. He has been offered asylum in Venezuela, Bolivia and Nicaragua but faces the
practical problem of how to get to any of these countries. The most recent poll, for the Huffington Post and YouGov, suggested a shift in support away for Snowden, with 38% saying they feel he did the wrong
thing in leaking documents against 33% who felt he did the right thing. After the first interview, 35% said he did the wrong thing while 38% said he had done the right thing. The interview took place immediately
after the Guardian published the first leak about a court order to Verizon ordering it to hand over US customers' call records to the NSA. Snowden explained why he thought that story and the other subsequent
leaks about the NSA and its partnership with the corporate sector had to be made public. "They are getting everyone's calls, everyone's call records and everyone's internet traffic as well." In reference to one
surveillance system Boundless Informant that he said allowed the NSA to track data it was accumulating, he said: "The NSA lied about the existence of this tool to Congress and to specific congressmen in

He was part of the internet generation that grew up on the


understanding that it was free, he said. The partnership between the intelligence
agencies and the corporate sector was a "dangerous collaboration", especially for an organisation like the the NSA
response to previous inquiries about their surveillance activities."

that has demonstrated time and again "it works to shield itself from oversight".

Key to Policymaking
Discourse about surveillance policies key to effective policy making
Greenwald 2014
(Glenn Edward Greenwald is an American lawyer, journalist and author. He was a columnist for Guardian US from August 2012 to
October 2013. He was a columnist for Salon.com from 2007 to 2012, and an occasional contributor to The Guardian. ; No Place to
Hide ; May 13, 2014 ; Epilogue ; AWEY)

Edward Snowden, he told me he had only one fear about coming


forward: that his revelations might be greeted with apathy and indifference, which would
mean he had unraveled his life and risked imprisonment for nothing. To say that this fear
has gone unrealized is to dramatically understate the case. Indeed, the effects of this
unfolding story have been far greater, more enduring, and more wide-ranging than we
ever dreamed possible. It focused the worlds attention on the dangers of ubiquitous
state surveillance and pervasive government secrecy. It triggered the first global debate
about the value of individual privacy in the digital age and prompted challenges to
Americas hegemonic control over the Internet. It changed the way people around the world viewed the
reliability of any statements made by US officials and transformed relations between countries. It radically altered
views about the proper role of journalism in relation to government power. And within the United States, it gave rise to an
ideologically diverse, trans-partisan coalition pushing for meaningful reform of the
surveillance state. One episode in particular underscored the profound shifts brought about by Snowdens revelations. Just a few weeks after my first Snowden-based article for the
Guardian exposed the NSAs bulk metadata collection, two members of Congress jointly introduced a bill to defund that
NSA program. Remarkably, the bills two cosponsors were John Conyers, a Detroit liberal
serving his twentieth term in the House, and Justin Amash, a conservative Tea Party
member in only his second House term. It is hard to imagine two more different members of Congress, yet here they were, united
in opposition to the NSAs domestic spying. And their proposal quickly gained dozens of cosponsors across the entire ideological spectrum, from the
In the very first online conversation I had with

most liberal to the most conservative and everything in betweena truly rare event in Washington. When the bill came up for a vote, the debate was televised on C-SPAN, and I watched it while chatting online
with Snowden, who was also watching CSPAN on his computer in Moscow. We were both amazed at what we saw. It was, I believe, the first time he truly appreciated the magnitude of what he had accomplished.
One House member after another stood up to vehemently denounce the NSA program, scoffing at the idea that collecting data on the calls of every single American is necessary to stop terrorism.

It was

by far the most aggressive challenge to the national security state to emerge from
Congress since the 9/11 attacks. Until the Snowden revelations, it was simply inconceivable that any bill designed to gut a major national security program could receive more than a handful of
votes. But the final vote tally on the Conyers-Amash bill shocked official Washington: it failed by just a tiny margin, 205217. Support for it was wholly bipartisan, with 111 Democrats joining 94 Republicans to vote
for the bill. This discarding of traditional partyline divisions was as exciting to Snowden and me as the substantial support for reining in the NSA. Official Washington depends upon blind tribalism engendered by

there is much more hope for policy making


based on the actual interests of the citizenry. Over the following months, as more and more NSA stories were published around the world, many
pundits predicted that the public would lose interest in the subject. But, in fact, interest in the surveillance discussion only intensified , not
rigid partisan warfare. If the red versus blue framework can be eroded, and then transcended,

just domestically but internationally. The events of a single week in December 2013more than six months after my first report appeared in the Guardianillustrate just how much Snowdens disclosures continue
to resonate and just how untenable the NSAs position has become. The week began with the dramatic opinion issued by US federal judge Richard Leon that the NSA metadata collection was likely to be found in
violation of the Fourth Amendment to the US Constitution, denouncing it as almost Orwellian in scope. As noted, the Bush-appointed jurist pointedly added that the government had not cited a single instance in
which analysis of the NSAs bulk metadata collection had stopped a terrorist attack. Just two days later, President Obamas advisory panel, formed when the NSA scandal first broke, issued its 308-page report.
That report, too, decisively rejected the NSAs claims about the vital importance of its spying. Our review suggests that the information contributed to terrorist investigations by the use of [the Patriot Acts] section
215 telephony meta-data was not essential to preventing attacks, the panel wrote, confirming that in not a single instance would the outcome have been different without the section 215 telephony meta-data
program.

Challenge Govt by Challenge Policy


The government is powerful, we can choose to challenge it by combatting
its policies.
Greenwald 2014
(Glenn Edward Greenwald is an American lawyer, journalist and author. He was a columnist for Guardian US from August 2012 to
October 2013. He was a columnist for Salon.com from 2007 to 2012, and an occasional contributor to The Guardian. ; No Place to
Hide ; May 13, 2014 ; Chapter 5: The Fourth Estate ; AWEY)

Other steps, too, can be taken to reclaim online privacy and limit state surveillance. International
efforts currently being led by Germany and Brazilto build new Internet infrastructure so that most network traffic no longer has to transit the United States could go a long

individuals also have a role to play in reclaiming their own


online privacy. Refusing to use the services of tech companies that collaborate with the NSA
and its allies will put pressure on those companies to stop such collaboration and will
spur their competitors to devote themselves to privacy protections. Already, a number of European tech
way toward loosening the American grip on the Internet. And

companies are promoting their email and chat services as a superior alternative to offerings from Google and Facebook, trumpeting the fact that they do notand will not

to prevent governments from intruding into personal


communications and Internet use, all users should be adopting encryption and
browsing-anonymity tools. This is particularly important for people working in sensitive areas, such as journalists, lawyers, and human rights activists.
provide user data to the NSA. Additionally,

And the technology community should continue developing more effective and user-friendly anonymity and encryption programs. On all of these fronts, there is a great deal of

less than a year after I first met Snowden in Hong Kong, there is no question
that his disclosures have already brought about fundamental, irreversible changes in many countries and many
work still to be done. But

realms. And beyond the specifics of NSA reform, Snowdens acts have also profoundly advanced the cause of government transparency and reform in general. He has created

e Obama administration,
which has brought more prosecutions against leakers than all prior presidencies
combined, has sought to create a climate of fear that would stifle any attempts at whistleblowing. But Snowden has destroyed that template. He has managed to remain free, outside
the grasp of the United States; whats more, he has refused to remain in hiding but proudly came
forward and identified himself. As a result, the public image of him is not a convict in orange
jumpsuit and shackles but an independent, articulate figure who can speak for himself,
explaining what he did and why. It is no longer possible for the US government to distract from the message simply by demonizing the messenger. There is a
powerful lesson here for future whistle-blowers: speaking the truth does not have to
destroy your life. And for the rest of us, Snowdens inspirational effect is no less profound. Quite simply, he
has reminded everyone about the extraordinary ability of any human being to change the
world. An ordinary person in all outward respects raised by parents without particular wealth or power, lacking even a high school diploma, working as an obscure
employee of a giant corporationhe has, through a single act of conscience, literally altered the course of history. Even the most committed
activists are often tempted to succumb to defeatism. The prevailing institutions seem too
powerful to challenge; orthodoxies feel too entrenched to uproot; there are always many parties with a vested interest in maintaining the status quo. But
it is human beings collectively, not a small number of elites working in secret, who can
decide what kind of world we want to live in. Promoting the human capacity to reason and make decisions: that is the purpose of
a model to inspire others, and future activists will likely follow in his footsteps, perfecting the methods he embraced. Th

whistle-blowing, of activism, of political journalism. And thats what is happening now, thanks to the revelations brought about by Edward Snowden.

Nuclear Weapons Impact


Terrorism threats are all fear mongering and justifies many NSA programs
used for economic espionage and surveillance
Rice 15 (Rebecca, University of Montana, Resisting NSA Surveillance: Glenn Greenwald and
the public sphere debate about privacy, pg online @
http://scholarworks.umt.edu/cgi/viewcontent.cgi?article=5439&context=etd)//akim
Creation of a Permanent Emergency Greenwald also explores how

the surveillance community stifles discussion . He says that

the community draws on appeals to fear , particularly of terrorism, to keep the public
complacent. These fear appeals confine debate to technical experts and keep debate away from
the public sphere. This idea functions similarly to Taylor's (2007) discussion of the creation of a permanent
emergency which prevents deliberation about US nuclear policy. Taylor (2007) says that secrecy
limits public knowledge of nuclear matters, and this limitation is in turn used to justify
excluding an 'uninformed' public from subsequent deliberation. Additionally, 'national security' is commonly
invoked to discourage public debate of nuclear policy on the assumption that such debate
might damage national security itself (p. 173). Greenwald attributes squelched discussion to claims of national security as well, and asserts that
prevention of terrorism as
justification for the NSA's power. The post-9/11 American veneration of security above all else has created a climate particularly
conducive to abuses of power, Greenwald writes as he introduces his encounter with Snowden (2014, p. 2). In NPTH, Greenwald says Surveillance
cheerleaders essentially offer only one argument in defense of mass surveillance: it is only carried out to stop terrorism and keep
people safe. Indeed, invoking an external threat is a historical tactic of choice to keep the population
these claims are unjustified. First, Greenwald says that surveillance advocates have repeatedly cited

submissive to government powers... Ever since the 9/11 attack, US officials reflexively produce the word
'terrorism' (2014, p. 202). Linking surveillance to a powerful fear appeal, like terrorism, keeps citizens complacent, according to
Greenwald. Greenwald says the government often claims there is a need for secrecy surrounding
these programs in order to protect national security. Every time a lawsuit is brought contesting the legality of
intercepting Americans' communications without warrants, the Obama DOJ raises claims of secrecy, standing and immunity to prevent any such
determination from being made, he writes in an article for The Guardian (2013d, p. 5). Taylor (2007) says that the US government often cites
the need for secrecy and exigency as reasons to limit public discussion on nuclear
weapons . Post-9/11, the Bush administration linked these arguments to rogue states and terrorists (Taylor, 2007). Greenwald says that the threat of terrorism is
invoked to keep surveillance away from public scrutiny. The threat of terrorism, along with the secrecy of proceedings previously discussed, allow the
government to maintain the state of permanent emergency with little evidence of the effectiveness
of surveillance. Greenwald, however, finds these claims to be exaggerated. He says, What is perhaps so remarkable about the
bottomless exploitation of the threat of terrorism is that it is so plainly exaggerated. The
risk of any American dying in a terrorist attack is infinitesimal, considerably less than
the chance of being struck by lightning (2014, p. 205). Greenwald finds that statistically, the threat of terrorism
is low. Responding to the argument that the NSA could prevent the next 9/11, Greenwald says the implication is rank
fearmongering and deceitful in the extreme (2014, p. 204). He points out that the CIA had several reports about al-Qaeda before 9/11, but failed to
act on the warnings. Therefore, collecting more information will not solve the intelligence problems (2014). Additionally, revealing NSA programs
does not threaten national security. When writing his first article based off of Snowden's documents, Greenwald says he contemplated any

any
terrorists capable of tying their own shoes would already know that the government was
trying to monitor their telephone communications (2014, p. 66). Greenwald argues that the fear of terrorism is overblown, and
that reporting about the NSA would not harm national security because terrorists are already
on guard against surveillance. Greenwald also says that many NSA programs are not being used to
prevent terrorism. Greenwald says that a substantial number of the agency's activities have nothing to do with antiterrorism efforts or even with national
security. Much of the Snowden archive revealed what can only be called economic espionage (2014, p. 134). This refers to NSA
documents that revealed economic spying on foreign energy companies. This spying
takes place in order to gain enormous advantage for American industry by providing
economic and trade information, which the NSA has supplied to the State Department
and others during negotiations with other countries (2014, p. 138-139). Greenwald argues that terrorism is an exaggerated
threats to national security, but believed the idea that 'terrorists' would benefit from exposing the order [to authorize Boundless Informant] was laughable:

threat, and is not the sole motivation for NSA surveillance. These arguments call into question the NSA's main justification for their unchecked power. Greenwald makes the
first move toward dismantling the technical sphere isolation of surveillance by claiming that Rice 36 arguments in favor of this isolation are invalid. The creation of a
permanent emergency stifles public discussion, keeping surveillance within the realm of technical deliberation. Greenwald thus spends time dismantling the illusion of the
constant threat of terrorism, which he sees as created to protect and expand governmental power.

General

Legal Debates Good


Simulated legal debates are crucial for social transformation---teaching
legal precision is net-better for eliminating oppression even if one-shot
legal solutions dont work the first time
Klare 11
(Karl Klare, George J. & Kathleen Waters Matthews Distinguished University Professor,
Northeastern University School of Law, Teaching Local 1330Reflections on Critical Legal
Pedagogy, (11). School of Law Faculty Publications. Paper 167.
http://hdl.handle.net/2047/d20002528)
Most
beginning students have found themselves in the situation of wanting to express their moral
intuitions in the form of legal arguments but of feeling powerless to do so. A common
attitude of Northeastern students is that a lawyer cannot turn moral and political convictions
into legal arguments in the context of case-litigation. If you are interested in directly pursuing a moral and/or
political agenda, at a minimum you need to take up legislative and policy work, and more likely
you need to leave the law altogether and take up grass roots organizing instead. I insist that we keep the
By now it has begun to dawn that one of the subjects of this class session is how lawyers translate their moral intuitions and sense of justice into legal arguments.

focus on litigation for this class period. After the straw poll, I ask the students to simulate the role of Staughton Lynd s legal assistants and to assume that the court has just
definitively rejected the claims based on contract, promissory estoppel, and the notion of a community property right. However, they should also assume, counter-factually, that
Judge Lambros stayed dismissal of the suit for ten days to give plaintiffs one last opportunity to come up with a theory. I charge the students with the task of making a
convincing common law argument, supported by respectable legal authority, that the plaintiffs were entitled to substantial relief. Put another way, I ask the students to prove that
Judge Lambros was mistakenthat he was legally wrongwhen he concluded that there was no basis in existing law to vindicate the workers and community s rights. In

All students are asked


to simulate the role of plaintiffs counsel and to make the best arguments they caneither because
they actually believe such arguments and/or because in their simulated role they are fulfilling
their ethical duty to provide zealous representation. A recurring, instant reflex is to say: its simplethe workers human rights were violated
some classroom exercises, I permit students to select the side for which they wish to argue, but I do not allow that in this session.

in the Youngstown case. I remind the class that the challenge I set was to come up with a common law theory. The great appeal of human rights discourse for today s students
is that it seems to provide a technical basis upon which their fervent moral and political commitments appear to be legally required. What human rights? I ask. The usual
answers are (1) they had a right to be treated like human beings or (2) surely there is some human right on which they can base their case. To the first argument I respond:
well, how they are entitled to be treated is exactly what the court is called upon in this case to decide. Counsel may not use a re-statement of the conclusion you wish the court
to reach as the legal basis supporting that conclusion. To the second response I reply: it would be nice if some recognized human right applied, but we are in the Northern
District of Ohio in 1980. Can you cite a pertinent human rights instrument? (Answer: no.) The students then throw other ideas on the table. Someone always proposes that
U.S. Steels actions toward the community were unconscionable. I point out that unconscionability is a defense to contract enforcement whereas the plaintiffs were seeking to
enforce a contract (the alleged promise not to close the plant if it were rendered profitable). In any case, we have assumed that the judge has already ruled that there was no
contract. Another suggestion is that plaintiffs go for restitution. A restitution claim arises when plaintiff gives or entrusts something of value to the defendant, and the defendant
wrongfully refuses to pay for or return it. But here we are assuming that Judge Lambros has already ruled that the workers did not endow U.S. Steel with any property or value
other than their labor power for which they were already compensated under the applicable collective bargaining agreements. If the community provided U.S. Steel with value in
the nature of tax breaks or infrastructure development, the effect of Judge Lambros ruling on the property claim is to say that these were not investments by the community but
no-strings-attached gifts given in the hope of attracting or retaining the companys business. At this point I usually give a hint by saying, if we ve ruled out contract claims, and
weve ruled property claims, what does that leave? Aha, torts! A student then usually suggests that U.S. Steel committed the tort of intentional infliction of emotional distress
(IIED).15 I point out that, even if it were successful, this theory would provide plaintiffs relief only for their emotional injuries, but not their economic or other losses, and most
likely would not provide a basis for an injunction to keep the plant open. In any event, IIED is an intentional tort. What, I ask, is the evidence that U.S. Steel intends the plant
shutdown to cause distress? The response that they should know that emotional distress will result is usually not good enough to make out an intentional tort. An astute
student will point out that in some jurisdictions it is enough to prove that the defendant acted with reckless disregard for the likelihood that severe emotional distress would
result. I allow that maybe theres something to that, but then shift ground by pointing out that a prima facie requirement of IIED is that the distress suffered go beyond what an
ordinary person may be expected to endure or beyond the bounds of civilized behavior.16 Everyone knows that plants close all the time and that the distress accompanying
job-loss is a normal feature of American life. A student halfheartedly throws out negligent infliction of emotional distress, to which my reply is: In what way is U.S. Steel s
proposed conduct negligent? The problem we are up against here is precisely that the corporation is acting as a rational profit-maximizer. A student always proposes that
plaintiffs should allege that what U.S. Steel did was against public policy. First of all, I say, public policy is not a cause of action; it is a backdrop against which conduct or
contract terms are assessed. Moreover, what public policy was violated in this case? The student will respond by saying it is against public policy for U.S. Steel to leave the
community devastated. I point out once again that that is the very conclusion for which we are contendingit is circular argument to assert a statement of our intended

One ineffective theory after another is put on


the table. Only once or twice in the decades I have taught this exercise have the students
gotten close to a viable legal theory. But this is not wasted time learning occurs in this phase of the
exercise. The point conveyed is that while law and morals/politics are inextricably
intertwined, they are not the same. For one thing, lawyers have a distinct way of talking about
conclusion as the rationale for that conclusion. This dialogue continues for awhile.

and analyzing problems that is characteristic of the legal culture of a given time and place. So-called legal reasoning is
actually a repertoire of conventional, culturally approved rhetorical moves and countermoves deployed by lawyers to create an appearance of the legal necessity of the results for which they contend.
In addition, good lawyers actually possess useful, specialized knowledge not generally absorbed by
political theorists or movement activists . Legal training sensitizes us to the many
complexities that arise whenever general norms and principles are implemented in the
form of rules of decision or case applications. Lawyers know, for example, that large stakes may turn on
precisely how a right is defined , who has standing to vindicate it, what remedies it provides, how
the right is enforced and in what venue(s), and so on. We are not doing our jobs properly if we argue, simply, what
the defendant did was unjust and the plaintiff deserves relief. No one needs a lawyer to make the what the
defendant did was unjust argument. As Lynds account shows, the workers of Youngstown did make that argument in their own, eloquent words and through their collective

If what the defendant did was unjust is all we have to offer, lawyers
bring no added value to the table. Progressive students sometimes tell themselves that law is
basically gobbledygook, but that you can assist movements for social change if you learn how to spout the right gobbledygook. In this view of legal practice,
resistance to the shut-downs.

creativity consists in identifying an appropriate technicality that helps your client. But in the Youngstown situation, we are way past that nave view. There is no technicality

a social justice lawyer must use the bits and pieces lying around to
generate new legal knowledge and new legal theories. And these new theories must say
something more than my client deserves to win (although it is fine to commence ones research on the basis of that moral
that can win the case. In this setting,

intuition). The class is beginning to get frustrated, and around now someone says well, what do you expect? This is capitalism. There s no way the workers were going to win.
The this-is-capitalism (TIC) statement sometimes comes from the right, sometimes from the left, and usually from both ends of the spectrum but in different ways. The TIC
statement precipitates another teachable moment. I begin by saying that we need to tease out exactly what the student means by TIC, as several interpretations are possible.
For example, TIC might be a prediction of what contemporary courts are most likely to do. That is, TIC might be equivalent to saying that it doesn t matter what theory you
come up with; 999 US judges out of 1,000 would rule for U. S. Steel.17 I allow that this is probably true, but not very revealing. The workers knew what the odds were before
they launched the case.

Even if doomed to fail , a legal case may still make a contribution to social

justice if the litigation creates a focal point of energy around which a community can
mobilize, articulate moral and political claims, educate the wider public, and conduct
political consciousness-raising. And if there is political value in pursuing a case, we might as well make good legal arguments. On an alternative
reading, the TIC observation is more ambitious than a mere prediction. It might be a claim that a capitalist society requires a legal structure of a certain kind, and that therefore
professionally acceptable legal reasoning within capitalist legal regimes cannot produce a theory that interrogates the status quo beyond a certain point. Put another way, some
outcomes are so foreign to the bedrock assumptions of private ownership that they cannot be reached by respectable legal reasoning. A good example of an outcome that is

This reading of the TIC


comment embodies the idea that legal discourse is encased within a deeper, extra-legal structure
given by requirements of the social order (capitalism), so that within professionally responsible
legal argument the best lawyers in the world could not state a winning theory in Local 1330. Ironically,
the left and the right in the class often share this belief. I take both conservative and progressive students on about this. I
insist that the claim that our law is constrained by a rigid meta-logic of capitalismwhich curiously parallels the
notion that legal outcomes are tightly constrained by legal reasoning is just plain wrong . Capitalist societies recognize all
sorts of limitations on the rights of property owners. Professor Singers classic article catalogues a multitude of them.18
The claim is not only false, it is a dangerous falsehood. To believe TIC in this sense is to
limit in advance our aspirations for what social justice lawyering can accomplish . Now the class
incompatible with capitalism, so the argument goes, is a court order interfering with U.S. Steel s decision to leave Youngstown.

begins to sense that I am not just playing law professor and asking rhetorical questions to which there are no answers. The students realize that I actually think that I have a
theory up my sleeve that shows that Judge Lambros was wrong on the law. If things are going well, the students begin to feel an emotional stake in the exercise. Many who
voted in the straw poll that the plaintiffs deserved to win are anxious to see whether I can pull it off. Other students probably engage emotionally for a different reasonthe ones
who have been skeptical or derisive of my approach all term hope that my theory, when I eventually reveal it, is so implausible that I will fall flat on my face. I begin to feed the
students more hints. One year I gave the hint, What do straying livestock, leaking reservoirs, dynamite blasting, and unsafe products have in common?but that made it too
easy. Usually my hints are more oblique, as in does anything you learned about accident law ring a bell? Whatever the form, the students take the hints, and some start
cooking with gas. Over the next few minutes, the pieces usually fall into place. The legal theory toward which I have been steering the students is that U.S. Steel is strictly liable
in tort for the negative social effects of its decision to disinvest in Youngstown. I contend that that is what the law provided in Ohio in 1980, and therefore a mechanism was
available for the District Court to order substantial relief. A basic, albeit contested theme of modern tort law, which all students learn in first year, is that society allows numerous
risky and predictably harmful activities to proceed because we deem those activities, on balance, to be worthwhile or necessary. In such cases, the law often imposes liability
rules designed to make the activity pay for the injuries or accidents it inevitably causes. For more than a century, tort rules have been fashioned to force actors to take account
of all consequences proximately attributable to their actions, so that they will internalize the relevant costs and price their products accordingly. The expectation is that in the
ordinary course of business planning, the actor will perform a cost/benefit analysis to make sure that the positive values generated by the activity justify its costs. Here, I remind
the students of the famous Learned Hand Carroll Towing formula19 comparing B vs. PL, where B represents the costs of accident avoidance (or of refraining from the activity
when avoidance is impossible or too costly); and P x L (probability of the harm multiplied by the gravity of the harm) reflects foreseeable accident costs.20 The tort theory that
evolved from this and similar cost/benefit approaches is called market deterrence. The notion is that liability rules should be designed to induce the actor who is in the best
position to conduct this kind of cost/benefit analysis with respect to a given activity to actually conduct it. Such actors will have incentives to make their products and activities
safer and/or to develop safer substitute products and activities.21 Actors will then pass each activitys residual accident costs on to consumers by fractionating and

spreading such costs through their pricing decisions. As a result, prices will give consumers an accurate picture of the true social costs of the activity, including its accident
costs. Consumers are thus enabled to make rational decisions about whether to continue purchasing the product or activity in light of its accident as well as its production costs.
In principle, if a particular actor produces an unduly risky product (in the sense that its accident costs are above market level), that actor s products will be priced above
market, and he/she will be driven out of business.22 Tort rules have long been crafted with an eye toward compelling risky but socially valuable activities or enterprises to
internalize their external costs. My examplesto which the students were exposed in first yearare the ancient rule imposing strict liability for crop damage caused by escaping
livestock;23 strict liability under the doctrine of Rylands v. Fletcher for the escape of dangerous things brought onto one s property;24 strict liability under Restatement (Second)
519 for damage caused by abnormally dangerous activities such as dynamite blasting;25 and most recently, strict products liability.26 Of course, there are many exceptions
to this approach. For example, unavoidably unsafe or Comment k products are deemed non-defective and therefore do not carry strict liability. And of course the U.S. largely
rejected Rylands. Why was that? Because, as was memorably stated in Losee v. Buchanan: We must have factories, machinery, dams, canals and railroads. They are
demanded by the manifold wants of mankind, and lay at the basis of all our civilization.27 In assuming that entrepreneurial capitalism would be stymied if enterprises were
obliged to pay for the harms they cause, the Losee court accepted a strong version of TIC. Time permitting, I touch briefly on the debate about whether the flourishing of the
negligence principle in the U.S. subsidized 19th century entrepreneurial capitalism,28 the possible implications of the Coase Theorem for our discussion of Local 1330,29 and
the debate about whether it is appropriate for courts to fashion common law rules with an eye toward their distributive as well as efficiency consequences.30 With this as
background, I argue that the District Court should have treated capital mobilityinvestors circulation of capital in search of the highest rate of returnas a risky but socially
valuable activity warranting the same legal treatment as straying cattle and dynamite blasting. Capital mobility is socially valuable. It is indispensable for economic growth and
flexibility. Capital mobility generates important positive externalities for winners, such as economic development and job-creation at the new site of investment. However,
capital mobility also predictably causes negative external effects on bystanders (the ones economists quaintly label the losers). We discussed some of these externalities at
the outset of the classthe trauma associated with income interruption and pre-mature retirement, waste or destruction of human capital, multiplier effects on the local
economy, and social pathologies and community decline of the kind experienced in Youngstown. The plaintiffs should have argued that capital mobility must internalize its social
dislocation costs for reasons of economic efficiency, and that this can be accomplished by making investors strictly liable in tort for the social dislocation costs proximately
caused by their capital mobility decisions. An investor considering shifting capital from one use to another will compare their respective rates of return. In theory, the investment
with the higher return is socially optimal (as well as more profitable for the individual investor). The higher-return investment enlarges the proverbial pie. But investors must
perform accurate comparisons of competing investment opportunities in order for the magic hand of the market to perform its magic. A rational investor bases her analysis
primarily on price signals reflecting estimated rates of return on alternative investment options. This comparison will yield an irrational judgment leading to a socially suboptimal
investment decision unless the estimated rate of return on the new investment reflects its external effects, both positive and negative. Investors often have public-relations
incentives to tout the positive economic consequences promised at the new location. To guarantee rational decision making, the law must force investors contemplating
withdrawal of capital from an enterprise to also carefully consider the negative social dislocation costs properly attributable to the activity of disinvestment. This can be achieved
by making capital mobility strictly liable for its proximately caused social dislocation costs.31 This approach erects no inefficient barriers to capital mobility, nor does it bar all
disinvestment decisions that may cause disruption and loss in the exit community. Other things being equal, if the new investment discounted by the social dislocation costs of
exit will generate a higher rate of return than the current use of the capital, the capital should be disinvested from the old use and transferred to the new use. However, if
investors are not forced by liability rules to take into account the social dislocation costs of disinvestment, the new investment opportunity will appear more attractive than it
really is in a social sense. The situation involves a classic form of market failure. The market is imperfect because investors are not obliged to take into account the negative
social dislocation costs proximately caused by their decisions. Inaccurate price signals lead to the overproduction of capital movement and therefore to a suboptimal allocation
of resources. Apart from any severance and unemployment benefits received by workers at the old plant, the social dislocation costs of disinvestment are almost entirely
externalized onto the workers and the surrounding community. Strict tort liability will induce investors and their downstream customers to fractionate and spread the dislocation
costs of capital mobility when pricing the products of the new activity. This will provide those who use or benefit from the new activity at the destination community more accurate
signals as to its true social costs and oblige them to fractionally share in the misfortunes afflicting the departure community. Suppose, for example, that U.S. Steel invested the
money it took out of Youngstown toward construction of a modern, high-tech steel mill in a Sunbelt state. The price of steel produced at the new mill should fractionally reflect
social dislocation costs in Youngstown. According to legal common sense and mainstream economic theory, the movement of capital from a lesser to a more profitable
investment is an unambiguous social good. Allowing capital to migrate to its highest rate of return guarantees that society s resources are devoted to their most productive
uses. Society as a whole is better off if capital is permitted freely to migrate to the new investment and there to grow the pie. In short, the free mobility of capital maximizes
aggregate welfare. We are all winners in the long run, even if some unfortunate losers might get hurt along the way. It follows as an article of faith that any legal inhibition on
the mobility of capital is inefficient and socially wasteful. This is why mainstream legal thinking refuses to accord long-term workers or surrounding communities any sort of
property interest in the enterprise which a departing investor is obliged to buy out before removal.32 An unwritten, bed-rock assumption of US law is that capital is not and
should not be legally responsible for the social dislocation costs occasioned by its mobility.33 Such costs are mostly externalized onto employees and the surrounding
community, even if the exit community had subsidized the old investment with tax breaks and similar forms of corporate welfare. The legal common sense about capital mobility
is mistaken. It is not a priori true that the movement of capital toward the greatest rate of return unambiguously enhances aggregate social welfare. Free capital mobility
maximizes aggregate welfare and allocates resources to their most productive uses only in a perfect market; that is, only in the absence of market failure. The claim that free
capital mobility is efficient is sometimes true, and sometimes it is not. It all depends on the particular facts and circumstances on the ground. Voil. Judge Lambros was wrong.
In 1980, a mechanism did exist in our law to recognize the plaintiffs claims and afford them substantial relief for economic, emotional, and other losses.34 All that was required
was a logical extension of familiar torts thinking. Had Judge Lambros correctly applied well-known and time-honored torts principles, he would have treated the social dislocation
costs of the plant closure as an externality that must be embedded in U.S. Steels calculations regarding the relative profitability of the old and new uses to which it might put its
capital. This would close the gap between private and social costs, thereby tending to perfect the market. Notice an important rhetorical advantage of this theoryits core value
is economic efficiency. The plaintiffs can get this far along in their argument without mentioning fairness, equity, or justice, let alone human rights, values that are often
fatal to legal argument in U.S. courts today.35 I now brace myself for the you gotta be kidding me phase of the discussion. Objections cascade in. The progressive students
want to be convinced that this is really happening. The mainstream students want to poke holes and debunk. A few of them are grateful at last for an opportunity to show how
misguided they always knew my teaching was. Always, students assert that my summary discussion of the cost/benefit analysis omitted various costs and benefits. For
example, one year I omitted to say that the social dislocation costs in the exit community must be discounted by ameliorative public expenditures such as unemployment
insurance benefits. My response to this type of objection is always the same: you are absolutely right, that cost or benefit should be included in the analysis. And here are a few
more considerations we would need to address to perfect the cost/benefit analysis which I left out only in the interest of time. But I learn from this discussion; not infrequently,
students contribute something I had not previously considered. A frequent objection is that the task of quantifying the social dislocation costs associated with capital mobility is
just too complicated and difficult. I concede that it is a complex task and that conservative estimates might be required in place of absolute precision. I ask, however, whether it
is preferable to allow investors to proceed on the basis of price-signals we know to be wrong or to induce them to use best efforts to arrive at fair estimates. Separation of
powers always comes up, as it should. I go through the usual riffs. Yes, I concede, these problems cry out for a comprehensive legislative solution rather than case-by-case
adjudication. But standard, well-known counter-arguments suggest that Judge Lambros should nevertheless have imposed tort liability in this case. For one thing, determining
the rules of tort liability has always been within the province of courts. Deferring to the status quo (that those who move capital are not legally responsible for negative
externalities) is every bit as much a choice, every bit as much activism or social engineering, as altering the status quo. Legal history is filled with cases in which the
legislature was only prompted to address an important public policy concern by the shock value of a court decision. Particularly is this so in cases involving the rights and
interests of marginalized, insular, and under-represented groups like aging industrial workers. I note that Congress eventually responded to the plant closing problem with the
WARN Act, a modest but not unimportant effort to internalize to enterprises some of the social dislocation costs of capital disinvestment. The statute liquidates these costs into a
sum equal to sixty days pay after an employer orders a plant closing or mass layoff without giving proper notice.36 I call the students attention to the provision of WARN
barring federal courts from enjoining plant closings37 and ask why Congress might have included that restriction. Another common objection concerns causation. A student will
say: The closedown of the mills, let alone the shutdown of any particular plant, could not have caused all of the suicides, heart failures, domestic violence, and so on, in
Youngstown. Surely many such tragedies would have occurred anyway, even if U.S. Steel had remained. It isn t fair to impose liability on U.S. Steel for everything bad that
happened in Youngstown during the statute-of-limitations period. I immediately say that this is a terrific point, and that I was hoping someone would raise it. I compliment the
student by saying that the question shows that he/she is now tapping legal knowledge. Typically, the class is concerned with causation-in-fact or but for causation. Their
question is, how do we know that a plant shutdown caused any particular case of heart failure or suicide in Youngstown? Problems of causal uncertainty are a familiar issue,
and I remind students that they were exposed to several well-known responses in Torts. A time-honored, if simplistic device is to shift the burden of proof regarding
causationinfact to the defendant, when everyone knows full well that the defendant has no more information than the plaintiff with which to resolve the problem of causal
uncertainty.38 In recent decades, courts have developed more sophisticated responses to problems of causal uncertainty as, for example, in the DES cases. As the court stated
in Sindell:39 In our contemporary complex industrialized society, advances in science and technology create fungible goods which may harm consumers and which cannot be
traced to any specific producer. The response of the courts can be either to adhere rigidly to prior doctrine, denying recovery to those injured by such products, or to fashion
remedies to meet these changing needs. Just as Justice Traynor in his landmark concurring opinion in Escola . . . recognized that in an era of mass production and complex
marketing methods the traditional standard of negligence was insufficient to govern the obligations of manufacturer to consumer, so should we acknowledge that some
adaptation of the rules of causation and liability may be appropriate in these recurring circumstances . . . .40 At this point, some of the progressive students are beginning to
salivate. They came to law school with the hope that legal reasoning would provide them a highly refined and politically neutral technology for speaking truth to power. The first
semester disabuses most of them of that crazy idea. They have learned that they will not find certainty or answers in legal discourse, and that legal texts are minefields of gaps,

conflicts, and ambiguities with moral and political implications. I can tell from the glint in their eyes that they are beginning to ask themselves whether this economics stuff, which
they formerly shunned like the plague, might provide a substitute toolbox of neutral technologies with which to demonstrate that redress for workers and other subordinated and
marginalized groups is legally required. I cannot allow them to think that. Therefore, unless an alert student has spotted it, I now reveal my Achilles heel. The weak link in my
argument is the age-old question of proximate causation. Assume we solve the causation-in-fact problem. For example, assume that by analogy to the Sindell theory of marketshare liability, the court arrives at a fair method of attributing to the plant shutdown some portion of the social trauma and injuries occurring in the wake of U.S. Steel s departure
from Youngstown. How do we know whether the plant closing proximately caused these harms? What do we mean by proximate causation anyway, and why does it matter?
These questions present another exciting, teachable moment. Naturally, the students havent thought about proximate cause since first year. They barely remember what it is
and how it differs from causation-in-fact. Some 3Ls shuffle uncomfortably knowing that the Bar examination looms, and they are soon going to need to know about this. I provide
a quick review of proximate causation which addresses the question, how far down the chain of causation should liability reach? I illustrate my points by referring to Palsgraf v.
Long Island R.R,41 which all law students remember. Perhaps U.S. Steel might fairly be held accountable for the suicide of steelworkers within ninety days of the plant closing,
but we might draw the line before holding U.S. Steel liable for a stroke suffered by a steelworker s spouse five years later. Now keyed in to what proximate cause doctrine is
about, the students eagerly wait for me to tell them what the answer is, that is, where proximate causation doctrine would draw the line in the Youngstown case. That s when I
give them the bad news. I explain that proximate causation doctrine does not provide a determinate analytical method for measuring the scope of liability. We pretend that
buzzwords like reasonable foreseeability or scope-of-the-risk give us answers, but ultimately decisions made under the rubric of proximate causation are always value
judgments.42 The conclusion that X proximately caused Y is a statement about the type of society we want to live in. At this juncture, the 3Ls grumpily realize that I am not
going to be much help in preparing them for their bar review course. I now distribute a one-page hand-out on proximate causation prepared in advance. The handout reprints
Justice Andrews remarkable observation in his Palsgraf dissent: What we . . . mean by the word proximate is, that because of convenience, of public policy, of a rough sense
of justice, the law arbitrarily declines to trace a series of events beyond a certain point. This is not logic. It is practical politics . . . . It is all a question of expediency. There are no
fixed rules to govern our judgment. There are simply matters of which we may take account.43 I point out that causation-in-fact analysis, too, always involves perspective and
value judgments.44 Why assume that water escaping the reservoir diminished the value of the neighboring coal mining company s land? Why not assume that the coal
companys decision to dig close to the border diminished the value of the manufacturer s land (by increasing the cost of using the type of reservoir needed in its production
process)? For that matter, why assume that the cattle trample on the neighbors crops? Why not assume that the crops get in the way of the cattle? My handout also contains
my variation on Robert Keetons famous definition of proximate cause45: When a court states that the defendant s conduct was the proximate cause of (some portion of) the
plaintiffs injuries, what the court means is that (1) the defendant s conduct was a cause-in-fact of that portion of plaintiff s injuries; and (2) the defendant s conduct and the
plaintiffs specified injuries are so related that it is appropriate, from the moral and social-policy points of view, to hold the defendant legally responsible for that portion of the
plaintiffs injuries. What we mean when we ask whether the social dislocation costs associated with the shutdown of the steel plant were proximately caused by capital mobility
is whether these costs are, in whole or in part, properly attributable from a moral/political point of view to U.S. Steel s decision to disinvest. Economic science does not and
cannot establish in a value-neutral manner that the social dislocation costs of the plant shutdown are a negative externality of capital mobility. A conclusion of that kind requires a
value judgment that we disguise under the rubric of proximate causation, a value judgment about whom it is appropriate to ask to bear what costs related to what injuries. The

in legal reasoning there is no escape from moral and political choice

lesson is that
. If things have gone
according to plan, time conveniently runs out, and the class is dismissed on that note. What am I trying to accomplish in a class like this? What are the objectives of critical legal

Legal education should empower students. It should put them in touch with their
own capacity to take control over their lives and professional education and
development. It should enable them to experience the possibility of participating, as lawyers, in
transformative social movements. But all too often classroom legal education is deadening. The law students job, mastering doctrine, appears
utterly unconnected to any process of learning about oneself or developing ones moral, political, or professional identity. Classroom legal education
tends to reinforce a sense of powerlessness about our capacity to change social
institutions. Indeed, it often induces students to feel that they are powerless to shape and alter
their own legal education. Much of legal education induces in students a pervasive and
pedagogy?

exaggerated sense of the constraint of legal rules and roles and the students inability to do
much about it. In capsule form, the goals of critical legal pedagogy are to disrupt the
socialization process that occurs during legal education; to unfreeze entrenched habits of mind and
deconstruct the false claims of necessity which constitute so-called legal reasoning; to urge students to see
their lifes work ahead as an opportunity to unearth and challenge law s dominant ideas
about society, justice, and human possibility and to infuse legal rules and practices with
emancipatory and egalitarian content ; to persuade students that legal discourses and
practices comprise a medium, neither infinitely plastic nor inalterably rigid, in which they
can pursue moral and political projects and articulate alternative visions of social
organization and social justice; to train them to argue professionally and respectably for the utopian and
the impossible; to alert them that legal cases potentially provide a forum for intense public
consciousness-raising about issues of social justice; to encourage them to view legal
representation as an opportunity to challenge, push, and relocate the boundaries between intra-systemic and extra-systemic activity, that is, an
opportunity to work within the system in a way that reconstitutes it ; and to show that the existing
social order is not immutable but is merely possible, and that people have the freedom and
power to act upon it.46 The most important point of the class is that social justice lawyers never
give up. The appropriate response when you think you have a hopeless case is to go back
and do more work in the legal medium.

Fiat Good AT: Agency Turn


Fiat good
Elizabeth SHOVE Sociology @ Lancaster AND Gordon WALKER Geography @ Lancaster
7 [CAUTION! Transitions ahead: politics, practice, and sustainable transition management,
Environment and Planning C 39 (4)]
For academic readers, our commentary argues for loosening the intellectual grip of innovation studies, for backing off from the
nested, hierarchical multi-level model as the only model in town, and for exploring other social scientific, but also systemic theories
of change. The

more we think about the politics and practicalities of reflexive transition


management, the more complex the process appears: for a policy audience, our words of caution could be read as an
invitation to abandon the whole endeavour. If agency, predictability and legitimacy are as limited as weve suggested, this might be
the only sensible conclusion.However, we

are with Rip (2006) in recognising the value, productivity and


everyday necessity of an illusion of agency , and of the working expectation that a
difference can be made even in the face of so much evidence to the contrary. The
outcomes of actions are unknowable, the system unsteerable and the effects of
deliberate intervention inherently unpredictable and, ironically, it is this that sustains
concepts of agency and management. As Rip argues illusions are productive because they
motivate action and repair work, and thus something (whatever) is achieved (Rip 2006: 94).
Situated inside the systems they seek to influence, governance actors and actors of other kinds
as well - are part of the dynamics of change : even if they cannot steer from the outside they
are necessary to processes within . This is, of course, also true of academic life. Here we are, busy
critiquing and analysing transition management in the expectation that somebody somewhere
is listening and maybe even taking notice. If we removed that illusion would we bother
writing anything at all? Maybe we need such fictions to keep us going, and maybe
fiction or no - somewhere along the line something really does happen , but not in ways that we
can anticipate or know.

Warming Debates Good


Its specifically true for climate policy
Mitchell 10
Gordon R. Mitchell is Associate Professor and Director of Graduate Studies in the Department
of Communication at the University of Pittsburgh, where he also directs the William Pitt
Debating Union. Robert Asens patient and thoughtful feedback sharpened this manuscript,
which was also improved by contributions from members of the Schenley Park Debate Authors
Working Group (DAWG), a consortium of public argument scholars at the University of
Pittsburgh that strives to generate rigorous scholarship addressing the role of argumentation
and debate in society. SWITCH-SIDE DEBATING MEETS DEMAND-DRIVEN RHETORIC OF
SCIENCE. MITCHELL, GORDON R.1 Rhetoric & Public Affairs; Spring2010, Vol. 13 Issue 1,
p95-120, 26p
The watchwords for the intelligence communitys debating initiative collaboration, critical thinking, collective awarenessresonate with key terms anchoring the study of
deliberative democracy. In a major new text, John Gastil defines deliberation as a process whereby people carefully examine a problem and arrive at a well-reasoned solution
aft er a period of inclusive, respectful consideration of diverse points of view.40 Gastil and his colleagues in organizations such as the Kettering Foundation and the National
Coalition for Dialogue and Deliberation are pursuing a research program that foregrounds the democratic telos of deliberative processes. Work in this area features a blend of

much of this literature concerns the relationship


between deliberation and debate, with the latter term often loaded with pejorative
baggage and working as a negative foil to highlight the positive qualities of deliberation.42 Most political discussions, however, are debates. Stories in the media turn
concrete interventions and studies of citizen empowerment.41 Notably, a key theme in

politics into a never-ending series of contests. People get swept into taking sides; their energy goes into figuring out who or what theyre for or against, says Kettering president
David Mathews and coauthor Noelle McAfee. Deliberation is different. It is neither a partisan argument where opposing sides try to win nor a casual conversation conducted
with polite civility. Public deliberation is a means by which citizens make tough choices about basic purposes and directions for their communities and their country. It is a way of

distrust of the debate process is almost paradigmatic

reasoning and talking together.43 Mathews and McAfees


amongst
theorists and practitioners of Kettering-style deliberative democracy. One conceptual mechanism for reinforcing this debate-deliberation opposition is characterization of debate
as a process inimical to deliberative aims, with debaters adopting dogmatic and fixed positions that frustrate the deliberative objective of choice work. In this register, Emily
Robertson observes, unlike deliberators, debaters are typically not open to the possibility of being shown wrong. . . . Debaters are not trying to find the best solution by
keeping an open mind about the opponents point of view.44 Similarly, founding documents from the University of HoustonDowntowns Center for Public Deliberation state,
Public deliberation is about choice work, which is different from a dialogue or a debate. In dialogue, people oft en look to relate to each other, to understand each other, and to
talk about more informal issues. In debate, there are generally two positions and people are generally looking to win their side.45 Debate, cast here as the theoretical

The Ketteringframework receives support from perversions of the debate process such as vapid
presidential debates and verbal pyrotechnics found on Crossfire-style television shows.46 In
contrast, the intelligence communitys debating initiative stands as a nettlesome anomaly for these
scapegoat, provides a convenient, low-water benchmark for explaining how other forms of deliberative interaction better promote cooperative choice work.
inspired

theoretical frameworks, with debate serving, rather than frustrating, the ends of deliberation. The presence of such an anomaly would seem to point to the wisdom of fashioning
a theoretical orientation that frames the debate-deliberation connection in contingent, rather than static terms, with the relationship between the categories shift ing along with
the various contexts in which they manifest in practice.47 Such an approach gestures toward the importance of rhetorically informed critical work on multiple levels. First, the
contingency of situated practice invites analysis geared to assess, in particular cases, the extent to which debate practices enable and/ or constrain deliberative objectives.
Regarding the intelligence communitys debating initiative, such an analytical perspective highlights, for example, the tight connection between the deliberative goals established
by intelligence officials and the cultural technology manifest in the bridge projects online debating applications such as Hot Grinds. An additional dimension of nuance emerging
from this avenue of analysis pertains to the precise nature of the deliberative goals set by bridge. Program descriptions notably eschew Kettering-style references to democratic
citizen empowerment, yet feature deliberation prominently as a key ingredient of strong intelligence tradecraft . Th is caveat is especially salient to consider when it comes to the
second category of rhetorically informed critical work invited by the contingent aspect of specific debate initiatives. To grasp this layer it is useful to appreciate how the name of
the bridge project constitutes an invitation for those outside the intelligence community to participate in the analytic outreach eff ort. According to Doney, bridge provides an
environment for Analytic Outreacha place where IC analysts can reach out to expertise elsewhere in federal, state, and local government, in academia, and industry. New
communities of interest can form quickly in bridge through the web of trust access control modelaccess to minds outside the intelligence community creates an analytic force
multiplier.48 This presents a moment of choice for academic scholars in a position to respond to Doneys invitation; it is an opportunity to convert scholarly expertise into an

switch-side
debating should be viewed as a cultural technology in light of Langdon Winners maxim that technological artifacts have
analytic force multiplier. In reflexively pondering this invitation, it may be valuable for scholars to read Greene and Hickss proposition that

politics.49 In the case of bridge, politics are informed by the history of intelligence community policies and practices. Commenter Th omas Lord puts this point in high relief in a
post off ered in response to a news story on the topic: [W]hy should this thing (bridge) be? . . . [Th e intelligence community] on the one hand sometimes provides useful
information to the military or to the civilian branches and on the other hand it is a dangerous, out of control, relic that by all external appearances is not the slightest bit reformed,
other than superficially, from such excesses as became exposed in the cointelpro and mkultra hearings of the 1970s.50 A debate scholar need not agree with Lords fullthroated criticism of the intelligence community (he goes on to observe that it bears an alarming resemblance to organized crime) to understand that participation in the
communitys Analytic Outreach program may serve the ends of deliberation, but not necessarily democracy, or even a defensible politics. Demand-driven rhetoric of science
necessarily raises questions about whats driving the demand, questions that scholars with relevant expertise would do well to ponder carefully before embracing invitations to
contribute their argumentative expertise to deliberative projects. By the same token, it would be prudent to bear in mind that the technological determinism about switch-side
debate endorsed by Greene and Hicks may tend to flatten reflexive assessments regarding the wisdom of supporting a given debate initiativeas the next section illustrates,
manifest differences among initiatives warrant context-sensitive judgments regarding the normative political dimensions featured in each case. Public Debates in the EPA
Policy Process Th e preceding analysis of U.S. intelligence community debating initiatives highlighted how analysts are challenged to navigate discursively the heteroglossia of

Public policy planners are tested in like manner when


they attempt to stitch together institutional arguments from various and sundry inputs
ranging from expert testimony, to historical precedent, to public comment. Just as intelligence managers find that algorithmic, formal methods of
analysis often dont work when it comes to the task of interpreting and synthesizing
copious amounts of disparate data, public-policy planners encounter similar challenges. In fact, the argumentative
turn in public-policy planning elaborates an approach to public-policy analysis that
foregrounds deliberative interchange and critical thinking as alternatives to decisionism, the formulaic application of objective
decision algorithms to the public policy process. Stating the matter plainly, Majone suggests, whether in written or oral form, argument is central in all
stages of the policy process. Accordingly, he notes, we miss a great deal if we try to understand
policy-making solely in terms of power, influence, and bargaining, to the exclusion of debate and argument.51
vast amounts of diff erent kinds of data flowing through intelligence streams.

One can see similar rationales driving Goodwin and Daviss EPA debating project, where debaters are invited to conduct on-site public debates covering resolutions craft ed to

For example,

reflect key points of stasis in the EPA decision-making process.


in the 2008 Water Wars debates held at EPA headquarters in Washington, D.C.,
resolutions were craft ed to focus attention on the topic of water pollution, with one resolution focusing on downstream states authority to control upstream states discharges
and sources of pollutants, and a second resolution exploring the policy merits of bottled water and toilet paper taxes as revenue sources to fund water infrastructure projects. In

Gannon and Seungwon Chung from Wake Forest University argued in favor of
downstream state control, with the Michigan State University team of Carly Wunderlich and Garrett Abelkop providing opposition. In the second debate on
taxation policy, Kevin Kallmyer and Matthew Struth from University of Mary Washington defended taxes on bottled water and
toilet paper, while their opponents from Howard University, Dominique Scott and Jarred McKee, argued against this proposal. Reflecting on the
project, Goodwin noted how the intercollegiate Switch-Side Debating Meets Demand-Driven Rhetoric of Science 107 debaters ability to
act as honest brokers in the policy arguments contributed positively to internal EPA
deliberation on both issues.52 Davis observed that since the invited debaters didnt have a dog in the fight, they were able to give voice to previously buried
arguments that some EPA subject matter experts felt reticent to elucidate because of their institutional affiliations.53 Such findings are consistent
with the views of policy analysts advocating the argumentative turn in policy planning. As
Majone claims, Dialectical confrontation between generalists and experts often succeeds in bringing out
unstated assumptions, conflicting interpretations of the facts, and the risks posed by
new projects.54 Frank Fischer goes even further in this context, explicitly appropriating rhetorical scholar Charles Willards concept of argumentative epistemics
the first debate on interstate river pollution, the team of Seth

to flesh out his vision for policy studies: Uncovering the epistemic dynamics of public controversies would allow for a more enlightened understanding of what is at stake in a
particular dispute, making possible a sophisticated evaluation of the various viewpoints and merits of diff erent policy options. In so doing, the diff ering, oft en tacitly held
contextual perspectives and values could be juxtaposed; the viewpoints and demands of experts, special interest groups, and the wider public could be directly compared; and

This would by no means sideline or even exclude scientific


assessment; it would only situate it within the framework of a more comprehensive
evaluation.55 As Davis notes, institutional constraints present within the EPA communicative milieu can complicate eff orts to provide a full airing of all relevant
arguments pertaining to a given regulatory issue. Thus, intercollegiate debaters can play key roles in retrieving and
amplifying positions that might otherwise remain sedimented in the policy process . Th e
the dynamics among the participants could be scrutizined.

dynamics entailed in this symbiotic relationship are underscored by deliberative planner John Forester, who observes, If planners and public administrators are to make
democratic political debate and argument possible, they will need strategically located allies to avoid being fully thwarted by the characteristic self-protecting behaviors of the
planning organizations and bureaucracies within which they work.56 Here, an institutions need for strategically located allies to support deliberative practice constitutes the

As an instance of rhetoric of
science scholarship, this type of switch-side public 108 Rhetoric & Public Affairs debate diff ers both
from insular contest tournament debating, where the main focus is on the pedagogical benefit for student participants, and first-generation rhetoric of science
scholarship, where critics concentrated on unmasking the rhetoricity of scientific
artifacts circulating in what many perceived to be purely technical spheres of knowledge production.58 As a form of demand-driven
rhetoric of science, switch-side debating connects directly with the communication
fields performative tradition of argumentative engagement in public controversya different route of
theoretical grounding than rhetorical criticisms tendency to locate its foundations in the English fields tradition of
literary criticism and textual analysis.59 Given this genealogy, it is not surprising to learn how Daviss response to the EPAs institutional
demand for rhetorically informed expertise, setting up what can be considered a demand-driven rhetoric of science.

need for rhetorical expertise took the form of a public debate proposal, shaped by Daviss dual background as a practitioner and historian of intercollegiate debate. Davis
competed as an undergraduate policy debater for Howard University in the 1970s, and then went on to enjoy substantial success as coach of the Howard team in the new
millennium. In an essay reviewing the broad sweep of debating history, Davis notes, Academic debate began at least 2,400 years ago when the scholar Protagoras of Abdera

Sophists such as Protagoras


taught Greek students the value of dissoi logoi, or pulling apart complex questions by
debating two sides of an issue.61 Th e few surviving fragments of Protagorass work suggest that his notion of dissoi logoi stood for the principle
(481411 bc), known as the father of debate, conducted debates among his students in Athens.60 As John Poulakos points out, older

that two accounts [logoi] are present about every thing, opposed to each other, and further, that humans could measure the relative soundness of knowledge claims by

engaging in give-and-take where parties would make the weaker argument stronger to activate the generative aspect of rhetorical practice, a key element of the Sophistical
tradition.62 Following in Protagorass wake, Isocrates would complement this centrifugal push with the pull of synerchesthe, a centripetal exercise of coming together
deliberatively to listen, respond, and form common social bonds.63 Isocrates incorporated Protagorean dissoi logoi into synerchesthe, a broader concept that he used flexibly to
express interlocking senses of (1) inquiry, as in groups convening to search for answers to common questions through discussion;64 (2) deliberation, with interlocutors gathering
in a political setting to deliberate about proposed courses of action;65 and (3) alliance formation, a form of collective action typical at festivals,66 or in the exchange of pledges
that deepen social ties.67 Switch-Side Debating Meets Demand-Driven Rhetoric of Science 109 Returning once again to the Kettering-informed sharp distinction between

one sees in Isocratic synerchesthe, as well as in the EPA debating initiative, a fusion of debate with
deliberative functions. Echoing a theme raised in this essays earlier discussion of intelligence tradecraft , such a fusion troubles categorical attempts to
debate and deliberation,

classify debate and deliberation as fundamentally opposed activities. Th e significance of such a finding is amplified by the frequency of attempts in the deliberative democracy
literature to insist on the theoretical bifurcation of debate and deliberation as an article of theoretical faith.

Tandem analysis of the EPA and intelligence

debating initiatives also brings to light

community
dimensions of contrast at the third level of Isocratic synerchesthe, alliance formation. Th
e intelligence communitys Analytic Outreach initiative invites largely one-way communication flowing from outside experts into the black box of classified intelligence analysis.

deliberative alliance building

On the contrary, the EPA debating program gestures toward a more expansive project of
. In this vein, Howard
Universitys participation in the 2008 EPA Water Wars debates can be seen as the harbinger of a trend by historically black colleges and universities (hbcus) to catalyze their
debate programs in a strategy that evinces Daviss dual-focus vision. On the one hand, Davis aims to recuperate Wiley Colleges tradition of competitive excellence in
intercollegiate debate, depicted so powerfully in the feature film The Great Debaters, by starting a wave of new debate programs housed in hbcus across the nation.68 On the
other hand, Davis sees potential for these new programs to complement their competitive debate programming with participation in the EPAs public debating initiative. Th is
dual-focus vision recalls Douglas Ehningers and Wayne Brockriedes vision of total debate programs that blend switch-side intercollegiate tournament debating with forms of
public debate designed to contribute to wider communities beyond the tournament setting.69 Whereas the political telos animating Daviss dual-focus vision certainly embraces
background assumptions that Greene and Hicks would find disconcertingnotions of liberal political agency, the idea of debate using words as weapons70there is little
doubt that the project of pursuing environmental protection by tapping the creative energy of hbcu-leveraged dissoi logoi diff ers significantly from the intelligence communitys
eff ort to improve its tradecraft through online digital debate programming. Such diff erence is especially evident in light of the EPAs commitment to extend debates to public

Having a public debater


argue against their convictions, or confess their indecision on a subject and subsequent embrace of argument as a way to seek clarity,
could shake up the prevailing view of debate as a war of words. Public uptake of the
possibility of switch-sides debate may help lessen the polarization of issues inherent in prevailing
realms, with the attendant possible benefits unpacked by Jane Munksgaard and Damien Pfister: 110 Rhetoric & Public Affairs

debate formats because students are no longer seen as wedded to their arguments. This could transform public debate from a tussle between advocates, with each public

in a Manichean struggle about the truth of their side, to a more inviting exchange
focused on the content of the others argumentation and the process of deliberative exchange.71 Reflection on
the EPA debating initiative reveals a striking convergence among (1) the expressed need
for dissoi logoi by government agency officials wrestling with the challenges of inverted rhetorical situations, (2) theoretical claims
by scholars regarding the centrality of argumentation in the public policy process, and (3) the practical wherewithal of
intercollegiate debaters to tailor public switch-side debating performances in specific ways requested
by agency collaborators. These points of convergence both underscore previously articulated theoretical assertions regarding the relationship of
debate to deliberation, as well as deepen understanding of the political role of deliberation in institutional decision making. But they also suggest how decisions by
debater trying to convince the audience

rhetorical scholars about whether to contribute switch-side debating acumen to meet demand-driven rhetoric of science initiatives ought to involve careful reflection. Such an

policy planning in the argumentative turn is designed to respond to the


weaknesses of formal, decisionistic paradigms of policy planning with situated,
contingent judgments informed by reflective deliberation. Conclusion Dilip Gaonkars criticism of first-generation rhetoric
approach mirrors the way

of science scholarship rests on a key claim regarding what he sees as the inherent thinness of the ancient Greek rhetorical lexicon.72 That lexicon, by virtue of the fact that it
was invented primarily to teach rhetorical performance, is ill equipped in his view to support the kind of nuanced discriminations required for eff ective interpretation and critique
of rhetorical texts. Although Gaonkar isolates rhetoric of science as a main target of this critique, his choice of subject matter Switch-Side Debating Meets Demand-Driven
Rhetoric of Science 111 positions him to toggle back and forth between specific engagement with rhetoric of science scholarship and discussion of broader themes touching on
the metatheoretical controversy over rhetorics proper scope as a field of inquiry (the so-called big vs. little rhetoric dispute).73 Gaonkars familiar refrain in both contexts is a
warning about the dangers of universalizing or globalizing rhetorical inquiry, especially in attempts that stretch the classical Greek rhetorical vocabulary into a hermeneutic
metadiscourse, one pressed into service as a master key for interpretation of any and all types of communicative artifacts. In other words, Gaonkar warns against the dangers of
rhetoricians pursuing what might be called supply-side epistemology, rhetorics project of pushing for greater disciplinary relevance by attempting to extend its reach into farflung areas of inquiry such as the hard sciences. Yet this essay highlights how rhetorical scholarships relevance can be credibly established by outsiders, who seek access to
the creative energy flowing from the classical Greek rhetorical lexicon in its native mode, that is, as a tool of invention designed to spur and hone rhetorical performance.

Analysis of the intelligence community and EPA debating initiatives shows how this is the case,
with government agencies calling for assistance to animate rhetorical processes such as dissoi logoi (debating
different sides) and synerchesthe (the performative task of coming together deliberately for the purpose of
joint inquiry, collective choice-making, and renewal of communicative bonds).74 Th is demand-driven epistemology is diff erent in kind
from the globalization project so roundly criticized by Gaonkar. Rather than rhetoric venturing out from its own academic home to proselytize about its epistemological
universality for all knowers, instead here we have actors not formally trained in the rhetorical tradition articulating how their own deliberative objectives call for incorporation of
rhetorical practice and even recruitment of strategically located allies75 to assist in the process. Since the productivist content in the classical Greek vocabulary serves as a

demand-driven rhetoric of science turns Gaonkars original


critique on its head. In fairness to Gaonkar, it should be stipulated that his 1993 intervention challenged the way rhetoric of science had been done to date,
critical resource for joint collaboration in this regard,

not the universe of ways rhetoric of science might be done in the future. And to his partial credit, Gaonkar did acknowledge the promise of a performance-oriented rhetoric of
science, especially one informed by classical thinkers other than Aristotle.76 In his Ph.D. dissertation on Aspects of Sophistic Pedagogy, Gaonkar documents how the

ancient sophists were the greatest champions 112 Rhetoric & Public Affairs of socially useful
science,77 and also how the sophists essentially practiced the art of rhetoric in a translational, performative register: Th e sophists could not blithely go about their
business of making science useful, while science itself stood still due to lack of communal support and recognition. Besides, sophistic pedagogy was becoming increasingly

dependent on the findings of contemporary speculation in philosophy and science. Take for instance, the eminently practical art of rhetoric. As taught by the best of the sophists,

strength and vitality of


sophistic rhetoric came from their ability to incorporate the relevant information obtained
from the on-going research in other fields.78 Of course, deep trans-historical diff erences make uncritical appropriation of classical
it was not simply a handbook of recipes which anyone could mechanically employ to his advantage. On the contrary, the

Greek rhetoric for contemporary use a fools errand. But to gauge from Robert Harimans recent reflections on the enduring salience of Isocrates, timely, suitable, and eloquent

appropriations can help us postmoderns forge a new political language suitable for
addressing the complex raft of intertwined problems facing global society. Such retrospection is long overdue, says Hariman,
as the history, literature, philosophy, oratory, art, and political thought of Greece and Rome have never been more accessible or less appreciated.79 Th is essay has explored

debate and deliberationcan be retrieved


in the contemporary milieu for cultural technologies capable of dealing
with one of our times most daunting challenges. This challenge involves finding meaning in
inverted rhetorical situations characterized by an endemic surplus of
ways that some of the most venerable elements of the ancient Greek rhetorical traditionthose dealing with
and adapted to answer calls

heterogeneous content.

The skills acquired in the process of debating warming policy are necessary and
sufficient to overcome political apathy and dissuasion
Herbeck and Isham 10
http://www.thesolutionsjournal.com/node/775
Jon Isham
Associate Professor of Economics, Middlebury College
In the fall of 1999, Jon joined the department of economics and the program in environmental
studies at Middlebury College. Jon teaches classes in environmental economics, environmental
policy, introductory microeconomics, social capital in Vermont, and global climate change. Jon is
co-editing a new book, Ignition: The Birth of the Climate Movement; has co-edited Social
Capital, Development, and the Environment (Edward Elgar Publications); has published articles
(several forthcoming) in Economic Development and Cultural Change, The Journal of African
Economies, The Nonprofit and Voluntary Sector Quarterly, The Quarterly Journal of Economics,
Rural Sociology, Society and Natural Resources, The Southern Economic Journal, The Vermont
Law Review, and the World Bank Economic Review; and has published book chapters in
volumes from Ashgate Press, The New England University Press, Oxford University Press, and
Cambridge University Press. His current research focuses on building the new climate
movement; the demand for water among poor households in Cambodia; information
asymmetries in low-income lending; and the effect of local social capital on environmental
outcomes in Vermont.
Herbeck, member of the Rubenstein School of Environment and Natural Resources and the
Honors College.

Getting to 350 parts per million CO2 in the atmosphere will require massive investments in cleanenergy infrastructureinvestments that can too often be foiled by a combination of special interests and political
sclerosis. Take the recent approval of the Cape Wind project by the U.S. Department of the Interior. In some ways, this was great news for clean-energy advocates:
the projects 130 turbines will produce, on average, 170 megawatts of electricity, almost 75 percent of the average electricity demand for Cape Cod and the islands of Marthas
Vineyard and Nantucket.1 But, because of local opposition by well-organized opponents, the approval process was lengthy, costly, and grueling and all for a project that will
produce only 0.04 percent of the total (forecasted) U.S. electricity demand in 2010.2,3 Over the next few decades, the world will need thousands of large-scale, low-carbon

How can the


decision-making process about such projects be streamlined so that public policy reflects the view of a
well-informed majority, provides opportunities for legitimate critiques, but does not
permit the opposition to retard the process indefinitely? One answer is found in a set of innovative
policy-making tools founded on the principle of deliberative democracy, defined as decision making by discussion among
electricity projectswind, solar, and nuclear power will certainly be in the mix. But if each faces Cape Windlike opposition, getting to 350 is unlikely.

free and equal citizens.4 Such approaches, which have been developed and led by the Center for Deliberative Democracy (cdd.stanford.edu), America Speaks
(www.americaspeaks.org), and the Consensus Building Institute (cbuilding.org), among others, are gaining popularity by promising a new foothold for effective citizen

democratic leadership should involve


educating constituents about issues at hand, and that citizens may significantly alter their opinions
when faced with information about these issues. Advocates of the approach state that democracy should shift away
from fixed notions toward a learning process in which people develop defensible
positions.5 While the approaches of the Center for Deliberative Democracy, America Speaks, and the Consensus Building Institute do differ, all of these
deliberative methodologies involve unbiased sharing of information and public-policy
alternatives with a representative set of citizens; a moderated process of deliberation among the selected citizens;
and the collection and dissemination of data resulting from this process. For example, in the deliberative polling approach used by the
participation in the drive for a clean-energy future. Deliberative democracy stems from the belief that

Center for Deliberative Democracy, a random selection of citizens is first polled on a particular issue. Then, members of the poll are invited to gather at a single place to discuss
the issue. Participants receive balanced briefing materials to review before the gathering, and at the gathering they engage in dialogue with competing experts and political
leaders based on questions they develop in small group discussions. After deliberations, the sample is asked the original poll questions, and the resulting changes in opinion
represent the conclusions that the public would reach if everyone were given the opportunity to become more informed on pressing issues.6 If policymakers look at deliberative
polls rather than traditional polls, they will be able to utilize results that originate from an informed group of citizens. As with traditional polls, deliberative polls choose people at
random to represent U.S. demographics of age, education, gender, and so on. But traditional polls stop there, asking the random sample some brief, simple questions, typically

participants of deliberative polls have the opportunity to access expert information and then
talk with one another before voting on policy recommendations. The power of this
approach is illustrated by the results of a global deliberative process organized by World Wide
Views on Global Warming (www.wwviews.org), a citizens deliberation organization based in Denmark.7 On September 26, 2009, approximately 4,000
people gathered in 38 countries to consider what should happen at the UN climate change negotiations in Copenhagen (338 Americans met in five major cities). The
results derived from this day of deliberation were dramatic and significantly different from results of
traditional polls. Overall, citizens showed strong concern about global warming and support for climate-change legislation, contrary to the outcomes of many
online or over the phone. However,

standard climate-change polls. Based on the polling results from these gatherings, 90 percent of global citizens believe that it is urgent for the UN negotiations to produce a new
climate change agreement; 88 percent of global citizens (82 percent of U.S. citizens) favor holding global warming to within 2 degrees Celsius of pre-industrial levels; and 74
percent of global citizens (69 percent of U.S. citizens) favor increasing fossil-fuel prices in developed countries. However, a typical news poll that was conducted two days
before 350.orgs International Day of Climate Action on October 24, 2009, found that Americans had an overall declining concern about global warming.7 How can

deliberative democracy help to create solutions for the climate-change policy process, to accelerate
the kinds of policies and public investments that are so crucial to getting the world on a path to 350? Take again the example of wind in the United States. In the mid-1990s, the
Texas Public Utilities Commission (PUC) launched an integrated resource plan to develop long-term strategies for energy production, particularly electricity.8 Upon learning
about the deliberative polling approach of James Fishkin (then at the University of Texas at Austin), the PUC set up deliberative sessions for several hundred customers in the
vicinity of every major utility provider in the state. The results were a surprise: it turned out that participants ranked reliability and stability of electricity supply as more important
characteristics than price. In addition, they were open to supporting renewable energy, even if the costs slightly exceeded fossil-fuel sources. Observers considered this a
breakthrough: based on these public deliberations, the PUC went on to champion an aggressive renewable portfolio standard, and the state has subsequently experienced little
of the opposition to wind-tower siting that has slowed development in other states.8 By 2009, Texas had 9,500 megawatts of installed wind capacity, as much as the next six

Deliberative
democracy has proven effective in a wide range of countries and settings. In the Chinese township of
states (ranked by wind capacity) in the windy lower and upper Midwest (Iowa, Minnesota, Colorado, North Dakota, Kansas, and New Mexico).9

Zeguo, a series of deliberative polls has helped the Local Peoples Congress (LPC) to become a more effective decision-making body.10 In February 2008, 175 citizens were
randomly selected to scrutinize the towns budgetand 60 deputies from the LPC observed the process. After the deliberations, support decreased for budgeting for national
defense projects, while support rose for infrastructure (e.g., rural road construction) and environmental protection. Subsequently, the LPC increased support for environmental
projects by 9 percent.10 In decades to come, China must be at the forefront of the worlds investments in clean-energy infrastructure. The experience of Zeguo, if scaled up and

Deliberative democracy offers one solution for


determining citizen opinions, including those on pressing issues related to climate change and clean
energy.
fully supported by Chinese leaders, can help to play an important role.

AT: Discourse First


Discourse fails
Schwartz, 9
(Joseph, Poli Sci Prof @ Temple, The Future of Democratic Equality, Routledge, pg 64-5)
'Discursive' performance is not the sole manner by which individuals deal with (and express) the
material and cultural structural realities that both empower and constrain individuals . For
example. individuals cannot readily "discursively perform" themselves out of their socioeconomic or class position. There is a certain materiality to poverty or to being "bossed"
that can't simply be "ironically" and "performatively" transformed. Class relations are structural, as well as
discursive. The greater difficulty in forming unions in the United States- as compared to other advanced industrial democracieshas much to do with American legal, ideological, and political constraints and not simply with the
rel- ative inefficacy of the "performative," "counter-hegemonic" behavior of (frag- mented)
individuals. Even the "parodic" possibilities of "gender" reversal are constrained by the
communities in which one resides. ls the "reversal" of "drag" a viable public possibility
in a violently homophobic community? Were not the "performative" options of a Matthew
Sheperd (extremely) more limited than those of a gay or lesbian student at a "progressive"
residential liberal arts college (and unsafe-and even degrading and violent-social spaces
confront gay and lesbian people and women and students of color in the most allegedly
"cosmopolitan" of social spaces). Simply put, distinct "social spaces" set differ- ential
constraints on "performative" choices.

Of course, how individuals express class, race, gender, and sexuality does, in part, involve how

we "perform" (or "racist") cultural and discursive "norms." Hence, the inevitable controversies over "authenticity" within racial, sexual, and ethnic communities, as well as

there are
material constraints to performative "choice": one can't "perform" one's way out of an
criticism of people taking on the mores of a class different from those who share their "place" in the labor process, neighborhood, or income strata. But

under-funded inner city school or out of being a laid-off auto worker with dim prospects
of finding a new job with comparable wages and benefits. Traditional sociological
theories of "structuration" provide greater insight into how these individuals would deal
with these social dilemmas than do micro-level theories of the discursive construction
of subjectivity.

To her credit, Wendy Brown is more concerned with issues of class and political economy than are many post-structuralist political theorists. She

expressly claims to bring class back into her political analysis and condemns identity poli- tics as a "phantasmagorical reflection of the 'middle-class' American dream." But

there is little attention in her work to developing a political strategy that could promise a structural and
material redistribution of power, rather than an alter- ation of how we think of epistemology,
discourse, and politics." While ideology and culture play a relatively autonomous role in constituting subjectivity, both have a material structure that must
be altered if society is to be democratized. Brown implies that radical social change does not as much involve
democratiz- ing social structural relations as it does popularizing a radical
epistemological approach to discourse. Brown argues that if we will ourselves to "surrender epis- temological foundations" and give up
"specifically moral claims" we will all be able to engage in "the sheerly political: 'wars of position' and amoral contests about the just and good in which truth is always grasped
as coterminous with power, as always already power, as the voice of power."""' Even if one resists asking whether democracy can rest on "amoral" principles, one can still ask

The post-structuralist
hyper-emphasis on "discourse" and the agonal construction of the self also overly devalues the state as an arena
whether Brown's Foucauldian assertation that power and truth are co-terminous can distinguish between more or less democratic forms of power?

for political reform. Brown's work makes a positive political contribution by warning social movements about fetishizing the struggle for group rights within
the law as potential minefields of "reversed" power/knowledge formations. State regulation and technocratic control which claim to defend the interests of newly, legally-

recognized identities may yield the perverse consequence of "domesticating" the identity of the insurgent social group (e.g. state micro-management of the work place in
"comparable worth legislation," or enforcement of patriarchal values in regard to punitive workfare or "child support" regulation)?" Sometimes, as Brown contends, new-found
rights may enhance separation and alienation between and within individuals and groups, as well as constitute new forms of state regulation in the name of the impersonal

Brown rejects the possibility (and historical reality) that new "rights" can, in other contexts,
con- tribute to human emancipation by enhancing individual choice and freedom. To deny this
is to ignore the elective affinity between the struggle for "rights" and struggles to
achieve political equality for formerly subordinate peoples. Not all new-found rights are "co-optative" and a
"reinscribing of domination.""7 Nor will the conflict within the American polity over how we should interpret and defend "rights" ever cease. One only has to
witness contemporary political con- conflict over "abortion rights," "voting rights," "gun rights," etc.
Rights are both politically contested and protective of certain forms of human choice and
agency. Rights do not "fix" identities as intransigently as Brown and other post-structuralists claim. Do rights only serve, as
Brown contends, to promote "the discursive denial of historically layered and institutionally
secured bounds, by denying with words the effects of relatively wordless, politically
invisible, yet material constraints"?"" Patri- cia Williams and other critical race theorists have
argued that being included under the state's equal protection law helped limit violence against people
of color." Despite legitimate fears about excessive state regulation of sexuality, would Brown
reject the use of state force to limit domestic violence? How does her philosophical fear of the
bureaucratic-regulatory powers of the state speak to the experience of hundreds of thousands of women
who have been spared the "privatization'' of domestic violence by the extension of the
rights of state author- ity (e.g. the police) to act against violence within the household? Are such prac- tices
solely evidence of the "reconstruction of domination by the regulation of the technocratic-bureaucratic state"? Of course, state regulation of domestic
violence may, in Brown's language, produce a female subject "dependent upon the pater- nal
state" for protection. But is this not preferable to the prior form of paternal state that let a man
subject. But

be the violent definer of "rights" in his home?

AT: Individual Ethics First


Individual ethical orientations arent effective its more productive to
rearticulate systems from the inside
Pugh 10
(Jonathan, Newcastle Postcolonial Geographer, The Stakes of Radical Politics have Changed:
Post-crisis, Relevance and the State, Globalizations, March-June, ebsco)
some from the radical Left were
incapable of being able to respond to the new stakes of radical politics. In particular, they
were not found at the state, where the passive public turned to resolve the crisis. I will
now go on to examine how in recent years significant parts of the radical Left have also
tended to prioritise raising awareness of our ethical responsibilities, over capturing state
power. I am going to say that it is important to create this awareness. However, in an effort to draw attention to the stakes of politics as we find them now, post-2008, I will also point out that we should not
In this polemical piece I have just been talking about how, following an ethos of radicalism as withdrawal from the state,

place too much faith in this approach alone. Against the backdrop of what I have just been saying, it is important to remember that while much attention is focused upon President Obama, in many other parts of
the world the Right and fundamentalism are gaining strength through capturing state power. The perception that the USA has changed is accompanied by a sense of relief among many radicals.

However, the European Elections of 2009, the largest trans-national vote in history,
heralded a continent-wide shift to the Right (and far Right) in many placesin Austria,
Belgium, Bulgaria, Cyprus, the Czech Republic, Denmark, Estonia, Finland, France,
Germany, Italy, Estonia, Lithuania, Luxembourg, Poland, Portgual, Slovenia, Spain,
Romania, as just some examples (Wall Street Journal, 2009). Despite Obamas election and a near depression, neo-liberalism continues to be implemented through a
world spanning apparatus of governmental and intergovernmental organisations, think tanks and trans-national corporations (Massey, 2009; Castree, 2009). The power of the Right in countries like Iran, while

Albertazzi et al. (2009) draw attention to how a disconnected Left is


leaving power in the hands of the Right in many other countries nationally, like Italy for
example. Reflecting upon contemporary radical politics, the British Labour politician Clare Short (2009, p. 67) concludes: In the fog of the future, I see a rise of fascistic movements . . . I am afraid it will
checked, remains unchallenged by the Left.

all get nastier before we see a rise in generous, radical politics, but I suspect that history is about to speed up in front of our eyes and all who oppose the radicalisation of fear, ethnic hatred, racialism and division
have to be ready to create a new movement that contains the solutions to the monumental historical problems we currently face. So, the stakes of politics are clear. The Right is on the rise. Neo-liberal ideology is
still dominant. How is the Left responding to these stakes? I have already discussed how some from the radical Left are placing too much faith in civil society organisations that seek to withdraw from the state. I

Post-crisis, the increasing popularity of


David Chandlers (2004, 2007, 2009a, 2009b) work reflects the sense that radicals too
often celebrate the ethical individual as a radical force, at the expense of wider
representational programmes for change. His central argument is that this leaves
will now turn to how others have too much faith in the power of raising awareness of our ethical responsibilities.

radicals impotent . Chandler (2009a, p. 7879) says that many radicals argue that there is nothing passive or conservative about radical political activist protests, such as the 2003 antiwar march, anti-capitalism and anti-globalisation protests, the huge march to Make Poverty History at the end of 2005, involvement in the World Social Forums or the radical jihad of Al-Qaeda. I disagree;

these new forms of protest are highly individualised and personal ones there is no
attempt to build a social or collective movement . It appears that theatrical suicide,
demonstrating, badge and bracelet wearing are ethical acts in themselves: personal
statements of awareness, rather than attempts to engage politically with society. In one way,
Chandlers reflective insight here is not particularly unique. Many others also seem to think that radicals today are too isolated and disengaged (Martin, 2009).5 Neither is it particularly original to say that there is
too much emphasis upon creativity and spontaneity (what Richard Sennett, 2004, calls social jazz), and not enough upon representational politics. Indeed, go to many radical blogs and you find radicals
themselves constantly complaining about how it has become too easy to sign up to ethical web petitions, email complaints, join a variety of ethical causes, without actually developing the political programmes
themselves that matter. So it is not Chandlers point about radicals being disengaged from instrumental politics that concerns me here. It is his related pointthat there has been a flight into ethics, away from
political accountability and responsibility that I find intriguing. Personal statements of ethical awareness have become particularly important within radical politics today. It is therefore interesting to note, as I will
now discuss, that we have been here before. In his earlier writings Karl Marx (1982) criticised the German Idealists for retreating into ethics, instead of seizing the institutions of power that mattered for

Unwilling to express their self-interests politically through capturing power, the


Idealists would rather make statements about their ethical awareness. Such idealism,
along with an unwillingness to be held accountable for political power, often goes hand
in hand. For Marx, it is necessary to feel the weight, but also the responsibility of power. Chandler argues that, just as when the early Marx critiqued German Idealism, we should now be drawing
themselves.

attention to the pitfalls of the flights to ethics today. He says: In the case of the German bourgeoisie, Marx concludes that it is their weakness and fragmentation, squeezed between the remnants of the ancien re
gime and the developing industrial proletariat, which explains their ideological flight into values. Rather than take on political responsibility for overthrowing the old order, the German bourgeoisie denied their
specific interests and idealised progress in the otherworldly terms of abstract philosophy, recoiling from the consequences of their liberal aspirations in practice. (Chandler, 2007, p. 717) Today we are witnessing a

Fragmented, many radicals retreat into abstract ethical slogans


like another world is possible, global human rights, or making poverty history. As discussed
renewed interest in ethics (Ladi, 1998; Badiou, 2002).

above, we are also of course seeing the return of Kants cosmopolitanism. While I think we should not attack the ethical turn for its values, as many of these around environmental issues and human rights are

, it is equally important to say that the turn to ethics seems to reflect a certain lack of
willingness to seize power and be held accountable to it. For the flight to ethics, as it often plays out in radical politics today, seems to
be accompanied by scepticism toward representational politics. Continuing with this theme for a moment, Slavoj Zizek (2008) also sheds some more light upon why ethics (when compared to
representational politics) has become so important to the Left in recent years. He says that many of us (he is of course writing for the Left) feel that we are
unable to make a real difference through representational politics on a larger scale, when it comes to the big political problems of life. Zizek (2008, p. 453) talks of this
admirable

feeling that we cannot ever predict the consequences of our acts; that nothing we do will guarantee that the overall outcome of our interactions will be satisfactory. And he is right to make this point. Today, our
geographical imaginations are dominated by a broader sense of chaos and Global Complexity (Urry, 2003; Stengers, 2005). These ways of thinking, deep in the psyche of many radicals on the Left may be one
other reason why so many have retreated into ethics. When we do not really believe that we can change the world through developing fine detailed instruments, capturing the state, or predictive models, we are
naturally more hesitant. It is better to try and raise ethical awareness instead. Whereas in the past power was something to be won and treasured, something radicals could use to implement a collective ideology,
today, with the risk posed by representation in fragmented societies, top-down power often becomes a hazard, even an embarrassment, for many on the Left (Ladi, 1998). This is, as I have already discussed,

Putting what I have just said another way, there is a


need to be clear, perhaps more so in these interdisciplinary timesethics and politics
(particularly representational politics) are different. Of course they are related. You cannot do politics without an ethical perspective. But
where the Right and neo-liberal ideologues are seizing the opportunity of the moment.

my point here is that the Right and neo-liberal ideologues will not simply go away if the
Left adopt or raise awareness of alternative ethical lifestyles. The Right are willing to
capture state power, particularly at this time when the state is increasingly powerful. When we compare the
concerted political programme of neo-liberalism, first developed by Reagan, Thatcher, the IMF, the World Bank, NATO, multi-national banks, and the G20, as just some of many examples, ethical individuals

But the 2008 crisis, and the response of protests like the Alternative
G20, demonstrated how weak ethical resistance is in the face of the institutions of the
neo-liberal economy. Another reason for this is because the ethical individual contributes
so much to neo-liberal societies themselves. To explain how, we must briefly step back. The new social movements of previous decades have, in
across the world offer some counter-resistance.

general, been effectively recuperated by the existing system of capital, by satisfying them in a way that neutralised their subversive potential. This is how capital has maintained its hegemonic position in post-

They say the new


social movements desire for autonomy, the ideal of self-management, the antihierarchical exigency, and the search for authenticity, were important in developing postFordism. These replaced the hierarchical framework of the Fordist period with new forms of networked control. And so, in this way, we see that the relationship between new social movements and
Fordist societies. Luc Boltanski and Eve Chiapello (2005) explain how capitalists have worked with, rather than against, the characteristics of new social movements.

capital has been productive. In turn, and this is the important point I want to make about the present moment, clearly the stakes of radical politics have now changed once more. As discussed earlier, it would now

Without the neoliberal state, and the


publics subordination to its actions, it would not now exist in anything like its present
form. Our subordination to the state has stopped a post-crisis implosion of neoliberalism. And this is of course where one of the central characteristics of the ethical
individual has been so productive. Endemic individualism, so dominant in liberal
societies, has been recuperated by the ethical individual who is unwilling to seize the
state. So the salient point here is that the ethical individual is reflective of the
seem that post-Fordist society is actually more hierarchical and controllable than many previously thought.

conservative forces in society today.

Roleplay k Agency
The aff solves Debate roleplaying specifically activates agency
Hanghoj 8
(Thorkild, University of Aarhus School of Education Assistant Professor, PLAYFUL
KNOWLEDGE An Explorative Study of Educational Gaming,
http://static.sdu.dk/mediafiles/Files/Information_til/Studerende_ved_SDU/Din_uddannelse/phd_
hum/afhandlinger/2009/ThorkilHanghoej.pdf)
debate games require teachers to balance the centripetal/centrifugal forces of gaming and
teaching, to be able to reconfigure their discursive authority, and to orchestrate the multiple voices of a dialogical game space in relation to particular goals. These
Thus,

Bakhtinian perspectives provide a valuable analytical framework for describing the discursive interplay between different practices and knowledge aspects when enacting

dialogical philosophy also offers an explanation of why debate


games (and other game types) may be valuable within an educational context. One of the central features of multi-player games is that players
are expected to experience a simultaneously real and imagined scenario both in relation to an
insiders (participant) perspective and to an outsiders (co-participant) perspective. According to Bakhtin, the outsiders
perspective reflects a fundamental aspect of human understanding: In order to understand, it is immensely
(debate) game scenarios. In addition to this, Bakhtins

important for the person who understands to be located outside the object of his or her creative understanding in time, in space, in culture. For one cannot even really see
one's own exterior and comprehend it as a whole, and no mirrors or photographs can help; our real exterior can be seen and understood only by other people, because they are
located outside us in space, and because they are others (Bakhtin, 1986: 7). As the quote suggests, every person is influenced by others in an inescapably intertwined way, and

it is in the interaction with other voices that individuals are


able to reach understanding and find their own voice. Bakhtin also refers to the ontological process
of finding a voice as ideological becoming, which represents the process of selectively assimilating the words of others (Bakhtin, 1981: 341).
Thus, by teaching and playing debate scenarios, it is possible to support students in their
process of becoming not only themselves, but also in becoming articulate and
responsive citizens in a democratic society.
consequently no voice can be said to be isolated. Thus,

SS = Reflexivity
And, Switch-Side debate causes reflexivity and tolerance
Koehle 10
(Joe, Phd candidate in communications at Kansas, former West Georgia debater
http://mccfblog.org/actr/wp-content/uploads/2010/12/Koehle_Paper_ACTR-editedPDF.pdf.)
criticism of switch side debate has been a constant feature

Much like criticism of the sophists has persisted throughout time;


since
the advent of tournament-style debating. Harrigan documents how numerous these criticisms have been in the last century, explaining that Page 15 Koehle 15 complaints about the mode of debate are as old as

The most famous controversy over modern switch side debate occurred in 1954, when the U.S. military
academies and the Nebraska teachers colleges decided to boycott the resolution: Resolved: That the United States should extend diplomatic
the activity itself (9).

relations to the communist government of China. The schools that boycotted the topic argued that it was ethically and educationally indefensible to defend a recognition of communists, and even went so far as to
argue that a pro-recognition stand by men wearing the countrys uniforms would lead to misunderstanding on the part of our friends and to distortion by our enemies (English et al. 221). Switch side debate was
on the defensive, and debate coaches of the time were engaged in virulent debate over the how to debate. The controversy made the national news when the journalist Edward Murrow became involved and

the debate about debate with helping


accelerate the implosion of the famous red- baiting Senator Joseph McCarthy (222). The debate about
opined on the issue in front of millions of TV viewers. English et al. even go so far as to credit

debate fell back out of the national spotlight after the high-profile incident over the China resolution, but it never ended in the debate community itself. The tenor of the debate reached a fever pitch when

outright accusations of modern sophistry (the bad kind) were published in the Spring 1983 edition of the National Forensic Journal, when
Bernard K. Duffy wrote, The Ethics of Argumentation in Intercollegiate Debate: A Conservative Appraisal. Echoing the old Platonic argument against sophistic practice, Duffy
argued that switch side debate has ignored ethical considerations in the pursuit of teaching cheap techniques for victory
(66). The 1990s saw a divergence of criticisms into two different camps. The first camp was comprised of traditional critics who argued that debate instruction and practice promoted form over substance. For
example, a coach from Boston College lamented that absent a change, Debate instructors and their students will become the sophists of our age, susceptible to the traditional indictments elucidated by Isocrates
and others (Herbeck). Dale Bertelstein published a response to the previously cited article by Muir about switch side debate that launched into an extended discussion of debate and sophistry. This article
continued the practice of coaches and communications scholars developing and applying the Platonic critique of the sophists to contemporary debate practices. Alongside this traditional criticism

newer set of critiques of switch side debate emerged.

Armed with the language of Foucauldian criticism, Critical Legal Studies, and
critiques of normativity and statism, many people who were uncomfortable with the debate tradition of arguing in favor of government action began to question the reason why one should ever be obliged to

to argue that switch side debate was a mode of debate that


unnecessarily constrained people to the hegemony of debating the given topic.
advocate government action. They began

These newer
criticisms of switch side debate gained even more traction after the year 2000, with several skilled teams using these arguments to avoid having to debate one side of the topic. William Spanos, a professor of
English at SUNY Binghamton decided to link the ethos of switch side debate to that of neo-conservatism after observing a debate tournament, saying that the arrogant neocons who now saturate the government
of the Bushlearned their disinterested argumentative skills in the high school and college debate societies and that, accordingly, they have become masters at disarming the just causes of the oppressed.
(Spanos 467)

Contemporary policy debate is now under attack from all sides, caught in its own dissoi logoi. Given the variety
how can switch side debate be justified

of assaults upon switch side debate by both sides of the political spectrum,
? Supporters of switch side debate have
made many arguments justifying the value of the practice that are not related to any defense of sophist Page 17 Koehle 17 techniques. I will only briefly describe them so as to not muddle the issue, but they are
worthy of at least a cursory mention. The first defense is the most pragmatic reason of all:

Mandating people debate both sides of a topic is

most fair to participants because it helps mitigate the potential for a topic that is biased towards one side. More theoretical justifications are given, however. Supporters of switch
side debate have argued that encouraging students to play the devil s advocate creates a sense of selfreflexivity that is crucial to promoting tolerance and preventing dogmatism (Muir 287). Others have
attempted to justify switch side debate in educational terms and advocacy terms, explaining that it is a path to diversifying a students knowledge by encouraging them to seek out paths they may have avoided

contemporary policy debate and its reliance upon


switching sides creates an oasis of argumentation free from the demands of advocacy, allowing students to
test out ideas and become more well-rounded advocates as they leave the classroom and enter the polis (Coverstone). Finally, debate empowers
individuals to become critical thinkers capable of making sound decisions (Mitchell, Pedagogical
otherwise, which in turn creates better public advocates (Dybvig and Iversen). In fact,

Possibilities, 41).

Rez as Heuristic
Evaluate the desirability of the plan---using the resolution as a heuristic for
deliberation is necessary to produce democratic citizens able to
responding to manipulation and deception used by those in power---this
does not necessitate excluding conjuring, but does necessitate having a
common background for normative evaluation
Dahlberg 5
(Lincoln The University of Queensland, Center for Critical and Cultural Studies, Visiting Fellow,
The Habermasian public sphere: Taking difference seriously?, Theory and Society (2005)
34:111-126)
I believe this

critique of power, transparency, and the subject is largely based upon a poor characterization
of Habermas position. There are three main misunderstandings that need to be cleared up here, to do with power as negative, as able
to be easily removed, and as able to be clearly identied. First, Habermas does not dene power as simply
negative and as therefore needing to be summarily removed from the public sphere. The public
sphere norm calls for coercion-free communication and not power-free communication .
Habermas emphasizes the positive power of communicative interaction within the public sphere
through which participants use words to do things and make things happen.60 Communicative
rationality draws on the force of better argument to produce more democratic citizens,
culture, and societies. Subjects are indeed molded through this constituting power, but
their transformation is towards freedom and autonomy rather than towards subjugation and
normalization. As Jeffrey Alexander points out, to act according to a norm is not the same as to be
normalized .61 The public sphere norm provides a structure through which critical
reection on constraining or dominating social relations and possibilities for freedom can take
place . As Chambers argues, rational discourse here is about the endless questioning of codes, the reasoned
questioning of normalization.62 This is the very type of questioning critics like Lyotard, Mouffe, and Villa
are engaged in despite claiming the normalizing and repressive power of communicative
rationality. These critics have yet to explain adequately how they escape this performative
contradiction, although they may not be too concerned to escape it.63 The form of power that is to be excluded
from discourse in the public sphere is that which limits and disables democratic participation
and leads to communicative inequalities . Coercion and domination are (ideally) excluded
from the public sphere, which includes forms of domination resulting from the
maldistribution of material and authoritative resources that lead to discursive inequalities . This
emphasis on the ideal exclusion of coercion introduces the second point of clari- cation, that the domination free public sphere is an idealization for
the purposes of critique. Habermas

is more than aware of the fact that, as Nancy Fraser, Mouffe, and Young remind us, coercive
never be completely separated from the

forms of power, including those that result from social inequality, can

public sphere.64 Claims that such power has been removed from any really-existing deliberative arena can only be made by ignoring or
hiding the operation of power. However,
and domination cannot

this does not mean that a reduction in coercion

be achieved. Indeed, this is precisely what a democratic politics must do. To


aid this project, the public sphere conception sets a critical standard for evaluation of

everyday communication . Chambers puts this nicely: Criticism requires a normative backdrop
against which we criticize. Crit-icizing the ways power and domination play themselves out in discourse presupposes a conception of
discourse in which there is no [coercive] power and domination. In other words, to defend the position that there is a mean- ingful difference between
talking and ghting, persuasion and coercion, and by extension, reason and power involves beginning with idealizations. That is, it involves drawing a
picture of undominated discourse.65 However, this discussion of the idealizing status of the norm does notanswer claims that it invokes a transparency
theory of knowledge. Iwould argue that such claims not only fall prey to another performa-tive contradiction of presupposing that the use of rational
discourse can establish the impossibility of rational discourse revealing truth and power but are also based on a poor reading of Habermas theory of

In contrast to the metaphysics of presence, the


differentiation of persusion from coercion in the public sphere does not posit a naive
theory of the transparency of power, and meaning more generally. The public sphere
conception as based upon communicative rationality does not assume a Cartesian
communicative rationality. This is the third point of clarication.

( autonomous, disembodied, decontextualized ) subject who can clearly distinguish


between persuasion and coercion, good and bad reasons, true and untrue claims, and then wholly
re-move themselves and their communications from such inuence. For Habermas, subjects are always
situated within culture. The public sphere is posited upon intersubjective rather than
subject-centered rationality. It is through the process of communicative rationality, and
not via a Cartesian subject, that manipulation, deception, poor reasoning, and so on, are
identied and removed, and by which meanings can be understood and communicated. In
other words, it is through rational-critical communication that discourse moves away from
coercion or non-public reason towards greater rational communication and a stronger public sphere. The circularity
here is not a problem, as it may seem, but is in fact the very essence of democratization: throughthe practice of democracy, democratic practice is
advanced. This democratizing process can be further illustrated in the important and challenging case of social inequalities. Democratic theorists
(bothdeliberative and difference) generally agree that social inequalities al-ways lead to some degree of inequalities in discourse. Thus, the ide-alized
public sphere of full discursive inclusion and equality requires that social inequalities be eliminated. Yet how is social inequality to

The idealization seems wholly in-adequate given contemporary


capitalist systems and associated social inequality. However, it is in the very process of
befullyidentied,letaloneeliminated?

argumentation, even if awed, that the identication and critique of social inequality,
and thus of communicative inequality, is able to develop.

Indeed, public sphere deliberation often comes into

existence when and where people become passionate about social injustice and publicly thematize problems of social inequality. Thus the negative

This is not to
say that subjects are merely effects of discourse, that there are no critical social agents acting in the process. It is
not to say that 125 subjects within discourse cannot themselves identify negative forms of power, cannot reexively
monitor their own arguments, cannot rationally criticize other positions, and so on. They
can, and in practice do, despite the instability of meaning . The point is that this reasoning and
power of social inequality as with other forms of coercion is brought to light and critique by the very discourse it is limiting.

understanding is (provisionally) achieved through the subjects situatedness in


discourse rather than via a pre-discursive abstract subject. As Kenneth Baynes argues, it is through discourse that subjects
achieve a degree of reective distance (what we could call autonomy) from their situations, enabling them
to revise their conceptions of what is valuable or worthy of pursuit,[and]to assess various courses
of action with respect to those ends. 66 Democratic discourse generates civic-oriented selves, intersubjective meanings and understandings, and democratic agreements that can be seen
as the basis of public sovereignty. How-ever, the idea of communicatively produced agreements, which in the public sphere are
known as public opinions, has also come under ex-tensive criticism in terms of excluding difference, criticism that I wantto explore in the next section.

The starting point of discourse is disagreement over


problematic validity claims. However, a certain amount of agreement, or at least mutual

The ends of discourse: Public opinion formation

understanding, is presupposed when interlocutors engage in argumentation. All


communication presupposes mutual understanding on the linguistic terms used that
interlocutors use the same terms in the same way.67 Furthermore, in undertaking rational-critical discourse, according to Habermas

formal pragmatic reconstruction,

interlocutors also presuppose the same formal conditions of

argumentation . These shared presuppositions enable rational-critical discourse to be


undertaken. However, as seen above, meaning is never fixed and understanding is always
partial. Understanding and agreement on the use of linguistic terms and of what it means to be reasonable, reflexive, sincere,
inclusive, non-coercive, etc. takes place within discourse and is an ongoing political process .

Impact Defense

No Endless Warfare
No risk of endless warfare
GRAY 7
(Colin, Director of the Centre for Strategic Studies and Professor of International Relations and
Strategic Studies at the University of Reading, graduate of the Universities of Manchester and
Oxford, Founder and Senior Associate to the National Institute for Public Policy, [July 2007, The
Implications of Preemptive and Preventive War Doctrines: A Reconsideration,
http://www.ciaonet.org/wps/ssi10561/ssi10561.pdf)
7. A policy that favors preventive warfare expresses a futile quest for absolute security. It could do so. Most controversial policies
contain within them the possibility of misuse. In

the hands of a paranoid or boundlessly ambitious political


leader, prevention could be a policy for endless warfare. However, the American political
system, with its checks and balances, was designed explicitly for the purpose of
constraining the executive from excessive folly. Both the Vietnam and the contemporary
Iraqi experiences reveal clearly that although the conduct of war is an executive prerogative, in practice
that authority is disciplined by public attitudes. Clausewitz made this point superbly with his designation of the
passion, the sentiments, of the people as a vital component of his trinitarian theory of war. 51 It is true to claim that power can be,
and indeed is often, abused, both personally and nationally. It is possible that a state could acquire a taste for the apparent swift
decisiveness of preventive warfare and overuse the option. One might argue that the easy success achieved against Taliban
Afghanistan in 2001, provided fuel for the urge to seek a similarly rapid success against Saddam Husseins Iraq. In other words, the
delights of military success can be habit forming. On balance, claim seven is not persuasive, though it certainly contains a germ of
truth. A country with unmatched wealth and power, unused to physical insecurity at homenotwithstanding 42 years of nuclear

we ought
not to endorse the argument that the United States should eschew the preventive war option
because it could lead to a futile, endless search for absolute security. One might as well
argue that the United States should adopt a defense policy and develop capabilities shaped strictly for homeland security
approached in a narrowly geographical sense. Since a president might misuse a military instrument that
had a global reach, why not deny the White House even the possibility of such misuse?
In other words, constrain policy ends by limiting policys military means. This argument has
danger, and a high level of gun crimeis vulnerable to demands for policies that supposedly can restore security. But

circulated for many decades and, it must be admitted, it does have a certain elementary logic. It is the opinion of this enquiry,
however, that the

claim that a policy which includes the preventive option might lead to a
search for total security is not at all convincing. Of course, folly in high places is always possible, which is
one of the many reasons why popular democracy is the superior form of government . It would be absurd to permit
the fear of a futile and dangerous quest for absolute security to preclude prevention as a
policy option. Despite its absurdity, this rhetorical charge against prevention is a stock
favorite among preventions critics. It should be recognized and dismissed for what it is,
a debating point with little pragmatic merit. And strategy, though not always policy, must be
nothing if not pragmatic.

No endless war
Rodwell 5

(Jonathan Rodwell is a PhD student at Manchester Met. researching the U.S. Foreign
Policy of the late 70's / rise of neo-cons and Second Cold War, Trendy But Empty:
A Response to Richard Jackson,
http://www.49thparallel.bham.ac.uk/back/issue15/rodwell1.htm)
if the U.S. and every other nation is continually reproducing identities through
it is a constant and universal phenomenon that fails to help us understand at
all why one result of the othering turned out one way and differently at another time . For
example, how could one explain how the process resulted in the 2003 invasion of Iraq but didnt
produce a similar invasion of Afghanistan in 1979 when that country (and by the logic of the Regan
administrations discourse) the West was threatened by the Evil Empire. By the logical of discourse
To be specific
othering

analysis in both cases these policies were the result of politicians being able to discipline and control the political

how do we
explain that the language of the War on Terror actually managed to result in the eventual
Afghan invasion in 2002? Surely it is impossible to explain how George W. Bush was able to
convince his people (and incidentally the U.N and Nato) to support a war in Afghanistan without
referring to a simple fact outside of the discourse; the fact that a known terrorist in Afghanistan
actually admitted to the murder of thousands of people on the 11h of Sepetember 2001. The point is that if the
discursive othering of an alien people or group is what really gave the U.S. the
opportunity to persue the war in Afghanistan one must surly wonder why Afghanistan.
Why not North Korea? Or Scotland ? If the discourse is so powerfully useful in its own right why could it
not have happened anywhere at any time and more often? Why could the British government not have
been able to justify an armed invasion and regime change in Northern Ireland throughout the
agenda to produce the outcomes. So why were the outcomes not the same? To reiterate the point

terrorist violence of the 1980s? Surely they could have just employed the same discursive trickery as George W.
Bush? Jackson is absolutely right when he points out that the actuall threat posed by Afghanistan or Iraq today may
have been thoroughly misguided and conflated and that there must be more to explain why those wars were
enacted at that time. Unfortunately that explanation cannot simply come from the result of inscripting identity and
discourse. On top of this there is the clear problem that the consequences of the discursive othering are not
necessarily what Jackson would seem to identify. This is a problem consistent through David Campbells original
work on which Jacksons approach is based[iii]. David Campbell argued for a linguistic process that always results
in an other being marginalized or has the potential for demonisation[iv]. At the same time Jackson, building upon
this, maintains without qualification that the systematic and institutionalised abuse of Iraqi prisoners first exposed
in April 2004 is a direct consequence of the language used by senior administration officials: conceiving of terrorist
suspects as evil, inhuman and faceless enemies of freedom creates an atmosphere where abuses become

The only problem is that the process of differentiation does not


actually necessarily produce dislike or antagonism. In the 1940s and 50s even subjected
to the language of the Red Scare its obvious not all Americans came to see the Soviets
as an other of their nightmares. And in Iraq the abuses of Iraqi prisoners are isolated cases, it is not the case
that the U.S. militarily summarily abuses prisoners as a result of language. Surely the massive protest
against the war, even in the U.S. itself, is also a self evident example that the language of evil
and inhumanity does not necessarily produce an outcome that marginalises or
demonises an other. Indeed one of the points of discourse is that we are continually
differentiating ourselves from all others around us without this necessarily leading us to
normalised and tolerated[v].

hate fear or abuse anyone .[vi] Consequently, the clear fear of the Soviet Union during the height of the
Cold War, and the abuses at Abu Ghirab are unusual cases. To understand what is going on we must ask how far can
the process of inscripting identity really go towards explaining them? As a result at best all discourse analysis
provides us with is a set of universals and a heuristic model

Endless violence is not a thing

Chandler 9
(David, Westminster IR professor, War Without End(s): Grounding the Discourse of `Global
War', Security Dialogue, 40.3, SAGE)
Western governments appear to portray some of the distinctive characteristics that Schmitt attributed to motorized partisans, in that the shift
from narrowly strategic concepts of security to more abstract concerns reflects the fact
that Western states have tended to fight free-floating and non-strategic wars of
aggression without real enemies at the same time as professing to have the highest
values and the absolute enmity that accompanies these. The government policy documents and critical
frameworks of global war have been so accepted that it is assumed that it is the
strategic interests of Western actors that lie behind the often irrational policy responses,
with global war thereby being understood as merely the extension of instrumental
struggles for control. This perspective seems unable to contemplate the possibility that
it is the lack of a strategic desire for control that drives and defines global war today.
Very few studies of the war on terror start from a study of the Western actors themselves rather
than from their declarations of intent with regard to the international sphere itself. This
methodological framing inevitably makes assumptions about strategic interactions and grounded interests
of domestic or international regulation and control, which are then revealed to explain the
proliferation of enemies and the abstract and metaphysical discourse of the war on
terror (Chandler, 2009a). For its radical critics, the abstract, global discourse merely reveals the
global intent of the hegemonizing designs of biopower or neoliberal empire, as critiques
of liberal projections of power are scaled up from the international to the global. Radical critics working within a broadly
Foucauldian problematic have no problem grounding global war in the needs of neoliberal or biopolitical governance or US hegemonic designs. These critics have
produced numerous frameworks, which seek to assert that global war is somehow inevitable,
based on their view of the needs of late capitalism, late modernity, neoliberalism or
biopolitical frameworks of rule or domination. From the declarations of global war and practices of military intervention, rationality,
instrumentality and strategic interests are read in a variety of ways (Chandler, 2007). Global war is taken very much on its own terms, with the declarations of Western
governments explaining and giving power to radical abstract theories of the global power and regulatory might of the new global order of domination, hegemony or empire

The alternative reading of global war rendered here seeks to clarify that the declarations
of global war are a sign of the lack of political stakes and strategic structuring of the
international sphere rather than frameworks for asserting global domination . We
increasingly see Western diplomatic and military interventions presented as justified on the basis of valuebased declarations, rather than in traditional terms of interest-based outcomes. This was
as apparent in the wars of humanitarian intervention in Bosnia, Somalia and Kosovo where there was no clarity of objectives and
therefore little possibility of strategic planning in terms of the military intervention or the post-conflict political outcomes as it is in the war on terror
campaigns, still ongoing, in Afghanistan and Iraq. There would appear to be a direct
relationship between the lack of strategic clarity shaping and structuring interventions
and the lack of political stakes involved in their outcome. In fact, the globalization of
security discourses seems to reflect the lack of political stakes rather than the urgency
of the security threat or of the intervention. Since the end of the Cold War, the central
problematic could well be grasped as one of withdrawal and the emptying of contestation
from the international sphere rather than as intervention and the contestation for control.
The disengagement of the USA and Russia from sub-Saharan Africa and the Balkans forms the backdrop to the policy debates about sharing responsibility for stability and the

It is the lack of political stakes in the international


sphere that has meant that the latter has become more open to ad hoc and arbitrary
management of failed or failing states (see, for example, Deng et al., 1996).

interventions as states and international institutions use the lack of strategic imperatives to construct

their own meaning through intervention . As Zaki Ladi (1998: 95) explains: war is not waged necessarily to achieve predefined
objectives, and it is in waging war that the motivation needed to continue it is found. In these cases of which there are very many war is no longer a
continuation of politics by other means, as in Clausewitzs classic model but sometimes the initial expression
of forms of activity or organization in search of meaning. . . . War becomes not the ultimate
means to achieve an objective, but the most efficient way of finding one. The lack of
political stakes in the international sphere would appear to be the precondition for the
globalization of security discourses and the ad hoc and often arbitrary decisions to go
to war.

In this sense,

global wars reflect the fact that the international sphere has been

reduced to little more than a vanity mirror for globalized actors who are freed from
strategic necessities and whose concerns are no longer structured in the form of
political struggles against real enemies . The mainstream critical approaches to global
wars, with their heavy reliance on recycling the work of Foucault, Schmitt and Agamben, appear to invert this reality, portraying the
use of military firepower and the implosion of international law as a product of the high
stakes involved in global struggle, rather than the lack of clear contestation involving the
strategic accommodation of diverse powers and interests .

No global war impact


Teschke 11
(Benno Gerhard, IR prof at the University of Sussex, Fatal attraction: a critique of Carl Schmitt's
international political and legal theory, International Theory (2011), 3 : pp 179-227)
For at the centre of the heterodox partly post-structuralist, partly realist neo-Schmittian analysis stands the conclusion of The Nomos: the thesis of a structural and
continuous relation between liberalism and violence (Mouffe 2005, 2007; Odysseos 2007). It suggests that, in sharp contrast to the liberal-cosmopolitan programme of perpetual
peace, the geographical expansion of liberal modernity was accompanied by the intensification and de-formalization of war in the international construction of liberal-

Liberal world-ordering proceeds via the


conduit of wars for humanity, leading to Schmitt's spaceless universalism. In this perspective, a straight line is drawn
constitutional states of law and the production of liberal subjectivities as rights-bearing individuals.

from WWI to the War on Terror to verify Schmitt's long-term prognostic of the 20th
century as the age of neutralizations and de-politicizations (Schmitt 1993). But this attempt to
read the history of 20th century international relations in terms of a succession of
confrontations between the carrier-nations of liberal modernity and the criminalized foes
at its outer margins seems unable to comprehend the complexities and specificities of
liberal world-ordering, then and now . For in the cases of Wilhelmine, Weimar and fascist
Germany, the assumption that their conflicts with the Anglo-American liberal-capitalist heartland were grounded in an antagonism between liberal modernity
and a recalcitrant Germany outside its geographical and conceptual lines runs counter to the historical evidence . For this reading
presupposes that late-Wilhelmine Germany was not already substantially penetrated by capitalism and fully incorporated into the capitalist world economy, posing the question
of whether the causes of WWI lay in the capitalist dynamics of inter-imperial rivalry (Blackbourn and Eley 1984), or in processes of belated and incomplete liberal-capitalist
development, due to the survival of re-feudalized elites in the German state classes and the marriage between rye and iron (Wehler 1997). It also assumes that the lateWeimar and early Nazi turn towards the construction of an autarchic German regionalism Mitteleuropa or Groraum was not deeply influenced by the international
ramifications of the 1929 Great Depression, but premised on a purely politicalexistentialist assertion of German national identity. Against a reading of the early 20th century
conflicts between the liberal West and Germany as wars for humanity between an expanding liberal modernity and its political exterior, there is more evidence to suggest that

confrontations were interstate conflicts within the crisis-ridden and nationally uneven
capitalist project of modernity. Similar objections and caveats to the binary opposition between the Western discourse of liberal humanity against
non-liberal foes apply to the more recent period. For how can this optic explain that the liberal West coexisted
these

(and keeps coexisting)

with a large number of pliant authoritarian client-regimes

(Mubarak's

Egypt, Suharto's

Indonesia, Pahlavi's Iran, Fahd's

Saudi-Arabia, even Gaddafi's pre-intervention Libya, to name but a few), which were and are

actively managed and supported by the West as anti-liberal Schmittian states of


emergency , with concerns for liberal subjectivities and Human Rights secondary to the
strategic interests of political and geopolitical stability and economic access? Even in the more
obvious cases of Afghanistan, Iraq, and, now, Libya, the idea that Western intervention has to be conceived as an
encounter between the liberal project and a series of foes outside its sphere seems to
rely on a denial of their antecedent histories as geopolitically and socially contested
state-building projects in pro-Western fashion, deeply co-determined by long histories of Western anti-liberal colonial and post-colonial legacies. If these
states (or social forces within them) turn against their imperial masters, the conventional policy expression is blowback. And as the Schmittian
analytical vocabulary does not include a conception of human agency and social forces
only friend/enemy groupings and collective political entities governed by executive
decision it also lacks the categories of analysis to comprehend the social dynamics
that drive the struggles around sovereign power and the eventual overcoming , for example, of
Tunisian and Egyptian states of emergency without US-led wars for humanity . Similarly, it
seems unlikely that the generic idea of liberal world-ordering and the production of
liberal subjectivities can actually explain why Western intervention seems improbable in
some cases (e.g. Bahrain, Qatar, Yemen or Syria) and more likely in others (e.g. Serbia, Afghanistan, Iraq , and Libya).
Liberal world-ordering consists of differential strategies of building, coordinating, and drawing liberal and anti-liberal states into the Western orbit, and overtly or covertly
intervening and refashioning them once they step out of line. These are conflicts within a world, which seem to push the term liberalism beyond its original meaning. The

generic Schmittian idea of a liberal spaceless universalism sits uncomfortably with the
realities of maintaining an America-supervised informal empire, which has to manage a
persisting interstate system in diverse and case-specific ways. But it is this persistence of a worldwide system of
states, which encase national particularities, which renders challenges to American supremacy possible in the first place.

AT: VTL
Life has intrinsic and objective value achieved through subjective
pleasures---its preservation should be an a priori goal
Kacou 8
(Amien WHY EVEN MIND? On The A Priori Value Of Life, Cosmos and History: The Journal of
Natural and Social Philosophy, Vol 4, No 1-2 (2008)
cosmosandhistory.org/index.php/journal/article/view/92/184)
Furthermore, that

manner of finding things good that is in pleasure can certainly not exist in
any world without consciousness (i.e., without life, as we now understand the word)
slight analogies put aside. In fact, we can begin to develop a more sophisticated definition of the concept of pleasure, in the
broadest possible sense of the word, as follows: it is the common psychological element in all
psychological experience of goodness (be it in joy, admiration, or whatever else). In this sense, pleasure can
always be pictured to mediate all awareness or perception or judgment of goodness: there is pleasure in all
consciousness of things good; pleasure is the common element of all conscious
satisfaction. In short, it is simply the very experience of liking things, or the liking of experience, in
general. In this sense, pleasure is, not only uniquely characteristic of life but also, the core expression of
goodness in lifethe most general sign or phenomenon for favorable conscious
valuation, in other words. This does not mean that good is absolutely synonymous
with pleasantwhat we value may well go beyond pleasure. (The fact that we value things needs not
be reduced to the experience of liking things.) However, what we value beyond pleasure remains a
matter of speculation or theory. Moreover, we note that a variety of things that may seem otherwise unrelated are
correlated with pleasuresome more strongly than others. In other words, there are many things the experience
of which we like. For example: the admiration of others; sex; or rock-paper-scissors. But, again,
what they are is irrelevant in an inquiry on a priori valuewhat gives us pleasure is a matter for
empirical investigation. Thus, we can see now that, in general, something primitively valuable is
attainable in living that is, pleasure itself. And it seems equally clear that we have a
priori logical reason to pay attention to the world in any world where pleasure exists.
Moreover, we can now also articulate a foundation for a security interest in our life: since
the good of pleasure can be found in living (to the extent pleasure remains attainable),[17] and only in
living, therefore, a priori, life ought to be continuously (and indefinitely) pursued at least
for the sake of preserving the possibility of finding that good. However, this platitude about the value
that can be found in life turns out to be, at this point, insufficient for our purposes. It seems to amount to very little more than
recognizing that our subjective desire for life in and of itself shows that life

has some objective value. For what


difference is there between saying, living is unique in benefiting something I value
(namely, my pleasure); therefore, I should desire to go on living, and saying, I have a
unique desire to go on living; therefore I should have a desire to go on living , whereas the
latter proposition immediately seems senseless? In other words, life gives me pleasure, says little more than, I like life. Thus, we
seem to have arrived at the conclusion that the fact that we already have some
(subjective) desire for life shows life to have some (objective) value. But, if that is the most we can
say, then it seems our enterprise of justification was quite superficial, and the subjective/objective distinction was uselessfor all we
have really done is highlight the correspondence between value and desire. Perhaps, our inquiry should be a bit more complex.

Life is always valuable

Torchia 2, Professor of Philosophy, Providence College, Phd in Philosophy, Fordham College


(Joseph, Postmodernism and the Persistent Vegetative State, The National Catholic Bioethics
Quarterly Summer 2002, Vol. 2, No. 2,
http://www.lifeissues.net/writers/torc/torc_01postmodernismandpvs1.html)
Ultimately, Aquinas'

theory of personhood requires a metaphysical explanation that is rooted


in an understanding of the primacy of the existence or esse of the human person. For humans
beings, the upshot of this position is clear: while human personhood is intimately connected with a broad
range of actions (including consciousness of oneself and others), the definition of personhood is not based
upon any specific activity or capacity for action, but upon the primacy of esse. Indeed, human
actions would have neither a cause nor any referent in the absence of a stable, abiding self that is rooted in the person's very being. A commitment to
the primacy of esse, then, allows for an adequate recognition of the importance of actions in human life, while providing a principle for the unification
and stabilizing of these behavioral features. In this respect ,

the human person is defined as a dynamic being


which actualizes the potentiality for certain behavior or operations unique to his or her
own existence. Esse thereby embraces all that the person is and is capable of doing. In the final analysis, any attempt to
define the person in terms of a single attribute, activity, or capability (e.g., consciousness) flies in
the face of the depth and multi-dimensionality which is part and parcel of personhood
itself. To do so would abdicate the ontological core of the person and the very center
which renders human activities intelligible. And Aquinas' anthropology, I submit, provides an effective philosophical lens
through which the depth and profundity of the human reality comes into sharp focus. In this respect, Kenneth Schmitz draws an
illuminating distinction between "person" (a term which conveys such hidden depth and profundity) and
"personality" (a term which pertains to surface impressions and one's public image).40 The preoccupation with the latter term, he shows, is
very much an outgrowth of the eighteenth century emphasis upon a human individuality that is understood in terms of autonomy and privacy. This
notion of the isolated, atomistic individual was closely linked with a subjective

focus whereby the "self" became the

ultimate referent for judging reality. By extension, such a presupposition led to the conviction that only self-consciousness
provides a means of validating any claims to personhood and membership in a community of free moral agents capable of responsibilities and worthy
of rights. In

contrast to such an isolated and enclosed conception (i.e., whereby one is a person by virtue of
being "set apart" from others as a privatized entity), Schmitz focuses upon an intimacy which presupposes a
certain relation between persons. From this standpoint, intimacy is only possible through genuine
self-disclosure, and the sharing of self-disclosure that allows for an intimate knowledge
of the other.41 For Schmitz, such a revelation of one's inner self transcends any specific
attributes or any overt capacity the individual might possess.42 Ultimately, Schmitz argues, intimacy is
rooted in the unique act of presencing, whereby the person reveals his or her personal existence. But such a mystery only admits of a metphysical
explanation, rather than an epistemological theory of meaning which confines itself to what is observable on the basis of perception or sense
experience. Intimacy, then, discloses

a level of being that transcends any distinctive properties.


Because intimacy has a unique capacity to disclose being, it places us in touch with the
very core of personhood. Metaphysically speaking, intimacy is not grounded in the
recognition of this or that characteristic a person has, but rather in the simple
unqualified presence the person is.43

Preventing death is the first ethical priority its the only impact you cant
recover from.

Bauman 95 Zygmunt Bauman, University of Leeds Professor Emeritus of Sociology, 1995,


Life In Fragments: Essays In Postmodern Morality, p. 66-71
being-for is like living towards-the-future: a being filled with anticipation, a being aware of the abyss between future foretold and future that will
eventually be; it is this gap which, like a magnet, draws the self towards the Other,as it draws life towards the future, making life into an activity of overcoming, transcending, leaving behind. The
self stretches towards the Other, as life stretches towards the future; neither can
grasp what it stretches toward, but it is in this hopeful and desperate, never
conclusive and never abandoned stretching-toward that the self is ever anew created
and life ever anew lived. In the words of M. M. Bakhtin, it is only in this not-yet accomplished world of anticipation and trial, leaning toward stubbornly an-other Other, that life
The

can be lived - not in the world of the `events that occurred'; in the latter world, `it is impossible to live, to act responsibly; in it, I am not needed, in principle I am not there at all." Art, the Other, the future:
what unites them, what makes them into three words vainly trying to grasp the same mystery, is the modality of possibility. A curious modality, at home neither in ontology nor epistemology; itself, like that
which it tries to catch in its net, `always outside', forever `otherwise than being'. The possibility we are talking about here is not the all -too-familiar unsure-of-itself, and through that uncertainty flawed, inferior
and incomplete being, disdainfully dismissed by triumphant existence as `mere possibility', `just a possibility'; possibility is instead `plus que la reahte' - both the origin and the foundation of being. The hope,
says Blanchot, proclaims the possibility of that which evades the possible; `in its limit, this is the hope of the bond recaptured where it is now lost."' The hope is always the hope of being fu filled, but what
keeps the hope alive and so keeps the being open and on the move is precisely its unfu filment. One may say that the paradox of hope (and the paradox of possibility founded in hope) is that it may pursue
its destination solely through betraying its nature; the most exuberant of energies expends itself in the urge towards rest. Possibility uses up its openness in search of closure. Its image of the better being is
its own impoverishment . . . The togetherness of the being-for is cut out of the same block; it shares in the paradoxical lot of all possibility. It lasts as long as it is unfulfilled, yet it uses itself up in never
ending effort of fulfilment, of recapturing the bond, making it tight and immune to all future temptations. In an important, perhaps decisive sense, it is selfdestructive and self -defeating: its triumph is its
death. The Other, like restless and unpredictable art, like the future itself, is a mystery. And being-for-the-Other, going towards the Other through the twisted and rocky gorge of affection, brings that mystery
into view - makes it into a challenge. That mystery is what has triggered the sentiment in the first place - but cracking that mystery is what the resulting movement is about. The mystery must be unpacked
so that the being-for may focus on the Other: one needs to know what to focus on. (The `demand' is unspoken, the responsibility undertaken is unconditional; it is up to him or her who follows the demand
and takes up the responsibility to decide what the following of that demand and carrying out of that responsibility means in practical terms.) Mystery - noted Max Frisch - (and the Other is a mystery), is an
exciting puzzle, but one tends to get tired of that excitement. `And so one creates for oneself an image. This is a loveless act, the betrayal." Creating an image of the Other leads to the substitution of the
image for the Other; the Other is now fixed - soothingly and comfortingly. There is nothing to be excited about anymore. I know what the Other needs, I know where my responsibility starts and ends.
Whatever the Other may now do will be taken down and used against him. What used to be received as an exciting surprise now looks more like perversion; what used to be adored as exhilarating
creativity now feels like wicked levity. Thanatos has taken over from Eros, and the excitement of the ungraspable turned into the dullness and tedium of the grasped. But, as Gyorgy Lukacs observed,
`everything one person may know about another is only expectation, only potentiality, only wish or fear, acquiring reality only as a result of what happens later, and this reality, too, dissolves straightaway

Only death, with its finality and irreversibility, puts an end to the musical-chairs game of the
real and the potential - it once and for all closes the embrace of togetherness which
was before invitingly open and tempted the lonely self." `Creating an image' is the dress rehearsal of that death. But creating an image is the inner urge, the constant temptation, the
into potentialities'.

must of all affection . . . It is the loneliness of being abandoned to an unresolvable ambivalence and an unanchored and formless sentiment which sets in motion the togetherness of being -for. But what
loneliness seeks in togetherness is an end to its present condition - an end to itself. Without knowing - without being capable of knowing - that the hope to replace the vexing loneliness with togetherness is
founded solely on its own unfulfilment, and that once loneliness is no more, the togetherness ( the being-for togetherness) must also collapse, as it cannot survive its own completion. What the loneliness
seeks in togetherness (suicidally for its own cravings) is the foreclosing and pre-empting of the future, cancelling the future before it comes, robbing it of mystery but also of the possibility with which it is
pregnant. Unknowingly yet necessarily, it seeks it all to its own detriment, since the success (if there is a success) may only bring it back to where it started and to the condition which prompted it to start on

The togetherness of being-for is always in the future

the journey in the first place.


, and nowhere else. It is no more once the self
proclaims: `I have arrived', `I have done it', `I fulfilled my duty.' The being-for starts from the realization of the bottomlessness of the task, and ends with the declaration that the infinity has been exhausted.

This is the tragedy of being-for - the reason why it cannot but be death-bound while
simultaneously remaining an undying attraction. In this tragedy, there are many happy moments, but no happy end. Death is
always the foreclosure of possibilities, and it comes eventually in its own time, even if not brought forward by the impatience of love. The catch is
to direct the affection to staving off the end, and to do this against the affection's nature. What follows is that, if moral relationship is grounded in the being-for togetherness (as it
is), then it can exist as a project, and guide the self's conduct only as long as its nature of a project (a not yet-completed project) is not denied. Morality, like the future itself, is forever
not-yet. (And this is why the ethical code, any ethical code, the more so the more perfect it is by its own standards, supports morality the way the rope supports the hanged man.) It is because of our
loneliness that we crave togetherness. It is because of our loneliness that we open up to the Other and allow the Other to open up to us. It is because of our loneliness (which is only belied, not overcome,

it is only through allowing the togetherness its


possibilities which only the future can disclose that we stand a chance of acting
morally, and sometimes even of being good, in the present.
by the hubbub of the being-with) that we turn into moral selves. And

War = Structural Violence


We turn structural violence
Goldstein 1
(Joshua Goldstein, Intl Rel Prof @ American U, 2001, War and Gender, p. 412)
First,

peace activists face a dilemma in thinking about causes of war and working for
peace. Many peace scholars and activists support the approach, if you want peace,
work for justice. Then, if one believes that sexism contributes to war one can work for
gender justice specifically (perhaps among others) in order to pursue peace. This approach brings
strategic allies to the peace movement (women, labor, minorities), but rests on the assumption that injustices
cause war. The evidence in this book suggests that causality runs at least as strongly the
other way. War is not a product of capitalism, imperialism, gender, innate aggression, or
any other single cause, although all of these influence wars outbreaks and outcomes. Rather, war has in part
fueled and sustained these and other injustices.9 So,if you want peace, work for peace.
Indeed, if you want justice (gender and others), work for peace. Causality does not run just upward through the
levels of analysis, from types of individuals, societies, and governments up to war. It runs downward too. Enloe suggests that

changes in attitudes towards war and the military may be the most important way to
reverse womens oppression. The dilemma is that peace work focused on justice brings to the peace movement
energy, allies, and moral grounding, yet, in light of this books evidence, the emphasis on injustice as the main
cause of war seems to be empirically inadequate.

No Prior Questions
No prior questions- scholarly orientations cause gridlock
Owen 2
(David, Southampton political theory reader, Re-orienting International Relations: On
Pragmatism, Pluralism and Practical Reasoning, Millennium 31.3, SAGE)
that [a] frenzy for words like epistemology and
ontology often signals this philosophical turn, although he goes on to comment that these terms are often used loosely.4
Commenting on the philosophical turn in IR, Wver remarks

However, loosely deployed or not, it is clear that debates concerning ontology and epistemology play a central role in the contemporary IR theory wars. In one respect, this is
unsurprising since it is a characteristic feature of the social sciences that periods of disciplinary disorientation involve recourse to reflection on the philosophical commitments of
different theoretical approaches, and there is no doubt that such reflection can play a valuable role in making explicit the commitments that characterise (and help individuate)

such a philosophical turn is not without its dangers and I will briefly mention three before turning to
consider a confusion that has, I will suggest, helped to promote the IR theory wars by motivating this philosophical turn. The first danger with the
philosophical turn is that it has an inbuilt tendency to prioritise issues of ontology and
epistemology over explanatory and/or interpretive power as if the latter two were merely
a simple function of the former. But while the explanatory and/or interpretive power of a
theoretical account is not wholly independent of its ontological and/or epistemological commitments (otherwise criticism of these features would not be
a criticism that had any value), it is by no means clear that it is, in contrast, wholly dependent on these philosophical
commitments. Thus, for example, one need not be sympathetic to rational choice theory
to recognise that it can provide powerful accounts of certain kinds of problems, such as
the tragedy of the commons in which dilemmas of collective action are foregrounded. It
may, of course, be the case that the advocates of rational choice theory cannot give a
good account of why this type of theory is powerful in accounting for this class of problems (i.e., how it is
that the relevant actors come to exhibit features in these circumstances that approximate
the assumptions of rational choice theory) and, if this is the case, it is a philosophical
weaknessbut this does not undermine the point that, for a certain class of problems,
rational choice theory may provide the best account available to us. In other words, while the
critical judgement of theoretical accounts in terms of their ontological and/or
epistemological sophistication is one kind of critical judgement, it is not the only or even
necessarily the most important kind. The second danger run by the philosophical turn is that because prioritisation of
ontology and epistemology promotes theory-construction from philosophical first
principles, it cultivates a theory-driven rather than problem-driven approach to IR. Paraphrasing
Ian Shapiro, the point can be put like this: since it is the case that there is always a plurality of possible true
descriptions of a given action, event or phenomenon, the challenge is to decide which is the most apt in
terms of getting a perspicuous grip on the action, event or phenomenon in question given the purposes of the inquiry; yet, from
this standpoint, theory-driven work is part of a reductionist program in that it dictates always
opting for the description that calls for the explanation that flows from the preferred
model or theory.5 The justification offered for this strategy rests on the mistaken belief that it is
necessary for social science because general explanations are required to characterise
the classes of phenomena studied in similar terms. However, as Shapiro points out, this is to
misunderstand the enterprise of science since whether there are general explanations
for classes of phenomena is a question for social-scientific inquiry, not to be prejudged
before conducting that inquiry.6 Moreover, this strategy easily slips into the promotion of the
pursuit of generality over that of empirical validity. The third danger is that the preceding
diverse theoretical positions. Yet,

combine to encourage the formation of a particular image of disciplinary debate in IRwhat might be called (only slightly tongue in
cheek) the Highlander viewnamely, an image of warring theoretical approaches with each, despite
occasional temporary tactical alliances, dedicated to the strategic achievement of sovereignty over the
disciplinary field. It encourages this view because the turn to, and prioritisation of, ontology and
epistemology stimulates the idea that there can only be one theoretical approach which
gets things right, namely, the theoretical approach that gets its ontology and epistemology right. This image feeds back into IR
exacerbating the first and second dangers, and so a potentially vicious circle arises.
two

Prior questions will never be fully settled---must take action even under
conditions of uncertainty
Cochran 99
(Molly, Assistant Professor of International Affairs at Georgia Institute for Technology, Normative
Theory in International Relations, 1999, pg. 272)
while modernist and postmodernist debates continue, while we are still unsure as to
what we can legitimately identify as a feminist ethical/political concern, while we still are
unclear about the relationship between discourse and experience , it is particularly important for
To conclude this chapter,

that we proceed with analysis of both the material (institutional and structural) as well as the discursive. This
holds not only for feminists, but for all theorists oriented towards the goal of extending further moral
inclusion in the present social sciences climate of epistemological uncertainty.
Important ethical/ political concerns hang in the balance. We cannot afford to wait for the
feminists

meta-theoretical questions to be conclusively answered . Those answers may be


unavailable. Nor can we wait for a credible vision of an alt ernative institutional order to appear
before an emancipatory agenda can be kicked into gear. Nor do we have before us a chicken and egg
question of which comes first: sorting out the metatheoretical issues or working out
which practices contribute to a credible institutional vision. The two questions can and should
be pursued together, and can be via moral imagination. Imagination can help us think beyond discursive and material conditions
which limit us, by pushing the boundaries of those limitations in thought and examining what yields. In this respect, I believe international ethics as pragmatic critique can be a
useful ally to feminist and normative theorists generally.

Prereqs bad
Cochran 99
(Molly, GIT international affairs professor, Normative Theory in International Relations, 272)
To conclude this chapter, while modernist and postmodernist

debates continue, while we are still unsure


as to what we can legitimately identify as a feminist ethical/political concern, while we still
are unclear about the relationship between discourse and experience , it is particularly
important for feminists that we proceed with analysis of both the material (institutional and structural) as well as
the discursive. This holds not only for feminists, but for all theorists oriented towards the goal of
extending further moral inclusion in the present social sciences climate of

epistemological uncertainty. Important

ethical/ political

concerns hang in the balance. We

cannot afford to wait for the meta-theoretical questions to be conclusively answered .


Those answers may be unavailable. Nor can we wait for a credible vision of an alternative
institutional order to appear before an emancipatory agenda can be kicked into gear. Nor do we have
before us a chicken and egg question of which comes first: sorting out the metatheoretical
issues or working out which practices contribute to a credible institutional vision. The
two questions can and should be pursued together, and can be via moral imagination. Imagination can
help us think beyond discursive and material conditions which limit us, by pushing the boundaries of those limitations in thought and examining what
yields. In this respect, I believe international ethics as pragmatic critique can be a useful ally to feminist and normative theorists generally.

Ethics

Extinction First
Extinction impacts must be prioritized-any alternative frame fails in the
context of public policy
Isaac 2
(Jeffrey, political science professor Indiana University, Ends, Means, and Politics, Dissent
Magazine, Spring 2002)
Power is not a dirty word or an unfortunate feature of the world. It is the core of politics. Power is the ability to effect outcomes in the world. Politics, in
large part, involves contests over the distribution and use of power. To

accomplish anything in the political world,


one must attend to the means that are necessary to bring it about. And to develop such means is to
develop, and to exercise, power. To say this is not to say that power is beyond moral- ity. It is to say that power is not reducible to morality. As writers
such as Niccolo Machiavelli, Max Weber, Reinhold Niebuhr, and Hannah Arendt have taught,

an unyielding concern with moral

goodness undercuts political responsibility. The concern may be morally laudable, reflecting a kind of personal integrity,
but it suffers from three fatal flaws: (1) It fails to see that the purity of ones intention does not ensure the
achievement of what one intends. Abjuring violence or refusing to make common cause
with morally compromised parties may seem like the right thing; but if such tactics entail
impotence, then it is hard to view them as serving any moral good beyond the clean conscience of their supporters; (2) it fails to see that in a world of real violence and injustice,
moral purity is not simply a form of powerless- ness; it is often a form of complicity in
injustice. This is why, from the standpoint of politicsas opposed to religionpacifism is always a potentially immoral stand. In
categorically repudiating violence, it refuses in principle to oppose certain violent
injustices with any effect; and (3) it fails to see that politics is as much about unintended
consequences as it is about intentions; it is the effects of action, rather than the motives
of action, that is most significant. Just as the alignment with good may engender
impotence, it is often the pursuit of good that generates evil. This is the lesson of
communism in the twentieth century: it is not enough that ones goals be sincere or
idealistic; it is equally important, always, to ask about the effects of pursuing these goals
and to judge these effects in pragmatic and historically contextualized ways. Moral
absolutism inhibits this judgment. It alienates those who are not true believers. It
promotes arrogance. And it undermines political effectiveness.

AT: Extinction Inevitable


Extinction does not make our efforts worthless.
Trisel 4
[Brooke Alan Trisel, Medical Facilities Consultant with the Ohio Department of Health, "Human
Extinction and the Value of Our Efforts," The Philosophical Forum, Volume XXXV, Number 3,
Fall, 2004, p. 390-391]
Although our works will not last forever, this should not matter if we accomplished what
we set out to do when we created these works. Wanting our [end page 390] creations to endure forever was not likely part of our goal when
we created them. If we accomplish our goals and then later in life conclude that these accomplishments were of no significance, then this is a sign that
a desire for long-lastingness has crept into the standards that we use to judge significance. Escalating desires can lead to escalating standards since
the standards that we establish reflect our goals and desires. Including long-lastingness as a criterion for judging the significance of our efforts is

There
is no need for humanity to live forever for our lives and works to be significant. If the standard that we
unreasonable. If one includes long-lastingness as part of the standard, then one will feel that it is necessary for humanity to persist forever.

adopt for judging significance does not include long-lastingness as part of the standard, then it will not matter whether humanity will endure for a long

We may also be unable to


keep from wanting our works to be appreciated forever. But we can refrain from turning
these desires into standards for judging whether our efforts and accomplishments are significant. If we can keep
time. Like Tolstoy, we may be unable to keep from wanting to have our achievements remembered forever.

from doing this, it will be to our advantage. Then, during those times when we look back on life from an imagined perspective that encompasses times
after humanity has become extinct, we will not conclude that our efforts amounted to nothing. Rather, we will conclude that many
remarkable accomplishments

people made

that made their lives, and possibly the lives of others, better than they would have been if
And if we expand our evaluation, as we should, to take into account all
experiences associated with living, not just goal-related experiences, we will conclude not that life was empty, but
that living was worthwhile.
these goals had never been pursued.

AT: Zizek Ethics


Their dont compromise style ethics are foolishrevolutionary
consistency trades off with compassion and good judgment.
Nicholas BROWN English @ Illinois (Chicago) 4 {,SS} {$}?: Or, Alain Badiou and Slavoj
iek, Waiting for Something to Happen New Centennial Review 4 (3) p. 315-319
Before at we arrive at the political thesis of Zizeks writings on Lenin, it is worth pointing out that Zizeks work is shot through with an ethical imperative which is not often

Zizeks ethics is one of

self-consistency

explicitly thematized. Like Badiou,


subjective
, sustained, like Badious, by Lacans ne pas
cder. This ethical self-consistency is not to be confused with the substantial self-consistency of the subjectwe know the Lacanian barred subject, written S, never fully

a refusal to compromise on ones desire, even if this


desire is ultimately self-destructive. Zizeks contempt for an inconsistent ethical
positionwhere the in-itself of a political position and its for-itself fail to coincideis palpable. Nobody provokes Zizeks ire
so much as pseudo-radical academic Leftists who adopt an attitude of utter disdain towards the Third Way, while their own
coincides with itselfbut rather refers to a kind of going all the way,

radicalism ultimately amounts to an empty gesture which obliges no one to do anything definite. (Revolution, 172) The great majority of todays radical academics silently count
on the long-term stability of the American capitalist model, with a secure tenured position as their ultimate professional goal (a surprising number of them even play the stock

The basis of this contempt is its inconsistency: the pseudo-radical academic


Leftist experiences his position as radical, while in fact it is nothing more than the
empty quietism of Hegels beautiful soul, benefiting from what it denounces while deriving disavowed pleasure from its presumed superiority to the same. There
can be no doubt that the figure of the beautiful soul remains a potent figure for political critique. In fact, one could turn it against Zizek
himself. But what is fundamentally wrong with the beautiful soul is not its inconsistency, but simply the quietism it masks. The consequence of Z
izeks valorization of ethical consistency is a strange admiration for bad but consistent
positions: With regard to this radical chic, our first gesture towards Third Way
ideologists and practitioners should be one of praise: at least they play their game straight, and are
honest in their acceptance of the global capitalist coordinates. (Revolution, 172) Zizek reproduces this gesture
market). (171)

several times, with regard to, for example, the refreshing Francis Fukuyama (In our intellectual space full of false protesters, here we finally have a fully pledged apologist for
the existing order [Organs, 132 n. 23), or, in the introduction to On Belief, his grudging preference for a reactionary fundamentalist preacher over the doctrinal equivocation of a

The problem with this ethicsas Brecht showed us, with ethics in general is that, under
capitalism, the only fully consistent ethical position is ruthless self-interest. There is no ethical
position that is both minimally compassionate and fully ethically consistent. Mauler in Saint Joan is
liberal rabbi and priest.

doomed to make money from all of his generous impulses; the good woman of Szechwan can only help her neighbors by taking advantage of them. In fact, this split constitutes
part of capitalisms dynamism. The ideological force of capitalism is that so many people are given a subjective interest in maintaining the stability of capitalism, even if this

Any opting out is at present simply


quixotic, and only possible on the basis of substantial privilege. Plainly, professors want tenured positions, for the
interest involves competing with neighbors who share an objective interest in ending it.

same reason the unemployed want jobs: because they exist. (As for playing the stock market, this criticism buys neoliberal rhetoric hook, line, and sinker: most academics who
play the stock market do so because universities, like many other U.S. employers, have shifted the burden of risk from their own retirement systems onto the individual
employees.) For the wealthyand this includes the famous tenured radicalself-interest only lines up with political interest in cynical defense of privilege. Our first gesture
towards the pseudoradical academic should therefore be the exact opposite of Zizeks. True, academics by and large live their lives in accordance with their interest in the
long-term stability of Capital; but at least they dont go all the way, contributing to its ideological legitimation in that small zone in which they timidly exercise their freedom.

The only fully consistent leftist ethical position is revolutionary. Zizek has this right, but
currently this position is easier to endorse than to occupy. (Of course, it would be easy to show that Zizeks own
position, or mine, is just as compromised as that of our hypothetical tenured radical. But if we are not going to get upset at liberal hypocrisy, we cant get upset at Zizeks either.
That would be hypocritical.) One of Zizeks hidden targets in Revolution at the Gates is Michael Hardt and Antonio Negris Empire (which appears here only in a footnote, while
it gets fuller treatment in Organs without Bodies). Zizeks main critique (an adaptation of Lukcss critique of utopianism) is simple but damning: their political horizon is
irredeemably abstract. Global citizenship, universal minimum wage, and so on: these are obviously wonderful ideas, but without any concrete proposals for getting there, they

. The more abstract the goal, the easier it is to achieve unanimity:


pretty much everyone can agree that the world ought to be just, but as soon as concrete
proposals for achieving justice are proposed, all unanimity disappears. The notion of the purely immanent,
might as well be anything else or nothing at all

unorganized desire of the multitude is undeniably attractive; but, since this desire is an abstraction from concrete desires that may be incompatible, how could it form itself into
a political subject without the imposition of an organization transcendent to ita Party? Hardt and Negri know this: The only event we are awaiting is the insurgence of a
powerful organization. We do not have any models to offer for this event (2000, 411). The similarity to the Pauline language of the Event as grace is striking, but is this not also
a genuinely shocking admission, a virtual admission of defeat? The event they can only await in silence is the only event that would matter. But in the absence of any attempt to

how does Zizeks question How do we invent the organizational structure


which will confer on [contemporary social unrest] the form of universal political
answer it,

demand? (Revolution, 296)get us any farther than Hardt and Negris empty expectation? While Zizek fully understands the theoretical need for
a Party (and here I am not talking about any particular party or even any particular form of organization, but simply the fact that organization is primary), this exigency remains
completely formal. The closest Zizek comes to attempting an answer to this fundamental question is by invoking (weirdly, it seems to me) a kind of belated cyberspace Utopia,
updating Lenins famous slogan Socialism = electrification + the power of the soviets as Socialism = free access to the internet + the power of the soviets (294). Well, this is
fine as far as it goesinternet technology has a liberatory edge that cannot be realized in a market economybut where are the soviets, exactly? The question itselfwhat is to

the current global political and economic instability may loosen the death grip
of neoliberalism, but it doesnt guarantee that what comes after will be better. Immanuel
Wallerstein even thinks the capitalist world system will collapse of its own internal
contradictions somewhere between 2025 and 2075. This assertion sounds less crazy now than it did a few
years ago, but it isnt very helpful for trying to influence what comes next. Once again, the
attitude is one of waiting until the right conditions emerge. When Heidegger famously
claimed that only a god can save us now, this was to be understood not theologically, but as a deus ex machina: as an
unforeseeable Event. It is a symptom of the weakness of the Left that we effectively agree with
Heidegger. Are we really condemned to await some unexpected turn, some unforeseen development that will emerge like a god from the machine of Capital? If, on
be done?is urgent:

the contrary, the question is how, without any quixotic passage lacte, one is to contribute to the organization of the multitude against Empire, then waiting for a messiah will
only waste time. The equation Lenin = Paulalready implicit in Badiou, made explicit by Zizekis false. To say there was no certainty that the revolution of 1917 would
succeed is true but banal: lots of things are hard to discern without belonging to an indiscernible register of being. Lenins writings do reveal a kind of excessiveness to his
conviction, yes. But they also show an extraordinarily detailed and insightful analysis of the concrete situation, and a remarkable theorization of the political opening offered by
an immanent crisis in Capital (an effort which, if we were really to repeat Lenin today, would have to focus on the inability of Capital to absorb the propertyless masses on its
periphery). What we are offered is not really the equation of Lenin and Paul, but a choice: Lenin or Paul? Do we wait for the Event, or do we organize to bring it about? Badiou,
Hardt and Negri, and Zizekalong with Heideggerall side with Paul. Do we?

Prevent Death Key


Preventing death is the first ethical priority its the only impact you cant
recover from.
Bauman 95
(Zygmunt Bauman, University of Leeds Professor Emeritus of Sociology, 1995, Life In
Fragments: Essays In Postmodern Morality, p. 66-71)
being-for is like living towards-the-future: a being filled with anticipation, a being aware of the abyss between future foretold and future that will
eventually be; it is this gap which, like a magnet, draws the self towards the Other,as it draws life towards the future, making life into an activity of overcoming, transcending, leaving behind. The
self stretches towards the Other, as life stretches towards the future; neither can
grasp what it stretches toward, but it is in this hopeful and desperate, never
conclusive and never abandoned stretching-toward that the self is ever anew created
and life ever anew lived. In the words of M. M. Bakhtin, it is only in this not-yet accomplished world of anticipation and trial, leaning toward stubbornly an-other Other, that life
The

can be lived - not in the world of the `events that occurred'; in the latter world, `it is impossible to live, to act responsibly; in it, I am not needed, in principle I am not there at all." Art, the Other, the future:
what unites them, what makes them into three words vainly trying to grasp the same mystery, is the modality of possibility. A curious modality, at home neither in ontology nor epistemology; itself, like that
which it tries to catch in its net, `always outside', forever `otherwise than being'. The possibility we are talking about here is not the all -too-familiar unsure-of-itself, and through that uncertainty flawed, inferior
and incomplete being, disdainfully dismissed by triumphant existence as `mere possibility', `just a possibility'; possibility is instead `plus que la reahte' - both the origin and the foundation of being. The hope,
says Blanchot, proclaims the possibility of that which evades the possible; `in its limit, this is the hope of the bond recaptured where it is now lost."' The hope is always the hope of being fu filled, but what
keeps the hope alive and so keeps the being open and on the move is precisely its unfu filment. One may say that the paradox of hope (and the paradox of possibility founded in hope) is that it may pursue
its destination solely through betraying its nature; the most exuberant of energies expends itself in the urge towards rest. Possibility uses up its openness in search of closure. Its image of the better being is
its own impoverishment . . . The togetherness of the being-for is cut out of the same block; it shares in the paradoxical lot of all possibility. It lasts as long as it is unfulfilled, yet it uses itself up in never
ending effort of fulfilment, of recapturing the bond, making it tight and immune to all future temptations. In an important, perhaps decisive sense, it is selfdestructive and self -defeating: its triumph is its
death. The Other, like restless and unpredictable art, like the future itself, is a mystery. And being-for-the-Other, going towards the Other through the twisted and rocky gorge of affection, brings that mystery
into view - makes it into a challenge. That mystery is what has triggered the sentiment in the first place - but cracking that mystery is what the resulting movement is about. The mystery must be unpacked
so that the being-for may focus on the Other: one needs to know what to focus on. (The `demand' is unspoken, the responsibility undertaken is unconditional; it is up to him or her who follows the demand
and takes up the responsibility to decide what the following of that demand and carrying out of that responsibility means in practical terms.) Mystery - noted Max Frisch - (and the Other is a mystery), is an
exciting puzzle, but one tends to get tired of that excitement. `And so one creates for oneself an image. This is a loveless act, the betrayal." Creating an image of the Other leads to the substitution of the
image for the Other; the Other is now fixed - soothingly and comfortingly. There is nothing to be excited about anymore. I know what the Other needs, I know where my responsibility starts and ends.
Whatever the Other may now do will be taken down and used against him. What used to be received as an exciting surprise now looks more like perversion; what used to be adored as exhilarating
creativity now feels like wicked levity. Thanatos has taken over from Eros, and the excitement of the ungraspable turned into the dullness and tedium of the grasped. But, as Gyorgy Lukacs observed,
`everything one person may know about another is only expectation, only potentiality, only wish or fear, acquiring reality only as a result of what happens later, and this reality, too, dissolves straightaway

Only death, with its finality and irreversibility, puts an end to the musical-chairs game of the
real and the potential - it once and for all closes the embrace of togetherness which
was before invitingly open and tempted the lonely self." `Creating an image' is the dress rehearsal of that death. But creating an image is the inner urge, the constant temptation, the
into potentialities'.

must of all affection . . . It is the loneliness of being abandoned to an unresolvable ambivalence and an unanchored and formless sentiment which sets in motion the togetherness of being -for. But what
loneliness seeks in togetherness is an end to its present condition - an end to itself. Without knowing - without being capable of knowing - that the hope to replace the vexing loneliness with togetherness is
founded solely on its own unfulfilment, and that once loneliness is no more, the togetherness ( the being-for togetherness) must also collapse, as it cannot survive its own completion. What the loneliness
seeks in togetherness (suicidally for its own cravings) is the foreclosing and pre-empting of the future, cancelling the future before it comes, robbing it of mystery but also of the possibility with which it is
pregnant. Unknowingly yet necessarily, it seeks it all to its own detriment, since the success (if there is a success) may only bring it back to where it started and to the condition which prompted it to start on

The togetherness of being-for is always in the future

the journey in the first place.


, and nowhere else. It is no more once the self
proclaims: `I have arrived', `I have done it', `I fulfilled my duty.' The being-for starts from the realization of the bottomlessness of the task, and ends with the declaration that the infinity has been exhausted.

This is the tragedy of being-for - the reason why it cannot but be death-bound while
simultaneously remaining an undying attraction. In this tragedy, there are many happy moments, but no happy end. Death is
always the foreclosure of possibilities, and it comes eventually in its own time, even if not brought forward by the impatience of love. The catch is
to direct the affection to staving off the end, and to do this against the affection's nature. What follows is that, if moral relationship is grounded in the being-for togetherness (as it
is), then it can exist as a project, and guide the self's conduct only as long as its nature of a project (a not yet-completed project) is not denied. Morality, like the future itself, is forever
not-yet. (And this is why the ethical code, any ethical code, the more so the more perfect it is by its own standards, supports morality the way the rope supports the hanged man.) It is because of our
loneliness that we crave togetherness. It is because of our loneliness that we open up to the Other and allow the Other to open up to us. It is because of our loneliness (which is only belied, not overcome,

it is only through allowing the togetherness its


possibilities which only the future can disclose that we stand a chance of acting
morally, and sometimes even of being good, in the present.
by the hubbub of the being-with) that we turn into moral selves. And

Consequences Good
Their desire to ignore the consequences of their advocacy causes alt
failure ---must evaluate consequences of proposals
Bracey 6
(Christopher A. Bracey 6, Associate Professor of Law, Associate Professor of African & African
American Studies, Washington University in St. Louis, September, Southern California Law
Review, 79 S. Cal. L. Rev. 1231, p. 1318)
reducing conversation on race matters to an ideological contest allows opponents to
elide inquiry into whether the results of a particular preference policy are desirable. Policy
positions masquerading as principled ideological stances create the impression that a racial
policy is not simply a choice among available alternatives, but the embodiment of some
higher moral principle. Thus, the "principle" becomes an end in itself, without reference to
outcomes. Consider the prevailing view of colorblindness in constitutional discourse.
Colorblindness has come to be understood as the embodiment of what is morally just,
Second,

independent of its actual effect upon the lives of racial minorities. This explains Justice Thomas's belief in the "moral and constitutional equivalence" between Jim Crow laws and
race preferences, and his tragic assertion that "Government cannot make us equal [but] can only recognize, respect, and protect us as equal before the law." 281 For Thomas,

Critics may
point out that colorblindness in practice has the effect of entrenching existing racial
disparities in health, wealth, and society. But in framing the debate in purely ideological terms,
opponents are able to avoid the contentious issue of outcomes and make viability
determinations based exclusively on whether racially progressive measures exude
fidelity to the ideological principle of colorblindness. Meaningful policy debate is
replaced by ideological exchange, which further exacerbates hostilities and deepens the
cycle of resentment.
there is no meaningful difference between laws designed to entrench racial subordination and those designed to alleviate conditions of oppression.

State/Law Key

2AC Reformism Good


Reformism is effective and brings revolutionary change closer rather than
pushing it away
Delgado 9
(Richard Delgado 9, self-appointed Minority scholar, Chair of Law at the University of Alabama
Law School, J.D. from the University of California, Berkeley, his books have won eight national
book prizes, including six Gustavus Myers awards for outstanding book on human rights in
North America, the American Library Associations Outstanding Academic Book, and a Pulitzer
Prize nomination. Professor Delgados teaching and writing focus on race, the legal profession,
and social change, 2009, Does Critical Legal Studies Have What Minorities Want, Arguing
about Law, p. 588-590)
CLS critique of piecemeal reform Critical scholars reject the idea of piecemeal reform.
Incremental change, they argue, merely postpones the wholesale reformation that must
occur to create a decent society. Even worse, an unfair social system survives by using
piecemeal reform to disguise and legitimize oppression. Those who control the system
weaken resistance by pointing to the occasional concession to, or periodic court victory of, a
black plaintiff or worker as evidence that the system is fair and just. In fact, Crits believe that teaching the common law
2. The

or using the case method in law school is a disguised means of preaching incrementalism and thereby maintaining the current power structure. To avoid this, CLS scholars urge
law professors to abandon the case method, give up the effort to nd rationality and order in the case law, and teach in an unabashedly political fashion.

The

CLS

critique of piecemeal reform is familiar, imperialistic and wrong . Minorities know from bitter
experience that occasional court victories do not mean the Promised Land is at hand . The critique is imperialistic in that it tells minorities and
other oppressed peoples how they should interpret events affecting them. A court order
directing a housing authority to disburse funds for heating in subsidized housing may
postpone the revolution, or it may not . In the meantime, the order keeps a number of
poor families warm. This may mean more to them than it does to a comfortable academic
working in a warm office. It smacks of paternalism to assert that the possibility of
revolution later outweighs the certainty of heat now, unless there is evidence for that possibility. The Crits do not

incremental changes may bring


revolutionary changes closer , not push them further away. Not all small

offer such evidence. Indeed, some

reforms induce complacency; some may whet the appetite for further combat. The
welfare family may hold a tenants union meeting in their heated living room. CLS scholars
critique of piecemeal reform often misses these possibilities, and neglects the question
of whether total change, when it comes, will be what we want.

2AC Law k for Oppressed


The law isnt perfect but the alts illusory radicalism fails and neglects
political openings to lessen violence
Roth 2k
(Brad R. Roth 2k, Assistant Professor or Legal Studies and Political Science and Adjunct
Professor of Law at Wayne State University, Governmental Illegitimacy and Neocolonialism:
Response to Review by James Thuo Gathii, Michigan Law Review, Vol. 98, No. 6, May 2000,
JSTOR)
Critical" scholars frequently seem to imagine that, in struggling against the methodological
norms of their disciplines, they are struggling against the very structure of the power
relations that exploit and repress the poor and weak the metaphor being, in their minds, somehow
transubstantiated into reality. The result is all too often, an illusory radicalism, rhetorically
"

colorful but programmatically vacuous . The danger is that a fantasized radicalism will
lead scholars to abandon the defense of the very devices that give the poor and weak a
modicum of leverage, when defense of those devices is perhaps the only thing of
practical value that scholars are in a position to contribute.*1 My main problem with Gathii's
critique, then, is not (as he might imagine) that it is political, but that it is politically dysfunctional. More specifically, for all of Gathii's
anticolonial posturing, my book is, I insist, far more effectively anticolonial than is his critique of it. I. The Law and Politics of Governmental Illegitimacy Professor Gathii is fully
justified in subjecting Governmental Illegitimacy in International Law to an essentially political critique, for the book, like all legal scholarship, has political implications in this

Law is a
purposive project, and thus not exclusively an empirical phenomenon; "law as it is"
cannot be wholly separated from "law as it ought to be."5 The purposes that drive the
project, however, must be demonstrably immanent in social reality, not merely
superimposed according to the predilections of the jurist; the jurist's task, at once creative and bounded, is to render a
case, designedly so.* This is not to say, as "critical" scholars sometimes seem to imply, that law or legal scholarship is reducible to ordinary politics.

persuasive account of how those immanent purposes bind powerful actors to worthy projects (such as the self-determination of Third-World peoples) that they would not
otherwise be inclined to undertake." That legal scholarship impress those who are not natural political allies is the test, not only of its scholarly merit, but also of its political merit;

This task is not to everyone's taste, and some in the


academy have devoted their considerable talents to discrediting the project of legal
reasoning, as conventionally understood.7 But their efforts, though often of great
that friends may be disappointed is of far lesser significance.

intellectual sophistication, are profoundly misguided. In their zeal to "unmask" law's


legitimation of exercises of power, they fail to appreciate that law can legitimate such
exercises only insofar as it simultaneously constrains them. Power holders seeking the imprimatur of legality can
benefit only to the extent that they accept its limits, for violation of the limits necessarily reverses the process of legitimation.8 To deny such a
relationship between legitimation and constraint is to assert that putative legal limits are
a remarkably effective ruse that legal rhetoric, rather improbably, fools most of the
people all of the time. (Presumably, the power holders are not thought to be fool- ing themselves, since if the constraints, though objectively illusory, seem
real enough to them, the rule of law would be a reality in politi- cal terms even if a chimera in philosophical terms.) On the other hand, if law does
constrain as well as legitimate the exercise of power, to neglect that point is to miss an
important political opportunity.

2AC Law Good


They are right about the state of the law but that is not a reason to reject
it that only causes nihilistic violence and conservative cooption we
should try to make it effective even if thats impossible
Ristroph 9
(Alice, Associate Professor of Law, Seton Hall University School of Law, Is Law? Constitutional
Crisis and Existential Anxiety, Constitutional Commentary Vol. 25, 431-459.
http://scholarship.law.georgetown.edu/cgi/viewcontent.cgi?article=1457&context=facpub)
law is in crisis concerns our own expectations of the function of law. A possible achievement is to offer an alternative to
violenceas we saw in Levinson and Balkins account of the Constitution as enabling nonviolent dispute resolution.66 This might be called the anti-Thrasymachus view
One reason to care whether

of law. Early in Platos Republic (before Socrates has tamed him), a young man called Thrasymachus describes justice as the advantage of the stronger.67 The claim is that
might makes right, and Western political and legal thought has produced many efforts to prove Thrasymachus and his heirs wrong. If

law distinguishes

right from might , then it becomes important to say what law is, and to show that it exists .
Hence, many ongoing jurisprudential debates about the criteria for a valid and functional
system of law (including worries about legal indeterminancy) are motivated by worries about arbitrary power and
violence.68 To show Thrasymachus to be mistaken, we want to show that the rule of law is
really different from the rule of (the strongest) men. In legal theory, we could view John Austins positivism law as commands
backed by threats of punishmentas a descendant of Thrasymachuss claim.69 Here, I want to examine briefly one of the most influential, and most plausible, efforts to show
that law is something more and different from the commands of a gunman: H. L. A. Harts response to Austin. Hart framed his discussion around the question, What is law?.70
But perhaps, as the Stoppard passage that opened this essay suggests, beginning with this question led us to conjure an image of law with various predicates that do not, as it
turns out, include existence. A second form of existential anxiety, one that I suspect shapes present talk of crisis, is the anxiety thast Thrasymachus and Austin were right and
law, if it is anything more than command and force, does not exist. For my purposes here, the critical features of Harts account are the rule of recognition and the internal point
of view. Since, in most of The Concept of Law, Hart takes laws existence for granted, it is helpful to look at the passages where laws existence, or at least the existence of a
particular form of law, is up for grabs. In his classic discussion of the question, Is international law really law?, H. L. A. Hart deployed the concepts of a rule of recognition and
the internal point of view to conclude that international law was at most in a state of transition toward fully legal law, moving toward law properly so called but certainly not yet
there.71 At the time he wrote The Concept of Law, Hart believed that international law departed from domestic (or municipal) law in that it lacked a widely accepted rule of
recognition and in that states could not be said to take the internal point of view toward international obligations. (Harts argument has been challenged by many contemporary
scholars of international law, but that particular dispute need not occupy us here.72) For law qua law to exist, Hart argued, there must be a rule of recognition under which the
authoritative status of other rules was accepted or denied, and the officials who would apply the rule of recognition must themselves take the internal point of view toward it. That
is, the officials needed to view the rule of recognition as a binding, authoritative guide to their own decisions. Suppose Hart was right and the rule of recognition and the internal
point of view are conditions for the existence of law. Two questions arise: what is the rule of recognition for constitutional law, and who must hold the internal standpoint toward
that rule? The Constitution itself initially seems a candidate for the rule of recognition, though the fact that the Constitution must itself be interpreted leads some theorists to
amend this account and say that the rule of recognition must include authoritative statements of the meaning of the Constitution, under prevailing interpretive standards.73 As
for the internal point of view, we might hope that all state officials would take this point of view toward constitutional rules.74 In other words, we might hope that every state actor
would comply with the U.S. Constitution because it is the Constitution, not simply to avoid injunctions, or judicial invalidation of legislative action, or liability under 42 U.S.C.
1983. But Harts theory does not demand universal adherence to an internal point of view. Even if legislators and other public officials complied with First or Fourth or Fourteenth
Amendment doctrine only to avoid invalidation or 1983 liability even if these public officials were the equivalent of Holmess bad manHart might find that constitutional law
still existed in a meaningful sense so long as the judges applying constitutional rules believed themselves to be bound by a constitutional rule of recognition.75 Here is a
possibility, one I believe we must take seriously and one that prompts anxiety about the existence of constitutional law itself: there is no common rule of recognition toward which
judges and other officials take an internal point of view.76 Individual judges may adhere to their particular understandings of the rule of recognition the Constitution as
interpreted by proper originalist methods, for example, or the Constitution as elucidated by popular understandings. But the fact that individual state actors follow their own rules
of recognition in good faith does not satisfy Harts account of law, and it does not provide a satisfying alternative to Thrasymachus. (There is no reason, on the might-makesright account, that the mighty cannot hold the good faith belief that they are pursuing a common good or acting pursuant to rule-governed authority. What matters is that their
power is in fact traceable to their superior strength.) There is reason for academic observers to doubt the existence of a single rule of recognition in American constitutional law.
There are too many core interpretive disputes, as discussed in Part I, and it is now widely accepted that constitutional rules are at least underdeterminatc. Should there be doubt
about this claim, consider this feature of constitutional law textbooks: they include majority and dissenting opinions, and questions after each case frequently ask the reader
which opinion was more persuasive. Those questions are not posed as rhetorical. For most constitutional decisions, we can say, it could have been otherwise. With a few votes
switched, with a different line-up of Justices, the same precedents (and in some cases, the same interpretive methodology) could have produced a different outcome. Moreover,
these suspicions of indeterminancy or underdetermi-nancy are not the unique province of the academy. Think of the discussions of Supreme Court appointments in presidential
elections. Many voters, law professors or not, understand their vote for president to be also a vote for a certain kind of Justice and for certain kinds of constitutional outcomes.
Discussions of Supreme Court appointments are often framed in terms of judicial methodology I will appoint judges who are faithful to the text of the Constitution but that
language may be more a matter of decorum than of real constitutional faith. Judges, of course, are not ignorant of the charges of indeterminancy or of the politicization of judicial
appointments. And it seems possible that the erosion of constitutional faith has reached the judiciary itself.771 claim no special insight into judicial psychology, but it seems
implausible that the reasons for constitutional skepticismthe discussions of underdetermined rules, the contingency of outcomes based on 5-4 votes, and the great attention to
swing justices such as Sandra Day OConnor or Anthony Kennedyhave not influenced judges themselves. Here again it seems worthwhile to consider dissenting opinions.
Justice Scalias polemics come to mind immediately; he has often accused his colleagues of acting lawlessly.78 Yet he keeps his post and continues to participate in a system
that treats as law the determinations of five (potentially lawless) Justices. It is possible, I suppose, that Justice Scalias dissents express earnest outrage, that he is shocked
(shocked) by decisions like Lawrence v. Texas79 and Boumediene. It is possible that he believes himself to be the last best hope of constitutional law properly so called. But it
seems more likely that he shares the skepticism of academic observers of the Court. Though one cant help but wonder whether judges are still constitutionally devout, I should
emphasize here that my argument does not turn on a claim that judges are acting in good or bad faith. Individual judges may well take the internal point of view, in Harts terms,
and strive faithfully to apply the principles they recognize as law.

But it seems clear that American judges do not all hold

the internal point of view toward a single, shared rule of recognition, given the nature of
disagreements among judges themselves. If there are multiple rules of recognition,
varying from judge to judge, then legal outcomes will depend on which judge is
empowered to make the critical decision, and Thrasymachus is not so far off the mark.
Contemporary judicial disagreement is profound, and it is not just a matter of Justice Scalias flair for colorful rhetoric. Consider Scott v. Harris, the recent decision granting
summary judgment (on the basis of qualified immunity) to a police officer who had rammed a passenger car during a high-speed chase, causing an accident that left the driver a
quadriplegic.80 Like most use-of-force opinions, the decision applies a deferential Fourth Amendment standard that gives police officers wide leeway. What is unusual about
Harris is that, because the case arose as a civil suit under 42 U.S.C. 1983, the critical question (whether the driver, Victor Harris, posed a sufficient threat to others bodily
safety such that the use of deadly force was reasonable) was nominally a jury question, and at summary judgment, the court should have taken the facts in the light most
favorable to the non-moving partythe injured driver. Thus, in earlier use-of-force cases that reached the Court as 1983 claims, the Court articulated the Fourth Amendment
standard and then remanded the case to the trial court.81 But in Harris, the Court had access to videotapes of the chase recorded by cameras on the dashboards of the police
vehicles involved.82 In the view of the eight-Justice majority, the videotape spoke for itself: it made Harriss threat to the public so clear that no reasonable juror could conclude
that the officers use of force was unreasonable.83 Accordingly, the Supreme Court found the officer to be entitled to summary judgment.84 Doubtless there are many instances
in which a court grants summary judgment to one party though non-judicial observers believe a reasonable juror could find for the other party. Harris is of particular interest,
though, because the reasonable juror who might have found in favor of Victor Harris was clearly visible to the majorityin fact, this juror had a spokesman on the Court.
Justice Stevens, the lone dissenter in Scott v. Harris, viewed the same videotape and found it to confirm the factual findings of the district court (which had denied the police
offic-ers motion for summary judgment).85 Though Justice Stevens was careful not to base his argument on an actual determination of the substantive Fourth Amendment
question (chiding his colleagues for doing just that and thereby acting as jurors rather than judges),86 he viewed the video evidence and explained how one might conclude,
perfectly reasonably, that Scott had used excessive force.87 In order for the eight Justices in the Harris majority to believe their own opinion, they would have to conclude that
Justice Stevens lived outside the realm of reason. Harris is nominally a dispute about what reasonable jurors could conclude, rather than a direct argument about the meaning of
a particular constitutional provision. But the two reactions to the videotape should call to mind Larry Tribes worry that American constitutional law is plagued by deep and thus
far intractable divisions between wholly different ways of assessing truth and experiencing reality.88 It is not just abortion and assisted suicide that reveal profound
disagreement about what is true and real. A videotape that speaks for itself in the eyes of eight Justices says something entirely different to the ninth. Looking beyond the
judiciary, consider the consequences of constitutional disagreement and constitutional indeterminancy for other government officials and for would-be critics of those officials.
Earlier I noted that with sufficient constitutional indeterminancy, theres no such thing as an unconstitutional president. A more extreme version of this argument is that with

Yoo wrote the Office of Legal Counsel memos that defend


practices formerly known as torture, he was simply doing to bans on torture what critics
had long argued it was possible to do for any law: he was trashing them.89 This was the
spawn of CLS put to work in the OLC; deconstructions on the left are now
sufficient legal indeterminancy, theres no such thing as illegality. When John

deconstructions on the right .90 And that, of course, is cause for anxiety among those who would like to argue that George W. Bush or members
of his administration acted illegally. As I suggested in the Introduction, this may be the Pyrrhic victory of critical legal studies: If the crits were correct, then there is no distinctively
legal form of critique. About torture, indefinite detention, warrantless wiretapping, and so on, we can say I don't like it or it doesnt correspond to my vision of the good, but we

To argue that the Bush administration violated the rule of law, we need to
believe that the rule of law exists. But for 30 years or more, we have found reasons to doubt that it does.91 Perhaps it will seem that I am
cannot say its illegal.

overstating the influence of legal realism and critical legal studies, or the doubts about laws existence. Im willing to entertain those possibilities, but I do want to emphasize that
the focus is on constitutional law. Its easy enough to believe in law when we see it applied and enforced by figures of authority in a recognized hierarchy. That is, the sentencing
judge or the prison warden can believe in lawhe has applied it himself. And the criminal should believe in law he has felt its force. But these examples illustrate Austinian
law: commands backed by force. What remains elusive, on my account, are laws that are truly laws given to oneself, and especially law given by a state to itself.92 That is why,

brute force is a poor candidate to distinguish ordinary politics, or


ordinary legal decisions, from extraordinary moments of crisis. What would be truly extraordinary is not the use of
in Part I of this essay, I suggested that

force, but its absence: a system of law truly based on consent and independent of sanction. The Constitution, in theory, is a law given unto oneself. By this I mean not simply
that the Founders gave the Constitution to future generations, but that

each successive generation must give the

Constitution to itself: each generation must adopt the internal point of view toward the
Constitution in order for it to be effective. Even once we have accepted the written text as authoritative, all but the strictest
constructionists acknowledge that many meanings can plausibly be extracted from that text. (And even the strict constructionists must acknowledge that as a factual matter
many meanings have been extracted; they deny only the plausibility of those varied readings.) Any law given unto oneself requires what Hart called the internal point of view,
and what one more cynical might describe as self-delusion: it requires a belief that one is bound though one could at any minute walk away. It is possible, I think, that we have

we have become too clever, too quick to notice indeterminancy, even too
post-modern to believe ourselves bound. A third possible explanation for contemporary references to crisis is professional malaise. It
outwitted the Constitution: that

could be, as I suggested earlier, that after too many years of chewing what judges had for breakfast, professors have lost their appetites. It could be that the problems of
originalists and historicists and popular constitutionalists dont amount to a hill of beans in this crazy world. And if these possibilities have not crossed the law professors mind,
they probably should. We might consider again Larry Tribes explanation of his decision to stop work on his treatise of American constitutional law. There are two questions of
meaning there, one of which Tribe confronts directly and the other which he brushes off quickly. Most obviously, there is the search for constitutional meaning, as Tribe
acknowledges, a search that cannot be concluded within the Constitutions own text. I see no escape from adopting some perspective... external to the constitution itself from
which to decide questions not indisputably resolved one way or the other by the text and structure--------9* Tribe goes on to wonder where these extra-constitutional criteria
come from, and who ratified the meta-constitution that such external criteria would comprise?.94 Supreme Court Justices (and other judges) must struggle with these
questions, given the public authority that they have the enormous responsibility and privilege to wield.95 But Tribe need not. He can simply decline to finish the treatise. If he
declines to finish the treatise, though, we cant help asking ourselves what was at stake, and what remains at stake. If the law professor lacks the responsibility of a judge, is his
constitutional theory just an amusing hobby? What was the point of the constitutional law treatise, or of other efforts to discern coherent principles of constitutional law? The
significance of a treatise is the question of meaning that Tribe brushes off quickly: he says a treatise is an attempt at a synthesis of some enduring value and insists that his
decision is not based on doubts about whether constitutional treatises arc ever worthwhile.96 But Tribes letter leaves the enduring value of a treatise rather underspecified,

Is constitutional
theory good for absolutely nothing? Only if we believe that the effort to resist
Thrasymachus is futile or pointless. Constitutional theory is a species of legal and political theory, and the most intriguing forms of such
and it is possible that current references to constitutional crisis in the academy stem from uncertainty about such questions of value.

theory are produced by worries that law and violence are too closely intertwined.97 Thus I suggested at the outset of this essay that existential anxiety is not always to be
regretted, cured, or mocked. Such

anxiety may be an important indication that we have noticed the ways

in which Thrasymachus seems right, and we still care enough to try to prove him wrong.98
After so much talk of crisis and anxiety, consider an illustration from the dramatic genre. Tom Stoppards play Jumpers features a
troupe of philosophy professors who double as acrobats: Logical positivists, mainly, with a linguistic analyst or two, a couple of Benthamite utilitarians ...
lapsed Kantians and empiricists generally... and of course the usual Behaviorists... a mixture of the more philosophical members of the university gymnastics team and the more

The Jumpers seem to practice what we would now identify as post-modern nihilism:
One shoots and kills another, then conceals the murder with cheerful aplomb. Against these
gymnastic members of the Philosophy School.99

intellectually and physically adroit colleagues, the clumsy and old-fashioned Professor George Moore struggles to defend the irreducible fact of goodness,100 the possibility of
a moral conscience, and the claim that there is more in me than meets the microscope.101 Is God? Moore wonders.

defend his faith.

He can neither shake nor

Law schools, I think, are filled with moral sympathizers to Professor Moore who possess the skills of modern-day Jumpers.102 The current

discourse of crisis is the latest manifestation of an old struggle between faith and doubt, and it is not one that we will resolve. On one hand, we have observed too much to

we are determined to have law, even if we must make it


ourselves. There was at least a smidgen of truth in John Finniss claim that scholars of critical legal studies were disappointed ... absolutists.103 But it is not
just crits that are disappointed when they look for law and see nothing. Few scholars of
any stripe want to vindicate Thrasymachus. All of this is just to reiterate the difficulty, and
perhaps the necessity, of giving a law unto oneself. If constitutional law did not exist, it
believe (in law) unquestioningly. And on the other hand,

would be necessary to invent it.

Negative State Action = N/L


Were NOT THE LAW OR STATE AFFIRMATION demands that a state not
exclude groups doesnt reaffirm the state
Newman 10
(Saul, Reader in Political Theory at Goldsmiths, U of London, Theory & Event Volume 13, Issue
2)
There are two aspects that I would like to address here. Firstly, the

notion of demand: making certain demands


on the state say for higher wages, equal rights for excluded groups, to not go to war, or an end to
draconian policing is one of the basic strategies of social movements and radical groups. Making
such demands does not necessarily mean working within the state or reaffirming its
legitimacy . On the contrary, demands are made from a position outside the political
order, and they often exceed the question of the implementation of this or that specific
measure. They implicitly call into question the legitimacy and even the sovereignty of
the state by highlighting fundamental inconsistencies between, for instance, a formal
constitutional order which guarantees certain rights and equalities, and state practices
which in reality violate and deny them.

Reform Possible

State Can be Reformed


The state can be reformed We need to change our thinking
Austin 2014
(Lisa M. Austin Associate Professor Faculty of Law University of Toronto ; Lawful Illegality: What
Snowden Has Taught us About the Legal Infrastructure of the Surveillance State; 2014 ;
http://poseidon01.ssrn.com/delivery.php?
ID=5060010130650220300940930800760011230250210680550700820250860910260701150
90016005031057122111022009036109118000093000001086073017084012076061126005006
0651150910950050290731220010950771070651040240670941020661030760940970910020
04004089105119091085&EXT=pdf&TYPE=2; B ; AWEY)
some of the post 9/11 debate regarding emergencies and the rule of law
concerns the different responses one might take to the existence of either legal black
holes or legal grey holes. A black hole is where the legislature seeks to carve out a space of no-law; a grey hole is one in which there is the facade or form of the rule of law
rather than any substantive protections.4 The space created by such holes is a space for executive discretion and the need
As Dyzenhaus has argued,

for such space derives from the perceived exceptional nature of national emergencies, where it is difficult to anticipate in advance what that emergency will be and how one should This framework of emergencies,

It is helpful in that the exceptional nature


of terrorism has deeply influenced contemporary methods of state surveillance. One aspect of the
with its themes of uncertainty and unforseeability, is both helpful and unhelpful when applied to state surveillance.

exceptional nature of terrorism is indeed its unpredictability. It is difficult to anticipate who will engage in acts of terrorism: agents of foreign powers, members of existing and known terrorist organizations, affiliates
abroad, or homegrown extremists? It is difficult to anticipate where an attack will take place, whether many civilians will be at risk, the potential scale of an attack, and so on. Another aspect of the exceptional
nature of terrorism is the type of risk it is seen to be not just a risk of potentially catastrophic harm but a deep political threat to the state. For example, the US considers itself to be at war against al Queda. 6

focusing on the
exceptional nature of emergencies can detract us from the most salient features of the
state surveillance methods Snowden has revealed to the world: they are in fact a rational,
systematic, planned response to the perceived need to prevent terrorist attacks. In other words,
the framework of emergencies concerns whether what is needed is a discretionary space
for executive authority either legal black holes or legal grey holes to nimbly respond to exceptional circumstances that cannot be foreseen in advance. But st ate
surveillance premised on the idea of collecting the haystack to find the needle is not
about preserving discretion at all. It is about applying rational analytic methods to the
problem of preventing certain kinds of threats that have been identified at least at some level of generality (e.g. terrorist threat).8 The
proper frame of the rule of law challenge is not about the question of whether executive discretionary authority in relation to emergencies can and
should be constrained by the reason of the law; instead, it is about whether mass surveillance as a mode of rational social
ordering is in conflict with the deepest commitments of law as a mode of rational social
ordering. When we talk about the legality of surveillance, therefore, we need to focus less on
the spaces of discretion and more on the systematic features of surveillance that put strain on our
traditional understandings of the rule of law. In particular, I want to flag three issues. The first is the issue of secrecy and the degree to
which it is demanded by the national security context. My claim is that it creates
pressure for unilateral, rather than objective and public, interpretations of the law. The
second is the issue of legal complexity, especially as it relates to law reform initiatives.
The extraordinary nature of the threat of terrorism also underpins the US response of seeking to prevent future terrorist attacks, with a never again mentality.7 However,

Where there is an increased blurring between regular law enforcement, border control, and terrorism investigations, as well as increasingly complex relationships between private sector communications

third is
the issue of jurisdiction and the extent to which national boundaries and questions of
status (like citizenship) affect the lawfulness of surveillance. In particular, I argue that instead of
providing us with the tools for accountability, status and jurisdiction allow for the
intermediaries and the state, gaining a clear public understanding of proposed changes to lawful access laws or the full significance of legal cases before the courts, is extremely difficult. The

leveraging of national boundaries to create an international surveillance regime with


questionable accountability.

Reforms Work
Pure resistance entrenches the status quo---must be willing to propose
imperfect reforms
Pyle 99
(Jefferey, Boston College Law School, J.D., magna cum laude Race, Equality and the Rule of
Law: Critical Race Theory's Attack on the Promises of Liberalism, 40 B.C.L. Rev. 787)
race-crits are strangely unrealistic in their proposals for reform. 1 m7 Most probably realize
crits generally
prefer not to suggest solutions, but to "resist" the dominant legal thought, doctrine and policy, whatever
For all their talk of "realism,"'"

that radical measures like racial or ethnic reparations are not likely to be granted, especially by a court. But even unrealistic proposals are rare, because race-

that happens to be.'" As Derrick Bell has put it, "most critical race theorists are committed to a program of scholarly resistance, and most hope scholarly resistance will lay the
groundwork for wide-scale resistance."'"

How this ivory tower oppositionalism would foment grassroots

revolt is unclear, because CRT professors rarely suggest anything practical. Rather, their
exhortations are meant, as Bell says, to "harass white folks" and thereby "make life bearable in a society where blacks are a permanent, subordinate class."'" One of the raceerns' few practical programs of "resistance" is Paul Butler's proposal that inner-city juries practice racially-based jury nullification.'91 jurors of color, Butler argues, have the
"moral responsibility" not to apply the criminal law to blacks and whites equally, but to "etnancipate some guilty black outlaws" because "the black community" would be "better
off" if there were fewer black men in prison.'" If enough juries were hung or not-guilty verdicts rendered, he imagines, the white-dominated government would change its
excessive reliance on incarceration.'" Butler rejects the ordinary democratic process of legal reform.' Democracy, he says, ensures a "permanent, homogenous majority" of
whites that "dominat[es]" African Ainericans.w5 Butler is probably correct that occasional acts of jury nullification might well express the resentment that many African Americans
justifiably feel towards discriminatory law enforcement.'"`' As Randall Kennedy has pointed out, however, black Americans are disproportionately the victims of crimes,'97 and

preference for "resistance"'99 over


democratic participation seems to flow from a fear of losing their status as "oppositional
scholars] "200 to the game of mainstream law and politics, which they regard as "an inevitably
co-optive process?"' Better to be radically opposed to the "doniinant political discourse""2 and remain an out than to work within the current system and lose
one's "authenticity?" In rejecting the realistic for the "authentic," however, race- crits begin to look
therefore tend to favor more, not less, criminal prosecution and punishment. 1 "8 The race-crits'

like academic poseursideological purists striking the correct radical stance, but doing
little within the confines of the real world, so sure are they that nothing much can be
done."

Change from Within k


Change must happen from within the state
Dussel| 2011
(Enrique Domingo Dussel Ambrosini (born December 24, 1934) is an Argentine-Mexican writer
and philosopher; A note regarding political organization: strategic action ; 2011 ; From Critical
Theory to the Philosophy of Liberation: Some Themes for Dialogue, page 33; AWEY)
In the same way, this lack of realism with respect to state institutions results in a lack of
critical realism in political strategy
. Lacking a strategic
institutional reference like the state
mediations become
insignificant.
. I am not speaking of a politics of organization, but rather a politics of events
(which should nevertheless always be transformed) strategic

In the end, for Negri, the global multitude faced with the specter of Empire (which for him lacks both an army and an exteriority) confronts the efforts of some organizations like the NGOs. This multitude will accumulate very

little power (potentia) while under the control of and managed by the mediocracy (as Giorgio Agamben, who knows well the political power of media magnates like Berlusconi, refers to it), which inevitably fabricates the interpretation of all political actions and
institutions. It is true that it is the interpretation of the event that creates the consciousness of the multitudes, as public opinion is manipulated through a sort of fabricated meaning that completely distorts those same events. In the end, what occurred in Seattle,
Genoa, or Cancun is what the media presented through the distortion of information. The media outlets don not argue; they present video and images, they grind them up and repeat them, and they create an imaginary, fetishized unanimity with complete control

It would seem that the reason and the political


will of the oppressed masses, of the particular postcolonial states, of the marginalized,
impoverished, of the original peoples of all the continents, of the excluded and the
wretched of the earth, also demands institutional mediation. Be they as they may, the
unforeseeable, the already known, or the transformable, such institutions are necessary
so that an empirical strategy might begin to clear the path.
over the meaning of the political. Is it possible in such a situation to passively await the maturation of a politics of events?

When the World Social Forum in Porto Alegre asserts that Another world is possible!

the practical postulate of all postulates it slowly begins to invent from below, without firm presuppositions, humble in the face of the experiences of that globally-networks and united base. This base comprises political organizations on all levels, from the economic,
household, neighborhood, sporting, artistic, cultural, and theoretical spheres, etc., new social movements whose participants know how to transform themselves into political actors in different institutions, not only in civil society but likewise in political society.

Political parties, which need to transform themselves as reality demands, would need to
play a new and more active role, not as a vanguard, but rather as a political school, as
the rearguard of the popular masses, as a critical institution in the exercise of power,
elaborating alternatives, as a site for the discussion of postulates, projects, models,
ends, strategies, tactics, means... so that the reproduction and development of life in
political community might be possible, so that its democratic, symmetrical, authorized
participation might be possible,

within a realism of that feasibility which is situated between anarchist impossibility (which is empirically impossible) and conservative impossibility (which is empirically

possible, but open to criticism). Hope, beyond conservative pessimism, but more proximate than the extreme optimism of the anarchist, becomes mobilized when it exercises a feasible power which transforms the existing structures from the perspective of a
postulate which needs only be filled with content: A world in which all worlds fit!, as the Zapatistas proclaim.

Law Key to Natives


Legal reform is vital to provide meaningful redress for past violence to
nativescant fix the law outside of the law
Bradford 2
(William Bradford 2, Chiricahua Apache. LL.M., 2001, Harvard Law School; Ph.D., 1995,
Northwestern University, "With a Very Great Blame on Our Hearts": 1 Reparations,
Reconciliation, and an American Indian Plea for Peace with Justice, 27 Am. Indian L. Rev. 1)
[*17] Nevertheless, even if the non-Indian majority would reject the American Myth in the interest of
mending national fences, the path to Indian redress winds through terrain unmapped
heretofore. Compensation and apologies, gestures potentially part of an amicable settlement, are not germane to the resolution of Indian claims for
injustices that cannot be remedied save by reinvestiture of lands and sovereignty in self-determining Indian tribes. 70 This requires not merely an abstract
acknowledgment of the value of pluralism but a comprehensive program of legal reform
that dispenses with doctrines and precedents perpetuating the denial of the human rights of
Indian tribes and people. 71 As law, more than any other social variable, has reproduced

the subordination of Indians in the United States, 72 legal reform occupies a central
position in the claim for Indian redress. 73 [*18] In short, proponents of Indian redress must not
only displace a flawed version of history : they must articulate a proposal for
remediation that transports the American people far beyond the strictures of existing law
to enable the peaceful restoration of Indian lands and powers of self-government. 74 Such a
transformative mission cannot be accomplished by positing Indians and the non-Indian majority as adversaries, as would reparations; rather, redress of Indian claims and the healing of the American nation --

this Article offers a


disquieting version of U.S-Indian history that accelerates erosion of the American Myth
crucial foci of the drive toward perfection -- necessitate dialogue, reconciliation, and joint authorship of a future history of peace, harmony, and justice. 75 Part II of

and acquaints the non-Indian majority with the necessary factual predicate to Indian
redress.

Parts III and IV contrast the assumptions, procedures, and remedies that distinguish reparations and reconciliation, 76 the dominant contending models of redress available to group victims

of human injustice, and demonstrate that,

because it offers the best hope for a peaceful American coexistence

marked by mutual respect for sovereignty , reconciliation is a more appropriate avenue


to Indian redress. Several preliminary proposals, including the introduction of traditional tribal peacemaking as perhaps the most appropriate form of [*19] reconciliation, will be offered to
stimulate thinking.

Defense

AT: Quick Fix Solutions


Chocolate laxatives K is silly
Olson 9
(Dan T. Olson 9 - MA in theology from Fuller Theological Seminary and am in the process of
completing another MA in philosophy at LMU, 8/1/9, Hauerwas, Zizek, Patagonia And Rob Bell:
Capitalisms Chocolate Laxatives And The Church
http://dtomolson.wordpress.com/2009/08/01/hauerwas-zizek-patagonia-and-rob-bellcapitalisms-chocolate-laxatives-and-the-church/)
Zizek suggests that the reason these liberal-communists and their chocolate laxatives must be
rejected is that they are the quick fixes that allow the system to continue to grind on, grinding up the
To get back to the point,

masses as it does. The chocolate laxative may momentarily loosen your stool, but it will not fix the main cause of the problem. In order to do that one must stop living off of
chocolate and eat some roughage, which doesnt taste all that good to a palate used to sugar, cocoa and butter.

One possible danger in Zizeks

answer , for me, is that it is in danger of making the same mistake that every other purely
human revolution, insurrection, or rescue mission has made . That is, in view of what must be
achieved some sacrifices must be madesome heads of the aristocracy, some infidels, a couple of thousand Iraqi or Afghani civilians
lives or the massive suffering and starvation of many of the wretched of the earth who
depend on the charity of the liberal communists. For the Christian this is always unacceptable. People are not goods to be used
no matter how lofty the purpose but persons to be loved. (Of course how this love works itself out is what is in question here.) Many theologians have told us, rightly, that the
poor are the sight of the in breaking and redeeming work of Christ. This is different than being the subjective location of, or potential for, the revolution.

One must be

careful that in order to achieve ones end, the end of capitalist hegemony, an end that I believe is
completely in line with the Gospel, one does not sacrifice those who cannot choose otherwise . This does
not mean that on the other hand we shut our eyes and continue to rejoice in liberal
communists and chocolate laxatives . We cannot be content to wear our Toms Shoes and Gap Red T-shirts while
drinking our fair trade coffee and going about our day pleased with the current social order . There has to

be a third or middle way, a political option better than Elizabeths ecclesial via media.

Law Inevitable
Law is inevitable repressing the state increases its abusive activity
McCormick 99
(John P., Assistant Political Science Pf - Yale, American Political Science Review. v93, p. June
1999)//akim

Conventionalism is often considered to be a way of avoiding the naked prejudices of


individual judges constituting the law. As a solution, whatever a community of judges in a particular culture could be
expected to agree upon in a similar case ought to be adopted as a guideline for a judge's particular opinion in a given case. Indeed, in his early legal
work, Schmitt ([1912] 1969, 71-9, 86) offers this as a solution to the problem of judicial subjectivism.8 Here Schmitt trusts in the [foot notes] [header]
hegemony of the conservative interpretive methods of the late Kaiserreich to promote judicial consensus in much the manner that Fiss (1982,
1985), for instance, trusts in something approaching a postwar American liberal consensus in his own formulation of conventionalism. These
examples, of course, potentially exacerbate rather than alleviate the problem of judicial arbitrariness: The

cultural prejudice of a
particular profession as a whole is no better solution to the problem of indeterminacy
from a normative perspective than, on the one hand, a subjectively all-powerful lawgiver judge,
or, on the other, a mechanically objective automaton judge. Collective prejudice is ultimately
just as problematic as individual prejudice. It may temporarily reduce the threat of
indeterminacy by guaranteeing more predictable results, but it does so in a way that is still, in the last instance,
arbitrary. Schmitt abandons conventionalism in his mature writings of the 1920s and early 1930s; CLS has
always rejected it, perhaps appropriately, as elitist. According to Schmitt, in their revulsion to arbitrary discretion, liberals sought to
eliminate the state institutionally from jurisprudential concerns, just as they wished to eliminate, hermeneutically, the personal, subjective decision
from such matters. But

they were equally unsuccessful in each endeavor. Specifically, the adherence


to legal formalism under conditions of an emerging welfare state allows more state
intervention than was ever dreamed of by absolute monarchs. To Schmitt's mind, in the early twentieth century, the formalism of
liberalism serves as an ideology that belies the so-called materialization of law that is
brought about by state activity in the new era of welfare-state or Sozialstaat interventionism. Schmitt observes presciently that
as state activity addresses complex and variegated social and economic situations, law is formulated in a more open-ended and less finely
discernible manner. In

the service of widescale state intervention into particular spheres of society,


more discretion becomes exercised by bureaucratic administrators, including judicial officers, in implementing
broadly defined social policy. Rather than a formal guideline for members of society, law
becomes a material part of social reality .9 By repressing the state, the legal formalists
not only do

not prevent arbitrary state functioning but also allow its activity to proliferate

more extensively and undetected to an even greater degree . Unlike CLS at a later date, however, Schmitt's
concern with this situation is not that state officials and the powerful interests they represent potentially abuse their power through such functioning;
rather, the state is actually, in the end, exhausted by it. The welfare state is a weakened state that overextends itself, not a dangerously strong
coercive state.

State Inevitable
State is inevitable only way to change is to work within it.
Dussel| 2011
(Enrique Domingo Dussel Ambrosini (born December 24, 1934) is an Argentine-Mexican writer
and philosopher; The sphere of feasibility: Empire and the dissolution of the particular state ;
2011 ; From Critical Theory to the Philosophy of Liberation: Some Themes for Dialogue, page
29; AWEY)
All institutions, responding in their founding moment to demands for the permanence
and extension of life, include a certain degree of discipline. Anti-institutional critics are quite right to show that this discipline
becomes quickly (or even from its origin) repressive. This was Marcuse's subject in Eros and Civilization. The state is a political macro-institution. More
recently, M. Foucault opposed the political binary of oppressor-oppressed as wielded by the only instance of the exercise of power (the state) and equally affirmed by standard Marxism.75 He tried to show that
power was disseminated, through multipolar rather than bipolar structures, in micro-institutions that discipline the body on different epistemic levels and justify the exercise of power. The panoptical power of
prisons, psychiatric clinics, schools etc., fragment power, and as such undermine the over-simplified view of power as based macro-institutionally in the state. The old tradition of which this sort of critique is a part
begun by Stirner and Bakunin, continued by Sorel and Pelloutier, and which runs deeply at present in Foucault himself, along with Negri and Holloway expresses a need for the dissolution of the state.

This, however, brings in the entire institutional question within the philogenetic development of
the human species that have progressively become fixed in systemic structures and
institutions. Finally, it is on these latter that the entire problematic of diagnosing the nature of politics and economics is based, which provides the foundation for a properly contingent level of
strategy. Political action that seeks to change or transform the world inevitably confronts
institutions. In a situation of chaos or pure original dissidence (disidencia originaria),
there can be no transformation or dissent. To chaos one can only con-form, institutionalizing it toward the permanence of life by way of this
institutionalizing (instituyente) power.76 Original dissidence, on the other hand, is death and non-power, because when there is no consensus or agreement the powers-to-posit" 77 of each member oppose and

The starting point should be some sort of


consensus. The form of the institution or consensus is open to change, to be transformed through a moment of overcoming chaos with creative dissidence, into a higher
form. To trans-form or change is not simply to destroy: it is to de-construct in order to innovate and move toward a better construction. Revolution is not only, or primarily, or principally destruction: it means
having a principle that orients the deconstruction just as much as it orients the new construction (it is not the business of destroying everything, only that which is irretrievable). Those who
lack criteria and principles for a new construction (note that I am not saying a re-construction), are not
revolutionaries but simply destructive and barbaric. It would not be possible for millions
of human beings to maintain and expand communal life without institutions. Should we irrationally return to
cancel out one another (and it is not possible to create any mediation to sustain life).

the Paleolithic era? No. We are dealing with the trans-formation (what Marx called Veraenderung) of those institutions which began as lifeenhancing mediations, but which have since become instruments of
death, impediments to life, instruments of an exclusion which can be observed empirically in the cry arising from the pain of the oppressed, the ones suffering under unjust institutions. Such entropically-repressive
institutions exercise a power-over78 their victims, whose power-to-posit79 their own mediations is negated, and who are thereby repressed.

Alt Worse

Individual Ethics Fail


Individual ethical orientations arent effective its more productive to
rearticulate systems from the inside
Pugh 10
(Jonathan, Newcastle Postcolonial Geographer, The Stakes of Radical Politics have Changed:
Post-crisis, Relevance and the State, Globalizations, March-June, ebsco)
some from the radical Left were
incapable of being able to respond to the new stakes of radical politics. In particular, they
were not found at the state, where the passive public turned to resolve the crisis. I will
now go on to examine how in recent years significant parts of the radical Left have also
tended to prioritise raising awareness of our ethical responsibilities, over capturing state
power. I am going to say that it is important to create this awareness. However, in an effort to draw attention to the stakes of politics as we find them now, post-2008, I will also point out that we should not
In this polemical piece I have just been talking about how, following an ethos of radicalism as withdrawal from the state,

place too much faith in this approach alone. Against the backdrop of what I have just been saying, it is important to remember that while much attention is focused upon President Obama, in many other parts of
the world the Right and fundamentalism are gaining strength through capturing state power. The perception that the USA has changed is accompanied by a sense of relief among many radicals.

However, the European Elections of 2009, the largest trans-national vote in history,
heralded a continent-wide shift to the Right (and far Right) in many placesin Austria,
Belgium, Bulgaria, Cyprus, the Czech Republic, Denmark, Estonia, Finland, France,
Germany, Italy, Estonia, Lithuania, Luxembourg, Poland, Portgual, Slovenia, Spain,
Romania, as just some examples (Wall Street Journal, 2009). Despite Obamas election and a near depression, neo-liberalism continues to be implemented through a
world spanning apparatus of governmental and intergovernmental organisations, think tanks and trans-national corporations (Massey, 2009; Castree, 2009). The power of the Right in countries like Iran, while

Albertazzi et al. (2009) draw attention to how a disconnected Left is


leaving power in the hands of the Right in many other countries nationally, like Italy for
example. Reflecting upon contemporary radical politics, the British Labour politician Clare Short (2009, p. 67) concludes: In the fog of the future, I see a rise of fascistic movements . . . I am afraid it will
checked, remains unchallenged by the Left.

all get nastier before we see a rise in generous, radical politics, but I suspect that history is about to speed up in front of our eyes and all who oppose the radicalisation of fear, ethnic hatred, racialism and division
have to be ready to create a new movement that contains the solutions to the monumental historical problems we currently face. So, the stakes of politics are clear. The Right is on the rise. Neo-liberal ideology is
still dominant. How is the Left responding to these stakes? I have already discussed how some from the radical Left are placing too much faith in civil society organisations that seek to withdraw from the state. I

Post-crisis, the increasing popularity of


David Chandlers (2004, 2007, 2009a, 2009b) work reflects the sense that radicals too
often celebrate the ethical individual as a radical force, at the expense of wider
representational programmes for change. His central argument is that this leaves
will now turn to how others have too much faith in the power of raising awareness of our ethical responsibilities.

radicals impotent . Chandler (2009a, p. 7879) says that many radicals argue that there is nothing passive or conservative about radical political activist protests, such as the 2003 antiwar march, anti-capitalism and anti-globalisation protests, the huge march to Make Poverty History at the end of 2005, involvement in the World Social Forums or the radical jihad of Al-Qaeda. I disagree;

these new forms of protest are highly individualised and personal ones there is no
attempt to build a social or collective movement . It appears that theatrical suicide,
demonstrating, badge and bracelet wearing are ethical acts in themselves: personal
statements of awareness, rather than attempts to engage politically with society. In one way,
Chandlers reflective insight here is not particularly unique. Many others also seem to think that radicals today are too isolated and disengaged (Martin, 2009).5 Neither is it particularly original to say that there is
too much emphasis upon creativity and spontaneity (what Richard Sennett, 2004, calls social jazz), and not enough upon representational politics. Indeed, go to many radical blogs and you find radicals
themselves constantly complaining about how it has become too easy to sign up to ethical web petitions, email complaints, join a variety of ethical causes, without actually developing the political programmes
themselves that matter. So it is not Chandlers point about radicals being disengaged from instrumental politics that concerns me here. It is his related pointthat there has been a flight into ethics, away from
political accountability and responsibility that I find intriguing. Personal statements of ethical awareness have become particularly important within radical politics today. It is therefore interesting to note, as I will
now discuss, that we have been here before. In his earlier writings Karl Marx (1982) criticised the German Idealists for retreating into ethics, instead of seizing the institutions of power that mattered for

Unwilling to express their self-interests politically through capturing power, the


Idealists would rather make statements about their ethical awareness. Such idealism,
along with an unwillingness to be held accountable for political power, often goes hand
in hand. For Marx, it is necessary to feel the weight, but also the responsibility of power. Chandler argues that, just as when the early Marx critiqued German Idealism, we should now be drawing
themselves.

attention to the pitfalls of the flights to ethics today. He says: In the case of the German bourgeoisie, Marx concludes that it is their weakness and fragmentation, squeezed between the remnants of the ancien re
gime and the developing industrial proletariat, which explains their ideological flight into values. Rather than take on political responsibility for overthrowing the old order, the German bourgeoisie denied their
specific interests and idealised progress in the otherworldly terms of abstract philosophy, recoiling from the consequences of their liberal aspirations in practice. (Chandler, 2007, p. 717) Today we are witnessing a

Fragmented, many radicals retreat into abstract ethical slogans


like another world is possible, global human rights, or making poverty history. As discussed
renewed interest in ethics (Ladi, 1998; Badiou, 2002).

above, we are also of course seeing the return of Kants cosmopolitanism. While I think we should not attack the ethical turn for its values, as many of these around environmental issues and human rights are

, it is equally important to say that the turn to ethics seems to reflect a certain lack of
willingness to seize power and be held accountable to it. For the flight to ethics, as it often plays out in radical politics today, seems to
be accompanied by scepticism toward representational politics. Continuing with this theme for a moment, Slavoj Zizek (2008) also sheds some more light upon why ethics (when compared to
representational politics) has become so important to the Left in recent years. He says that many of us (he is of course writing for the Left) feel that we are
unable to make a real difference through representational politics on a larger scale, when it comes to the big political problems of life. Zizek (2008, p. 453) talks of this
admirable

feeling that we cannot ever predict the consequences of our acts; that nothing we do will guarantee that the overall outcome of our interactions will be satisfactory. And he is right to make this point. Today, our
geographical imaginations are dominated by a broader sense of chaos and Global Complexity (Urry, 2003; Stengers, 2005). These ways of thinking, deep in the psyche of many radicals on the Left may be one
other reason why so many have retreated into ethics. When we do not really believe that we can change the world through developing fine detailed instruments, capturing the state, or predictive models, we are
naturally more hesitant. It is better to try and raise ethical awareness instead. Whereas in the past power was something to be won and treasured, something radicals could use to implement a collective ideology,
today, with the risk posed by representation in fragmented societies, top-down power often becomes a hazard, even an embarrassment, for many on the Left (Ladi, 1998). This is, as I have already discussed,

Putting what I have just said another way, there is a


need to be clear, perhaps more so in these interdisciplinary timesethics and politics
(particularly representational politics) are different. Of course they are related. You cannot do politics without an ethical perspective. But
where the Right and neo-liberal ideologues are seizing the opportunity of the moment.

my point here is that the Right and neo-liberal ideologues will not simply go away if the
Left adopt or raise awareness of alternative ethical lifestyles. The Right are willing to
capture state power, particularly at this time when the state is increasingly powerful. When we compare the
concerted political programme of neo-liberalism, first developed by Reagan, Thatcher, the IMF, the World Bank, NATO, multi-national banks, and the G20, as just some of many examples, ethical individuals

But the 2008 crisis, and the response of protests like the Alternative
G20, demonstrated how weak ethical resistance is in the face of the institutions of the
neo-liberal economy. Another reason for this is because the ethical individual contributes
so much to neo-liberal societies themselves. To explain how, we must briefly step back. The new social movements of previous decades have, in
across the world offer some counter-resistance.

general, been effectively recuperated by the existing system of capital, by satisfying them in a way that neutralised their subversive potential. This is how capital has maintained its hegemonic position in post-

They say the new


social movements desire for autonomy, the ideal of self-management, the antihierarchical exigency, and the search for authenticity, were important in developing postFordism. These replaced the hierarchical framework of the Fordist period with new forms of networked control. And so, in this way, we see that the relationship between new social movements and
Fordist societies. Luc Boltanski and Eve Chiapello (2005) explain how capitalists have worked with, rather than against, the characteristics of new social movements.

capital has been productive. In turn, and this is the important point I want to make about the present moment, clearly the stakes of radical politics have now changed once more. As discussed earlier, it would now

Without the neoliberal state, and the


publics subordination to its actions, it would not now exist in anything like its present
form. Our subordination to the state has stopped a post-crisis implosion of neoliberalism. And this is of course where one of the central characteristics of the ethical
individual has been so productive. Endemic individualism, so dominant in liberal
societies, has been recuperated by the ethical individual who is unwilling to seize the
state. So the salient point here is that the ethical individual is reflective of the
seem that post-Fordist society is actually more hierarchical and controllable than many previously thought.

conservative forces in society today.

Micro-Resistance Fails State Key


The alt doesnt adequately create counterhegemonies-micro resistance
empirically relies on the myth they will spill up but never do-the AFFs
vision of institutional change is more effective.
Lobel 7
(Orly, UCSD law professor, The Paradox of Extralegal Activism: Critical Legal Consciousness
and Transformative Politics, http://www.harvardlawreview.org/media/pdf/lobel.pdf)
the practical failures and the fallacy of rigid boundaries generated by extralegal activism
rhetoric permit us to broaden our inquiry to the underlying assumptions of current proposals regarding
transformative politics that is, attempts to produce meaningful changes in the political and socioeconomic landscapes. The suggested
alternatives produce a new image of social and political action. This vision rejects a shared theory of social reform, rejects formal programmatic agendas, and
embraces a multiplicity of forms and practices. Thus, it is described in such terms as a plan of no plan,211 a project of
Both

projects,212 anti-theory theory,213 politics rather than goals,214 presence rather than power,215 practice over theory,216 and chaos and openness over order and formality. As a result, the contemporary
message rarely includes a comprehensive vision of common social claims, but rather engages in the description of fragmented efforts. As Professor Joel Handler argues, the commonality of struggle and social
vision that existed during the civil rights movement has disappeared.217 There is no unifying discourse or set of values, but rather an aversion to any metanarrative and a resignation from theory. Professor

[T]he opposition
is not playing that game . . . . [E]veryone else is operating as if there were Grand Narratives . . . .218
Intertwined with the resignation from law and policy, the new bromide of neither left nor
right has become axiomatic only for some.219 The contemporary critical legal consciousness informs the scholarship of those who are interested in
Handler warns that this move away from grand narratives is self-defeating precisely because only certain parts of the political spectrum have accepted this new stance:

progressive social activism, but less so that of those who are interested, for example, in a more competitive securities market. Indeed, an interesting recent development has been the rise of conservative public

conservative advocacy groups


have rapidly grown both in number and in their vigorous use of traditional legal strategies to promote
their causes.221 This growth in conservative advocacy is particularly salient in juxtaposition to the
decline of traditional progressive advocacy. Most recently, some thinkers have even suggested that there may be something inherent in the
lefts conception of social change focused as it is on participation and empowerment that
produces a unique distrust of legal expertise.222 Once again, this conclusion reveals flaws parallel to
the original disenchantment with legal reform. Although the new extralegal frames present themselves as apt alternatives to legal reform models and as capable
of producing significant changes to the social map, in practice they generate very limited improvement in existing social arrangements. Most
strikingly, the cooptation effect here can be explained in terms of the most profound risk of the typology that of legitimation. T he common pattern of extralegal
scholarship is to describe an inherent instability in dominant structures by pointing, for example, to grassroots
strategies,223 and then to assume that specific instances of counterhegemonic activities translate into a
more complete transformation. This celebration of multiple micro-resistances seems to rely on
an aggregate approach an idea that the multiplication of practices will evolve into something
substantial. In fact, the myth of engagement obscures the actual lack of change being
produced, while the broader pattern of equating extralegal activism with social reform
produces a false belief in the potential of change. There are few instances of meaningful reordering of social and economic arrangements and
interest lawyer[ing].220 Although public interest law was originally associated exclusively with liberal projects, in the past three decades

macro-redistribution. Scholars write about decoding what is really happening, as though the scholarly narrative has the power to unpack more than the actual conventional experience will admit.224 Unrelated
efforts become related and part of a whole through mere reframing. At the same time, the elephant in the room the rising level of economic inequality is left unaddressed and comes to be understood as
natural and inevitable.225 This is precisely the problematic process that critical theorists decry as losers self-mystification, through which marginalized groups come to see systemic losses as the product of their

micro-instances of activism are


often fundamentally performative, obscuring the distance between the descriptive and the prescriptive. The manifestations of extralegal activism
the law and organizing model; the proliferation of informal, soft norms and norm-generating actors; and the celebrated, separate nongovernmental sphere of action all produce a
fantasy that change can be brought about through small-scale, decentralized
transformation. The emphasis is local, but the locality is described as a microcosm of the
whole and the audience is national and global. In the context of the humanities, Professor Carol Greenhouse poses a comparable challenge to ethnographic studies from the 1990s, which utilized the
own actions and thereby begin to focus on minor achievements as representing the boundaries of their willed reality. The explorations of

genres of narrative and community studies, the latter including works on American cities and neighborhoods in trouble.226 The aspiration of these genres was that each individual story could translate into a time

of the nation body of knowledge and motivation.227 In contemporary legal thought, a corresponding gap opens between the local scale and the larger, translocal one. In reality, although there has been a recent
proliferation of associations and grassroots groups, few new local-statenational federations have emerged in the United States since the 1960s and 1970s, and many of the existing voluntary federations that
flourished in the mid-twentieth century are in decline.228 There is, therefore, an absence of links between the local and the national, an absent intermediate public sphere, which has been termed the missing

social movements have for the most part failed in sustaining coalitions or
producing significant institutional change through grassroots activism. Professor Handler concludes that this failure
is due in part to the ideas of contingency, pluralism, and localism that are so embedded
in current activism.230 Is the focus on small-scale dynamics simply an evasion of the need to engage in broader substantive debate? It is important for
next-generation progressive legal scholars, while maintaining a critical legal
consciousness, to recognize that not all extralegal associational life is transformative. We
must differentiate, for example, between inward-looking groups, which tend to be self-regarding
and depoliticized, and social movements that participate in political activities, engage the
public debate, and aim to challenge and reform existing realities.231 We must differentiate between professional
associations and more inclusive forms of institutions that act as trustees for larger segments of the community.232 As described above, extralegal activism tends to
operate on a more divided and hence a smaller scale than earlier social movements, which had national
reform agendas. Consequently, within critical discourse there is a need to recognize the limited
capacity of small-scale action. We should question the narrative that imagines consciousness-raising as directly translating into action and action as directly translating into
middle by Professor Theda Skocpol.229 New

change. Certainly not every cultural description is political. Indeed, it is questionable whether forms of activism that are opposed to programmatic reconstruction of a social agenda should even be understood as

groups are situated in opposition to any form of institutionalized power,


they may be simply mirroring what they are fighting against and merely producing moot
activism that settles for what seems possible within the narrow space that is left in a
rising convergence of ideologies. The original vision is consequently coopted, and
contemporary discontent is legitimated through a process of self-mystification.
social movements. In fact, when

Alt Fails/State Good/Reformable


Either the alternative is ontologically incompatible and the alternative does
nothing or it necessitates dissolution of the state which fails
Gay 12
(Paul, CBS department of organization professor, Leviathan calling Some notes on sociological
anti-statism and its consequences, Journal of Sociology, 48.4, SAGE)
Because the state is imperfect indeed, in being born it had to renounce perfection, as Weber
famously argued in The Profession and Vocation of Politics (1994), both its own and that of those it sought to rule, making
do instead with its capacity to enforce peace and their capacity to act peaceably it is
not necessarily wise to insist on benchmarking the state against moral ideals which are
alien to its constitution . In seeking to make the state conform to certain politically and
morally expressivist ideals whether liberal or communitarian, cosmopolitan or transnationalist, or any other such ideals critics of the
state can come dangerously close to endorsing its retheologization, or its dissolution.
Michael Ignatieff (2001: 35) makes precisely this point when he argues that individual rights cannot and should not function as transcendental limits on state action because they
are themselves the product of action by sovereign states. They are historical entitlements to legal action, contingent upon the states capacity to establish and maintain security.
Under conditions of peace the sovereign state, in its guise as security state, can almost seem to disappear, and the state becomes the addressee of a wide range of additional
demands and expectations. As soon as the security envelope is threatened, however, whether internally (incitement to insurrection, domestic terrorism, economic crisis) or
externally (foreign terrorism, invasion, economic crisis) then civil liberties and rights are retracted to the extent that is necessary to protect the space within which they were
unfolded in the first place (Hunter, 1998, 2005). It should therefore come as no surprise in fact it should be taken as evidence of its stateness that contemporary liberaldemocratic states default to their foundational security setting when under threat. It would be a worry if they did not, for this would be evidence of their transcendence or

What still seems so difficult for those seeking to moralize the state, and what
conjoins them with earlier critics, is their distrust of the independence or autonomy of
the state as an impersonal structure of rule. In particular, these modern critics seem unable, or unwilling, to accept that the state
hollowing out.

cannot possibly undertake its core functions of pacification and security unless it can decide for itself, without internal impediment, what can be publicly expressed or just who

It is the state, and the state alone, that can and must: judge the degree of
jeopardy in every instance. The state carries, and must carry, the authority of its own subjects will and choice to make that judgement on their behalf
and to act, decisively, upon it. Indeed, each subject has a right against every other that it should do just this. (Dunn, 2000: 84) It is in this sense, as I
indicated earlier, the authority of the state is both binding and content-independent:
Part of the whole point of having such a free-standing coercive structure (the state) is
that it be independent. As long as the basic fact remains, there is always going to be a
can own what, and why.

gap between the political power of the state and the effective powers of the populace,
and, on this argument, that is a good thing in a world populated by other states, many
of them predatory, it is essential for the minimal self-defence of a certain population that it be organized as a state, or
one might think that it was necessary [as recent events indicate all too clearly] to have an independent power that could intervene in the economy to prevent it from

The very rationale of the state as an independent coercive


apparatus, one beyond the direct control of its subjects, means that it will always be
antithetical to the moralizing ideal of popular sovereignty. From a statist point of view,
this is a major factor in its favour. Attempts by critics of various sorts to neuter the
selfdestruction. (Geuss, 2001: 129, emphasis in original)

authoritarian or absolutist pedigree of the state through, for instance, abrogating its
room for manoeuvre by opposing its discretionary powers, are, from this statist
viewpoint, misguided, not least because they can in certain circumstances undermine
the very conditions that made the state and the liberal rights associated with it possible
in the first place (Geuss, 2001; Hunter, 2005). The critique and expose line of reasoning, which seeks

to somehow evacuate, water down or occlude the continuing and crucial practical
importance of sovereignty and an associated absolutist capacity, also extends to
contemporary sociological discussions of the state. Here, the effects of the ongoing moment of theory, in particular some of its
constructionist and constructivist variants, are evident (for a detailed discussion see Hunter, 2006, 2007, 2009). Much important work in the latter vein has sought to puncture
reifications of the state as a free-standing entity, indicating instead how state effects are produced in, and as a result of, the relations established between a diverse range of
materials, mundane practices and devices. However, while ostensibly signaling an adherence to empirical history and positive description, this body of work has tended to be
highly theoretical and epochal in orientation. A number of consequences have flowed from this, not least of which has been the effective disappearance of the state as an object
of sociological analysis. Two pieces inspired by the work of Michel Foucault Rose and Millers (1992) Political Power beyond the State (with its focus on governmentality)
and Timothy Mitchells (1999) Society, Economy, and the State Effect (with its focus on disciplinary society) serve as representative examples of this tendency. They do so
precisely because they are more epochal theoretical positions than they are historical descriptions. In following a certain Foucauldian line, one which programmatically
distinguishes between a before and an after, instituting an epochal break between singular power exercised via absolutist sovereignty and plural powers exercised via
normalizing disciplines and techniques of conduct, these and other such analyses are simply trading on an ahistorical theoretical distinction which supposedly contrasts perfectly

they have little to say about the historical emergence of the state
and its instituted purposes, and thus equally little to say about how and why the manner
of the states historical existence makes it unamenable to being reduced to theoretical
abstractions such as the juridico-discursive or governmentality. Their epochal
theoretical orientation and concomitant lack of historical contextualization effectively
render them incapable of comprehending how the absolutist security state was and
remains the default setting for the contemporary liberal-democratic state (Hunter, 2005).
antithetical ways of exercising power. As such,

Institutions KT Solve Anti-Blackness


the law is obviously problematic, but thats a reason we should hold it
accountable to live up to its ideals
Crenshaw 88
(Kimberle, Law @ UCLA, RACE, REFORM, AND RETRENCHMENT: TRANSFORMATION
AND LEGITIMATION IN ANTIDISCRIMINATION LAW, 101 Harv. L. Rev. 1331, lexis)
Questioning the Transformative View: Some Doubts About Trashing The Critics' product is of limited utility to Blacks in its present form. The implications for Blacks of trashing liberal legal ideology are troubling,
even though it may be proper to assail belief structures that obscure liberating possibilities. Trashing legal ideology seems to tell us repeatedly what has already been established -- that legal discourse is unstable
and relatively indeterminate. Furthermore

, trashing offers no idea of how to avoid the negative consequences of

engaging in reformist discourse or how to work around such consequences. Even if we


imagine the wrong world when we think in terms of legal discourse, we must
nevertheless exist in a present world where legal protection has at times been a blessing -albeit a mixed one. The fundamental problem is that, although Critics criticize law because it functions to
legitimate existing institutional arrangements, it is precisely this legitimating function
that has made law receptive to certain demands in this area. The Critical emphasis on deconstruction as
the vehicle for liberation leads to the conclusion that engaging in legal discourse should
be avoided because it reinforces not only the discourse itself but also the society and the world
that it embodies. Yet Critics offer little beyond this observation. Their focus on delegitimating rights rhetoric seems to
suggest that, once rights rhetoric has been discarded, there exists a more productive strategy for change, one
which does not reinforce existing patterns of domination. Unfortunately, no such strategy has yet
been articulated , and it is difficult to imagine that racial minorities will ever be able to
discover one. As Frances Fox Piven and Richard Cloward point out in their [*1367] excellent account of the civil rights movement, popular struggles are a
reflection of institutionally determined logic and a challenge to that logic. 137 People can
only demand change in ways that reflect the logic of the institutions that they are
challenging . 138 Demands for change that do not reflect the institutional logic -- that is, demands that
do not engage and subsequently reinforce the dominant ideology -- will probably be
ineffective . 139 The possibility for ideological change is created through the very process of legitimation, which is triggered by crisis. Powerless people can
sometimes trigger such a crisis by challenging an institution internally, that is, by using
its own logic against it. 140 Such crisis occurs when powerless people force open and
politicize a contradiction between the dominant ideology and their reality. The political consequences [*1368] of
maintaining the contradictions may sometimes force an adjustment -- an attempt to close the gap or to make things appear fair. 141 Yet, because the adjustment is triggered by the political consequences of the

This approach to understanding legitimation and


change is applicable to the civil rights movement. Because Blacks were challenging their exclusion from political society, the only claims that were likely to achieve
contradiction, circumstances will be adjusted only to the extent necessary to close the apparent contradiction.

recognition were those that reflected American society's institutional logic: legal rights ideology. Articulating their formal demands through legal rights ideology, civil rights protestors exposed a series of

Rather than using the


contradictions to suggest that American citizenship was itself illegitimate or false, civil
rights protestors proceeded as if American citizenship were real, and demanded to
exercise the rights that citizenship entailed. By seeking to restructure reality to reflect
American mythology, Blacks relied upon and ultimately benefited from politically inspired efforts to
resolve the contradictions by granting formal rights. Although it is the need to maintain legitimacy that presents powerless groups with the opportunity
contradictions -- the most important being the promised privileges of American citizenship and the practice of absolute racial subordination.

to wrest concessions from the dominant order, it is the very accomplishment of legitimacy that forecloses greater possibilities. In sum, the potential for change is both created and limited by legitimation. 139 The
possibility for ideological change is created through the very process of legitimation, which is triggered by crisis.

Powerless people can sometimes trigger

such a crisis by challenging an institution internally, that is, by using its own logic
against it. 140 Such crisis occurs when powerless people force open and politicize a
contradiction between the dominant ideology and their reality. The political consequences [*1368] of maintaining the
contradictions may sometimes force an adjustment -- an attempt to close the gap or to make things appear fair. 141 Yet, because the adjustment is triggered by the political consequences of the contradiction,

This approach to understanding legitimation and change


is applicable to the civil rights movement. Because Blacks were challenging their exclusion from political society, the only claims that were likely to achieve recognition were
circumstances will be adjusted only to the extent necessary to close the apparent contradiction.

those that reflected American society's institutional logic: legal rights ideology. Articulating their formal demands through legal rights ideology, civil rights protestors exposed a series of contradictions -- the most

Rather than using the contradictions to


suggest that American citizenship was itself illegitimate or false, civil rights protestors
proceeded as if American citizenship were real, and demanded to exercise the rights
that citizenship entailed. By seeking to restructure reality to reflect American mythology,
Blacks relied upon and ultimately benefited from politically inspired efforts to resolve the contradictions
by granting formal rights. Although it is the need to maintain legitimacy that presents powerless groups with the opportunity to wrest concessions from the dominant order, it is the very
important being the promised privileges of American citizenship and the practice of absolute racial subordination.

accomplishment of legitimacy that forecloses greater possibilities. In sum, the potential for change is both created and limited by legitimation.

Movements Fail
Have to transform systems from the inside out-otherwise rhetoric changes
but not policies.
McCormack 10
(Tara, PhD in International Relations from the University of Westminster, Critique, Security and
Power: The political limits to emancipatory approaches, pg 59-61)
In chapter 7 I engaged with the human security framework and some of the problematic implications of emancipatory security policy frameworks. In
this chapter I argued that the

shift away from the pluralist security framework and the elevation of cosmopolitan
and emancipatory goals has served to enforce international power inequalities rather than lessen
them. Weak or unstable states are subjected to greater international scrutiny and international institutions and
other states have greater freedom to intervene, but the citizens of these states have no
way of controlling or influencing these international institutions or powerful states. This
shift away from the pluralist security framework has not challenged the status quo, which may help to
explain why major international institutions and states can easily adopt a more cosmopolitan rhetoric
in their security policies. As we have seen, the shift away from the pluralist security framework has entailed a shift towards a more
openly hierarchical international system, in which states are differentiated according to, for example, their ability to provide human security for their
citizens or their supposed democratic commitments. In this shift, the old pluralist international norms of (formal) international sovereign equality, nonintervention and blindness to the content of a state are overturned. Instead, international institutions and states have more freedom to intervene in

theorists argue that


the goal of the emancipation of the individual means that security must be
reconceptualised away from the state. As the domestic sphere is understood to be the sphere of insecurity and disorder, the
international sphere represents greater emancipatory possibilities, as Tickner argues, if security is to start with the
individual, its ties to state sovereignty must be severed (1995: 189). For critical and emancipatory theorists
weak or unstable states in order to protect and emancipate individuals globally. Critical and emancipatory security

there must be a shift towards a cosmopolitan legal framework, for example Mary Kaldor (2001: 10), Martin Shaw (2003: 104) and Andrew Linklater
(2005). For

critical theorists, one of the fundamental problems with Realism is that it is


unrealistic. Because it prioritises order and the existing status quo, Realism attempts to impose a particular
security framework onto a complex world, ignoring the myriad threats to people emerging
from their own governments and societies. Moreover, traditional international theory serves to obscure power relations
and omits a study of why the system is as it is: [O]mitting myriad strands of power amounts to exaggerating the simplicity of the entire political system.
Todays conventional portrait of international politics thus too often ends up looking like a Superman comic strip, whereas it probably should resemble a
Jackson Pollock. (Enloe, 2002 [1996]: 189) Yet as I have argued, contemporary critical security

theorists seem to show a

marked lack of engagement with their problematic (whether the international security context, or the Yugoslav breakup and wars). Without concrete engagement and analysis, however, the critical project is
undermined and critical theory becomes nothing more than a request that people behave
in a nicer way to each other. Furthermore, whilst contemporary critical security theorists argue that
they present a more realistic image of the world, through exposing power relations , for
example, their lack of concrete analysis of the problematic considered renders them actually
unable to engage with existing power structures and the way in which power is being
exercised in the contemporary international system. For critical and emancipatory theorists the central place of the
values of the theorist mean that it cannot fulfil its promise to critically engage with contemporary power relations and emancipatory possibilities. Values
must be joined with engagement with the material circumstances of the time.

Institutional Focus Key


Institutional focus is necessary --- alt fails
Myers 13,
(Ella Assistant Professor of Political Science and Gender Studies at the University of Utah,
2013, Worldly Ethics: Democratic Politics and Care for the World, p. 44-45)
Unfortunately, Connolly is inconsistent in this regard, for he also

positions Foucauldian self- artistry as an essential


preliminary to, and even the necessary condition of, change at the macropolitical
level.104 That is, although Connolly claims that micropolitics and political movements work
in tandem, each producing effects on the other,105 he sometimes privileges action by the self on itself as a
starting point and necessary prelude to macropolitical change. This approach not only
avoids the question of the genesis of such reflexive action and its possible harmful
effects but also indicates that collective efforts to alter social conditions actually await
proper techniques of the self . For example, in a rich discussion of criminal punishment in the United States, Connolly
contends that today the micropolitics of desire in the domain of criminal violence has
become a condition for a macropolitics that reconfigures existing relations between
class, race, crime and punishment.106 Here and elsewhere in Connollys writing the sequencing renders
these activities primary and secondary rather than mutually inspiring and reinforcing.107 It
is a mistake to grant chronological primacy to ethical self-intervention , however. How,
after all, is such intervention, credited with producing salient effects at the macropolitical
level, going to get off the ground , so to speak, or assuredly move in the direction of
democratic engagement (rather than withdrawal , for example) if it is not tethered, from the

beginning, to public claims that direct attention to a specific problem, defined as


publicly significant and changeable? How and why would an individual take up
reflexive work on the desire to punish if she were not already attuned, at least partially, to
problems afflicting current criminal punishment practices? And that attunement is

fostered, crucially, by the macropolitical efforts of democratic actors who define a public
matter of concern and elicit the attention of other citizens.108

For reflexive self- care to be democratically

significant, it must be inspired by and continually connected to larger political


mobilizations . Connolly sometimes acknowledges that the arts of the self he celebrates are
not themselves the starting point of collaborative action but instead exist in a dynamic,
reciprocal relation with cooperative and antagonistic efforts to shape collective
arrangements. Yet the selfs relation with itself is also treated as a privileged site , the very
source of democratic spirit and action. This tendency to prioritize the selfs reflexive relationship over
other modes of relation defines the therapeutic ethics that ultimately emerges out of Foucaults and, to a lesser
degree, Connollys work. This ethics not only elides differences between caring for oneself and
caring for conditions but also celebrates the former as primary or, as Foucault says,
ontologically prior . An ethics centered on the selfs engagement with itself may have value, but it

is not an ethics fit for democracy .

State k Change
Social movements end and resort to institutional bureaucratic agencies
empirics prove the only way for progressive change is through the state
Schneiberg and Lounsbury 07 (Marc Schneiberg, PhD University of Wisconsin, and Michael Lounsbury,
Canada Research Chair in Entrepreneurship & Innovation Associate Dean of Research Univ. of Alberta, Social
Movements and Institutional Analysis, ph online @ http://citeseerx.ist.psu.edu/viewdoc/download?
doi=10.1.1.465.3710&rep=rep1&type=pdf )//akim
Schneiberg (2007; Schneiberg, King & Smith 2008) takes this avenue of research a step further in analyzing the development of
mutual, cooperative and publicly owned enterprise in the US economy. For the most part, populists

and the radical


anti-corporate movements of the late nineteenth and early twentieth centuries faced decisive defeats in
their efforts to forge alternatives to corporate capitalism. But even though they collapsed,
these movements nevertheless left behind organizational, cultural and institutional legacies bit and pieces
of the paths they had pursued, including theories of order, regulatory fragments, local movement
chapters, and alternative systems of enterprise in key industries. These legacies of previous
mobilization, in turn, served as legitimating structures, platforms and infrastructures for subsequent collective
mobilization in the same or related industries during the Progressive era, and then in the early New
Deal. Indeed, successive waves of reformers and anti-corporate forces built or transposed theories,
moral sentiments and cooperative forms out from insurance and other early sites of alternative enterprise into the
dairy and grain industries, the electrical utility industry and banking, elaborating what amounts to a secondary
path of industrial order in the US economy. Haveman, Rao and Paruchuris (2007) study of
Progressivism and savings and loans associations likewise highlights the distal and often unintended
effects of movements on organizational fields. Progressive activists quite deliberately and directly
sought to reform a variety of economic institutions, from the railroads to savings and loan associations. Yet they also fostered
Progressive models of rationality, bureaucratization and expert management within the
thrift industry indirectly, via two intermediary institutions. Activists formed Progressive newspapers that
exposed corruption and promulgated reform principles, and promoted city-manager forms of municipal government that
exemplified those principles, providing tangible analogies for reformers within the thrift industry. Both institutions

promoted the constitutive legitimacy of bureaucracy, prompting saving and loans associations to adopt
organizational forms more consistent with modernist moral sentiments. Nor are these processes confined to
economic industries or organizational dynamics. As Armstrong (2002, 2005) illustrates, the legacy of initial movements may also
include the establishment of new identities, cultural tools such as frames and logics, and creative contexts that enable
subsequent groups to continue struggles, mobilize and realize new gains in their efforts. The rise of the New Left in the 1960s
enabled the creation of new kinds of lesbian/gay organizational identities in San Francisco in the early 1970s. The development
of gay identity politics, in turn, proved crucial in structuring subsequent lesbian/gay organizations as well as enabling changes
within mainstream organizations such as the establishment of domestic partner benefits (Creed & Scully 2000; Scully & Creed
2005). While this work traces the sequencing and layering from outsider to insider movements, it

would be
interesting to also understand how insider movements facilitate outsider
mobilizations. Overall, the approach to movements and institutions that we advocate celebrates the
heterogeneity of actors, multiple logics and practice variation . A focus on such multiplicity revises
the isomorphic imagery of the canonical two-stage diffusion and punctuated equilibrium models (e.g., Tolbert & Zucker 1983).

Such a perspective concentrates less on the contagion of unitary practices or a singular rationality, but rather on
multiple forms of rationality that inform the decision making of actors in fields (Bourdieu 1984),
and provide foundations for ongoing struggle and contestation. This conceptualization of institutionalization and fields as
multiple, fragmented and contested (Schneiberg & Soule SOCIAL MOVEMENTS AND INSTITUTIONAL ANALYSIS 665
9781412931236-Ch27 11/22/07 7:09 PM Page 665 2005; Washington & Ventresca 2004; Lounsbury 2007) is

a crucial

ontological starting point for a new wave and generation of institutional scholars. And
when combined with a renewed attention to movements, it directs analytical attention to how historical
legacies of prior social action become embedded in existing fields, providing bases for
sequences of mobilization, and the construction of new paths from the elements or
ruins of old or forgotten orders. The early work in this direction has proven fruitful and
promises to propel institutional analysis for many years to come.

The alt fails and locks in the status quo approach to surveillance ---the
embrace of <contingency, radical democracy, alterity> discounts the role of
political institutions in modern life and reduces politics to an individualistic
project of ethical self-improvement and becoming
Lavin 6
(Chad, teaches political theory at Tulane University, Fear, Radical Democracy, and Ontological
Methadone, Polity (2006) 38, 254-275
radical democracy suggests

While liberalism exhorts us to take comfort in the promises and assurances of a fixed identity,
revealing this identity for
the reification that it is so that we can move beyond a tolerant modus vivendi and toward an ethical comportment of generosity and meaningful democracy. They prescribe, as a
cure for postmodern agency panic, an enthusiastic embrace of the contingency of everyday life, a series of practices of the self that force an examination of the existential

The problem of fear, they


suggest, resides in a dogmatic clinging to an untenable and disintegrating myth of
subjectivity with an unrealizable promise of control. Their response to the fear of homelessness runs directly counter to the
resentment felt by subjects of liberal capitalism: Don't adhere to a manufactured map! Learn to be at home in homelessness!

conspiracy theorist: with a model of contingency rather than conspiracy, there are no villains, thus no reservoir for depositing and segregating one's fear. Rather than depositing
this fear at the feet of a scapegoat, face it; overcome it. While challenging the liberal fetish for reifying a historical production and thus closing the door on possibilities for
alternative subjects to emerge, radical democrats also acknowledge the work done by liberalism to remove metaphysical obstacles to the expression of agency. As such, their
claim is not antiliberal so much as it is postliberal, interrogating the limits of the liberal subject in pursuit of a more open approach to identity and difference. For the identities and
attachments that have been forged through liberalism are anything but fetters to the unhindered exercise of democracy and production of difference. They are not only
productive of many of the freedoms that we enjoy today in the liberal world (e.g., civil rights and liberties, relative government accountability), they also offer valuable solace
from a hostile and increasingly unpredictable existence. But because the prescription for those suffering from agency panic tends to be found in the capitalist myths of merit,

we might look at liberalism as the opiate of the people, easing our pain, but
preventing us from doing the work necessary to attend to the source of our ills. Perhaps liberalism, then, for all its value and appeal, is a habit we need to kick. Radical
autonomy, and authenticity,

democrats recommend sobering up and facing our fears head-on. But where's the Betty
Ford Center for radical democracy? If the fundamentalist drive to reify assumptions and the democratic drive to challenge them are two
responses to the same panic that results from social pluralization (as Connolly convincingly claims), the question remains: why do some
subjects flee from this fear (through fundamentalism), while others draw from it strength
and forbearance (through radical democracy)? What resources allow some subjects to
respond to revealed inadequacies of their foundational beliefs with a gracious humility
while others respond by desperately clinging to and fundamentalizing the contested
position? The short answer is that the democratic move is not an easy one to make. While liberal cognitive maps
respond to the complexity of contemporary power networks with a promise of predictability and stability through the claims to fixed identity, rational control, and market

radical democracy promises precisely what subjects of liberalism fear: enduring


instability. The ethos of forbearance asks subjects not only to accept social and political instability today, but ontological instability forever after (whereas the
fundamentalizing moves promise order not only in the future, but now). By introducing contingency into the very substance
of being, this approach leaves us in indeterminacy without even a promise of finding
answers; it is "tentative, experimental," emphasizing the need for perpetual work of
"cultivation" without the promise of attaining a stable system ;41 it is "a risky and ambiguous enterprise" always
efficiency,

threatening to destroy self-confidence as soon as it is built;42 it offers "no necessary political consequences," but only possible gains in freedom.43 Going back to Hobbes,

radical democrats suggest that we should embrace rather than run from or even merely
tolerate the very conditions we fear: unknowability, instability, and discord. Further, Connolly

insinuate

that liberal authenticity has been thoroughly debunked

and Butler
unmistakably
, still adhered to only by
potential or actual fascists who (at best) have been thoroughly colonized by the culture industry and the ideological apparatuses of the state or (at worst) have no sympathy for

They thus imply that it is the


social critic who bears the mantle of democracy, and that it is not fear but a stubborn
authoritarian desire that obliges resistors to slap the table to illustrate its non-discursiveness and pound their chests to
prove their individuated, corporeal, nonporous existence. (A spectre is haunting criticismthe spectre of Leninism.) We
can almost hear radical democrats responding that liberal individualists have nothing to
lose but their chains. But if the reigning paradigms of identity and order offer solace
from an increasingly indeterminate and unpredictable world (complete with massive
layoffs, abandonment of traditional gender norms, declining national sovereignty, and
other existential disintegrations), then by what standard can radical democrats call upon
the subjects of liberal capitalism to reject the myths that make lives livable? These calls
might be the ontological counterpart of ham-fisted approaches to drug abuse, exhorting
addicts to take responsibility for themselves and quit cold turkey, with little attention to
the social and psychological dynamics that lead millions of users to abuse narcotics, hallucinogens,
opiates, and amphetaminesbe they consumers of heroin, religious fervor, Xanax, or extreme corn chips and suburban assault vehicles. The sales pitch for
radical democracy rarely accounts for the degree to which citizens are interpellated as
subjects of fear. Connolly is more than aware of the interpellations that subjects of capitalism receive; large sections of The Ethos of Pluralization are dedicated
the contemporary movements interested in expanding the democratic web to include currently disenfranchised populations.

to the relationship between economic anxiety and the drive to fundamentalism, and Why I Am Not a Secularist is largely an indictment of secularism (and much of American
liberalism) for failing to account for the habits inscribed on the body that rational argumentation cannot erase.44 His attention to what he calls "the visceral register," and his

Unfortunately,
Connolly's intellectual trajectory has shifted away from the social and political
recommendation that the left take a lesson from William Bennett on how to appeal to this register is both compelling and timely.45

apparatuses which inculcate these habits and toward the level of neurology to explain
the inward mechanisms of habits and ethics ;46 while he never denies the relevance of
social and political institutions (he merely argues that "too many cultural theorists" ignore the biological components of thinking and ethics), his
trajectory has led him to speak more about what practices of the self might lead to more
democracy and less about the institutional production of subjects who actually fear
democracy .47 As White convincingly argued well before this latest turn toward biological studies, Connolly takes attitudes to have
primacy over issues of justice, leading him to prematurely curtail pursuit of the obviously
compatible institutional dimension of criticism.48 The common scoundrel in such critiques of liberalism, the prototypical
antidemocratic citizen, is typified by Archie Bunker and his white working-class buddies. Resentful, socially and economically conservative, and easily seduced by reactionary
propagandists, this was the face of fundamentalism before 9/11. Notably, this is a population whose economic viability and (therefore) ability to live up to conventional norms of

Connolly notes the familiar


targets: civil rights, Vietnam, feminism, dismantling of the welfare state,
environmentalism, and a decline in the availability of industrial jobs.49 Along with these
transformations in established relations of power, we can also add to this list (which
Connolly may or may not have intended to be exhaustive) the supposed death of grand
narratives declared by critical intellectuals. Traditional cognitive maps are not only
becoming obsolete in the wake of shifting terrain, but they are being criticized for being
defective from the get-go. Given this, is the rise in public fear a surprise? As Jameson's, Melley's, and Furedi's attention to the paucity of concepts
suggests, isn't the declining availability of cognitive maps to those threatened by shifting networks of power a prime candidate for Connolly's list?50 If
fundamentalisms feed on the anxieties endemic to late capitalist (dis)order; if market
ideology, conspiracy theories, and aggressive nostalgia are degraded attempts to construct cognitive maps that
might provide threatened subjects with a sense of agency; what does radical democracy
offer in their stead? Traditional fundamentalisms are popular precisely because they treat the agency panic and existential angst accelerated by global
masculinity have been taking it in the gut in recent decades. Identifying the threats that have constituted this subject,

capitalism and postmodern culture. Again, Connolly recognizes the absence of viable democratic supplements to the decline of grand narratives, and he laments that the world's
most zealous moralizers have effectively filled that void. But claims that rethinking liberalism's ontological commitments cannot be an afterthought of political emancipation (but
must be coterminous with or preliminary to it), betray an inattention to how a postmodern ethos is intimately tied to the various psychological, economic, and political securities
that cognitive maps often provide.

While Connolly's diagnosis of fundamentalist epidemics clearly

highlights their social and economic roots, he presents a postmodern sensibility as a


treatment for the anxieties of identity without considering the role that this move may
have played in actually producing those anxieties by threatening the cherished existence
of those perhaps most in need of their help.51 My concern is not the diagnosis, but the
prescription, as it is for a regimen that few have any interest in adopting. Radical
democracy is premised on the idea that its ontology and ethics are necessary to an
enactment of the pluralism that liberalism purports to offer, transcending mere tolerance
toward a generous fostering of otherness and a meaningfully democratic way of life. At the
same time, however, it is insensitive to the profound threat that genealogy poses to those who
receive their only respite from a truly tenuous existence from their myths. While its pitch suggests that it
is ontologically more defensible, ethically more admirable, and practically more conducive to a generous ethical-political comportment (contentions with which I agree,

), it does not consider the sheer difficulty (and often terror) in adopting such a
position. Certainly, it does not speak to the members of society who are the most
economically, politically, and/or socially disadvantaged, whose daily survival is so
tenuous as to not provide the luxury for the practices of self-cultivation which it
advocates. I am far from the first to call attention to the dangers of and aversions to genealogy. The aristocratic baggage of Nietzscheanism, from which radical
incidentally

democrats have vigorously sought to distance themselves, stems largely from Nietzsche's acknowledgement of the difficulties of abandoning foundations. No friend of pity,
Nietzsche nevertheless prescribes revaluing our constitutive values only to a privileged class, maintaining that we cannot ask lambs to be birds of prey.52 Similarly, William
James discusses the differential capacities of modern subjects to be at home in homelessness, expressing concern for those who might not be ready or able to make such a

the "healthy" has a responsibility to attend to the "sick" soul,


and that some are not ready, willing, or able to abandon their most cherished myths : "Some
move. He argues (with unfortunate terminology) that

constitutions need them too much."53 Returning to our liberal fundamentalist, Rorty emphasizes that "most people" are not interested in facing the contingency of their identities,
and that "there is something very cruel about" revealing their contestability and groundlessness.54 While these arguably elitist arguments might sit uncomfortably with the
radically democratic impulses of Connolly and Butler, which is more democratic: Throwing everybody into the same pool? Or realizing that not everybody knows how to swim?
Or realizing that some citizens, having seen their parents devoured by sharks, might think the waters unsafe? Connolly reads Rorty's courteous capitulation to fear as an

I do not mean
to suggest that consistency would oblige radical democrats to respect discourses
hostile to pluralization (Connolly addresses this issue repeatedly). Nor do I want to imply that we should refrain
from attacking the antidemocratic presumptions of various fundamentalisms. But I am
concerned that the approach to fundamentalism in this literature is something much
closer to hostility than sympathy (to say nothing of empathy). For all of its talk of generosity, radical
democracy remains quite stingy with regard to this issue. Accepting that there remains a
compatibility between foundational thought and undemocratic or authoritarian politics,
the apparent expectation that the articulation of this compatibility might convince any
lingering fundamentalists of their error is distressing . In fact, Connolly effectively labels
those lacking in the material, philosophical, or psychological resources necessary for
the move he recommends latent brownshirts, thus betraying not only a commitment to
liberal voluntarism, but also a striking lack of generosity toward those still in need of
comforting maps.56 Radical democrats clearly recognize the material conditions that may not provide the comfort necessary to develop an ethos of
generosity. As diagnosticians, they are certainly attuned to the tactics applied by institutions
(markets, ideologies, militaries) to subjects and the constant interpellation of subjects of
fear. Nevertheless, the prescribed micropolitics of desire summons a heroic capacity to
respond generously to the myriad threats we encounter. Recommending that subjects abandon the cognitive maps
(Christianity, conspiracy theories, market fundamentalism) that allow them to find their bearings in increasingly complicated and vast networks of power, they
ultimately reduce liberal fundamentalism to a problem of individual priorities and ethical
failures rather than a medical condition requiring professional treatment or (better) a
public health issue not easily or appropriately tied to any individual's preferences. As in
abandonment of irony precisely where it is most important. Whereas Rorty coddles subjects of fear, Connolly scolds them for their cowardice.55

contemporary U.S. drug policy rhetoric, the addiction is all-too-easily reduced to an


issue of personal responsibility, taking focus away from the institutions responsible for
the daily interpellation of subjects of fear. Conclusion Despite a relatively mature Constitution,

Americans are no strangers to instability. In recent decades, the assurances of job security and the welfare state have fallen victim to a
vicious market ideology; traditional codes of masculinity and femininity have become increasingly unrealizable with Martha Stewart and J. Lo doing their best to humiliate the
ladies, Bill Gates and Vin Diesel the gentlemen; and illusions of effective agency have been ravaged by unprecedented technological and geopolitical transformations. In this

though radical democrats denounce the comforting liberal myth of the autonomous
individual as a ruse of antidemocratic power, its ideology of identity, authenticity, and
responsibility provides relief from the ontological homelessness endemic to a world with
ever fewer uncontested narratives. Liberalism allows us to personalize and segregate various anonymous hostilities, facilitating displacements
condition,

of general anxieties onto welfare moms, homosexual teachers, professional women, non-white street criminals, Zionist Occupied Governments, and Islamic fundamentalists.
While the reactionary grasp at liberal fundamentalism certainly constitutes an obstacle to a democratic politics of difference, it is also the case that subjects of late capitalism are
interpellated as subjects of fear, reared to understand every component of society with suspicion. Navigating the breathtaking and impersonal forces of bureaucratic capitalism
with categories which emphasize (indeed, almost exclusively mention) the powers of isolated individuals, we are led to an ever more hopeless situation. As history moves on
and our cognitive maps seem less and less relevant for helping us chart networks of power, we feel an overwhelming sense of powerlessness. The ideology of autonomy,
authenticity, and responsibility seems an entirely logical aid for coping with this agency panic. Indeed, liberalism both creates and then assuages the fears of late capitalism. In

radical democrats
trenchantly reveal liberal ideology as an addictive and distorting source of relief. But in
prescribing a micropolitics of desire to kickstart the stalled project of democratization,
they locate the work of politics in the contested site of the individual and discount the
degree to which these individuals actually fear the unpredictable and radically
democratic order being promised. Contesting the validity of the cognitive maps most commonly used to steer a course through institutions of
social and political power, they ask subjects to abandon the anesthetizing components of liberal
fundamentalism for a world composed of unstable identities and provisional reconciliations that are grounded in a set
of ontological commitments that are weak at best. But as all but the most steadfast purveyors of simplification in
this age of Panic, in which our surroundings appear at least comparably if not more alive and efficacious than our selves,

the so-called war on drugs realize, addiction is inscribed on the body, and requires both
desire and treatment to be overcome. Radical democrats realize why we are identity
junkies. But where, oh where, are the ontological treatment centers and methadone
clinics?

Psychoanalysis

2AC Psycho Bad


Psychoanalysis is not empirical and has no explanatory power --- prefer
social science because it can explain events based on causal relationships
Sadovnikov 7
(Slava York University, "Escape from Reason: Labels as Arguments and Theories", Dialogue
XLVI (2007), 781-796, philpapers.org/archive/SADEFR.pdf)
The way McLaughlin shows the rosy prospects of psychoanalytical social theory boils down to
this: there are people who labour at it. He reports on Neil Smelsers lifelong elaborations of psychoanalytical sociology, which prescribed the use of
Freudian theories. Then he presents a powerful psychoanalytical theory of creativity of Michael Farrell, commenting on how the theorist usefully utilizes psychoanalytic

He correctly expects that I might not view his examples as


scientic. Their problems begin well before that. First, due to their informative emptiness, or
insights, though McLaughlin does not specify them.

tautological character , all they amount to is rewordings of everyday assumptions. Second,


due to their vagueness these accounts are compatible with any outcomes; in other
words, they lack explanatory and predictive power . The proposed ideas are too
inarticulate to subject to intersubjective criticism, and to call them empirical or scientic
theories would be, no matter how comforting, a gross misuse of words. On the constructive side, a
psychoanalytic theorist may be challenged to unambiguously formulate her suppositions and specify conditions of their disproof, to leave out what we already well know and
smooth out internal inconsistencies, and revise the theories in view of easily available counter-examples and competing accounts. Only after having done this can one present
candidate theories to public criticism and thus make them part of science, and fruitfully discuss their further renements. Another suggestion is not to label them powerful

That criticism and disagreement are indispensable


for science is not a Popperian orthodoxy , although Popper does champion this idea; it is the pivot of the
tradition (which we owe to the Greeks) which identies rationalism with criticism. 4 McLaughlin ostensibly bows to the critical
theories, classics, or anything else before their real scrutiny begins.

tradition but does not put it to use. Instead of critical evaluation of the theories in question he writes of compelling case, powerful analytic model, and useful conceptual tool.

On the methodological side of the issue, we should inquire into the mode of thinking
common to Fromm and all adherents of conrmation-ism. The trick consists in mere replacement of familiar
words with new, more peculiar ones; customary expressions are substituted by instrumental intimacy, collaborative circles, and idealization of a selfobject. Since the new, funnier, and pseudo-theoretical tag does the job of naming just as well, it
shows how things work. The new labels in the cases criticized here do not add anything to our
knowledge; nor do they explain. We have seen Fromm routinely abuse this technique. The vacuity of Fromms explanations by character type

was the central point in my analysis of Escape , yet McLaughlin conveniently ignores it and, like Fromm, uses the method of labelling as somehow supporting his cause.

The widely popular practice of mistaking new labels for explanations has been exposed
by many methodologists in the history of philosophy, but probably the most famous example of such critique comes from Molire. In the now oftenquoted passage, his character delivers a vacuous explanation of opiums property to induce sleep by renaming the property with an offhand Latinism, virtus dormitiva. The
satire acutely points not only at the impostor doctors hiding his lack of knowledge behind foreign words, but also at the emptiness of his alleged explanation. (Pseudotheoretical literature is boring precisely because of its dormitive virtue, its shufing of labels without rewarding inquiring minds.) Let me review notable criticisms of this
approach in the twentieth century by Hempel, Homans, and Weber leaving aside their forerunners. This problem was discussed in the famous debate between William Dray and

social scientists often try to answer the


question, What is this phenomenon? by giving an explanation-by-concept (Dray 1959, p. 403).
Carl Hempel. Dray argues, contra the nomological account of explanation, that historians and

A series of events may be better understood if we call it a social revolution; or the appropriate tag may be found in the expressions reform, collaboration, class struggle,
progress, etc.; or, to take Fromms suggestions, we may call familiar motives and actions sadomasochistic, and any political choice save the Marxist escape from freedom.

such concepts may be explanatory, but they are so only if the chosen
labels or classicatory tags refer to some uniformities, or are based on nomic analogies. In other words, our new
label has explanatory force if it states or implies some established regularity (Hempel 1970, pp. 453-57).
For example, you travel to a foreign country and, strolling along the street, see a
boisterous crowd. Your guide may explain the crowd with one of several terms: that it is the
local soccer teams fans celebrating its victory, or it is a local religious festival, or a teachers strike,
Hempel agrees with Dray that

etc. The labels applied herecelebration, festival, strike have explanatory value ,
because we know that things they refer to usually manifest themselves in noisy or unruly
mass gatherings. If, on the other hand , by way of explaining the boisterous crowd the
guide had invoked some hidden social or psychological forces , or used expressions such
as embodiment, mode of production, de-centring, simulacra, otherness, etc., its causes would remain obscure .
If she had referred to psychoanalytic character types (say, Fromms authoritarian, anal, or necrophiliac types), the
explanation would not make much sense either . Nothing prevents us nevertheless from unconditionally attaching all these labels
to any event. The mistake McLaughlin and conrmationists persistently make is in thinking that labelling
social phenomena alone does theoretical and explanatory work. 5 George Homans observed the prevalence of
this trick some decades ago: Much modern sociological theory seems to us to possess every virtue except
that of explaining anything . . . . The theorist shoves various aspects of behavior into his
pigeonholes, cries Ah-ha! and leaves it at that. Like magicians in all times and places, the theorist
thinks he controls phenomena if he is able to give them names, particularly names of his
own invention. (1974, pp. 10-11)

Non-Falsifiable
Do not let them shield their criticism with unfalsifiablity - sidelines truth
and makes it impossible to debate - research proves
Kay et al 15
(Justin P. Friesen [Ph.D in Psychology at The University of Winnipeg] Troy H. Campbell [Ph.D
student at Duke University and in Fall 2015 a professor at the University of Oregon] Aaron C.
Kay [Associate Professor of Management and Associate Professor of Psychology &
Neuroscience at Duke University], 2015, The Psychological Advantage of Unfalsifiability: The
Appeal of Untestable Religious and Political Ideologies, Journal of Personality and Social
Psychology, Vol. 108, No. 3, http://dx.doi.org/10.1037/pspp0000018, Pages 524-525, MX)
unfalsifiability may lead to polarization and intractability
at a societal level and how people may actively wish to remove science from the discussion in
an order to give their cherished beliefs an armor of unfalsifiability. Experiment 4 in particular may resonate with anyone
We conclude with a discussion of the macro social effects of unfalsifiability on belief, particularly how

who has watched cable news and seen a pundit tell a scientist that the data is just the scientists opinion and immediately turn a conversation of objective metrics into a conversation of beliefs. It seems as if

people are motivated not only to deny isolated scientific findings but reduce the role of scientific inquiry in
answering important social questions as doing so shields ones own beliefs in an armor
of unfalsifiability. For example, 2012 Republican vicepresidential candidate Paul Ryan said: Even if Washington could be good at picking winners and loserswhich theyre notthey
shouldnt be in the business of picking winners and losers. Thats not the role of government. (Stewart, 2012) Ryan believes the facts are on his side, but should he turn out to be wrong, he has already
preemptively couched the issue of limited government as unfalsifiable moral principle instead of the more falsifiable question of government effectiveness. It seems increasingly difficult to have rational discussion

current research suggests that this difficulty may be, at least in part, because
peoples beliefs are not based on falsifiable statements. Moreover, if including unfalsifiability is
one defensive response to threat, popular beliefs systems may evolve to include more aspects
of unfalsifiability over time, such as by marginalizing the relevance of science if they suspect that science
in the polarized political climate (DiMaggio, Evans, & Bryson, 1996). The

does not support their beliefs. To the extent that scientific inquiry is the best method to test hypotheses and falsify beliefs, people who have salient psychological needs, other than needs for accuracy, may

even flawless and clearly communicated


science may not convince people because people often have a powerful trump card
hidden up their sleeve: unfalsifiability. Our findings also have applied applications for the communication of science, and future research should take into
progressively distance their beliefs from science and the corresponding social discourse. Scientists must be aware of such tendencies and see that

account the defensive functions of unfalsifiability. For instance when unfalsifiability is most likely to be incorporated, under what mindsets people will permit themselves to hold falsifiable beliefs, how does

unfalsifiability may be a
dangerous force in society at large. Though it might benefit individuals psychologically or groups socially, unfalsifiability might
also lead people and societies to continually make truthdefying decisions. To the extent that the success of a
temporary versus ultimate unfalsifiability influence beliefs, and how unfalsifiability might feel uncomfortable. These results and speculations suggest that

society largely depends on its ability to respect good data and change behavior accordingly (Sheikh, 2013), a devotion not just to ideas but to testing those ideas is necessary for the welfare and improvement of
the society. Understanding the appeal of unfalsifiability is therefore an important question for a world that seems, on the one hand, actively interested in testable scientific data as evidenced by recent reforms in
the United States to evidence-based policy (Baron, 2012; Borstein, 2012) but, on the other hand, at times seems to wish to remove the data from the discussion. This puzzle must be solved if evidence-based
policy is to win out.

Robinson
Lacanianism relies on a prior assumptions and functions at the ontological
rather than the political- its use of universals obfuscates the fact that its
neither original nor applicable
Robinson 4
(Andrew Robinson [Postgraduate Student, School of Politics, University of Nottingham], 2004,
The Politics of Lack, British Journal of Politics & International Relations, Volume 6, Pages 260262, MX)
the return of the Real is always a disruptive, almost revolutionary event which
shatters the entire social totality constructed around its exclusion. Every social order,
therefore, has a single touchy nodal point which it must maintain, or else it will collapse
Lacanians urge that one reconcile oneself to the inevitability of
lack
The acceptance of lack takes the form
of an Act
in which the myth of subjective completeness is rejected and the
incompleteness of the self is embraced. The primary ethical imperative in Lacanian
politics is to accept the primacy of antagonism
For Lacanians,

Since the exclusion of a Real element is supposed to be necessary,

. Lacanian politics is therefore about coming to terms with violence, exclusion and antagonism, not about resolving or removing these.
or Event,

, i.e. the central ontological claim of the Lacanian edifice itself. Mouffe, for instance, demands that one accept an element

of hostility among human beings as something akin to a fact of human nature (2000, 130132). She attacks deconstructive and dialogical approaches to ethics for being unable to come to terms with the political in its antagonistic dimension. Such approaches
lack a proper reflection on the moment of decision which characterises the field of politics and which necessarily entail[s] an element of force and violence (ibid., 129130). Mouffes alternative involves a politics which acknowledges the real nature of [the]
frontiers [of the social] and the forms of exclusion that they entail instead of trying to disguise them under the veil of rationality or morality (ibid., 105). She celebrates democracy, but her version of democracy depends on the possibility of drawing a frontier between
us and them and always entails relations of inclusionexclusion (ibid., 43). The derivation of such views is unclear from the text, but seems to be that, since everyone needs a master-signifier as an element in their psyche, and since such a signifier arises
through the machinations of the political, therefore the exclusionary and violent operations of coercive state apparatuses must be accepted as an absolute necessity for any kind of social life. This is Hobbesian statism updated for a post-modern era. As should by

the central claims of Lacanian theory are ontological rather than political. Indeed,
since Lacans work deals with politics only very occasionally, the entire project of using
Lacan politically is fraught with hazards.
Lacanian theorists put ontology in the
driving seat, allowing it to guide their political theorising. Political discourse and events
are subsumed into a prior theoretical framework in a manner more reminiscent of an
attempt to confirm already-accepted assumptions than of an attempt to assess the
theory itself
now be clear,

With rare exceptions,

. Among the authors discussed here, i ek takes this the furthest: the stuff of theory is notions, which have a reality above and beyond any referent, so that, if reality does not conform to the notions, it is so much the worse for

reality (in Butler, Laclau and i ek 2000, 244). The selection and interpretation of examples, whether in concrete analysis of political discourse or in theoretical exegesis, is often selective in a way which appears to confirm the general theory only because
inconvenient counterexamples are ignored. The entire edifice often appears wholly a priori and non-falsifiable, and the case for its acceptance is extremely vague. Most often, the imperative to adopt a Lacanian as opposed to (say) a Rawlsian or an orthodox Marxist
approach is couched in terms of dogmatically-posited demands that one accept the idea of constitutive lack. A failure to do so is simply denounced as shirking, blindness, inability to accept and so on. In this way, Lacanian theory renders itself almost immune to
analytical critique on terms it would find acceptable. Furthermore, a slippage frequently emerges between the external acceptance of antagonism and its subjective encouragement. For instance, Ernesto Laclau calls for a symbolisation of impossibility as such as a
positive value (in Butler, Laclau and i ek 2000, 1999, original emphasis). The differences between the texts under review mainly arise around the issue of how to articulate Lacanian themes into a concrete political discourse. This becomes especially clear in the
Butler, Laclau and i ek volume from which the above quotation is taken. Laclau and i ek share a theoretical vocabulary and agree on a number of issues of basic ontology. However, they bothand each in an equally dogmatic wayinsist on a particular
decontestation of this vocabulary in their analysis of contemporary events. For Laclau, Lacanian analysis dovetails with radical democracy, whereas for i ek, it entails a radical refusal of the status quo from a standpoint cross-fertilised with insights from Hegel, Kant

disagreement represents a broader split of Lacanian theorists into two camps


radical democrats
and more-or-less nihilistic Lacanians
Butler
not sufficiently committed to an ontology of lack to accept the other protagonists
inability to provide substantial argumentation for their positions. She calls Lacanian
theory a theoretical fetish, because the theory is applied to its examples, as if already
true, prior to its exemplification
that the Lacanian project is in a certain sense a theological project, and that its
heavy reliance on a priori assumptions impedes its ability to engage with practical
political issues, using simplification and a priori reasoning to avoid the rather messy
psychic and social entanglement involved in studying specific political cases
She
could perhaps have added that, in practice, the switch between ontology and politics is
usually accomplished by the transmutation of single instances into universal facts by
means of a liberal deployment of words such as always, all, never and necessity
and the Marxist tradition. This

competition,

who follow Laclaus line that liberal democracy is a realisation of the Lacanian model through the acceptance of antagonism and its conversion into symbolically accepted electoral and interest-group
such as i ek and Badiou who maintain that a Lacanian politics requires a radical break with the present political system.

, for

her part, is

. Articulated on its own self-sufficiency, it shifts its basis to concrete matters only for pedagogical purposes (in Butler, Laclau and i ek 2000, 2627). She suggests, quite

accurately,

(ibid., 155156).

; it is by this

specific discursive move that the short-circuit between theology and politics is achieved. Butler questions the political motivations involved in such practices. Are we using the categories to understand the phenomena, or marshalling the phenomena to shore up the

the two
Lacanian camps both face enormous problems once they attempt to specify their
political agendas
The claim
that liberal democracy is necessary to take the bite out of intractable conflicts arising
from human nature, and the resultant condemnation of utopian theories such as
Marxism for ungrounded optimism and resultant totalitarian dangers, is hardly original
Since the appeal of Lacanian
theory depends on its claims to be offering a new, radical approach to politics, such
similarities must be downplayed.
categories in the name of the father [i.e. the master-signifier]? (ibid., 152). The problems raised by Butler are serious, and reflect a deeper malaise. Aside from the absence of any significant support for their basic ontological claims,

. For the Laclauians, the greatest difficulty is that of maintaining a critical position even while endorsing assumptions remarkably close to those of the analytical-liberal mainstream.

. To take

one example, it arises in Rawls discussion of reasonable pluralism and the burdens of difference in Political Liberalism (1996, Lecture 2 and passim).

much of

AT: Lack
Desire does not lack - their conception ignores the reality that need is a
product of desire - turns the criticism because only desire can solve
Deluze and Guatarri 72
(Gilles Deleuze [French philosopher], Flix Guattari [French psychiatrist and political activist],
1972, Anti - Oedipus: Capitalism and Schizophrenia, Translated from the French by Robert
Hurley, Mark Seem, and Helen R. Lane, Pages 26-28, MX)
If desire produces, its product is real. If desire is productive, it can be productive only in the real world
and can produce only reality. Desire is the set of passive synthes s that engineer partial objects, flows, and bodies, and that function as units of production. The real is
the end product, the result of the passive syntheses of desire as autoproduction of the unconscious. Desire does not lack anything; it
does not lack its object. It is, rather, the subject that is missing in desire, or desire that lacks a fixed subject; there is no fixed subject unless there is repression. Desire
and its object are one and the same thing: the machine, as a machine of a machine. Desire is a machine, and the object of desire is another machine
connected to it. Hence the product is something removed or deducted from the process of producing: between the act of producing and the product, something becomes detached, thus giving the vagabond,

The objective being of desire is the Real in and of itself.* There is no particular form of existence that can be
labeled "psychic reality." As Marx notes, what exists in fact is not lack, but passion, as a "natural and sensuous
object." Desire is not bolstered by needs, but rather the contrary; needs are derived from desire: they are counterproducts within the real that desire produces.
Lack is a countereffect of desire; it is deposited, distributed, vacuolized within a real that is natural and social. Desire always remains
in close touch with the conditions of objective existence; it embraces them and follows them, shifts when they shift, and does not
outlive them. For that reason it so often becomes the desire to die, whereas need is a measure of the
withdrawal of a subject that has lost its desire at the same time that it loses the passive syntheses of these conditions. This is
precisely the significance of need as a search in a void: hunting about, trying to capture or become a parasite of passive syntheses in
whatever vague world they may happen to exist in. It is no use saying: We are not green plants; we have long since been unable to synthesize chlorophyll, so it's necessary to eat. . . . Desire then
becomes this abject fear of lacking something. But it should be noted that this is not a phrase uttered by the poor or the dispossessed. On the
contrary, such people know that they are close to grass, almost akin to it, and that desire "needs" very few thingsnot those leftovers that chance to come their way, but
the very things that are continually taken from themand that what is missing is not things a subject feels the lack of
somewhere deep down inside himself, but rather the objectivity of man, the objective
being of man, for whom to desire is to produce, to produce within the realm of the real. The
real is not impossible; on the contrary, within the real everything is possible, everything becomes possible. Desire does not
express a molar lack within the subject; rather, the molar organization deprives desire of its objective being. Revolutionaries, artists, and seers are content to be objective,
nomad subject a residuum.

merely objective: they know that desire clasps life in its powerfully productive embrace, and reproduces it in a way that is all the more intense because it has few needs. And never mind those who believe that this
is very easy to say, or that it is the sort of idea to be found in books. "From the little reading I had done I had observed that the men who were most in life, who were moulding life, who were life itself, ate little,

had no illusions about duty

slept little, owned little or nothing. They


, or the perpetuation of their kith and kin, or the preservation of the State. . . . The phantasmal world is the
world which has never been fully conquered over. It is the world of the past, never of the future. To move forward clinging to the past is like dragging a ball and chain."30 The true visionary is a Spinoza in the garb

Lack (manque)* is created, planned, and organized


in and through social production. It is counterproduced as a result of the pressure of antiproduction;the latter falls back on (serab at sur) the forces of production and
appropriates them. It is never primary; production is never organized on the basis of a pre-existing need or lack
(manque). It is lack that infiltrates itself, creates empty spaces or vacuoles, and propagates itself in accordance with the organization of an already existing organization of production. The
deliberate creation of lack as a function of market economy is the art of a dominant class.
of a Neapolitan revolutionary. We know very well where lackand its subjective correlativecome from.

This involves deliberately organizing wants and needs (manque) amid an abundance of production; making all of desire teeter and fall victim to the great fear of not having one's needs satisfied; and making the
object dependent upon a real production that is supposedly exterior to desire (the demands of rationality), while at the same time

categorized as fantasy and nothing but fantasy.

the production of desire is

Perm/No Link Stuff

Permutation
Permutation do the Alternative - Severing the 1AC allows us to pass from
attachment to detachment thus reclaiming our identities as Lacanian
subjects and re-instituting desire
Scott 15
(Maria Scott [PhD in French at Trinity College Dublin], 2015, Deciphering the Gaze in Lacan's
"Of the gaze as objet petit a"', DS Project, MX)
The always renewed re-institution of desire is at the core of Lacanian ethics. In order to renew ones desire, it is
necessary first to fall sway to the drive. In Seminar XI, the satisfaction of the drives requirements, as for example in the viewing of art, becomes the ethical
goal of analysis. After the seminars on the gaze, the object a appears in Seminar XI as the libido or genital drive. This pure life instinct is the organ from which the
subject separates in order to exist in society. Because this instinct is irrepressible, however, the subject must renew continually her
detachment from the libido, so as neither to revert to an undetermined hommelette, nor to become frozen as a signifier. The subject is truly
itself, according to Lacan, only in the passage from attachment to detachment: The subject is this emergence which, just before, as
subject, was nothing, but which, having scarcely appeared, solidifies into a signifier. The Lacanian subject has, therefore, no other being
than as a breach in discourse and manifests itself in daily life as a fleeting interruption
of something foreign or extraneous, a pulsation, an occasional impulse or interruption that immediately dies away or is extinguished.

Perm Do Both
Permutation do both - Pragmatism and psychoanalysis are more similar
than most realize
Hanlon 1
(Christopher Hanlon [PhD in Philosophy at University of Massachusetts Amherst], May 2001,
Pragmatism and the Unconscious: Language and Subject in Psychoanalytic Theory, Pragmatist
Philosophy, and American Narrative, Scholarwork@UMassAmherst,
http://scholarworks.umass.edu/dissertations/AAI3012135/, Page 19, MX)
pragmatism and psychoanalysis may already be
on speaking terms, though in ways quite apart from those Rorty suggests in his essay on Freud. It explores this possibility by
investigating the relations between classical American pragmatist philosophy and the
psychoanalysis of Jacques Lacan, relations sometimes secured through textual transmission and, at other moments,
striking conceptual affinity. Through readings of a handful of signal textsboth "pragmatist" and "psychoanalytic"--which stand at the
intersections of postmodern theory and American intellectual history, it enters into a
network of problems: problems of language and consciousness, of body and mind, of
gender and race, of power and know ledge, of fantasy and reality, of philosophy, of
literature, of reading. Thus, while working to produce an acquaintance between some key
issues of concern for Americanist intellectual historians and some of the key concepts of
Lacan's theoretical corpus, it will continually pose questions which are normally thought
of as "clinical" in their psychoanalytic context, but which may also serve to enliven what
have become a series of ossified debates within American philosophical, literary, and
cultural critical circles.
This dissertation represents an attempt to seriously entertain the notion that

No Death Drive
No death drive - human destructiveness is a complex development process
De Masi 14
(Franco De Masi [Training and Supervising Analyst of the Italian Psychoanalytical Society and
former President of Centro Milanese di Psicoanalisi and Secretary of the Training Milanese
Institute. He is a medical doctor and a psychiatrist who worked for twenty years in psychiatric
hospitals.], 10/29/14, Is the concept of the death drive still useful in the clinical field?, The
International Journal of Psychoanalysis, 96:445458, doi: 10.1111/1745-8315.12308, Pages
546-547, MX)
the concept of
death instinct does not find a convincing explanation in the analytic clinical practice. By
contrast, it is possible to consider destructiveness as a transformative potential inherent in
the human mind, when life conditions become objectively and subjectively impossible to
tolerate. In the sphere of mental phenomena, destructiveness underlies severe psychopathologies such as perversion, anorexic and borderline syndromes, drug addiction and psychoses. In the social
In this contribution I have attempted to distinguish aggression, which can assume the form of hate and violence, from destructiveness. The point of view suggested is that

and political field, destructiveness was responsible for the greatest tragedies of the last century, such as Nazism and the derivatives of ideological communism. In addressing the problems presented by the

While acknowledging
the role of infantile trauma in the generation of adult suffering, we find that in some
cases it is not always easy to establish evidence of repeated violent trauma in infancy. The
analytic treatment of some patients, we face considerable uncertainty when seeking to establish a link between infantile traumas and destructive pathologies.

common element favouring such pathology is in fact the mental absence of the parents their indifference to the childs emotional development. This emotional remoteness allows a child, already in early infancy,
to stand aloof from reality and to take refuge in (megalomanic, phantasy-related or sexualized) psychic withdrawal involving incipient pathological identification with grandiose, destructive characters, who are felt

human destructiveness does not derive from


an original availability (the death instinct) but needs a long and complex development
and is sometimes prompted by environmental destructiveness. In this regard, it seems essential to assume the existence of
an internal pathological organization that forms gradually and tends to conquer the mind. The transformative process favouring the
construction of an inhuman world takes place in a state of mind that is dissociated from
the rest of the personality. In this psychic retreat destructiveness gives rise to a mental excitation that makes evil pleasurable and irresistible. Whereas
aggression is exhausted once the aim of lowering tension has been accomplished,
destructiveness, sustained as it is by pleasure, tends to be self-perpetuating. The power acquired by the
to be providers of pleasure, so that the world of relationships is progressively abandoned. My idea is that

pathological destructive organization over the rest of the personality depends on the role of the superego which coincides with a perverse psychopathological structure that has an intimidatory, seductive power. In

pleasure does not depend only on the defence of the self


against the demands of external reality, or on drive discharge. In its production, the destructive mechanisms, directed against
my opinion (De Masi, 1999) the problem of pleasure is very complex:

the subjects own self or others, seem as efficacious as those involved in the survival of the individual through self-affirmation and satisfaction of the drives. Thus, human destructiveness is very strongly linked to
pleasure that gets satisfaction from evil. To achieve this goal, and for primal aggressiveness to be transformed into destructiveness, a long process is required in which the innate resources are combined with the
environmental causes so as to get to a perverse constellation that enhances the destructive act as evidence of power, superiority and omnipotence.

No Scapegoating
No Scapegoating - empirical study proves
Gollwitzer 4
(Mario Gollwitzer [PhD in Psychology from University of Trier], 4/7/4, Do Normative
Transgressions Affect Punitive Judgments? An Empirical Test of the Psychoanalytic Scapegoat
Hypothesis, Personality and Social Psychology Bulletin, Pages 1657-1659, MX)
The psychoanalytic scapegoat hypothesis implies (a) that individuals who are guilty of violating a law or norm tend to impose harsher punitive judgments on comparable wrongdoers than nonguilty individuals and

If
individuals were really funda mentally concerned with avoiding self-ascriptions of blame
by applying defensive attributions (Shaver, 1970, 1985), then it follows that transgressors should
impose more lenient punishment on comparable offenders, especially under
circumstances of similarity. Although the blame-avoidance hypothesis directly contradicts the implications of the scapegoat hypothesis, both effects might be present
(b) that transgressors punitiveness should be amplified by the subjectively perceived decision conflict. A new alter native hypothesis was derived from research on blame avoidance and empathy:

depending on situation specific or person-specific factors. If these factors are not controlled for, the paradoxical situation could occur that the two effects cancel each other out (Schmitt et al., 2003). Among the

: The scapegoat
effect is expected to occur among high authoritarians, whereas the leniency effect
should occur among low authoritarians. The data presented here largely support the
blame avoidance motivation account: Punitive judgment scores were higher among nontransgressors, regarding both vignette transgressions in immoral
most theoretically plausible of such person-specific factors is authoritarianism. This variable is expected to moderate the transgression-punitiveness relation. In other words

temptation situations and actually committed transgressions reported by par ticipants. This effect was consistent across all six criminal cases in both vignette and actual transgressions. Further more

compared with a baseline measure of punitive judgments, the present results suggest
that it is not a harshness bias on the side of the nontransgressors that causes this effect
but rather, a mildness bias on the side of the transgressors. Although this finding is a clear corroboration for the blame-avoidance
hypothesis, one could argue (a) that transgressors simply do not care about the norm they transgress and (b) that transgression decision and punitiveness share a common cause, that is, the moral

conflict scores
wereexcept for the moonlighter scenarioconsistently higher among transgressors than among
nontransgressors. The suggestion that transgressors mildness bias was simply due to their recklessness can therefore be ruled out. The sec ond explanation would result in the argument
reprehensibility of the deed. The first explanation implies that conflict scores do not differ between trans gressors and nontransgressors. This is, however, not the case, because

that the relation between transgression decision and punitive judgments is spurious, because they can both be traced back to the same cause, that is, the moral evaluation of the deed. The mediation effect of
moral evaluation was not significant in the present analyses: The effect sizes of transgression decisions on punitive judgments remained largely unaffected by entering moral evaluation scores as a covariate. This
finding contradicts the alternative hypothesis that moral evaluation was the common cause for transgression decisions as well as for punitive judgments. Besides investigating the main effect of transgression

we tested whether the subjectively experienced decision conflict in a dilemma


situa tion affects punitive judgments among transgressors. Although the size of this correlation was
consistently positive in sign across the six scenarios, it was not statistically deviate from 0. Thus, this particular finding cannot be used as an argument for or
against the scapegoat hypothesis. Taking authoritarianism as a possible moderator variable into account was
a further attempt to demonstrate scapegoating effects in a subpopulation where they
appear to be most likely to occur. The negative sign of the three-way interaction
regression coefficient shows that this was not the case. One might think of the possibility that this three-way interaction did not emerge
decision on punitiveness,

because authoritarianism and transgression decisions were con founded. This would be a reasonable interpretation because authoritarianism implies conventionalism, which means refraining from committing
something unlawful because it is unlawful (Adorno et al., 1950). Therefore, it could be that individuals high in authoritarianism principally do not give in to immoral temptations. Contrary to this speculation,
however, transgression decisions in each of the six vignettes were correlated only weakly with authoritarianism (.05 < r < .12, average r = .04). Thus, nontransgressors are not automatically high authoritarians.
Taken together, the present study presents both weak and strong results. The major aim was to test the psycho analytic scapegoat hypothesis against an alternative hypothesis based on a blame-avoidance

Concerning the effect of


transgressors decision conflict on punitive judgments, however, the results support
neither the scapegoat nor the blame-avoidance motivation hypothesis. One could think of other empirical accesses
motivation. Concerning the main effect of transgression on punitive judgments, the results strongly support the blame avoidance hypothesis.

to the effect of decision conflicts. For example, the subjective quality of experienced decision conflicts in tempting situations might have trait-like qualities: Individuals might consistently differ in the way they
perceive and solve moral decision conflicts and in the amount of unease connected to these decision conflicts. These inter individual differences could be assessed and tested as predictors for punitive judgments.
Second, moral conflicts may be better captured by assessing them on a more idiosyncratic level. Participants could be interviewed about temptation situations in which they recall having experienced a strong
decision conflict; subsequently, their punitive judgments could be assessed in an adaptively constructed criminal case vignette that resembles this particular temptation. Finally, a possible conflict-punitiveness
relation might be moderated by other vari ables than authoritarianism. Because fear of an Id breakthrough is considered the crucial motor of scapegoating by Freud (1923/1990), trait anxiety appears to be a

. It is remarkable that a prominent theoretical account such as the


psychoanalytic scapegoat hypothesis, which is discussed among principal penological theories in stan dard textbooks on legal studies (e.g., Gppinger, 1997;
Kaiser, 1996; Schwind, 2002), has never received the empirical attention that it might deserve . The present study was a first
suitable moderator variable

It seems that although the process of scapegoating and


its psycho dynamic roots, as conceptualized in psychoanalytic writings, is not testable
as such, its implications can be incorporated into contemporary accounts of
punitiveness and retributive justice. If these implications repeatedly fail to receive empirical backup, the scapegoat hypothesis
should be at least reformulated, or dismissed.
attempt to test specifically derived impli cations of the scapegoat hypothesis.

Vote Aff: Abandon Search


The gaze is resistant to understanding because Lacan wants it to be - vote
affirmative to abandon the search
Scott 15
(Maria Scott [PhD in French at Trinity College Dublin], 2015, Deciphering the Gaze in Lacan's
"Of the gaze as objet petit a"', DS Project, MX)
The four-fold
structure worked out here might be read as corresponding to Lacans description of the
subjects relation to the object a as one of envelopment-development-conjunction-disjunction. The quartet is a recurrent figure in Seminar XI, in
Each of my four readings of the gaze has been shown to correspond to a mode of the subjects relation to the object a ($ a) as formulated by Lacan.

which the four fundamental concepts of psychoanalysis, the four partial drives, the four elements composing the montage of the drive, and the four vicissitudes of the drive are
all invoked. Indeed, Lacan draws attention to the pattern by stating that it is curious that there are four vicissitudes as there are four elements of the drive. Furthermore, while
for Freud the drive combines active, passive, and reflexive modes, Lacan invents in Seminar XI a fourth mode, which contains the others, and which he claims involves a
movement of appeal toward the Other. Lacans seminars on the gaze mix and match the various versions of the object a in a way that seems designed to confuse the auditor-

the concept of the


gaze outlined in Seminar XI is resistant to understanding precisely because it obeys laws similar to
those governing the gaze itself. In other words, Lacans logic of the gaze may operate to captivate
us by means of its elusiveness; an elusiveness that may ultimately be maddening
enough to force us to renounce our search for it and symbolize our failure.
reader. Their logic would seem to mimic the visual anamorphosis that they take to emblematize the workings of the gaze. It is possible that

Alt Offense

Psycho Fails in IR
Psychoanalysis treats entire nations as single patient on a couch that can
be treated like an individual psyche - this approach dooms alt solvency
Rosen-Carole 10
(Adam Rosen-Carole [a Visiting Professor in the Philosophy Department at Bard College.],
2010, Menu Cards in Time of Famine: On Psychoanalysis and Politics, The Psychoanalytic
Quarterly, Volume LXXIX, Number 1, Pages 226-227, MX)
we might think here of the

Also,
innumerable discussions of Americas death drive as propelling the recent invasions in the Middle East, or of the ways in which the motivation
for the Persian Gulf Wars of the 1990s was a collective attempt to kick the Vietnam War Syndrome that is, to solidify a national sense of power and prominence in the recognitive regard of the international

psychoanalytic speculations concerning the psychodynamics of various


nations involved in the Cold War (here, of course, I have in mind Segals [1997] work), or of the collective racist
fantasies and paranoiac traits that organize various nation-statess domestic and foreign
policies.7 Here are some further examples from iek, who, as a result of his popularity, might be said to function as a barometer of incipient trends:
What is therefore at stake in ethnic tensions is always the possession of the national
Thing. We always impute to the other [ethnic group, race, nation, etc.] an excessive enjoyment: he wants to steal our enjoyment (by ruining our way of life) and/or he has access to some secret, perverse
communityor of the

enjoyment. [1993, pp. 202-203] Beneath the derision for the new Eastern European post- Communist states, it is easy to discern the contours of the wounded narcissism of the European great nations. [2004,

There is in fact something of a neurotic symptom in the Middle Eastern conflict


everyone recognizes the way to get rid of the obstacle, yet nonetheless, no one wants
to remove it, as if there is some kind of pathological libidinal profit gained by persisting
in the deadlock. [2004, p. 39, italics added] If there was ever a passionate attachment to the lost object, a refusal to come to terms with its loss, it is the Jewish attachment to their land and
p. 27, italics added]

Jerusalem . . . . When the Jews lost their land and elevated it into the mythical lost object, Jerusalem became much more than a piece of land . . . . It becomes the stand-in for . . . all that we miss in our earthly

Rather than explore collective subjects through analyses of their individual


members, this type of psychoanalytically inclined engagement with politics treats a
collective subject (a nation, a region, an ethnic group, etc.) as if it were simply amenable
to explanation, and perhaps even to intervention, in a manner identical to an individual
psyche in a therapeutic context.
lives. [2004, p. 41]

Lacan is a pretentious charlatan whose writings are illiterate falsehoods prefer Chomsky - hes actually alive and comprehensible
Wolters 13
(Eugene Wolter [the founder and editor of Critical-Theory.com; freelance writer; New School
University graduate], 2/28/13, Noam Chomsky Calls Jacques Lacan a Charlatan, Critical
Theory: Theory and Theorists, http://www.critical-theory.com/noam-chomsky-calls-jacqueslacan-a-charlatan/, MX)
When asked by
the interviewer if he thought the work of Slavoj Zizek had any relevence, Chomsky
started to rail against what he considers fancy words that mean nothing once they are
decoded. Zizek is an extreme example, I dont see anything in what hes saying, said
Noam Chomsky appeared in a podcast on the site Veterans Unplugged back in December 2012 to give his thoughts about American militarism.

Chomsky. Chomsky then took a swipe at Jascques Lacan, the French psychoanalyst.
Jacques Lacan I actually knew. I kind of liked him. We had meetings everyone once in a
while but quite frankly I thought he was a total charlatan, just posturing before the
television cameras the way many Paris intellectuals do. Why this is influential I havent
the slightest idea I dont see anything that should be influential You can listen to the full interview at Veterans
Unplugged, the convervation of Zizek and Lacan starts around 6:20. Its actually not the first time Chomsky has made this claim about Lacan and other postmodernists. An
alleged text from Chomsky has been circulating on usenet and the web since 1995 where
Chomsky calls Lacan an amusing and perfectly self-conscious charlatan. The interview seems to
bolster the evidence that the 1995 text in fact originated from Chomsky. In the text, Chomsky speaks of his discussions with Kristeva, Foucault, and others. Some of
the people in these cults (which is what they look like to me) Ive met: Foucault (we even have a several-hour discussion, which is in print, and
spent quite a few hours in very pleasant conversation, on real issues, and using language that was perfectly comprehensible he speaking French, me English); Lacan
(who I met several times and considered an amusing and perfectly self-conscious
charlatan, though his earlier work, pre-cult, was sensible and Ive discussed it in print); Kristeva (who I met only briefly during the period when she was a fervent Maoist);
and others. Many of them I havent met, because I am very remote from from these circles, by choice, preferring quite different and far broader ones the kinds where I give

Ive dipped into what they write out of


curiosity, but not very far, for reasons already mentioned: what I find is extremely
pretentious, but on examination, a lot of it is simply illiterate, based on extraordinary
misreading of texts that I know well (sometimes, that I have written), argument that is appalling in its
casual lack of elementary self-criticism, lots of statements that are trivial (though dressed up in
complicated verbiage) or false; and a good deal of plain gibberish . When I proceed as I do in other areas where I do not understand, I
talks, have interviews, take part in activities, write dozens of long letters every week, etc.

run into the problems mentioned in connection with (1) and (2) above. So thats who Im referring to, and why I dont proceed very far. I can list a lot more names if its not
obvious.

Psycho =/= Ptix


Lacanian theory pretends to be radical but falls silent on the question of
politics, its claims at revolution are undercut by a mythical explanation
that justifies the status quo
Robinson 5
(Andrew Robinson [Ph.D. in Political Theory at the University of Nottingham], 2005, The
Political Theory of Constitutive Lack: A Critique, Theory & Event Volume 8, Issue 1, The Johns
Hopkins University Press, 1092-311X, MX)
There is more than an accidental relationship between the mythical operation of the
concept of constitutive lack and Lacanians conservative and pragmatist politics. Myth
is a way of reducing thought to the present:
Lacanian theory can be very radical,
This
radicalism
never translates into political conclusions
It is as if there is a magical barrier between theory
and politics which insulates the latter from the former.
Lacanians have a radical theory oriented towards happiness, but politically,
their primary concern is security. As long as they are engaged in politically ineffectual
critique, Lacanians will denounce and criticize the social system, but once it comes to
practical problems, the order not to think becomes operative
The Lacanian gesture, however,
is instead to present the evil and then add a word such as always to it. In this way, a
present problem becomes eternal and social change becomes impossible
Lacanian theory operates as an alibi: it offers a little
bit of theoretical radicalism to inoculate the system against the threat posed by a lot of
politicized radicalism
yes, liberal democracy involves
violent exclusions, but what is this compared to the desert of the real outside it
the isolated signs which are included in the mythical gesture are thereby attached to extra-historical abstractions. On an analytical

level,

unscrupulously exposing the underlying relations and assumptions concealed beneath officially-sanctioned discourse.

, however,

: as shown above, a radical rejection of anti-crime rhetoric turns into an endorsement of

punishment, and a radical critique of neo-liberalism turns into a pragmatist endorsement of structural adjustment.

One should recall a remark once made by Wilhelm Reich: You plead for happiness in life, but security

means more to you (1974, 27).

. This magic barrier is the alibi function of myth. The short-circuit between

specific instances and high-level abstractions is politically consequential. A present evil can be denounced and overthrown if located in an analysis with a middle level.

. At the very most, such change cannot affect

the basic matrix posited by Lacanian theory, because this is assumed to operate above history. In this way,

(cf. Barthes, 2000, 41-2). In Laclau and Mouffes version, this takes the classic Barthesian form:

? The iekian version is

more complex: yes, there can be a revolution, but after the revolution, one must return to the pragmatic tasks of the present. A good example is provided in one of ieks texts. The author presents an excellent analysis of a Kafkaesque incident in the former
Yugoslavia where the state gives a soldier a direct, compulsory order to take a voluntary oath - in other words, attempts to compel consent. He then ruins the impact of this example by insisting that there is always such a moment of forced choice, and that one

There is a danger of a
stultifying conservatism arising from within Lacanian political theory
Stavrakakis
ims
that attempts to find causes and thereby to solve problems are always fantasmatic
It is not clear whether such hostility
applies to all instances of solution, or whether there is a difference between constitutive
lack and some kind of surplus lack arising from contingent conditions. Certainly,
Lacanians often revert to contingent, empirical explanations, even when these seem
contrary to their own theoretical assumptions
a Lacanian approach to an
instance of lack, such as environmental crisis, famine or political repression, carries a
large danger that a contingent phenomenon will be labelled as constitutive and thereby
placed beyond criticism
should not attempt to escape it lest one end up in psychosis or totalitarianism (1989, 165-6). The political function of Lacanian theory is to preclude critique by encoding the present as myth.

. Echoing the terrifying conservatism Deleuze suggests is

active in any reduction of history to negativity (1994, 53). The addition of an always to contemporary evils amounts to a pessimism of the will, or a repressive reduction of thought to the present.

, for instance, cla

(1999, 87), while

iek states that an object which is perceived as blocking something does nothing but materialize the already-operative constitutive lack (1992c, 89).

(e.g. Laclau and Mouffe, 1985, 131). In any case,

. For instance, the argument that, since existing food production is sufficient for the worlds population, the existence of famine is an intolerable indictment of the world trade system and global

power relations would be severely damaged by a Lacanian claim that an inclusive distribution system is an impossible totalitarian fantasy. Contingent explanations - for instance, that the current famine in southern Africa is a result of IMF demands that governments
sell food stocks - are in competition with the Lacanian mythical gesture of explaining shortages and conflicts by reference to a constitutive impossibility of completion. Even if Lacanians believe in surplus/contingent as well as constitutive lack, there are no standards
for distinguishing the two. How does one tell an expression of constitutive lack from an effect of a particular regime of power, or for that matter from an imagined, nonexistent bogeyman? Perhaps all instances fall into the former category anyway: if it is not possible

iek effectively
admits that no element in the world is Real per se, reducing his affirmation of the idea to
to know whether any specific impasse is an instance of constitutive lack or not, it is not possible to know that any of them are, and there is therefore no basis for claiming with any certainty that constitutive lack exists. (

a suggestion that its rejection would lead to liberal conclusions


Even if constitutive lack
exists, Lacanian theory runs a risk of misdiagnoses which have a neophobe or even
reactionary effect. To take an imagined example, a Lacanian living in France in 1788
would probably conclude that democracy is a utopian fantasmatic ideal and would settle
for a pragmatic reinterpretation of the ancin regime
The construction of
the relation between
colonizer and colonized in terms of constitutive antagonism
turns
colonialism into an expression of an unchangeable ontology and impedes the possibility
of anti-colonial rebellion
The pervasive negativity and cynicism of Lacanian theory
offers little basis for constructive activity. Instead of radical transformation, one is left
with a pragmatics of containment which involves a conservative de-problematization of
the worst aspects of the status quo.
the
political theory of constitutive lack does not hold together as an analytical project and
falls short of its radical claims as a theoretical and political one. It relies on central
concepts which are constructed through the operation of a mythical discourse
with the result that it is unable to offer sufficient openness to engage with complex
issues.
the
idea of constitutive lack turns Lacanian theory into
a plague of
fantasies.
[iek and Salecl, 1996, 41-2]. This suggests that he is prepared to affirm

whatever he must affirm to avoid a conclusion he has decided in advance to view as unacceptable - a far flight from his official image as a daredevil revealing repressed truths).

. Laclau and Mouffes hostility to workers councils and ieks insistence on the need for a state and a Party

(e.g. Laclau and Mouffe, 1985, 178; iek, 2002b, 296-7; 1997a, 157) exemplify this neophobe tendency.

(for instance)

(see Laclau and Mouffe 1985, 129)

. It is also interesting that Newman begins his book with an intention to destroy the place of power, but concludes with the view that this is impossible. Instead of the anarchic desire to destroy

hierarchy, he demands that power merely be reinterpreted and displaced (2001, 37, 118-19).

The inactivity it counsels would make its claims a self-fulfilling prophecy by acting as a barrier to transformative activity. To conclude,

in the Barthesian sense,

If political theory is to make use of poststructuralist conceptions of contingency, it would do better to look to the examples provided by Deleuze and Guattari, whose conception of contingency is active and affirmative. In contrast,
something its most vocal proponent, iek, claims to attack:

Psycho Masquerades As Politics


Psychoanalysis masquerades as politics while turning the world into a
couch - offering hyperactive apolitical solutions to a vicious world that
needs marginal improvements just like the aff
Rosen-Carole 10
(Adam Rosen-Carole [a Visiting Professor in the Philosophy Department at Bard College.],
2010, Menu Cards in Time of Famine: On Psychoanalysis and Politics, The Psychoanalytic
Quarterly, Volume LXXIX, Number 1, Pages 230-231, MX)
the worry about psychoanalytically inclined political theory goes something like
this. At the level of form, the question is: can the diagnoses and proposals that proceed
from psychoanalytically inclined political theoryor any other form of political theory, for
that matterbe anything more than formal placeholders or standins for viable, vibrant
political life? Arent these diagnoses and proposals what becomes of the practice of
political judgment under conditions of large-scale alienation from politics, that is, under
conditions in which the diagnoses and demands articulated by psychoanalytically
informed political theory are wanting for the public institutions through which they could
be realized? And if the so-called interventions and proposals of psychoanalytically inclined
political theory sidestep the question of the institutional transformations necessary for
their realization, arent they conspiring with our blindness to the enormous institutional
impediments to a progressive political future? Doesnt psychoanalytic political theory
and I would say contemporary political theory in generalbear the burden of political
alienation? And might this burden be evident in, and at least partially responsible for, (1)
its reactive tendency to turn all the world into a couch, and (2) its hyperactive
prescriptiveness? At the level of content, the question is: might not psychoanalytically inclined political theory,
precisely in homing in on the unconscious dynamics of political investment, ideological
captivation, and so forth, indicate precisely the apolitical dynamics of what passes for
politics? If the value of psychoanalytic political theory is in its disclosure of motivations
that are self-secluding, self-distorting, or otherwise constitutively opaque and so
unshareable, then isnt the allure of a psychoanalytic interpretation of ostensibly political
phenomena an indication that politics has fallen under eclipse; and so, in the name of politics, might there
be reason to be wary about the whole enterprise of psychoanalytically inclined political
theory? Finally, then, the worry is that frustrations with the limited viability of the habits and forms of life cultivated via psychoanalytic practice (discussed in the first part of this paper) may feed the
By way of conclusion,

temptation to interpretive intervention (discussed in the second), and so menu cards in time of famine provoke the offering of meals, i.e., political proposals, that cant be delivered. If so, again, what I want to

psychoanalytic theory and practice, though not to be belittled, cannot be


thought apart from the viciousness of the world in which its marginal contributions and
interventions are necessary.
insist on is that the value of

K = Repression
The negatives claim that we must accept posited conditions founded on
transgression eliminates any possibility for incremental or revolutionary
reform and creates conflict by promoting antagonism as inescapable
Robinson 5
(SCHOOL OF POLITICS NOTTINGHAM, 2K5 [ANDREW, THEORY & EVENT, 8:1, The
Political Theory of Constitutive Lack: A Critique)
The basic claim of Lacanian theory is that identity - whether individual or social - is founded on a lack.
Therefore, social relations are always irreducibly concerned with antagonism, conflict, strife and
exclusion. Chantal Mouffe, for instance, writes of 'the primary reality of strife in social life'4, while Slavoj iek seeks an 'ethics grounded in reference to the traumatic Real
which resists symbolization'5. 'Lack ("castration") is original; enjoyment constitutes itself as "stolen"'6. According to Stavrakakis, the Real is 'inherent in human experience' and
'doesn't stop not being written'7. Hence, the primary element of social life is a negativity, which prevents the emergence of any social "whole". In Mouffe's words, 'society is the
illusion... that hides the struggle and antagonism behind the scenes', putting the 'harsh reality' of antagonism behind a 'protective veil'. For Newman, 'war is the reality', whereas
'society is the illusion... that hides the struggle and antagonism behind the scenes'9. For Stavrakakis, 'personal trauma, social crisis and political rupture are constant

Such claims have political consequences, because they rule out the
possibility of achieving substantial improvements (whether "reformist" or
"revolutionary") in any area on which this fundamental negativity bears. The dimension of antagonism is, after all,
'ineradicable'. Instead of the imperative to overcome antagonism which one finds in forms as diverse as Marxian revolution and deliberative democracy, Lacanian
characteristics of human experience'10.

political theory posits as the central political imperative a demand that one "accept" the underlying lack and the constitutive character of antagonism. While the various authors

Lacanian theory thus entails an ethical commitment


to create conflict and antagonism. This ethics mostly expresses itself via a detour into ontology: the ethical imperative is to 'accept' or 'grasp'
disagree about the means of achieving this, they agree on its desirability.

the truth of the primacy of lack, and the accusation against opponents is that they fall into some kind of fallacy (illusion, delusion, blindness, failure to accept, and so on). At
other times, however,

one finds a direct ethical advocacy of exclusion and conflict as almost goods in themselves.

Alt Offense
Turn: Imperialism of the analyst. Their psyschoanalytic reading of the
social world is unethical. Their alternative and method presumes you can
master the interpretation of any social system.
Dean 2
(Tim DEAN English and the Center for the Study of Psychoanalysis and Culture @ SUNY
Buffalo 2 Zizek and the Ethics of Psychoanalytic Criticism diacritics 32.2 p. 21-41 MUSE)
a problem that is exemplified by, but not restricted to, Slavoj iek's work: the tendency to treat
aesthetic artifacts as symptoms of the culture in which they were produced . Whether or not one employs the
vocabulary and methods of psychoanalysis to do so, this approach to aesthetics has become so widespread in the humanities that it qualifies as a
contemporary critical norm. As a norm, it may be subject to debate and even contestation. Today it is normative to read literature, film, and other
This paper tackles

cultural texts primarily as evidence about the societies that made themevidence that necessarily requires our hermeneutic labor in order to yield its significance. This
methodological protocol remains in place whether one inhabits critical perspectives as ostensibly disparate as historicist, materialist, or psychoanalytic modes of thinking; it is
also a grounding assumption of cultural studies, irrespective of how one defines that critical practice. Indeed, the issue I want to address is quite as much a Marxist problem as it
is a psychoanalytic one, and therefore the way in which iek articulates Lacan with Marx makes his work especially fertile terrain on which to engage this matter. As iek
reminds us in The Sublime Object of Ideology, Lacan claimed that Marx invented the symptom [SO 11]an observation that iek has been keen to exploit from the very
beginning of his work. iek's combining psychoanalysis with Marxism in novel ways has helped make Lacan more palatable to contemporary critical sensibilities by politicizing
psychoanalysis, demonstrating how it offers less an account of the individual than of society and culture. In iek's hands psychoanalytic theory appears less vulnerable to the
standard criticisms that it is ahistorical and apolitical. While a number of critiques of iek have reiterated these common objections, nevertheless his politicizing of
psychoanalysis has been particularly important during a period that witnessed the rise of new historicism, the institutionalization of cultural studies, and the escalating
importance of "the political" as a signperhaps the signof humanities professors' seriousness.1 iek's work has gone a long way toward making Lacan seem indispensable
to cultural studies, just as Juliet Mitchell's and Jacqueline Rose's work a decade earlier made Lacan seem indispensable to theoretically rigorous feminism. At a moment when
the poststructuralist variant of Lacanian theory was being displaced by historicist modes of thought, iek emerged on the scene to revivify psychoanalysis and make it exciting
again. Thus his work's appeal has an historical basis quite apart from iek's own personal charisma and his remarkable productivity. It is his politicizing of psychoanalysis, as

his style of politicizing


psychoanalysis carries a significant ethical cost, one that follows partly as a consequence of iek's failure to work through his
much as his [End Page 21] rendering Lacan newly accessible, that has made iek popular. I want to argue, however, that

theoretical relation to Althusser, from whom he derives the practice of symptomatic reading while claiming to displace the latter's version of psychoanalytic Marxism. Thus I shall
be arguing for a significant distinction between a political and an ethical psychoanalysis, suggesting that we have been cultivating the former at the expense of the latter.
Spaghetti Psychoanalysis The notion of the symptom is central to iek's thinking about politics and culture. Although in his work and in psychoanalytic theory more generally
the term symptom carries a range of conceptual meanings, symptomatology remains the governing trope of iek's oeuvre. Following Lacan, who continued to modify the
concept of the symptom throughout his career, iek argues that just about anything can be understood as symptomatic: [I]n the final years of Lacan's teaching we find a kind
of universalization of the symptom: almost everything that is becomes in a way symptom, so that finally even woman is determined as the symptom of man. We can even say
that "symptom" is Lacan's final answer to the eternal philosophical question "Why is there something instead of nothing?"this "something" which "is" instead of nothing is

If, for reasons to be elaborated, virtually anything can be considered a symptom, then this
conceptual move illuminates how iek can write about everything, how he seems able
to render all cultural phenomena as grist to his theoretical mill. Having grasped the
structural logic of the symptom, one may submit practically anything of interest to its
explanatory grid. And while iek expounds more than merely one logic of symptom formation, his structural logicslike his many bookstend to remain
indeed the symptom. [SO 71-72]

variations on a single theme. If, according to Lacan at the end of his career, the symptom has become a condition of subjective existence rather than a contingent problem, then

while this universalizing of


symptomatology sidelines the question of cure, it does nothing to diminish the
psychoanalytic zeal for diagnosis and interpretation. Instead, the opposite is true: universalizing the symptom
fuels the motive for diagnosis and interpretation, since symptoms are no longer localized and self-evident
but lurking everywhere. A hermeneutic operation becomes necessary before we can see how, for example, woman is the symptom of man.2 By shifting
there can be no possibility of curing symptoms in the manner that Freud envisioned when he invented psychoanalysis. Yet

symptoms from the category of the exception to that of the rule, iek to some extent depathologizes the symptom, converting it into a subjective norm. But to the degree that

his method requires a diagnostic stance (insofar as it encourages an intensified hermeneutic vigilance vis--vis the cultural field),
iek's symptomatology raises questions about the ethics of diagnosis. While Lacan's universalizing of
the symptom provokes fundamental [End Page 22] epistemological questions too, my primary concern lies with the ethical implications of a critical approach that regards the

Reading one's world in terms of symptoms positions one as a


with a particular relation to the worlda relation of suspicion and putative mastery.
Although iek repeatedly points out that one can never master one's "own" symptom
(but only enjoy it), his method nonetheless situates the critic in a position of hermeneutic
mastery over the social and cultural symptoms he or she diagnoses. One cannot help noticing that in his
dozen or so books no cultural artifact poses any resistance to iek's hermeneutic energy; there is
no social system or movie or opera or novel that he cannot interpret. We might say that there seems to be no
cultural phenomenon that, with his Lacanian schema, iek cannot master.3 iek's hermeneutic voracitywhat
universe as perpetually in need of interpretation.
hermeneut

could be understood as
psychoanalysis's imperialism, its tendency to find exemplifications of its
principles everywhere it turns [see Derrida; Meltzer]. I would suggest, however, that viewing cultural phenomena through the lens of
symptomatology points to a larger problem, one that pertains to not only psychoanalytic criticism but also
Marxism, historicism, and cultural studies. The problem lies in the way that treating aesthetic artifacts as cultural symptoms elides the specificity of art, making cultural
Tom Cohen characterizes as his approaching "the vast samples of American popular culture with vampirelike urgency" [356]
but one more instance of

forms too readily apprehensible as what iek, in one definition of the symptom, calls "the point of emergence of the truth about social relations" [SO 26]. Of course, the
category of artand, more broadly, that of aesthetic experiencedoes not appear in iek's work; speaking of "art" when discussing post-Lacanian ideology critique may
appear as quaintly anachronistic. But that is exactly my point. Despite his interest in Kantian philosophy and his evocation of the sublime, iek's approach to culture and society
leaves little conceptual space for any consideration of aesthetic effects or their significance.4

Alt Fails: Politics


Sweeping psychological generalizations have no explanatory power for
politics. They represent the worst of non-falsifiable hindsight thinking.
Samuels 93
(Andrew, Training Analyst Society of Analytical Psychology and Science Associate American
Academy of Psychoanalysis, Free Associtions, The mirror and the hammer: depth psychology
and political transformation, Vol. 3D, Psychoanalytic Electronic Publishing)
It is a contribution to the longstanding
ambition of depth psychology to develop a form of political and cultural analysis that will, in Freud's
words, understand the riddles of the world . It has to be admitted that there is an equally longstanding reluctance in the non-psychological
The paper is about the depth psychology of political processes, focusing on processes of political change.

community to accept the many and varied ideas and suggestions concerning political matters that have been offered by analysts of all persuasions. I do not believe this can all

There is something offensive above reductive interpretations of complex


socio-political problems in exclusively psychological terms. The tendency to
panpsychism on the part of some depth psychologists has led me to wonder if an
adequate methodology and ethos actually exists with which to make an engagement of depth psychology with the public sphere possible.
By politics I mean the arrangements within a culture for the organization and
distribution of power, especially economic power, and the way in which power is deployed to maintain the survival and enhance
be put down to resistance.

the quality of human life. Economic and political power includes control of processes of information and representation as well as the use of physical force and possession of
vital resources such as land, food and water. On a more personal level, political power reflects the ability to choose freely whether to act and what action to take in a given
situation. Politics refers to the interplay between the personal and public dimensions of power. That is, there is an articulation between public, economic power and power as
expressed on the personal, private level. This articulation is demonstrated in family organization, gender and race relations, and in religious and artistic assumptions as they
affect the life of individuals. (I have also tried to be consistent in my use of the terms culture, society and collective.)1 Here is an example of the difficulty with psychological

a distinguished psychoanalyst referred to the


revolutionary students in Paris in 1968 as functioning as a regressive group. Now, for a large group
of students to be said to regress, there must be, in the speaker's mind, some sort of normative developmental starting point for them to regress to . The social
group is supposed to have a babyhood, as it were. Similarly, the speaker must have had in mind the possibility of a healthier, progressive
group process what a more mature group of revolutionary students would have looked like. But complex social and political
phenomena do not conform to the individualistic, chronological, moralistic,
pathologizing framework that is often imported. The problem stems from treating the
entire culture, or large chunks of it, as if it were an individual or, worse, as if it were a baby.
Psychoanalysts project a version of personality development couched in judgemental
terms onto a collective cultural and political process. If we look in this manner for
pathology in the culture, we will surely find it. As we are looking with a psychological
theory in mind, then, lo and behold, the theory will explain the pathology. But this is a
reductionism to which I am referring. At a conference I attended in London in 1990,

retrospective prophecy (to use a phrase of Freud's), twentytwenty hindsight. In this psychoanalytic tautologizing
there is really nothing much to get excited about. Too much psychological writing on the
culture, my own included, has suffered from this kind of smug correctness when the material
proves the theoretical point. Of course it does! If we are interested in envy or greed, then we will find envy or greed in capitalistic organization. If
we set out to demonstrate the presence of archetypal patterns, such as projection of the shadow, in geopolitical
relations, then, without a doubt, they will seem to leap out at us. We influence what we analyse and so psychological
reflection on culture and politics needs to be muted; there is not so much aha! as one
hoped.

Alt Fails/Threats Real


Threats arent psychological projections and the alt fails
Hoffman 86
(Stanley Hoffman 86, Center for European Studies at Harvard, On the Political Psychology of
Peace and War: A Critique and an Agenda, Political Psychology 7.1 JSTOR)
it is unrealistic to expect
statesmen to stand above the fray: By definition, the statesmen are there to worry not only about planetary survival, but
first of allabout national survival and safety. To be sure, they ought to be able to see how certain policies, aimed at enhancing security, actually increase in-security all around. But
there are sharp limits to how far they can go in their mutual empathy or in their acts (unlike intellectuals in their advice), as long as the states'
The traditionalists, even when, in their own work, they try scrupulous-ly to transcend national prejudices and to seek scientific truth, believe that

antagonisms persist, as long as uncertainty about each other's intentions prevails, and as long as there is reason to fear that one side's wise restraint, or unilateral moves toward "sanity," will be met, not by the
rival's similar restraint or moves, but either by swift or skillful political or military exploitation of the opportunity created for unilateral gain, or by a for-midable domestic backlash if national self-restraint appears to

the
radicals' stance condemnation from the top of Mount Olympus can only impede understanding of the limits and possibilities of reform. To be sure, the
fragmentation of mankind is a formidable obstacle to the solution of many problems that cannot be handled well in a national framework, and a deadly peril insofar as the use of force, the very
distinctive feature of world politics, now entails the risk of nuclear war. But one can hardly call
anachronistic a phenomenonthe assertion of national identity that, to the bulk of [HU]mankind, appears
not only as a necessity but also as a positive good, since humanity's fragmentation results
from the very aspiration to self-determination. Many people have only recently emerged from foreign mastery, and have reason to fear that the
result in ex-ternal losses, humiliations or perceptions of weakness. There is little point in saying that the state of affairs which imposes such limits is "anachronistic" or "unrational." To traditionalists,

alternative to national self-mastery is not a world government of assured fairness and efficiency, but alien domination. As for "unrationality," the drama lies in the contrast between the ra-tionality of the whole,
which scholars are concerned aboutthe greatest good of the greatest number, in utilitarian terms and the rationality or greatest good of the part, which is what statesmen worry about and are responsible for.
What the radicals denounce as irrational and irresponsible from the viewpoint of mankind is what Weber called the statesman's ethic of responsibility. What keeps ordinary "competitive conflict processes"
(Deutsch, 1983) the very stuff of society from becoming "unrational" or destructive, isprecisely what the nature of world politics excludes: the restraint of the partners either because of the ties of affection or
responsibility that mitigate the conflict, or because of the existence of an outsider marriage counselor, arbitrator, judge, policeman or legislator capable of inducing or imposing restraints. Here we come to a

The very absence of such safeguards of rationality, the obvious discrepancy between what each part intends, and what it (and
the whole world) ends with, the crudeness of some of the psychological mechanisms at work in international affairsas one can see from the
statements of leaders, or from the media, or from inflamed publics have led many radicals, especially among those whose training or
profession is in psychoanalysis or mental health, to treat the age-old contests of states in terms, not of the psychology of politics, but of
third point of difference.

individual psychology and pathology. There are two manifestations of this. One is the tendency to
look at nations or states as individuals writ large, stuck at an early stage of development (similarly, John
Mack (1985) in a recent paper talks of political ideologies as carrying "forward the dichotomized structures of childhood"). One of my predecessors writes about "the correspondence between
development of the individual self and that of the group or nation," and concludes "that intergroup or international conflict contains the basic elements of the conflict each individual experiences psychologically"

The
second related aspect is the tendency to look at the notions statesmen or publics have of "the enemy," not only as
residues of childhood or adolescent phases of development, but as images that express "disavowed aspects of
the self" (Stein, 1985), reveal truths about our own fears and hatreds, and amount to masks we put on
the "enemy," because of our own psychological needs. Here is where the clash between traditionalists and radicals is strongest.
(Volkan, 1985). Robert Holt, from the viewpoint of cognitive psychology, finds "the largest part of the American public" immature, in a "phase of development below the Conscientious" (Holt, 1984).

Traditionalists do not accept a view of group life derived from the study of individual development or family relations, or a view of modern society derived from the simplistic Freudian model of regressed followers
identifying with a leader. They don't see in ideologies just irra-tional constructs, but often rationally selected maps allowing individuals to cope with reality. They don't see national identification as pathological, as
an appeal to the people's baser instincts, more aggressive impulses or un-sophisticated mental defenses; it is, as Jean-Jacques Rousseau so well understood, the competition of sovereign states that frequently
pushes people from "sane" patriotism to "insane" nationalism (Rousseau's way of preventing the former from veering into the latter was, to say the least, im-practical: to remain poor in isolation). Nor do they see

the "intra-psychic" approach


distorts reality. Enemies are not mere projections of negative identities; they are often
anything "primitive" in the nation's concern for survival: It is a moral and structural requirement. Traditionalists also believe that

quite real . To be sure, the Nazis' view of the Jews fits the metaphor of the mask put on the enemy for one's own needs. But were, in return, those Jews who understood what enemies they had in
the Nazis, doing the same? Is the Soviet domination of Eastern Europe, is the Soviet regime's treatment of dissidents, was the Gulag merely a convenient projection of
our intrapsychic battles? Clichs such as the one about how our enemy "understands
only force" may tell us a great deal about ourselves; but sometimes they contain half-truths about him, and not
just revelations about us. Our fears flow not only from our private fantasies but also from concrete

realities and from the fantasies which the international state of nature generates. In other words, the psychology of politics which traditionalists deem adequate is
not derived from theories of psychic development and health; it is derived from the logic of the
international milieu, which breeds the kind of vocabulary found in the historians and theorists of the
state of nature: fear and power, pride and honor, survival and security, self-interest and reputation, distrust and misunderstanding, commitment and credibility. It is also derived from the social
psychology of small or large groups, which resorts to the standard psychological vocabulary that describes mental mechanisms or maneuvers and cognitive processes: denial, projection, guilt,
repression, closure, rigidity, etc.... But using this vocabulary does not imply that a group whose style of politics
is paranoid is therefore composed of people who, as private individuals, are paranoid. Nor does it
relieve us of the duty to look at the objective reasons and functions of these mental
moves, and of the duty to make explicit our assumptions about what constitutes a "healthy,"
wise, or proper social process. Altogether, traditionalists find the mental health approach to world affairs unhelpful. Decisions about war and
peace are usually taken by small groups of people; the temptation of analyzing their behavior either, literal-ly, in terms of their
personalities, or, metaphysically, in terms borrowed from the study of human development, rather than in those of group dynamics or principles of international politics is
understandable. But it is misleading. What is pathological in couples, or in a well-ordered community, is, alas, frequent, indeed normal, among states, or in a
troubled state. What is malignant or crazy is usually not the actors or the social process in which they are engaged: it is the possible results. The grammar of motives which the mental health approach brands as
primitive or immature is actually rational for the actors. to the substitution of labels for explanations, to bad analysis and fanciful prescriptions. Bad analysis: the tendency to see in group coherence a regressive
response to a threat, whereas it often is a rational response to the "existential" threats entailed by the very nature of the international milieu. Or the tendency to see in the effacement or minimization of individual
differences in a group a release of unconscious instincts, rather than a phenomenon that can be perfectly adaptivein response to stress or threatsor result from governmental manipulation or originate in the
code of conduct inculcated by the educational system, etc.. . The habit of comparing the state, or modern society, with the Church or the army, and to analyze human relations in these institutions in ways that
stress the libidinal more than the cognitive and superego factors, or equate libidinal bonds and the desire for a leader. The view that enemies are above all products of mental drives, rather than inevitable
concomitants of social strife at every level. Or the view that the contest with the rival fulfills inter-nal needs, which may be true, but requires careful examination of the nature of these needs (psychological?

The
radicals are so (justifiably) concerned with the nuclear peril that the traditional ways in which
statesmen and publics behave seem to vindicate the pathological approach. But this, in turn, incites
radicals to overlook the fundamental ambiguity of contemporary world politics. On the one hand, there
bureaucratic? economic?), obscures the objective reasons of the contest, and risks confusing cause and function. Indeed, such analysis is particularly misleading in dealing with the pre-sent scene.

is a nuclear revolutionthe capacity for total destruction. On the other hand, many states, without nuclear weapons, find that the use of force remains rational (in terms of a rationality of means) and beneficial at
home or abroadask the Vietnamese, or the Egyptians after October 1973, or Mrs. Thatcher after the Falklands, or Ronald Reagan after Grenada. The superpowers themselves, whose contest has not been
abolished by the nuclear revolution (it is the stakes, the costs of failure that have, of course, been transformed), find that much of their rivalry can be conducted in traditional ways including limited uses of force
below the level of nuclear alarm. They also find that nuclear weapons, whileperhapsunusable rationally, can usefully strengthen the very process that has been so faulty in the prenuclear ages: deterrence
(this is one of the reasons for nuclear proliferation). The pathological approach interprets deterrence as expressing the deterrer's belief that his country is good, the enemy's is bad. This is often the case, but it
need not be; it can also reflect the conviction that one's country has interests that are not mere figments of the imagination, and need to be protected both because of the material costs of losing them,

As for war planning, it is not a case of "psychological denial of


unwelcome reality" (Montville, 1985). but a perhaps futile, perhaps dangerousnecessity in a
world where deterrence may once more fail. The prescriptions that result from the radicals' psychological approach also run into traditionalist
objections. Even if one accepts the metaphors of collective disease or pathology, one must understand that the "cure" can only be
and because of the values embedded in them.

provided by politics . All too often, the radicals' cures consist of perfectly sensible
recommendations for lowering tensions, but fail to tell us how to get them carried out they only tell us
how much better the world would be, if only "such rules could be established" (Deutsch, 1983).
Sometimes, they express generous aspirations for common or mutual securitywithout much awareness of the obstacles which conflict-ing interests, fears about allies or clients, and the nature of the weapons
themselves, continue to erect. Sometimes, they too neglect the ambiguity of life in a nuclear world: The much lamented redundancy of weapons, a calamity if nuclear deterrence fails, can also be a cushion
against failure. Finally, many of the remedies offered are based on an admirable liberal model of personality and politics: the ideal of the mature, well-adjusted, open-minded person (produced by liberal education
and healthy family relations) transposed on the political level, and thus accompanied by the triumph of democracy in the community, by the elimination of militarism and the spread of functional cooperation
abroad. But three obstacles remain unconquered: first, a major part of the world rejects this ideal and keeps itself closed to it (many of the radicals seem to deny it, or to ignore it, or to believe it doesn't matter).
Second, the record shows that real democracies, in their behavior toward non-democratic or less "advanced" societies, do not conform to the happy model (think of the US in Central America). Third, the task of
reform, both of the publics and of the statesmen, through consciousness raising and education is hopelessly huge, incapable of being pursued equally in all the important states, and indeed too slow if one
accepts the idea of a mortal nuclear peril. These, then, are the dimensions of a split that should not be minimized or denied

Alt Fails: Cant Explain


One thing can explain all action psychoanalysis fails
Mootz 2k
[Francis J, Visiting Professor of Law, Pennsylvania State University, Dickinson School of Law;
Professor of Law, Western New England College School of Law, Yale Journal of the Law &
Humanities, 12 Yale J.L. & Human. 299, p. 319-320]
Freudian psychoanalysis increasingly is the target of blistering criticism from a wide variety of commentators.
54 In a recent review, Frederick Crews reports that independent studies have begun to converge toward a verdict...
that

there is literally nothing to be said, scientifically or therapeutically , to the advantage

of the entire Freudian system or any of its component dogmas Analysis as a whole
remains powerless ... and understandably so, because a thoroughgoing epistemological
critique, based on commonly acknowledged standards of evidence and logic decertifies every
distinctively psychoanalytic proposition. 55 The most telling criticism of Freud's psychoanalytic theory is
that it has proven no more effective in producing therapeutic benefits than have other forms of psychotherapy. 56
Critics draw the obvious conclusion that the benefits (if any) of psychotherapy are neither explained nor facilitated
by psychoanalytic theories. Although Freudian psychoanalytic theory purports to provide a truthful account of the

psychoanalytic theory
may prove in the end to be nothing more than fancy verbiage that tends to obscure
whatever healing effects psychotherapeutic dialogue may have. 57 Freudian psychoanalysis
failed because it could not make good on its claim to be a rigorous and empirical
science. Although Freud's mystique is premised on a widespread belief that psychoanalysis was a profound
operations of the psyche and the causes for mental disturbances, critics argue that

innovation made possible by his genius, Freud claimed only that he was extending the scientific research of his day

Freud's adherents created


the embarrassing cult of personality and the myth of a self-validating psychoanalytic
method only after Freud's empirical claims could not withstand critical scrutiny in
accordance with the scientific methodology demanded by his metapsychology. 59 The
record is clear that Freud believed that psychoanalysis would take its place among the
sciences and that his clinical work provided empirical confirmation of his theories. This
belief now appears to be completely unfounded and indefensible. Freud's quest for a
within the organizing context of a biological model of the human mind. 58 [*320]

scientifically grounded psychotherapy was not amateurish or naive. Although Freud viewed his "metapsychology as
a set of directives for constructing a scientific psychology," n60 Patricia Kitcher makes a persuasive case that he
was not a blind dogmatist who refused to adjust his metapsychology in the face of contradictory evidence. n61
Freud's commitment to the scientific method, coupled with his creative vision, led him to construct a comprehensive
and integrative metapsychology that drew from a number of scientific disciplines in an impressive and persuasive

the natural and social sciences upon which he built his derivative and
interdisciplinary approach developed too rapidly and unpredictably for him to respond .
n63 As developments in biology quickly undermined Freud's theory, he "began to look to
linguistics and especially to anthropology as more hopeful sources of support," n64 but this strategy
later in his career proved equally [*321] unsuccessful. n65 The scientific justification claimed
by Freud literally eroded when the knowledge base underlying his theory collapsed ,
leaving his disciples with the impossible task of defending a theory whose
presuppositions no longer were plausible according to their own criteria of validation. n66
manner. n62 However,

Environment K Answers

Alt Fails
Advocacy for specific policy reform is key to environmental justice
movements---refusal of policy relevance ensures marginalization
Noonan 5
(Douglas S. Noonan 5, Assistant Professor, School of Public Policy, Georgia Institute of
Technology, 2005, DEFINING ENVIRONMENTAL JUSTICE: POLICY DESIGN LESSONS
FROM THE PRACTICE OF EJ RESEARCH,
http://www.prism.gatech.edu/~dn56/EJ.APPAM.pdf)
The negotiated nature of environmental policymaking holds some stark lessons for policymakers and
analysts alike. Even if there were no uncertainty and all of the useful scientific evidence was available the heterogeneous
interests of affected parties would persist. When policies ultimately seek to reconcile these
competing interests, essentially answering questions of social choice (for which optimal solutions may not be available either in theory or due to practical
limits to policy), only rarely or never would a policy process be such that selfish advocacy by interest
groups yields both individually and socially optimal outcomes. In the environmental policy arena, the
disconnect between the pursuit of individual interests and the pursuit of collective goals
is paramount. In this sense, the acrimony surrounding many environmental policy debates is both undersandable and inevitable. Although this preface might
apply equally well to discussions of climate change policy or species/wilderness preservation policy, the application to environmental
justice (EJ) provides an opportune arena in which to observe the interplay between
environmental policymaking and the (allegedly) relevant research. Environmental justice is a major
theme in environmental and social policy. Its researchers are legion. Their output is
voluminous. A debate about the empirical evidence and about appropriate policies
continues among academics. In more public forums, interest groups routinely cite
environmental justice in advocating for policy reforms. As is typical in policy debates,
advocates select evidence to cite in support of their position. The influence of scholarly
EJ research on policymakers , however, is less than straightforward . If the mounting evidence
provides only partial answers or, as is common, answers to questions only marginally relevant
to policymakers , then even hundreds of books 1 on the subject may do little to sway
public policy . Or, conversely, the evidences influence may far outstrip its limited relevance. Regardless, like many other environmental policy topics, the role of
The purpose of this paper is to offer some insight
about policy design from the scholarly literature on EJ. After scaling this mountain of literature, what are the
important lessons to be learned for making EJ policy? From this vantage, this paper critiques the
scholarly research in policy design is inevitably contentious and complex.

field of EJ research . It also offers some suggestions for a more policy-relevant research agenda. The conclusion returns to the broad assessment of EJ
policy and suggests some future directions for designing policy and framing the discourse.

Environment impact is insulated from the alt


Minteer 5

(Ben, Arizona State biology professor, Environmental Philosophy and the Public Interest: A
Pragmatic Reconciliation, Environmental Values 14, ebsco)
This call for revisiting and rethinking the philosophical roots of Western culture, which for White were the
techno-scientific worldview and its underlying religious and secular foundations in the medieval period, implied nothing less than an overhaul of the tradition, a
foundation-razing process in which a new philosophy of science, technology, and nature and perhaps a new, less arrogant relationship to
the natural world would be unearthed and absorbed into the modern worldview. Early environmental philosophers such as Routley and Rolston, then,
apparently following White in their call for a new ethic able to account for the independent value of the natural world, assumed that the anthropocentric worldview (and its destructive instrumentalisation of nature)
had to be replaced with a new, nonanthropocentric outlook. Here, White s thesis about the anti-environmental implications of the Judeo-Christian religion, particularly his sweeping claim that the latter was the
most anthropocentric religion the world has seen , offered a point of departure for environmental philosophers, who would respond in subsequent years with a series of influential criticisms of the moral humanism

an exclusivist nonanthropocentric agenda established itself as the dominant approach in the field, with a few notable
exceptions (of the latter, see Norton 1984, 1991; Weston 1985, and Stone 1987). The result of these developments is that the public interest never became part of
the agenda of environmental philosophy in the same way, for example, that it appears to have made lasting impressions in other branches of applied
philosophy such as business, engineering, and biomedical ethics. Concerned with what it perceived to be more pressing and fundamental
questions of moral ontology that is, with the nature of environmental values and the moral standing of nonhuman nature environmental
philosophers pursued questions selfconsciously cordoned off from parallel discussions in mainstream moral and
political theory, which were apparently deemed too anthropocentric to inform a philosophical field preoccupied with the separate issue of the moral
considerability and significance of nonhuman nature. As a consequence, instead of (for example) providing a conceptual or analytic framework for
evaluating cases, practices, and policies from the perspective of ostensibly human-centred concepts such as the public interest, many environmental
philosophers preferred to focus exclusively on the independent status of natural values. I would argue that this original
failure to link environmental values and claims to recognised moral and political concerns also helps to explain
the relative inability of environmental philosophy to have a significant impact within public
and private institutions over the years, again, especially when compared with other applied ethics counterparts. Environmental
philosophy is and always has been concerned with nature s interest , not that of the public. This situation has also produced a number of unfortunate
consequences for the contribution of environmental philosophy to policy discussion and
debate, not to mention more concrete and on-the-ground forms of social action. One example here is the largely
missed opportunity for philosophers to study and contribute to some of the more important environmental reform
movements and institutional initiatives of the past three decades. Chief among these developments, perhaps, is the public interest
movement that developed alongside environmental ethics in the late 1960s and early 1970s, which united consumer protection with
environmental advocacy through organisations like Ralph Naders Public Interest Research Groups (PIRGs). This list of emerging direct-action environmental movements would
also have to include the growing number of grassroots organisations and groups, commonly lumped under the environmental justice banner, which
of the Western philosophical inheritance (e.g., Taylor 1986, Rolston 1988, Callicott 1989, Westra 1994, Katz 1996). As the field matured in the 1980s and 1990s,

have sought to link the concerns of public health, safety, and community well-being to environmental protection through the language and tactics of social justice and civil rights (Gottlieb 1993, Shutkin 2000,

Had environmental philosophy worked a serious notion of the public interest into its agenda, it
doubtless would have been (and would now be) much more engaged with these influential
movements in citizen environmental action, not to mention a range of discussions in areas
such as risk communication, pollution prevention and regulatory reform, public understanding of
science, and so on.
Shrader-Frechette 2002).

AT: Movements
Movements/the alternative cant address immediate problemsthey are
multigenerational, educational ventures. Working within the system is
necessary to solve particular instances of the climate problemtheres no
guarantee their revolution will solve
-Working within existing political institutions is key
-Its too late to solve the whole environmental crisis, but can work to mitigate the damage
-No guarantee the alternatives regression to socialism wont have same environmental
problems

PARENTI 13
(Christian, professor of sustainable development at the School for International Training,
Graduate Institute, A Radical Approach to the Climate Crisis, Dissent, Summer 2013,
http://www.dissentmagazine.org/article/a-radical-approach-to-the-climate-crisis)
Several strands of green thinking maintain that capitalism is incapable of a sustainable
relationship with non-human nature because, as an economic system, capitalism has a growth imperative while the earth is finite. One finds versions of
this argument in the literature of eco-socialism, deep ecology, eco-anarchism, and even among many mainstream greens who, though typically declining to actually name the
economic system, are fixated on the dangers of growth. All this may be true. Capitalism, a system in which privately owned firms must continuously out-produce and out-sell

However, that is not the same question as


whether capitalism can solve the more immediate climate crisis. Because of its
magnitude, the climate crisis can appear as the sum total of all environmental problems
deforestation, over-fishing, freshwater depletion, soil erosion, loss of biodiversity,
chemical contamination. But halting greenhouse gas emissions is a much more specific problem,
the most pressing subset of the larger apocalyptic panorama . And the very bad news is, time
has run out. As I write this, news arrives of an ice-free arctic summer by 2050. Scientists once assumed that would not happen for hundreds of years. Dealing
with climate change by first achieving radical social transformationbe it a socialist or
anarchist or deep-ecological/neo-primitive revolution, or a nostalgia-based localista conversion back to a mythical small-town capitalismwould be
a very long and drawn-out , maybe even multigenerational, struggle . It would be marked
by years of mass education and organizing of a scale and intensity not seen in most core
capitalist states since the 1960s or even the 1930s. Nor is there any guarantee that the new system
would not also degrade the soil, lay waste to the forests, despoil bodies of water, and
find itself still addicted to coal and oil. Look at the history of actually existing socialism
before its collapse in 1991. To put it mildly, the economy was not at peace with nature. Or consider the vexing
complexities facing the left social democracies of Latin America. Bolivia, and Ecuador, states run by
socialists who are beholden to very powerful, autonomous grassroots movements, are still very
dependent on petroleum revenue. A more radical approach to the crisis of climate change begins
not with a long-term vision of an alternate society but with an honest engagement with
the very compressed timeframe that current climate science implies. In the age of climate change,
these are the real parameters of politics .
their competitors, may be incapable of accommodating itself to the limits of the natural world.

State Good
Climate change mitigation and adaptation requires a revamped role of the
statetheir total insistence n overthrow of capitalism is a delusion
Christian PARENTI, professor of sustainable development at the School for International
Training, Graduate Institute, 13 [A Radical Approach to the Climate Crisis, Dissent, Summer
2013, http://www.dissentmagazine.org/article/a-radical-approach-to-the-climate-crisis]
Climate change is a problem of an entirely different order of magnitude, but these past
solutions to smaller environmental crises offer lessons. Ultimately, solving the climate crisislike
the nineteenth-century victory over urban squalor and epidemic contagionswill require a re-legitimation of the states role in
the economy. The modern story of local air pollution offers another example of the rebellion of nature. As Jim McNeil outlines in Something New Under The Sun,
smog inundations in industrial cities of the United States and Europe used to kill many people. In 18791880 smog killed 3,000 Londoners, and in Glasgow a 1909 inversion
where cold air filled with smoke from burning coal was trapped near the groundkilled 1,063. As late as 1952, a pattern of cold and still air killed 4,000 people in London,
according to McNeil, and even more according to others. By 1956, the Britons had passed a clean air act that drove coal out of the major cities. In the United States there was a
similar process. In 1953, smog in New York killed between 170 and 260 people, and as late as 1966 a smog inversion killed 169 New Yorkers. All of this helped generate
pressure for the Clean Air Act of 1970. Today, a similar process is underway in China. Local air quality is so bad that it is forcing changes to Chinese energy policy. A major
World Bank study has estimated that the combined health and non-health cost of outdoor air and water pollution for Chinas economy comes to around $US 100 billion a year
(or about 5.8% of the countrys GDP). People across China are protesting pollution. Foreign executives are turning down positions in Beijing because of the toxic atmospheric
stew that western visitors have taken to calling airpocalypse. The film director Chen Kaige, who won the Palme dOr for his 1993 film Farewell My Concubine, told the world he
couldnt think or make films because of the Chinese capitals appallingly bad air. These local pressures are a large part of what is driving Chinese investment in renewable
energy. Last year China added more energy capacity from wind than from the coal sector. Capitalism vs. Nature? Some of the first thinkers to note a conflict between capitalism
and non-human nature were Karl Marx and Friedrich Engels. They came to their ecology through examining the local problem of relations between town and country
expressed simultaneously as urban pollution and rural soil depletion. In exploring this question they relied on the pioneering work of soil chemist Justus von Liebig. And from this
small-scale problem, they developed the idea of capitalism creating a rift in the metabolism of natural processes. Here is how Marx explained the dilemma: Capitalist production
collects the population together in great centers, and causes the urban population to achieve an ever-growing preponderance. This has two results. On the one hand it
concentrates the historical motive force of society; on the other hand, it disturbs the metabolic interaction between man and the earth, i.e., it prevents the return to the soil of its
constituent elements consumed by man in the form of food and clothing; hence it hinders the operation of the eternal natural condition for the lasting fertility of the soil.All

And as with soil robbing, so


too concentrations of atmospheric CO2: the natural systems are out of sync; their
elements are being rearranged and redistributed, ending up as garbage and pollution . It
may well be true that capitalism is incapable of accommodating itself to the limits of the
natural world. But that is not the same question as whether or not capitalism can solve
the climate crisis. Climate mitigation and adaptation are merely an effort to buy time to
address the other larger set of problems that is the whole ecological crisis. This is both a
pessimistic and an optimistic view. Although capitalism has not overcome the fundamental
conflict between its infinite growth potential and the finite parameters of the planets
pollution sinks, it has , in the past, addressed specific environmental crises . Anyone who
thinks the existing economic system must be totally transformed before we can deal with
the impending climate crisis is delusional or in willful denial of the very clear findings of
climate science. If the climate system unravels/// , all bets are off. The many progressive visions
born of the Enlightenment will be swallowed and forgotten by the rising seas or
smashed to pieces by the wrathful storms of climate chaos.
progress in capitalist agriculture is a progress in the art, not only of robbing the worker, but of robbing the soil.

The political demands of climate change require reformist politicsthe


alternatives all or nothing attitude guarantees extinction. Perm is the best
way out

Parenti 13
(Christian, professor of sustainable development at the School for International Training,
Graduate InstituteA Radical Approach to the Climate Crisis, Dissent, Summer 2013,
http://www.dissentmagazine.org/article/a-radical-approach-to-the-climate-crisis)
scientific consensus, expressed in peer-reviewed and professionally vetted and published scientific literature, runs as
follows: For the last 650,000 years atmospheric levels of CO2the primary heat-trapping gashave hovered at around 280 parts per million (ppm). At no
point in the preindustrial era did CO2 concentrations go above 300 ppm. By 1959, they had reached
316 ppm and are now over 400 ppm. And the rate of emissions is accelerating. Since 2000, the
world has pumped almost 100 billion tons of carbon into the atmosphereabout a quarter of all CO2 emissions
since 1750. At current rates, CO2 levels will double by mid-century . Climate scientists believe
that any increase in average global temperatures beyond 2 degrees Celsius above preindustrial levels
will lead to dangerous climate change, causing large-scale desertification, crop failure,
inundation of coastal cities, mass migration to higher and cooler ground, widespread
extinctions of flora and fauna, proliferating disease, and possible social collapse. Furthermore, scientists now
understand that the earths climate system has not evolved in a smooth linear fashion. Paleoclimatology has uncovered evidence of
sudden shifts in the earths climate regimes. Ice ages have stopped and started not in a matter of centuries, but decades. Sea
levels (which are actually uneven across the globe) have risen and fallen more rapidly than was once believed.
Throughout the climate system, there exist dangerous positive-feedback loops and tipping points . A positivefeedback loop is a dynamic in which effects compound, accelerate, or amplify the original cause. Tipping points in the climate system reflect
the fact that causes can build up while effects lag. Then, when the effects kick in, they do so
all at once, causing the relatively sudden shift from one climate regime to another. Thus, the UNs
Intergovernmental Panel on Climate Change says rich countries like the United States must cut emissions 25 percent to
40 percent below 1990 levels by 2020only seven years awayand thereafter make precipitous cuts to 90
percent below 1990 levels by 2050. This would require global targets of 10 percent reductions
in emissions per annum, starting now. Those sorts of emissions reductions have only occurred during
Hard Facts The

economic depressions . Russias near total economic collapse in the early 1990s saw a 37 percent decrease in CO2 emissions from 1990 to 1995, under
conditions that nobody wants to experience. The political implications of all this are mind-bending. As daunting as it may sound, it
means that it is this society and these institutions that must cut emissions. That means, in the
short-term, realistic climate politics are reformist politics , even if they are conceived of
as part of a longer-term anti-capitalist project of totally economic re-organization.

Institutions Key
Their call to reject all existing institutions guarantees warming happens
Monbiot 8
(George, English Writer and Environmental and Political Activist, 9-4, Identity Politics in Climate
Change Hell, http://www.celsias.com/article/identity-politics-climate-change-hell/)
If you want a glimpse of how the movement against climate change could crumble faster than a summer snowflake, read Ewa Jasiewiczs article , published on the Guardians Comment is Free site. It is a fine

identity politics

example of the
that plagued direct action movements during the 1990s, and from which the new generation of activists has so far been mercifully free. Ewa rightly celebrates
the leaderless, autonomous model of organising that has made this movement so effective. The two climate camps I have attended this year and last were among the most inspiring events Ive ever
witnessed. I am awed by the people who organised them, who managed to create, under extraordinary pressure, safe, functioning, delightful spaces in which we could debate the issues and plan the actions
which thrust Heathrow and Kingsnorth into the public eye. Climate camp is a tribute to the anarchist politics that Jasiewicz supports. But in seeking to extrapolate from this experience to a wider social plan, she

claims to want to stop global warming, but she makes that


task 100 times harder by rejecting all state and corporate solutions. It seems to me that what she really wants to do is to
create an anarchist utopia, and use climate change as an excuse to engineer it. Stopping runaway climate change must take
precedence over every other aim. Everyone in this movement knows that there is very little time: the window of
opportunity in which we can prevent two degrees of warming is closing fast. We have to
use all the resources we can lay hands on, and these must include both governments and corporations.
makes two grave errors. The first is to confuse ends and means. She

Or perhaps she intends to build the installations required to turn the energy economy around - wind farms, wave machines, solar thermal plants in the Sahara, new grid connections and public transport systems herself? Her article is a terryifying example of the ability some people have to put politics first and facts second when confronting the greatest challenge humanity now faces. The facts are as follows.

Runaway climate change is bearing down on us fast. We require a massive political and
economic response to prevent it. Governments and corporations, whether we like it or
not, currently control both money and power. Unless we manage to mobilise them, we
stand a snowballs chance in climate hell of stopping the collapse of the biosphere. Jasiewicz
would ignore all these inconvenient truths because they conflict with her politics. Changing our sources of energy without changing our sources of economic and political power, she asserts, will not make a

before we are allowed to begin cutting greenhouse gas


emissions, we must first overthrow all political structures and replace them with
autonomous communities of happy campers. All this must take place within a couple of
months, as there is so little time in which we could prevent two degrees of warming. This
is magical thinking of the most desperate kind. If I were an executive of E.On or Exxon, I would
be delighted by this political posturing, as it provides a marvellous distraction from our
real aims. To support her argument, Jasiewicz misrepresents what I said at climate camp. She claims that I confessed not knowing where to turn next to solve the issues of how to generate the
difference. Neither coal nor nuclear are the solution, we need a revolution. So

changes necessary to shift our sources of energy, production and consumption. I confessed nothing of the kind. In my book Heat I spell out what is required to bring about a 90% cut in emissions by 2030.
Instead I confessed that I dont know how to solve the problem of capitalism without resorting to totalitarianism. The issue is that capitalism involves lending money at interest. If you lend at 5%, then one of two
things must happen. Either the money supply must increase by 5% or the velocity of circulation must increase by 5%. In either case, if this growth is not met by a concomitant increase in the supply of goods and
services, it becomes inflationary and the system collapses. But a perpetual increase in the supply of goods and services will eventually destroy the biosphere. So how do we stall this process? Even when usurers
were put to death and condemned to perpetual damnation, the practice couldnt be stamped out. Only the communist states managed it, through the extreme use of the state control Ewa professes to hate. I dont

Yes, let us fight both corporate power and the undemocratic


tendencies of the state. Yes, let us try to crack the problem of capitalism and then fight for a different system. But let us not confuse this
task with the immediate need to stop two degrees of warming, or allow it to interfere with
the carbon cuts that have to begin now. Ewas second grave error is to imagine that society could be turned into a giant climate camp. Anarchism is a great means of organising a self-elected
yet have an answer to this conundrum. Does she?

community of like-minded people. It is a disastrous means of organising a planet. Most anarchists envisage their system as the everyone is to be free from the coercive power of the state, this must apply to the
oppressors as well as the oppressed. The richest and most powerful communities on earth - be they geographical communities or communities of interest - will be as unrestrained by external forces as the poorest

if anarchist utopia arrives, the first


thing that will happen is that every Daily Mail reader in the country will pick up a gun and
go and kill the nearest hippy. This is why, though both sides furiously deny it, the
outcome of both market fundamentalism and anarchism, if applied universally, is
identical. The anarchists associate with the oppressed, the market fundamentalists with the oppressors. But by eliminating the state, both remove such restraints as prevent the strong from crushing
the weak. Ours is not a choice between government and no government. It is a choice between
government and the mafia. Over the past year I have been working with groups of climate protesters who have changed my view of what could be achieved. Most of them are
and weakest. As a friend of mine put it, when the means by which the oppressed can free themselves from persecution. But

under 30, and they bring to this issue a clear-headedness and pragmatism that I have never encountered in direct action movements before. They are prepared to take extraordinary risks to try to defend the
biosphere from the corporations, governments and social trends which threaten to make it uninhabitable. They do so for one reason only: that they love the world and fear for its future. It would be a tragedy if,
through the efforts of people like Ewa, they were to be diverted from this urgent task into the identity politics that have wrecked so many movements.

Reps Good

Framework
Imagining possible changes is necessary to motivate action on the climate
Elizabeth SHOVE Sociology @ Lancaster AND Gordon WALKER Geography @ Lancaster
7 [CAUTION! Transitions ahead: politics, practice, and sustainable transition management,
Environment and Planning C 39 (4)]
For academic readers, our commentary argues for loosening the intellectual grip of innovation studies, for backing off from the nested, hierarchical multi-level model as the only

The more we think about the politics and


practicalities of reflexive transition management, the more complex the process appears: for a policy audience, our words of caution could
model in town, and for exploring other social scientific, but also systemic theories of change.

be read as an invitation to abandon the whole endeavour. If agency, predictability and legitimacy are as limited as weve suggested, this might be the only sensible

we are with Rip (2006) in recognising the value, productivity and everyday
necessity of an illusion of agency , and of the working expectation that a difference can
be made even in the face of so much evidence to the contrary. The outcomes of actions
are unknowable, the system unsteerable and the effects of deliberate intervention
inherently unpredictable and, ironically, it is this that sustains concepts of agency and
management. As Rip argues illusions are productive because they motivate action and repair
work, and thus something (whatever) is achieved (Rip 2006: 94). Situated inside the systems they
seek to influence, governance actors and actors of other kinds as well - are part of the dynamics of
conclusion.However,

change : even if they cannot steer from the outside they are necessary to processes
within . This is, of course, also true of academic life. Here we are, busy critiquing and analysing transition
management in the expectation that somebody somewhere is listening and maybe even
taking notice. If we removed that illusion would we bother writing anything at all? Maybe
we need such fictions to keep us going, and maybe fiction or no - somewhere along the
line something really does happen, but not in ways that we can anticipate or know.

AT: warming distancing


In the context of warming it is important to privilege scientific data.
Personal experience is a flawed approach to climate changecreates
systemic bias and inaction
-cant detect climate change locally b/c of temperature fluctuations
-people are misled by extreme events instead of long term trends
-more likely to underestimate the impacts of warming

Weber and Stern 11


(professor @ Center for Research on Environmental Studies @ ColumbiaElke U. and Paul C
Stern, Public Understanding of Climate Change in the United States American Psychologist
Vol. 66, No. 4, 315328 May/June)
The power and limitations of personal experience. Personal experience is a powerful teacher, readily available to everyone from an early age.
Decisions based on personal experience with the outcomes of actions (e.g., touching a hot stove or losing money in the stock market) involve

However, learning from personal


experience can lead to systematic bias in understanding climate change. First, there are serious
problems detecting the signal. In most U.S. locales at this time, it is virtually impossible to detect the signal of climate
change from personal experience, amid the noise of random fluctuations around the central trend (Hansen,
Sato, Glascoe, & Ruedy, 1998). Second, people are likely to be misled by easily memorable extreme
events. Such events have a disproportionate effect on judgment (Keller, Siegrist, & Gutscher, 2006)
even though they are poor indicators of trends. Extreme events by definition are highly infrequent, and it takes a long
associative and affective processes that are fast and automatic (Weber, Shafir, & Blais, 2004).

time to detect a change in the probability of an event that occurs, on average, once in 50 years or less frequently. The likelihood of an increase in the
frequency or intensity of extreme climaterelated events large enough to be noticed by humans will be small for some time in many regions of the world.

Even individuals whose economic livelihood depends on weather and climate events (e.g.,
farmers or fishers) might not receive sufficient feedback from their daily or yearly personal
experience to reliably detect climate change, though recent surveys conducted in Alaska and Florida (two states in which
the climate signal has been relatively strong) show that such personal exposure greatly increases the concern and willingness of citizens in these
states to take action (Arctic Climate Impact Assessment, 2004; Leiserowitz & Broad, 2008).6 These studies are noteworthy for examining peoples
attempts to learn about climate change from personal experience, providing direct empirical evidence about the power as well as the shortcomings of
this form of learning in this domain, rather than extrapolating from results of research in other domains. Third, experiential learning tends to bias the
publics understanding because of a tendency to over-weight recent events (Hertwig, Barron, Weber, & Erev, 2004). The evaluation of probabilistic
outcomes follows classical reinforcement learning models, in which positive (negative) consequences increase (decrease) the likelihood of a behavior
that gave rise to them. Such learning processes give recent events more weight than distant events, which is adaptive in dynamic environments where
circumstances change with the seasons or other cycles or trends (Weber et al., 2004). Because extreme events have a small probability of having
occurred recently, they usually have a smaller impact on the decision than their objective likelihood of occurrence would warrant. But when they do
occur, recency weighting gives them a much larger impact on judgment and decision than their probability warrants, making decisions from experience
more volatile across past outcome histories than decisions from description (Yechiam, Barron, & Erev, 2005).

As a result,

nonscientists can be expected to overreact to rare events like a hurricane or a heat wave (Li, Johnson, & Zaval,
2011) but most of the time to underestimate the future adverse consequences of climate
change. Beliefs in climate change have been shown to be affected by local weather conditions (Li et al., 2011), and a relatively cool 2008 may
have influenced the drop in American concern about climate change in 20082009 (Woods Institute for the Environment, 2010). Confusing weather

Climate scientists can also overreact to


single vivid events, but their greater reliance on analytic processing, accumulations of data,
statistical descriptions and model outputs, and scientific deliberation and debate can be
expected to dampen this tendency. Without such correctives, nonscientists are more likely than scientists to accept evidence that
with climate increases the potential for these sorts of error (Weber, 2010).

confirms preexisting beliefs and to fail to search out disconfirming evidence (Evans, 1989). The scientific method can be seen as a cultural adaptation
designed to counteract the emotionally comforting desire for confirmation of ones beliefs, which is present in everyone (M. Gardner, 1957). Finally,

nonscientists differ from scientists in the way they react to uncertainty. Rather than using probability
theory to gauge and express the degree of belief in possible future events and to incorporate new evidence, nonscientists respond to uncertainty in
ways that are more emotional than analytic (Loewenstein, Weber, Hsee, & Welch, 2001) and in qualitatively different ways depending on whether the
uncertain events are perceived as favorable or adverse (Smithson, 2008). Nonscientists prefer concrete representations of uncertainty that relate to
their experience (Marx et al., 2007). To satisfy this preference, some scientists translate probabilistic forecasts into a small set of scenarios (e.g., bestto worst-case) to facilitate strategic planning by professional groups such as military commanders, oil company managers, and policymakers
(Schoemaker, 1995).

AT: Apoc Rhetoric Bad (Warming)


Disaster framing of warming overcomes fear and disbelief.
Joe ROMM, Fellow at American Progress, editor of Climate Progress, Ph.D. in physics from
MIT, 12 [February 26, 2012, Apocalypse Not: The Oscars, The Media And The Myth of
Constant Repetition of Doomsday Messages on Climate,
http://thinkprogress.org/romm/2012/02/26/432546/apocalypse-not-oscars-media-myth-ofrepetition-of-doomsday-messages-on-climate/#more-432546]
The two greatest myths about global warming communications are 1) constant repetition
of doomsday messages has been a major, ongoing strategy and 2) that strategy doesnt
work and indeed is actually counterproductive! These myths are so deeply ingrained in the environmental and progressive political
community that when we finally had a serious shot at a climate bill, the powers that be decided not to focus on the threat posed by climate change in any serious fashion in their

These myths are so


deeply ingrained in the mainstream media that such messaging, when it is tried, is
routinely attacked and denounced and the flimsiest studies are interpreted exactly
$200 million communications effort (see my 6/10 post Can you solve global warming without talking about global warming?).

backwards to drive the erroneous message home (see Dire straits: Media blows the story of UC Berkeley study on climate
messaging) The only time anything approximating this kind of messaging not doomsday but what Id call blunt, science-based messaging that also makes clear the
problem is solvable was in 2006 and 2007 with the release of An Inconvenient Truth (and the 4 assessment reports of the Intergovernmental Panel on Climate Change and

data suggest that strategy measurably moved the public to


become more concerned about the threat posed by global warming (see recent study here). Youd think it would be pretty obvious that the
public is not going to be concerned about an issue unless one explains why they should
be concerned about an issue. And the social science literature , including the vast literature on advertising and
media coverage like the April 2006 cover of Time). The

could not be clearer that only repeated messages have any chance of sinking in
and moving the needle. Because I doubt any serious movement of public opinion or
mobilization of political action could possibly occur until these myths are shattered , Ill do a
marketing,

multipart series on this subject, featuring public opinion analysis, quotes by leading experts, and the latest social science research. Since this is Oscar night, though, it seems

climate change
has been mostly an invisible issue for several years and the message of conspicuous
consumption and business-as-usual reigns supreme. The motivation for this post actually came up because I received an eappropriate to start by looking at what messages the public are exposed to in popular culture and the media. It aint doomsday. Quite the reverse,

mail from a journalist commenting that the constant repetition of doomsday messages doesnt work as a messaging strategy. I had to demur, for the reasons noted above. But
it did get me thinking about what messages the public are exposed to, especially as Ive been rushing to see the movies nominated for Best Picture this year. I am a huge movie
buff, but as parents of 5-year-olds know, it isnt easy to stay up with the latest movies. That said, good luck finding a popular movie in recent years that even touches on climate
change, let alone one a popular one that would pass for doomsday messaging. Best Picture nominee The Tree of Life has been billed as an environmental movie and even
shown at environmental film festivals but while it is certainly depressing, climate-related it aint. In fact, if that is truly someones idea of environmental movie, count me out.
The closest to a genuine popular climate movie was the dreadfully unscientific The Day After Tomorrow, which is from 2004 (and arguably set back the messaging effort by
putting the absurd global cooling notion in peoples heads! Even Avatar, the most successful movie of all time and the most epic piece of environmental advocacy ever
captured on celluloid, as one producer put it, omits the climate doomsday message. One of my favorite eco-movies, Wall-E, is an eco-dystopian gem and an anti-consumption
movie, but it isnt a climate movie. I will be interested to see The Hunger Games, but Ive read all 3 of the bestselling post-apocalyptic young adult novels hey, thats my job!
and they dont qualify as climate change doomsday messaging (more on that later). So, no, the movies certainly dont expose the public to constant doomsday messages on

The broad American public is


exposed to virtually no doomsday messages , let alone constant ones, on climate change
climate. Here are the key points about what repeated messages the American public is exposed to:

in popular culture (TV and the movies and even online). There is not one single TV show on any network devoted to this subject, which is, arguably, more consequential than

The same goes for the news media, whose coverage of climate
change has collapsed (see Network News Coverage of Climate Change Collapsed in 2011). When the media do cover
climate change in recent years, the overwhelming majority of coverage is devoid of any
doomsday messages and many outlets still feature hard-core deniers. Just imagine what
the publics view of climate would be if it got the same coverage as, say, unemployment,
the housing crisis or even the deficit? When was the last time you saw an employment denier quoted on TV or in a newspaper? The
public is exposed to constant messages promoting business as usual and indeed
idolizing conspicuous consumption. See, for instance, Breaking: The earth is breaking but how about that Royal Wedding? Our
any other preventable issue we face.

political elite and intelligentsia, including MSM pundits and the supposedly liberal media like, say, MSNBC, hardly even talk
about climate change and when they do, it isnt doomsday. Indeed, there isnt even a single
national columnist for a major media outlet who writes primarily on climate. Most
liberal columnists rarely mention it. At least a quarter of the public chooses media that devote a vast amount of time to the notion that
global warming is a hoax and that environmentalists are extremists and that clean energy is a joke. In the MSM, conservative pundits routinely trash climate science and mock

The major energy companies


bombard the airwaves with millions and millions of dollars of repetitious pro-fossil-fuel
ads. The environmentalists spend far, far less money. As noted above, the one time they did run a
major campaign to push a climate bill, they and their political allies including the
president explicitly did NOT talk much about climate change, particularly doomsday
messaging Environmentalists when they do appear in popular culture, especially TV, are routinely mocked. There
is very little mass communication of doomsday messages online. Check out the most popular websites. General
clean energy. Just listen to, say, Joe Scarborough on MSNBCs Morning Joe mock clean energy sometime.

silence on the subject, and again, what coverage there is aint doomsday messaging. Go to the front page of the (moderately trafficked) environmental websites. Where is the

If you want to find anything approximating even modest, blunt, science-based


messaging built around the scientific literature, interviews with actual climate scientists
and a clear statement that we can solve this problem well, youve all found it, of course, but the only
people who see it are those who go looking for it. Of course, this blog is not even aimed at the general public. Probably 99% of
doomsday?

Americans havent even seen one of my headlines and 99.7% havent read one of my climate science posts. And Climate Progress is probably the most widely read, quoted,

Anyone dropping into America from another country or another


planet who started following popular culture and the news the way the overwhelming
majority of Americans do would get the distinct impression that nobody who matters is
terribly worried about climate change. And, of course, theyd be right see The failed presidency of Barack Obama, Part 2. It is
total BS that somehow the American public has been scared and overwhelmed by
repeated doomsday messaging into some sort of climate fatigue. If the publics concern
has dropped and public opinion analysis suggests it has dropped several percent (though is bouncing back a tad) that is primarily due to
the conservative medias disinformation campaign impact on Tea Party conservatives
and reposted climate science blog in the world.

and to the treatment of this as a nonissue by most of the rest of the media, intelligentsia
and popular culture.

Apocalyptic Rhet Good


Its good and solves
Veldman 12
(Robin, Flordida religion PhD student, Narrating the Environmental Apocalypse: How Imagining
the End Facilitates Moral Reasoning Among Environmental Activists, Ethics and the
Environment, 17.1, JSTOR)
As we saw in the introduction, critics often argue that apocalyptic rhetoric induces feelings of hopelessness or fatalism. While it certainly does for some

apocalypticism also often goes hand in hand with activism. Some of the
strongest evidence of a connection between environmental apocalypticism and activism
comes from a national survey that examined whether Americans perceived climate change to be dangerous.
As part of his analysis, Anthony Leiserowitz identified several interpretive communities, which had consistent demographic
characteristics but varied in their levels of risk perception. The group who perceived the risk to be the greatest, which he labeled
alarmists, described climate change ETHICS & THE ENVIRONMENT, 17(1) 2012 using apocalyptic
language, such as Badbadbadlike after nuclear warno vegetation, Heat waves, its gonna kill the world, and Death of the planet
people, in this section I will present evidence that

(2005, 1440). Given such language, this would seem to be a reasonable way to operationalize environmental apocalypticism. If such apocalypticism
encouraged fatalism, we would expect alarmists to be less likely to have engaged in environmental behavior compared to groups with moderate or low

alarmists were significantly more likely to have


taken personal action to reduce greenhouse gas emissions (ibid.) than respondents who perceived climate change to pose less of a
levels of concern. To the contrary, however, Leiserowitz found that

threat. Interestingly, while one might expect such radical views to appeal only to a tiny minority, Leiserowitz found that a respectable eleven percent of

a separate national survey conducted in 2008,


Maibach, Roser-Renouf, and Leiserowitz found that a group they labeled the Alarmed (again, due to
their high levels of concern about climate change) are the segment most engaged in the issue of global warming.
They are very convinced it is happening, humancaused, and a serious and urgent threat. The Alarmed are already making
changes in their own lives and support an aggressive national response (2009, 3, emphasis
added). This group was far more likely than people with lower levels of concern over climate change to have engaged
in consumer activism (by rewarding companies that support action to reduce global warming with their business, for example) or to have
Americans fell into this group (ibid). Further supporting Leiserowitzs findings, in

contacted elected officials to express their concern. Additionally, the authors found that [w]hen asked which reason for action was most important to
them personally,

the Alarmed were most likely to select preventing the destruction of most life

on the planet (31%) (2009, 31)a finding suggesting that for many in this group it is specifically the
desire to avert catastrophe, rather than some other motivation, that encourages pro-environmental behavior. Taken
together, these and other studies (cf. Semenza et al. 2008 and DerKarabetia, Stephenson, and
Poggi 1996) provide important evidence that many of those who think environmental problems
pose a severe threat practice some form of activism, rather than giving way to fatalistic resignation.

Climate Reps Good


Our heuristic overcomes disbelief and mobilizes public responses
Romm 12 (Joe Romm is a Fellow at American Progress and is the editor of Climate Progress,
which New York Times columnist Tom Friedman called "the indispensable blog" and Time
magazine named one of the 25 Best Blogs of 2010. In 2009, Rolling Stone put Romm #88 on
its list of 100 people who are reinventing America. Time named him a Hero of the
Environment and The Webs most influential climate-change blogger. Romm was acting
assistant secretary of energy for energy efficiency and renewable energy in 1997, where he
oversaw $1 billion in R&D, demonstration, and deployment of low-carbon technology. He is a
Senior Fellow at American Progress and holds a Ph.D. in physics from MIT., 2/26/2012,
Apocalypse Not: The Oscars, The Media And The Myth of Constant Repetition of Doomsday
Messages on Climate, http://thinkprogress.org/romm/2012/02/26/432546/apocalypse-notoscars-media-myth-of-repetition-of-doomsday-messages-on-climate/#more-432546)
The two greatest myths about global warming communications are 1) constant repetition
of doomsday messages has been a major, ongoing strategy and 2) that strategy doesnt
work and indeed is actually counterproductive! These myths are so deeply ingrained in the environmental and progressive political
community that when we finally had a serious shot at a climate bill, the powers that be decided not to focus on the threat posed by climate change in any serious fashion in their

These myths are so


deeply ingrained in the mainstream media that such messaging, when it is tried, is
routinely attacked and denounced and the flimsiest studies are interpreted exactly
backwards to drive the erroneous message home (see Dire straits: Media blows the story of UC Berkeley study on climate
$200 million communications effort (see my 6/10 post Can you solve global warming without talking about global warming?).

messaging) The only time anything approximating this kind of messaging not doomsday but what Id call blunt, science-based messaging that also makes clear the
problem is solvable was in 2006 and 2007 with the release of An Inconvenient Truth (and the 4 assessment reports of the Intergovernmental Panel on Climate Change and
media coverage like the April 2006 cover of Time). The data suggest that strategy measurably moved the public to become more concerned about the threat posed by global

the public is not going to be concerned about an


issue unless one explains why they should be concerned about an issue. And the social
science literature, including the vast literature on advertising and marketing, could not be clearer that only repeated
messages have any chance of sinking in and moving the needle. Because I doubt any serious
movement of public opinion or mobilization of political action could possibly occur until
these myths are shattered, Ill do a multipart series on this subject, featuring public opinion analysis, quotes by leading experts, and the latest social
warming (see recent study here). Youd think it would be pretty obvious that

science research. Since this is Oscar night, though, it seems appropriate to start by looking at what messages the public are exposed to in popular culture and the media. It

climate change has been mostly an invisible issue for several years
and the message of conspicuous consumption and business-as-usual reigns supreme.
aint doomsday. Quite the reverse,

The
motivation for this post actually came up because I received an e-mail from a journalist commenting that the constant repetition of doomsday messages doesnt work as a
messaging strategy. I had to demur, for the reasons noted above. But it did get me thinking about what messages the public are exposed to, especially as Ive been rushing to
see the movies nominated for Best Picture this year. I am a huge movie buff, but as parents of 5-year-olds know, it isnt easy to stay up with the latest movies. That said, good
luck finding a popular movie in recent years that even touches on climate change, let alone one a popular one that would pass for doomsday messaging. Best Picture nominee
The Tree of Life has been billed as an environmental movie and even shown at environmental film festivals but while it is certainly depressing, climate-related it aint. In
fact, if that is truly someones idea of environmental movie, count me out. The closest to a genuine popular climate movie was the dreadfully unscientific The Day After
Tomorrow, which is from 2004 (and arguably set back the messaging effort by putting the absurd global cooling notion in peoples heads! Even Avatar, the most successful
movie of all time and the most epic piece of environmental advocacy ever captured on celluloid, as one producer put it, omits the climate doomsday message. One of my
favorite eco-movies, Wall-E, is an eco-dystopian gem and an anti-consumption movie, but it isnt a climate movie. I will be interested to see The Hunger Games, but Ive read
all 3 of the bestselling post-apocalyptic young adult novels hey, thats my job! and they dont qualify as climate change doomsday messaging (more on that later). So, no,
the movies certainly dont expose the public to constant doomsday messages on climate. Here are the key points about what repeated messages the American public is

The broad American public is exposed to virtually no doomsday messages, let


alone constant ones, on climate change in popular culture (TV and the movies and even online). There is not one single TV show on any
network devoted to this subject, which is, arguably, more consequential than any other preventable issue we face. The same goes for the news
media, whose coverage of climate change has collapsed (see Network News Coverage of Climate Change Collapsed in
2011). When the media do cover climate change in recent years, the overwhelming majority
of coverage is devoid of any doomsday messages and many outlets still feature hardexposed to:

core deniers. Just imagine what the publics view of climate would be if it got the same
coverage as, say, unemployment, the housing crisis or even the deficit? When was the last time you saw
an employment denier quoted on TV or in a newspaper? The public is exposed to constant messages promoting
business as usual and indeed idolizing conspicuous consumption. See, for instance, Breaking: The earth is
breaking but how about that Royal Wedding? Our political elite and intelligentsia, including MSM pundits and the supposedly liberal
media like, say, MSNBC, hardly even talk about climate change and when they do, it isnt doomsday .
Indeed, there isnt even a single national columnist for a major media outlet who writes
primarily on climate. Most liberal columnists rarely mention it. At least a quarter of the public chooses media
that devote a vast amount of time to the notion that global warming is a hoax and that environmentalists are extremists and that clean energy is a joke. In the MSM, conservative

The
major energy companies bombard the airwaves with millions and millions of dollars of
repetitious pro-fossil-fuel ads. The environmentalists spend far, far less money. As noted above, the
one time they did run a major campaign to push a climate bill, they and their political
allies including the president explicitly did NOT talk much about climate change,
particularly doomsday messaging Environmentalists when they do appear in popular culture, especially TV, are
routinely mocked. There is very little mass communication of doomsday messages
online. Check out the most popular websites. General silence on the subject, and again, what coverage there is aint doomsday messaging. Go to the front page of the
(moderately trafficked) environmental websites. Where is the doomsday? If you want to find anything approximating even modest,
blunt, science-based messaging built around the scientific literature, interviews with
actual climate scientists and a clear statement that we can solve this problem well, youve all
found it, of course, but the only people who see it are those who go looking for it . Of course, this blog is not even
pundits routinely trash climate science and mock clean energy. Just listen to, say, Joe Scarborough on MSNBCs Morning Joe mock clean energy sometime.

aimed at the general public. Probably 99% of Americans havent even seen one of my headlines and 99.7% havent read one of my climate science posts. And Climate Progress

Anyone dropping into America from


another country or another planet who started following popular culture and the news
the way the overwhelming majority of Americans do would get the distinct impression
that nobody who matters is terribly worried about climate change. And, of course, theyd be right see The
failed presidency of Barack Obama, Part 2. It is total BS that somehow the American public has been scared
and overwhelmed by repeated doomsday messaging into some sort of climate fatigue . If
the publics concern has dropped and public opinion analysis suggests it has dropped several percent (though is bouncing back a tad)
that is primarily due to the conservative medias disinformation campaign impact on Tea
Party conservatives and to the treatment of this as a nonissue by most of the rest of the
media, intelligentsia and popular culture.
is probably the most widely read, quoted, and reposted climate science blog in the world.

There is intrinsic value to the future-counterfactual simulation of the 1ac


representations of future climate change impacts leads to new solutions external
to the aff and avoid traditional pedagogical dilemmas of role-playing
--sex edited
--attempts to project conceptions of the past into the future rely on an anchoring bias that is flawed new scenarios must be injected
to understand new futures

Junio 13 PhD in Political Science @ Penn, currently @ Stanford


(Timothy and Thomas Mahnken, Conceiving of Future War: The Promise of Scenario Analysis
for International Relations, International Studies Review, 15)
We argue that future
counterfactuals may assist scholars in the same ways as historical ones and offer
As noted in our discussion of the counterfactuals literature, most scholars writing about the method focus on history.

additional benefits. Scenarios may be useful for theory building and development,
identifying new hypotheses, analyzing data-poor research topics, articulating world
views, setting new research agendas, avoiding cognitive biases, and teaching. Theory Building
and Development The structured analysis of future counterfactuals offers a unique approach for the study of causal effects in social systems. The first category, and perhaps

This is an approach
sometimes rec- ommended for qualitative research; it consists of writing a notional
depiction of what a case study might look like. This exercise helps researchers to think
through what variables are of greatest interest, what values those variables might take
on, and how they interact to cause values of the dependent variable. Sce- nario analysis is one way in which
researchers may conduct such a notional case study. Rather than introduce a timeless or historical vignette regarding fictional circumstances, the researcher
may find it beneficial to place their case in the future. This helps orient the research
project toward current and anticipated political issuesthus increasing the relevance of the workeven if the actual case
most significant, is the ability of researchers to use scenarios to identify variables of interest and consider ways to measure them.

studies are historical. Thinking through the causal process in this way helps the researcher to identify a wider range of explanatory variables, including those that have not yet

Scenario analysis also helps


the researcher to consider the range of values that the identified indepen- dent variables
may take on, as exploration of different worlds pushes the boundaries of the
researchers predispositions going into the research project. Robust scenario analysis thus helps the researcher to
occurred or may be of very low probability (but are still consis- tent with existing or proposed theoretical arguments).

identify the upper and lower bounds of their theory. Second, a commonly cited advantage of counterfactual reasoning that is useful for this process of theory building is a
researchers attempt to manipulate one variable in a causal process while holding others constant, thus isolating the effects of different values of the independent variable on the
dependent vari- able. Manipulating one variable at a time to do a better job of analyzing causal processes is often very difficult to do, as, in the real world, interactions between
variables often lead to unpredictable and nonlinear outcomes (Jervis 1997:34 60). For instance, a scholar conducting an analysis of tax rates and other domes- tic legislation
regarding oil may use a counterfactual of a different average oil price in the 1970s. Such a counterfactual would have some fairly obvious implica- tions for the domestic political
question, but a world in which that one variable were manipulated would have a large number of equally plausible second- and third-order consequences for regional politics in

Those conse- quences could conceivably feed back into domestic US politics,
thus affecting the social system under analysis in a way the researcher may not have
controlled for in the original scenario. Despite these acknowledged difficulties in using a manipulate one variable approach for the
the Middle East.

purpose of assaying real-world policy options, it is a useful input to the processes of building theory and research design. The best defense of such an approach is that all

even highly unrealistic modelssuch as James Fearons


famous ideal condition in which war should never occurare useful for studying real
events (Fearon 1995). Furthermore, manipulating one variable at a time is more appro- priate to some kinds of
forms of modeling involve abstractions from real- ity, and

counterfactual reasoning than others. Consider the three main categories of scenario use: political narratives, game theory and formal modeling, and experimentation. The
manipulate one variable approach seems least useful to political narratives, which often try to tackle such tough questions as What is the future of the international system?
Although scenarios offer advantages to developing and extending theory in regard to these sorts of ques- tions, particularly in assessing key drivers and articulating world views
(discussed in the next subsections), a scientific approach of controlling for various social factors is unlikely to succeed. In these projects, manipulating one variable at a time
serves only to develop one of many possible futures in the interest of extending the range of the theorys explanatory power. On the other hand, the manipulate one variable
approach offers more direct advantages for formal modeling and experimentation. The reasoning for each follows a comment made by Elinor Ostrom in her 1997 American

from...scenarios, one can proceed to formal


models and empirical testing in field and laboratory settings (Ostrom 1998). The experimental method with human
Political Sci- ence Association presidential address. Ostrom suggested that

subjects benefits strongly from the use of scenarios. In one study of how values factor into Ameri- cans economic decision making, a team of researchers sought to attribute
sig- nificant differences in average responses between conditions to the independent variables manipulated in the hypothetical scenario; that is, to the factors intuitive
neorealists should weigh heavily and intuitive economists should weigh lightly (Herrmann, Tetlock, and Diascro 2001). That is to say, one variable related to individuals world
views could be manipulated at once in the experiment, and the researcher may test for the significance of variance between the test and con- trol groups. After using scenarios
to better identify variables of interest and the role of their specific values in a causal process, a third category of applications of scenario analysis to theory building is to develop
new hypotheses and ways to test them. This follows from using scenarios to identify new independent variables and how their values may effect changes on the dependent
variable; each new causal argu- ment may (and should) be expressed as a hypothesis to be tested in the broader research project for which the scenario analysis was
developed. Additionally, day- after scenarios that seek to walk back the causal processes that may have led to a consequential event are particularly well suited to developing
hypotheses (Holmes and Yoshihara 2008).

By definition, this type of scenario analysis seeks to discover

causal pathways. For instance, one might seek to chart various paths by which a particular type of social revolution may occur in a country of interest. Each
narrative of how such a revolution could come to pass would result in at least one hypothesis regarding the links between the many variables of interest. These hypotheses may
then be tested against historical data or used to develop new kinds of data collection methods (discussed further in the next section). Finally, scenario analysis helps to explore
completely new theoretical projects in a deductive way, whereas a great deal of qualitative work in political science tends to be inductive from the case study method. The use of
scenario analysis may help scholars to pursue an abductive, or hybrid, method of theory build- ing that draws on both deductive reasoning and insights from cases (Mayer and
Pirri 1995). For example, a data-poor research subject, such as how states may respond to computer network attack, has few historical precedents (Mahnken 2011; Rid 2012). If
a researcher were interested in identifying the circumstances under which states are more likely to resort to violence in response to cyber attack, he would be confounded by

Scenario analysis beginning with the value


of violent counter-attack on the dependent variable (the DV being a states strategy choice) would help the
researcher to deduce likely circumstances under which such an outcome may occur.
Historical analysis, such as regarding other kinds of information threats, would be helpful for such a
project, but the differ- ences between cyber and other kinds of information transmission would result in an incomplete causal
narrative based on inductive reasoning alone. Data-Poor Research Topics Scenarios are a useful
the problem that never in history has a state responded with violence to such an attack.

method for theory building and research design for topics that, despite being of high
importance, lack an empirical base. The best example of this type of research is scholarship on nuclear warfare. An enormous literature
evolved during the Cold War regarding how a nuclear war might be fought and how escalation dynamics might occur (Kahn 1962; Brown and Mahnken 2011). This literature was
based almost exclusively on future counterfactuals, as there were no nuclear wars to study and a very low nconsisting of the Cuban Missile Crisis and very few other crises

Other
topics that are of high impor- tance but have a very low or zero n include great-power war, global epidemics, climate change ,
for publicly acknowledged close calls (Sagan 1995). Indeed, in our survey of the use of scenarios in the discipline, more than 25% were about nuclear warfare.

large-scale cyber attack, and weapon of mass destruction terrorism. The points made earlier regarding the identification of new variables and hypotheses are relevant here. In

scenario analysis helps to identify new sources of data.

addition to these advantages to new research topics,


This is
partially because scenarios help to identify new independent variables, thus leading the researcher to think about how to measure their values, but also by helping him to think

For instance, a day-after analysis of a scenario of


interest would cause the researcher to ask what [s]he would have needed to know to
predict the occurrence of the future counterfactuals and in turn help the researcher to
think about ways in which the discipline could identify that low-probability process if it
begins to happen in the real world. Articulation of World Views In addition to the process of building theory, scenarios are useful in helping
of proxies for measurement when direct observation is not possible.

to link theories. This is known as the articulation of a world view, which is a set of guiding logics for how an international system operates, such as realism, con- structivism,
and neoliberal institutionalism (Doyle 1997). For any world view, one may use scenario analysis to narrate what the theory logically dictates ought to happen in the world. Similar
to the benefit to theory building, but on a greater scale, this approach offers an opportunity for empirical validation of the world view over time. For instance, a scenario at a high
level of abstraction, such as the structure of a political system, allows for the validation or invalidation of multiple theories and interactions of theories. Should behavior in the
world fail to conform to the expectations of the world view, this of course offers scholars an opportunity to reconsider the guiding logics of their world view. In the early 1990s,
for example, a team of defense analysts had been tasked with developing a set of scenarios for the postCold War security environment (Project 2025 1994). They crafted a
scenario of a future world dominated by conflict between radical Islam and the West that resembled in some key respects the current struggle against Al Qaeda and its
Associated Movements. The sce- nario should not be seen as a prediction of current events, however. Rather, the scenario was developed to test whether or not one could
plausibly make the case for a global ideological conflict at a time when IR scholars such as Francis Fukuy- ama were arguing that ideological conflict had become a thing of the
past (Fukuyama 1992). Islam appeared to be the most plausible universalist ideology that could trigger widespread conflict. Another example is that a realist scholar may
benefit from a scenario analysis of the likely futures of the European Union. Such an analysis would project dif- ferent possible outcomesranging from dissolution through
more coherent for- eign policy and security integrationfrom basic premises consistent with the realist world view pitted against the premises of competing world views. The
final section of this article offers an example of when John Mearsheimer conducted such an analysis more than 20 years ago; he developed scenarios regarding the future of
Europe by comparing the expectations of offensive realism to other theoretical approaches. Setting Research Agendas The remainder of this section describes ways in which

Scenarios are often used in the business and


national security policy communities to have smarter con- versations. This use of the
scenario method differs from positivist social science and instead seeks to improve
knowledge through participation in scenario exer- cises. Such exercises usually involve a facilitated discussion. One way
scenarios are useful to political scientists in ways other than developing theory.

in which scenario conversations may make researchers smarter is to identify new research questions. Thinking about critical drivers of the future may help scholars to
understand areas that presently have no useful theories and to avoid the ten- dency of the political science discipline to consistently focus on a small number of questions. For
instance, the New Era Foreign Policy Conference, initiated by the University of California, Berkeley, and currently cosponsored by the Ameri- can University, University of
California, Berkeley, and Duke University, seeks to bring together graduate students of political science to engage in scenario analy- sis and identify future research topics.8
Avoiding Cognitive Biases In a methods book on scenarios, James Ogilvy discussed the inability of extre- mely bright people to see ahead due to cognitive biases, that is, what
people found unthinkable, though those futures eventually came into being. In the context of discussing the future of US and USSR nuclear arsenals during a 1980s scenario
exercise, Ogilvy wrote in hindsight that two decades later we now take for granted what was then unthinkable to some very good thinkers (Ogilvy 2002:191). This comment is
very similar to an insight by Philip Tetlock that polit- ical scientists tend to view surprises as overdetermined in hindsight, but as incon- ceivable ahead of time (Weber

Scenarios, whether formally written or developed for purposes of conversation, offer a powerful way for researchers
to compensate for cognitive biases endemic to any kind of human research. Scenarios do this by forcing researchers to con- front their most
1996:281).

basic assumptions about how the world operates and by teasing out the logical implications of extreme values on the independent variables of interest. As Peter Schwartz put

Scenario building is] all part of a process of self-reflection: understanding your- self
and your biases, identifying what matters to you, and perceiving where to put your
attention. It takes persistent work and honesty to penetrate our internal mental defenses. To ensure the success of our efforts, we need a clear under- standing of the
it, [

relationship between our own concerns and the wider world around us. To achieve that, it helps to have a constant stream of rich, diverse, and thought-provoking information
(Schwartz 1996:59). One of the most common forms of bias that scenarios help to compensate for is an anchoring bias; that is, the tendency to interpret new information in

One might also think about this as a linear


projection bias; scholars who have a view of how the world is operating in the present
(theoretically informed or not) may well project this view into the future. In addition to forcing scholars to explicitly confront these beliefs,
the scenario process offers a way to think about sorting new information. A com- pleted scenario project allows a researcher
to think about multiple futures (with differing plot lines for how the world arrived there) at once. Thus, new informa- tion is not automatically
ways that conform with our preexisting beliefs (Jervis 1976:143202).

compared against a single linear projection into the future, but rather weighted relative to alternative futures. New information might be consistent with all hypothesized futures,
in which case the new information may not lead to a new understanding. An example is the current debate regarding whether China is rising or declining relative to the United
States. This literature, despite access to the same data, offers remarkably different projections regarding the future strength of each state and what this portends for the
international sys- tem (Pape 2005; Layne 2009; Beckley 2011/2012; Subramanian 2011). In many cases, however, new information is likely to favor one trend line over another,
thus changing how a researcher assigns probabilities to various futures. Pedagogy Scenarios offer many of the same benefits as simulations, recently a hot topic in the
pedagological literature, to teaching in political science (Newkirk and Hamil- ton 1979; Smith and Boyer 1996; Newmann and Twigg 2000; Simpson and Kauss- ler 2009; Sasley

Indeed, scenarios are often a key part of simulation learning.

2010).
For instance, in a decision-making simulation in
which students are assigned the roles of heads of state, the students are often offered a scenario vignette to respond to with policy choices. The emphasis of scenarios and
simu- lations in pedagogy, however, is different. The literature on simulations tends to focus on experiential learning, but recent scholarship has cast some doubt on whether or
not this kind of learning improves students knowledge of core course concepts (Raymond 2010). Scenarios offer a way to make classroom exercises more explicitly oriented
toward the incorporation of theories.

For instance, rather than asking students to take on the roles of the

President, National Security Advisor, etc., the students may be presented with a vignette
and asked to analyze the strategic implications of the scenario for the United States. Both
coauthors of this article have used scenarios in classroom exercises. Tom Mahnken has taught the use of scenarios for stategic planning at the Naval War College. Tim Junio
used scenario exer- cises at the University of Pennsylvania. Students in the class International Secu- rity, having been assigned Thomas Schellings Arms and Influence and
other core readings on strategy, were asked to evaluate a scenario in which the United States had committed itself to military action, but was subsequently held hostage by a
foreign power. In Junios scenario, a future US President was led to believe that due to an intel- ligence breakthrough, North Korean nuclear weapon targets were rendered
vulnerable to a US first strike with conventional weapons. The United States and close allies saw this as an opportunity for regime change and pre-positioned US forces in the
region. The US President then issued an ultimatum to the North Korean regime to vacate the country within 48 hours, akin to the US threat to Sad- dam Hussein in 2003, or face
a forceful regime change at the hands of the US-led coalition. To the surprise of US leaders, North Koreas Supreme Leader went on television to announce that an unspecified
number of nuclear warheads had been smuggled into the United States as a contingency against such a situation. The Supreme Leader then declared that any act of
aggression against the North Korean people would be met with retaliation against the US homeland. Students in the class were asked to first discuss the strategic situation for
the United States. What mistakes had been made to get the United States into that scenario? What issues were at stake? Then, the students were asked to apply strategic
concepts to discuss how the United States might seek to extricate itself from the situation. The North Koreas blackmail scenario is an example of an extremely low-prob- ability
event that almost certainly would not justify much further analysis by the intelligence and defense policy communities, but is extremely useful for peda- gogy. This kind of

Rather than focus on


policy decisions alone, as simulations are likely to do, students are forced to bring deductive logic to
bear to assess the boundaries of the scenario. Demonstrations of Robust Scenario Analysis Mearsheimer 1990a,b A widely
scenario increases student interest in the material and forces them to engage with the theories and concepts of the course.

read example of scenario analysis, though one not often considered methodologically interesting, is John Mearsheimers Back to the Future: Insta- bility in Europe After the
Cold War (published more accessibly in The Atlantic as Why We Will Soon Miss the Cold War) (Mearsheimer 1990a,b). Mearshei- mer offered several scenarios of what post
Cold War Europe might look like; the one he deemed most probable suggested that Germany (and possibly others) would develop nuclear weapons and that European states
would resume security competition. Mearsheimer contrasted this scenario, driven by the theoretical expectations of offensive realism, with outcomes predicted by the democratic
peace and economic interdependence perspectives. The dependent variable of interest in Mearsheimers scenario analysis was the risk of war in Europe following the end of
the Cold War. His primary research question was, Would the end of the bipolar power structure result in a higher or lower risk of war? He considered several independent
variables, including the distribution of military power (possible values: a range along a spectrum from bipolar to multipolar [unipolar perhaps being logically possible, but he does
not explicitly include it as a possibility]); the character of military power, defined in terms of the distribution of nuclear weapons (possible values: aboli- tion, sustenance of
existing levels, unmitigated proliferation, or mitigated prolif- eration [current nuclear powers manage their spread]); and domestic politics (possible values: degree of nationalism,
ranging from high to low). Mearsheimer was explicit regarding how he believed these independent variables should effect values on his dependent variable of war proneness.
Bipolar power distributions were believed to be more stable than multipolar. Nuclear weapons were expected to increase the probability of war in the first three of his four
categori- cal values. High values of nationalism were expected to increase the risk of war, while low values of nationalism would reduce or keep even the risk of war. The
shortest formulation of Mearsheimers theory of offensive realism is that states seek to maximize their relative power. A relevant aspect of the theory that Mearsheimer
articulated is states perceptions of the costs and risks of going to war; he believed a competitive world under offensive realism might still be peaceful if the costs and risks of
going to war were perceived to be high and the benefits of going to war low (Mearsheimer 1990a:12). Mearshei- mer also noted competing theoretical approaches regarding
how European states were likely to behave during his analyzed time period: an international institutions perspective (IV: the strength of international institutions, ordinal), which
Mearsheimer deemed irrelevant as power-seeking behavior should trump institutional concerns; democratic peace theory (IV: joint democracy, binary), which he found
unpersuasive for a few reasons, including nationalism, defec- tion, and uneven spread of democracy among post-Soviet states; and pacifism (IV: binary, learned war is bad or
did not), which Mearsheimer believed lacks an empirical basis. Although Mearsheimer discussed these other theoretical perspectives superfi- cially, he developed a detailed
account based on how he believed offensive real- ism would effect values on his three main independent variables of interest. He believed it is certain that bipolarity will
disappear, and multipolarity will emerge in the new European order (Mearsheimer 1990a:31). He believed this because with the end of the US and Soviet spheres of influence,
European states would be strongly incentivized by the anarchic character of the international system to provide for their own security. Thus, no two states were likely to emerge
as clear poles in the European state system; rather, power would be diffused as many states competed with one other. Mearsheimer was highly confident in this out- come and
treated it as more of a background condition than an important determinant of outcomes on his dependent variable. The second two independent variables were nuclear
proliferation and nation- alism. Mearsheimer viewed the most critical uncertainty regarding the future of security in Europe as the distribution and deployment patterns of nuclear
weap- ons. He wrote that the best new order would incorporate the limited, managed proliferation of nuclear weapons. This would be more dangerous than the cur- rent order,
but considerably safer than 19001945. The worst order would be a non-nuclear Europe in which power inequalities emerge between the principal poles of power
(Mearsheimer 1990a:31). Mearsheimer offered scenarios of what it would look like if each of these outcomes resulted. Finally, Mearsheimer con- sidered the future that actually
resulted, or the continuation of existing nuclear weapon ownership patterns. He argued that his theory predicted this future would not come to be, as Germany was expected to
desire nuclear weapons so that they would not have to rely on Poland and Czechoslovakia to provide a bar- rier against a Soviet invasion, and because small East European
states would simi- larly perceive nuclear weapons to be of the highest security interest. Nationalism was less important and factored into Mearsheimers analysis as an
interaction effect between nuclear proliferation and nationalism that may make war more likely under some conditions. In summary, Mearsheimers article provides an excellent
example of scenario analysis being used to extend an existing theory and develop testable hypotheses that were subsequently falsified. His futures also constitute plot lines
formed by the interaction of multiple variables of interest. Mearsheimer explained why he expected a particular outcome in Western Europe in the 1990s and why he did not
expect other outcomes. The historical record clearly falsifies the hypoth- eses derived from his theory of offensive realism. Various reasons may explain why his theory was
incorrectsuch as normative claims, continued reliance on US security guarantees, and so onbut it is at least clear that his scenario-based approach framed a debate in a
rigorous and clearly articulated way and has led to new areas of exploration for the discipline. A second example of robust scenario analysis is a chapter from Keith Paynes
book The Fallacies of Cold War Deterrence and a New Direction (Payne 2001). In this chapter, Payne used a scenario to demonstrate how the assumptions underlying
traditional deterrence analysis, which were developed during the Cold War, may not apply to a conflict between the United States and China. Payne also used this scenario to
demonstrate why in the theoretical sections of his book he emphasized some variables that traditional deterrence theorists downplay. His self-proclaimed purpose was to use a
scenario analysis to test whether a more empirical approach, drawing on cultural and domestic political contexts, is more applicable to future conflicts than the deductive
reasoning applied to the Cold War. The dependent variable in Paynes analysis was whether or not an adversary is deterred. He then took traditional independent variables
from the existing deterrence literature and drew on extensive secondary sources to question what relaxing assumptions about the values of those variables would do to the
depen- dent variable. For instance, in deductive reasoning, such factors as a states cul- tures (organizational and in the usual sense of tradition) are assumed to be either
constant on both sides or irrelevant because other variables matter more. Payne suggested that in a particular context, these variables are not only impor- tant, but also may
dominate outcomes. To explain how variation in these independent variables may yield an unde- terred adversary, Payne developed an excellent full qualitative scenario. His
pri- mary interest was Chinese decision making. The context is whether or not the United States could deter China from escalating to violent conflict during a cri- sis over the
status of Taiwan. Rather than assume constant values for China on the independent variables, Payne manipulated these variables (several at once, not a one variable at a
time approach) to show how China: is more risk toler- ant than notional adversaries in the traditional deterrence literature; considers many political issues of lower importance
than the status of Taiwan; perceives it has little freedom to back down; and has difficulty understanding US demands and viewing them as credible. Factors that cause them to
have these different val- ues are related to the Chinese regimes culture and incentive structures. For instance, Payne focused on how the erosion of communist ideology has
led the regime to emphasize national unity and stability as justifications for its continued hold on power, thus making the Taiwan issue of high importance. His points regarding
risk tolerance come from Chinese strategic culture; Payne follows ana- lysts who place great meaning in the fact that the Chinese word for crisis has connotations of both
danger and opportunity. Variation on all of these indepen- dent variables may, Payne argued, lead to an undeterred China, although tradi- tional deterrence theory would yield a
deterred China. Conclusion The role of academics in policymaking is a cyclical debate in the IR subfield. Following the 9/11 terrorist attacks and subsequent wars in Iraq and
Afghani- stan, a vocal group has once again elevated the perspective that political science professors should be contributing to these pressing national security problems
(Andres and Beecher 1989; George 1993; Putnam 2003; Monroe 2005; Nye 2009; Mead 2010). Nearly all of the discourse on bridging the gap between academia and the
policy world emphasizes how academics may help policymak- ers, particularly with rigorous methods for testing social science hypotheses. The scenario method is one way in
which political scientists may improve the policy relevance of their work. It also shows that ideas flowing in the other direction are promising: the policy community and other
disciplines have potential to improve the quality of political science research. The future counterfactual approach has been used by policymakers and wealth creators to improve

Thinking and writing about the future in


a robust way offers politi- cal scientists an exciting opportunity to push the boundaries of
deci- sions for decades, while our discipline has consistently relied to a great degree on the past.

current debates and to generate new ones, while also improving the processes of
teaching and theory building.

We need apocalyptic framing to give us planning time for adaptive and


mitigating strategies. Ongoing messages about the danger of inaction are
key
-Impact is bigalarm is good because it gives us planning time and encourages adaptive
measures
-We need an ongoing message about the dangers of inactionthis should be alarmist
Tom FOWLER ET AL. 14, Public Health, Epidemiology and Biostatistics @ University of
Birmingham, United Kingdom, Sally C. Davies, Chief Medical Officer for England, and David
Walker, Deputy Chief Medical Officer for England [The risk/benefit of predicting a post-antibiotic
era: Is the alarm working? Annals of the New York Academy of Sciences, Vol. 1323, p. 1-10
(September 2014)]
Conclusions The

impact of a post-antibiotic era would be enormous . We would see the reemergence of many infectious diseases and the risks of many conventional medical
treatments would rise, increasing the potential morbidity and mortality from many disease processes. Costs of medical
care and the social impact of caring for the sick and those with disabilities would be great. The likelihood of a postantibiotic era and the characteristics of such a scenario will depend very much on the
action that we take collectively over the next few decades. Without further action, the
probability of this eventuality is high owing to the inexorable rise of antimicrobial
resistance and the declining rate of discovery of new antimicrobial agents. The benefit of
raising the alarm about this issue is twofold. First, it enables us to explore adaptation
strategies should such a scenario arise through research into potential problems and
effective solutions. It is essential that this process begins early to allow for timely
responses. A move to more isolation facilities, for example, would require planning, funding,
and construction time. Second, it enables us to more accurately assess the true costs of
antimicrobial resistance and to consider the prioritization of mitigation strategies, such as
the development of global programs for antibiotic stewardship and the negotiation of new business models for the development of
new antibiotics. We might also want to prioritize research and development activities into alternative diagnostic, prevention, and
treatment strategies for infectious diseases, for example, genomic sequencing and innovative immunization technology. Of course,

these processes also take time. Concerns have been raised about antimicrobial
resistance for many decades but the real possibility of a post-antibiotic era has only
emerged recently , as the antibiotic development pipeline has begun to dry up. This
factor seems to have elevated concerns internationally to a level where international
collaboration and government/private sector partnerships are real possibilities. Many individual
countries, the WHO, and other international communities are now prioritizing this issue as a key threat, and work is
underway to mitigate the risk and adapt to the consequences. The alarm has been raised and
there is clear evidence of response within individual healthcare systems, nationally and globally.

However, experience shows that to truly confront this issue requires the leadership of health
professionals and a concerted, unified, and ongoing message that ensures that the public and
politicians know the potential dangers of inaction .

CLS

Reform Effective
Reformism is effective and brings revolutionary change closer rather than
pushing it away
Delgado 9
(Richard Delgado 9, self-appointed Minority scholar, Chair of Law at the University of Alabama
Law School, J.D. from the University of California, Berkeley, his books have won eight national
book prizes, including six Gustavus Myers awards for outstanding book on human rights in
North America, the American Library Associations Outstanding Academic Book, and a Pulitzer
Prize nomination. Professor Delgados teaching and writing focus on race, the legal profession,
and social change, 2009, Does Critical Legal Studies Have What Minorities Want, Arguing
about Law, p. 588-590)
CLS critique of piecemeal reform Critical scholars reject the idea of piecemeal reform.
Incremental change, they argue, merely postpones the wholesale reformation that must
occur to create a decent society. Even worse, an unfair social system survives by using
piecemeal reform to disguise and legitimize oppression. Those who control the system
weaken resistance by pointing to the occasional concession to, or periodic court victory of, a
black plaintiff or worker as evidence that the system is fair and just. In fact, Crits believe that teaching the common law
2. The

or using the case method in law school is a disguised means of preaching incrementalism and thereby maintaining the current power structure. To avoid this, CLS scholars urge
law professors to abandon the case method, give up the effort to nd rationality and order in the case law, and teach in an unabashedly political fashion.

The

CLS

critique of piecemeal reform is familiar, imperialistic and wrong . Minorities know from bitter
experience that occasional court victories do not mean the Promised Land is at hand . The critique is imperialistic in that it tells minorities and
other oppressed peoples how they should interpret events affecting them. A court order
directing a housing authority to disburse funds for heating in subsidized housing may
postpone the revolution, or it may not . In the meantime, the order keeps a number of
poor families warm. This may mean more to them than it does to a comfortable academic
working in a warm office. It smacks of paternalism to assert that the possibility of
revolution later outweighs the certainty of heat now, unless there is evidence for that possibility. The Crits do not

incremental changes may bring


revolutionary changes closer , not push them further away. Not all small

offer such evidence. Indeed, some

reforms induce complacency; some may whet the appetite for further combat. The
welfare family may hold a tenants union meeting in their heated living room. CLS scholars
critique of piecemeal reform often misses these possibilities, and neglects the question
of whether total change, when it comes, will be what we want. 3. CLS Idealism The CLS program is also
idealistic. CLS scholars idealism transforms social reality into mental construct . Facts become
intelligible only through the categories of thought that we bring to experience. Crits argue that the principal impediments to
achieving an ideal society are intellectual. People are imprisoned by a destructive system
of mental categories that blocks any vision of a better world." Liberal capitalist ideology so shackles individuals that they willingly accept
a truncated existence and believe it to be the best available. Changing the world requires primarily that we begin to think about it differently. To
help break the mental chains and clear the way for the creation of a new and better world,

Crits practice "trashing"a process by which law and social structures are shown to be
contingent, inconsistent and irrationally supportive of the status qua without good
reason. CLS scholars' idealism has a familiar ring to minority ears. We cannot help but be reminded of those
fundamentalist preachers who have assured us that our lot will only improve once we
"see the light" and are "saved."

Misconstrues Law
CLS cant distinguish between rules and principles over generalizes laws
as rules and constructs false situations solely for liberal resolution
McCormick 99
(John P., Assistant Political Science Pf - Yale, American Political Science Review. v93, p. June
1999)//akim
For Habermas, the de facto strategy of reducing all cases to "exceptions" leads CLS to the conclusion that since there is no
coherence to liberal-democratic law, there is no justice. Because each law is capable of contradicting another, a fact exacerbated
by subjective [footnotes] 418 judging, the whole legal order is suspect. But Habermas (1996, 216-7) argues that at base

CLS confuses legal rules with legal principles. Rules are norms specified for application
in particular cases, such as stipulations for drawing up wills or traffic laws. Rules are often preceded by an
"if" clause, which clearly identifies the application situation: If circumstance X obtains, then procedure Y is in order (p. 208).
Principles are general legal standards with far fewer prespecified application guidelines. The meaning of
procedural rights, human rights, or equal protection is ultimately made specific in
interpretation and application (pp. 172, 208). Rules may collide in irreconcilable ways, but they do not constitute
the broader workings of justice as such, as do principles. In fact, rules are supposed to conflict because they are so finely
specified for particular situations that the legal system depends on their conflict. This kind of conflict can be inter preted as
facilitating the determinacy of the system, rather than as thrusting it into indeterminacy jeopardy (Habermas 1996, 217). Rules
are highly determinate because they set the conditions of their own application. They are what Habermas calls "self-executing"
laws, whereas such principles as equality or mutual recognition are "implemented along administrative paths," where their
meaning is fully fleshed out (p. 172). Principles are necessarily indeterminate, to some extent, because they do not apply
themselves but, rather, require "additional specification" (p. 217). Unlike rules that virtually apply themselves in appropriate
circumstances, in jurisprudential practice, a principle must be carefully examined to see whether another better conforms to a
particular case. There is, for Habermas, an intimate relationship between rules

and principles, but they ought not


to be collapsed in the way that CLS and Schmitt collapse them.21 Because Schmitt and
CLS perceive all law as rules and, more important, caricature liberalism as ultimately doing the same, they set
up a false either/or dilemma for liberal adjudication. Thus viewed, all legal materials are potentially in
chaotic disagreement with one another. The conflating of rules and principles is at the root of Schmitt's and CLS's exaggerated
characterization of formalism. Since

Schmitt and CLS group all liberal law under the blanket term
"formal" and do not acknowledge that formal and substantive modes of law can coexist
in the liberal rule of law, they encourage the appearance of arbitrary and conflicting
practices that may not actually exist upon more differentiated analy- [footnotes] [header] sis.
In fact, it is precisely because Schmitt, like the positivism he criticizes, views the whole liberal legal system as
merely a set of rules that he can move effortlessly from statutes to constitutions, gaps to
exceptions, generally with malicious political intent. Habermas (1996, 208) remarks that a traditional way of avoiding rule conflict
in the law in cases in which it might be confusing rather than instructive is the recourse to exception clauses. Schmitt

exploits this by reading a built-in exception clause into the entire statutory legal system
(understood as a collection of exceptionable rules) with disastrous results. If all laws have exception
conditions, then so does the legal order itself, which is therefore suspendable in the name of a
sovereign popular will.

CLS Flawed
CLS is flawed contradictions
McCormick 99
(John P., Assistant Political Science Pf - Yale, American Political Science Review. v93, p. June
1999)//akim

CLS also misrepresents the formal quality of liberal legalism. In its caricature of formalism, CLS very often assails a
straw man: Even the most stringent legal formalists acknowledge a necessary but not devastating lack of
total determinacy in the liberal rule of law.22 For Habermas, the formalism of the law has a very precise
meaning. Formalism pertains primarily to the law's construction; that is, it is logically correct and established
through prearranged procedures. The decisive aspect of the adjudication of principles, however, is not necessarily an
air-tight formalism but, rather, appropriateness for a particular case (1996, 218). According to Habermas, in
the application of principles all suitable normative reasons must be collected and then the situation itself interpreted. From a high
level of abstraction, the formal modes of law associated, for instance, with such principles of criminal justice as
retribution, appear irreconcilable with the more concrete, or material, modes of law that actualize the principle of substantive
equality through social welfare provisions. But each is appropriate to a vastly different set of cases. To
illustrate this point, Rosenfeld and Weinrib have each used the distinction between corrective and distributive justice to clear up
misunderstandings stemming from conflations of the two. The primarily backward-looking

quality of corrective
justice-redressing a wrong already perpetrated by means of procedures previously establishedmakes its workings appear relatively transparent. The future-directed quality of distributive
justice deals with quantities not always known in advance-how much of what, to whom, and by what means? This seems
terribly vague by contrast with criminal justice codes (Rosenfeld 1992; Weinrib 1988). CLS lumps the two
modes together in an attack on "the law." Just because the conditions of principle
adjudication cannot, and should not, be fully laid out in advance does not render them
hemorrhaging wounds in the body legal. It is largely their open-endedness that gives
the process a public and democratic quality. According to Habermas (1996, 172), in legal discourses
of application competing parties and state authorities present arguments over what norms are most
relevant to the facts of a case. There are two interpretive steps for[foot notes] . judges that are not preordained: the
description of facts and the description of norms (p. 218). According to Habermas, "once the situational features of the case have
been described as exhaustively as possible from all normatively relevant points of view," the appropriate principle can be selected
(p. 260). When one norm is selected over another for application, it does not invalidate, nullify, or render irrational those not
chosen. "Recessive

principles," according to Habermas, those that are decided to be less relevant, do not lose
validity for a case as do rules, but only their "contextual relevance" (p. 209). CLS does not distinguish
between principles that contradict one another per se and those that collide in a particular
case (p. 216). Because a particular principle was not applied does not render it at war with
other principles or with the system as a whole. "A plausible connection [is maintained]
between the pertinent norm and the norms that ... do not prevail such that the coherence
of'the rule system as a whole remains unaffected" (p. 260). This is not an ideological weighing or privileging of
one over the other but, rather, a logical selection.

State Resilient
Critical legal theorists ignore state resiliency to civil and social principles
McCormick 99
(John P., Assistant Political Science Pf - Yale, American Political Science Review. v93, p. June
1999)//akim
In the previous section, I sympathetically presented Habermas's

distinction between rules and principles in


adjudication, particularly how they relate to earlier historical paradigms of law in a liberaldemocratic framework. It is precisely this distinction, however, that at this juncture is especially
problematic at the supranational level. It is presently either too easy or too difficult to delineate juridical
principles in supranational configurations, even one as relatively homogeneous as the EU.34 If the governing principles
of a [foot notes] [header] juridical unit like the EU pertain merely to the regulation of an economic freetrade zone, for instance, then those principles will clearly be too thin to preserve the gains of
liberal or social democracy associated with two or three centuries of expanding civil and
social rights. According to skeptics (Garrett, Keleman, and Schulz 1998), appeals by European citizens or associations to
the ECJ on the grounds of more substantive principles of justice, and attempts by the ECJ to address them, will be resisted by
nation-states. While the autonomy of the ECJ is a hotly contested issue in European studies, the

court's activities are


no doubt constrained to a significant degree by the preemptive and reactive pressure of
EU member states. Habermas asserts supranational legal protection as the necessary
response to the expanding transnational power of economic and hence political actors. But evidence suggests
that this development is presently stymied by the still quasi-sovereign nation-states. EU members may have suffered some loss of
sovereignty to the extent that they participate in the process of integration as compensation for economic losses that many
attribute to international developments. These

states are, however, still sufficiently strong, and jealous of what sovereignty
to block the extension of civil and social principles of justice to the
adjudication processes of the ECJ (Garrett, Keleman, and Schulz 1998). Habermas takes note of this retrenchist
behavior on the part of the member states (1998, 124), but he does not construe it as a particularly significant
they still hold, to be able

obstacle to the development of his prospective communicatively and constitutionally democratic European continental regime (p.
127). On the contrary, he

urges state actors to undertake the "heroic effort" to promote actively


the development of supranational institutions that could replace the actors themselves,
or at least replicate themselves on a continental level (p. 124): institutions such as "a European-wide, integrated public sphere . . .,
a civil society encompassing interest associations, nongovernmental organizations, citizens' movements, etc., and naturally a
party system appropriate to a European arena" (p. 160). On Habermas's own terms, however, within the

gap between
the nation-state's declining ability to secure and advance principles of social justice (as a result
of evaporating tax bases, increased environmental threats, antiimmigrant and minority-unfriendly policies, and so on) and their
reluctance to accede fully these responsibilities to fora like the ECJ, lies

the abyss of the supranational


democratic possibilities of Habermas's theoretical framework.35 If the ECJ is to adjudicate [foot
notes] 424 exclusively on the basis of market-related rules, and merely make hortatory appeals to principles associated with civil,
social, and human rights, then the subtle distinction and interplay of rules and principles upon which Habermas bases his
rationally democratic theory of adjudication would appear virtually ineffectual in the EU context for the conceivable future. There is
obviously significant debate on the extent to which pessimism is warranted concerning social welfare in contemporary Europe.36
But it

is precisely the existence and importance of such debates that renders problematic
their absence from Habermas's discourse theory of law and reflections on European
integration, especially given his stated concern with "empirically grounded arguments" as
much as a "normative perspective" (1998, 158). At one point in the discussion of the reflexive theory of law in
Between Facts and Norms, Habermas highlights the historical limitations of understanding law in terms of paradigms. But he has
in mind the traditional resistance of the bourgeois formal law to materialized welfare-state law, not the resistance of his own
understanding of the two paradigms to new, perhaps supranational, paradigms: Paradigms

harden into

ideologies insofar as they systematically close themselves off from the perception of
radically new situations and resist different interpretations of rights and principles,
interpretations that press for acknowledgment in the light of radically new historical experiences .... [They] stabilize
themselves through professionally and judicially institutionalized monopolies on interpretation and permit
only internal revision according to their own standards (1996, 221, emphasis added). This could, ironically, be said of

Habermas's framework itself. His discourse theory of law has to some extent closed off, resisted, and
insulated itself against reflections on the law's radically new historical situation. To that
extent, Habermas falls somewhat short of the standards set in his early work and his own adoption of the strictures of critical
theory. Obviously, the

simple assertion of "history" or the injunction toward the more finegrained analysis of "context" will not automatically or satisfactorily address the deficiencies
of liberal normative theory or its concrete-fixated assailants (Shapiro 1990, 207-13). But, as I conclude
below, the combination of the concretely historical and categorically universal method practiced in Habermas's [foot notes]
[header] early efforts, and even still alluded to in his later ones, offers a viable provisional guide for thinking critically about the
law in the coming years.

Feminisms

F/W & Tech Discourse Good


Using technical discourse strategically is key to solve the K
Caprioli 4
(Mary, Dept. of Political Science @ the University of Tennessee, PhD from the University of
Connecticut, Feminist IR Theory and Quantitative Methodology: A Critical Analysis,
International Studies Review, Vol. 6, No. 2 (Jun., 2004), pp. 253-269)
We should learn from the research of feminist scholars to engage in a dialogue that can be
understood . Carol Cohn (1987), for example, found that one could not be understood or taken
seriously within the national security arena without using a masculine-gendered language. In other
words, a common language is necessary to understand and be understood. This insight could be applied to feminist research within international relations. Why not, as
Charlotte Hooper (2001:10) suggests, make "strategic use of [expert jargon ] to gain credibility for feminist
arguments (or otherwise subvert it for feminist ends)." Little justification exists for abandoning the liberal
empiricists who reason that "the problem of developing better knowledge lies not with
the scientific method itself but with the biases in the ways in which our theories have
been focused and developed" (Tickner 2001:13).

AT: excludes women in debate


The strength of feminist theory comes from the interface between theory
and practical legal approaches. Their notion of law is that of a closed space
we must advance a model of informed engagement to turn law into a site
of struggle rather than just a tool of struggle.
Bottomley and Conaghan 93
(ANNE BOTTOMLEY and JOANNE CONAGHAN, Kent Law School, University of Kent,
Canterbury, Kent CT2 2NY, England) Feminist Theory and Legal Strategy 20 J.L. & Soc'y 1
1993
this could be read as no more than an attempt to
bracket together themes and issues developed in theoretical debate with concrete,
specific examples by way of application. Indeed some of the papers may be read this way: such a project has a recognized academic
history with which we have all engaged at some stage in our own work. Equally, the move from particularities to more
general principles has frequently been employed in (feminist) academic work; locating strategies within a broader
We have entitled this collection feminist theory and legal strategy. At one level,

theoretical context is a common approach and one which allows us to continually remind ourselves of the potential ramifications of our immediate

Feminist work within law has been


characterized by both of these trajectories; it is in the very mode of our operation as lawyers that we are continually faced
project. Again, there are aspects of this in some of the papers which follow.'

with specific instances. It is also in our lives as women that our feminism is informed by the particular operations and instances of our engagement with
law. This collection, however, is premised on a rather different base. It is a base which we would assert has been the particular strength of feminist

our work is not so much concerned with the application of 'theory' to 'law'
or vice versa, but rather that it is the interface between the two which is the very site of our
work within and on law; it is that

theoretical work . What we are posing here is twofold. On the one hand, it is a model of theory which sees theory as simply that - a model;
one which offers us ideas which we return to and use as a series of tools to raise questions, test insights, and find a language to bring together and

the strength of
feminist jurisprudence is tested not by claims to internal coherence but rather by an
ability to deliver. This may sound harshly pragmatic but we firmly believe that those aspects of theoretical work
which have proven of value to us over the years are those which have enabled us to
develop and access concrete material gains in our work. There are undoubtedly those who would read this as an anticommunicate our ideas. It does not exist in and for itself, but only in relation to the use to which it is put. For this reason,

theoretical stance. Carol Smart might, for instance, identify us with what she terms as 'resistance to theory': based on the argument that, because law
is a practice which has actual material consequences for women, what is needed in response is counterpractice not theory. This constituency demands
'practical engagement' and continually renders (mere?) theoretical practice inadequate. This argument comes from certain feminist constituencies
which may define 'doing' theory as male. These... elements present a major obstacle to proponents of feminist legal theory as they (we) meet with the
frustrations of being ignored or seen as outmoded in and by law and are simultaneously moved to renounce theory by the moral imperative of doing
something through or in law.' In our view, it

is not a question of whether different forms of theoretical


practice are seen as male or not. Equally, it is not a question of demanding practical
engagement at the expense of theory. It is rather that the mode and strength of feminist theory
in law has been the interface between the use of abstract models and ideas to
interrogate the practices of law .' It may not have been articulated in quite this way before - in that sense it has not yet been
'theorized' - but if we examine our feminist academic heritage, it is the one point which consistently characterizes the work, in all its rich plurality and
diverse perspectives. Thus, our

stance is not anti-theory, neither is it concerned simply with the


application of theory. It is theory, but it is not one which traditionalists would recognize in
their limited use of the term. If the counterpoint to our approach is the idea that theory stands for theory's sake, then certainly our
notion of theory will be too radical. If we need (and we are not convinced that we do need) to find parallels in the work of 'theorists' who would
exemplify a similar approach it would be with, for instance, Braidotti5 or Deleuze.6 They exemplify a theoretical stance based on engagement, on what
used to be called 'praxis'. It

is not so much a question of application but rather of movement

between 'theory' and 'practice'. This theoretical stance allows, perhaps reflects, a pluralism
within which there can be no one analytical model deployed but rather a series of
engagements and incursions . In a legal context, this precludes seeing 'law' as a closed model
but rather recognizes it as a series of ideas, practices, and engagements, all of which
are loosely held together under the rubric of the term 'law' . We would be the first to acknowledge the
experience of any marginal or disadvantaged group within our society which has (rightly) felt, and continues to feel, in many ways and in many
instances, the oppressive (or in terms of their own needs, unresponsive) operation of law.7 But to then characterize law simply in these terms is to fly in
the face of our experience of law in other ways. One of the most important insights to emerge from the critical legal studies movement has been to
highlight the internal inconsistencies in law, the paradoxes, silences, and contingencies. This matches our experiences as women and feminists
engaged with law. It

is not an internally coherent system which operates smoothly to oppress us


at all times and in all ways. The 'power' of law is certainly present but not undifferentiated. It is uneven and it allows us space: space
to argue, to engage, and (in the active sense of the word) to ,resist'.' We have only to listen to our sisters in practice to learn this lesson in a very
pragmatic way. The operation of law involves a continual use of strategies, in which one is constantly balancing the -possibilities against the

it is a more than uneven fight. As law is constituted in a


society which still privileges sections of that society in terms of gender, race, class, and socially defined standards
of ability, so it is by no means a 'free space' for equal engagement. But neither is it closed
probabilities.' Again, let us be clear - in general,

space which we must continually struggle against rather than within.

When Smart states that 'the

entry of feminists into law has turned law into a site of struggle rather than being taken
only as a tool of struggle "' we would agree, but we would add that it is a site within which we find tools; it was never a tool because it
is not a single entity or practice. Indeed, one of our problems has been that (ironically) the sense of this is far more available in the practice of law than
in the academy. It

is in the academy that law has been (partly under the influence of the social sciences) most
frequently presented as an internally coherent, undifferentiated model which leaves one only with the
choice of submission or resistance in the narrow sense of the word. In practice we experience law as
more complex and hence more open to incursions.

The problem is that this experience of law must not only be

placed against a context of generally negative expectations and experiences (in other words, we must keep a perspective), but also that it is difficult to

there is still (properly) a tendency to focus on those areas of law which


operate most to women's detriment. This necessarily reinforces our sense of neglect and
exclusion. Secondly, the interstices of practice - not merely the operation of substantive law but the elements of process in
bargaining and negotiation - are less visible than the record of judgments in the higher courts or the
passing of legislation. Small (but significant) developments which we could characterize
record. We mean this in two ways. First,

as victories go unnoticed but radically improve the position of particular women in


particular ways . We might think here of developments requiring the sharp thinking of women solicitors and barristers in helping to devise
means by which rape victims might receive representation in criminal cases or strategies to protect the claims to ownership made by women who do
not hold legal title to property. This low-profile lawyering, reflected neither in case material nor academic texts, is a rich tapestry of strategies which

it leads us to
dismiss an entirely negative stance towards the tools of lawyering, whether in practice

often cross conventional legal categories." Of course this must be read against a generally negative background but, importantly,

or within the academy. Our stance then is a politics of engagement , informed and
considered, moving continually between the abstract and the specific. 2 The overwhelming tone of this
particular collection is with the problems we encounter when we engage in this way, problems which go beyond substantive law, to issues of procedure
and process and beyond that to the very languages with which we engage. 3 They also reflect issues relating to the diversity of women's position and
needs, 4 and to the general problems encountered by taking abstract stances, reliant upon concepts such as equality or rights. 5 In this sense, read as
a whole, the collection reflects the modus operandi of our claim to a particular type of theoretical work - the interface between the particular, concrete
circumstance, and the general, abstract model. It is that interface that we understand as 'strategy'.

AT: ontological power


Ontologizing sexual power and identity as a lens for worldly phenomena
reproduces binary discourses end elides difference, reproducing
heteronormativity.
Prasad 12
(Ajnesh, Australian School of Business, Beyond analytical dichotomies, Human Relations May
2012 vol. 65 no. 5 567-595)
poststructuralism may be typified by what Lyotard (1984)
famously expressed as its incredulity towards metanarratives (also see Alvesson and Deetz, 1999; Kilduff and
A poststructuralist critique At the most rudimentary level,

Mehra, 1997), or what Fraser and Nicholson (1997) describe as grand theorizing of social macrostructures. Akin to other critical traditions, it is explicitly historical, attuned to the
cultural specificity of different societies and periods and . . . inflected by temporality, with historically specific institutional categories (Fraser and Nicholson, 1997: 1434). The
aim of the poststructuralist mandate is to critique metanarratives and, from there, to define human consciousness and social existence through engagements with contextualized
subjectivity (Agger, 1991).11 To appreciate this idea, it is important to understand how metanarratives are problematically situated within, and are informed by, socially
constructed identity binaries, be they along the fault lines of gender (e.g. Butler, 1990; Hird, 2000), race (e.g. Gilroy, 2000; Miles and Torres, 2007), or sexuality (e.g. Zita, 1994).

poststructuralist critique, such as Derridas conjecture of difference (Mumby and Putnam, 1992), illustrates how the
preservation of the privileged identity (white, man, heterosexual) is existentially
dependent upon a corresponding relegated identity (black, woman, homosexual) (see Tyler
and Cohen, 2008); or, to posit it in Butlerian (1991) phrasing, heterosexuality presumes the being of homosexuality. Working from the same
current, Harding (2003) extends this idea through consideration of the central dyadic
relationship in management; she notes that every individual in the western workforce is identified as a
worker or as a manager and that the identity of the latter is wholly contingent upon the
Indeed,

binary existence of the former . Elsewhere, again assuming a poststructuralist position, Harding (2008: 44; emphasis in original) explains that
identities cannot be resolved into an essence or into a coherent whole; rather, they are
post hoc impositions of a seemingly unified [label] upon a disparate and disconnected
population. As such, the central aim of the poststructuralist is to repudiate deterministic and binary logic by drawing attention to the discursive processes that culturally
(re)produce social realities and the dichotomous modes of thinking embedded within them (Butler, 1990; Calas, 1993). [T]he ways in which sex was put into Western discourse
from the end of the 16th century onwards, the proliferation of sex during the 18th century and the modern incitement to discuss sex in endless detail have simultaneously

Tangentially related to the


Foucauldian reading of sexuality, Katz (2004) offers a genealogical investigation into how
heterosexuality was invented and came to be ideologically defined from the late 19th-century onwards. Katz writes that
because the concept of heterosexuality is only one particular historical way of perceiving,
categorizing, and imagining the social relations of the sexes, it ought to be studied with
the purposeful aim to dislocate its socio-cultural privilege as being the normal and the
natural form of sexual expression (p. 69). Given its socially manifest nature, Butler (1991, 1993) contends that heterosexuality is perpetually
established heterosexuality as the unassailable norm and constituted other sexualities as abnormal.

at risk and must continually engage in a set of repetitive, or what she calls parodic, practices such as, heterosexual sex which functions to stringently affirm the hegemonic
ideals of femininity (passive) and masculinity (active). Incidentally, the very redundancy of these parodic practices function to consolidate the discursive authority and cultural
stability of heterosexuality (Butler, 1991; see also Butlers [1993] writing on performativity). A related stream of poststructuralist-inflected scholarship reveals how

sexual

identities that are predicated on ontological sexual difference produce


heteronormativity, which can be described as the the normative idealization of heterosexuality (Hird, 2004: 27) or the
centrality of heterosexual norms in social relations (Pringle, 2008: S111). While feminists have long critiqued the tacit and the
explicit claims of ontological sexual difference, essentialist definitions of female and male continue to prevail
in popular culture and in certain academic disciplines (Frye, 1996).12 On this point, Hird (2004) adopts a position in feminist
science studies to develop a substantive critique into how the ontology of sexual difference is often rendered concrete in research propagated by the natural and particularly,

The influence of ontological sexual difference within and


outside of academia, lends credence to Broadbridge and Hearns (2008: S39) recent
the biological sciences (also see Martin, 1991).

observation that, [s]ex and sex differences are still often naturalized as fixed, or almost
fixed, in biology. It is equally important to note, here, that the alchemy of ontological sexual difference is
wholly dependent upon the patriarchal conflation of biological sex and cultural gender
(Hird, 2004). As Pringle (2008: S112; also see Borgerson and Rehn, 2004) notes, [g]ender [does] not avoid the oppositional
duality embodied in the concept of sex, but reflect[s] the interdependent relationship of
masculinity and femininity. This reflection pivots on genital determinism, which declares that males naturally embrace masculinity while females
naturally embrace femininity (Bornstein, 1994; Hird, 2000). This initial conflation of sex and gender leads to the conventional model of heterosexuality, which dictates that a man

It is precisely
these corresponding relationships whereby the heterosexual matrix is constructed
(Butler, 1990). According to Butler, this matrix serves as the grid of cultural intelligibility
through which bodies, gender, and desires are naturalized (see Ringrose, 2008: 511).13
will desire-to-be a male and will desire-for a female, while a woman will desire-to-be a female and will desire-for a male (Sinfield, 2002: 126).

Gender is not the root cause of environmental degradationsuch claims are


simplistic and wrong

Fox 98 Center for Environmental Studies


Fellow @ Cent. Env Studies, The Deep-Ecology-Ecofem debate, in Environmental
Philosophy ed. Zimmerman, p 232-3
To begin with, deep ecologists completely agree with ecofeminists that men have been

far more implicated in the history of ecological destruction than women. However,
deep ecologists also agree with similar charges derived from other social
perspectives: for example, that capitalists, whites, and Westerners have been far more
implicated in the history of ecological destruction than pre-capitalist peoples, blacks,
and non-Westerners .21 If ecofeminists also agree with these points, then the question arises
as to why they do not also criticize deep ecology for being neutral with respect to
issues concerning such significant social variables as socioeconomic class, race, and
Westernization. There appears to be two reasons for this. First, to do so would detract
from the priority that econfeminists wish to give to their own concern with
androcentrism. Second, and more significantly, these charges could also be applied
with equal force to the ecofeminist focus on androcentrism itself.14 How does one
defend the ecofeminist charge against deep ecology (i.e., that androcentrism is "the
real root" of ecological destruction) in the face of these charges?" For deep
ecologists, it is simplistic on both empirical and logical grounds to think that one
particular perspective on human society identifies the real root of ecological
destruction. Empirically, such thinking is simplistic (and thus descriptively poor)
because it fails to give due consideration to the multitude of interacting factors at
work in any given situation. (While on a practical level it can be perfectly reasonable to
devote most of one's energy to one particular 'cause-if only for straightforward reasons to
do with time and energy-that, of course, is no excuse for simplistic social theorizing.)

Such thinking fails, in other words, to adopt an ecological perspective with respect to
the workings of human society itself. Logically, such thinking is simplistic (and thus
facile) because it implies that the solution to our ecological problems is close at handall we have to do is remove "the real root" of the problem-when it is actually perfectly
possible to conceive of a society that is nonandrocentric, socioeconomically
egalitarian, nonracist, and nonimperialistic with respect to other human societies, but

whose members nevertheless remain aggressively anthropocentric in collectively


agreeing to exploit their environment for their collective benefit in ways that
nonanthropocentrists would find thoroughly objectionable. Indeed, the "green" critique of
socialism proceeds from precisely this recognition that a socially egalitarian society does not
necessarily imply an ecologically benign society.

Deliberation/Gender Consistent
Deliberation doesnt exclude gendered analysis
Dahlberg 5
(Lincoln, The University of Queensland, Center for Critical and Cultural Studies, Visiting Fellow,
The Habermasian public sphere: Taking difference seriously?, Theory and Society (2005)
34:111-126)
this critique of power, transparency, and the subject is largely based upon a poor characterization of
Habermas position. There are three main misunderstandings that need to be cleared up here, to do with power as negative, as able to be easily removed,
and as able to be clearly identied. First, Habermas does not dene power as simply negative and as therefore
needing to be summarily removed from the public sphere. The public sphere norm calls for
coercion-free communication and not power-free communication. Habermas
emphasizes the positive power of communicative interaction within the public sphere through which
participants use words to do things and make things happen.60 Communicative rationality draws on
the force of better argument to produce more democratic citizens, culture, and
societies. Subjects are indeed molded through this constituting power, but their
transformation is towards freedom and autonomy rather than towards subjugation and
normalization. As Jeffrey Alexander points out, to act according to a norm is not the same as to be
I believe

normalized .61 The public sphere norm provides a structure through which critical
reection on constraining or dominating social relations and possibilities for freedom can take
place . As Chambers argues, rational discourse here is about the endless questioning of codes, the reasoned
questioning of normalization.62 This is the very type of questioning critics like Lyotard, Mouffe, and Villa are
engaged in despite claiming the normalizing and repressive power of communicative
rationality. These critics have yet to explain adequately how they escape this performative
contradiction, although they may not be too concerned to escape it.63 The form of power that is to be excluded from
discourse in the public sphere is that which limits and disables democratic participation and
leads to communicative inequalities . Coercion and domination are (ideally) excluded
from the public sphere, which includes forms of domination resulting from the
maldistribution of material and authoritative resources that lead to discursive inequalities . This
emphasis on the ideal exclusion of coercion introduces the second point of clari- cation, that the domination free public sphere is an idealization for the purposes of critique.

Habermas is more than aware of the fact that, as Nancy Fraser, Mouffe, and Young remind us, coercive forms of power, including those that
result from social inequality, can never be completely separated from the public sphere.64 Claims that such power has
been removed from any really-existing deliberative arena can only be made by ignoring or hiding the operation of power. However, this does not
mean that a reduction in coercion and domination cannot be achieved. Indeed, this is precisely what a
democratic politics must do. To aid this project, the public sphere conception sets a
critical standard for evaluation of everyday communication . Chambers puts this nicely: Criticism
requires a normative backdrop against which we criticize. Crit-icizing the ways power and domination play themselves
out in discourse presupposes a conception of discourse in which there is no [coercive] power and domination. In other words, to defend the position that there is a mean- ingful
difference between talking and ghting, persuasion and coercion, and by extension, reason and power involves beginning with idealizations. That is, it involves drawing a picture
of undominated discourse.65 However, this discussion of the idealizing status of the norm does notanswer claims that it invokes a transparency theory of knowledge. Iwould
argue that such claims not only fall prey to another performa-tive contradiction of presupposing that the use of rational discourse can establish the impossibility of rational

In
contrast to the metaphysics of presence, the differentiation of persusion from coercion
discourse revealing truth and power but are also based on a poor reading of Habermas theory of communicative rationality. This is the third point of clarication.

in the public sphere does not posit a naive theory of the transparency of power, and
meaning more generally. The public sphere conception as based upon communicative
rationality does not assume a Cartesian ( autonomous, disembodied, decontextualized )
subject who can clearly distinguish between persuasion and coercion, good and bad reasons, true
and untrue claims, and then wholly re-move themselves and their communications from such
inuence. For Habermas, subjects are always situated within culture. The public sphere is
posited upon intersubjective rather than subject-centered rationality. It is through the
process of communicative rationality, and not via a Cartesian subject, that manipulation,
deception, poor reasoning, and so on, are identied and removed, and by which
meanings can be understood and communicated. In other words, it is through rational-critical
communication that discourse moves away from coercion or non-public reason towards greater rational
communication and a stronger public sphere. The circularity here is not a problem, as it may seem, but is in fact the very essence of
democratization: throughthe practice of democracy, democratic practice is advanced. This democratizing process can be further illustrated in the important and challenging case
of social inequalities. Democratic theorists (bothdeliberative and difference) generally agree that social inequalities al-ways lead to some degree of inequalities in discourse.
Thus, the ide-alized public sphere of full discursive inclusion and equality requires that social inequalities be eliminated. Yet how is social inequality to

The idealization seems wholly in-adequate given contemporary


capitalist systems and associated social inequality. However, it is in the very process of
befullyidentied,letaloneeliminated?

argumentation, even if awed, that the identication and critique of social inequality,
and thus of communicative inequality, is able to develop. I ndeed, public sphere deliberation often comes into existence
when and where people become passionate about social injustice and publicly thematize problems of social inequality. Thus the negative power of social inequality as with

This is not to say that subjects are merely


effects of discourse, that there are no critical social agents acting in the process. It is not to say that 125 subjects within discourse
cannot themselves identify negative forms of power, cannot reexively monitor their own arguments, cannot
rationally criticize other positions, and so on. They can, and in practice do, despite the
other forms of coercion is brought to light and critique by the very discourse it is limiting.

instability of meaning . The point is that this reasoning and understanding is (provisionally)
achieved through the subjects situatedness in discourse rather than via a pre-discursive abstract subject. As Kenneth
through discourse that subjects achieve a degree of reective distance (what we could call autonomy)
from their situations, enabling them to revise their conceptions of what is valuable or worthy of
pursuit,[and]to assess various courses of action with respect to those ends. 66 Democratic discourse
generates civic-oriented selves, inter-subjective meanings and understandings, and
democratic agreements that can be seen as the basis of public sovereignty. How-ever, the idea of
Baynes argues, it is

communicatively produced agreements, which in the public sphere are known as public opinions, has also come under ex-tensive criticism in terms of excluding difference,

The starting point of discourse is


disagreement over problematic validity claims. However, a certain amount of agreement,

criticism that I wantto explore in the next section. The ends of discourse: Public opinion formation

or at least mutual understanding, is presupposed when interlocutors engage in


argumentation. All communication presupposes mutual understanding on the linguistic
terms used that interlocutors use the same terms in the same way.67 Furthermore, in undertaking rational-critical discourse, according to
Habermas formal pragmatic reconstruction, interlocutors also presuppose the same formal conditions of
argumentation . These shared presuppositions enable rational-critical discourse to be
undertaken. However, as seen above, meaning is never fixed and understanding is always partial.
Understanding and agreement on the use of linguistic terms and of what it means to be reasonable, reflexive, sincere, inclusive, non-coercive, etc.
takes place within discourse and is an ongoing political process.

Public sphere not inevitably gendered---debates about public policy can


include embodied experience
Lincoln Dahlberg 5, The University of Queensland, Center for Critical and Cultural Studies,
Visiting Fellow, The Habermasian public sphere: Taking difference seriously?, Theory and
Society (2005) 34:111-126
this critique of power, transparency, and the subject is largely based upon a poor characterization of
Habermas position. There are three main misunderstandings that need to be cleared up here, to do with power as negative, as able to be easily removed,
and as able to be clearly identied. First, Habermas does not dene power as simply negative and as therefore
needing to be summarily removed from the public sphere. The public sphere norm calls for
coercion-free communication and not power-free communication. Habermas
emphasizes the positive power of communicative interaction within the public sphere through which
participants use words to do things and make things happen.60 Communicative rationality draws on
the force of better argument to produce more democratic citizens, culture, and
societies. Subjects are indeed molded through this constituting power, but their
transformation is towards freedom and autonomy rather than towards subjugation and
normalization. As Jeffrey Alexander points out, to act according to a norm is not the same as to be
I believe

normalized .61 The public sphere norm provides a structure through which critical
reection on constraining or dominating social relations and possibilities for freedom can take
place . As Chambers argues, rational discourse here is about the endless questioning of codes, the reasoned
questioning of normalization.62 This is the very type of questioning critics like Lyotard, Mouffe, and Villa are
engaged in despite claiming the normalizing and repressive power of communicative
rationality. These critics have yet to explain adequately how they escape this performative
contradiction, although they may not be too concerned to escape it.63 The form of power that is to be excluded from
discourse in the public sphere is that which limits and disables democratic participation and
leads to communicative inequalities . Coercion and domination are (ideally) excluded
from the public sphere, which includes forms of domination resulting from the
maldistribution of material and authoritative resources that lead to discursive inequalities . This
emphasis on the ideal exclusion of coercion introduces the second point of clari- cation, that the domination free public sphere is an idealization for the purposes of critique.

Habermas is more than aware of the fact that, as Nancy Fraser, Mouffe, and Young remind us, coercive forms of power, including those that
result from social inequality, can never be completely separated from the public sphere.64 Claims that such power has
been removed from any really-existing deliberative arena can only be made by ignoring or hiding the operation of power. However, this does not
mean that a reduction in coercion and domination cannot be achieved. Indeed, this is precisely what a
democratic politics must do. To aid this project, the public sphere conception sets a
critical standard for evaluation of everyday communication . Chambers puts this nicely: Criticism
requires a normative backdrop against which we criticize. Crit-icizing the ways power and domination play themselves
out in discourse presupposes a conception of discourse in which there is no [coercive] power and domination. In other words, to defend the position that there is a mean- ingful
difference between talking and ghting, persuasion and coercion, and by extension, reason and power involves beginning with idealizations. That is, it involves drawing a picture
of undominated discourse.65 However, this discussion of the idealizing status of the norm does notanswer claims that it invokes a transparency theory of knowledge. Iwould
argue that such claims not only fall prey to another performa-tive contradiction of presupposing that the use of rational discourse can establish the impossibility of rational

In
contrast to the metaphysics of presence, the differentiation of persusion from coercion
in the public sphere does not posit a naive theory of the transparency of power, and
meaning more generally. The public sphere conception as based upon communicative
discourse revealing truth and power but are also based on a poor reading of Habermas theory of communicative rationality. This is the third point of clarication.

rationality does not assume a Cartesian (autonomous, disembodied, decontextualized)

subject who can clearly distinguish between persuasion and coercion, good and bad reasons, true
and untrue claims, and then wholly re-move themselves and their communications from such
inuence. For Habermas, subjects are always situated within culture. The public sphere is
posited upon intersubjective rather than subject-centered rationality. It is through the
process of communicative rationality, and not via a Cartesian subject, that manipulation,
deception, poor reasoning, and so on, are identied and removed, and by which
meanings can be understood and communicated. In other words, it is through rational-critical
communication that discourse moves away from coercion or non-public reason towards greater rational
communication and a stronger public sphere. The circularity here is not a problem, as it may seem, but is in fact the very essence of
democratization: throughthe practice of democracy, democratic practice is advanced. This democratizing process can be further illustrated in the important and challenging case
of social inequalities. Democratic theorists (bothdeliberative and difference) generally agree that social inequalities al-ways lead to some degree of inequalities in discourse.
Thus, the ide-alized public sphere of full discursive inclusion and equality requires that social inequalities be eliminated. Yet how is social inequality to

The idealization seems wholly in-adequate given contemporary


capitalist systems and associated social inequality . However, it is in the very process of
befullyidentied,letaloneeliminated?

argumentation, even if awed, that the identication and critique of social inequality,
and thus of communicative inequality, is able to develop.

Indeed, public sphere deliberation often comes into existence

when and where people become passionate about social injustice and publicly thematize problems of social inequality. Thus the negative power of social inequality as with

This is not to say that subjects are merely


effects of discourse, that there are no critical social agents acting in the process. It is not to say that 125 subjects within discourse
cannot themselves identify negative forms of power, cannot reexively monitor their own arguments, cannot
rationally criticize other positions, and so on. They can, and in practice do, despite the
other forms of coercion is brought to light and critique by the very discourse it is limiting.

instability of meaning . The point is that this reasoning and understanding is (provisionally)
achieved through the subjects situatedness in discourse rather than via a pre-discursive abstract subject. As Kenneth
through discourse that subjects achieve adegree of reective distance (what we could call autonomy)
from their situations, enabling them to revise their conceptions of what is valuable or worthy of
pursuit,[and]to assess various courses of action with respect to those ends. 66 Democratic discourse
generates civic-oriented selves, inter-subjective meanings and understandings, and
democratic agreements that can be seen as the basis of public sovereignty. How-ever, the idea of
Baynes argues, it is

communicatively produced agreements, which in the public sphere are known as public opinions, has also come under ex-tensive criticism in terms of excluding difference,

The starting point of discourse is


disagreement over problematic validity claims. However, a certain amount of agreement,

criticism that I wantto explore in the next section. The ends of discourse: Public opinion formation

or at least mutual understanding, is presupposed when interlocutors engage in


argumentation. All communication presupposes mutual understanding on the linguistic
terms used that interlocutors use the same terms in the same way.67 Furthermore, in undertaking rational-critical discourse, according to
Habermas formal pragmatic reconstruction, interlocutors also presuppose the same formal conditions of
argumentation . These shared presuppositions enable rational-critical discourse to be
undertaken. However, as seen above, meaning is never fixed and understanding is always partial.
Understanding and agreement on the use of linguistic terms and of what it means to be reasonable, reflexive, sincere, inclusive, non-coercive, etc.
takes place within discourse and is an ongoing political process.

AT: Patriarchy R/C


Patriarchys not the root cause
Bell 6
(Duncan, senior lecturer Department of Politics and International Studies @ Cambridge
University, Beware of false prophets: biology, human nature and the future of International
Relations theory, International Affairs 82, 3 p. 493510)
Fukuyama, tireless promulgator of the end of history and now a member of the Presidents Council on Bioethics,
employed EP reasoning to argue for the central role in world politics of masculine values, which are rooted in
Writing in Foreign Aff airs in 1998, Francis

biology. His argument starts with the claim that male and female chimps display asymmetric behaviour, with the males far more prone to violence and domination. Female
chimps have relationships; male chimps practice realpolitik. Moreover, the line from chimp to modern man is continuous and this has signifi cant consequences for international

He argues that the world can be divided into two spheres, an increasingly peaceful
and cooperative feminized zone, centred on the advanced democracies, and the brutal world outside this insulated space,
where the stark realities of power politics remain largely masculine. This bifurcation heralds dangers, as masculine policies are essential in
politics.46

dealing with a masculine world: In anything but a totally feminized world, feminized policies could be a liability. Fukuyama concludes the essay with the assertion that the form
of politics best suited to human nature issurprise, surprisefree-market capitalist democracy, and that other political forms, especially those promoted by feminists and

the authority of science is invoked in order to


naturalize a particular political objective. This is a pattern that has been repeated across the history of
modern biology and remains potent to this day.48 It is worth noting in brief that Fukuyamas argument is badly
flawed even in its own terms. As anthropologist R. Brian Ferguson states, Fukuyamas claims about the
animal world display a breathtaking leap over a mountain of contrary evidence.49 Furthermore,
Joshua Goldstein concludes in the most detailed analysis of the data on war and gender that
although biological differences do play a minor role, focusing so heavily on them is profoundly
misleading.50 The simplistic claims, crude stereotyping and casual use of evidence that
characterize Fukuyamas essay unfortunately recur throughout the growing literature on the biology of international politics.
socialists, do not correspond with our biological inheritance.47 Once again

Relationships are complex and multiple types of feminism disproves their


argument
Crenshaw2
(Carrie, PhD, Perspectives In Controversy: Selected Articles from CAD, Scholar)
Increasingly, students rely on feminist authors to inform their
such arguments only exemplify the
general absence of sound causal reasoning in debate rounds. Poor causal reasoning results from a debate practice that privileges
empirical proof over rhetorical proof, fostering ignorance of the subject matter being debated. To illustrate my point, I claim that debate arguments about
feminists suffer from a reductionism that tends to marginalize the voices of significant feminist authors. David
Zarefsky made a persuasive case for the value of causal reasoning in intercollegiate debate as far back as 1979. He argued that causal arguments are
desirable for four reasons. First, causal analysis increases the control of the arguer over
events by promoting understanding of them. Second, the use of causal reasoning increases
rigor of analysis and fairness in the decision-making process. Third, causal arguments promote understanding
of the philosophical paradox that presumably good people tolerate the existence of evil.
Finally, causal reasoning supplies good reasons for "commitments to policy choices or to
systems of belief which transcend whim, caprice, or the non-reflexive "claims of
Feminism is not dead. It is alive and well in intercollegiate debate.

analysis of resolutions. While I applaud these initial efforts to explore feminist thought, I am concerned that

immediacy" (117-9). Rhetorical proof plays an important role in the analysis of causal relationships. This is true despite the common assumption that the identification
of cause and effect relies solely upon empirical investigation. For Zarefsky, there are three types of causal reasoning. The first type of causal reasoning describes the application
of a covering law to account for physical or material conditions that cause a resulting event This type of causal reasoning requires empirical proof prominent in scientific
investigation. A second type of causal reasoning requires the assignment of responsibility. Responsible human beings as agents cause certain events to happen; that is,
causation resides in human beings (107-08). A third type of causal claim explains the existence of a causal relationship. It functions "to provide reasons to justify a belief that a
causal connection exists" (108). The second and third types of causal arguments rely on rhetorical proof, the provision of "good reasons" to substantiate arguments about
human responsibility or explanations for the existence of a causal relationship (108). I contend that the practice of intercollegiate debate privileges the first type of causal
analysis. It reduces questions of human motivation and explanation to a level of empiricism appropriate only for causal questions concerning physical or material conditions.
Arguments about feminism clearly illustrate this phenomenon. Substantive debates about feminism usually take one of two forms. First, on the affirmative, debaters argue that
some aspect of the resolution is a manifestation of patriarchy. For example, given the spring 1992 resolution, "[rjesolved: That advertising degrades the quality of life," many
affirmatives argued that the portrayal of women as beautiful objects for men's consumption is a manifestation of patriarchy that results in tangible harms to women such as rising
rates of eating disorders. The fall 1992 topic, "(rjesolved: That the welfare system exacerbates the problems of the urban poor in the United States," also had its share of patriarchy cases. Affirmatives typically argued that women's dependence upon a patriarchal welfare system results in increasing rates of women's poverty. In addition to these
concrete harms to individual women, most affirmatives on both topics, desiring "big impacts," argued that the effects of patriarchy include nightmarish totalitarianism and/or
nuclear annihilation. On the negative, many debaters countered with arguments that the some aspect of the resolution in some way sustains or energizes the feminist
movement in resistance to patriarchal harms. For example, some negatives argued that sexist advertising provides an impetus for the reinvigoration of the feminist movement
and/or feminist consciousness, ultimately solving the threat of patriarchal nuclear annihilation. likewise, debaters negating the welfare topic argued that the state of the welfare
system is the key issue around which the feminist movement is mobilizing or that the consequence of the welfare system - breakup of the patriarchal nuclear family -undermines

Such arguments seem to have two assumptions in common. First, there is a


single feminism. As a result, feminists are transformed into feminism. Debaters speak of feminism as a single, monolithic,
theoretical and pragmatic entity and feminists as women with identical m otivations, methods, and goals. Second, these
arguments assume that patriarchy is the single or root cause of all forms of oppression.
patriarchy as a whole.

Patriarchy not only is responsible for sexism and the consequent oppression of women, it also is the cause of totalitarianism, environmental degradation, nuclear war, racism,

These reductionist arguments reflect an unwillingness to debate about the


complexities of human motivation and explanation. They betray a reliance upon a framework of proof that can explain only
and capitalist exploitation.

material conditions and physical realities through empirical quantification. The transformation of feminists to feminism and the identification of patriarchy as the sole cause of all
oppression is related in part to the current form of intercollegiate debate practice. By "form," I refer to Kenneth Burke's notion of form, defined as the "creation of appetite in the
mind of the auditor, and the adequate satisfying of that appetite" (Counter-Statement 31). Though the framework for this understanding of form is found in literary and artistic
criticism, it is appropriate in this context; as Burke notes, literature can be "equipment for living" (Biilosophy 293). He also suggests that form "is an arousing and fulfillment of
desires. A work has form in so far as one part of it leads a reader to anticipate another part, to be gratified by the sequence" (Counter-Statement 124). Burke observes that there
are several aspects to the concept of form. One of these aspects, conventional form, involves to some degree the appeal of form as form. Progressive, repetitive, and minor
forms, may be effective even though the reader has no awareness of their formality. But when a form appeals as form, we designate it as conventional form. Any form can
become conventional, and be sought for itself - whether it be as complex as the Greek tragedy or as compact as the sonnet (Counter-Statement 126). These concepts help to
explain debaters' continuing reluctance to employ rhetorical proof in arguments about causality.

Debaters practice the convention of

poor causal reasoning as a result of judges' unexamined reliance upon conventional form. Convention is the practice of arguing single-cause links to monolithic
impacts that arises out of custom or usage. Conventional form is the expectation of judges that an argument will take this form. Common practice or convention dictates that a
case or disadvantage with nefarious impacts causally related to a single link will "outweigh" opposing claims in the mind of the judge. In this sense, debate arguments

Debaters practice the convention of establishing single-cause


relationships to large monolithic impacts in order to conform to audience expectation. Debaters practice poor causal reasoning because
themselves are conventional.

they are rewarded for it by judges. The convention of arguing single-cause links leads the judge to anticipate the certainty of the impact and to be gratified by the sequence. I
suspect that the sequence is gratifying for judges because it relieves us from the responsibility and difficulties of evaluating rhetorical proofs. We are caught between our
responsibility to evaluate rhetorical proofs and our reluctance to succumb to complete relativism and subjectivity. To take responsibility for evaluating rhetorical proof is to admit

when we abandon our responsibility to rhetorical proofs,


we sacrifice our students' understanding of causal reasoning. The sacrifice has
consequences for our students' knowledge of the subject matter they are debating. For
example, when feminism is defined as a single entity, not as a pluralized movement or theory, that single entity results in the identification of patriarchy
as the sole cause of oppression. The result is ignorance of the subject position of the
particular feminist author, for highlighting his or her subject position might draw attention to the incompleteness of the causal relationship between link
and impact Consequently, debaters do not challenge the basic assumptions of such
argumentation and ignorance of feminists is perpetuated. Feminists are not feminism. The topics of feminist inquiry are
that not every question has an empirical answer. However,

many and varied, as are the philosophical approaches to the study of these topics. Different authors have attempted categorization of various feminists in distinctive ways. For
example, Alison Jaggar argues that feminists can be divided into four categories: liberal feminism, marxist feminism, radical feminism, and socialist feminism. While each of
these feminists may share a common commitment to the improvement of women's situations, they differ from each other in very important ways and reflect divergent
philosophical assumptions that make them each unique. Linda Alcoff presents an entirely different categorization of feminist theory based upon distinct understandings of the
concept "woman," including cultural feminism and post-structural feminism. Karen Offen utilizes a comparative historical approach to examine two distinct modes of historical
argumentation or discourse that have been used by women and their male allies on behalf of women's emancipation from male control in Western societies. These include
relational feminism and individualist feminism. Elaine Marks and Isabelle de Courtivron describe a whole category of French feminists that contain many distinct versions of the
feminist project by French authors. Women of color and third-world feminists have argued that even these broad categorizations of the various feminism have neglected the
contributions of non-white, non-Western feminists (see, for example, hooks; Hull; Joseph and Lewis; Lorde; Moraga; Omolade; and Smith). In this literature, the very definition of
feminism is contested. Some feminists argue that "all feminists are united by a commitment to improving the situation of women" (Jaggar and Rothenberg xii), while others have
resisted the notion of a single definition of feminism, bell hooks observes, "a central problem within feminist discourse has been our inability to either arrive at a consensus of

The controversy over the


very definition of feminism has political implications. The power to define is the power
both to include and exclude people and ideas in and from that feminism. As a result, [bjourgeois white
opinion about what feminism is (or accept definitions) that could serve as points of unification" (Feminist Theory 17).

women interested in women's rights issues have been satisfied with simple definitions for obvious reasons. Rhetorically placing themselves in the same social category as
oppressed women, they were not anxious to call attention to race and class privilege (hooks. Feminist Wieory 18). Debate arguments that assume a singular conception of
feminism include and empower the voices of race- and class-privileged women while excluding and silencing the voices of feminists marginalized by race and class status. This

when we examine the second assumption of arguments about feminism in intercollegiate debate - patriarchy is the
sole cause of oppression. Important feminist thought has resisted this assumption for good
reason. Designating patriarchy as the sole cause of oppression allows the subjugation of
resistance to other forms of oppression like racism and classism to the struggle against
sexism. Such subjugation has the effect of denigrating the legitimacy of resistance to
racism and classism as struggles of equal importance. "Within feminist movement in the West, this led to the
assumption that resisting patriarchal domination is a more legitimate feminist action
than resisting racism and other forms of domination" (hooks. Talking Back 19). The relegation of struggles against racism
and class exploitation to offspring status is not the only implication of the "sole cause" argument In addition, identifying patriarchy as the single
source of oppression obscures women's perpetration of other forms of subjugation and
domination, bell hooks argues that we should not obscure the reality that women can and do partici- pate in politics of domination, as perpetrators as well as victims that we dominate, that we are dominated. If focus on patriarchal domination masks this reality or becomes the means by
which women deflect attention from the real conditions and circumstances of our lives, then women cooperate in suppressing and
promoting false consciousness, inhibiting our capacity to assume responsibility for
transforming ourselves and society (hooks. Talking Back 20). Characterizing patriarchy as the sole
cause of oppression allows mainstream feminists to abdicate responsibility for the
exercise of class and race privilege. It casts the struggle against class exploitation and racism as secondary concerns. Current debate
practice promotes ignorance of these issues because debaters appeal to conventional form, the expectation of judges that they will isolate a single link to a large impact
position becomes clearer

Feminists become feminism and patriarchy becomes the sole cause of all evil. Poor causal arguments arouse and fulfill the expectation of judges by allowing us to surrender our

The result is either the mar-ginalization or


colonization of certain feminist voices. Arguing feminism in debate rounds risks trivializing feminists. Privileging the act
of speaking about feminism over the content of speech "often turns the voices and
beings of non-white women into commodity, spectacle" (hooks, Talking Back 14). Teaching
sophisticated causal reasoning enables our students to learn more concerning the
subject matter about which they argue. In this case, students would learn more about the
multiplicity of feminists instead of reproducing the marginalization of many feminist
voices in the debate itself. The content of the speech of feminists must be investigated to subvert the colonization of exploited women. To do so, we
responsibility to evaluate rhetorical proof for complex causal relationships.

must explore alternatives to the formal expectation of single-cause links to enormous impacts for appropriation of the marginal voice threatens the very core of selfdetermination and free self-expression for exploited and oppressed peoples. If the identified audience, those spoken to, is determined solely by ruling groups who control
production and distribution, then it is easy for the marginal voice striving for a hearing to allow what is said to be overdetermined by the needs of that majority group who
appears to be listening, to be tuned in (hooks, Talking Back 14). At this point, arguments about feminism in intercollegiate debate seem to be overdetermined by the expectation
of common practice, the "game" that we play in assuming there is such a thing as a direct and sole causal link to a monolithic impact To play that game, we have gone along
with the idea that there is a single feminism and the idea that patriarchal impacts can account for all oppression. In making this critique, I am by no means discounting the
importance of arguments about feminism in intercollegiate debate. In fact, feminists contain the possibility of a transformational politic for two reasons. First, feminist concerns
affect each individual intimately. We are most likely to encounter patriarchal domination "in an ongoing way in everyday life. Unlike other forms of domination, sexism directly
shapes and determines relations of power in our private lives, in familiar social spaces..." (hooks. Talking Back 21). Second, the methodology of feminism, consciousnessraising, contains within it the possibility of real societal transformation. "lE]ducation for critical consciousness can be extended to include politicization of the self that focuses on
creating understanding the ways sex, race, and class together determine our individual lot and our collective experience" (hooks, Talking Back 24). Observing the incongruity
between advocacy of single-cause relationships and feminism does not discount the importance of feminists to individual or societal consciousness raising.

Perm
Their advocacy is not exclusive with the 1AC. The judge can endorse the
affirmative and the negatives methods of resistance to oppression as two
separate, worthwhile methods.
Kathleen Higgins, University of Texas-Austin, Philosophy Professor, Winter 2013, Post-Truth
Pluralism: The Unlikely Political Wisdom of Friedrich Nietzche, Kindle
Progressives are right that we live increasingly in a post-truth era, but rather than rejecting it and pining
nostalgically for a return to a more truthful time, we should learn to better navigate it. Where the New York Times and Walter
Cronkite were once viewed as arbiters of public truths, today the Times competes with the Wall Street Journal, and CBS News with FOX News and

The diversity of
viewpoints opened up by new media is not going away and is likely to intensify. This diversity of interpretations of
reality is part of a longstanding trend. Democracy and modernization have brought a proliferation of
worldviews and declining authority of traditional institutions to meanings. Citizens have
more freedom to create new interpretations of facts. This proliferation of viewpoints makes
the challenge of democratically addressing contemporary problems more complex. One
consequence of all this is that our problems become more wicked and more subject to
conflicting meanings and agendas. We cant agree on the nature of problems or their
solutions because of fundamentally unbridgeable values and worldviews . In attempting
to reduce political disagreement to black and white categories of fact and fiction, progressives
themselves uniquely ill-equipped to address our current difficulties, or to advance liberal values in the
culture. A new progressive politics should have a different understanding of the truth than the one
suggested by the critics of conservative dishonesty. We should understand that human beings make meaning
and apprehend truth from radically different standpoints and worldviews, and that our great wealth
and freedom will likely lead to more, not fewer, disagreements about the world. Nietzsche was no democrat, but the
pluralism he offers can be encouragement to todays political class, as well as the rest of us, to
become more self-aware of, and honest about, how our standpoint, values, and power affect our
determinations of what is true and what is false. In the posttruth era, we should be able to
MSNBC, in describing reality. The Internet multiplies the perspectives and truths available for public consumption.

articulate not one but many different perspectives. Progressives seeking to govern and
change society cannot be free of bias, interests, and passions, but they should strive to be aware of
them so that they can adopt different eyes to see the world from the standpoint of their
fiercest opponents. Taking multiple perspectives into account might alert us to more
sites of possible intervention and prime us for creative formulations of alternative
possibilities for concerted responses to our problems. Our era, in short, need not be an
obstacle to taking common action. We might see todays divided expert class and fractions public
not as temporary problems to be solved by more reason, science, and truth, but rather as permanent
features of our developed democracy. We might even see this proliferation of belief systems
and worldviews as an opportunity for human development. We can agree to disagree and
still engage in pragmatic action

in the World.

AT: Gender Monolithic


While gender has a large impact it isnt monolithic, nor unified
Hooper 1
(Charlotte, University of Bristol research associate in politics, Manly States: Masculinities,
International Relations, and Gender Politics pp 45-46.)
Spike Peterson and Anne Sisson Runyan (1993), in their discussion of gendered dichotomies, appear to drop Lacanian psychoanalytic discourse as an explanation for
gendered dichotomies in favor of a more straightforward- ly political account.14Gendered dichotomies, rather than uniformly con- structing gendered social relations through
universal psychoanalytic mecha- nisms, are seen more ambiguously, as playing a dual role. Where gendered dichotomies are used as an organizing principle of social life
(such as in the gendered division of labor) they help to construct gender differences and in- equalities and thus are constitutive of social reality, but in positing a grid of polar
opposites, they also serve to obscure more complex relationships, commonalties, overlaps, and intermediate positions (Peterson and Runyan 1993, 2425). Elaborating on
this view, it can be argued that gendered dichotomies are in part ideological tools that mystify, masking more complex social realities and reinforcing stereotypes. On one level,
they do help to produce real gen- der differences and inequalities, when they are used as organizing principles that have practical effects commensurate with the extent that
they become embedded in institutional practices, and through these, human bodies. They constitute one dimension in the triangular nexus out of which gender identities and

institutional practices are not always completely or


unambiguously informed by such dichotomies, which may then operate to obscure more
complex relationships. It is a mistake to see the language of gendered dichotomies as a
unied and totalizing discourse that dictates every aspect of social practice to the extent
that we are coherently produced as subjects in its dualistic image. As well as the
disruptions and discontinuities engendered by the intersections and interjections of
other discourses (race, class, sexuality, and so on) there is always room for evasion,
reversal, resistance, and dissonance between rhetoric, practice, and embodiment, as well
as reproduction of the symbolic order, as identities are negotiated in relation to all three
dimensions, in a variety of complex and changing circumstances. On the other hand, the symbolic gender order
the gender order are produced. But at the same time,

does inform practice, and our subjectivities are produced in relation to it, so to dismiss it as performing only an ideological or propagandistic role is also too simplistic.

State/Reform Good

Legal/State Key
Abdication of legal reformism makes concrete challenges to material
barriers to feminism impossible
Crawford 7
(1-1-2007 Toward a Third-Wave Feminist Legal Theory: Young Women, Pornography and the
Praxis of Pleasure Bridget J. Crawford Pace University School of Law,
bcrawford@law.pace.edu)
CONCLUSION In spite of third-wave feminisms appeal, at this point in its development,

third-wave feminism lacks an overall theoretical

view of how the law functions. Third-wave feminism is largely a reactive critique that fails to
advance its own positivistic view of how certain goals should be accomplished . Third-wave feminists respond
to incomplete and distorted images of second-wave feminism. Their indictment of second-wave feminism has led to a significant tension between older and younger feminists. Gloria Steinem, for one, has said
that when reading third-wave feminist writings, she feels "like a sitting dog being told to sit."384 Women on the younger cusp of second-wave feminism, who demographically are not part of the third wave, report
that they feel adrift between the competing "waves."385 And even some younger women, perhaps articulating the most decidedly third-wave stance of all, state that they do not want selfidentity as part of a "third-

thirdwave feminism is a helpful elaboration of some of the issues first raised by earlier
feminists, but that it is not so decidedly different from what has come before. Third-wave feminism's emphasis on personal pleasure, the fluidity of gender roles, the internet and coalition-building
contribute to the feminist conversation, but third-wave feminists have not yet altered the terms and conditions of
wave" of feminism, because that identification implies a group affiliation or branding that should be rejected in favor of a third-wave embrace of individualism. So one is left with the sense that

that conversation . It remains for lawyers and legal theorists to take up the challenge from this
generation of young women to develop laws that enhance womens autonomy and wellbeing.

Feminist critiques of the state get coopted to justify neoliberal


deregulation---engagement is key
Gupta 12
(Rahila, freelance journalist and writer, Has neoliberalism knocked feminism sideways?,
https://www.opendemocracy.net/5050/rahila-gupta/has-neoliberalism-knocked-feminismsideways)
Feminism needs to recapture the state from the neoliberal project to which it is in hock in order to make it
deliver for women. It must guard against atomisation and recover its transformative aspirations to
shape the new social order that is hovering on the horizon, says Rahila Gupta How should feminists read our current times? A major economic crisis
rocks the developed world. While austerity measures dont appear to be working across Europe, the mildly Keynesian efforts of Obama to kick-start the US economy have had
only a marginal effect. The Occupy movement has gone global and the public disorder in the summer, with more disorder being predicted by the police, are an indication of deep
discontent with the system. Yet we have seen an enthusiastic and vibrant third wave of youthful feminism emerge in the past decade. At the rate at which these waves arise, it
will be some time before the rock of patriarchy will be worn smooth. The current phase of capitalism neo-liberalism which began with Thatcher and Reagan in the 1970s,
promotes privatisation and deregulation in order to safeguard the freedom of the individual to compete and consume without interference from a bloated state. According to
David Harvey, a Marxist academic, the world stumbled towards neo-liberalism in response to the last major recession in the 70s when the uneasy compact between capital and
labour brokered by an interventionist state broke down. The UK government, for example, was obliged by the International Monetary Fund to cut expenditure on the welfare
state in order to balance the books. The post-war settlement had given labour more than its due, and it was time for the upper classes to claw these gains back. The fact that

feminism served to
legitimate a structural transformation of capitalist society . I am with Nancy Fraser in so far as she says that there is a
convergence, a coinciding of second wave feminism and neo-liberalism, even that feminism thrived in these conditions. It is well known that in an attempt to
renew and survive, capitalism co-opts the opposition to its own ends. If part of the
second wave feminism and neoliberalism flourished from the 1970s onwards has led some to argue, notably Nancy Fraser, that

project of neoliberalism is to shrink the size of the state, it serves its purpose to co-opt
the feminist critique that the state is both paternalistic and patriarchal. Critiques of the nanny state from the
right may chime with feminist concerns. However, the right has little to say about patriarchy. What is left out of the co-option process is equally significant. The
critique of the state mounted by feminists such as Elizabeth Wilson when state capitalism was at the
height of its powers suited neoliberal capitalists seeking deregulation and a reduced role
for the state. Frasers analysis does not explain the current resurgence of feminism at a time when the shine of neoliberalism has faded. It is not so much that
feminism legitimised neoliberalism, but that neoliberal values created a space for a bright, brassy and ultimately fake feminism - the I really, really want girl-power ushered in by
the Spice Girls. This transitional period between second wave and the current wave of feminism (which some commentators characterised as post-feminist) represented the
archetypal appropriation of the feminist agenda, shorn of its political context, by neoliberalism. Incidentally, many of us rejected the label post-feminist because it felt like an
attempt to chuck feminism into the dustbin of history and to deny the continuing need for it. In hindsight, there was something different going on in that lull between the two

If the culture of neoliberalism had


something to offer women, it was the idea of agency, of choice freely exercised, free even of
patriarchal restraints. It emphasised self-sufficiency of the individual while at the same time
undermining those collective struggles or institutions which make self-sufficiency
possible. The world was your oyster all you needed to do was compete successfully in the marketplace. The flexible worker, in order to make herself acceptable to the
waves in the 70s and 80s and today; the voice of feminism was being drowned out by its loud, brassy sisters.

world of work, may even go so far as to remodel herself through cosmetic surgery, all the while under the illusion that she was in control of her life. In her essay on Feminism in

liberal capitalism is committed to what she calls


the fetishism of choice . If women choose things that disadvantage them and entrench
differences, it legitimates inequality because the inequality arises from the choices they
make. The few women who do well out of the sex industry do not believe that their work
entrenches inequality because it is freely chosen, because prostitution is seen as a
liberation from the drudgery of cleaning jobs. Choice is their weapon against feminist objections. In their so-called free
expression of their sexuality, they are challenging nothing in the neoliberal schema because
the work reduces women to the status of meat and commodity.
a forthcoming book, Oxford Handbook of Political Ideologies, Clare Chambers argues that

State Inevitable
State influence inevitable---only mobilizing focus on reforms can effectively
challenge patriarchy
Connell 90
(R. W., The State, Gender, and Sexual Politics: Theory and Appraisal, Theory and Society, Vol.
19, No. 5, (Oct., 1990), pp. 507-544, http://www.jstor.org/stable/657562)
the state is a major stake in gender politics; and the exercise of
that power is a con- stant incitement to claim the stake . Thus the state becomes the
Because of its power to regulate and its power to create,

focus of interest-group formation and mobilization in sexual politics. It is worth recalling just how wide the
liberal state's activity in relation to gender is. This activity includes family policy, population policy, labor force and labor market management, housing
policy, regulation of sexual behavior and expression, provision of child care, mass educa- tion, taxation and income redistribution, the creation and use of mili- tary forces - and that is not the whole of it. This is not
a sideline; it is a major realm of state policy.

Control of the machinery that conducts these activities is a massive

asset in gender politics . In many situations it will be tactically decisive . The state is
therefore a focus for the mobilization of interests that is central to gender politics on the
large scale . Feminism's historical con- cern with the state, and attempts to capture a
share of state power, appear in this light as a necessary response to a historical reality . They are
not an error brought on by an overdose of liberalism or a capitula- tion to patriarchy . As
the state is unavoidable for feminism. The question is not whether feminism will
deal with the state, but how: on what terms, with what tactics, toward what goals.5" The
same is true of the politics of homosexuality among men. The ear- liest attempts to agitate for toleration produced a half-illegal, half-aca- demic mode
Franzway puts it,

of organizing that reached its peak in Weimar Germany, and was smashed by the Nazis. (The Institute of Sexual Science was vandalized and its library burnt in 1933; later, gay men were sent to concentration

The
gay liberation movement changed the methods and expanded the goals to include social
revolution, but still dealt with the state over policing, de-criminalization, and antidiscrimination. Since the early 1970s gay politics has evolved a complex mixture of confron- tation,
cooperation, and representation. In some cities, including San Francisco and Sydney, gay men as such have successfully run for public office. Around
the AIDS crisis of the 1980s, in countries such as the United States and Australia, gay community based organizations and state
health services have entered a close - if often tense - long-term relationship.' In a longer historical perspective,
all these forms of politics are fairly new. Fantasies like Aristophanes's Lysistrata aside, the open mobiliza- tion of
groups around demands or programs in sexual politics dates only from the midnineteenth century. The politics that characterized other patriarchal gender orders in history were constructed along other lines, for instance as a politics of kinship, or faction formation
in agri- cultural villages. It can plausibly be argued that modern patterns re- sulted from a reconfiguration of gender
politics around the growth of the liberal state. In particular its structure of legitimation
through plebiscite or electoral democracy invited the response of popular mobilization
camps or shot.) A long period of lobbying for legal reform followed, punctuated by bouts of state repression. (Homosexual men were, for instance, targeted in the McCarthyite period in the United States.)

State influence is inevitable but depth of oppression matters---reform is


effective and only way to solve---they exaggerate states internal coherence
Connell 90

(R. W., The State, Gender, and Sexual Politics: Theory and Appraisal, Theory and Society, Vol.
19, No. 5, (Oct., 1990), pp. 507-544, http://www.jstor.org/stable/657562)
Appraisals

Is the state patriarchal? Yes, beyond any argument, on the evidence dis- cussed above. It is not

"essentially patriarchal " or "male"; even if one could speak of the "essence" of a social
institution, this would exaggerate the internal coherence of the state . Rather the state is
historically patriarchal, patriarchal as a matter of concrete social practices . State structures in recent history institutionalize
the European equation be- tween authority and a dominating masculinity; they are effectively con- trolled by men; and they operate with a massive bias towards hetero- sexual men's interests. At the
same time the pattern of state patriarchy changes. In terms of the depth of oppression
and the historical possibilities of resistance and transformation, a fascist regime is
crucially different from a liberal one, and a liberal one from a revolutionary one. The
most favorable histori- cal circumstance for progressive sexual politics seems to be the
early days of social-revolutionary regimes; but the later bureaucratization of these regimes is devastating. Next best is a
liberal state with a reformist government; though reforms introduced under its aegis are
vulnerable in periods of reaction. Though the state is patriarchal, progressive gender politics cannot
avoid it . The character of the state as the central institutionalization of power , and its
historical trajectory in the regulation and constitution of gender relations, make it
unavoidably a major arena for challenges to patriarchy . Here liberal feminism is on
strong ground . Becoming engaged in practical struggles for a share of state power
requires tactical judgments about what developments within the state provide
opportunities. In the 1980s certain strategies of reform have had a higher relative pay-off than
they did before. In Australia, for instance, the creation of a network of "women's services" was a feature of the 1970s, and the momentum of this kind of action has died away. Reforms that
have few budgetary implications but fit in with other state strategies, such as modernizing the bureaucracy, become more promi- nent. Equal employment opportunity
and anti-discrimination legisla- tion have been highlighted; decriminalizing
homosexuality is consistent with this.

Reform Better
State influence is inevitable but depth of oppression matters---reform is
effective and only way to solve---they exaggerate states internal coherence
Connell 90
(R. W. Connell 90, The State, Gender, and Sexual Politics: Theory and Appraisal, Theory and
Society, Vol. 19, No. 5, (Oct., 1990), pp. 507-544, http://www.jstor.org/stable/657562)
Appraisals

Is the state patriarchal? Yes, beyond any argument, on the evidence dis- cussed above. It is not

"essentially patriarchal " or "male"; even if one could speak of the "essence" of a social
institution, this would exaggerate the internal coherence of the state . Rather the state is
historically patriarchal, patriarchal as a matter of concrete social practices . State structures in recent history
institutionalize the European equation be- tween authority and a dominating masculinity; they are effectively con- trolled by men; and they operate with a massive bias towards

At the same time the pattern of state patriarchy changes. In terms of the
depth of oppression and the historical possibilities of resistance and transformation, a
fascist regime is crucially different from a liberal one, and a liberal one from a
revolutionary one. The most favorable histori- cal circumstance for progressive sexual
politics seems to be the early days of social-revolutionary regimes; but the later bureaucratization of these
regimes is devastating. Next best is a liberal state with a reformist government; though reforms
introduced under its aegis are vulnerable in periods of reaction. Though the state is patriarchal,
progressive gender politics cannot avoid it . The character of the state as the central
hetero- sexual men's interests.

institutionalization of power , and its historical trajectory in the regulation and


constitution of gender relations, make it unavoidably a major arena for challenges to
patriarchy . Here liberal feminism is on strong ground . Becoming engaged in practical
struggles for a share of state power requires tactical judgments about what
developments within the state provide opportunities. In the 1980s certain strategies of reform
have had a higher relative pay-off than they did before. In Australia, for instance, the creation of a network of "women's
services" was a feature of the 1970s, and the momentum of this kind of action has died away. Reforms that have few budgetary implications but fit in with other state strategies,

Equal employment opportunity and anti-discrimination


legisla- tion have been highlighted; decriminalizing homosexuality is consistent with
this.
such as modernizing the bureaucracy, become more promi- nent.

Our Strat Best


Thats the best starting point struggles for political power in the name of
narrow goals sanitizes the desire for dominance and arrest radical
potential. The thought experiment of recognizing the interconnectedness
of political violence is key
Wang 14
(Hongyu, Ph.d., Professor, STCL - Curriculum Studies, 20, A Nonviolent Perspective on
Internationalizing Curriculum Studies, International Handbook of Curriculum Research, Ch. 5
Routledge)
In the first edition of this Handbook. William K Pinar (2003) discusses the importance of focusing on education and curriculum, rather than international political tensions, for the
internationalization of curriculum studies. If we have scholars acting as if diplomatic representatives of their own countries, the intellectual and educational possibility will be lost
in power struggles. Actually,

in political and social movements, the egocentric pursuit of political

authority and control, either for an individual or for a group, can hardly lead to any
success , (lhandi (1942/2007) specifically points out that the nonviolence movement is "not a program of
seizure of power" but "a program of transformation of relationships" (p. 40). In the Liberian women's peace
movement in 2003, they adopted the strategy of not criticizing the political policies of the dictatorshipeven though there were more than plenty to criticizebut demanding of
peace unyieldingly and wholeheartedly (Disney & Ritickcr, 2(108; Cibowce, 2011).

Paradoxically, the key to winning social and

political victories in nonviolence movements is to abandon the politics of power


struggle and instead to mobilize every participant in the powerful process of
transforming the nature of relationships from dominating/being dominated to organic
interconnectedness. If we cannot go beyond the confinement of national, group, or
individual self-interest, there is no possibility of achieving "heart unity" with others who arc
distant or/and different from us. Here it is essential not only to dwell in international space, but also to move towards transnational space. The
inter-space and trans-space are both important for creating nonviolent dynamics of the local, the national. and the global through
transforming relationships. The term "international" acknowledges the "in between" fluid spaces
where multiplicity and differences are neither excluded nor self-contained. Moreover,
internationalization as a conceptsupports the decentering of both the national and the global
through a focus on interaction and relationship that lead to the transformation of both
locality and globalness. To borrow the language of chaos and complexity theory (Doll. 20121, the newness of the global comes from a dynamic interaction
of local parts. Also as Peter Hershock (2009) argues, it is a fallacy lo assume that "whatever is good for each and every one of us (individually) will be good for all of us
(communally or ecologically)" (p. 156) since what is good for the local may become detrimental to the ecological or the global. Therefore, the global us the whole is more than
the addition of (he national or the local, but emerges from interactive dynamics and is marked by organic relationality. Noel Gough (2003) suggests that

internationalizing curriculum inquiry might best be understood as a process of creating transnational


spaces in which scholars from different localities collaborate in reframing and decentering their own
knowledge traditions and negotiate trust in each other's contributions to their collective
work" (p. 6S). The very usage of "trans-"' indicates both an intense experiencing of (the boundary and an effort to go beyond that boundary. Such
transnational spaces not only sustain hybrid movements but also support embodied work to
negotiate collaborative trust. Nonviolence education must be an embodied process. Sherry B. Shapiro (2002) asserts that it is the joy and suffering
"

of the human body that extends "beyond the boundaries of nationality, rice, ethnicity, gender, social class, or sexual or religious preferenceall the ways of marking ourselves

Peace and nonviolence education need to sensitize us to the collective


body, and pedagogically we need to begin with the body as the connector between the
public and the private, and between social identity and a wider shared experience. In such
off from others" (p. I4U).

dynamics of international and transnational movements, identity is destabilized, power struggles are
displaced into fluid modes of relationships, and nonviolent relationality across
differences become multidimensionalboth horizontal (among the local) and vertical (between the local and the global l. and both topdown (from the global to the individual body) and bottom-up (from the local to the international)to form a network of nonviolence. Instead of intensifying
the fragmentation (due to dualism) that marks the fragility of the modern life we share, the nonviolent modes of relationalily
we choose to establish can contribute to the integrative potential of the network. For the dynamics
of intergroup relationships within the national, I reference the American field of curriculum studies as an example due to my familiarity with it. Pinar (2013) identifies "power,
identity, and discourse" as the key concepts of the reconceptualized curriculum field in the United States, hut he suggests that these concepts have become assumptionsdue
to their successand that these newly taken-for-granted concepts have tendencies toward totalization and reductionism. Now

the assumption that

"power predominates, that identity is central, and that discourse is determinative (e.g. our research provides only narratives, never
truth)are widely shared" (p. 8). Accepted as given , they have become "abstractions split off
from the concrete complexity of the historical moment " <p. S) and exhausted in self-referentiality. Ironically, the
central emphasis of identity leads to the casually of individual agency and subjective
specificity. As both an observer and participant of the American field of curriculum studies who came from China in 1996, I also would like to add another causality:
organic relationality. The complexity and richness in the singularity of each individual or group
coexists with the complicated and organic relationality of humanity and life, and when one side of the
coin is undermined, ihe other side deteriorates as well. While Pinar (2013) discusses the proliferation of "uncertainty" and "dispersion" in post structural dis-courses and their
elicits. I also think ihe distance between self and other stretched by the post-structural discourses of otherness and the unknown Other may lead to the difficulty of nm being able
to bring self and other back into the fabric of relationality (Wang, in press). In addressing "difference-centered politics of recognition and respect." drawing upon the Buddhist
philosophy. Peter Hershock I20O9) argues for "a concerted shift from considerations of how much we are the same or different from each |sic | another to how we might best
differ for one another" (p. 1611: emphasis in original). In a nondualistic. nonviolent view, subject and object, body and mind, and self and other exist interdependent!).

Hershock's perception of differences as essential for mutual contribution and shared welfare, as
something positive that should not be erased or elevated, but as a part of a relationship network, is a
challenge not only to the liberal notion of the individual as autonomous, but also to the identity politics of static diversity or
the postmodern radicalization of singularity. The nonviolent relational dynamics of
"differing for" rather than "differing from" are particularly imperative under the context
of a profoundly shared sense of crisis in American public education. While particular
differences such as racial or gendered differences must he discussed, the discussions
need to orient towards changing our ways of relating to others and addressing the root course of social violence,
rather than fixing on any particular social identity. Nonviolence cannot exist without social justice, but social justice for one
I
suggest that challenging the violence of the conservative forces and working through the
depressive position of educators in relation to the external attack from non-education sectors, we are called to form
nonviolent relationships among different social groups and their affiliated scholarly
camps. Identity-based struggles, when contextualized in the interconnected web of life,
have played a progressive role in the field. However, without contextualizing and complicating
one's own investment in a broader project of education for all, without taking a step
group at the expense of the welfare of others docs not do justice to the shared human struggle for the common good of all. Confronting the crisis in American public education,

back from one's own particular subjective positioning to see a bigger picture, any fixation upon one
group's strugglealong or within the lines of cither nice, gender, class, sexuality, nation,
or other social factorsat the expense of the collective good arrests democracy as an
unfulfilled dream, If we can initiate and participate in nonviolent dynamics of "differing
for" an educationally informed, compassion-ale community across local and national borders, we are also challenging the
international domination of American politics , along with its domestically repressive educational "reform" demand for raising test
scores and maintaining global control.

This suggestion is certainly not about subsuming diversity into

uniformity , as any network has room for breaks and fragmentations. The organic relationalty of nonviolence
welcomes differences and does not avoid conflicts because it has the ability to stretch, transform, and rebuild. Moving from the
national to the international level, the dualism of "us" versus "them" has played a violent role

, and the possibility of moving beyond such a fixed boundary depends upon
our capacity for refusing to dehumanize the other, both the friendly other and the hostile
in global relationships

other . Through the psychoanalytic notion of "the stranger to ourselves." Julia Kristeva (1993) invites us "to recognize ourselves as strange in order better to appreciate the
foreigners outside us instead of striving to bend them to the norms of our own repression" (p. 29). If we are aware of our subconscious rather than repressing it. aliens are no
longer a threat to us. Kristeva believes that a transnational or international position is situated at the crossing of boundaries, which simultaneously affirms and transcends
national borders. The idea of nation "at the same time affirmed as a space of freedom and dissolved in its own identity" (p. 32) affirms both the protective function of
identification and the necessity of border-crossing. Situated at the fluid border, "nations without nationalism" support nonviolent relationality.

AT Specific Strategies

Alt no solve
The alternative cant solve access but a focus on legalism does
Schwartz9
(Joseph, Poli Sci Prof @ Temple, The Future of Democratic Equality, Routledge, pg 64-5)
'Discursive' performance is not the sole manner by which individuals deal with (and express) the
material and cultural structural realities that both empower and constrain individuals . For
example. individuals cannot readily "discursively perform" themselves out of their socioeconomic or class position. There is a certain materiality to poverty or to being "bossed"
that can't simply be "ironically" and "performatively" transformed. Class relations are structural, as well as
discursive. The greater difficulty in forming unions in the United States- as compared to other advanced industrial democracieshas much to do with American legal, ideological, and political constraints and not simply with the
rel- ative inefficacy of the "performative," "counter-hegemonic" behavior of (frag- mented)
individuals. Even the "parodic" possibilities of "gender" reversal are constrained by the
communities in which one resides. ls the "reversal" of "drag" a viable public possibility
in a violently homophobic community? Were not the "performative" options of a Matthew
Sheperd (extremely) more limited than those of a gay or lesbian student at a "progressive"
residential liberal arts college (and unsafe-and even degrading and violent-social spaces
confront gay and lesbian people and women and students of color in the most allegedly
"cosmopolitan" of social spaces). Simply put, distinct "social spaces" set differ- ential
constraints on "performative" choices.

Of course, how individuals express class, race, gender, and sexuality does, in part, involve how

we "perform" (or "racist") cultural and discursive "norms." Hence, the inevitable controversies over "authenticity" within racial, sexual, and ethnic communities, as well as

there are
material constraints to performative "choice": one can't "perform" one's way out of an
criticism of people taking on the mores of a class different from those who share their "place" in the labor process, neighborhood, or income strata. But

under-funded inner city school or out of being a laid-off auto worker with dim prospects
of finding a new job with comparable wages and benefits. Traditional sociological
theories of "structuration" provide greater insight into how these individuals would deal
with these social dilemmas than do micro-level theories of the discursive construction
of subjectivity.

To her credit, Wendy Brown is more concerned with issues of class and political economy than are many post-structuralist political theorists. She

expressly claims to bring class back into her political analysis and condemns identity poli- tics as a "phantasmagorical reflection of the 'middle-class' American dream." But

there is little attention in her work to developing a political strategy that could promise a structural and
material redistribution of power, rather than an alter- ation of how we think of epistemology,
discourse, and politics." While ideology and culture play a relatively autonomous role in constituting subjectivity, both have a material structure that must
be altered if society is to be democratized. Brown implies that radical social change does not as much involve
democratiz- ing social structural relations as it does popularizing a radical
epistemological approach to discourse. Brown argues that if we will ourselves to "surrender epis- temological foundations" and give up
"specifically moral claims" we will all be able to engage in "the sheerly political: 'wars of position' and amoral contests about the just and good in which truth is always grasped
as coterminous with power, as always already power, as the voice of power."""' Even if one resists asking whether democracy can rest on "amoral" principles, one can still ask

The post-structuralist
hyper-emphasis on "discourse" and the agonal construction of the self also overly devalues the state as an arena
whether Brown's Foucauldian assertation that power and truth are co-terminous can distinguish between more or less democratic forms of power?

for political reform. Brown's work makes a positive political contribution by warning social movements about fetishizing the struggle for group rights within
the law as potential minefields of "reversed" power/knowledge formations. State regulation and technocratic control which claim to defend the interests of newly, legally-

recognized identities may yield the perverse consequence of "domesticating" the identity of the insurgent social group (e.g. state micro-management of the work place in
"comparable worth legislation," or enforcement of patriarchal values in regard to punitive workfare or "child support" regulation)?" Sometimes, as Brown contends, new-found
rights may enhance separation and alienation between and within individuals and groups, as well as constitute new forms of state regulation in the name of the impersonal

Brown rejects the possibility (and historical reality) that new "rights" can, in other contexts,
con- tribute to human emancipation by enhancing individual choice and freedom. To deny this
is to ignore the elective affinity between the struggle for "rights" and struggles to
achieve political equality for formerly subordinate peoples. Not all new-found rights are "co-optative" and a
"reinscribing of domination.""7 Nor will the conflict within the American polity over how we should interpret and defend "rights" ever cease. One only has to
witness contemporary political con- conflict over "abortion rights," "voting rights," "gun rights," etc.
Rights are both politically contested and protective of certain forms of human choice and
agency. Rights do not "fix" identities as intransigently as Brown and other post-structuralists claim. Do rights only serve, as
Brown contends, to promote "the discursive denial of historically layered and institutionally
secured bounds, by denying with words the effects of relatively wordless, politically
invisible, yet material constraints"?"" Patri- cia Williams and other critical race theorists have
argued that being included under the state's equal protection law helped limit violence against people
of color." Despite legitimate fears about excessive state regulation of sexuality, would Brown
reject the use of state force to limit domestic violence? How does her philosophical fear of the
bureaucratic-regulatory powers of the state speak to the experience of hundreds of thousands of women
who have been spared the "privatization'' of domestic violence by the extension of the
rights of state author- ity (e.g. the police) to act against violence within the household? Are such prac- tices
solely evidence of the "reconstruction of domination by the regulation of the technocratic-bureaucratic state"? Of course, state regulation of domestic
violence may, in Brown's language, produce a female subject "dependent upon the pater- nal
state" for protection. But is this not preferable to the prior form of paternal state that let a man
subject. But

be the violent definer of "rights" in his home?

Alt Fails Strategy Key


Alternative alone cant solve focus on tactics and strategy key
Saloom 6
(Rachel, JD Univ of Georgia School of Law and M.A. in Middle Eastern Studies from U of
Chicago, Fall 2006, A Feminist Inquiry into International Law and International Relations, 12
Roger Williams U. L. Rev. 159, Lexis)
Because patriarchy is embedded within society, it is no surprise that the theory and practice of both international law and
international relations is also patriarchal. 98 Total

critique, however, presents no method by which to


challenge current hegemonic practices. Feminist scholars have yet to provide a coherent
way in which total critique can be applied to change the nature of international law and
international relations. Some [*178] feminist scholars are optimistic for the possibility of changing the way the current
system is structured. For example, Whitworth believes that "sites of resistance are always available to those who oppose the status
quo." 99 Enloe suggests that since the world of international politics has been made it can also be remade. 100 She posits that
every time a woman speaks out about how the government controls her, new theories are being made. 101 All

of these
theorists highlight the manner in which gender criticisms can destabilize traditional
theories. They provide no mechanism , however, for the actual implementation of their
theories into practice. While in the abstract, resistance to hegemonic paradigms seems like a promising concept, gender
theorists have made no attempt to make their resistance culminate in meaningful change. The notion of rethinking
traditional approaches to international law and international relations does not go far enough in
prescribing an alternative theoretical basis for understanding the international arena.
Enloe's plea for women to speak out about international politics does not go nearly far
enough in explaining how those acts could have the potential to actually change the
practice of international relations. Either women are already speaking out now, and their
voices alone are not an effective mechanism to challenge the system, or women are not even
speaking out about world politics currently. Obviously it is absurd to assume that women remain silent about world politics. If that is
the case, then one must question women's ability to speak up, challenge, and change the system.

AT: Negativity Alt


Optimism and solidarity are our only hope---their pessimism accepts the
foundational premises of patriarchy/racism as its starting point for politics
Hooks 96
(bell hooks 96, Killing Rage: Ending Racism, Google Books, 269-272)
black Americans are succumbing to and internalizing the racist
assumption that there can be no meaningful bonds of intimacy between blacks and
whites. It is fascinating to explore why it is that black people trapped in the worst situation of racial oppres sionenslavementhad the foresight to see that it would be
269More than ever before in our history,

disempowering for them to lose sight of the capacity of white people to transform themselves and divest of white supremacy, even as many black folks today who in no way

black folks, like their


white counterparts, have passively accepted the internalization of white supremacist
assumptions. Organized white supremacists have always taught that there can never be
trust and intimacy between the superior white race and the inferior black race. When
suffer such extreme racist oppression and exploitation are convinced that white people will not repudiate racism. Con temporary

black people internalize these sentiments, no resistance to white supremacy is taking


place; rather we become complicit in spreading racist notions . It does not matter that so many black people feel
white people will never repudiate racism because of being daily assaulted by white denial and refusal of accountability. We must not allow the actions of white folks who blindly
endorse racism to determine the direction of our resistance. Like our white allies in struggle we must consistently keep the faith, by always sharing the truth that 270white

Of course many white people are


comfortable with a rhetoric of race that suggests racism cannot be changed, that all white people
are inherently racist simply because they are born and raised in this society. Such misguided thinking socializes white
people both to remain ignorant of the way in which white supremacist attitudes are learned and to assume a posture of learned
people can be anti-racist, that racism is not some immutable character flaw.

helplessness as though they have no agencyno capacity to resist this thinking. Luckily we have many autobiographies by white folks committed to
anti-racist struggle that provide documentary testimony that many of these individuals repudiated racism when they were children. Far from passively
accepting It as inherent, they instinctively felt it was wrong. Many of them witnessed bizarre acts of white racist aggression towards black folks in
everyday life and responded to the injustice of the situation. Sadly, in our times so many white folks are easily convinced by racist whites and bLack
folks who have internalized racism that they can never be really free of racism. These feelings aso then obsc]re the reality of white privi lege. As long
as white folks are taught to accept racism as natura] then they do not have to see themselves as con sciously creating a racist society by their
actions, by their political choices. This means as well that they do not have to face the way in which acting in a racist manner ensures the maintenance
of white privilege. Indeed, denying their agency allows them to believe white privilege does not exist even as they daily exercise it. If the young white
woman who had been raped had chosen to hold all black males account able for what happened, she would have been exercising white privilege and
reinforcing the structure of racist thought which teaches that all black people are alike. Unfortunately, 271so many white people are eager to believe
racism cannot be changed because internalizing that assumption downplays the issue of accountability. No responsibility need be taken for not
changing something fit is perceived as immutable. To accept racism as a system of domination that can be changed would demand that everyone who
sees him- or herself as embracing a vision of radai social equality would be required to assert anti-racist habits of being. We know from histories both
present and past that white people (and everyone else) who commit themselves to living in anti-racist ways need to make sacrifices, to courageously

Whites, people of color, and black folks are reluctant to


commit themselves fully and deeply to an anti-racist struggle that is ongoing because there is such a
pervasive feeling of hopelessnessa conviction that nothing will ever change . How any
of us can continue to hold those feelings when we study the history of racism in this
society and see how much has changed makes no logical sense. Clearly we have not
gone far enough. In the late sixties, Martin Luther King posed the question Where do we go from here. To live in anti-racist society we
must collectively renew our commitment to a democratic vision of racial justice and
equality. Pursuing that vision we create a culture where beloved community flourishes
and is sustained. Those of us who know the joy of being with folks from all walks of life, all races, who are fundamentalls anti-racist in their
endure the uncomfortable to challenge and change.

habits of being. need to give public testimony. Ve need to share not only what we have experienced but the conditions of change that make such an
experience possible. The interracial circle of love that I know can happen because each individual present in it has made his or her own commitment to
living an anti- racist life and to furthering the struggle to end white supremacy 272 will become a reality for everyone only if those of us who have
created these communities share how they emerge in our lives and the strategies we use to sustain them. Our devout commitment to building diverse

communities is cen tral. These commitments to anti-racist living are just one expression of who we are and what we share with one an other but they
form the foundation of that sharing. Like all beloved communities we affirm our differences. It is this generous spirit
of affirmation that gives us the courage to challenge one another, to work through misunderstandings, especially those that have to do with race and
racism. In

a beloved community solidarity and trust are grounded in profound commitment


to a shared vision. Those of us who are always anti-racist long for a world in which evezyone can form a beloved community where
borders can be crossed and cultural hybridity celebrated. Anyone can begin to make such a community by truly seeking to live in an anti-racist world.

If that longing guides our vision and our actions, the new culture will be born and antiracist communities of resis tance will emerge everywhere. That is where we must go
from here.

AT: Individual Experience Method


They presume the purity of their experience in place of facts---we must
examine multiple perspectives
Conway 97
(philosophy, Penn State, Daniel, Nietzsche and the political, 135-6)
This preference is clearly political in nature

, and Haraway makes no pretense of aspiring to epistemic purity or foundational innocence. For Haraway,

any

epistemic privilege necessarily implies a political (i.e., situated) preference


All perspectives are partial, all
standpoints situated
It is absolutely crucial
that we
acknowledge claims about situated knowledge as themselves situated within the
political agenda
feminist s must
accept
the self-referential
implications of their own epistemic claims The political agenda of
assigns
to some) subjugated standpoints a political preference or priority.

. Her postmodern orientation

elides the boundaries traditionally drawn between politics and epistemology, and thus renders otiose the ideal of epistemic purity.
including those of feminist theorists.

to Haraway's postmodern feminist project

her

she sets for postmodern feminism;

theorist

therefore

and accommodate

postmodern feminism thus

Haraway, for example, believes that some subjugated

standpoints may be more immediately revealing, especially since they have been discounted and excluded for so long. They may prove especially useful in coming to understand the political and psychological mechanisms whereby the
patriarchy discounts the radically situated knowledges of others while claiming for its own (situated) knowledge an illicit epistemic privilege:

The standpoints of the subjugated ... are savvy to modes of denial through repression,

forgetting, and disappearing acts ways of being nowhere while claiming to sec comprehensively. The subjugated have a decent chance to be on to the god-trick and all its dazzlingand, therefore, blindingilluminations.34

But these subjugated standpoints do not afford


view of the world,
against the

serious

feminist theorists

an epistemically privileged

independent of the political agendas they have established. Reprising elements of Nietzsche's psychological profile of the "slave" type,

danger of

romanticizing and/or

Haraway warns

appropriating the vision of the less powerful while

claiming to see from their positions. To see from below is neither easily learned nor
unproblematic , even if "we" "naturally" inhabit the great underground terrain of
subjugated knowledges The positionings of the subjugated are not exempt from
.

critical re-examination
The standpoints of
the subjugated are not "innocent" positions A subjugated standpoint may shed new
light on the ways of an oppressor, but it in no way renders superfluous
the
standpoint of the oppressor . Because neither standpoint fully comprises the other the
aggregation of the two would move both parties
closer to a more objective
understanding of the world If some
have political reasons for disavowing this
project of aggregation, or for adopting it selectively, then they must pursue their
, decoding, deconstruction, and interpretation; that is, from both semiological and hermeneutic modes of critical enquiry.

.35

or redundant

(or a third party)

feminists

political agenda at the expense of the greater objectivity that they might otherwise
have gained

Micro focus fails


Cuomo 11
(Chris Professor of Philosophy and Women's Studies, and an affiliate faculty member of the Environmental Ethics Certificate
Program and the Institute for African-American Studies. The author and editor of many articles and several books in feminist,
postcolonial, and environmental philosophy, Cuomo served as Director of the Institute for Women's Studies from 2006-2009. Her
book, The Philosopher Queen, a reflection on post-9/11 anti-war feminist politics, was nominated for a Lambda Award and an APA

book award, and her work in ecofeminist philosophy and creative interdiciplinary practice has been influential among those seeking
to bring together social justice and environmental concerns, as well as theory and practice. She has been a recipient of research
grants from the Rockefeller Foundation, the National Science Foundation, the Ms. Foundation, the National Council for Research on
Women, and the Institute for Sustainability and Technology Policy, and she has been a visiting faculty member at Cornell University,
Amherst College, and Murdoch University in Perth, Australia, Climate Change, Vulnerability, and Responsibility, Hypatia 26 no4
Fall 2011 p. 690-714, AM)

individual and household reductions in greenhouse-gas


pollution will be effective only if they are deep and widespread, and only if they are
accompanied by meta-level efforts, but meta-level policies and corporate practices seem
unlikely to emerge without significant support from below . Addressing climate change
through mitigation and transnational funding for adaptation requires administrative action in the form of binding
treaties, laws and regulations, taxes, incentives for technological development, and
increased international aid, but such policies and practices require mass popular support. An unfair
and possibly unmanageable degree of practical responsibility therefore falls on citizens and
consumers, who may turn out to be ineffective as political actors because of the problems of insufficiency and disempowerment, among other
Due to the scale of change that is needed,

things. Nonetheless, if national and corporate policies will not go in a more sustainable direction without a great swell of public support in places like
the United States, then it is ethically and practically necessary that the significant minority who hopes to effectively address the problem of climate
change find ways to build that support.

It would be tragic if increasing disempowerment fueled by wellmessaging were to magnify political ineffectiveness among environmentalists
and global human rights advocates by making it more attractive to focus on personal or
private-sphere changes, rather than investing time or energy in work for change at higher
levels. Perhaps money and energy otherwise spent on highpriced home retrofitting or demanding lifestyle changes should be aimed directly toward
intentioned green

growing movements that increase green consciousness and political influence and that effectively demand full corporate responsibility for pollution. If
such efforts were to result in a few very significant policy changes, such as a global moratorium on gas flaring or a greening of the military, the payoff in
terms of long-term mitigation could be great. Such successes could in turn energize cultural shifts toward more effective alternative technologies. What
can a well-organized collection of people who care accomplish through democratic politics and cultural transformation? Can advocates for
environmental integrity and human rights better help us all to effectively reduce greenhouse gas emissions in due time? The problem of climate change
provides opportunities to foster regenerating movements toward more sustainable and humane futures, and so inevitably some will step up and take

Given the urgency created by the industrial


greenhouse effect, an ethically motivated minority must effectively act on their caring
while also making it contagious through the creation of a more effective political will . The
insufficiency problem might be reduced if those who care about climate change and
climate justice channel their mitigation efforts more effectively to influence decisionmakers and policies at higher levels, where actions can be carried out with significant
and immediate effects on emission levels and matters of social justice. If more corporate
and governmental actors are pressured (or inspired) to take responsibility for the causes of climate
change, their decisions and innovations can in turn create more options for carbon-free
lifestyles, which will also help reduce the insufficiency and disempowerment problems for average consumers. The knowledge we need to avert
a more extreme climate disaster already exists, in many places and in multiple forms. Those who care about humanity and
Earths green growing mantle of life need the power to turn dominant practices and policies
toward better futures. Grand successes along those lines are needed very soon.
responsibility for addressing the problem. Could they possibly succeed?

AT: Affirmation of Self


We have to extend politics beyond the realm of our immediate experience
and the confines of the debate space---structural analysis key
Rob 14
(Carleton College, Robtheidealist, My Skinfolk Ain't All Kinfolk,
www.orchestratedpulse.com/2014/03/problem-identity-politics/)
identity politics merely means
political activity that caters to the interests of a particular social group. In a certain
sense, all politics are identity politics. However, its one thing to intentionally form a
group around articulated interests; its another matter entirely when group membership
is socially imposed. Personal identities are socially defined through a combination of systemic
rewards/marginalization plus actual and/or potential violence. We cant build politics from that
foundation because these socially imposed identities dont necessarily tell us anything
about someones political interests. Successful identity politics requires shared
interests, not shared personal identities. Im not here to tell you that personal identity
doesnt matter; we rightfully point out that systemic power shapes peoples lives. Simply put, my
message is that personal identity is not the only thing that matters. We spend so much energy labeling people
privileged/marginalized, oppressor/oppressedthat we often neglect to build spaces
Some people look at these flaws and call for an end to identity politics, but I think thats a mistake. At its most basic level,

that antagonize the systems that cause our collective trauma . All You Blacks Want All the Same Things We assume that if a
person is systemically marginalized, then they must have a vested interest in dismantling that system. Yet, thats not always the case. Take Orville Lloyd Douglas, who last summer wrote an article in the Guardian
in which he admitted that he hates being Black. I can honestly say I hate being a black male I just dont fit into a neat category of the stereotypical views people have of black men. I hate rap music, I hate most
sports, and I like listening to rock music I have nothing in common with the archetypes about the black male I resent being compared to young black males (or young people of any race) who are lazy, not

membership in a marginalized group


is no guarantee that a person can understand and effectively combat systemic
oppression. Yet, we seem to treat all marginalized voices as equal, as if they are all
insightful, as if there is no diversity of thought, as ifin the case of race All you Blacks
want all the same things. Shared identity does not equal shared interests. John Ridley, the Oscar-winning
disciplined, or delinquent. Orville Lloyd Douglas, Why I Hate Being a Black Man As we can see from Douglas cry for help,

screenplay writer of 12 Years a Slave, is a good example. Hes written screenplays based on Jimi Hendrix, the L.A. riots, and other poignant moments and icons within Black history. He wants to see more Black
people in Hollywood and he has a long history of successfully incorporating Black and Brown characters into comic book stories and franchises. However, in 2006, Ridley made waves with an essay in which he
castigated Black people who did not live up to his standards; saying, Its time for ascended blacks to wish niggers good luck. So I say this: Its time for ascended blacks to wish niggers good luck. Just as whites
may be concerned with the good of all citizens but dont travel their days worrying specifically about the well-being of hillbillies from Appalachia, we need to send niggers on their way. We need to start extolling the
most virtuous of ourselves. It is time to celebrate the New Black Americansthose who have sealed the Deal, who arent beholden to liberal indulgence any more than they are to the disdain of the hard Right. It
is time to praise blacks who are merely undeniable in their individuality and exemplary in their levels of achievement. The Manifesto of Ascendancy for the Modern American Nigger While Ridley and I share
cultural affinity, and we both want to see Black people doing well, shared cultural affinity and common identity are not enough which recent history makes abundantly clear. Barack Obama continues to deport

Don Lemon, speaking in support of Bill


OReilly, said that racism would be lessened if Black people pulled up their pants and
stopped littering. Last fall, 40% of Black U.S. Americans supported airstrikes against
Syria. My skinfolk aint all kinfolk, and the Left needs to catch up. NO MORE ALLIES John Ridley, Barack Obama, myself, and Don Lemon are all Black males. We also have conflicting political positions
and interests, but how can we decide which paths are valid if we only pay attention to personal identity? Instead of learning to recognize how the
overarching systems maintain their power and then attacking those tools, we spend our
energy finding an other to embody the systemic marginalization and legitimize our
spaces and ideals. In some interracial spaces I feel like nothing more than an
interchangeable token whose only purpose is to legitimize the politics of my White peers.
If not me, then some other Black person would fill the slot. We use these others as authorities on various issues, and we
use concepts like privilege to ensure that people stay in their lanes. People of color are the authorities on race,
while LGBTQ people are the authorities on gender and sexuality, and so forth and so on. Yet, experience is not the same as expertise, and
record numbers of Brown immigrants here at home, while mercilessly bombing Brown folks abroad.

privilege doesnt automatically make you clueless . As Ive discussed, these groups are not oriented
around a singular set of political ideals and practices. Furthermore, as we see in Andrea Smiths work, there are often
competing interests within these groups . We mistake essentialism for intersectionality
as we look for the ideal subjects to embody the various forms of oppression; true intersectionality is a
description of systemic power, not a call for diversity. If we dont develop any substantive analysis of systemic power ,
then its impossible to know what our interests are, and aligning with one another
according to shared interests is out of the question. In this climate all that remains is the
ally, which requires no real knowledge or political effort, only the willingness to appear
supportive of an other. We cant build power that way. After having gathered to oppose
organized White supremacy at the University of North Carolina, a group of organizers in
Durham, North Carolina found that the Lefts emphasis on personal identity and allyship was a major
reason why their efforts collapsed. They proposed that we adopt the practice of forming
alliances rather than identifying allies . (h/t NinjaBikeSlut) Much of the discourse around being an ally seems to presume a relationship of one-sided support,
In an alliance,
the two parties support each other while maintaining their own self-determination and
autonomy, and are bound together not by the relationship of leader and follower but by a
shared goal. In other words, one cannot actually be the ally of a group or individual with
whom one has no political affinity and this means that one cannot be an ally to an
entire demographic group, like people of color, who do not share a singular cohesive
political or personal desire. The Divorce of Thought From Deed While its vital for me to learn the politics and
history of marginalized experiences that differ from my own, listen to their voices, and respect their spaces and contributions
its also important for me to understand the ways in which these same systems have
shaped my own identity/history as well. Since we know that oppression is systemic and multidimensional, then Im
with one person or group following anothers leadership. While there are certainly times where this makes sense, it is misleading to use the term ally to describe this relationship.

going to have to step outside of personal experience and begin to develop political
ideals and practices that actually antagonize those systems . I have to understand and
articulate my interests, which will allow me to operate from a position of strength and
form political alliances that advance those interests interests which speak to issues
beyond just my own immediate experience .

AT: Lesbian Separatism


The alt essentializes identity reproduces heteronormativity that
implicates every level of their argument
Prasad 12
(Ajnesh, York University's Schulich School of Business PhD candidate, Beyond analytical
dichotomies, Human Relations, 65.5, Sage)
poststructuralism may be typified by what Lyotard (1984) famously
expressed as its incredulity towards metanarratives (also see Alvesson and Deetz, 1999; Kilduff and Mehra, 1997), or what Fraser and
A poststructuralist critique At the most rudimentary level,

Nicholson (1997) describe as grand theorizing of social macrostructures. Akin to other critical traditions, it is explicitly historical, attuned to the cultural specificity of different societies and periods and . . . inflected
by temporality, with historically specific institutional categories (Fraser and Nicholson, 1997: 1434). The aim of the poststructuralist mandate is to critique metanarratives and, from there, to define human
consciousness and social existence through engagements with contextualized subjectivity (Agger, 1991).11 To appreciate this idea, it is important to understand how metanarratives are problematically situated
within, and are informed by, socially constructed identity binaries, be they along the fault lines of gender (e.g. Butler, 1990; Hird, 2000), race (e.g. Gilroy, 2000; Miles and Torres, 2007), or sexuality (e.g. Zita,

poststructuralist critique, such as Derridas conjecture of difference (Mumby and Putnam, 1992), illustrates how the
preservation of the privileged identity (white, man, heterosexual) is existentially
dependent upon a corresponding relegated identity (black, woman, homosexual) (see Tyler
and Cohen, 2008); or, to posit it in Butlerian (1991) phrasing, heterosexuality presumes the being of homosexuality. Working from the same current, Harding
(2003) extends this idea through consideration of the central dyadic relationship in management; she
notes that every individual in the western workforce is identified as a worker or as a manager
and that the identity of the latter is wholly contingent upon the binary existence of the
1994). Indeed,

former . Elsewhere, again assuming a poststructuralist position, Harding (2008: 44; emphasis in original) explains that identities cannot be resolved into
an essence or into a coherent whole; rather, they are post hoc impositions of a
seemingly unified [label] upon a disparate and disconnected population. As such, the central aim of the poststructuralist
is to repudiate deterministic and binary logic by drawing attention to the discursive processes that culturally (re)produce social realities and the dichotomous modes of thinking embedded within them (Butler, 1990;
Calas, 1993). [T]he ways in which sex was put into Western discourse from the end of the 16th century onwards, the proliferation of sex during the 18th century and the modern incitement to discuss sex in

Tangentially related to the


Foucauldian reading of sexuality, Katz (2004) offers a genealogical investigation into how
heterosexuality was invented and came to be ideologically defined from the late 19th-century onwards. Katz writes that because
the concept of heterosexuality is only one particular historical way of perceiving,
categorizing, and imagining the social relations of the sexes, it ought to be studied with
the purposeful aim to dislocate its socio-cultural privilege as being the normal and the
natural form of sexual expression (p. 69). Given its socially manifest nature, Butler (1991, 1993) contends that heterosexuality is perpetually at risk and must
endless detail have simultaneously established heterosexuality as the unassailable norm and constituted other sexualities as abnormal.

continually engage in a set of repetitive, or what she calls parodic, practices such as, heterosexual sex which functions to stringently affirm the hegemonic ideals of femininity (passive) and masculinity (active).
Incidentally, the very redundancy of these parodic practices function to consolidate the discursive authority and cultural stability of heterosexuality (Butler, 1991; see also Butlers [1993] writing on performativity).
A related stream of poststructuralist-inflected scholarship reveals how

sexual identities that are predicated on ontological sexual

difference produce heteronormativity, which can be described as the the normative idealization of heterosexuality (Hird,
2004: 27) or the centrality of heterosexual norms in social relations (Pringle, 2008: S111). While feminists have long critiqued the tacit
and the explicit claims of ontological sexual difference, essentialist definitions of female and male continue to prevail in
popular culture and in certain academic disciplines (Frye, 1996).12 On this point, Hird (2004) adopts a position in feminist science studies to
develop a substantive critique into how the ontology of sexual difference is often rendered concrete in research propagated by the natural and particularly, the biological sciences (also see Martin, 1991).

The influence of ontological sexual difference within and outside of academia, lends
credence to Broadbridge and Hearns (2008: S39) recent observation that, [s]ex and sex
differences are still often naturalized as fixed, or almost fixed, in biology. It is equally important to note, here,
that the alchemy of ontological sexual difference is wholly dependent upon the patriarchal
conflation of biological sex and cultural gender (Hird, 2004). As Pringle (2008: S112; also see Borgerson and Rehn, 2004) notes,
[g]ender [does] not avoid the oppositional duality embodied in the concept of sex, but

reflect[s] the interdependent relationship of masculinity and femininity. This reflection pivots on genital
determinism, which declares that males naturally embrace masculinity while females naturally embrace femininity (Bornstein, 1994; Hird, 2000). This initial conflation of sex and gender leads to the conventional

It is
precisely these corresponding relationships whereby the heterosexual matrix is
constructed (Butler, 1990). According to Butler, this matrix serves as the grid of cultural
intelligibility through which bodies, gender, and desires are naturalized (see Ringrose, 2008: 511).13
model of heterosexuality, which dictates that a man will desire-to-be a male and will desire-for a female, while a woman will desire-to-be a female and will desire-for a male (Sinfield, 2002: 126).

AT: rage alt

The valorization of rage as a political leads to a vicious cycle of repetitive


violence

Wenning 09
(Mario, Phd., Assistant professor of philosophy @ the University of Macau, The Return of
Rage, Parrhesia No. 8 pg. 89-99)
The valorization of erotic emotions and virtues over thymotic ones is as old as philosophy itself.
Aristotle already insists that the virtuous person cultivates mildness of temper the even
tempered person confesses to be calm and not carried away by his feelings, but to be cross
only in the way, at the things, and for the length of time that reason dictates. 15 Compassion
is introduced as an antidote to revenge. The virtuous character does not lose the control
that is necessary to provide for a self-sufficient emotional economy, which is the
precondition for achieving a life that is marked by wisdom, even-temperedness, and justice.
Senecas influential work on rage, De ira, which was immensely influential for Christian and
humanist ethics, calls for a Stoic control of the dangerous affect. The general suspicion
against the destructive consequences of this aggressive emotion is not limited to the European
tradition. Confucius already warns his students to let a sudden fit of anger make you
forget the safety of your own person or even that of your parents, is that not misguided
judgment? 16 Daoism and ZenBuddhism promote meditative practices and compassion to
overcome our fixation on the need of being angry with ourselves and the world surrounding us.
More recently, Martha Nussbaum argued that we should aim to understand how to
channel emotional development in the direction of a more mature and inclusive and less
ambivalent type of love. 17 According to Nussbaum, anger should at best operate as a tool of
compassion. Acts of punishment are then seen as merciful rather than vindictive because
they aim at the good of the victim. These representative examples illustrate that the
erotization of the psyche replaced what is regarded as archaic forms of militancy that, it
is contended, mistakenly suggest that honor, pride and craving for recognition (and the
rage that results from the violation of these) has been considered to be more important
than a concern for justice, equality and compassion. We might think that the dislike of
negative emotions in general and potentially aggressive ones in particular results from an
insight into the misfortunes these emotions bring about. Revenge, then, is undesirable
because it tends to be too costly in producing long term damages. Hegel, for example,
reminds us in the Philosophy of Right of the infinite chain of violence , the economy of
pay-back that results from blind vengeance and selfadministered acts of justice. 18 The
excesses of rage can easily lead to tragic repetitions of an original act of violence that
might be impossible to get out of . Honor killings often lead to new honor killings rather than
the reestablishment of justice and the fight against terror breed more terrorists.

Utilize rage as a means to mobilize ati-surveillance movements---targeted


rage is key
Lesage 85
(Julia, in Marxism and the Interpretation of Culture, ed. Nelson and Grossberg,
http://pages.uoregon.edu/jlesage/Juliafolder/womensRage.html)
Feminism by itself is not the motor of change. Class, anti-imperialist, and antiracist struggles demand our participation. Yet how,
specifically, does women's consciousness change? How do women move into action? How does change occur? What political strategies should feminists pursue? How, in our
political work, can we constantly challenge sexual inequality when the very social construction of gender oppresses women? In 1981 I visited Nicaragua with the goal of finding
out how and why change occurred there so quickly in women's lives. "The revolution has given us everything," I was told. "Before the revolution we were totally devalued. We
weren't supposed to have a vision beyond home and children." In fact, many Nicaraguan women first achieved a fully human identity within the revolution. Now they are its most
enthusiastic supporters. For example, they form over 50 percent of the popular militias, the mainstay of Nicaragua's defense against United States-sponsored invasions from
Honduras and Costa Rica. In the block committees, they have virtually eliminated wife and child abuse. Yet in Nicaragua we still see maids, the double standard sexually,
dissatisfaction in marriage, and inadequate childcare. Furthermore, all the women I talked to defined their participation in the revolution in terms of an extremely idealized notion
of motherhood and could not understand the choice not to reproduce. I bring up this example of Nicaragua because Nicaraguan women are very conscious of the power of their
own revolutionary example. They know they have been influenced by the Vietnamese and Cuban revolutions and are very much shaping how Salvadoran women militants are
looking at women's role in the Salvadoran revolution. Because of the urgency and violence of the situation, unity between men and women was and is necessary for their
survival, but the women also want to combat, in an organized and self-conscious way, specific aspects of male supremacy in the workplace, politics, and daily life. Both here and
in Nicaragua, women's daily conversation is about the politics of daily life. They talk to each other often, complaining about men and about managing the domestic sphere.
Women's talk also encompasses complaints about poor and unstable work conditions, and about the onerous double day. However, here in the United States that conversation
usually circulates pessimistically, if supportively, around the same themes and may even serve to reconfirm women's stasis within these unpleasant situations. Here such
conversation offers little sense of social change; yet in our recent political history, feminists have used this preexisting social form--women's conversation in the domestic
sphere--to create consciousness-raising groups. But to what degree is consciousness raising sufficient to change women's behavior, including our self-conception and our own

We do not live in a revolutionary situation in the United States. There is no leftist political organization here
Within
such a context, women need to work on another, intermediate level, both to shape our revolutionary
consciousness and to empower us to act on our own strategic demands . That is, we need to promote
self-conscious, collectively supported, and politically clear articulations of our anger
and rage. Furthermore, we must understand the different structures behind different women's rage. Black women rage against poverty and racism at the same time
colonized minds?

providing leadership and a cohesive strategy, and in particular the struggle against women's oppression is not genuinely integrated into leftist activity and theory.

that they rage against sexism. Lesbians rage against heterosexual privilege, including their denial of civil rights. Nicaraguan women rage against invasions and the aggressive
intentions of the United States. If, in our political work, we know this anger and the structures that generate it, we can more genuinely encounter each other and more
extensively acknowledge each other's needs, class position, and specific form of oppression. If we do not understand the unique social conditions shaping our sisters' rage, we
run the risk of divisiveness, of fragmenting our potential solidarity. Such mutual understanding of the different structures behind different women's anger is the precondition of
our finding a way to work together toward common goals. I think a lot about the phenomenon of the colonized mind. Everything that I am and want has been shaped within a
social process marked by male dominance and female submission. How can women come to understand and collectively attack this sexist social order? We all face, and in
various ways incorporate into ourselves, sexist representations, sexist modes of thought. Institutionally, such representations are propagated throughout culture, law, medicine,
education, and so on. All families come up against and are socially measured by sexist concepts of what is "natural"--that is, the "natural" roles of mother, children, or the family
as a whole. Of particular concern to me is the fact that I have lived with a man for fifteen years while I acutely understand the degree to which heterosexuality itself is socially
constructed as sexist. That is, I love someone who has more social privilege than me, and he has that privilege because he is male. As an institution, heterosexuality projects
relations of dominance and submission, and it leads to the consequent devaluation of women because of their sex. The institution of heterosexuality is the central shaping factor
of many different social practices at many different levels--which range, for example, from the dependence of the mass media on manipulating sexuality to the division of labor,
the split between the public and private spheres, and the relations of production under capitalism. Most painfully for women, heterosexuality is a major, a social and
psychological mode of organizing, generating, focusing, and institutionalizing desire, both men's and women's. Literally, I am wedded to my own oppression. Furthermore, the
very body of woman is not her own--it has been constructed by medicine, the law, visual culture, fashion, her mother, her household tasks, her reproductive capacity, and what
Ti-Grace Atkinson has called "the institution of sexual intercourse." When I look in the mirror, I see my flaws; I evaluate the show I put on to others. How do I break through
representations of the female body and gain a more just representation of my body for and of myself? My social interactions are shaped by nonverbal conventions which we all
have learned unconsciously and which are, as it were, the glue of social life. As Nancy Henley describes it in Body Politics, women's nonverbal language is characterized by
shrinking, by taking up as little space as possible. Woman is accessible to be touched. When she speaks in a mixed group, she is likely to be interrupted or not really listened to
seriously, or she may be thought of as merely emotional. And it is clear that not only does the voyeuristic male look shape most film practice, but this male gaze, with all its
power, has a social analog in the way eye contact functions to control and threaten women in public space, where women's freedom is constrained by the threat of rape. We
need to articulate these levels of oppression so as to arrive at a collective, shared awareness of these aspects of women's lives. We also need to understand how we can and
already do break through barriers between us. In our personal relations, we often overcome inequalities between us and establish intimacy. Originally, within the women's
movement we approached the task of coming together both personally and politically through the strategy of the consciousness-raising group, where to articulate our experience
as women itself became a collective, transformative experience. But these groups were often composed mostly of middle-class women, sometimes predominantly young,
straight, single, and white. Now we need to think more clearly and theoretically about strategies for negotiating the very real power differences between us. It is not so
impossible. Parents do this with children, and vice versa; lovers deal with inequalities all the time. The aged want to be in communion with the young, and third-world women
have constantly extended themselves to their white sisters. However, when women come together in spite of power differences among them, they feel anxiety and perhaps
openly express previously suppressed hostility. Most likely, such a coming together happens when women work together intensively on a mutual project so that there is time for
trust to be established. Yet as we seek mutually to articulate the oppression that constrains us, we have found few conceptual or social structures through which we might

Women's anger is pervasive, as pervasive as our oppression, but it


frequently lurks underground. If we added up all of women's depression--all our compulsive smiling, ego-tending, and sacrifice; all our
psychosomatic illness, and all our passivity--we could gauge our rage's unarticulated, negative force. In the sphere of cultural production
there are few dominant ideological forms that allow us even to think " women's rage." As
authentically express our rage.

ideological constructs, these forms end up containing women. Women's rage is most often seen in the narratives that surround us. For example: Classically, Medea killed her
children because she was betrayed by their father. Now, reverse-slasher movies let the raped woman pick up the gun and kill the male attacker. It is a similar posture of dead
end vengeance. The news showed Patty Hearst standing in a bank with a gun embodying that manufactured concept "terrorist," and then we saw her marrying her FBI
bodyguard long after her comrades went up in flames. In melodrama and film noir, as well as in pornography, women's anger is most commonly depicted through displacement
onto images of female insanity or perversity, often onto a grotesque, fearful parody of lesbianism. These displacements allow reference to and masking of individual women's

rage, and that masked rage is rarely collectively expressed by women or even fully felt. We have relatively few expressions of women's authentic rage even in women's art.
Often on the news we will see a pained expression of injustice or the exploitative use of an image of a third- world woman's grief. Such images are manipulated purely for
emotional effect without giving analysis or context. Some great feminist writers and speakers such as Mary Wollstonecraft, Virginia Woolf, Elizabeth Cady Stanton, and Harriet
Tubman have provided models by which we can understand ourselves, but too often the very concept of "heroine" means that we hold up these women and their capacity for

women chained themselves together in the state


house when it was clear that the ERA would not pass; the women sought to express our collective anger at our legislators' cowardice and to do so in a conspicuous, public
way. But actions such as these often have little effect beyond their own time span. We need to
think beyond such forms to more socially effective ones. It is a task open to all our creativity and skill--to tap our
anger as a source of energy and to focus it aesthetically and politically. We may have to combine images of anger with
angry self-expression as the exception rather than the rule. In Illinois,

something else--say, images of how women can construct the collectivity as a whole. It is here that, by their example, our third-world sisters have often taken the lead. Rosa
Parks refusing to sit in the back of the bus, Harriet Tubman leading slaves to the North, an Angolan mother in uniform carrying a baby and a rifle, a Vietnamese farmer tilling and
defending her land, Nicaraguan women in their block committees turning in wife abusers to the police--these images let us see that

women can gain more

for themselves than merely negating the bad that exists. And it is in their constant need to attack both sexism and racism,
as well as poverty and imperialist aggression, that third-world feminists now make us all see much more clearly both the urgent need for and the possibility of reconstructing the
whole world on new terms. Artistically, emotionally, and politically women seem to need to glimpse dialectically the transcendence of our struggle against sexism before we can

our suppressed rage feels so immense that the


open expression of it threatens to destroy us. So we often do not experience anger directly
and consciously, nor do we accurately aim our rage at its appropriate target. To
transcend negation and to build on it means that we have to see what is beyond our
fully express sexism's total negation, that is, our own just rage. Sometimes

rage . An example of such transcendence was demonstrated by Nicaraguan mothers of "martyred" soldiers (those killed by U.S.-paid counterrevolutionaries) to Pope John
Paul II when he visited Managua in April 1983. They stood in the rows closest to the podium where the Pope spoke and they all bore large photos of their dead children. As the
events of the day unfolded, the women created an image that stirred the whole people, one that the Pope could not go beyond or even adequately respond to. Here is what
happened: The Pope spoke on and on to the gathered crowd about obeying the hierarchy and not getting involved with the things of this world. In frustration and anger, the
women began to shout, "We want peace," and their chant was taken up by the 400,000 others there. The women's rage at personal loss was valorized by the Nicaraguan
people as a whole, as the grieving mother became a collective symbol of the demand for peace. The chant, "We want peace," referred simultaneously to national sovereignty,
anti-imperialism, religion, and family life. The women spoke for the whole. This brings me back to my original question about women's political action in the United States today.
One of the major areas of investigation and struggle in the women's movement has been the sphere of daily life. This struggle, represented by an early women's movement
phrase--"the personal is the political"--derives from women's real material labor in the domestic sphere and in the sphere of social relations as a whole. Women have traditionally
done the psychological labor that keeps social relations going. In offices, in neighborhoods, at home, they often seek to make the social environment safe and "better," or more
pleasant. That such labor is invisible, particularly that it is ignored within leftist theory and practice, is one of the more precise indices of women's oppression. And it is feminists'
sensitivity to and analysis of social process that clarifies for them the sexism on the Left. Often at a leftist conference or political meeting, many men continue to see women and
women's concerns as "other," and they do not look at what the Left could gain from feminist theory or from women's subcultural experience or from an analysis of women's
labor. Women who come to such an event have already made a commitment to learn and to contribute, so they make an effort to continue along with the group as a whole but
are impeded by sexist speakers' intellectual poverty (e.g., use of the generic "he"), macho debating style, and distance from political activism. Furthermore, not only women feel
this political invisibility at leftist events. When black labor and black subcultural experience in the United States is not dealt with, nor is imperialism, or when racism is
theoretically subsumed under the rubric of "class oppression" and not accorded its specificity, then third-world participants face the same alienation. To demonstrate this process
and analyze what divides us, I will describe an incident that occurred at the Teaching Institute on Marxist Cultural Theory in June 1983. It is worth discussing because it is the
kind of incident that happens all too often among us on the Left. Early in that summer session, a coalition of students and the two women faculty members, Gayatri Spivak and
me, formed to present a protest statement to the faculty. It was read in every class. Here is what it said: The Marxist-Feminist Caucus met on Friday June 17th and concluded
that the "limits, frontiers and boundaries" of Marxist cultural theory as articulated by the Teaching Institute excluded and silenced crucial issues of sexism, racism and other
forms of domination. We find ourselves reproducing in the classrooms of the Teaching Institute the very structures which are the object of our critique. The Marxist-Feminist
Caucus therefore proposes that each class set aside an hour weekly to discuss strategic silences and structural exclusions. A Marxism that does not problematize issues of
gender and race, or of class consciousness in its own ranks, cannot hope to be an adequate tool for either social criticism or social transformation. The institute had a format of
having famous Marxist intellectuals lecture, specifically males with job security who have never incorporated a feminist analysis into their theoretical work. Both the format and
the content of their lectures enraged some of us, but not others. In a sense, writing a protest statement divided the school's participants between the political ones and the
consumers of Marxist theory. This is because critical theory itself has become a pathway for elitist advancement in the humanities and social sciences in universities where
these areas are facing huge cutbacks. And the canon of that critical theory is based on Marx and Freud and their contemporary interpretants, Althusser and Lacan. Both at the
Teaching Institute and at prestigious universities, young academics could get their quick fix of Marxism, the knowledge of which could help greatly in their academic career. This
is a capitalist mode of consuming knowledge. Too many students, especially career- pressured graduate students, want only a well conceived lecture, a digest of Marxist theory
and social analysis, something that can be written in a notebook, taken home, and quoted from in a future paper or journal article. Furthermore, we intellectuals fall into this
capitalist competitive mode. We feel pressured inside ourselves to be the best. Students are told to buy the best. All the faculty at the Teaching Institute felt that they could not
make a mistake, that they had to read and show they had read everything, that they had been challenged on their political practice, accused of being racist or sexist or
undemocratic. Our control over the classroom and studied theoretical polish became a kind of professional hysteria and worked against the collective building of Marxist
knowledge and theory that we have needed for more effective social change. Since the early 1970s women have come together in meetings like these, in feminist seminars,
caucuses, and workshops, partly in resistance to a certain macho leftist or academic style and partly to build a new body of knowledge and feminist political practice. And we
have been successful at doing this but it has meant double or triple work for us. Feminist scholarship does not usually lead to academic promotion for a woman. The knowledge
women produce is easily marginalized, as was made painfully obvious at that summer school. Feminists and third-world students came to the Teaching Institute knowing how
much they needed Marxist theory. They understood that abolishing capitalism and imperialism was the precondition for liberation. They came as political participants expecting
to learn theoretical tools to use in fighting oppression. But sex and race were too often ignored--I would say stupidly ignored--as social determinants in the theories presented
about social change. (Beyond that, students felt intimidated by name-dropping and teachers' and other students' failure to explain terms. They felt they had to give a polished
rebuttal or a cohesive "strategic intervention" before they could speak to refute a lecturer's point.) And when students raised issues of sexism or racism, deflection became the
all too frequent tactic used by teachers or some of the white male students in response. No wonder that women, with their sex-role socialization, were often too intimidated to
speak. This is a sad analysis, but not an infrequent one in academia. It speaks about political theory and academic sexism and racism, and elitism and class privilege. The
incident reveals much of what divides politically progressive people in the United States. These differences must be acknowledged in depth if we are to work together politically
in a coalition form. In particular, I understand the texture of women's silence in a forum that demanded a highly rational and developed intervention. Many of the women students
at the Teaching Institute already produced feminist theory, but the intimidating nature of this kind of aggressive public speaking made them seem like nonparticipants. And it
often happens to me, too. I know that we watch and despair of our own colonized psyches which hold us back in silence precisely when we would choose to be political actors,
especially in a Marxist forum. What we have seen in the 1970s and 1980s in North America and Europe is a supercession of political forms related to developments in radical
consciousness. Conditions have evolved in the United States that make it impossible to conceive of a revolutionary organizing strategy that does not embrace a black and
minority revolution and a feminist revolution. The lesson of the civil rights/black power movement was that blacks will organize autonomously. Now it is the offspring of that
movement, Jesse Jackson's Rainbow Coalition, that has taken the lead in building an anti-imperialist coalition that addresses the specific struggles and organizing forms of
blacks, Latinos, women, and gays. Such a coalition relates to the existence of the women's movement, the gay and lesbian movement, the anti-imperialist movement, by
supporting these groups' autonomous organizing and granting new respect, not by subsuming or controlling them. Furthermore, at this point in U.S. history, issues of mass
culture and mass communication have to be dealt with, so that minority figures such as Jesse Jackson or Harold Washington, Chicago's black major, have developed an
ongoing analysis about racism in the press. As a feminist who has worked both in the cultural sphere and in anti-imperialist work, I have experienced this supercession of forms.

In the early 1970s a politically active woman was either "on the Left" or "in the independent women's movement." Some socialist feminists within leftist organizations formed
caucuses to try to influence their organizations. In the 1970s I chose to work mostly within the independent women's movement, especially in creating a women's studies
program at an urban university. In developing feminist media now within the women's movement, I find many of my sisters addressing broader issues of imperialism, racism,
class oppression, and the nuclear threat. Many of us are joining progressive coalitions around these issues. Within these coalitions we must be able openly to declare, "I am a
feminist and our feminist position represents the most advanced stand. You men have to join us." Indeed, many men, often younger men, have. As feminists, we are the ones
who are building a whole theoretical critique of mass culture and mass communication; we are the ones who are learning how to appropriate all of culture in an oppositional way.
And because of our historical position in advanced capitalism, we are one of the first social movements to address cultural issues in such a thorough and complex way. Many
feminists are eager to participate in coalitions, the major political strategy for us in the 1 980s. In Chicago, we saw the women's movement and the Left work to elect Harold
Washington. In the San Francisco area, gays and lesbians have formed a Central America support group. Both in the United States and abroad, the antinuclear movement
contains within it all-women's affinity groups. Latinos in various areas identify and organize as Puerto Rican or Mexican-American according to their ethnic origins and
concentration, and also unite in Central American solidarity work. This great diversity of sectoral organizing enriches all of us who are working for social change. Some of the
best aspects of current progressive organizing have, in fact, derived specifically from the development of the contemporary women's movement. I mentioned the consciousnessraising groups earlier. I think the women's movement has introduced into political discourse an open and direct critique of the macho style and political posturing of many male
leaders. As feminist activists, we have created among ourselves new forms of discussion and a creative, collective pursuit of knowledge--in contrast to an older, more
aggressive, male debating style. Particularly important for me, the women's movement has pursued and validated as politically important cultural and artistic work. In Chicago,
where I live, I experience a strong continuum and network among community-based artists and women in the art world. We have built up intellectual ties between academic
women and feminist film- and videomakers who have created an analysis of how sexuality is manipulated in the visual culture that surrounds us. As a consequence, feminist film
criticism has developed a new theoretical framework for analyzing ideology and the mass media. In fact, I think that our building of a feminist cultural theory has made a key

consider how unleashing our anger might


capacitate us to act for change, I reconsider Frantz Fanon's essay "Concerning Violence" in The Wretched of the Earth. In that essay he describes
decolonization, particularly the process by which the native sheds the colonizer's values and the colonizer's ways. I understand that my black and Latina sisters in the
United States experience a rage against the economic and racial violence perpetrated every day against them; in a way that is similar to what Fanon describes:
this rage knows its resolution lies in a complete change of the economic order in which we live. At the same time, I must ask what kind of rage it
would be that would effectively contest women's oppression--given all the levels at which gender inequality and
contribution to the Left and to revolutionary movements throughout the world. When I want to

women's oppression is articulated in social and personal life. What Fanon describes to us is a specific historical moment at which mental colonization can be and is surpassed.
As I look at women's mental colonization, I see our internalized sense of powerlessness, our articulation into masochistic structures of desire, and our playing out of personae
that on the surface seem "passive," "self-defeating," "irrational," "hesitant," "receptively feminine," or even "crazy." Much of this behavior stems from internalized and suppressed

active rage

rage. Fanon describes such behavior in the colonized and posits


, the violent response to violence, as its cure. What would the overturning of male
supremacy and women's colonization mean to women? How would it be accomplished? Fanon understands that a whole social structure and a new kind of person must come
into being, and that those with privilege know, fear, and resist this. His call to armed struggle, based on the very clear demarcations and abuses of power that the native always
sees,

signals a survival struggle that does not characterize the war between the sexes . As I read

Fanon for what he can teach me about women's resistance to oppression in nonrevolutionary society, I read him as a communist psychiatrist talking about how social
movements can change the mentality of the oppressed. When I ask about revolution for women now, minimally I see that our contestation cannot be conducted in the mode of
nice girls, of managing the egos of and patiently teaching those who oppress, which is a skill and duty we learned from our mothers in the domestic sphere. If we do so, once

Angry contestation may take us the extra


step needed to overcome our own colonized behavior and tardy response. Let me now rewrite for you parts of Fanon's
essay to show its power when discussing the relation between psychological and social change. The distance between the violence of
colonization and its necessary response in armed struggle, and the emotional rage I am
referring to here in combating sexism, marks the distance between the periphery and the center of
again we will be placed in that very role of "helpmate" that we are trying to overcome.

international capitalism. By using Fanon in this way, I do not wish to co-opt him for the women's movement but to learn from him, just as I learned from the Nicaraguan women's
courage and tenacity. If women must learn to be openly angry, we must learn to draw links between ourselves and those who are more oppressed, to learn new methods of
struggle and courageous response. Combating women's oppression as we know it is a historical process: that is to say, it cannot become intelligible or clear to itself except in
the exact measure that we can discern the movements that give it historical form and content. Combating women's oppression is the meeting of two intrinsically opposed forces,
which in fact owe this originality to that sort of substantification that results from and is nourished by the social construction of gender. The husband is right when he speaks of
knowing "them" well--for it is men who perpetuate the function of wife. Men owe the reproduction of their bodies and psyches to the family. Feminist revolution never takes place
unnoticed, for it influences individuals and modifies them fundamentally. It transforms passive femininity crushed with inessentiality into privileged agency under the floodlights of
history. A new kind of woman brings a new rhythm into existence with a new language and a new humanity; combating women's oppression means the veritable creation of new
women who become fully human by the same process by which they freed themselves. Feminists who decide to put their program into practice and become its moving force are
ready to be constantly enraged. They have collectively learned that this narrow world, strewn with prohibitions, can only be called into question by absolute contestation. The
sex-gender system is a world divided into compartments. And if we examine closely this system of compartments, we will at last be able to reveal the lines of force it implies and

At the level of individuals, anger is a cleansing


force. It frees the woman from her inferiority complex and from despair and inaction; it makes her fearless and restores her self-respect. At this point I will stop
to mark out the lines on which a nonoppressive society will be reorganized.

deliberately at the point of individual rage. Now is a time when we need


to work in coalitions, but we must be very honest about what divides us and what are the
preconditions we need before we can work together. I have made the decision to work in leftist and feminist cultural work and
citing from and reworking Fanon,

in Latin American solidarity work. I think in all our strategies we must analyze the relation of that strategy to feminist, antiracist, and anti- imperialist demands. Women comprise
over half the population; any class issues in the United States are intimately tied to the question of racism; we all live off the labor of workers, often underpaid women, in the
Third World; and socialist revolution is being waged very near us. Personally, I know that it is by my contact with Nicaraguan women, who insist that men and women must
struggle together for our mutual liberation, that I have been politically and emotionally renewed. The problems grow more acute. We know that the Right is racist, homophobic,

the women's movement must stop turning our anger against each other and
learn the most effective ways to work together for social change. We can focus our
and sexist. We in

anger and harness it , but to do that we must clearly analyze cause and effect. If theory
accompanies anger, it will lead to effective solutions to the problems at hand. We have great emotional and
social power to unleash when we set loose our all too often suppressed rage, but we may only feel free to do so when we know that we can use our anger in an astute and
responsible way.

Their deployment of essentialism fails its rejected as a descriptive theory


and cant achieve political ends
Stone 04 (Alison, Institute for Environment, Philosophy and Public Policy Lancaster
University, Essentialism and Anti-Essentialism in Feminist Philosophy, Journal of Moral
Philosophy 1.2, 2004)
An objection immediately arises to this strategic essentialist position. Any political strategy is effective only inasmuch as it allows agents to recognize and intervene into the real social events, processes and forces

it seems reasonable to think that a strategy can be effective, in this sense,


only insofar as it embodies an accurate understanding of the character of social
processes. This implies that a strategy of affirming fictitious commonalities among women will fail to
which make up the social field. But

facilitate effective action given a world where women do not really have any common
social characteristics or locations. Rather, such a strategy appears destined to mislead
women into fighting against difficulties which are either non-existent or more likelyreally affect only some
privileged subgroup of women. This objection can be resisted, however, as it (implicitly) is by Denise Riley in Am I That Name? . Riley claims that it is compatible to
suggest that women dont existwhile maintaining a politics of as if they existed since
the world behaves as if they unambiguously did. 15 In other words, for Riley, the fiction that women share a common social experience is politically effective because the social world actually does treat women as

this false idea


informs and organizes the practices and institutions that shape womens experiences, so that
thosevery differentexperiences become structured by essentialist assumptions. A strategy of affirming fictitious commonalities
therefore will be effective given this world in which (false) descriptive essentialist
assumptions undergird womens social existence. Rileys argument has a problem, though: she cannot
consistently maintain both that womens social experience is fully diverse and that this
experience is uniformly structured by essentialist assumptions. If essentialism informs and organizes the structures that
shape womens social experience, then this experience will be organized according to certain shared models and will acquire certain common patterns and features. More concretely,
the idea that women are a homogeneous group will structure social institutions so that they position all
women homogeneously, leading to (at least considerable areas of) shared experience. Thus, Riley (and other strategic essentialists)may be right that essentialist
constructions are socially influential, but they cannot, consistently with this, also maintain that descriptive essentialism is false. Furthermore, it is not obviously true that
any uniform set of essentialist constructions informs all social experience. These
constructions may all identify women as a homogeneous group, but they vary widely in their
account of the context of womens homogenous features. Consequently, these constructions will influence social structures in
if they comprise a unitary group. Riley accepts that women are not a unitary group and that the socially prevalent idea that they are unified is false. Nevertheless,

correspondingly varying directions, against which no counter-affirmation of common experience can be expected to be effective Strategic essentialists, then, have attempted to resuscitate essentialism by arguing

this attempt proves unsuccessful, because one cannot defend


essentialism on strategic grounds without first showing that there is a homogeneous set of essentialist assumptions that exerts a coherent influence on womens
social experiencewhich amounts to defending essentialism on descriptive grounds(as well). Advocates of essentialism therefore need to
show that it accurately describes social reality. Here, though, critics can retort that essentialism is
descriptively false, since women do not even share any common mode of construction by essentialist discourses. Yet this retort reinstates the
problem of anti-essentialism: its paralysing effect on social criticism and political
activism. Strategic essentialism has not resolved this problem, for it has not stably
demarcated any merely political form of essentialism from the descriptive essentialism
that it can take a merely political and non-descriptive form. But

which critics have plausibly condemned as false and oppressive.

Foucault Answers

General Foucault

Aff = Consistent
Their reading of Foucauldian philosophy is all wrong - the affirmative
supports Foucaults idea to deconstruct power systems instead of their
radical critique.
Ferguson 11
Professor of Anthropology at Stanford Univeristy (James, Toward a left art of government: from
Foucauldian critique to Foucauldian politics, History of the Human Sciences 24(4),
http://hhs.sagepub.com/content/24/4/61.full.pdf)//DWB
One of the founding premises of this special issue and the conference with which it began is that Foucault has been read, and
used, in different ways in different academic disciplines. In this article I will discuss one common way of using Foucaults thought in

using Foucauldian modes of analysis to


critique power (as it is often put) has frequently led to a rather sterile form of political
engagement. Attention to some of Foucaults own remarks about politics hints at a different
political sensibility, in which empirical experimentation rather than moralistic denunciation
takes center place. I will reference some examples of such experimentation that come out of my current research on the
my own discipline of anthropology. I will suggest that the strategy of

politics of social assistance in southern Africa (though I do not have space here to give a full exposition of these). The sort of use
of Foucault that I have in mind is well represented in the anthropology of development (and the related field of what is sometimes
called critical development studies). Here, the

characteristic strategy is to use Foucauldian analysis to


reveal the way that interventions, projects, etc., which claim to be merely technical or
benevolent, really involve relations of power. This is a perfectly reasonable thing to do, but too often, in this
field, such a simple demonstration is apparently seen as the end of the exercise. Power has been critiqued, an
oppressive system has been exposed as such, and that seems to be taken as a
satisfactory end to the matter. This impasse in development studies and anthropology is related, I think, to a wider
predicament that progressive or left politics seems to find itself in today. The predicament is that the left seems
increasingly to be defined by a series of gestures of refusal what I call the antis (anti-globalization,
anti-neo-liberalism, anti-privatization, anti-Bush, sometimes even anti-capitalism but always anti, never pro). The
current world system, the politics of the anti- points out, rests on inequality and exploitation. The global poor are
being screwed, while the rich are benefiting. The powerless are getting the short end of the stick. This is all perfectly true, of
course, if not terribly illuminating. But such

lines of argument typically have very little to propose by


way of an alternative art of government. Governing is exercising power over others, which is what the
powerful do to the downtrodden. It appears as something to be resisted or denounced, not improved or experimented with. My first
observation about this sort of analysis is that it

rests on what seems to me a very un-Foucauldian idea


of the political. Foucault did, certainly, valorize certain forms of resistance, and worked tirelessly to
undermine and denaturalize taken-for-granted arrangements of power. But he never
suggested that power ought not be exercised, or that it was illegitimate for some to seek
to govern the conduct of others. On the contrary, he repeatedly insisted that it made no sense
(in his scheme of things) to wish for a world without power.1 Naive readings of Foucault turned
his skeptical analytics of power into a simple denunciation. Thus the question (once posed to him by
an interviewer) of whether it would be an intolerable use of power for a parent to prevent a child from scribbling on the walls of a
house. Foucaults instructive answer was: If I accepted the picture of power that is frequently adopted namely, that its something
horrible and repressive for the individual its clear that preventing a child from scribbling would be an unbearable tyranny. But
thats not it. I

say that power is a relation. A relation in which one guides the behavior of
others. And theres no reason why this manner of guiding the behavior of others should
not ultimately have results which are positive, valuable, interesting, and so on. If I had a kid, I

assure you he would not write on the walls or if he did, it would be against my will. The very idea! (Foucault, 1988a: 1113) In the
same interview, he complained of those who ... think Im a sort of radical anarchist who has an absolute hatred of power. No!

What Im trying to do is to approach this extremely important and tangled phenomenon


in our society, the exercise of power, with the most reflective, and I would say prudent,
attitude. ... To question the relations of power in the most scrupulous and attentive manner possible, looking into all the
domains of its exercise, thats not the same thing as constructing a mythology of power as the
beast of the apocalypse. (ibid.: 1113)

Alt = Inconsistent
Their static position of critique is exactly what Foucault opposed.
Ferguson 11 Professor of Anthropology at Stanford Univeristy (James, Toward a left art of
government: from Foucauldian critique to Foucauldian politics, History of the Human Sciences
24(4), http://hhs.sagepub.com/content/24/4/61.full.pdf)//DWB
Such a stance, I suggest, brings

us much closer toward a truly Foucauldian politics. For politics, for


Foucault, was always more about experimentation than denunciation. In an interview on social
security, Foucault insisted that what was required for a progressive rethinking of social
policy was not a theoretically derived line, but, as he put it, a certain empiricism. We
have to transform the field of social institutions into a vast experimental field, in such a way as to decide which taps need turning,
which bolts need to be loosened here or there, to get the desired change. ... What

we have to do ... is to increase


the experiments wherever possible in this particularly interesting and important area of
social life. (Foucault, 1988b: 165) What this implies is a form of politics that has less to do with
critique and denunciation than with experimentation and assessment. It is a matter not of
refusing power, but rather exercising it in a way that would be provisional, reversible,
and open to surprise. If we are indeed to arrive at viable left arts of government, we will need to be open to
the unexpected, ready to increase the experiments wherever possible, and attentive to
the ways that governmental techniques originally deployed for nefarious purposes can
be appropriated toward other ends. To do this, we will need to forgo the pleasures of theeasy, dismissive critique,
and instead turn a keen and sympathetic eye toward the rich world of actual social and
political practice, the world of tap-turning and experimentation. That is a world still full of invention and surprise, where
the landscape of political possibility and constraint that we have come to take for
granted is being redrawn, even as we speak.

Liberalism Good

Lib k HR/Env
International Relations and Liberalism is key to solve human rights,
environment, and poverty constitutents provide a legitimate check.
Devitt 11 Masters in International Relations (Sept. 1, Rebecca, Liberal Institutionalism: An
Alternative IR Theory or Just Maintaining the Status Quo?, E-International Relations Students,
http://www.e-ir.info/2011/09/01/liberal-institutionalism-an-alternative-ir-theory-or-just-maintainingthe-status-quo/)//DWB
Domestic interests have had a major impact on cooperation in International Relations ,
public reactions to the deaths of US soldiers in Somalia led to the Clinton administration pulling out of a peacekeeping mission to the
war torn country whilst public demand for action on Climate Change has led to member states such as Australia to sign up to the
Kyoto Protocol. [15] This reflects the fact that domestic

issues and policies have a major influence on


how states cooperate with other states on an international stage. If a state was to go into agreement
with another on trade relations, but that state had a bad record on human rights , the government might reconsider
doing a deal if its constituents elected them on the basis of a strong human rights
stance. This reflects the need for greater development of the liberal institutionalists
argument in terms of domestic influences on decision making at an international level.
Furthermore whilst Liberal institutionalists have acknowledged the influence non-state actors in world affairs such as transnational
organizations and non-governmental organizations play, they have

failed to recognize the role that global


political advocacy networks have had in international relations. Advocacy work on
human rights, the environment and poverty has had major effects on the way states are
viewed and global movements have challenged the notion that sovereignty is
sacrosanct. Technological advances and telecommunication networks have allowed for
swifter mobilization and organization of groups that can lobby governments and
organizations about issues such as human rights and questioned the liberal institutionalists argument that
states remain the key actors in world affairs. As R.B.J Walker has stated:

Lib k Peace/Democ
Liberalism is key to democracy that spills over to more peaceful
societies.
Burchill 05 Researcher of International Relations at Deakin University (Scott, Theories of
International Relations Third edition, Chapter 3 Liberalism, pp. 55-83)//DWB
At the beginning of this chapter, it was argued that liberalism was an inside-out approach to international relations, because

liberals favour a world in which the endogenous determines the exogenous. Their challenge
is to extend the legitimacy of domestic political arrangements found within democratic states to the relationships between all
nationstates. To put it another way, liberals

believe that democratic society, in which civil liberties


are protected and market relations prevail, can have an international analogue in the
form of a peaceful global order. The domestic free market has its counterpart in the open, globalized world
economy. Parliamentary debate and accountability is reproduced in international fora such as the United Nations. And the legal
protection of civil rights within liberal democracies is extended to the promotion of
human rights across the world. With the collapse of Communism as an alternative political and economic order, the
potential for continuity between the domestic and the international became greater than
in any previous period. Fukuyama had reason to be optimistic. The spread of liberal democracies and the zone of
peace was an encouraging development, as is the realization by states that trade and commerce is more closely correlated with
economic success than territorial conquest. The number

of governments enjoying civilian rather than


military rule is increasing, and there are signs that ethical considerations and ideas of
human justice have a permanent place on the diplomatic agenda. The collapse of Marxism as a
legitimate alternative political order removes a substantial barrier to the spread of liberal democracies, and there can be
little doubt that the great powers are now much less inclined to use force to resolve
their political differences with each other. It appears that liberal democracies are in the
process of constructing a separate peace . The globalization of the world economy means that there are few
obstacles to international trade. Liberals want to remove the influence of the state in commercial relations between businesses and
individuals, and the

decline of national economic sovereignty is an indication that the


corrupting influence of the state is rapidly diminishing. TNCs and capital markets wield significant
influence over the shape of the world economy, in the process homogenizing the political economies of every member state of the
international community.

Free Trade Solves War


Liberal free trade prevents war and fosters international dialogue.
Burchill 05 Researcher of International Relations at Deakin University (Scott, Theories of
International Relations Third edition, Chapter 3 Liberalism, pp. 55-83)//DWB
Free trade, however, was a more peaceful means of achieving national wealth because, according to the
theory of comparative advantage, each economy would be materially better off than if it had been
pursuing nationalism and self-sufficiency (autarky). Free trade would also break down the divisions
between states and unite individuals everywhere in one community. Artificial barriers to
commerce distorted perceptions and relations between individuals, thereby causing international
tension. Free trade would expand the range of contacts and levels of understanding between
the peoples of the world and encourage international friendship and understanding.
According to Kant, unhindered commerce between the peoples of the world would unite them in a common, peaceful enterprise. Trade
would increase the wealth and power of the peaceloving, productive sections of the
population at the expense of the war-orientated aristocracy, and would bring men of different nations into constant contact with one
another; contact which would make clear to all of them their fundamental community of interests (Howard 1978: 20; Walter 1996). Similarly Ricardo
believed that free trade binds together, by one common tie of interest and intercourse, the universal society of nations
throughout the civilised world (Ricardo 1911: 114). Conflicts were often caused by states erecting barriers which distorted and concealed the natural
harmony of interests commonly shared by individuals across the world. The solution to the problem, argued Adam Smith and Tom Paine, was the
free movement of commodities, capital and labour. If

commerce were permitted to act to the universal extent


it is capable, it would extirpate the system of war and produce a revolution in the
uncivilised state of governments (Howard 1978: 29). Writing in 1848, John Stuart Mill also claimed that free trade was the
means to bring about the end of war: it is commerce which is rapidly rendering war obsolete, by strengthening and multiplying the personal
interests which act in natural opposition to it (Howard 1978: 37). The

spread of markets would place societies on an

entirely new foundation. Instead of conflicts over limited resources such as land, the industrial revolution raised the prospect of
unlimited and unprecedented prosperity for all: material production, so long as it was
freely exchanged, would bring human progress. Trade would create relations of mutual dependence which
would foster understanding between peoples and reduce conflict. Economic self-interest would then be
a powerful disincentive for war. Liberals have always felt that unfettered commercial exchanges would encourage
links across frontiers and shift loyalties away from the nationstate. Leaders would eventually come to
recognize that the benefits of free trade outweighed the costs of territorial conquest and colonial expansion. The attraction of going to war to promote
mercantilist interests would be weakened as societies learn that war can only disrupt trade and therefore the prospects for economic prosperity.
Interdependence would replace national competition and defuse unilateral acts of aggression and reciprocal retaliation.

Lib Solves War


Liberalism is key to solve war. All other solutions lead to chaos.
Burchill 05 Researcher of International Relations at Deakin University (Scott, Theories of
International Relations Third edition, Chapter 3 Liberalism, pp. 55-83)//DWB
For liberals, peace is the normal state of affairs: in Kants words, peace can be perpetual. The
laws of nature dictated harmony and cooperation between peoples. War is therefore both unnatural and irrational,
an artificial contrivance and not a product of some peculiarity of human nature. Liberals
have a belief in progress and the perfectibility of the human condition. Through their faith in the
power of human reason and the capacity of human beings to realize their inner potential, they remain confident that the stain of
war can be removed from human experience (Gardner 1990: 2339; Hoffmann 1995: 15977; Zacher and Matthew 1995: 107
50). A common thread, from Rousseau, Kant and Cobden, to Schumpeter and Doyle, is that

wars were created by

militaristic and undemocratic governments for their own vested interests. Wars were
engineered by a warrior class bent on extending their power and wealth through territorial conquest. According to Paine in The
Rights of Man, the war

system was contrived to preserve the power and the employment of princes,
statesmen, soldiers, diplomats and armaments manufacturers, and to bind their tyranny ever more firmly upon
the necks of the people (Howard 1978: 31). Wars provide governments with excuses to raise taxes, expand their bureaucratic
apparatus and increase their control over their citizens. The people, on the other hand, were

peace-loving by
nature, and plunged into conflict only by the whims of their unrepresentative rulers. War
was a cancer on the body politic. But it was an ailment that human beings, themselves, had the capacity to cure. The treatment

the disease of war could be


successfully treated with the twin medicines of democracy and free trade. Democratic
processes and institutions would break the power of the ruling elites and curb their propensity for violence. Free trade and
commerce would overcome the artificial barriers between individuals and unite them
everywhere into one community. For liberals such as Schumpeter, war was the product of the
aggressive instincts of unrepresentative elites. The warlike disposition of these rulers drove the reluctant
which liberals began prescribing in the eighteenth century had not changed:

masses into violent conflicts which, while profitable for the arms industries and the military aristocrats, were disastrous for those

republican forms of government in which rulers were


lead to peaceful international relations because
the ultimate consent for war would rest with the citizens of the state (Kant 1970: 100). For both
Kant and Schumpeter, war was the outcome of minority rule, though Kant was no champion of democratic
who did the fighting. For Kant, the establishment of

accountable and individual rights were respected would

government (MacMillan 1995). Liberal states, founded on individual rights such as equality before the law, free speech and civil
liberty, respect for private property and representative government, would not have the same appetite for conflict and war.

Peace was fundamentally a question of establishing legitimate domestic orders


throughout the world. When the citizens who bear the burdens of war elect their governments, wars become
impossible (Doyle 1986: 1151).

I/R = Global Stability


International relations guarantee global stability even tense relations
wont escalate
Shuja 01 - Adjunct Assistant Professor of International Relations at Bond University, Australia
(January 1, Sharif, The Historical Myopia of International Relations, Contemporary Review,
Vol. 278,
http://www.thefreelibrary.com/THE+HISTORICAL+MYOPIA+OF+INTERNATIONAL+RELATION
S.-a070396453)//DWB
ALL nations are members of the community of nations and part of the international
political system; each needs to deal in some measure with the other members. No one can stop the world and get off. We have to share the
planet with each other, and the way in which we share it is what foreign policy is all
about. Some nations depend a great deal on the international community for their security, their economic development and their way of life. Others may flourish in a more
self-reliant or autarchic way. The international system has been described by some writers as representing a billiard table, on which the 191 or so sovereign independent

, seeking only their own national interests;


much of the energy generated by these collisions gives rise to conflict and war. It is argued
that there is no guiding hand, no predictable behaviour, no pattern or order; the key
principle is survival of the fittest; violence and war are integral to the system. At one
extreme is thus pure conflict. It occurs when the interests of actors, such as states, or non-state actors, such as multinational corporations, are
countries behave like balls which bounce off each other in an unpredictable fashion

diametrically opposed: if one actor realises its goal, the other cannot achieve its objective. For example, in a conflict over territory, one state will gain the territory and the other

At the other extreme, the system is painted as a cobweb of relationships in which


states, in spite of their formal claims to sovereignty and to complete control over their own affairs, are in fact closely linked together in
an interdependence they cannot escape. It is an interdependence based on common global needs of economic development,
will lose it.

environmental protection, ideological convergence and the need to avoid war. In such a situation, actors have a common interest, and all benefit from the pursuit of this shared

Allies

have a common interest in ensuring their common defence,

interest.
, for example,
or colonies often have a
common interest in achieving independence, or trading partners have a common interest in maintaining beneficial trading relations. For large and small states alike,

survival and welfare depend on close cooperation. Violence and war are seen as aberrations, rather than the norm, since they
destroy the fabric of the cobweb and everyone loses. Because of this, most states recognise that violence is more or less
becoming counter-productive and that more and more of their sovereignty must be
surrendered for the common goal. The cobweb represents an ordered framework of behaviour which the great majority of states adhere to for
most of the time. Most international interaction contains elements of both conflict and cooperation.
Even in situations of extreme conflict, there is often an element of co-operation . For example,
despite the confrontation between the United States and the Soviet Union over placing missiles in Cuba in 1962, both superpowers' interest in preventing the escalation of these

Conversely, in situations involving high levels of


co-operation, there is often an element of conflict; and even when actors share interests,
there is usually conflict over specific interests and specific solutions. For example, all states may want to
conflicts into nuclear war and holocaust led to co-operation as their resolution.

establish and maintain a stable international monetary system; but some of these states may prefer a particular type of monetary system, such as a fixed exchange rate or a

within a framework of common goals, as argued


in 1992, in The Politics of International Economic Relations, by Joan Edelman Spero, states disagree over the best means to
achieve their common end.
floating exchange rate regime, which satisfies their more specific national interests. Thus,

Institutions k Peace
International institutions are key to ensure a safe global political
environment
Shuja 01 - Adjunct Assistant Professor of International Relations at Bond University, Australia
(January 1, Sharif, The Historical Myopia of International Relations, Contemporary Review,
Vol. 278,
http://www.thefreelibrary.com/THE+HISTORICAL+MYOPIA+OF+INTERNATIONAL+RELATION
S.-a070396453)//DWB
there are a number of major global issues with
which policy makers of all nations grapple. These are the imminence of violence; the containing and resolving
Irrespective of the view we have of the international system,

of international conflict; the development gap between the wealthy few and the underdeveloped many; the protection of the
environment, and the protection of human rights. Greater

levels of co-operation between nation states are


needed in meeting these challenges. In domestic politics, goal-seeking behaviour is
regulated by government, which has the authority to make decisions for a society and
the power to enforce those decisions. The characteristic that distinguishes international
politics from internal politics is the absence of government. In the international system, no
legitimate body has the authority to manage conflict or achieve common goals by
making and enforcing decisions for the system; instead, the decision-making authority is
dispersed among many governmental, inter-governmental and non-governmental
groups. Because there is no international government, the central problems of international politics are
the adjustment or management of conflict and the achievement of co-operation. Over the
years, actors have devised rules, institutions and procedures to manage international
conflict and co-operation. These forms of managing international order have varied over time and issues. They range
from balances of power to alliances and to international organisations, from hegemony to colonialism and to international law. Even
during the Cold War period, efforts were made to contain and reduce armed conflict through several 'layers of management'.
Firstly, the two superpowers - the USA and the USSR - established a loose duopoly, and between them they managed global
strategy, more or less effectively, at the nuclear level and they controlled, to a considerable degree, the behaviour of their allies and
clients at other levels, all according to a set of mostly tacit rules and understandings. Secondly, this superpower 'management' was
codified and elaborated by a range of explicit bilateral and multilateral regional alliances between each superpower and its
respective allies. This network

of military arrangements, though sometimes brittle and unstable,


has brought about a reasonably 'safe' distribution of military power. At the third level, the
international community has put in place the apparatus of global collective security in
the United Nations. Many of the key provisions of the UN Charter, such as, the role of the Security Council, the
establishment of peace-keeping forces, the process of the peaceful resolution of conflict, the International Court of Justice and so

different layers of management in


practice merge into each other. Together they give us the relative degree of safety from
violence that we currently enjoy.
on, are all designed to prevent or limit the use of violence between nations. These

N/L: Lib =/= Conflict Focus


Liberalism focuses on more than just political power it recognizes the
importance of everyday interactions instead of just conflicts
Shuja 01 - Adjunct Assistant Professor of International Relations at Bond University, Australia
(January 1, Sharif, The Historical Myopia of International Relations, Contemporary Review,
Vol. 278,
http://www.thefreelibrary.com/THE+HISTORICAL+MYOPIA+OF+INTERNATIONAL+RELATION
S.-a070396453)//DWB
A leading exponent of realism is Hans Morgenthau. Realists

focus on power and conflict as the dynamic


elements of international politics and pay particular attention to the military dimensions
of power. States are seen to be motivated by the pursuit of their national interests; in
pursuing those interests, states are influenced by the prevailing 'balance of power'. Although the concept of 'balance of power' can
be interpreted in various ways, it may be defined

as the way in which power is distributed within


international politics, particular among the major states. A more recent version of realism is
'neorealism'. Kenneth Waltz is often considered its leading exponent. Neorealism is not so much concerned
with the alleged metaphysical dimensions of power as with the logical character of state
behaviour in an anarchical world. This theory contends that a major influence on a state's behaviour is the fact that it
has to look to its survival in a state of anarchy. Neorealists believe that the structure of the system enables them to predict the

one can
consider liberalism or liberal institutionalism as an alternative to realism and neorealism .
This approach puts more emphasis on the economic dimensions of power. The state
remains important but less so than in the realist paradigm, and more attention is given to the ways in
likelihood of a state's actions given that particular state's location in this anarchical world.

On the other hand,

which states cooperate as well as compete. Interdependence is suggested as an alternative. The latter term points to the way in

liberalism emphasises the role of


'low politics', liberal theorists pay more attention to the 'everyday' character of these
interactions and distinguish them from the more dramatic developments in the 'high
politics' of realism.
which the various actors in the world are linked in a variety of ways. Because

Lib/Govt k Stability
Sovereign governments are key to ensure political stability worth it even
at a small violation of rights
Shuja 01 - Adjunct Assistant Professor of International Relations at Bond University, Australia
(January 1, Sharif, The Historical Myopia of International Relations, Contemporary Review,
Vol. 278,
http://www.thefreelibrary.com/THE+HISTORICAL+MYOPIA+OF+INTERNATIONAL+RELATION
S.-a070396453)//DWB
These perspectives

are helpful in understanding the dynamics of international politics. But


the peculiarities of the international system insist that each nation state has a
compulsion to strengthen its economy, to deepen its defence capability and to maximise
its economic welfare, if necessary at the expense of other states. To achieve these
objectives, each nation state maintains sovereign control over its resources and its
people, and uses and disposes of both as it thinks fit. The system expects that each
nation state play a self-interested game in order to survive. Where then is there a place for principles of
humanity, compassion and morality in efforts to reduce the development gap? To what extent can Great Powers in the post-Cold
War period participate in attempts to introduce more equity, peace and order into the international community? Despite hopes to the
contrary, the post-Cold War world has not witnessed the end of international conflict. In Africa, Asia, Eastern Europe and the Middle
East, issues of war and peace continue to present challenges to the international community. More than ever, world

problems require careful thinking, creative research, fresh ideas, and practical
approaches, if they are to be solved. These questions are relevant since the world now is
becoming a global village. And we are all enmeshed in a 'cobweb of interdependence', and we all have to deal with
these difficult global issues. These, along with other issues including globalisation and an expansion of information technology that
have given rise to a new wave of changes in international relations, will be the focus of my next article.

Lib Fails
Liberalism fails through a litany of reasons inconsistent theorists,
unregulated political judging, no brightline to objective-subjective politics
McCormick 99
(John P., Assistant Political Science Pf - Yale, American Political Science Review. v93, p. June
1999)//akim
In the 1920s Schmitt

criticized liberal legal theory for avoiding the reality of jurisprudence by


denying the existence of gaps within the law: the fact that the formal rules of statutory law
cannot possibly cover all instances of concrete reality. Liberalism (1) demotes judges to
the status of mere vending machines that mechanically dispense the law upon cases,
without intellectual reflection or active contribution, and (2) leaves the legal theorist
inadequately equipped to analyze exactly how the law is applied. According to Schmitt's
"decisionist" alternative to liberalism, only a person, not a rule within a larger system, can determine
how to enforce or realize the law. Purely formal jurisprudence endangers the "personality" of judges, their ability to engage in the
concrete particularity of a given case, by confining them to the mechanical application of a pre-given statute. For Schmitt, between the law
and concrete reality there will always be a gap that must be mediated by a judge. This is the
"humanity" or "life" of the law. According to Schmitt's account, liberal jurists refuse to acknowledge anything but formal
imperatives in adjudication.6 Moreover, they pretend that the law is applied consistently and appropriately
most or all of the time. CLS ultimately indicts postwar North American liberal adjudication on the
same grounds. Less concerned with metaphysical assertions about juridical "reality" or "life" than Schmitt-indeed, questioning the very
existence of such phenomena-CLS points primarily to the uncertain nature of meaning in language
as the source of the indeterminacy of deciding cases. Legal rules are so far from clear regarding where and how
they should be applied that there is no possibility of their being applied consistently or objectively. According to CLS, this shakes the very
foundations of liberal formalist notions of efficiency and justice. As a result of this lack of determinacy, both Schmitt and CLS conclude that, to some
extent, judges make the law. The difference is that CLS wishes to expose the subjective decision making that goes on behind the supposedly
objective formal rule of law in order to undermine the generally conservative rulings of judges. Schmitt wants to emphasize the personal quality of
decisions so that conservative rulings may be rendered with less liberal constraint. But the theories may converge in more fundamentally
disconcerting ways, as will be seen below. The

subjective-judging criticism also allows both [foot notes] 414


Schmitt and CLS to criticize legal liberalism for falsely separating politics from the law.
For both there is no sharp distinction between objective law and subjectively biased
politics. Such scholars as Tushnet (1984, 1986, 1988), Boyle (1985), Unger (1975), -and most recently and elaborately Kennedy (1997) argue
that political value judgments infiltrate the law at every turn. The law embodies specific political and, especially, economic values, such as self-

There is no consensus in the


CLS literature on the exact nature of these judicial prejudices (see Tushnet 1981). Kennedy (1997, 1-5,
interest, individualism, and advantage, and it reflects the personal prejudices of particular judges.

115) is perhaps most cynical in his assertion that judges generally pursue "ideological projects" of varying stripes but mostly decide cases so as to
curry favor with, and secure the interests of, what he calls the "intelligentsia"-New York intellectuals and Beltway insiders. Moreover, judges deny
such strategic political behavior, according to Kennedy (1997, 5), in deliberate "bad faith."7 CLS acknowledges the presence of more progressive
countervalues in legal discourse, such as solidarity and altruism, which also constitute the human psyche, but these seem overwhelmed by the
more conservative ones mentioned above (Tushnet 1985). Worse

still for CLS, the very pervasiveness of


contradictory values embedded in different laws and precedents makes manifest the
incoherence of the whole system, a system in perpetual war with itself. For Schmitt as well, legal
formalism is compromised politically, despite claims of being insulated from politics. The fact that the "personality" of judges interacts with the law
makes for unacknowledged state action. The emphasis on "abstractly

valid" principles, in particular, betrays


liberalism's prejudice against Schmitt's preferred model; namely, the more subjective, personalist theories of
judging associated with European absolutism. Yet, as a result of this denial and bias, liberalism allows such
political judging to proliferate undetected, unmonitored, and unregulated.

Institutions K Solve Warm


Their call to reject all existing institutions guarantees warming happens
Monbiot 8
(George, English Writer and Environmental and Political Activist, 9-4, Identity Politics in Climate
Change Hell, http://www.celsias.com/article/identity-politics-climate-change-hell/)
If you want a glimpse of how the movement against climate change could crumble faster than a summer snowflake, read Ewa Jasiewiczs article , published on the Guardians Comment is Free site. It is a fine

identity politics

example of the
that plagued direct action movements during the 1990s, and from which the new generation of activists has so far been mercifully free. Ewa rightly celebrates
the leaderless, autonomous model of organising that has made this movement so effective. The two climate camps I have attended this year and last were among the most inspiring events Ive ever
witnessed. I am awed by the people who organised them, who managed to create, under extraordinary pressure, safe, functioning, delightful spaces in which we could debate the issues and plan the actions
which thrust Heathrow and Kingsnorth into the public eye. Climate camp is a tribute to the anarchist politics that Jasiewicz supports. But in seeking to extrapolate from this experience to a wider social plan, she

claims to want to stop global warming, but she makes that


task 100 times harder by rejecting all state and corporate solutions. It seems to me that what she really wants to do is to
create an anarchist utopia, and use climate change as an excuse to engineer it. Stopping runaway climate change must take
precedence over every other aim. Everyone in this movement knows that there is very little time: the window of
opportunity in which we can prevent two degrees of warming is closing fast. We have to
use all the resources we can lay hands on, and these must include both governments and corporations.
makes two grave errors. The first is to confuse ends and means. She

Or perhaps she intends to build the installations required to turn the energy economy around - wind farms, wave machines, solar thermal plants in the Sahara, new grid connections and public transport systems herself? Her article is a terryifying example of the ability some people have to put politics first and facts second when confronting the greatest challenge humanity now faces. The facts are as follows.

Runaway climate change is bearing down on us fast. We require a massive political and
economic response to prevent it. Governments and corporations, whether we like it or
not, currently control both money and power. Unless we manage to mobilise them, we
stand a snowballs chance in climate hell of stopping the collapse of the biosphere. Jasiewicz
would ignore all these inconvenient truths because they conflict with her politics. Changing our sources of energy without changing our sources of economic and political power, she asserts, will not make a

before we are allowed to begin cutting greenhouse gas


emissions, we must first overthrow all political structures and replace them with
autonomous communities of happy campers. All this must take place within a couple of
months, as there is so little time in which we could prevent two degrees of warming. This
is magical thinking of the most desperate kind. If I were an executive of E.On or Exxon, I would
be delighted by this political posturing, as it provides a marvellous distraction from our
real aims. To support her argument, Jasiewicz misrepresents what I said at climate camp. She claims that I confessed not knowing where to turn next to solve the issues of how to generate the
difference. Neither coal nor nuclear are the solution, we need a revolution. So

changes necessary to shift our sources of energy, production and consumption. I confessed nothing of the kind. In my book Heat I spell out what is required to bring about a 90% cut in emissions by 2030.
Instead I confessed that I dont know how to solve the problem of capitalism without resorting to totalitarianism. The issue is that capitalism involves lending money at interest. If you lend at 5%, then one of two
things must happen. Either the money supply must increase by 5% or the velocity of circulation must increase by 5%. In either case, if this growth is not met by a concomitant increase in the supply of goods and
services, it becomes inflationary and the system collapses. But a perpetual increase in the supply of goods and services will eventually destroy the biosphere. So how do we stall this process? Even when usurers
were put to death and condemned to perpetual damnation, the practice couldnt be stamped out. Only the communist states managed it, through the extreme use of the state control Ewa professes to hate. I dont

Yes, let us fight both corporate power and the undemocratic


tendencies of the state. Yes, let us try to crack the problem of capitalism and then fight for a different system. But let us not confuse this
task with the immediate need to stop two degrees of warming, or allow it to interfere with
the carbon cuts that have to begin now. Ewas second grave error is to imagine that society could be turned into a giant climate camp. Anarchism is a great means of organising a self-elected
yet have an answer to this conundrum. Does she?

community of like-minded people. It is a disastrous means of organising a planet. Most anarchists envisage their system as the everyone is to be free from the coercive power of the state, this must apply to the
oppressors as well as the oppressed. The richest and most powerful communities on earth - be they geographical communities or communities of interest - will be as unrestrained by external forces as the poorest

if anarchist utopia arrives, the first


thing that will happen is that every Daily Mail reader in the country will pick up a gun and
go and kill the nearest hippy. This is why, though both sides furiously deny it, the
outcome of both market fundamentalism and anarchism, if applied universally, is
identical. The anarchists associate with the oppressed, the market fundamentalists with the oppressors. But by eliminating the state, both remove such restraints as prevent the strong from crushing
the weak. Ours is not a choice between government and no government. It is a choice between
government and the mafia. Over the past year I have been working with groups of climate protesters who have changed my view of what could be achieved. Most of them are
and weakest. As a friend of mine put it, when the means by which the oppressed can free themselves from persecution. But

under 30, and they bring to this issue a clear-headedness and pragmatism that I have never encountered in direct action movements before. They are prepared to take extraordinary risks to try to defend the
biosphere from the corporations, governments and social trends which threaten to make it uninhabitable. They do so for one reason only: that they love the world and fear for its future. It would be a tragedy if,
through the efforts of people like Ewa, they were to be diverted from this urgent task into the identity politics that have wrecked so many movements.

Institutions Inevitable
You cant just wish away institutions
McCormack 10
(Tara, Leicester international politics lecturer, Critique, Security and Power: The Political Limits
to Emancipatory Approaches, pg 58)
Contemporary critical and emancipatory approaches

reject the possibility of reaching an objective


evaluation of the world or social reality because they reject the possibility of differentiating
between facts and values. For the contemporary critical theorists, theory can only ever be for someone
and for some purpose. As this is so then quite logically critical theorists elevate their own values to be the most important aspect of
critical theory. As a result of the rejection of the fact/value distinction we see within the work of
contemporary critical theorists a highly unreflective certainty about the power of their moral
position. Critical theorists argue that all theory is normative, they offer in its place better
norms: ones, as we have seen, that will lead to emancipation and will help the marginalised. The claims made
for the central role of the values of the theorist reveal the theoretical limits of critical and
emancipatory theory today. Yet even good or critical theory has no agency, and only political
action can lead to change. Theory does of course play an important role in political change. This must be the first step towards a
critical engagement with contemporary power structures and discourses. In this sense, we can see that it is critical theory that really has the potential to
solve problems, unlike problem-solving theory which seeks only to ensure the smooth functioning of the existing order. Through substantive analysis
the critical theorist can transcend the narrow and conservative boundaries of problem-solving theory by explaining how the problematic arises. Unlike
problem-solving theory, critical theory makes claims to be able to explain why and how the social world functions as it does, it can go beyond the given
framework for action. The

critical theorist must therefore be able to differentiate between facts

(or social reality) and values , this ability is what marks the critical theorist apart from the traditional or problem-solving theorists,
who cannot, because of their values and commitment to the existing social world, go beyond the given framework for action. If we cannot
differentiate between our desires or values or norms (or our perspective, to put it in Coxs terms) and actually
occurring social and political and historical processes and relationships, it is hard to see
how we can have a critical perspective (Jahn, 1998: 614). Rather, through abolishing this
division we can no longer draw the line between what we would like and everything else,
and thereby contemporary critical theories are as much of a dogma as problem-solving
theories. Contemporary critical theorists are like modern-day alchemists, believing that
they can transform the base metal of the unjust international order into a golden realm
of equality and justice through their own words. For contemporary critical theorists, all that seems that
the crucial step towards progress to a better world order is for the theorist to state that
their theory is for the purposes of emancipation and a just world order.

Wilderson/Anti-Blackness

AT: structural antagonism


Antiblackness is not the foundational structure of violence their analysis
is overdetermined by discrete historical events that do not make sense
together their reading of black social life is ontological fatalism, this is
based on social factors that are subject to change
Hudson 13
(Peter Hudson 13, Political Studies Department, University of the Witwatersrand,
Johannesburg , South Africa, has been on the editorial board of the Africa Perspective: The
South African Journal of Sociology and Theoria: A Journal of Political and Social Theory and
Transformation, and is a member of the Johannesburg Workshop in Theory and Criticism, The
state and the colonial unconscious, Social Dynamics: A journal of African studies, 2013)
There always has to exist an outside, which is also inside,
to the extent it is designated as the impossibility from which the possibility of the
existence of the subject derives its rule (Badiou 2009, 220). But although the excluded place which
isnt excluded insofar as it is necessary for the very possibility of inclusion and identity
may be universal (may be considered ontological), its content (what fills it) as well as the mode of this
filling and its reproduction are contingent. In other words, the meaning of the signifier of exclusion is
not determined once and for all: the place of the place of exclusion, of death is itself overdetermined, i.e. the very framework for deciding the other and the same, exclusion and
inclusion, is nowhere engraved in ontological stone but is political and never terminally
settled. Put differently, the curvature of intersubjective space (Critchley 2007, 61) and thus, the specific modes of
the othering of otherness are nowhere decided in advance (as a certain ontological fatalism
might have it) (see Wilderson 2008). The social does not have to be divided into white and
Thus the self-same/other distinction is necessary for the possibility of identity itself.

black , and the meaning of these signifiers is never necessary because they are signifiers.
To be sure, colonialism institutes an ontological division, in that whites exist in a way barred to blacks who are not. But this
ontological relation is really on the side of the ontic that is, of all contingently constructed
identities, rather than the ontology of the social which refers to the ultimate unfixity, the
indeterminacy or lack of the social. In this sense, then, the white man doesnt exist, the black man doesnt exist (Fanon 1968, 165); and
neither does the colonial symbolic itself, including its most intimate structuring relations division is constitutive of the social, not the
colonial division. Whiteness may well be very deeply sediment in modernity itself, but respect for
the ontological difference (see Heidegger 1962, 26; Watts 2011, 279) shows up its ontological status as ontic. It may be so deeply sedimented
that it becomes difficult even to identify the very possibility of the separation of
whiteness from the very possibility of order, but from this it does not follow that the void of black
being functions as the ultimate substance, the transcendental signified on which all
possible forms of sociality are said to rest . What gets lost here, then, is the specificity of
colonialism, of its constitutive axis, its ontological differential. A crucial feature of the colonial symbolic is that the real is not screened
off by the imaginary in the way it is under capitalism. At the place of the colonised, the symbolic and the imaginary give way
because non-identity (the real of the social) is immediately inscribed in the lived experience (vcu) of
the colonised subject. The colonised is traversing the fantasy (Zizek 2006a, 4060) all the time; the void of the verb to be is the very content of his
interpellation. The colonised is, in other words, the subject of anxiety for whom the symbolic and the

imaginary never work, who is left stranded by his very interpellation.4 Fixed into non-fixity, he is eternally
suspended between element and moment5 he is where the colonial symbolic falters in
the production of meaning and is thus the point of entry of the real into the texture itself
of colonialism. Be this as it may, whiteness and blackness are (sustained by) determinate and
contingent practices of signification; the structuring relation of colonialism thus itself comprises a
knot of significations which, no matter how tight, can always be undone. Anti-colonial i.e.,
anti-white modes of struggle are not (just) psychic 6 but involve the reactivation (or de-sedimentation)7 of
colonial objectivity itself. No matter how sedimented (or global), colonial objectivity is not ontologically immune to antagonism. Differentiality, as Zizek
insists (see Zizek 2012, chapter 11, 771 n48), immanently entails antagonism in that differentiality both makes possible the existence of any identity whatsoever and at the same
time because it is the presence of one object in another undermines any identity ever being (fully) itself. Each element in a differential relation is the condition of possibility
and the condition of impossibility of each other. It is this dimension of antagonism that the Master Signifier covers over transforming its outside (Other) into an element of itself,

symbolisation produces an ineradicable excess over itself,


something it cant totalise or make sense of, where its production of meaning falters. This is its internal limit point, its
real:9 an errant object that has no place of its own, isnt recognised in the categories of the
system but is produced by it its part of no part or object small a.10 Correlative to this object a is the
subject stricto sensu i.e., as the empty subject of the signifier without an identity that pins it down.11 That is the subject of antagonism in
confrontation with the real of the social, as distinct from subject position based on a
determinate identity.
reducing it to a condition of its possibility.8 All

Afro-pessimisms dogmatism reifies the failures of defining static identities


blackness is reduced to incapacity which recreates violence of
universality by actively refusing to understand contingency
David Marriott 12, Black Cultural Studies, Years Work Crit Cult Theory (2012) 20 (1): 37-66
However, this is also not the entire story of Red, White, and Black, as I hope to show. For example, in Chapter One (The Structure of Antagonisms), written as a theoretical

Wilderson is prepared to
say that black suffering is not only beyond analogy, it also refigures the whole of being: the
introduction, and which opens explicitly on the Fanonian question of why ontology cannot understand the being of the Black,

essence of being for the White and non-Black position is non-niggerness, consequently, [b]eing can thus be thought of, in the first ontological instance, as non-niggerness, and

It is not hard when reading such sentences to suspect a kind of


absolutism at work here, and one that manages to be peculiarly and dispiritingly
slavery then as niggerness (p. 37).

dogmatic : throughout Red, White, and Black, despite variations in tone and emphasis, there is always the desire to have black
lived experience named as the worst , and the politics of such a desire inevitably
collapses into a kind of sentimental moralism : for the claim that Blackness is incapacity in
means merely that the black has to embody this abjection without
reserve (p. 38). This logicand the denial of any kind of ontological integrity to the Black/Slave due
to its endless traversal by force does seem to reduce ontology to logic , namely, a logic of nonits most pure and unadulterated form

recuperability moves through the following points: (1) Black non-being is not capable of symbolic resistance and, as such, falls outside of any language of
authenticity or reparation; (2) for such a subject, which Wilderson persists in calling death, the symbolic remains foreclosed (p. 43); (3) as such, Blackness is the record of an
occlusion which remains ever present: White (Human) capacity, in advance of the event of discrimination or oppression, is parasitic on Black incapacity (p. 45); (4) and, as an

Wilderson describes White (or non-Black) film


theory and cultural studies as incapable of understanding the suffering of the Black
the Slave (they cannot do so because they are erroneously wedded to humanism and to the psychoanalysis of Jacques Lacan, which Wilderson takes as two
examples of what the Afro-pessimist should avoid) (p. 56); as a corrective, Wilderson calls for a new language of
abstraction, and one centrally concerned with exposing the structure of antagonisms
between Blacks and Humans (p. 68). Reading seems to stop here, at a critique of Lacanian full speech: Wilderson wants to say that Lacans notion
example of the institutions or discourses involving violence, antagonisms and parasitism,

of the originary (imaginary) alienation of the subject is still wedded to relationality as implied by the contrast between empty and full speech, and so apparently cannot grasp
the trauma of absolute Otherness that is the Blacks relation to Whites, because psychoanalysis cannot fathom the structural, or absolute, violence of Black life (pp. 74; 75).
Whereas Lacan was aware of how language precedes and exceeds us, he did not have Fanons awareness of how violence also precedes and exceeds Blacks (p. 76). The

violence of such abjectionor incapacityis therefore that it cannot be communicated or avowed, and is always already delimited by desubjectification and dereliction (p. 77).
Whence the suspicion of an ontology reduced to a logic (of abjection). Leaving aside the fact that it is quite mistaken to limit Lacans notion of full speech to the search for

according to Wildersons own logic, his


description of the Black is working, via analogy, to Lacans notion of the real but, in his
insistence on the Black as an absolute outside Wilderson can only duly reify this void at
communication (the unconscious cannot be confined to parole), it is clear that,

the heart of universality . The Black is beyond the limit of contingencybut it is worth saying
immediately that this beyond is indeed a foreclosure that defines a violence whose traces can
only be thought violently (that is, analogically), and whose nonbeing returns as the theme for
Wildersons political thinking of a non-recuperable abjection. The Black is nonbeing and,
as such, is more real and primary than being per se: given how much is at stake, this insistence on a racial
metaphysics of injury implies a fundamental irreconcilability between Blacks and
Humans (there is really no debate to be had here: irreconcilability is the condition and possibility of what it means to be Black).

Sequencing question immunity is the nexus of antagonism and gratuitous


violence recognizing the political bases of violence is key

Warren Montag, Brown Family Professor of European Literature at Occidental College, 12


(Between Interpellation and Immunization: Althusser, Balibar, Esposito, Postmodern Culture
Volume 22, Number 3, May 2012)
The work of Roberto Esposito on immunity may well appear to be inscribed in a different register than that of Althusser and Balibar. As a response to some of the theoretical problems engendered by Foucault's

it would
seem to address the question of the appropriation of life by politics that in a certain way
would allow us to sidestep or circumvent the question of the subject altogether. Reading Esposito in
the light of the foregoing discussion, however, we cannot escape the sense that the opposition between communitas
and immunitas is another way to think the constitutive aporia of the subject and the
irreducible conflict at the heart of the notion of citizen. As Esposito demonstrates, the concept of
immunity is political before it is biological. Communitas or community is not only not "a property belonging to subjects that joins them together," the
totality to which they would pertain before their separation into individuals; it is not even a thing (the res publica) that could be substantified as that which individuals "share" or possess (Communitas 8). If
we can speak of a sharing, it would be the sharing of a munus, an obligation, a debt, a
tax, even the paradox of an obligatory gift, the sharing of which Esposito describes as a
lack or void. Communitas is thus not more than the individual, but less; it does not add but subtracts and is finally the experience of destitution and expropriation (depropriazione) (6-7).
Such a subject is subject of and to debt and despoliation: an outside that marks its own
limit is rediscovered within, as that which separates each subject from itself. It is here, in a gesture whose
work on biopower, particularly that "interval of meaning which remains open in Foucault's text between the constitutive poles of the concept of biopolitics, namely biology and politics,"

timing in relation to the argument concerning the subject's relation to communitas as well as whose content can only remind us of Althusser, that Esposito turns to Scripture and the example not of Moses but of
Paul. Even fellowship, , in the community of the faithful, the , is a "taking part in" that entails a loss and a diminution, a participation in a loss of freedom that is or slavery in Christ, in
whose death one must participate (11). These, of course, are the images of subjection so total that the subjected subject is threatened with extinction, a destitution that leaves no remainder. It is in relation to the
"unacceptable" as well as "unbearable" munus, in which the subject is lost to itself at the heart of the Christian communitas, that the countermovement of immunization takes place. Immunity emerges as the
demand for exemption from debt, obligation, service, and above all from the shared debt of communitas, which is now experienced at the extreme as a claim on one's life and therefore as the threat of death.

Every social tie becomes a threat against which the subject must be immunized by as
complete a separation as possible from others whose claims on him can be ignored with
impunity (a term closely related to immunity). Immunity evacuates the void of the munus by emptying the
common: there is nothing in common. But the subject thus free from obligation and from
the movement of alienation and exteriorization represented by the munus, a subject therefore restored to the self-sovereignty and selfproprietorship that community had denied it--the subject, that is, in the modern sensecan only become and
remain a subject in this sense by virtue of its subjection to that which will protect or
immunize it against the threats to its autonomy. The subject thus can exercise

sovereignty over itself (over its life, property, and liberty) only to the extent that it is subjected or subjects
itself to the sovereign power capable of assuring its immunity. To translate this into Althusser's idiom, there is no subject
except by and through subjection. It is, at least in part, on the terrain of Hobbes's philosophy that the link
between interpellation and immunization becomes intelligible: "the Leviathan-state coincides with the breaking [la
dissociazione] of every communitarian bond, with the squelching [l'abolizione] of every social relation that is foreign to the vertical exchange of protection-obedience" (14). As Esposito argues, this state is the

the entire transindividual


dimension from which the juridical individual cannot be disentangled and extricated
without violence and loss and without being deprived of that part of himself that is in common with
others and nevertheless proper to him. In fact, taken to its logical conclusion, which is, as Esposito
notes, paradoxically a remnant of irrationality, individuals are "preserved" and "protected"
only at the cost of the very sociality that makes their existence in a material as well as cultural sense possible: "they live in and of their refusal to live together
[convivere]" (14). Every possible combination, coagulation, and confusion in which the juridical
individual might become lost or dispersed - or, on the contrary, might become part of a composite singularity ranging from couples, in particular (the
denial of the very possibility of relation, not only of the cum, as if human beings are originally separated and unite only secondarily, but even of

INSERT DESCRIPTION - inline graphic or "one flesh" of the first couple--a phrase establishing that their separation could only be a tearing or laceration of the composite but irreducible body they had become), to

In fact, politics itself becomes


prevention, not only the prevention of war, but just as importantly a prophylaxis against
any exposure to others that would automatically carry the danger of transindividual
contagion and a weakening of the vertical relation of subjection. Hence the immunitary practice that both precedes and
collectivities of all kinds, whether "natural" or "artificial"-is abolished and replaced by the relation between subject and Subject.

accompanies interpellation as its simultaneous condition and result.

AT: Affirmation of Self


Their deployment of identity/social location/privilege arguments shuts
down effective collective action this reifies racism and leads to endless
squabbling about authenticity
Rob 13
(the Idealist, Carleton College, JD candidate, 10/1, Tim Wise & The Failure of Privilege
Discourse, www.orchestratedpulse.com/2013/10/tim-wise-failure-privilege-discourse/)
I dont find it meaningful to criticize Tim Wise the person and judge whether hes living up to some anti-racist bona fides. Instead, I choose to focus on the paradigm of White

Although the personal is political, not all


politics is personal; we have to attack systems . To paraphrase the urban poet and philosopher Meek Mill: there are
levels to this shit. How I Define Privilege There are power structures that shape individuals lived
experiences. Those structures provide and withhold resources to people based on factors like class, disability status,
privilege upon which his work is based, and its conceptual and practical limitations.

gender, and race. Its not a benefit to receive resources from an unjust order because ultimately, injustice is cannibalistic. Slavery binds the slave, but destroys the master. So,

the point then becomes not to assimilate the underprivileged, but to instead eradicate
the power structures that create the privileges in the first place. The conventional wisdom on privilege often says
that its benefits are unearned. However, this belief ignores the reality and history that privilege is earned and maintained through violence. Systemic advantages are

The
history and modern reality of violence is why Tim Wise comparison between whiteness and
tallness fails. White supremacy is not some natural evolution, nor did it occur by
happenstance. White folks *murdered* people for this thing that we often call White privilege; it was bought and paid for by blood and terror. White
supremacy is not some benign invisible knapsack. The same interplay between violence and advantage is true of any systemic
hierarchy (class, gender, disability, etc). Being tall, irrespective of its advantages, does not follow that pattern of violence. Privilege is Failing Us Unfortunately, I
think our use of the term privilege is no longer a productive way for us to gain a
thorough understanding of systemic injustice, nor is it helping us to develop collective
strategies to dismantle those systems. Basically , I never want to hear the word privilege
allocated and secured as a class, and simply because an individual hasnt personally committed the acts, it does not render their class dominance unearned.

again because the term is so thoroughly misused at this point that it does more harm than
good. Andrea Smith, in the essay The Problem with Privilege, outlines the pitfalls of misapplied privilege theory. Those who had little
privilege did not have to confess and were in the position to be the judge of those who did have privilege. Consequently,
people aspired to be oppressed. Inevitably, those with more privilege would develop new
heretofore unknown forms of oppression from which they suffered Consequently, the goal became not to
actually end oppression but to be as oppressed as possible. These rituals often substituted
confession for political movement-building. Andrea Smith, The Problem with Privilege Dr. Tommy Curry says it more bluntly, Its
not genius to say that in an oppressive society there are benefits to being in the superior
class instead of the inferior one. Thats true in any hierarchy, thats not an aha moment. Conceptually, privilege is best used when narrowly
focused on explaining how structures generally shape experiences. However, when we overly personalize the problem, then
privilege becomes a tit-for-tat exercise in blame, shame, and guilt. In its worst manifestations, this
dynamic becomes oppression Olympics and people tally perceived life advantages and
identities in order to invalidate one another. At best, we treat structural injustice as a personal
problem, and moralizing exercises like privilege confessions inadequately address the
nexus between systemic power and individual behavior. The undoing of privilege occurs
not by individuals confessing their privileges or trying to think themselves into a new subject position, but
through the creation of collective structures that dismantle the systems that enable these privileges. The

activist genealogies that produced this response to racism and settler colonialism were not initially focused on racism as a problem of individual prejudice. Rather, the purpose
was for individuals to recognize how they were shaped by structural forms of oppression. Andrea Smith, The Problem with Privilege Bigger than Tim Wise However, the problem
with White privilege isnt simply that Tim Wise, a white man, can build a career off of Black struggles. As Ive already said, White people need to talk to White people about the
historical and social construction of their racial identities and power, and the foundation for that conversation often comes from past Black theory and political projects. The
problem for me is that

privilege work has become a cottage industry of self-help moralizing that in

no way attacks the systemic ills that create the personal injustices in the first place. A
substantive critique of privilege requires us to get beyond identity politics . Its not about
good people and bad people; its a bad system. Its not just White people that participate in the White privilege industry, although
not everyone equally benefits/profits (see: Tim Wise). Dr. Tommy Curry takes elite Black academics to task for their role in
profiting from the White privilege industry while offering no challenge to White
supremacy. These conversations about White privilege are not conversations about race, and certainly not about racism; its a business where Blacks market
themselves as racial therapists for White people The White privilege discourse became a bourgeois distraction. Its a
tool that we use to morally condemn whites for not supporting the political goals of elite
black academics that take the vantages of white notions of virtue and reformism and
persuade departments, journals, and presses into making concessions for the benefit of a select
species of Black intellectuals in the Ivory Tower, without seeing that the white racial
vantages that these Black intellectuals claim theyre really interested in need to be
dissolved, need to be attacked all the way to the very bottom of American society. Dr. Tommy
Curry, Radio Interview The truth is that a lot of people, marginalized groups included, simply want more
access to existing systems of power. They dont want to challenge and push beyond these systems; they just want to
participate. So if we continue to play identity politics and persist with a personal privilege
view of power, then we will lose the struggle . Barack Obama is president, yet White
supremacy marches on, and often with his help (record deportations, expanded a drone war based on profiling, fought on behalf of US corporations to
repeal a Haitian law that raised the minimum wage). Adolph Reed, writing in 1996, predicted the quagmire of identity politics in the Age of Obama. In Chicago, for instance,
weve gotten a foretaste of the new breed of foundation-hatched black communitarian voices; one of them, a smooth Harvard lawyer with impeccable do-good credentials and
vacuous-to-repressive neoliberal politics, has won a state senate seat on a base mainly in the liberal foundation and development worlds. His fundamentally bootstrap line was
softened by a patina of the rhetoric of authentic community, talk about meeting in kitchens, small-scale solutions to social problems, and the predictable elevation of process
over program the point where identity politics converges with old-fashioned middle-class reform in favoring form over substance. I suspect that his ilk is the wave of the future

Obamas
election and subsequent presidency has made it starkly clear that its not just White
people that can perpetuate White supremacy. Systems of oppression condition all members of society to accept systemic injustice,
and there are (unequal) incentives for both marginalized and dominant groups to perpetuate these structures. Our approaches to injustice must
reflect this reality. This isnt a nave plea for unity, nor am I saying that talking about
identities/experiences is inherently divisive. Many of these privilege discussions use empathy to build personal and collective character, and
there certainly should be space for us to work together to improve/heal ourselves and one another. People will always make mistakes and
our spaces have to be flexible enough to allow for reconciliation. Though we dont have
to work with persistently abusive people who refuse to redirect their behavior, theres a
difference between establishing boundaries and puritanism. Fighting systemic
marginalization and exploitation requires more than good character, and we cannot fetishize personal
in U.S. black politics. Adolph Reed Jr., Class Notes: Posing As Politics and Other Thoughts on the American Scene Although it has always been the case,

morals over collective action .

Ethics B4 Ontology
Ethics/language precedes ontology---black body doesnt have any fixed
meaning until its constructed in a discursive community---we arent
speaking from nowhere---the body can reject the ontological fixity of the
white gaze
Yancy 5
(George, Duquesne University, Whiteness and the Return of the Black Body, The Journal of
Speculative Philosophy 19.4 (2005) 215-241)
I write out of a personal existential context. This context is a profound source of knowledge connected to my "raced" body. Hence, I write from a place of lived embodied
experience, a site of exposure. In philosophy, the only thing that we are taught to "expose" is a weak argument, a fallacy, or someone's "inferior" reasoning power. The
embodied self is bracketed and deemed irrelevant to theory, superfluous and cumbersome in one's search for truth. It is best, or so we are told, to reason from nowhere. Hence,
the white philosopher/author presumes to speak for all of "us" without the slightest mention of his or her "raced" identity. Self-consciously writing as a white male philosopher,
Crispin Sartwell observes: Left to my own devices, I disappear as an author. That is the "whiteness" of my authorship. This whiteness of authorship is, for us, a form of authority;

to speak (apparently) from nowhere, for everyone, is empowering, though one wields
power here only by becoming lost to oneself. But such an authorship and authority is also
pleasurable: it yields the pleasure of self-forgetting or [End Page 215] apparent transcendence of the mundane and the particular, and the pleasure of power
expressed in the "comprehension" of a range of materials. (1998, 6) To theorize the Black body one must "turn to the
[Black] body as the radix for interpreting racial experience" (Johnson [1993, 600]).1 It is important to note that this
particular strategy also functions as a lens through which to theorize and critique whiteness; for the Black body's "racial" experience is fundamentally linked to the oppressive
modalities of the "raced" white body.

However, there is no denying that my own "racial" experiences or the

social performances of whiteness can become objects of critical reflection . In this paper, my
objective is to describe and theorize situations where the Black body's subjectivity, its
lived reality, is reduced to instantiations of the white imaginary, resulting in what I refer to
as "the phenomenological return of the Black body."2 These instantiations are embedded within and evolve out of the complex
social and historical interstices of whites' efforts at self-construction through complex acts of erasure vis--vis Black people. These acts of self-construction, however, are
myths/ideological constructions predicated upon maintaining white power. As James Snead has noted, "Mythification is the replacement of history with a surrogate ideology of

How I understand and theorize the body


relates to the fact that the bodyin this case, the Black bodyis capable of undergoing a
[white] elevation or [Black] demotion along a scale of human value" (Snead 1994, 4).

sociohistorical process of "phenomenological return" vis--vis white embodiment . The


body's meaningwhether phenotypically white or blackits ontology, its modalities of aesthetic performance, its comportment, its
"raciated" reproduction, is in constant contestation. The hermeneutics of the body, how
it is understood , how it is "seen," its "truth," is partly the result of a profound historical, ideological
construction. "The body" is positioned by historical practices and discourses. The body is codified as
this or that in terms of meanings that are sanctioned, scripted, and constituted through processes of negotiation that are embedded within and serve various ideological interests

The historical plasticity of the body , the fact that it is a


site of contested meanings, speaks to the historicity of its "being" as lived and meant within the
interstices of social semiotics. Hence: a) the body is less of a thing/being than a shifting/changing historical
that are grounded within further power-laden social processes.

meaning that is subject to cultural configuration/reconfiguration . The point here is to


interrogate the "Black body" as a "fixed and material truth" that preexists "its relations
with the world and with others "3 ; b) the body's meaning is fundamentally symbolic (McDowell 2001, 301),
and its meaning is congealed through symbolic repetition and iteration that emits certain signs and
presupposes

certain norms; and, c) the body is a battlefield, one that is fought over again and again across

the concept of the body provides only the


illusion of self-evidence , facticity, 'thereness' for something [End Page 216] fundamentally
ephemeral, imaginary, something made in the image of particular social groups" (301). On this score, it is
not only the "Black body" that defies the ontic fixity projected upon it through the white
gaze, and, hence, through the episteme of whiteness, but the white body is also
fundamentally symbolic, requiring demystification of its status as norm, the paragon of beauty, order,
innocence, purity, restraint, and nobility. In other words, given the three suppositions above, both the "Black body" and the "white
body" lend themselves to processes of interpretive fracture and to strategies of
particular historical moments and within particular social spaces. "In other words,

interrogating and removing the veneer of their alleged objectivity .

Wilderson = Reductive
Wilderson is overly reductive---he has no way to explain historical
resistance to anti-blackness because his theory pigeon holes all
oppression into the non-falsifiable register of psychoanalysis
B 11
(Sar Maty, prof of film at Portsmouth University, The US Decentred,
http://epress.lib.uts.edu.au/journals/index.php/csrj/article/view/2304/2474)
As we shall see below, blacks in the US cannot and do not have ontology, or so Wilderson argues, denying with the same breath the workability of analogy as a method, because analogy can only be a ruse.
Thus, what he calls the ruse of analogy grants those who fall for it, for example, Black film theorists or Black academics, an opportunity to reflect on (black) cinema only after some form of structural alteration.
(38) Analogy does seem tricky if one follows Wildersons line of thought, that is, the Holocaust/Jews and slavery/Africans. Jews entered and came out of Auschwitz as Jews whereas Africans emerged from the
slave ships as Blacks.2 Two types of holocaust: the first Human, the second Human and metaphysical, something which leads to Wilderson saying that the Jews have the Dead ... among them; the Dead have

for Wilderson, blacks are socially and ontologically dead

the Black among them. (38) It bears reiterating that


in the sense that the
black body has been violently turned into flesh, ripped apart literally and imaginatively, that it is a body vulnerably open, an object made available (fungible) for any subject and not in the world or civil society

differences between black and white ethical dilemmas


separate them dialectically into incompatible zones . As illustration Wilderson reflects on black women suffering in US prisons in the 1970s
the way white bodies are. (38) Furthermore, Wilderson argues that

and then juxtaposes the suffering with white womens concurrent public preoccupations in civil society. For example, the violence and neglect underwent by Safya Bukhari Alston3 in solitary confinement at the
Virginia Correctional Center for Women is linked to the similar plight of another black woman, Dorothy, in Haile Gerimas Bush Mama (1977) before Wilderson questions what both situations mean in relation to
images of [w]hite women burning bras in Harvard Square ... marching in ... Manhattan campaigning for equal rights. (135) Wildersons answer is that the images of female black pain and white activism are
irreconcilable precisely because they cannot be read against one another without such an exercise appearing intellectually sloppy. However,

he does not develop this

point , preferring instead to examine suffering through a libidinal economy (131) leading ,
predictably, to the conclusion that white radicalism, white political cinema and white supremacy
are one and the same thing. Most unfortunate though inevitable is the reason Wilderson gives to justify this: a socalled anti
Blackness that, [wilderson quote begins] as opposed to white apathy, is necessary to White political radicalism and to White political cinema because it sutures affective, emotional, and even
ethical solidarity between the ideological polar extremes of Whiteness. This necessary antiBlackness erects a structural prohibition that one sees in White political discourse and in White political cinema. (131)
[wilderson quote ends] undamentally, the first three chapters of Red, White and Black are concerned with what it takes to think blackness and agency together ethically, or to permit ourselves intellectual mindful
reflections upon the homicidal ontology of chattel slavery. Wilderson posits ways through which the dead (blacks) reflect on how the living can be put out of the picture. (143) There seems to be no let off or way
out for blacks (The Slave) in Wildersons logic, an energetic and rigorous, if unforgiving and sustained, treadmill of damning analysis to which Indians (The Savage/The Red) will also be subjected, first

yet Wildersons highlighting is problematic because it overlooks the Diaspora or African Diaspora, a
key component in Yearwoods thesis that, crucially, neither navel gazes (that is, at the US or black America) nor pretends
to properly engage with black film. Furthermore, Wilderson separates the different waves of black film theory and approaches them, only, in terms of how a most
through Savage film analysis. <cont> And

recent one might challenge its precedent. Again, his approach is problematic because it does not mention or emphasise the interconnectivity of/in black film theory. As a case in point, Wilderson does not link
Tommy Lotts mobilisation of Third Cinema for black film theory to Yearwoods idea of African Diaspora. (64) Additionally, of course, Wilderson seems unaware that Third Cinema itself has been fundamentally

ignoring the African Diaspora is that it exposes


Wildersons corpus of films as unable to carry the weight of the transnational argument he
attempts to advance. Here, beyond the UScentricity or social and political specificity of [his] filmography, (95) I am talking about Wildersons choice of films. For example, Antwone
questioned since Lotts 1990s theory of black film was formulated. Yet another consequence of

Fisher (dir. Denzel Washington, 2002) is attacked unfairly for failing to acknowledge a grid of captivity across spatial dimensions of the Black body, the Black home, and the Black community (111) while films
like Alan and Albert Hughess Menace II Society (1993), overlooked, do acknowledge the same grid and, additionally, problematise Street Terrorism Enforcement and Prevention Act(STEP) policing. The above

Wildersons propensity for


exaggeration and blinkeredness. In chapter nine, Savage Negrophobia, he writes [wilderson quote begins] The philosophical anxiety of Skins is all too aware that through the Middle

examples expose the fact of Wildersons dubious and questionable conclusions on black film. Red, White and Black is particularly undermined by

Passage, African culture became Black style ... Blackness can be placed and displaced with limitless frequency and across untold territories, by whoever so chooses. Most important, there is nothing real Black
people can do to either check or direct this process ... Anyone can say nigger because anyone can be a nigger. (235)7 [wilderson quote ends] Similarly, in chapter ten, A Crisis in the Commons, Wilderson

Black is irredeemable

addresses the issue of Black time.


, he argues, because, at no time in history had it been deemed, or deemed through the right historical moment and place. In
other words, the black moment and place are not right because they are the ship hold of the Middle Passage: the most coherent temporality ever deemed as Black time but also the moment of no time at all

I also wonder what Wilderson makes


of the countless historians and sociologists works on slave ships, shipboard insurrections and/during the
Middle Passage,8 or of groundbreaking jazzstudies books on crosscultural dialogue like The
Other Side of Nowhere (2004). Nowhere has another side, but once Wilderson theorises blacks as socially and ontologically
dead while dismissing jazz as belonging nowhere and to no one, simply there for the taking, (225) there seems to be
on the map of no place at all. (279) Not only does Pinhos more mature analysis expose this point as preposterous (see below),

no way back. It is therefore hardly surprising that Wilderson ducks the need to provide a

solution or alternative to both his sustained bashing of blacks and anti Blackness.9 Last but not least, Red, White and Black ends like a badly plugged
announcement of a bad Hollywood films badly planned sequel: How does one deconstruct life? Who would benefit from such an undertaking? The coffle approaches with its answers in tow. (340)

AT: Libidinal Economy


Their insidious targeting arg relies on the libidinal economy which is wrong
and cannot be the foundation for ethics or the political
Gordon 1
(Paul, psychotherapist living and working in London Psychoanalysis and Racism : The politics
of defeat Race & Class v. 42, n. 4)
postmodernists' problem is that they cannot live with disappointment. All the tragedies of the political
project of emancipation the evils of Stalinism in particular are seen as the inevitable product of men and women
trying to create a better society. But, rather than engage in a critical assessment of how , for
The

radical political movements go wrong, they discard the emancipatory project and impulse itself.
academics have fled into discourse and deconstruction
and representation as though to interpret the world is more important than to change it, as though
instance,

The postmodernists, as Sivanandan puts it, blame modernity for having failed them: `the intellectuals and

changing the interpretation is all we could do in a changing world' .58 To justify their flight
from a politics holding out the prospect of radical change through self-activity, the disappointed intellectuals and abundant intellectual alibis for
themselves in the very work they champion, including, in Cohen's case, psychoanalysis. What Marshall Berman says of Foucault seems true also of
psychoanalysis ; that it offers `a world-historical alibi' for the passivity and helplessness felt by
many in the 1970s,

and that it has nothing but contempt for those naive enough to imagine that it

might be possible for modern human-kind to be free. At every turn for such theorists, as
Berman argues, whether in sexuality, politics, even our imagination, we are nothing but prisoners: there is no freedom in Foucault's world, because his language forms a
seamless web, a cage far more airtight than anything Weber ever dreamed of, into which no life can break . . . There is no point in trying to resist the
oppressions and injustices of modern life, since even our dreams of freedom only add more links to our
chains; however, once we grasp the futility of it all, at least we can relax.59 Cohen's political
defeatism and his conviction in the explanatory power of his new faith of psychoanalysis lead him to be

contemptuous and

dismissive of any attempt at political solidarity or collective action. For him,


`communities' are always `imagined', which, in his view, means based on fantasy, while different forms of working-class organisation, from
the craft fraternity to the revolutionary group, are dismissed as `fantasies of self-sufficient combination'.60 In this scenario, the idea that people might
come together , think together, analyse together and act together as rational beings is impossible . The idea
of a genuine community of equals becomes a pure fantasy , a `symbolic retrieval' of
something that never existed in the first place: `Community is a magical device for conjuring something apparently solidary out of the thin air of
modern times, a mechanism of re-enchantment.' As for history, it is always false, since `We are always dealing with invented traditions.'61 Now, this is not only non- sense,
but dangerous nonsense at that. Is history `always false'? Did the Judeocide happen or did it not? And did not some people even try to resist it? Did slavery exist or did it not, and did
not people resist that too and, ultimately, bring it to an end? And are communities always `imagined'? Or, as Sivanandan states, are they beaten out on the smithy of a people's collective struggle? Furthermore, all
attempts to legislate against ideology are bound to fail because they have to adopt `technologies of surveillance and control identical to those used by the state'. Note here the Foucauldian language to set up the
notion that all `surveillance' is bad. But is it? No society can function without surveillance of some kind. The point, surely, is that there should be a public conversation about such moves and that those responsible
for implementing them be at all times accountable. To equate, as Cohen does, a council poster about `Stamping out racism' with Orwell's horrendous prophecy in 1984 of a boot stamping on a human face is

Cohen reveals himself


to be deeply ambivalent about punitive action against racists, as though punishment or other firrm action against them (or
ludicrous and insulting. (Orwell's image was intensely personal and destructive; the other is about the need to challenge not individuals, but a collective evil.)

anyone else transgressing agreed social or legal norms) precluded `understand- ing' or even help through psychotherapy. It is indeed a strange kind of `anti-racism' that portrays active racists as the `victims',
those who are in need of `help'. But this is where Cohen's argument ends up.

In their move from politics to the academy and the

world of `discourse', the postmodernists may have simply exchanged one grand

narrative , historical materialism, for another, psychoanalysis .62 For psychoanalysis is a grand narrative, par excellence. It is a theory that seeks to
account for the world and which recognises few limits on its explanatory potential . And the claimed radicalism
of psycho- analysis, in the hands of the postmodernists at least, is not a radicalism at all but a prescription for a politics of
quietism, fatalism and defeat. Those wanting to change the world, not just to interpret it,
need to look elsewhere.

Impact Defense

AT: social death


Their politics is the wrong way to deal with social exclusion resurrection
from social death within soviereign spaces gets coopted and fails
Christopher Peterson, 2007, Kindred Specters, Death, Mourning, and American Affinity,
University of Minnesota Press 2007
That Americanist literary criticism on the subject of mortality remains implicated in the larger cultural
disavowal of dying suggests that we ought to reassess our critical energies, particularly as
these powers are enlisted to address how American political ideology produces the
"death" of racial and sexual others. Indeed, I would argue that such criticism remains invested despite all claims to the contrary - in an American exceptionalist project. American
exceptionalism names, in part, a fetishization of novelty and futurity that initially defined America
against an ostensibly decaying and moribund Europe. As David Noble has argued, the doctrine of exceptionalism excluded
America from "the human experience of birth, death, and rebirth" by figuring Europe in terms of time and America in terms of timeless space." If, as George Berkeley put it,

America thus becomes


eternally present while "Europe breeds in her decay." If the "new world" qua new must
deny mortality, then reanimating the excluded from within the terms of a dialectical
reversal renews rather than dismantles the American exceptionalist project. Challenging
the ideology of American exceptionalism is particularly crucial for a post-9/11 politics that aims
to resist the transformation of American exposure to injury and death into a newly
reconsolidated sense of innocence and immortality. As Donald Pease has argued, 9/11 transformed "virgin land" into "ground zero," effecting
an ideological shift from a "secured innocent nation to a wounded, insecure emergency state."16 Drawing from the work of Giorgio Agamben. Pease describes
the emergency state as a nation that - by exempting itself from its own democratic rules of free speech, due process, and above all, the rules
of war - marks a division between those whom the state protects from injury and those whom
the state is free to injure and kill with impunity (13). The reduction of the Arab other to that which cannot be killed because it is
already dead works to cover over the wound that ground zero opens up under the surface of virgin land. The emergency state (or what Agamben calls the
"state of exception") thus also names a nation that attempts to except itself from the universal condition
of mortality. As Bauman notes, "if mortality and transience are the norm among humans,
durability may be attained only as an exception" (67, his emphasis).
America is "time's noblest offspring," history gives birth to its final progeny in order that the latter might escape time altogether.

Their historical reading is wrong


Vincent Brown, Prof. of History and African and African-American Studies @ Harvard Univ.,
December 2009, "Social Death and Political Life in the Study of Slavery," American Historical
Review, p. 1231-1249
THE PREMISE OF ORLANDO PATTERSONS MAJOR WORK, that enslaved Africans were natally alienated and culturally isolated, was challenged even before he published
his influential thesis, primarily by scholars concerned with survivals or retentions of African culture and by historians of slave resistance. In the early to mid-twentieth century,
when Robert Parks view of the Negro predominated among scholars, it was generally assumed that the slave trade and slavery had denuded black people of any ancestral

historians Carter G. Woodson and W. E. B. Du Bois and the anthropologist Melville J. Herskovits argued the opposite. Their research
supported the conclusion that while enslaved Africans could not have brought intact social, political,
and religious institutions with them to the Americas, they did maintain significant
aspects of their cultural backgrounds.32 Herskovits ex- amined Africanismsany practices that seemed to be identifiably Africanas
heritage from Africa. The

useful symbols of cultural survival that would help him to analyze change and continuity in African American culture.33 He engaged in one of his most heated scholarly disputes
with the sociologist E. Franklin Frazier, a student of Parks, who empha- sized the damage wrought by slavery on black families and folkways.34 More recently, a number of

studies have
evolved productively from assertions about general cultural heritage into more precise
demonstrations of the continuity of worldviews, categories of belonging, and social
practices from Africa to America. For these scholars, the preservation of distinctive cultural forms
has served as an index both of a resilient social personhood, or identity, and of
resistance to slavery itself. 35 Scholars of slave resistance have never had much use for the concept of social death. The early efforts of writers such
scholars have built on Herskovitss line of thought, enhancing our understanding of African history during the era of the slave trade. Their

as Herbert Aptheker aimed to derail the popular notion that American slavery had been a civilizing institution threatened by slave crime.36 Soon after, studies of slave revolts
and conspiracies advocated the idea that resistance demonstrated the basic humanity and intractable will of the enslavedindeed, they often equated acts of will with humanity
itself. As these writ- ers turned toward more detailed analyses of the causes, strategies, and tactics of slave revolts in the context of the social relations of slavery, they had
trouble squaring abstract characterizations of the slave with what they were learning about the en- slaved.37 Michael Craton, who authored Testing the Chains: Resistance to
Slavery in the British West Indies, was an early critic of Slavery and Social Death, protesting that what was known about chattel bondage in the Americas did not confirm
Pattersons definition of slavery. If slaves were in fact generally dishonored, Craton asked, how does he explain the degrees of rank found among all groups of slavesthat

How could they have formed the


fragile families documented by social historians if they had been natally alienated by
definition? Finally, and per- haps most tellingly, if slaves had been uniformly subjected to permanent
violent domination, they could not have revolted as often as they did or shown the
varied manifestations of their resistance that so frustrated masters and compromised their power, sometimes fatally.38 The
dynamics of social control and slave resistance falsified Pattersons description of slavery even
as the tenacity of African culture showed that enslaved men, women, and children had
arrived in the Americas bearing much more than their tropical temperament. The cultural continuity and resistance schools of thought
is, the scale of reputation and authority accorded, or at least acknowledged, by slave and master alike?

come together pow- erfully in an important book by Walter C. Rucker, The River Flows On: Black Re- sistance, Culture, and Identity Formation in Early America. In Ruckers
analysis of slave revolts, conspiracies, and daily recalcitrance, African concepts, values, and cul- tural metaphors play the central role. Unlike Smallwood and Hartman, for

the perseverance of African culture even among


second, third, and fourth generation creoles.39 He looks again at some familiar events in North AmericaNew York
Citys 1712 Coromantee revolt and 1741 conspiracy, the 1739 Stono rebellion in South Carolina, as well as the
whom the rupture was the story of slavery, Rucker aims to reveal

plots, schemes, and insurgencies of Gabriel Prosser, Denmark Vesey, and Nat Turnerdeftly teasing out the African origins of many of the attitudes and actions of the black

the transformation of a shared cultural heritage that shaped collective


action against slavery corresponded to the various steps Africans made in the process
of becoming African American in culture, orientation, and identity.40
rebels. Rucker outlines how

Social death is an incorrect homogenization of the past AND reifies colonial


power relations by placing European whiteness at the center of symbolic
examination
Walker 12
(Tracey, Birkbeck University Masters in Psychosocial Studies, The Future of Slavery: From
Cultural Trauma to Ethical Remembrance, Graduate Journal of Social Science, 9.2, July,
JSTOR)
To argue that there is more to the popular conception of slaves as victims who experienced social
death within the abusive regime of transatlantic slavery is not to say that these
subjectivities did not exist. When considering the institution of slavery we can quite confidently rely on the assumption that it did indeed destroy the
self-hood and the lives of millions of Africans. Scholar Vincent Brown (2009) however, has criticised Orlando Pattersons (1982) seminal book
Slavery and Social Death for positioning the slave as a subject without agency and
maintains that those who managed to dislocate from the nightmare of plantation life
were not in fact the living dead, but the mothers of gasping new societies (Brown 2009, 1241).
The Jamaican Maroons were one such disparate group of Africans who managed to band
together and flee the Jamaican plantations in order to create a new mode of living under their own rule.

These runaways were in fact ferocious fighters and master strategists, building towns and military bases which enabled them to fight and successfully win the war against the
British army after 200 years of battle (Gotlieb 2000,16). In addition, the story of the Windward Jamaican Maroons disrupts the phallocentricism inherent within the story of the
slave hero by the very revelation that their leader, Queen Nanny was a woman (Gotlieb 2000). As a leader, she was often ignored by early white historians who dismissed her
as an old hagg or obeah woman (possessor of evil magic powers) (Gotlieb 2000, xvi). Yet, despite these negative descriptors, Nanny presents an interesting image of an
African woman in the time of slavery who cultivated an exceptional army and used psychological as well as military force against the English despite not owning sophisticated
weapons (Gotlieb 2000). As an oral tale, her story speaks to post-slavery generations through its representation of a figure whose gender defying acts challenged the patriarchal
fantasies of the Eurocentric imaginary and as such the study of her experiences might change the lives of people living under paternalistic, racist, classist and gender based

The label of social death is rejected here on the grounds that it is a


narrative which is positioned from the vantage point of a European hegemonic ideology.
Against the social symbolic and its gaze, black slaves were indeed regarded as non-humans since their lives were stunted, diminished and deemed less valuable in
comparison to the Europeans. However, Fanons (1967) assertion that not only must the black man be black; he
must be black in relation to the white man (Fanon 1967, 110) helps us to understand that this
classification can only have meaning relative to the symbolic which represents the
aliveness of whiteness against the backdrop of the dead black slave (Dyer 1997). Butler (2005) makes it clear
that the death one suffers relative to the social symbolic is imbued with the fantasy that
having constructed the Other and interpellated her into life, one now holds the
sovereignty of determining the subjects right to live or die: this death, if it is a death, is only the death of a certain kind
oppression (Gotlieb 2000, 84).

of subject, one that was never possible to begin with, the death of the fantasy of impossible mastery, and so a loss of what one never had, in other words it is a necessary grief

the concept of social death has proved useful for theorists to describe the metaphysical
experience of those who live antagonistically in relation to the social symbolic, it is nevertheless a colonial narrative within which the
slaves are confined to a one dimensional story of terror. In keeping with Gilroys (1993b) argument that the memory of
slavery must be constructed from the slaves point of view, we might instead concentrate, not on the way in which the
slaves are figured within the European social imaginary, but on how they negotiated their
own ideas about self and identity. We might therefore find some value in studying a group
like the Maroons who not only managed to create an autonomous world outside of the
hegemonic discourse which negated them, but also, due to their unique circumstances,
were forced to create new modes of communication which would include a myriad of
African cultures, languages and creeds (Gottlieb 2000). This creative and resistive energy of slave
subjectivity not only disrupts the colonial paradigm of socially dead slaves, but also implies
the ethical tropes of creation, renewal and mutual recognition. In contrast, the passive
slave proved to feature heavily in the 2007 bicentenary commemorations causing journalist Toyin Agbetu to interrupt the official speeches and exclaim
that it had turned into a discourse of freedom engineered mostly by whites with stories of black
agency excluded8. Youngs argument that one of the damaging side effects of the focus on white peoples role in abolition is that Africans are represented as
(Butler 2005, 65). The point to make here is that although

being passive in the face of oppression, appears to echo the behaviour in the UK today given that a recent research poll reveals that the black vote turnout is significantly lower
than for the white majority electorate and that forty percent of second generation immigrants believe that voting doesnt matter.9 Yet, Gilroy (1993a) argues that this political
passivity may not simply be a self fulfilling prophecy, but might allude to the lived contradiction of being black and English which affects ones confidence about whether

Without considering the


slaves capacity for survival and their fundamental role in overthrowing the European
regime of slavery, we limit the usevalue of the memory and risk becoming overly
attached to singular slave subjectivities seeped in death and passivity. The Maroons
story however, enables slave consciousness to rise above the mire of slaverys abject
victims and establishes an ethical relation with our ancestors who lived and survived in
the time of slavery.
opinions will be validated in a society that, at its core, still holds on to the fantasy of European superiority (Gilroy 1993a).

AT: Wilderson Humanism Tuch


Blackness is not the experience of social death---their very articulation of
their critique proves that the concept of humanity is malleable and not
subject to closure
Steven A. Tuch 7, professor of sociology, GWU and * Yoku Shaw-Taylor. The Other African
Americans: Contemporary African and Caribbean Immigrants in the United States
Accompanying the expansion of counmes whose consolidation became known as Europe were emerging differences
in the order of knowledge through which human beings were classified. As scholars such as V. Y.
Mudimbe (1988). Oyrnk Oyewm (1997). and Cor nd West (2002) have shown, drawing upon the resources of archaeo logical and genealogical
poststructuralism, the emergence of systematic forms of inquiry premised upon white supremacy as the basis of human
normality resulted in notions of deviation that structured black people in a derivative
relationship with whites. In other words, a link between the formation of knowledge and processes of inquiry, on the one hand, and
mechanisms of power and the effecting of new and differentiating forms of life/identities, on the other, emerged with their
correlative subjectivities. From aesthetic criteria for human beauty to measurements of intelligence, blacks became the comparison model of deviation instead of ever serving as

blacks
are, in other words, as Franiz Fanon (1967) declared, white constructions. The account suffers, however,
in that it simply declares the conditions for the emergence of certain forms of
the standard. Even worse, the very production of their classificationas blacks at the least. niggers at worst was a function of a logic that was not their own:

phenomena: it does not articulate the lived reality of such phenomena, especially in
terms of the consciousness that experiences them . For even such a consciousness
would be subjected to discursive determinations of its emergence: it would, in other words, be
accepted if, and only if, theorized from its outside, that is, in terms of what constitutes it
as such. So meaning, form, definition, and determination cannot make sense inside of
things. Given the conceptual work brought to bear in making things meaningful, there is
always an outside, public, hence social, dimension to the constitution of meaning. We
should thus study not the essence of the thing but how such an essence is formed . A
problem is raised, however. by a thing that is capable of raising such questions, a thing that
can raise the question of its own meaning and subjectivity. In effect, it subjects itself, or is
able to raise the problem of its existence to itself. This means that it faces a paradox: in
the face of the rejection of an inner logic of the self, it poses the question of itself as an
external matter from inside of itself This rather complex development amounts to saying
that the thing in question is not fully a thing. It is a human being. The human being poses
a challenge to mechanisms of discursive closure . This means that there is always a world of relations
of subjectiv ities and intersubjectivities. of a shared experience of meanings that are
both given and being made. This dialectical relationship of subjection and
intersubjectivity is a manifestation of what is often called agency. That human beings are aware of. or have to interpret and even
appre hend. their situations means that the archaeological and genealogical accounts of the constitution of racialized forms of life tell only a part of the story. Missing. as
well, are the varieties of ways subjects of racism live in their historical and everyday social
situations. That they live a constant struggle for the assertion of their humanity means
that social lines are constantly challenged. expanded. and retracted, and in their course, a more dialectical story unfolds. The
DuBoisian concept of double consciousness is a case in point. In one instance. It is simply an effect of the system blacks seeing them selves as seen by the dominating
perspectives and resources of mean ing in an antiblack racist society. On the other hand. as Paget Henry (2005) points out, there is palentiated double consciousness, where
there is realization of the false, universalistic claims of the system of oppression, where blacks realize that the reality of injustice posed as justice in a racist system hides the
racism as racism by (as DuBois [1898. 19031 also observed) making black people into problems instead of engaging black people as people facing problems. The latter expec
tation requires an admission of a social relationship between blacks and nonbiacks. in short, the reality of intersubjective or shared participation in the constitution of meaning in

That black people have posed much difficulty for the modern world is a sign of a
healthy consciousness. It means a refusal to submit to at tempts of human erasure . It is
not that all black individuals subscribe to such resistance. It is simply sufficient that
enough resistance has existed from the start of racialized slavery in the sixteenth century to make
the world.

the

anthropological

question of what it is to be a human being a constantly unfolding discourse

and material praxis of the modern age . For black people, the concrete formulation is the
reduction of blacks to forms of inert labor, as labor without a point of view, as property. Even for many freed blacks, the institutional imposition of
labor with black ness mearn a constant struggle for the assertion of claimed freedom in a world that had no room for blacks to have leisure time: to be black and not laboring
amounted to an illicit laziness. But even more, the plethora of lines drawn against human assertion meant a constant struggle against illegitimate being. Any category of social
life becomes stained with indiscretion in black form; how does one live when one lacks a right to exist?

AT: Social Death/Contemporary Slavery Impact


Their ontological account of social death is wrongthe focus on gratuitous
violence is ahistorical and equates all conditions
Nadine Ehlers, Professor, School of Social Sciences, Media, and Communication Faculty of
Law, Humanities, and Arts University of Wollongong, 12 [Racial Imperatives: Discipline,
Performativity, and Struggles against Subjection, p. 9-12, footnote from p. 145]
the study pivots on the desire to make dear tbe false
homogeneity of subjects that are denoted by these terms and the arbitrariness of race per se. In the same moment that I
employ these terms as critical tools of analysis, then, I hope to expose the mechanisms of their production and
mark possibilities for their rearticulation. The final portion of this study is concerned with
examining what forms of agency and resistance are possible within the context of this binary
construction of black and white identities. Guiding this analysis is the question of how individuals struggle against subjection and how
While I deploy these terms for analytic convenience,

racial norms might be recited in new directions, given that the coercive demands of discipline and performative constraints make it seem like race is an insurmountable limit or

That race operates as a limit appears particularly so for black subjects. For despite
the fact that all subjects are produced and positioned within and by the discursive
formations of race, the impact of that positioning and what it means for experience is
markedly different. Black subjects are situated within an antiblack context where the black body/self
continues to be torn asunder within the relations of civil society. This means that, as Yancy (2008, 134 n. n) insists, " the capacity
to imagine otherwise is seriously truncated by ideological and material forces that are systematically linked to the history of
white racism!' A number of scholars have examined these realities and advanced critical accounts of what they identify as the resulting condition of black existence.
David Marriot, for instance, argues that "the occult presence of racial slavery" continues to haunt
our political and social imagination: "nowhere, but nevertheless everywhere, a dead time which never
closed system.

arrives and does not stop arriving" (2007, xxi). Saidiya Hartman, in her provocative Lose Your Mother: A journey Along the Atlantic Slave Route (2007) refers to this haunting as
slavery's afterlife. She insists that we do not live with the residue or legacy of slavery but, rather, that slavery lives on. It 'survives' (Sexton 2010, 15), through what Loic
Wacquant (2002, 41) has identified as slavery's fu nctional surrogates: Jim Crow, the ghetto, and the prison. For Hartman, as echoed by other scholars, slavery has yet to be
undone: Black lives are still imperiled and devalued by a racial calculus and a political arithmetic that were entrenched centuries ago. This is the afterlife of slavery- skewed life

Wilderson
III, in his Red, White, and Black: Cinema and the Structures of U.S. Antagonisms (2009), powerfully frames slavery's afterlife as
resulting in a form of social death for black subjects and, more than this, he argues that black
subjectivity is constituted as ontological death. For Wilderson, " the Black [is) a subject who is always already positioned as
Slave" (2009, 7) in the United States, while everyone else exists as "Masters" (2009, 10 ).8 Studies of slavery's afterlife and the
concept of social death have inarguably made essential contributions to understandings of race.9 The strengths of
chances, limited access to health and education, premature death, incarceration, and impoverishment. I, too, am the afterlife of slavery. (2007, 6) Frank B.

such analyses lie in the salient ways they have theorized broad social systems of racism and how they have demanded the foregrounding of suffering, pain, violence, and death.
Much of this scholarship can be put or is productively in conversation with Foucault's account ofbiopolitics that, as I noted earlier, regulates at the level of the population. Where
sovereignty 'took life and let live,' in the contemporary sphere biopolitics works to 'make live.' However, certain bodies are not in the zone of protected life, are indeed
expendable and subjected to strategic deployments of sovereign power that 'make die.' It is here that Foucault positions the function of racism. It is, he argues, "primarily a way
of introducing a break into the domain of life that is under power's control: the break between what must live and what must die" (2003b, 254). Thus, certain bodies/subjects are
killed - or subjected to sovereign power and social death- so that others might prosper. 10 In Scenes of Subjection: Terror, Slavery, and Self-Making in Nineteenth-Century

Hartman examines the 'must die' imperative of social death understood broadly
as a lack of social being-but she also illuminates how, within such a context, slave
"performance and other modes of practice . .. exploit[ed), and exceed[ed] the constraints
of domination" (1997, 54, my emphasis). Hartman analyzes quotidian enactments of slave agency to
highlight practices of "(counter)investment" (1997, 73) that produced "a reconstructed self
that negates the dominant terms of identity and existence" (1997, 72). 11 She thus argues that a form of
agency is possible and that, while "the conditions of domination and subjugation determine what
kinds of actions are possible or effective" (1997, 54), agency is not reducible to these
America (1997),

conditions (1997, 55).'2 The questions that I ask in this analysis travel in this direction, and aim to build on this aspect of Hartman's work. In doing so I make two key
claims: first, that despite undeniable historical continuities and structural d)'namics, race is also marked by
discontinuity; and second, race is constantly reworked and transformed within relations of power by
subjects . 13 For Vincent Brown , a historian of slavery, ''violence, dislocation, and death
actually generate politics, and consequential action by the enslaved" (2009, 1239) . He warns that
focusing on an overarching condition or state potentially obscures seeing these politics . More
than this, however, it risks positioning relations of power as totalizing and transhistorical , and
essentializing experience or the lived realities of individuals. 14 I scale down to the level of the subject to
analyze both (a) how subjects are formed, and (b) how subjects black and white alike have struggled against
conditions in ways that refuse totalizing, immutable understandings of race. This book does
not seek to mark a condition or situa tion then, but instead takes up Brown's challenge (made within the context of studies of slavery) to pay
attention to efforts to remake condition. Looking to those efforts to remake condition and identity grapples with the microphysics of power
it risks

and the practices of daily life, enacted by individuals and i11 collective politics, to consider what people do with situations: those dynamic, innovative contestations of (a never

my work focuses then on "examining ... social and


political lives rather than assuming . . . lack of social being " in order to think about how
subjects can and have "made a social world out of death itself" (Brown 2009, 1233) or how, more generally,
race can be reconfigured within the broader workings of what I am calling racial
discipline and performative imperatives. But in addressing the quotidian and those efforts to remake condition and identity, this
study insists on a shift in perspective in terms of how power is thought about . As I have remarked, I
totalizing) power. Echoing the call raised by Brown (2009, 1239),

am not focused on biopolitics or what can be seen as solely sovereign forms of power that are deployed to condition who will live and who will die. Instead, I am concerned with
disciplinary power, which is articulated simultaneously but at a different level to biopolitics (and despi te the exercise of sovereign forms of power} (Foucault 2003a, 250). For

this form of power is not absolute , nor does it exist in opposition to resistance. Rather,
power is seen as always fragmentary and incoherent, and power and resistance are seen
as mutually constitutive. Disciplinary power is productive, in that it generates particular capacities and forms of subjectivity (and, necessarily, agency).
And finally, though subjects are formed in power, they are not reducible to it, not determined
by power. [BEGIN ENDNOTE] 14. Historian Vincent Brown, in his "Social Death and Political Life in the Study of Slavery" (2009), has examined a
number of scholars who seemingly take up such a viewpoint, in that they broadly position blackness as a totalizing
state that, historically and in the present, renders slavery synonymous with social death
and blackness/// as always already synonymous with slavery. Brown focuses specifically
on the academic uptake and what he sees as the problematic distillation and extension of
Orlando Patterson's (1981) concept of"slavery as social death;' where social death indicates a lack of social being. As a scholar
of slavery, Brown is most concerned with examining the limitations of this idea in relation to the enslaved, but he is also interested in how the
idea is used in relation to the present. For Brown, Patterson's "slavery as social death," and
contemporary usages of this concept to account for the present, advance a troubling
transhistorical characterization of slavery He argues in line with I-Ierman Bennett (quoted in Brown 1009, 1133), who has observed: As
Foucault,

the narrative of the slave experience, soclardeath assumes a uniform African, slave, and ultimately black subject rooted in a static New World history whose logic originated in
being property and remains confined to slavery. It absorbs and renders exceptional evidence that underscores the contingent nature of experience and consciousness. Thus,

normative assumptions about the experiences of peoples of African descent assert a


timeless, ahistorical, epiphenomenal "black" cultural experience.
[END ENDNOTE]

Sexual Power/ID = Heteronorm


Ontologizing sexual power and identity as a lens for worldly phenomena
reproduces binary discourses end elides difference, reproducing
heteronormativity.
Prasad 12
(Ajnesh, Australian School of Business, Beyond analytical dichotomies, Human Relations May
2012 vol. 65 no. 5 567-595)
poststructuralism may be typified by what Lyotard (1984)
famously expressed as its incredulity towards metanarratives (also see Alvesson and Deetz, 1999; Kilduff
A poststructuralist critique At the most rudimentary level,

and Mehra, 1997), or what Fraser and Nicholson (1997) describe as grand theorizing of social macrostructures. Akin to other critical traditions, it is
explicitly historical, attuned to the cultural specificity of different societies and periods and . . . inflected by temporality, with historically specific
institutional categories (Fraser and Nicholson, 1997: 1434). The aim of the poststructuralist mandate is to critique metanarratives and, from there, to
define human consciousness and social existence through engagements with contextualized subjectivity (Agger, 1991).11 To appreciate this idea, it is
important to understand how metanarratives are problematically situated within, and are informed by, socially constructed identity binaries, be they
along the fault lines of gender (e.g. Butler, 1990; Hird, 2000), race (e.g. Gilroy, 2000; Miles and Torres, 2007), or sexuality (e.g. Zita, 1994). Indeed,

poststructuralist critique, such as Derridas conjecture of difference (Mumby and Putnam, 1992), illustrates how the
preservation of the privileged identity (white, man, heterosexual) is existentially
dependent upon a corresponding relegated identity (black, woman, homosexual) (see
Tyler and Cohen, 2008); or, to posit it in Butlerian (1991) phrasing, heterosexuality presumes the being of homosexuality. Working from the
same current, Harding (2003) extends this idea through consideration of the central
dyadic relationship in management; she notes that every individual in the western workforce is
identified as a worker or as a manager and that the identity of the latter is wholly
contingent upon the binary existence of the former . Elsewhere, again assuming a poststructuralist position,
Harding (2008: 44; emphasis in original) explains that identities cannot be resolved into an essence or into a
coherent whole; rather, they are post hoc impositions of a seemingly unified [label]
upon a disparate and disconnected population. As such, the central aim of the poststructuralist is to repudiate
deterministic and binary logic by drawing attention to the discursive processes that culturally (re)produce social realities and the dichotomous modes of
thinking embedded within them (Butler, 1990; Calas, 1993). [T]he ways in which sex was put into Western discourse from the end of the 16th century
onwards, the proliferation of sex during the 18th century and the modern incitement to discuss sex in endless detail have simultaneously established

Tangentially related to the


Foucauldian reading of sexuality, Katz (2004) offers a genealogical investigation into how
heterosexuality was invented and came to be ideologically defined from the late 19th-century onwards. Katz
writes that because the concept of heterosexuality is only one particular historical way of
perceiving, categorizing, and imagining the social relations of the sexes, it ought to be
studied with the purposeful aim to dislocate its socio-cultural privilege as being the normal
and the natural form of sexual expression (p. 69). Given its socially manifest nature, Butler (1991, 1993) contends that
heterosexuality as the unassailable norm and constituted other sexualities as abnormal.

heterosexuality is perpetually at risk and must continually engage in a set of repetitive, or what she calls parodic, practices such as, heterosexual sex
which functions to stringently affirm the hegemonic ideals of femininity (passive) and masculinity (active). Incidentally, the very redundancy of these
parodic practices function to consolidate the discursive authority and cultural stability of heterosexuality (Butler, 1991; see also Butlers [1993] writing
on performativity). A related stream of poststructuralist-inflected scholarship reveals how

sexual identities that are predicated

on ontological sexual difference produce heteronormativity, which can be described as


centrality of heterosexual norms in social
relations (Pringle, 2008: S111). While feminists have long critiqued the tacit and the explicit claims of ontological sexual difference,
essentialist definitions of female and male continue to prevail in popular culture and in
certain academic disciplines (Frye, 1996).12 On this point, Hird (2004) adopts a position in feminist science studies to develop a
the the normative idealization of heterosexuality (Hird, 2004: 27) or the

substantive critique into how the ontology of sexual difference is often rendered concrete in research propagated by the natural and particularly, the

The influence of ontological sexual difference within and


outside of academia, lends credence to Broadbridge and Hearns (2008: S39) recent
observation that, [s]ex and sex differences are still often naturalized as fixed, or almost
fixed, in biology. It is equally important to note, here, that the alchemy of ontological sexual difference is
wholly dependent upon the patriarchal conflation of biological sex and cultural gender
(Hird, 2004). As Pringle (2008: S112; also see Borgerson and Rehn, 2004) notes, [g]ender [does] not avoid the
oppositional duality embodied in the concept of sex, but reflect[s] the interdependent
relationship of masculinity and femininity. This reflection pivots on genital determinism, which declares that males
biological sciences (also see Martin, 1991).

naturally embrace masculinity while females naturally embrace femininity (Bornstein, 1994; Hird, 2000). This initial conflation of sex and gender leads
to the conventional model of heterosexuality, which dictates that a man will desire-to-be a male and will desire-for a female, while a woman will

It is precisely these corresponding


relationships whereby the heterosexual matrix is constructed (Butler, 1990). According
to Butler, this matrix serves as the grid of cultural intelligibility through which bodies,
gender, and desires are naturalized (see Ringrose, 2008: 511).13
desire-to-be a female and will desire-for a male (Sinfield, 2002: 126).

Their politics which produces nostalgia for non-engagement casting


movement as stasis makes theorizing for the future impossible
Rosalind C. Morris, Professor of Anthropology @ Columbia, 10
(Can the Subaltern Speak?: Reflections on the History of an Idea, pg. 8)

Subalternity is not that which could, if given a ventriloquist, speak the truth of its oppression or
disclose the plenitude of its being. The hundreds of shelves of well-intentioned books
claiming to speak for or give voice to the subaltern cannot ultimately escape the problem
of translation in ii> full sense. Subalternity is less an identity than what we might call a
predicament , but this is true in very odd sense. For, in Spivak's definition, it is the structured
place from which the capacity to access power is radically obstructed. To the extent that
anyone escapes the muting of subalternity, she ceases being a subaltern. Spivak says this is to
be desired. And who could disagree? There is neither authenticity nor virtue in the position
of the oppressed. There is simply (or not so simply) oppression. Even so, we are moved to
wonder, in this context, what burden this places on memory work in the aftermath of education.
What kind of representation becomes available to the one who, having partially escaped the
silence of subalternity, is nonetheless possessed by the consciousness of having been
obstructed, contained, or simply misread for so much of her life? Is there any alternative to
either the positivist euphoria that would claim to have recovered the truth of her past or
the conflation of historiography with therapeutic adaptation by which ideology finally makes the
silence of subalternity seem normal?
Today in the halls of the academy it is possible to discern a certain displacement of the
critique of power and class, and hence of history, by the cultural analysis of memory. If the
latter offers itself as an alternative to the positivism of empiricist historiography, and as a
critique of the teleologies implicit in so much Marxist theory, it nonetheless tends to
surrender uto-pianism only to embrace nostalgia . Nostalgia, in this sense, is but the
inverse of utopianism, a utopianism without futurity. Ironically, this nostalgia often bears a

secret valorization and hypostatization of subalternity as an identityto be recalled,


renarrated, reclaimed, and revalidated. We need to resist the narcissism implicit in this
gesturewhich ultimately demands a whole image as the mirror of ourselves, not merely
as the basis for misrecognition (and hence our own subject formation) but also as the alibi for a
politics that imagines the project of emancipation to be over. A quick survey of the
contemporary social landscape demands the recognition that it is not.

Affirming difference as the starting point for revolution is a failed strategy that
ends in ambivalence maroonage collapses into aescthetic boundary drawing
makes the skill building necessary for radical politics inevitable
Thomas Nail, Post-doctoral Lecturer in European Philosophy at the University of Denver,
2013, Deleuze, Occupy, and the Actuality of Revolution, Theory & Event Volume 16, Issue 1

(1) Political Ambivalence


Affirming Difference in the state of permanent revolution [affirmer la Diffrence dans ltat
de rvolution permanente], as Deleuze says in Difference and Repetition (75/53),8 may escape
the previous problems of vanguardism and the party-state, but it also poses a new danger:
that the pure affirmation of Difference will be ultimately ambivalent. Revolution may
provide a new non-representational space of liberty, or it may provide a ruptured open
domain for a new discourse of rights and military occupation by the state, or it may merely
reproduce a complicity with the processes of capitalist deterritorialization necessary for
new capitalist reterritorializations. Slavoj iek, in particular, frequently attributes this
capitalist ambivalence to Deleuze and Guattaris politics (2004, 184). But to say, with Alain
Badiou, that affirming the potentiality for transformation as such is to affirm a purely
ideological radicality that inevitably changes over into its opposite: once the mass
festivals of democracy and discourse are over, things make place for the modernist
restoration of order among workers and bosses, would be to overstate the problem (Badiou
and Balms 1976, 83).
Rather, it would be much more appropriate to say, with Paolo Virno, that [t]he multitude is
a form of being that can give birth to one thing but also to the other: ambivalence (Virno
2003, 131). Accordingly, the affirmation of this ambivalence as a political commitment, and
the politico-ontological optimism and unapologetic vitalism it assumes in Hardt, Negri,
and Deleuzes work, according to Bruno Bosteels, remains radically insufficient (2004, 95).
While the purely creative power of the multitude may be the condition for global liberation
from Empire, it is also the productive condition for Empire as well. With no clear political
consistency to organize or motivate any particular political transformation vitalist
optimism is politically ambivalent, speculative, and spontaneous. Showing the nonfoundational or ungrounded nature of politics provides no more of a contribution for
organized politics than does the creative potentiality of desire. A subjects intervention,
Bosteels suggests, cannot consist merely in showing or recognizing the traumatic
impossibility, void, or antagonism around which the situation as a whole is structured (2004,
104). Rather, following Badiou, a political organization is necessary in order for the

intervention, as wager, to make a process out of the trajectory that goes from an
interruption to a fidelity. In this sense, organization is nothing but the consistency of
politics (Badiou 1985, 12). And in so far as Deleuze and Guattari, and those inspired by
their work, do not offer developed concepts of political consistency and organization that
would bring differential multiplicities into specific political interventions and
distributions, they remain, at most, ambivalent toward revolutionary politics.
(2) Virtual Hierarchy
In addition to this first danger of revolutionary ambivalence, Deleuzes concept of revolution,
according to Badiou and Hallward, risks a second danger; namely, that of creating a political
hierarchy of virtual potential. Badiou argues at length in The Clamor of Being that,
contrary to all egalitarian or communitarian norms, Deleuzes conception of thought is
profoundly aristocratic. Thought only exists in a hierarchized space. This is because, for
individuals to attain the point where they are seized by their preindividual determination
and, thus, by the power of the One-Allof which they are, at the start, only meager local
configurationsthey have to go beyond their limits and endure the transfixion and
disintegration of their actuality by infinite virtuality, which is actualitys veritable being.
And individuals are not equally capable of this. Admittedly, Being is itself neutral, equal,
outside all evaluation But things reside unequally in this equal being (Deleuze 1994,
60/37). And, as a result, it is essential to think according to a hierarchy which considers things
and beings from the point of view of power
(Deleuze 1994, 60/37; Badiou 1999, 1213).
The political thrust of this argument is that, if we understand revolutionary change as the
pure potential for change as such, and not actual change for or against certain forms, then,
contrary to any kind of egalitarianism, there will instead be a hierarchy of actual political
beings that more or less participate in this degree of pure potential transformation. The more
actual political beings renounce their specific and local determinations and affirm their
participation in the larger processes of difference-in-itself, the more powerful they
become. Thus, if the point of examining any local political intervention is in every case to
show to what degree it renounces its concrete determinations and might become other
than it is (as a virtuality or potentiality), there is, according to Badiou, a risk of asceticism
and hierarchy in such a relationship of potential (Badiou 1999, 13).
Similarly, Peter Hallward has argued that Deleuzes political philosophy is indifferent to the
politics of this world (2006, 162). Hallward claims that once a social field is defined less by
its conflicts and contradictions than by the lines of flight running through it any distinctive space
for political action can only be subsumed within the more general dynamics of creation, life, and
potential transformation (2006, 62n16). And since these dynamics are themselves antidialectical if not anti-relational, there can be little room in Deleuzes philosophy for relations of
conflict and solidarity (2006, 162). If each concrete, localized, actual political being is
important only in so far as it realizes a degree of pure potentiality of a virtual event, and every
mortal event in a single Event (Deleuze 1990, 178/152), then the processional telos of
absolute political deterritorialization is completely indifferent to the actual politics of this world
(2006, 97). By valorizing this pure potentiality for transformation as such against all
actual political determinations, Hallward argues, Deleuze is guilty of affirming an

impossible utopianism . By posing the question of politics in the starkly dualistic terms of war
machine or state, Hallward argues, by posing it, in the end, in the apocalyptic terms of a
new people and a new earth or else no people and no earththe political aspect of
Deleuzes philosophy amounts to little more than utopian distraction (2006, 162).
(3) Subjective Paralysis
The differential reading of Deleuze and Guattaris concept of revolution may be able to avoid the
problem of representational subjectivitythat it can reject or affirm particular desires but never
change the nature of the self that desiresbut it does so only at the risk of diffusing the self
into an endless multiplicity of impersonal drives: a self in perpetual transformation. This leads to
the third danger, that of subjective paralysis. Firstly, to read Deleuze and Guattaris theory of
revolutionary subjectivity as the simple fact of one's own existence as possibility or potentiality
(Agamben 1993, 43), or as Paul Patton calls it, ones critical freedom (the freedom to
transgress the limits of what one is presently capable of being or doing, rather than just the
freedom to be or do those things (2000, 85) suggests an ambivalence of action. What are the
conditions and factors by which one might decide to take an action or not? Emancipation and
enslavement in this sense are merely just different things to be done.
Secondly, without a pre-given unity of subjectivity, how do agents qua multiplicities deliberate
between and distinguish (in themselves) different political decisions? Without the
representational screen of reason, or the state-guaranteed grounds of political discourse, what
might something like a dispute or agreement look like? If becoming other is not a capacity
liberated individuals possess to constitute themselves as autonomous singularities, but what
defines autonomy itself (2006, 146), as Simon Tormey argues, then the political danger,
according to Hallward, is that the subject is simply replaced by the larger impersonal process of
transformation as such: pure autonomy. The radical affirmation of the ambivalent and
unlocalizable processes of subjective potentiality (qua pure multiplicities) seems then to have
nothing to contribute to an analysis of the basic function of participatory democracy and
collective decision-making, which remains at the core of many of todays radical political
struggles (See Starr, Martinez-Torres, and Rosset 2011). Insofar as a theory of subjectivity
is defined only by its potential for transformation, it is stuck in a kind of paralysis of
endless potential change no less disempowering than subjective stasis. Or, as Hallward
frames this criticism, Deleuze abandons the decisive subject in favor of our more immediate
subjection to the imperative of creative life or thought (2006, 163).

AT: Ballot/Strategy

AT: white suicide

Instead of focusing politics on the personal, we should take collectivize


action against social illsa personality litmus test dooms political projects
Rob the Idealist, Carleton College, 10/1/13, Tim Wise & The Failure of Privilege Discourse,
www.orchestratedpulse.com/2013/10/tim-wise-failure-privilege-discourse/
I dont find it meaningful to criticize Tim Wise the person and judge whether hes living up to some anti-racist bona fides. Instead, I choose to focus on the paradigm of White

Although the personal is political, not all


politics is personal; we have to attack systems. To paraphrase the urban poet and philosopher Meek Mill: there are
levels to this shit. How I Define Privilege There are power structures that shape individuals lived
experiences. Those structures provide and withhold resources to people based on factors like class, disability status,
privilege upon which his work is based, and its conceptual and practical limitations.

gender, and race. Its not a benefit to receive resources from an unjust order because ultimately, injustice is cannibalistic. Slavery binds the slave, but destroys the master. So,

the point then becomes not to assimilate the underprivileged, but to instead eradicate
the power structures that create the privileges in the first place. The conventional wisdom on privilege often says
that its benefits are unearned. However, this belief ignores the reality and history that privilege is earned and maintained through violence. Systemic advantages are

The
history and modern reality of violence is why Tim Wise comparison between whiteness and
tallness fails. White supremacy is not some natural evolution, nor did it occur by
happenstance. White folks *murdered* people for this thing that we often call White privilege; it was bought and paid for by blood and terror. White
supremacy is not some benign invisible knapsack. The same interplay between violence and advantage is true of any systemic
hierarchy (class, gender, disability, etc). Being tall, irrespective of its advantages, does not follow that pattern of violence. Privilege is Failing Us Unfortunately, I
think our use of the term privilege is no longer a productive way for us to gain a
thorough understanding of systemic injustice, nor is it helping us to develop collective
strategies to dismantle those systems. Basically , I never want to hear the word privilege
allocated and secured as a class, and simply because an individual hasnt personally committed the acts, it does not render their class dominance unearned.

again because the term is so thoroughly misused at this point that it does more harm than
goo d. Andrea Smith, in the essay The Problem with Privilege, outlines the pitfalls of misapplied privilege theory. Those who had little privilege
did not have to confess and were in the position to be the judge of those who did have privilege. Consequently, people
aspired to be oppressed. Inevitably, those with more privilege would develop new heretofore
unknown forms of oppression from which they suffered Consequently, the goal became not to
actually end oppression but to be as oppressed as possible. These rituals often substituted
confession for political movement-building. Andrea Smith, The Problem with Privilege Dr. Tommy Curry says it more bluntly, Its
not genius to say that in an oppressive society there are benefits to being in the superior
class instead of the inferior one. Thats true in any hierarchy, thats not an aha moment. Conceptually, privilege is best used when narrowly
focused on explaining how structures generally shape experiences. However, when we overly personalize the problem, then
privilege becomes a tit-for-tat exercise in blame, shame, and guilt. In its worst manifestations, this
dynamic becomes oppression Olympics and people tally perceived life advantages and
identities in order to invalidate one another. At best, we treat structural injustice as a personal
problem, and moralizing exercises like privilege confessions inadequately address the
nexus between systemic power and individual behavior. The undoing of privilege occurs
not by individuals confessing their privileges or trying to think themselves into a new subject position, but
through the creation of collective structures that dismantle the systems that enable these privileges. The
activist genealogies that produced this response to racism and settler colonialism were not initially focused on racism as a problem of individual prejudice. Rather, the purpose
was for individuals to recognize how they were shaped by structural forms of oppression. Andrea Smith, The Problem with Privilege Bigger than Tim Wise However, the problem
with White privilege isnt simply that Tim Wise, a white man, can build a career off of Black struggles. As Ive already said, White people need to talk to White people about the

historical and social construction of their racial identities and power, and the foundation for that conversation often comes from past Black theory and political projects. The

privilege work has become a cottage industry of self-help moralizing that in


no way attacks the systemic ills that create the personal injustices in the first place. A
substantive critique of privilege requires us to get beyond identity politics . Its not about
good people and bad people; its a bad system. Its not just White people that participate in the White privilege industry, although
not everyone equally benefits/profits (see: Tim Wise). Dr. Tommy Curry takes elite Black academics to task for their role in
profiting from the White privilege industry while offering no challenge to White
supremacy. These conversations about White privilege are not conversations about race, and certainly not about racism; its a business where Blacks market
themselves as racial therapists for White people The White privilege discourse became a bourgeois distraction. Its a
tool that we use to morally condemn whites for not supporting the political goals of elite
black academics that take the vantages of white notions of virtue and reformism and
persuade departments, journals, and presses into making concessions for the benefit of a select
species of Black intellectuals in the Ivory Tower, without seeing that the white racial
vantages that these Black intellectuals claim theyre really interested in need to be
dissolved, need to be attacked all the way to the very bottom of American society. Dr. Tommy
Curry, Radio Interview The truth is that a lot of people, marginalized groups included, simply want more
access to existing systems of power. They dont want to challenge and push beyond these systems; they just want to
participate. So if we continue to play identity politics and persist with a personal privilege
view of power, then we will lose the struggle. Barack Obama is president, yet White
supremacy marches on, and often with his help (record deportations, expanded a drone war based on profiling, fought on behalf of US corporations to
problem for me is that

repeal a Haitian law that raised the minimum wage). Adolph Reed, writing in 1996, predicted the quagmire of identity politics in the Age of Obama. In Chicago, for instance,
weve gotten a foretaste of the new breed of foundation-hatched black communitarian voices; one of them, a smooth Harvard lawyer with impeccable do-good credentials and
vacuous-to-repressive neoliberal politics, has won a state senate seat on a base mainly in the liberal foundation and development worlds. His fundamentally bootstrap line was
softened by a patina of the rhetoric of authentic community, talk about meeting in kitchens, small-scale solutions to social problems, and the predictable elevation of process
over program the point where identity politics converges with old-fashioned middle-class reform in favoring form over substance. I suspect that his ilk is the wave of the future

Obamas
election and subsequent presidency has made it starkly clear that its not just White
people that can perpetuate White supremacy. Systems of oppression condition all members of society to accept systemic injustice,
and there are (unequal) incentives for both marginalized and dominant groups to perpetuate these structures. Our approaches to injustice must
reflect this reality. This isnt a nave plea for unity, nor am I saying that talking about
identities/experiences is inherently divisive. Many of these privilege discussions use empathy to build personal and collective character, and
there certainly should be space for us to work together to improve/heal ourselves and one another. People will always make mistakes and
our spaces have to be flexible enough to allow for reconciliation. Though we dont have
to work with persistently abusive people who refuse to redirect their behavior, theres a
difference between establishing boundaries and puritanism. Fighting systemic
marginalization and exploitation requires more than good character, and we cannot fetishize personal
in U.S. black politics. Adolph Reed Jr., Class Notes: Posing As Politics and Other Thoughts on the American Scene Although it has always been the case,

morals over collective action .

The alt aestheticizes violence the position of the master is not produced by
violent interactions stubborn endorsement of violence leads to a self-fulfilling
prophecy of opposition rather than focusing on the factors that produce the need
for radical politics
Simon Swift, Ph.d., Senior Lecturer in Critical and Cultural Theory, University of Leeds, 2013
Hannah Arendt, violence and vitality, European Journal of Social Theory 16(3) 357376

A key aspect of Arendts critique of modern political life as it increasingly embodies violence
that has emerged throughout this article has been the ways in which political ideologies and
philosophical ideas fail to keep pace with the flux of events that they try to understand
and control. This insight, and the important role played by violence within it, are taken further in
Arendts account of the global phenomenon of the student protest movement in 1968 in her late
essay On Violence (Arendt, 1972: 117). Arendt claims that the students misrecognize the
sources for their own turn to violence, which she repeatedly associates with the rise of
technology, and which she frames especially in the context of the nuclear standoff or what she
calls the weird suicidal development of modern weapons (Arendt, 1972: 116). The general
context that Arendt defines for the growing commitment to violence on the New Left is
uncertainty about the future. In the context of nuclear armament, the means of violence
overwhelm the ends of politics in a particularly dramatic way, as societys technological
development is continually geared towards preparation for a thermonuclear war that never
happens. The students aesthetic critique of society, which takes the form of a growing
commitment to violence as a furthering and enhancement of the life force, misrecognizes
its continuity with this wider political abandonment of the instrumentality of violence.
Both the Cold War powers and the students, in other words, pass beyond an instrumental model
of violence, to a view of violence as an end in itself. But Arendt also asks why it is that the
students cling with such stubborn tenacity to concepts and doctrines that have not only
been refuted by factual developments but are clearly inconsistent with their own politics
(Arendt, 1972: 124). The disconnect with reality and what she thinks of as the
outmodedness of their ideas are in fact part and parcel of the same phenomenon. The
students cling tenaciously to an ideology of creativity in the face of the fact that an increasingly
consumerist, technology-driven society has done away with any notion of work. Behind their
critique of consumerism, she argues, stands the illusion of Marxs society of free producers, the
liberation of the productive forces of society; a revolution which has in fact been accomplished,
Arendt thinks, by science and technology (Arendt, 1972: 117). Like the life philosophy that, in
Arendts account, inspires it, the student protest movements aestheticization of violence
blocks out the technological context that determines it and that it reproduces in its radical
abandonment of an instrumental view of violence. As Benjamin remarked in his essay On
Some Motifs in Baudelaire, Bergsons philosophy, in his earlier Matter and Memory, manages
above all to stay clear of that experience from which his own philosophy evolved or, rather, in
reaction to which it arose. It was the inhospitable, blinding age of big-scale industrialism. In
shutting out this experience the eye perceives an experience of a complementary nature in the
form of its spontaneous afterimage, as it were. (Benjamin, 1992: 154) But while the students
are guilty of misapprehending, like Marx before them, the real process-character of
modern life and labour because they cling to an outmoded, humanistic idea of man as a
creator-producer, a figure who, in the work of Sartre and Fanon, is seen to produce
himself through violence, their expression of this doctrine of creativity is also, for Arendt,
internally inconsistent . Unlike Marx, who proposes that man reproduces himself through
labour, the students turn to the Nietzschean thesis that the joy of destruction is a
creative joy (Arendt, 1972: 117). They at once inhabit and depart from a Marxist
humanism, and in doing so they make contact, in their seemingly so novel biological
justification of violence with the most pernicious elements in our oldest traditions of

political thought (Arendt, 1972: 171). These elements enable a thinking of power in
biological terms as violence. Specifically, violence is understood as creative in its expressing
of what Bertrand de Jouvenel calls an inner urge to grow (Arendt, 1972: 171). Just as in the
realm of organic life everything either grows or declines or dies, this theory runs as Arendt
rehearses it, so in the realm of human affairs power supposedly can sustain itself only through
expansion; otherwise it shrinks and dies. The endpoint of this biological theory of power is
found in the view that kings are killed not because of their tyranny but because of their
weakness and that the people erect scaffolds as a biological penalty for weakness (p. 171). In
fact, Arendt had herself employed such an organicist theory of power in her effort to understand
the agency of anti-Semitism in The Origins of Totalitarianism, where she had compared the
Jews to aristocrats in pre-Revolutionary France. When the aristocrats lost their privileges,
among others the privilege to exploit and oppress, the people felt them to be parasites, without
any function in the rule of the country. Comparably, [a]ntisemitism reached its climax when
Jews had similarly lost their public functions and their influence, and finally were left with nothing
but their wealth (Arendt, 1968: 4).

The alt amounts to an arbitrary targeting mechanism based on the presumed


knowledge and charictarization of others bodies which is a double turn with their
first strategy, but also creates infinite culpability for action they cant solve until
everyone who is not them is symbolically dead
Butler 2 Paul Butler, Professor of Law, George Washington University BA Yale JD Harvard By
Any Means Necessary: Using Violence and Subversion to Change Unjust Law UCLA Law
Review 50 2002-2003 761-762

Just war doctrine, applied to insurgents, would limit violence more than it would authorize it. Jus
in bello requires that there be no injury to civilians. If the usual definitions of "terrorism" and

"civilians" are invoked, this requirement seems to rule out virtually all terrorism, for terrorism is
commonly thought of as politically motivated violence against civilians. Sometimes terrorists
argue that, in their particular conflict, there are no "noncomba-tants ." An example of this
argument is the claim that even nonmilitary citizens of Israel arc permissible targets for
Palestinian protestors because all Israelis benefit from and help maintain the subordination of
Palestinians.101 The problem with this argument is the same as the problem with most
terrorism: It is indiscriminate. It grants insufficient weight to the value of human life when it does
not acknowledge that there are degrees of culpability. Surely, for example, children are not as
responsible as adults, and surely a poor laborer is not as responsible as a high government
official. On the other hand, it is possible to defend a construct of "combatants," that is,

permissible targets of violence, that includes nonmilitary actors. The objective of just war is to
change the regime, or the way that it operates. To attack the foot soldiers, but to ignore
the authorities responsible for creating and implementing the oppressive policies, seems
inefficient. The suggestion that insurgents should distinguish among civilians and limit their
targets to "well-known officials, notorious collaborators, and bo on," seems reasonable,"" Thus,
just war applied to insurgents would not eliminate the "combatant" restriction but would broaden
the concept, in the manner described by Professor Walzer. "Combatant" could be defined as

any person directly responsible for creating, administering, or defending the human rights
violations, including genocide or race discrimination, that are the subject of the conflict. As
discussed below, in the context of the "war" against race-based capital punishment, combatants
would include those directly responsible for creating and implementing it. These people are
culpable in a way that the ordinary civilian is not. Even then, violence against them is not
necessarily moral: The other conditions of just war must also be satisfied. I want to emphasize

the purpose of applying just war doctrine to insurgents, because I understand that any moral
construct that tolerates political violence by nongovernmental entities is controversial. Just war
doctrine accepts that violencekilling peoplecan be morally justified, if certain conditions are
satisfied. One of the necessary conditions is that the targets must be military. This condition
seems inconsistent with other common constructs of morality, including those found in criminal
and international law, and in popular culture. The heroic status that many now accord those who
led slave revolts is evidence of that view of morality. Yet definitions of combatants proffered by
terrorists are overly broad when they include those who are not directly responsible for the
oppression of others (even if they benefit from that oppression) . This view of combarants
discounts the sanity of human life that must underlie any construct of morality (even if that

construct allows for the taking of life in certain cases). The proposed application of just war
doctrine to "terrorists" assumes that they, like nation-states, are open to persuasion about their
methods. Their embrace of violence does not mean they have abandoned their claim to
morality. If just war doctrine is to remain relevant in the twenty-first century, it should be applied
to every actornot just governmentsthat uses violence to accomplish political objectives.

AT: give us the ballot


Confessing ballot just reinstates domination
Andrea Smith, Ph.D., co-founder of Incite! Women of Color Against Violence, UC Riverside
Associate Professor, 2013, Geographies of Privilege, Unsettling the Privilege of Self-Reflexivity,
Kindle
In my experience working with a multitude of anti-racist organizing projects over the years, I frequently found
myself participating in various workshops in which participants were asked to reflect on their
gender/race/sexuality/class/etc. privilege. These workshops had a bit of a self-help orientation to them : I
am so and so, and I have x privilege. It was never quite clear what the point of these confessions were. It was not
as if other participants did not know the confessor in question had her/his proclaimed privilege. It did not
appear that these individual confessions actually led to any political projects to dismantle the
domination that enabled their privilege. Rather, the confessions became the political project
themselves. The benefits of these confessions seemed to be ephemeral. For the instant the confession took
place, those who do not have that privilege in daily life would have a temporary position of
power as the hearer of the confession who could grant absolution and forgiveness. The sayer of the confession
could then be granted temporary forgiveness for her/his abuses of power and relief from white/male/heterosexual/etc guilt. Because
of the perceived benefits of this ritual, there was generally little critique of the fact that in the end, it primarily served to reinstantiate the
structures of

structures of domination it was supposed to resist. One of the reasons there was little critique of this practice is that it
Those who had little privilege did not have to confess and were
in the position to be the judge of those who did have privilege. Consequently, people aspired to be oppressed.
Inevitably, those with more privilege would develop new heretofore unknown forms of oppression
from which they suffered. I may be white, but my best friend was a person of color, which
caused me to be oppressed when we played together. Consequently, the goal became not to actually end oppression but to be
bestowed cultural capital to those who seemed to be the most oppressed.

oppressed as possible. These rituals often substituted confession for political movementbuilding. And despite the cultural capital that was, at least temporarily, bestowed to those who seemed to be the most oppressed, these rituals
ultimately reinstantiated the white majority subject as the subject capable of selfreflexivity and the colonREized/racialized subject as the occasion for self-reflexivity. These
as

rituals around self-reflexivity in the academy and in activist circles are not without merit. They are informed by key insights into how the logics of domination that structure the

for this
process to work, individual transformation must occur concurrently with social and political
world also constitute who we are as subjects. Political projects of transformation necessarily involve a fundamental reconstitution of ourselves as well. However,

transformation. That is, the undoing of privilege occurs not by individuals confessing their privileges or trying to
but through the creation of collective structures that dismantle the
systems that enable these privileges. The activist genealogies that produced this response to racism and settler colonialism were not initially focused on racism as a
problem of individual prejudice. Rather, the purpose was for individuals to recognize how they were shaped by structural forms of oppression. However, the
response to structural racism became an individual one individual confession at the
expense of collective action. Thus the question becomes, how would one collectivize
individual transformation? Many organizing projects attempt and have attempted to do precisely this, such Sisters in Action for Power, Sista II Sista,
think themselves into a new subject position,

Incite! Women of Color Against Violence, and Communities Against Rape and Abuse, among many others. Rather than focus simply on ones individual privilege, they address
privilege on an organizational level. For instance, they might assess is everyone who is invited to speak a college graduate? Are certain peoples always in the limelight?
Based on this assessment, they develop structures to address how privilege is exercised collectively. For instance, anytime a person with a college degree is invited to speak,
they bring with them a co-speaker who does not have that education level. They might develop mentoring and skills-sharing programs within the group. To quote one of my

the current
social structure conditions us to exercise what privileges we may have. If we want to
activist mentors, Judy Vaughn, You dont think your way into a different way of acting; you act your way into a different way of thinking. Essentially,

undermine those privileges, we must change the structures within which we live

so that we become

different peoples in the process.

Dont take self-understanding as an incontrovertible truth we dont perfectly


know ourselves and have a lot to gain by making our beliefs subject to external
criticism

McBride 3 Professor of Government @ London School of Economics


(Cillian, Self-transparency and the possibility of deliberative politics, Journal of Political
Ideologies, 8.3)
self-transparency which underwrites the politics of presence. This connects
situation, identity, and perspective in such a way as to be incompatible with deliberative
ABSTRACT I argue against the notion of

politics and treats self-understand- ing as authoritative, rendering it insensitive to the


possibility that our self-under- standings may be distorted . I propose a hermeneutic, narrative,
conception of selfhood on which

we relate

to our lives as authors, constructing our identities by employing the linguistic and narrative resources

which our respective situations make available to us.

This admits the possibility that others may provide us

with superior interpretations of our lives , which is a precondition of deliberative politics.


Given the possibility that our self-understandings may be distorted, deliberative citizens
have a duty to challenge problematic self-under- standings . Anchoring criticism to
public deliberation, together with the her- meneutic premise that a measure of self-opacity is
universal, secures such challenges against the charge of authoritarianism levelled at traditional
ideol- ogy-critique. I want to focus here on the challenge posed to dialogue-centred politics by a particular discourse connecting situation, identity and
politics, which Anne Phillips has termed the

politics of presence.1 While apparently providing a particularly firm basis for arguments
for the inclusion of hitherto marginalized groups within the democratic process, this discourse embodies highly problem- atic
views about political dialogue and the nature of the self. I hope to build on an analysis of these flaws to clarify
the ontological preconditions of deliberative politics and, furthermore, to draw some conclusions about the nature of the obligations of parties to
deliberation. The

politics of presence rests on a model of the self as transparent to itself, but


not to others, with the consequence that a persons self-understanding, at the very least, must
be acknowledged to be authoritative, or incorrigible. This model of the self cannot, however, be
made to cohere with a plausible account of communication, and consequently it must be
discarded in favour of a broadly hermeneutic model of selfhood as situated in and
constituted through a network of language and interpretative traditions. While this self is inevitably opaque to itself in certain
respects, reflection on this opacity lends a point to dialogue which it cannot have on the assumption of self-transparency. A dialogic
politics, which is sensitive to the possibility of distorted self-understanding and aims at facilitating the transformation of perspectives and selfinterpretations, must acknowledge as a fundamental premise the provisional character of selfunder- standing. This, in turn, provides a basis for viewing deliberative citizens as having
obligations, in certain circumstances, to challenge rather than defer to the self-understandings of
others .

ROB
Our permutation frees us from the demand of immediacy in the debate
space. Imperfect, aspirational performatives are crucial for democratic
agency means you should prioritize our impacts
Leela GANDHI English @ Chicago 14 The Common Cause: Postcolonial Ethics and the
Practice of Democracy p. 164-166
4. To speak an ordinary language, that is, to be ordinary in the sense of being-in-common or being-with-others (in a word,
democratic), is an art of the possible.
"Our investigation," Wittgenstein famously notes in Philosophical Investigations, "is directed not toward phenomena, but, as one might say, toward the "possibilities" of

performative utterances
that he distinguishes as bringing something about rather than referring to something
already in place do not themselves need to be sui generis and original. 49 Thus the words that launch a marriage or a ship, or
cause an arrest to be made are, indeed, coded, iterable, and therefore citational. 50 So too, their powers of conjuration have a
great deal to do with the institutional and modular nature of the things they bring about (marriage, prison,
journeys). Nonetheless, performative meanings are always subject to creative deviation, deferral,
and/or variation. A behabitive commendation may fail to make the person being commended feel commended. An apology may take years to
have effect, or it may communicate way more or less regret than intended. Likewise, an expositive utterance plainly announcing or explaining what we are about to do
phenomena."48 The clearest tenets for the potentiality of shared meanings, thus conceived, occur in Austin's oeuvre. The

may falter mid-route. Famously (and very likely apocryphally), when training as a barrister in London, the young M.K. Gandhi once failed entirely to give birth to the speech he
was signaling repeatedly with the words "I conceive-," "I conceive-". It is this type of synapse or interval between utterance and signification-or, indeed, between utterance,

Performativity, she helps us understand,


is not, then, the certainty of effect so much as a "possibility of agency," which need not
conform to spatial and temporal specificities, and which, we might add, may also remain
unrealized. 52 To summarize, at the admittedly minor scene (though one embedded in the first half of the twentieth century) of an academic spat between Cambridge
English and Oxford ordinary-language philosophy, we find another variant on the binary of perfectionism/imperfectionism. Where Leavisite English
studies took up the cause of humanistic elevation against the threat of egalitarianism, OLP c ommitted to disciplinary dethronement in its
signification, and resignification that Judith Butler so eloquently describes as the "open temporality of the speech act."51

name. Both forms proclaimed their own wider application in the world beyond academe, in different ways. But the special-case, ahimsaic path not taken by Cambridge English,

it reformulates
ordinariness as the basis for a democratic exegetics, which consists in cultivating our most unremarkable faculties, being
and, perhaps, not fully appreciated in the descent of Western philosophy, either, is as yet available for recuperation. Very clearly, if aphoristically,

suspicious of the facts, and taking something like mere possibility very seriously as a form of politics and of knowledge. This sometimes entails giving free play to the fictionality
of any given text situation, by which, in turn, we must mean its socially critical or reparative-because-unfinished impulse. There is apt transcription of OLP principles into an
ethico-political hermeneutic by William Empson, the antiprovincial critic-poet admired by Wittgenstein, who spent many years between and after the first two world wars teaching
in China. 53 In terms especially resonant with the "possibility" thematic in OLP, Empson draws attention to the principle of ambiguity encoded within all literary experience. A
poem may be fully steeped in its context-a social work, as it were-yet there will come a point when its language begins to venture beyond the grammar of its own graspable
world. Add to this the muddy business of reading, and what you often get besides the inaugural "faint and separate judgments of probability," is the random surprise of a wildly

the
democratic critic's proper terrain is to exacerbate ambiguity. She must prevent the foreclosure of
collective sense that comes from dealing conclusively or expertly with the material, or from forcing people to attend
to "what is really there" in a text or situation. 55 Much better, Empson advises, to leave the
understanding ultimately unsatisfied and the text or literary experience ongoingly sociable. Complex permutations obtain from
such recommendations. For instance, we have all encountered textual situations the meanings of which simply
cannot be brought into view no matter how hard we try because they seem too insignificant or
unintelligible in relation to the grammar and moment of their prevailing circumstances.
Yet, a la M. K. Gandhi, such textual situations can be hypothesized, precisely in their semantic failure,
as aspirational-as bearing the strictly groundless character of a hoping and wishing that is either at odds with its
ambient circumstance or that seeks to be circumstance-altering: as in, "I wish x could be otherwise," or, "I hope against hope
that y."56 Guessing at this aspirational content or, rather, gleaning insignificance, failure, and unintelligibility as aspirational
modes is, thus, an exigent critical task. Similarly, we can also simply refuse to disclose the
autonomous final meaning. 54 These ambiguities are a measure of the plurality of likely assumptions secreted within the work and up for grabs to anybody. In context,

meanings of a textual situation by pointing to what it actually achieves or references in


the world to which it belongs. In other words, we can enforce its failure. When meanings are thus actively retracted from
realization, that is, with the disappearance of a fixed or sole referent/object/ objective, a galactic if inchoate
relational field often comes into view. This comprises the textual situation's "implication
threads," "contiguous fields" and "neighboring worlds" (to borrow some themes from Gilbert Ryle); or, ala Wittgenstein, its
sympathetic resemblance and kinship to other not even necessarily coeval textual situations .57 It goes without saying that such close-up readings
will be indistinct and blurry. 58 But they may well be proportionally more inclusive, cosmopolitan, and
global. This is but another low-key wager in the spirit of imperfection. In all such cases the risks of self-reduction
are finally negligible. If democracy is already here, we win. If it is yet to come, it is still well worth the
price. We can gamble thus from a range of venues and in the midst of miscellaneous
activities: in Makkali Gosala's cowshed; in Diogenes' bathtub; during a peace protest; as we
chat with a friend; in a naval mess hall; during wartime; in the rituals of Kant's complicated couture;
even in a library.

AT: pedagogy k/subjectivity


Pedagogy is not a new-found mode of educational inculturation, but rather
a means for achieving consciousness by foregrounding teaching moments
AGAINST power, and not just OUTSIDE power that distinction is crucial
and proves the alt can never solve
Henry Giroux 13, Chair of Secondary education at Pennsylvania State University, Henry
Giroux on the Militarization of Public Pedagogy,
http://www.counterpunch.org/2013/09/27/teaching-and-learning-with-henry-giroux/
Public schools are not simply being
corporatized, they are also subjected increasingly to a militarizing logic that disciplines
the bodies of young people, especially low income and poor minorities, and shapes their
desires and identities in the service of military values and social relations. For a lot of
these young people, there are only a few choices here: you can be unemployed and hopefully be
able to participate in some way in the social safety net, you can take a low-income job, you can end up in prison
or you can go into the military. And it seems to me that increasingly the military is becoming the
best option of all of those. So you have a whole generation which by virtue of this massive inequality
really has very limited choices. But also you have these institutions that are basically there to socialize
kids, telling them the only way to succeed is to join the military-industrial complex, and
that there really are no other options, at least for them. Moreover, as these young people are
subject to the warring logics of a militarized society, a society in which life itself is
increasingly absorbed into a war machine, it becomes difficult for them to imagine a
social order that can be otherwise, one that is organized around democratic values. SK: Like this program Ive been following:
its called STARBASE. This is a Defense Department program that every year reaches around
70,000 students in over one-thousand schools the majority of them in fifth grade. Pitched as
HG: This is an important issue and symptomatic of a much larger problem.

a way to supplement school curriculum in the STEM (science, technology, engineering, mathematics) fields, theres an insidious element of military
marketing at work: soldiers

mentor students enrolled in this program and most of the instruction takes
place at military installations. As part of the program students are given plenty of time to horse
around on cool military hardware. HG: Its mind-blowing. I think what we often forget and this is something that you
and others like yourself are trying to make clear is that when you talk about the militarization of American
society youre not just talking about increasing the military budget or arming the police
with military-style weapons and so forth. Youre also talking about the militarization of a
culture in which military values and relationships permeate every aspect of what C. Wright Mills
called the cultural apparatus schools, fashion, movies and screen culture. Violence becomes the only shared
relationship that we have to each other, the only mediating form through which people
can now solve problems. More insidiously, it defines our sense of identity and personal
liberation through violence both as a mediating force and as a source of pleasure and
entertainment. Its one of the reasons why the majority of people in the U.S. support
state-sanctioned torture. How do you explain that? Its really a culture thats become so saturated in
this military/violent mindset that it has lost any sense of critical thought and ethical
responsibility and has little understanding of what a democratic society might look like. SK: Militarism in the schools is of course just one
aspect of a larger culture of militarism in the U.S. And this gets at your notion of public pedagogy, doesnt it? HG: I may be terribly wrong but I think

the central issue here is that first of all you have to realize that the educational force of the culture represents the most important
pedagogical force at work in the United States, Canada, and in many other countries. This is not to suggest that schools are not involved in the

process of teaching and learning. But I think we commit a grave mistake when we assume that schools are the only place where learning goes on. I
would be willing to argue and I have argued that the most powerful educational force in the US is not the schools, its outside the schools.

Young people are awash in a public pedagogy that is distributed across numerous sites that extend
from movies and the Internet, readily amplified through a range of digital apparatuses that include cell phones,
computers and other electronic registers of the new and expanded cultural flows. When schools fail to make a connection between
knowledge and everyday life between knowledge and these ever expanding cultural apparatuses they fail to understand, interrogate, and question

The ongoing commercial carpet-bombing of


kids through a range of ever expanding technologiesthat make possible new social networks and information
flowsis aggressively commodifying every aspect of their lives . Not to address this and
make it pedagogically problematic, not to interrogate the massive violence kids are
exposed to through screen culture and the new digital technologies is to do an
enormous disservice to the way in which young people are being educated by the wider
culture. SK: But young people are resisting, in various ways. You were obviously inspired to write your latest book because
the educational forces that are having an enormous influence on children.

you believe youth have a role to play in fighting and changing the system. HG: As someone from the generation of the 60s, Im enormously inspired by
what theyre doing. Right now they may be the only chance that we have. Consider their courage: the bravery of these young kids in Occupy Wall
Street fighting against state-sanctioned violence in the form of police pepper spray, police dragging them off to jail and arresting them en masse.
Theyve become a model for what it is to stand up to this one percent that has turned the US into an authoritarian society. I think that what these

kids are doing is not only producing a new language to talk about inequality and power
relations in the US but theyre actually trying to create public spaces where new forms of
social relationships inspired by democratic and cooperative values are really becoming
meaningful. These young people are rethinking the very nature of politics and asking
serious questions about what democracy is and why it no longer exists in many capitalist countries
across the globe. They have been written out of the discourses of justice, equality, and democracy and are not only resisting how neoliberalism has
made them expendable, but they are also arguing for a collective future very different from the one that is on display in the current political and
economic systems in which they feel trapped. Thats important. But they face enormous challenges. They dont have access to the dominant media.
Theyre trying to use new media to create new modes of communication. Theyre trying to understand what democratic processes might mean in terms
of sustaining collective struggles, and all of this takes time. I think that rather than saying that Occupy Wall Street has died, we can say that theyre in

As conditions get worse in the


U.S. this movement will grow and take on an international significance. Hopefully theyll
join with young people in other countries to figure out how to address the biggest
problem that the global community faces politics is local and power is global. Nationstates cant control the flow of capital; its outside the boundaries of nation-states. So, we need a politics
thats global to be able to deal with that. SK: In reflecting on my own research Ive seen examples of school administrators
the process of understanding what the long march through alternative institutions might mean.

treating student activists in two distinctly different ways. In my area, Western Massachusetts, for example, there are high school students who are very
heavily involved in organizing around issues of ecology and sustainability. They lobby for locally grown foods to be served in the cafeteria, install small
garden plots for community members, school officials give them land on school property to grow vegetables, and so on. But then you have the students
in San Diego that I mentioned before. Because they were fighting against the military presence in their schools they were seen as agitators. School
administrators and police would conduct video surveillance of the students marches, and one of their leaders was prevented from taking part in the

As long as these modes of


resistance dont challenge relations of power, thats fine with school officials and others in a
position of authority. As long as theyre focused on students finding a happy spot in themselves, positive thinking, thats fine. But as
soon as they start talking about power, militarization, inequality, racism all those things
that point to deep structural problemsstudent resistance and dissent is viewed as
exceeding its possibilities and limits. Just look at what happened in places like Arizona, where these racist educators and
graduation ceremony with the rest of his class. What might explain the differential response here? HG:

politicians succeeded in banning ethnic studies. When young people protested against their history, culture, and forms of witnessing being excluded
from the curriculum, they were labeled as criminals, communists, and agitators. What is most important in terms of these youth movements is that you
have a lot of young people making connections, saying Look you cant talk about the rise in tuition unless you talk about the attack on the social state
and social protections. You cant talk about whats happening in education unless you talk about the rise of the punishing state. In a place like
California where more is spent on prisons than on education clearly those connections are what give force to a generation of students who are simply
refusing to isolate these issues. It no longer makes sense to say that these are spoiled kids who dont want to spend much for their education.

These young people are developing a conversation about society at large, calling into
question its most fundamentally oppressive economic, political, and educational
structures. Also, young people are recognizing that theyre not going to find their voice in the
Democratic Party or in the existing labor unions. What they really need to fight for are new mass and collective
organizations that can call the entirety of society into question and mobilize so as to develop the policies and institutions that make a new and radically
democratic society possible. SK: Heres a paradox for you:

How do you teach social change or resistance to

authority within public schools institutions that many have criticized for being
authoritarian and resistant to change? HG: You cant do it if you believe these institutions are so authoritarian that theres
simply no room for resistance. Thats a mistake. Power is never so overwhelming that theres no room for
resistance. Power and the forms it takes are always contradictory in different ways and
there is always some room for resistance. What needs to be understood is the intensity of
dominant power in different contexts and how it can be named, understood, and fought.
The issue here is to seize upon the contradictions at work in these institutions and to
develop them in ways that make a difference. During the sixties, the term for this was the long march through institutions
and the reference had little to do with reform but with massive restructuring of the instruments of democracy. And we also need to impose a certain kind
of responsibility upon adults in the schools whether they be social workers, university professors, or high school teachers. Clearly its not enough to
say they operate under terrible burdens that make them voiceless. I understand those structural conditions but it doesnt mean they shouldnt resist
either. That means they not only have to promote particular kinds of pedagogies in their classrooms but they also have to join social movements that

The greatest battle that were


facing in the U.S. today is around the question of consciousness. If people dont have an
understanding of the nature of the problems they face theyre going to succumb to the
right-wing educational populist machine. This is a challenge that the Left has never taken
seriously because it really doesnt understand that at the center of politics is the
question of pedagogy. Pedagogy is not marginal, it is not something that can be reduced
to a method, limited to what happens in high schools, or to what college professors say
in their classes. Pedagogy is fundamental not only to the struggle over culture but also,
if not more importantly, the struggle over meaning and identity. Its a struggle for
consciousness, a struggle over the gist of agency, if not the future itself a struggle to
convince people that society is more than what it is, that the future doesnt simply have
to mimic the present. SK: What would this look like in practice? One encouraging experiment I had the privilege
give them the force of a collective voice that can bear down on these problems and create change.

of observing up close is taking place at the Emiliano Zapata Street Academy in Oakland. There, in an alternative high school within the Oakland
Unified School District, student interns working with a group called BAY-Peace lead youth in interactive workshops on topics relevant to their lives:

two things have to go on here, and you


just mentioned one of them. Weve got to talk about alternative institutions. There has to be some
way to build institutions that provide a different model of education . On the Left, we had this in the 20s
street violence, the school-to-prison pipeline, military recruiters in their schools, and so on. HG: I think

and 30s: socialists had Sunday schools, they had camps; they found alternative ways to educate a generation of young people to give them a different
understanding of history, of struggle. We need to reclaim that legacy, update it for the twenty-first century, and join the fight over the creation of new

second level is what Rudi Dutschke called what I referred to


long march through the institutions. Its a model that makes a tactical claim to
having one foot in and one foot out. You cant turn these established institutions over to
the Right. You cant simply dismiss them by saying theyre nothing more than hegemonic
institutions that oppress people. Thats a retreat from politics. You have to fight within
these institutions. Not only that, you have to create new public spheres. SK: Henry, weve covered a lot of territory. Is there anything we
havent addressed that you would like to bring up before closing? HG: We need both a language of critique and a
language of hope. Critique is essential to what we do but it can never become so
overwhelming that all we become are critics and nothing else. It is counterproductive for
the left to engage in declarations of powerlessness, without creating as Jacques Rancire argues
new objects, forms, and spaces that thwart official expectations . What we need to do is
theorize, understand and fight for a society that is very different from the one in which
we now live. That means taking seriously the question of pedagogy as central to any
notion of viable progressive politics; it means working collectively with others to build
social movements that address a broader language of our society questions of
inequality and power (basically the two most important issues we can talk about now.) And I think that we need to find ways to support
modes of thinking, acting, and engaging ourselves and our relations to others. On the
earlier as the

young people because the most damage thats going to be done is going to be heaped upon the next generations. So what were really fighting for is

critique is not enough; we need a language of critique


and we need a language of possibility to be able to go forward with this.
not just democracy; were fighting for the future. And so

Alt Answers

AT: Negativity Alt


Optimism and solidarity are our only hope---their strategy of negativity
accepts the foundational premises of racism as its starting point for
politics
Hooks 96
(Bell hooks 96, Killing Rage: Ending Racism, Google Books, 269-272)
black Americans are succumbing to and internalizing the racist
assumption that there can be no meaningful bonds of intimacy between blacks and
whites. It is fascinating to explore why it is that black people trapped in the worst situation of racial oppres sionenslavementhad the foresight to see that it would be disempowering for them to lose
269More than ever before in our history,

sight of the capacity of white people to transform themselves and divest of white supremacy, even as many black folks today who in no way suffer such extreme racist oppression and exploitation are convinced

black folks, like their white counterparts, have passively


accepted the internalization of white supremacist assumptions. Organized white
supremacists have always taught that there can never be trust and intimacy between the
superior white race and the inferior black race. When black people internalize these
that white people will not repudiate racism. Con temporary

sentiments, no resistance to white supremacy is taking place; rather we become


complicit in spreading racist notions . It does not matter that so many black people feel white people will never repudiate racism because of being daily
assaulted by white denial and refusal of accountability. We must not allow the actions of white folks who blindly endorse racism to determine the direction of our resistance. Like our white allies in struggle we must

Of course many white


people are comfortable with a rhetoric of race that suggests racism cannot be changed, that
all white people are inherently racist simply because they are born and raised in this society. Such misguided thinking socializes white people
both to remain ignorant of the way in which white supremacist attitudes are learned and to assume a posture of learned helplessness as though they have no agencyno capacity to
consistently keep the faith, by always sharing the truth that 270white people can be anti-racist, that racism is not some immutable character flaw.

resist this thinking. Luckily we have many autobiographies by white folks committed to anti-racist struggle that provide documentary testimony that many of these individuals repudiated racism when they were
children. Far from passively accepting It as inherent, they instinctively felt it was wrong. Many of them witnessed bizarre acts of white racist aggression towards black folks in everyday life and responded to the
injustice of the situation. Sadly, in our times so many white folks are easily convinced by racist whites and bLack folks who have internalized racism that they can never be really free of racism. These feelings aso
then obsc]re the reality of white privi lege. As long as white folks are taught to accept racism as natura] then they do not have to see themselves as con sciously creating a racist society by their actions, by their
political choices. This means as well that they do not have to face the way in which acting in a racist manner ensures the maintenance of white privilege. Indeed, denying their agency allows them to believe white
privilege does not exist even as they daily exercise it. If the young white woman who had been raped had chosen to hold all black males account able for what happened, she would have been exercising white
privilege and reinforcing the structure of racist thought which teaches that all black people are alike. Unfortunately, 271so many white people are eager to believe racism cannot be changed because internalizing
that assumption downplays the issue of accountability. No responsibility need be taken for not changing something fit is perceived as immutable. To accept racism as a system of domination that can be changed
would demand that everyone who sees him- or herself as embracing a vision of radai social equality would be required to assert anti-racist habits of being. We know from histories both present and past that white

Whites, people
of color, and black folks are reluctant to commit themselves fully and deeply to an anti-racist
struggle that is ongoing because there is such a pervasive feeling of hopelessnessa conviction
people (and everyone else) who commit themselves to living in anti-racist ways need to make sacrifices, to courageously endure the uncomfortable to challenge and change.

that nothing will ever change . How any of us can continue to hold those feelings when
we study the history of racism in this society and see how much has changed makes no
logical sense. Clearly we have not gone far enough. In the late sixties, Martin Luther King posed the question Where do we go from here. To
live in anti-racist society we must collectively renew our commitment to a democratic vision of racial
justice and equality. Pursuing that vision we create a culture where beloved community
flourishes and is sustained. Those of us who know the joy of being with folks from all walks of life, all races, who are fundamentalls anti-racist in their habits of being. need to
give public testimony. Ve need to share not only what we have experienced but the conditions of change that make such an experience possible. The interracial circle of love that I know can happen because each
individual present in it has made his or her own commitment to living an anti- racist life and to furthering the struggle to end white supremacy 272 will become a reality for everyone only if those of us who have
created these communities share how they emerge in our lives and the strategies we use to sustain them. Our devout commitment to building diverse communities is cen tral. These commitments to anti-racist

Like all beloved communities we


affirm our differences. It is this generous spirit of affirmation that gives us the courage to challenge one another, to work through misunderstandings, especially those that have to do with race and
racism. In a beloved community solidarity and trust are grounded in profound commitment to
a shared vision. Those of us who are always anti-racist long for a world in which evezyone can form a beloved community where borders can be crossed and cultural hybridity celebrated.
Anyone can begin to make such a community by truly seeking to live in an anti-racist world. If that longing guides our vision and our actions,
living are just one expression of who we are and what we share with one an other but they form the foundation of that sharing.

the new culture will be born and anti-racist communities of resis tance will emerge
everywhere. That is where we must go from here.

AT: Alt Impossible Demands Bad


impossible demands bad
iek 7
(Slavoj, 11/15, Resistance Is Surrender, http://www.lrb.co.uk/v29/n22/print/zize01_.html)
Left to this predicament is to call for a new politics of resistance. Those who
still insist on fighting state power, let alone seizing it, are accused of remaining stuck within the old
paradigm: the task today, their critics say, is to resist state power by withdrawing
from its terrain and creating new spaces outside its control. This is, of course, the obverse of accepting the triumph
of capitalism. The politics of resistance is nothing but the moralising supplement to a Third Way Left. Simon Critchleys recent book, Infinitely Demanding, is an
The response of some critics on the postmodern

almost perfect embodiment of this position.[*] For Critchley, the liberal-democratic state is here to stay. Attempts to abolish the state failed miserably; consequently, the new
politics has to be located at a distance from it: anti-war movements, ecological organisations, groups protesting against racist or sexist abuses, and other forms of local selforganisation. It must be a politics of resistance to the state, of bombarding the state with impossible demands, of denouncing the limitations of state mechanisms. The main
argument for conducting the politics of resistance at a distance from the state hinges on the ethical dimension of the infinitely demanding call for justice: no state can heed this
call, since its ultimate goal is the real-political one of ensuring its own reproduction (its economic growth, public safety, etc). Of course, Critchley writes, history is habitually
written by the people with the guns and sticks and one cannot expect to defeat them with mocking satire and feather dusters. Yet, as the history of ultra-leftist active nihilism
eloquently shows, one is lost the moment one picks up the guns and sticks. Anarchic political resistance should not seek to mimic and mirror the archic violent sovereignty it

what should, say, the US Democrats do? Stop competing for state power and withdraw to the interstices
of the state, leaving state power to the Republicans and start a campaign of anarchic resistance to
opposes. So

it? And what would Critchley do if he were facing an adversary like Hitler? Surely in such a case one should mimic and mirror the archic violent sovereignty one opposes?
Shouldnt the Left draw a distinction between the circumstances in which one would resort to violence in confronting the state, and those in which all one can and should do is

if the state is here to stay, if it is


or capitalism why retreat from it? Why not act with(in) the state?

use mocking satire and feather dusters? The ambiguity of Critchleys position resides in a strange non sequitur:

impossible to abolish it (
),
Why not accept the
basic premise of the Third Way? Why limit oneself to a politics which, as Critchley puts it, calls the state into question and calls the established order to account, not in order to

These words
simply demonstrate that todays liberal-democratic state and the dream of an infinitely
demanding anarchic politics exist in a relationship of mutual parasitism: anarchic agents
do the ethical thinking, and the state does the work of running and regulating society.
do away with the state, desirable though that might well be in some utopian sense, but in order to better it or attenuate its malicious effect?

Critchleys anarchic ethico-political agent acts like a superego, comfortably bombarding the state with demands; and the more the state tries to satisfy these demands, the more
guilty it is seen to be. In compliance with this logic, the anarchic agents focus their protest not on open dictatorships, but on the hypocrisy of liberal democracies, who are
accused of betraying their own professed principles. The big demonstrations in London and Washington against the US attack on Iraq a few years ago offer an exemplary case
of this strange symbiotic relationship between power and resistance. Their paradoxical outcome was that both sides were satisfied. The protesters saved their beautiful souls:
they made it clear that they dont agree with the governments policy on Iraq. Those in power calmly accepted it, even profited from it: not only did the protests in no way prevent
the already-made decision to attack Iraq; they also served to legitimise it. Thus George Bushs reaction to mass demonstrations protesting his visit to London, in effect: You see,
this is what we are fighting for, so that what people are doing here protesting against their government policy will be possible also in Iraq! It is striking that the course on
which Hugo Chvez has embarked since 2006 is the exact opposite of the one chosen by the postmodern Left: far from resisting state power, he grabbed it (first by an
attempted coup, then democratically), ruthlessly using the Venezuelan state apparatuses to promote his goals. Furthermore, he is militarising the barrios, and organising the
training of armed units there. And, the ultimate scare: now that he is feeling the economic effects of capitals resistance to his rule (temporary shortages of some goods in the
state-subsidised supermarkets), he has announced plans to consolidate the 24 parties that support him into a single party. Even some of his allies are sceptical about this move:
will it come at the expense of the popular movements that have given the Venezuelan revolution its lan? However, this choice, though risky, should be fully endorsed: the task
is to make the new party function not as a typical state socialist (or Peronist) party, but as a vehicle for the mobilisation of new forms of politics (like the grass roots slum
committees). What should we say to someone like Chvez? No, do not grab state power, just withdraw, leave the state and the current situation in place? Chvez is often
dismissed as a clown but wouldnt such a withdrawal just reduce him to a version of Subcomandante Marcos, whom many Mexican leftists now refer to as Subcomediante
Marcos?

Today, it is the great capitalists Bill Gates, corporate polluters, fox hunters who resist the

state. The lesson here is that the truly subversive thing is not to insist on infinite demands we know those in power cannot fulfil. Since they know that we know it, such an
infinitely demanding attitude presents no problem for those in power: So wonderful that,
with your critical demands, you remind us what kind of world we would all like to live in.
Unfortunately, we live in the real world, where we have to make do with what is
possible. The thing to do is, on the contrary, to bombard those in power with strategically wellselected, precise, finite demands, which cant be met with the same excuse.

Gilbert
No one method can achieve social change cultural and representational
experiments should be supplemented by collaboration with existing
institutions.
Jeremy GILBERT Cultural and Political Theory @ East London 14 Common Ground:
Democracy and Collectivity in an Age of Individualism p. 200-206
Having said this, it would be problematic simply to dismiss projects such as Burning Man or even the Boom festival for their failure to exhibit these qualities: to some extent their
lack of self-doubt and political ambition are conditions of possibility for their success as sites of relatively safe affective and symbolic experimentation. If they lack effective
connections with wider political movements then the fault lies as much with those movements, or with all of us who might have built new ones but haven't yet.

Although

it is right to try to catalyse processes of democratic self-questioning and the


interrogation of internal hierarchies in all social situations, it remains the case that in the face of neoliberalism's
attempt to privatise every aspect of existence, it becomes politically crucial to defend every such possible site of' collective joy' (Ehrenreich 2007). It is also important to note, of
course, that raves and festivals are very far indeed from exhausting the possible forms of such sites. I have referred to them consistently here because they constitute an easy
illustration of what such a site can look like, and they have been through interesting processes of radicalisation and reterritorialisation over recent decades. There are many
other such sites which are arguably much more important, however. Given its importance to the cultural life of millions, for example, the commercialisation and celebritisation of
sport in recent years is arguably a far greater cause for concern. 16 And instances in which physical crowds gather are not of course the only significant potential sites of shared
joyous affect. Social networking protocols are just the latest manifestation of the inherent capacity of advanced communications technologies to manifest this potential, at an
extraordinary level of distribution and complexity, enabling all kinds of empowering connections to occur between disparate elements of human experience; and it is clearly a
potential which companies such as Facebook wish to contain, delimit, and exploit carefully: ensuring, for example, that users maintain a single .individual profile, identifying

I want to take this


complexification of the idea of a 'site of collective joy' even further. I would suggest, in fact, that this kind of joy need not
have anything to do with the physical proximity of bodies or the noise of the electronic crowd. Even
an activity as superficially solitary as reading in a library can be understood as an
experience of such a site, to the extent that it involves a creative and productive
interaction between singularities: those elements of the reader's conscious and
unconscious attention which are engaged in reading; the multiple ideas and possible
uses thereof which are partially expressed in the books they read; the elements of the
physical, architectural, economic, social, cultural and political assemblage which make
the very existence of a library possible; and so on. A library can only exist - can only be individuated - as the
consequence of a complex process of social interactions, and can only function well to
the extent to which it works as a commons to increase the capacities of its users, while
remaining sufficiently flexible and open in form and function to accommodate the
invention of multiple and changing uses. Perhaps most importantly for our purposes, a library is not simply a public
space or a private space, but must be at the same time both and neither, enabling a
diverse population to share resources while also enabling each person, if necessary, to
do so relatively undisturbed. This, just as much as the raving crowd (Gilbert 1997), incarnates the ideal of the multitude as a collectivity which
themselves as easily trackableconsumers, and do not engage in any kind of activity which cannot be profitably data-mined. But

empowers but does not suppress the singularity of its constituent elements; and it is possible to experience the democratic sublime in a moment of exemplary clarity or

What such spaces have


in common is that they are all, in a certain sense, spaces of decision, within and from
which new individuations and new becomings can emerge. This is not to say they are necessarily spaces within which actual conscious choices
are made (although they might be). In fact they are spaces within which we can only experience the
ultimate impossibility of making a 'decision' or 'choice' according to the classical liberal
model of the rational, intentional, autonomous and autochthonous subject: a decision which is final, which
is ours alone, and which is an expression of only our rational interests. But it is by virtue of this fact that
they are spaces conducive to the expansion of a field of potentiality and possibility, without
which no new decisions, no new individuations, no collective joy, and hence no democracy are ever possible.
exhilarating confusion (or to experience the disempowerment of bored frustration} at least as much in the one place as the other.

Problems of Strategy and Government The arguments presented in this chapter so far invite some obvious reactions: How far is it possible to construct systems and institutions
which facilitate the emergence of the kind of hetero-affective collective decisions described earlier in the chapter, not just at the level of political organisation, but also at the level

of government? At the same time, at the level of political organisation itself: what happens when waiting for such events of collective individuation to emerge simply will not do,

social change
is of course not ultimately possible without determined efforts by broad-based aggregations
of political forces co-ordinated by a viable political strategy: without, in other words, hegemonic
projects. The democratic potential of the Latin American multitude would not have
reached any kind of realisation without the strategic co-ordination enabled by the
political parties led by Chavez, Morales and Lula. The women's movement only secured
significant results by successfully transforming the common sense of a majority of the
public in many countries in the world and implementing consequent legislative change. The inability of, for example, the rave and festival movements in
when circumstances demand that determinate, conscious choices have to be made here and now? One answer to this question is to observe that

the United Kingdom to defend themselves from criminalisation and commercialisation is testament to how far being 'an affective process without a subject' gets you in the long

One of the characteristics that


all of the most successful political and cultural projects for democratisation share including but
not limited to those already discussed in this chapter- is what I have called a 'strategic orientation' (Gilbert 2008b). By this I mean not a
determinate strategy, but merely an awareness of the complexity and specificity of their strategic
situation: their strengths, limitations, threats, opportunities and opponents in the
broader field of political forces. However, if this sounds like a rather minimal concept, then so it should, because it is important not to have
unreasonable expectations of those types of political and cultural intervention which necessarily operate on a 'molecular' scale. No single project,
organisation, tendency or process can be expected to deliver radical social change; such
change can only ever come about as the result of a complex distribution and aggregation
of forces. The gains (and failures) of the women's movement, for example, have always been a product of relationships between interventions in the domains of
term, if your enemies are stronger and better organised than you are (McKay 1996, Gilbert and Pearson 1999).

affective relations, symbolic culture (for example, involving questions of the representation of women in print and broadcast media), political organisation (for example, involving
questions of women's representation within political and governmental institutions), and institutional management (Fraser 2013). To put it crudely: the upshot of this observation
is that the necessary task of hard political strategising should not be understood as falling on every group who wants to make social change happen: under conditions of
advanced neoliberal post-democracy, it may well have to fall to established and well-resourced mainstream political organisations: the unions and the social-democratic parties,
in particular, or those intermediary organisations which seek to mediate between them and a range of other social actorsY Conversely, professional political organisers and
leaders may have to accept that they cannot bring about the cultural change which would make their political projects viable, and instead be on the lookout for sympathetic and

The complex interdependence between the 'molar'


and 'molecular' dimensions of politics is not, of course, a new phenomenon and does not work in only one direction. For
example, the efflorescence of radical democratic demands in the 1960s was itself partly a
product of the successes of the social-democratic governmental projects from the 1930s
onwards, which freed large populations from the immediate fear of poverty for the first
time since the industrial revolution, so enabling a profounder imaginative challenge
amongst many of them to existing social relations than would otherwise have been
possible;18 this social-democratic success was itself made possible in part by the 'molecular' cultural changes of the inter-war period: in particular the diffusion of'
modernism', in both its avant-garde and popular variants (Williams 1989). It is therefore obviously a mistake to imagine that either the
strategic, molar and hegemonic or the molecular, affective and experimental dimensions
of political struggle can ever be ignored. Nor can any one of them be expected to bear the
full weight of hopes and demands for social change. In most contemporary contexts, it is
to be expected that the multiple tasks required to make change possible are likely to be
borne by quite different kinds of agent: from art movements to think tanks to university
departments to civil society organisations to political parties. Such tasks include
generating new modes of thought and perception which might contribute to cultural
change; crystallising those affective changes into meaningful political demands;
strategically co-ordinating a range of demands and constituencies into a viable political
coalition; delivering a coherent programme for government which instantiates some of
those changes; recruiting and mobilising a cadre of professional politicians who can
implement this programme; sustaining the affective and semiotic potency of those demands to the point that such realisation becomes likely; and
many others. Because such tasks require quite different dispositions and competences, it is not
surprising that their agents often dislike each other and find mutual comprehension
difficult; but it is probably necessary for any kind of democratic progress that there
should exist a degree of what we might call 'molecular sympathy' between them. Arguably one of
potentially important tendencies as they emerge within wider culture.

the most debilitating features of the political Left - mainstream and radical - in recent decades has been its inability to connect or even resonate at all with sites of radical cultural
experimentation. 19 This raises once again the issue of what kinds of political organisation and institutional innovation might make radical democratic hopes concretely
realisable. This is an issue addressed by Erik Olin Wright and Archon Fung in their co-edited book Deepening Democracy: Institutional Innm,ations in Empowered Participat01y
Gwernance (Wright and Fung 2003). This work is largely a collection of studies of localised experiments in participatory and deliberative democracy, such as the famous Porto
Alegre participatory budgeting process, or the decentralised planning process deployed by the leftist government of Kerala.20 In itself the collection stands as evidence for the
viabilityof participatory democratic forms in wildly varying contexts; but what is particularly interesting for us here is the conclusion reached by the editors in their epilogue.

experiments in participatory democracy, at least under present socio-political conditions, are


always in danger of degeneration into democratic inefficacy, or co-option, or
neutralisation by more powerful political and commercial forces. They argue that these
outcomes are only avoidable where there exists sufficient 'countervailing power' - in other words,
sufficiently well-organised and mobilised political constituencies- to defend their democratic status. Going further, they suggest that this countervailing
power must be deployed by forces which are strong and well-organised, but whose
relationship to government is not habitually adversarial , but instead collaborative :
engaging in complex tactical problem-solving, and constructive institutional
engagement, but from a position of strategic strength. Wright and Fung underscore, with a justified degree of pessimism, the difficulty of mobilising on such terms
Wright and Fung argue that such

movements and organisations whose identities and practices are grounded in adversarial relationships to existing power structures. A number of the conceptual distinctions we
have encountered in this study are relevant to understanding this argument. In Laclau's terms, Wright and Fung can be read as suggesting that such radical democratic
innovation requires an 'institutionalist' (or perhaps we might coin the term 'counter-institutionalist'21 ) practice on the part of movements formed on a populist basis. Following
Deleuze and Guattari, we could say that the difficulty they highlight is that of enacting a molecular deterritorialisation of existing institutions from the position of a molar
collectivity. And yet the great value of the schizoanalytic perspective is that it demonstrates the extent to which every such molarity is already an assemblage, constituted by its
lines of flight and its molecular processes, and so would suggest that such a transition need not be understood simply as the reversal of a group's constituted nature, but as an
activation and intensification of its most dynamic constituent tendencies. This last phrase of mine deliberately echoes a key distinction made by Antonio Negri between
'constituent power' (the creative power of the multitude of which all true democracy is an expression) and 'constituted power' (actually existing institutions of government) (Negri
1999). In fact what seems to be at stake here is a generalised extension of Gramsci's concept of political struggle as a 'war of position', a sort of ongoing trench or siege warfare
which is distinguished from the full-frontal revolutionary assault of the classic 'war of manoeuvre' or 'war of movement'. The same thing happens in the art of politics as happens
in military art: war of movement increasingly becomes war of position, and it can be said that a State will win a war in so far as it prepares for it minutely and technically in
peacetime. The massive structures of the modem democracies, both as State organisations, and as complexes of associations in civil society, constitute for the art of politics as
it were the 'trenches' and the permanent fortifications of the front in the war of position. (Gramsci 1971: 243) Wright and Fung's formulation develops an element which is

even while it is strategising against its opponents and inventing


institutions of its own, a radical force must have a constructive, creative dimension. The
implication of the foregoing argument is that this creative dimension cannot be
expressed only through the positive, ex nihilo construction of new institutions; it also
requires processes of molecular, transformative engagement with existing systems.
already implicit in Gramsci's:

Permutation Stuff

Perm Negativity Combined with Positivity


Their insistence on negativity as the starting point and generating
competition is problematiconly our inclusive approach can create
movements and tangible change
Brand-Jacobsen 5
(Kai Frithjof, is founder and Director of the Peace Action, Training and Research Institute of Romania (PATRIR) and Co-Director of
TRANSCEND, and is on the Executive Board of the TRANSCEND Peace University (TPU) where he is Course Director for the
courses Peacebuilding and Empowerment and War to Peace Transitions. He has worked in Afghanistan, India, Pakistan, Nepal,
Russia, South Eastern Europe, North America, Colombia, Somalia, Cambodia, Aceh-Indonesia and the Middle East at the invitation
of governments, inter-governmental organisations, UN agencies, and local organisations and communities. He has written and
published widely, and is author of The Struggle Continues: The Political Economy of Globalisation and People's Struggles for Peace
(Pluto, forthcoming), co-author, together with Johan Galtung and Carl Jacobsen, of Searching for Peace: The Road to TRANSCEND
(Pluto, 2000 & 2002) and Editor of the TRANSCEND book series published together with Pluto Press, Constructive Peace Studies:
Peace by Peaceful Means. He is a member of the Executive Board of the Journal of Peace and Development and the Executive
Board of the Centre for Peace and Conflict Resolution. In 1999 he was founder and Director of the Coalition for Global Solidarity and
Social Development, and in 2000, together with Johan Galtung, he was founder of the Nordic Institute for Peace Research (NIFF).
Since 1996 he has provided more than 250 training programmes in peacebuilding, development, and constructive conflict
transformation to more than 4000 participants in 30 countries. http://www.globalsolidarity.org/articles/peace_means_kai.html)
Peace by Peaceful Means Dear Friends, The discussions which have taken place over e-mail over the past few days have been extremely interesting. I have just returned from Oslo where the 100th anniversary
of the Nobel Peace Prize was being celebrated. The obvious contrast between the rather elite 'suit' dominated celebrations in Oslo and the realities of what is occurring in the world today was stark.

Questions of strategy, tactics and visions for how we work to bring about change, to transform all
forms of violent conflict -- direct, structural, and cultural -- and to empower, mobilise, and involve people in a mass, broadbased movement for peace and to build the alternatives we are looking for, are vital . In Norway alone, to take one example, perhaps 80% of people think what is
happening now in and over Afghanistan is wrong, either completely or at least in part, and yet all they hear from the media, academics and politicians is constant support and acclaim for the 'justness' of this war

Small groups of people and 'NGOs', in Norway as in every single country, are trying to bring
forward alternatives, to raise their voices, and to protest/oppose what they think is wrong. While these organisations are in every case much smaller than our governments and
militaries going to war, they often represent the social majority. A major challenge they face, however, is how to reach out to people ,
how to involve people, and how to develop alternatives which make sense to people tired of war and violence (whether of the kind we
are seeing in Afghanistan, or of a global economic system killing 100,000 a day). Negative slogans and opposition to what is wrong is
(or indeed, any war in which it is 'we' against 'them').

not enough however. It is not enough, but it is necessary. 'Basta!', 'Enough!' was perhaps the most 'revolutionary'
cry of the last decade, and still is in many parts of the world. The simple, courageous act, of standing up when we see that
something is wrong, and stating that it is wrong, not cooperating with it, can be a powerful and evocative symbol. When we are
having our conferences, discussions and meetings in whichever city, town or village of the world we may be found, we should always remember that the vast majority of people in our own city, town or village, as
well as the entire rest of the world, have no idea that we are there, meeting.

The vision, hope and ideas which bring people to these

conferences are, in the vast majority of cases, kept marginalised, on the periphery. Yet that is also part of
our own responsibility, technique and methods. Basta! became a cry to inspire millions, because those who said it lived it, refusing to cooperate any longer with what they know
to be wrong. While Basta! may be the most revolutionary cry or word today, transforming all
forms of direct, structural, and cultural violence is the greatest challenge. The two are
inclusive and complementary, not exclusive. We need to state clearly our opposition to
violence, war, injustice and exploitation (the 'peace movement' has often been willing to do the first two, not always as willing on the last two), and we need also to build a
constructive, positive programme . It is not only a question of what we are against, but
what we are for. When we criticize what we think is wrong, people will also want to know what we think could be
done instead. In these cases, our answers must seem real and viable to people. The 'anti-globalisation' movement is
therefore also a social justice movement; 'non-governmental organisations' should also be people's organisations or people's movements; and one of our challenges today will be to build upon the growing 'antiwar' movement, transforming it also into a peace movement. A step further, as many social and peace activists have recognised, will be to link the peace and social justice movements. Slogans and messages are
important, as are practice and vision

. It will not be possible today to unite broad numbers of people around

issues which they feel are too abstract and divorced from them. The 'abolish the debt'
campaign/movement was successful because people were able to see the clear linkages
between debt and the effective colonisation and enslavement of countries and people
across the south, as well as the incredible suffering and destruction it brought. The Jubilee 2000 'campaign' however, unlike the Jubilee South movement which continues today, did not
reach its objective of having the debt cancelled. Instead, while many people around the world believe the problem has been solved, the debt-system and the burden it places upon countries has become even

it is not a question of 'either/or' but 'both/and' with


individual campaigns extremely useful and effective at times for involving people, raising
awareness and mobilising around specific issues, strengthening further the broader
movements of which they may be a part. Today, a movement for demos kratos is necessary , and vital for any movement or work
more extreme. Going from 'campaigns' to movements will also be important, though even here

towards peace. To speak about the United States or any government in the world today as a 'democracy' is a ridiculous farce. They are highly elite dominated systems built upon massive structures and cultures of
violence, and willing to use overwhelming (Powel Doctrine) violence when necessary to enforce their needs and/or interests. At best they may be demagogia's, where elites maintain power by promising the
people what they will do for them (we call this 'elections'), but they are not system's or societies built upon people's power, demos kratos. Decisions to go to war are made by tiny numbers of people. Our economic
and political policies are constructed for us, often to the detriment of the social majorities who are told to 'leave well enough alone' and trust in the experts. This is sometimes as true of politicians as it is of nongovernmental organisations who themselves frequently prefer the conference halls and well-funded projects to actually working democratically with people as part of the people themselves. An alternative today,

discussions at every level, focussing not only on what is


wrong, but also on what we want therapy, ideas, alternatives. In one form or another many of these dialogues are taking place. In
what Johan Galtung has called for, with 10,000 dialogues, meetings,

a way they are therapy for the massive amounts of violence we are all being exposed to today, in our cultures, in our world, on our television sets or in the speeches of our 'democratically elected' rulers (the

are also
empowering, if we take the step beyond saying what is wrong to what could be done _, what
question, for those who do not support their policies, should not be 'who put them in power' -- though this is also important -- but why haven't we removed them from power yet_). They

should be done_, and then go further to discussing what I/we can do about it. Mobilising people for peace today is not simply about a slogan (though coming up with clearly expressed messages in a few words

What is necessary, beyond any single issue or top-level


strategy for how to change the world, is the process . The way is the goal. Perhaps the greatest achievement of the social justice/antiwill of course help us to link people together and raise awareness).

globalisation movement is that it has mobilised, involved, and empowered millions of people around the world in discussing, thinking about, and acting upon the realities around them. On the streets of Seattle,
Praha, Okinawa, Melbourne, Gotheburg, Washington, Quebec, Genoa, Ottawa, people, many of whom refuse to vote, have been discussing foreign policy, domestic politics, people to people movements, and all

We have our 'manifestos', our policies and plans which


often addressing them to 'politicians' and 'elites' believing, in a fundamentally undemocratic way, that they will be
the ones to bring about and implement change for us. This is not to say that that is not an important level which we also need to work at. The broader vision
here is both/and, not either or, in terms of strategy as well often of vision. We also need,
however, to be willing to take part in the much slower, more timely, and more empowering
process, of tens of thousands of dialogues together with people, communities, and organisations at every level. Solidarity
today is being built upon and carried further into alliances not just supporting people in their struggles for
social justice, peace and freedom, but carrying forward those struggles ourselves in our own communities, our own towns, cities
and villages. If we wish to change the injustices taking place in the world today we must of
course work on a global level, but we must also work, just as importantly, within our communities.
Again, both / and rather than either or. We should also be wary when we say 'we must
the issues which politicians and well-established NGOs are not able and often not willing to discuss with people.
we wish to put forward in the name of people,

begin here', or 'this must be done first !', even when the message is very positive and constructive. 'We must begin with the
individual!'. 'We must begin by changing society!'. 'We must begin with a culture of
peace!'. 'We must begin by ending the debt!'. All of these , and the many others put forward, are
extremely important issues. They are also all linked together . Again, both/and. Exclusive and elitist
visions will only serve to further fragment our efforts, creating division and separation
where what is needed is dialogue, solidarity, cooperation and alliances between
movements/organisations which often take diverse strategies and approaches to addressing
deeply interlinking injustices and structures and cultures of violence. Conscientisation (raising awareness, often political awareness -- but also
social, cultural, economic), organisation (we can do more together than we can apart, and it is necessary to organise -- though in many different ways -- to be able to bring about changes, both
against what we think is wrong and for what we think is right), mobilisation (bringing in more and more people, involving people in dialogues, discussion, action, and work
for change/transformation), and empowerment (I/we can, rather than 'I/we can't'; also important recognising the power we have to bring about change, rather than simply accepting
existing, often extremely violent, power structures and believing that change can/should/must be implemented by those 'in power', whether slave owners, men, politicians, or fuhrers) are all
necessary .

Institutions KT Solve Anti-Blackness


the law is obviously problematic, but thats a reason we should hold it
accountable to live up to its ideals
Crenshaw 88
(Kimberle, Law @ UCLA, RACE, REFORM, AND RETRENCHMENT: TRANSFORMATION
AND LEGITIMATION IN ANTIDISCRIMINATION LAW, 101 Harv. L. Rev. 1331, lexis)
Questioning the Transformative View: Some Doubts About Trashing The Critics' product is of limited utility to Blacks in its present form. The implications for Blacks of trashing liberal legal ideology are troubling,
even though it may be proper to assail belief structures that obscure liberating possibilities. Trashing legal ideology seems to tell us repeatedly what has already been established -- that legal discourse is unstable
and relatively indeterminate. Furthermore

, trashing offers no idea of how to avoid the negative consequences of

engaging in reformist discourse or how to work around such consequences. Even if we


imagine the wrong world when we think in terms of legal discourse, we must
nevertheless exist in a present world where legal protection has at times been a blessing -albeit a mixed one. The fundamental problem is that, although Critics criticize law because it functions to
legitimate existing institutional arrangements, it is precisely this legitimating function
that has made law receptive to certain demands in this area. The Critical emphasis on deconstruction as
the vehicle for liberation leads to the conclusion that engaging in legal discourse should
be avoided because it reinforces not only the discourse itself but also the society and the world
that it embodies. Yet Critics offer little beyond this observation. Their focus on delegitimating rights rhetoric seems to
suggest that, once rights rhetoric has been discarded, there exists a more productive strategy for change, one
which does not reinforce existing patterns of domination. Unfortunately, no such strategy has yet
been articulated , and it is difficult to imagine that racial minorities will ever be able to
discover one. As Frances Fox Piven and Richard Cloward point out in their [*1367] excellent account of the civil rights movement, popular struggles are a
reflection of institutionally determined logic and a challenge to that logic. 137 People can
only demand change in ways that reflect the logic of the institutions that they are
challenging . 138 Demands for change that do not reflect the institutional logic -- that is, demands that
do not engage and subsequently reinforce the dominant ideology -- will probably be
ineffective . 139 The possibility for ideological change is created through the very process of legitimation, which is triggered by crisis. Powerless people can
sometimes trigger such a crisis by challenging an institution internally, that is, by using
its own logic against it. 140 Such crisis occurs when powerless people force open and
politicize a contradiction between the dominant ideology and their reality. The political consequences [*1368] of
maintaining the contradictions may sometimes force an adjustment -- an attempt to close the gap or to make things appear fair. 141 Yet, because the adjustment is triggered by the political consequences of the

This approach to understanding legitimation and


change is applicable to the civil rights movement. Because Blacks were challenging their exclusion from political society, the only claims that were likely to achieve
contradiction, circumstances will be adjusted only to the extent necessary to close the apparent contradiction.

recognition were those that reflected American society's institutional logic: legal rights ideology. Articulating their formal demands through legal rights ideology, civil rights protestors exposed a series of

Rather than using the


contradictions to suggest that American citizenship was itself illegitimate or false, civil
rights protestors proceeded as if American citizenship were real, and demanded to
exercise the rights that citizenship entailed. By seeking to restructure reality to reflect
American mythology, Blacks relied upon and ultimately benefited from politically inspired efforts to
resolve the contradictions by granting formal rights. Although it is the need to maintain legitimacy that presents powerless groups with the opportunity
contradictions -- the most important being the promised privileges of American citizenship and the practice of absolute racial subordination.

to wrest concessions from the dominant order, it is the very accomplishment of legitimacy that forecloses greater possibilities. In sum, the potential for change is both created and limited by legitimation. 139 The
possibility for ideological change is created through the very process of legitimation, which is triggered by crisis.

Powerless people can sometimes trigger

such a crisis by challenging an institution internally, that is, by using its own logic
against it. 140 Such crisis occurs when powerless people force open and politicize a
contradiction between the dominant ideology and their reality. The political consequences [*1368] of maintaining the
contradictions may sometimes force an adjustment -- an attempt to close the gap or to make things appear fair. 141 Yet, because the adjustment is triggered by the political consequences of the contradiction,

This approach to understanding legitimation and change


is applicable to the civil rights movement. Because Blacks were challenging their exclusion from political society, the only claims that were likely to achieve recognition were
circumstances will be adjusted only to the extent necessary to close the apparent contradiction.

those that reflected American society's institutional logic: legal rights ideology. Articulating their formal demands through legal rights ideology, civil rights protestors exposed a series of contradictions -- the most

Rather than using the contradictions to


suggest that American citizenship was itself illegitimate or false, civil rights protestors
proceeded as if American citizenship were real, and demanded to exercise the rights
that citizenship entailed. By seeking to restructure reality to reflect American mythology,
Blacks relied upon and ultimately benefited from politically inspired efforts to resolve the contradictions
by granting formal rights. Although it is the need to maintain legitimacy that presents powerless groups with the opportunity to wrest concessions from the dominant order, it is the very
important being the promised privileges of American citizenship and the practice of absolute racial subordination.

accomplishment of legitimacy that forecloses greater possibilities. In sum, the potential for change is both created and limited by legitimation.

Non-Mutually Exclusive
Must take action---pessimism doesnt necessitate passivity
Seidman 11
(Louis Michael Seidman 11, Professor of Constitutional Law at Georgetown, HyperIncarceration and Strategies of Disruption: Is There a Way Out?,
moritzlaw.osu.edu/osjcl/Articles/Volume9_1/Seidman.pdf)
My worry about myself is not that I would make a wrong choice between these alternatives, but that I would
avoid the necessity of choice by adopting a stance of world-weary passivity or
complacent indifference . There is no good way to deal with overwhelming evil, but there
is surely a way not to deal with it. Willful blindness, craven apologetics, and flaccid acquiescence are
not options when millions of our fellow inhabitants are behind bars. Like the people of good will trapped in
Nazi Europe, we opponents of hyper-incarceration need to confront the truth of our situation and be honest about how bleak the
prospects are. But pessimism should not be confused with passivity. Even if our efforts
accomplish nothing at all, and whatever form our politics take, we must do what we can
to stop the horror. Our personal dignity demands no less.

Reforms Work Pyle


Pure resistance entrenches the status quo---must be willing to propose
imperfect reforms
Pyle 99
(Jefferey, Boston College Law School, J.D., magna cum laude Race, Equality and the Rule of
Law: Critical Race Theory's Attack on the Promises of Liberalism, 40 B.C.L. Rev. 787)
race-crits are strangely unrealistic in their proposals for reform. 1 m7 Most probably realize
crits generally
prefer not to suggest solutions, but to "resist" the dominant legal thought, doctrine and policy, whatever
For all their talk of "realism,"'"

that radical measures like racial or ethnic reparations are not likely to be granted, especially by a court. But even unrealistic proposals are rare, because race-

that happens to be.'" As Derrick Bell has put it, "most critical race theorists are committed to a program of scholarly resistance, and most hope scholarly resistance will lay the
groundwork for wide-scale resistance."'"

How this ivory tower oppositionalism would foment grassroots

revolt is unclear, because CRT professors rarely suggest anything practical. Rather, their
exhortations are meant, as Bell says, to "harass white folks" and thereby "make life bearable in a society where blacks are a permanent, subordinate class."'" One of the raceerns' few practical programs of "resistance" is Paul Butler's proposal that inner-city juries practice racially-based jury nullification.'91 jurors of color, Butler argues, have the
"moral responsibility" not to apply the criminal law to blacks and whites equally, but to "etnancipate some guilty black outlaws" because "the black community" would be "better
off" if there were fewer black men in prison.'" If enough juries were hung or not-guilty verdicts rendered, he imagines, the white-dominated government would change its
excessive reliance on incarceration.'" Butler rejects the ordinary democratic process of legal reform.' Democracy, he says, ensures a "permanent, homogenous majority" of
whites that "dominat[es]" African Ainericans.w5 Butler is probably correct that occasional acts of jury nullification might well express the resentment that many African Americans
justifiably feel towards discriminatory law enforcement.'"`' As Randall Kennedy has pointed out, however, black Americans are disproportionately the victims of crimes,'97 and

preference for "resistance"'99 over


democratic participation seems to flow from a fear of losing their status as "oppositional
scholars] "200 to the game of mainstream law and politics, which they regard as "an inevitably
co-optive process?"' Better to be radically opposed to the "doniinant political discourse""2 and remain an out than to work within the current system and lose
one's "authenticity?" In rejecting the realistic for the "authentic," however, race- crits begin to look
therefore tend to favor more, not less, criminal prosecution and punishment. 1 "8 The race-crits'

like academic poseursideological purists striking the correct radical stance, but doing
little within the confines of the real world, so sure are they that nothing much can be
done."

Coalitions Key
Perms best---the alt alone causes imperialistic domination
Churchill 3
(Ward, former professor of ethnic studies at the University of Colorado at Boulder Acts of
Rebellion: The Ward Churchill Reader pg 106)
It is in this last connection that the greatest current potential may be found, not only for the Newes in their struggle to retain (or regain) their homeland, but for (re)as sertion of indigenous land rights more generally,

In the combination of forces presently


coalescing in the Nevada desert lie the seeds of a new sort of communication, understanding, respect, and the growing
promise of mutually beneficial joint action between native and nonnative peoples in this
hemisphere.294 For the Shoshones, the attraction of a broad and broadening base of popular support for their rights
and for the struggles of nonindians who seek genuinely positive alternatives to the North American status quo.

offers far and away the best possibility of bringing

to bear the kind and degree of

pressure necessary to compel

the federal government to restore all, or at least some sizable portion, of their territory. For the nonindian individuals and organizations
involved, the incipient unity they have achieved with the Newes represents both a conceptual breakthrough and a seminal practical experience of the fact that active support of native land rights can tangibly
further their own interests and agendas.295 For many American Indians, particularly those of traditionalist persuasion, the emerging collaboration of nonindian groups in the defense of Western Shoshone lands
has come to symbolize the possibility that there are elements of the dominant population that have finally arrived at a position in which native rights are not automatically discounted as irrelevancies or presumed
to be subordinate to their own.296 On such bases, bona fide

alliances can be built. Herein lies what may be the most important lesson to be learned by those attempting to forge a
Native Americans cannot hope to achieve

truly American radical vision, and what may ultimately translate that vision into concrete reality:

restoration of

the

lands and liberty which are legitimately theirs without the support and assistance of nonindians , while nonindian

activists cannot hope to effect any transformation of the existing social order which is not fundamentally imperialistic, and thus doomed to replicate some of the most negative aspects of the present system,
unless they accept the necessity of liberating indigenous land and lives as a matter of first priority.297 Both sides of the equation are at this point bound together in all but symbiotic fashion by virtue of a shared
continental habitat, a common oppressor, and an increasingly interactive history.

There is thus no viable option but to go forward together , fig-

uratively joining hands to ensure our collective well-being, and that of our children, and our children's children.

AT: Coalition Link


Critique of the racial state shouldnt preclude appeals to state-based
politics.
Lipsitz 4 George Black Studies @ UC SB Abolition democracy and global justice
Comparative American Studies 2 (3) p. 271-276
Abstract As

new social relations produce new kinds of social subjects, scholars in American Studies and
The Civil Rights tradition of the 14th
Amendment plays an important role within progressive American Studies scholarship,
but in the course of seeking equality and exclusion within the USA, this tradition runs the
risk of occluding the role of the nation in the world and its central role in creating and preserving inequality
and injustice in other nations. An emerging emphasis on struggles for social justice without seeking
state power encapsulates many of the most progressive impulses within Area Studies and transnational
studies, yet this perspective runs the risk of occluding the enduring importance of the
Area Studies experience anxieties about disciplinary as well as geographic borders.

nation-state in inflecting global developments with local histories and concerns. The present moment challenges us
to draw on both traditions, and to use each to critique the shortcomings of the other,
while at the same time promoting an inclusionary, nonsectarian , and mutually
supportive dialogue about our differences. Keywords American Studies Area Studies inequality transnationalism In Jack
Conroys 1935 short story The Weed King, a stubborn Missouri farmer wages a one person war
against the weeds that spring up in his fields. Believing that farming would be an easy
job if it were not for the weeds, he dedicates himself to their eradication with a zeal that astounds his
fellow workers. The weed king embraces his war against weeds as his reason for being. His only vanity, Conroy tells us, is his belief that he has put
the quietus to more weeds than any man, woman, child or beast west of the Mississippi (Conroy, 1985: 101). Even in the winter time when snow
covers the ground, the zealot worries night and day about the tiny seeds waiting to bloom in the spring. One of his neighbors points out that weeds
have their uses too, that many of them have greatly-needed medicinal powers. However, the weed king is not deterred. He soon succeeds in
suppressing most of the weeds on his property. His

singleminded zealotry has its costs, however. The


measures he takes to kill the weeds prove fatal to his crops as well. At the present moment of
tumultuous transformation and change, scholars in American Studies and Area Studies might be tempted to emulate the
weed king, to keep a keen eye on our fields to protect what we have been cultivating for
so many years, to view each others work with trepidation and counter-insurgent zeal . American Studies
scholars worry that the growing enthusiasm for transnational studies threatens to focus too much on exchanges across national boundaries, in the
process occluding the unique, particular, and specific inflections given to those processes by distinct national histories, cultures, and politics. Area
Studies specialists, many of whom have been part of a decades-long tradition dedicated to constructing epistemologies and ontologies that resist the
hegemony of the monolingual, monocultural, and nationalist scholarship of the US academy, rightly fear that a transnational or postnational American
Studies might simply project American Exceptionalism onto a broader geographic terrain. Outside the USA, specialists in both American Studies and
Area Studies have reason to fear that (wittingly or unwittingly) scholars from the USA will use the power of US capital, communications media, and
commerce to substitute a US-centric monologue masquerading as a dialogue for the greatly needed polylateral communication and collaboration that a
transnational world requires. At a time when substantive changes in social structures, technology, and politics are radically reconfiguring the relations
linking culture, time, and place, policing the boundaries of disciplines speaks to deep desires for continuity and certainty. It is possible to look at the
current ferment in our fields and see only what is being lost, to become subsumed with melancholy about lost conversations and conventions. Yet
scholarly research should be conducted out of conviction, rather than out of habit. If we are not careful, our work can come to resemble Swedish
anthropologist Ulf Hannerzs definition of Scandinavian cooking something passed down from generation to generation for no apparent reason
(Hannerz, 1992: 42). Like

the weed king, we can worry night and day about the purity of our
fields. As new social relations throw forth fundamentally new social subjects with new
epistemologies, ontologies, archives, and imaginaries, new patterns of scholarly inquiry
will inevitably emerge. Will shallow forms of cultural and ideological critique eclipse the
grounded insights produced by ethnography or social history? Will the fetishes of archival and ethnographic research methods produce
empiricist and myopic work lacking in self-reflexivity? Will comparative work lack the cultural and linguistic depth traditionally produced by primarily
national studies? Will national studies ignore the ways in which nationalism itself is a transnational project? Will the proliferation of new social subjects

and new objects of study come at the expense of marginalizing aggrieved social groups or will it teach us how social identities become conflated with
power in richly generative and productive ways? It is understandable that these kinds of questions arise when we try to do our work. Anything worth

Yet counterinsurgency is a poor model for scholarly work, and too much attention to pulling out
weeds can kill the crops. Even more important, weeds can have curative powers if we learn to use them correctly. The author of The
doing can nonetheless be done badly, and principled questions from colleagues protect our interests as well as theirs.

Weed King confided to his biographer that his mother believed that weeds were simply plants for which no use had yet been found (Wixon, 1994: 32).

The weeds that invade a field can also inform it in crucially important ways if we learn to
recognize their curative powers. Within American Studies, the tradition of 14th Amendment Americanism may seem like the
quintessential expression of American exceptionalism. Forged from the freedom dreams and collective struggles of an enslaved people, the 14th

More
than a specific Constitutional provision promising equal treatment under law, the 14th
Amendment has functioned as a widely shared social warrant authoring and authorizing
new ways of knowing and new ways of being. In his indispensable work, Black Reconstruction in America, W.E.B. Du
Amendment stands as an enduring symbol of the accomplishments of the abolition democracy that ended slavery in the wake of the Civil War.

Bois demonstrated how slaves fighting for their freedom soon realized that it would not be enough to be merely free in a society premised on their
exclusion.

In the course of staging a general strike in the fields, running away from slavery to
swell the ranks of the Union army, and joining together to work land liberated by military
force, they formulated a political perspective that Du Bois named abolition democracy
(Du Bois, 1995). They fought for the 13th, 14th, and 15th Amendments to the Constitution. At the Charleston Black Convention in 1865 they called for
more than nominal freedom, for the development of their full being as humans. Between 1865 and 1877 they fashioned alliances with poor whites to
elect progressive majorities to office, and their successes led to the first universal public education systems in the South, to governments that
subsidized the general economic infrastructure rather than just the privileges and property of the elite. Although betrayed by the Compromise of 1877,
by the removal of federal troops from the South, by the legal consolidation of the combination of sharecropping and Jim Crow Segregation, and by
Supreme Court decisions that took protections away from black people and extended them to corporations, abolition democracy and the 14th
Amendment successfully challenged the hegemony of white male Protestant propertied power. It opened the door for subsequent claims for social
justice by immigrants and their children, religious minorities, women, workers and people with disabilities. From voting rights to affirmative action, from
fair housing to fair hiring, the 14th Amendment is an enduring and abiding force for social justice in US society. Yet American Studies scholarship that
subsumes social justice under the rubric of the 14th Amendment runs the risk of ignoring the position of the USA in the world. Celebrating struggles for
citizenship inside the USA can work to strengthen the distinctions between citizens and aliens, providing legitimation for nationalist and nativist policies
that impose enormous suffering on humans precisely because they are not US citizens. The legacy of the 14th Amendment has not prevented women
and blacks in contemporary California from supporting anti-immigrant nativism through Proposition 187, aimed at denying immigrants and their children
needed state services, or through Proposition 227, banning bilingual education in the states classrooms. Post-1965 immigrants from Asia, who owe
their entry into to the USA to the civil rights movement and its exposure of previous national origin quotas as racist, have not been immune to pursuing
the privileges of whiteness for themselves by opposing affirmative action and school desegregation policies vital to the well-being of blacks and
Latinos. At the same time, the power inequalities that separate even the most aggrieved US citizens from the masses of poor and working people
around the world can render struggles for full 14th Amendment rights by US citizens to be little more than what Martin Luther King, Jr used to describe
as an equal right to do wrong. Certainly the prominence of Colin Powell and Condoleeza Rice in forging the rationale for the 2003 invasion and

If abolition democracy emblematizes the


emancipatory tradition within American Studies, the idea of collective and linked
struggles for change without aiming for control over any one state expresses the uniquely
generative stance within transnational social movements and transnational scholarship. Articulated in the form
occupation of Iraq demonstrates the limits of this form of inclusion.

of a manifesto in John Holloways Change the World Without Taking Power, this sensibility has taken on activist form in the work of the EZLN in Mexico,
the Gabriela Network in the Philippines, and the Okinawan Women Act Against Military Violence in that Japanese prefecture (Holloway, 2002).

These movements make demands on the state and recognize the specificity of national
histories, cultures and politics, but their aspirations and activities cannot be contained
with any single national context. The activities of the Okinawan Women Act Against
Military Violence (OWAAMV) demonstrate the importance of a transnational perspective
that goes beyond the history, culture, and politics of any single nation state (Fukumura and
Matsuoka, 2002). Coming from a country that has been serially colonized since the 17th century and occupied militarily by both the USA and Japan,

Disadvantaged by colonial status, race,


and gender, they cannot turn to national liberation, anti-racism or feminism as their sole
OWAAMV activists cannot solve their problems within a single national context.

context for struggle . Coming from a small island with a limited population in a corner of the world far removed from metropolitan centers
of power, they must forge alliances with outsiders based on political affinities and identifications, rather than counting on the solidarities of sameness
that sustain most social movements. As eyewitnesses to brutal combat on the island in 1945 that killed more than 130,000 Okinawan civilians (onethird of the local population) and tens of thousands of Japanese and US military personnel, they find it impossible to celebrate organized violence and
masculinist militarism (Hein and Selden, 2003: 13). As women confronted with the pervasive presence of commercial sex establishments, sex tourism
and rapes of civilian women and girls by military personnel, they see gender as a central axis of power and struggle. The complicated history that
brought the OWAAMV into existence, and which vexes them in so many ways, has produced new ways of being and new ways of knowing that contain
enormous generative power for scholars in Ethnic Studies and American Studies. They do not seek to make their nation militarily superior to others.
Instead, they argue that massive preparation for war increases rather than decreases the likelihood of violence. Moreover, they argue that military

spending creates security for states and financial institutions but not for people. They charge that expenditures on war serve to contain and control
people like themselves who oppose the global economic system, who challenge neoliberal policies designed to privatize state assets, lower barriers to
trade and limit the power of local entities to regulate the environment. Perhaps most important, they call for a new definition of security, one that
places the security of women, children and ordinary people before the security of the state and financial institutions. They queer the nation not
because they take an explicit position on the rights of gays and lesbians, but because they interrupt and contest the narrative of patriarchal protection
upon which the nation-state so often rests. By necessity, the OWAAMV go beyond the categories and cognitive mappings of area studies. They are
citizens of Japan, but also victims of Japanese and US colonialism. On most issues, they feel more in solidarity with the indigenous Sovereignty
Movement in Hawaii or the Gabriela network mobilizing against sex tourism and sex work near military bases than they do with their fellow citizens of
Japan. The nature of US imperialism forces them to seek alliances with pacifists and feminists in the USA, with Puerto Rican activists fighting against
US military exercises on the island of Vieques, and with the Okinawans transported to Bolivia during the Cold War era when the Japanese and US
governments relocated them in that South American nation so their land could be appropriated for military uses. They feel solidarity with witnesses to
war and empire everywhere, recognizing that the things that have happened in their part of the Pacific cannot be contained within any one area of
study. Transnational

organizing of mobilizations for change, without directly seeking to take


state power, speak directly to the new circuits and networks of power emerging from new
forms of production, consumption, communication and repression. They often display brilliant ingenuity in
fashioning seemingly unlikely short-term alliances, affinities and identifications with people
across class, gender, race and national lines. Yet this very tactical dexterity makes it
difficult to turn temporary victories into long-term institutional changes . Strategies that
manifest the mobility and dynamism required for challenging transnational corporations
and financial institutions often lack the concentrated power needed to challenge the
enduring power of the state and its control over the prisons, armies and police agencies deployed in
support of private power everywhere. Even more important, flexible, fluid and dynamic coalitions often lack both the organic solidarity and the

Groups engaged in this kind of struggle can become


unexpected allies in each others struggles, but they can also easily be manipulated into
connecting ideology that make movements successful.

fighting against each other if they do not develop a systemic analysis of global power. Scholars can be pitted against each other as
easily as aggrieved communities can. In an era of carefully orchestrated challenges to public education,
scholarly independence and critical thinking, it is likely in the near future that every
department, discipline and field will be encouraged to defend its own worth by belittling
others, to compete for scarce and declining resources by inflating its own achievements
at the expense of others . A losing proposition in politics, this race to the bottom would
be even more disastrous for scholarship because it encourages parochialism and
defensive localism at precisely the moment when we most need dialogue , generosity
and cosmopolitanism. It is important in this context to identify and learn from scholarly
works that offer models of principled and productive synthesis between American Studies and Area
Studies. Fortunately, both well established classics and promising new work in both American Studies and Area Studies contain this generative
potential. The scholarly works of W.E.B. Du Bois and Walter Rodney provide especially useful and generative models from the past, while recent
studies by Melani McAlister, Lise Waxer, Roderick Ferguson and Clyde Woods pose bold and exciting challenges in the present (Ferguson, 2004;
McAlister, 2001; Waxer, 2002; Woods, 1998).

AT: Anti-blackness link


Their use of the black-white binary accords no power to other non-white
groups to contest the racial landscape. This undermines their analysis of
race
Michael OMI, Associate Professor at UC Berkeley, AND Howard WINANT, Sociology at the
University of California, Santa Barbara, 13 [Resistance is futile?: a response to Feagin and
Elias, Ethnic and Racial Studies, Vol. 36, Issue 6, 2013]
it is important not to frame race in a bipolar manner. The black/white paradigm
made more sense in the past than it does in the twenty-first century. The racial make-up
of the nation has now changed dramatically. Since the passage of the Immigration Reform Act of 1965, the USA has become more coloured. A
majorityminority national demographic shift is well underway. Predicted to arrive by the mid-twenty-first century, the
numerical eclipse of the white population is already in evidence locally and regionally. In
In the USA today

California, for example, non-Hispanic whites constitute only 39.7 per cent of the state's population. While the decline in the white population cannot be correlated with any decline of white racial dominance,

the dawning and deepening of racial multipolarity calls into question a sometimes
implicit and sometimes explicit black/white racial framework that is evident in Feagin and Elias's essay. Shifting racial
demographics and identities also raise general questions of race and racism in new ways that the systemic racism approach is not prepared to explain.3 Class questions and
issues of panethnicizing trends, for example, call into question what we mean by race, racial
identity and race consciousness. No racially defined group is even remotely uniform;
groups that we so glibly refer to as Asian American or Latino are particularly
heterogeneous. Some have achieved or exceeded socio-economic parity with whites,
while others are subject to what we might call engineered poverty in sweatshops, dirty and dangerous labour settings, or prisons.
Tensions within panethnicized racial groups are notably present, and conflicts between
racially defined groups (black/brown conflict, for example) are evident in both urban and rural
settings. A substantial current of social scientific analysis now argues that Asians and Latinos
are the new white ethnics, able to work toward whiteness4 at least in part, and that the
black/white bipolarity retains its distinct and foundational qualities as the mainstay of US
racism (Alba and Nee 2005; Perlmann 2005; Portes and Rumbaut 2006; Waters, Ueda and Marrow 2007). We question that argument in light of
the massive demographic shifts taking place in the USA. Globalization, climate change and above all neoliberalism on a global scale, all drive
migration. The country's economic capacity to absorb enormous numbers of immigrants, lowwage workers and their families (including a new, globally based and very female, servant class) without generating the sort of
established subaltern groups we associate with the terms race and racism, may be more
limited than it was when the whitening of Europeans took place in the nineteenth and twentieth centuries. In other words this
argument's key precedent, the absorption of white immigrants of a different color (Jacobson
1998), may no longer apply. Indeed, we might think of the assimilationist model itself as a general theory of immigrant incorporation that was based on a historically specific case study
one that might not hold for, or be replicated by, subsequent big waves of immigration. Feagin and Elias's systemic racism model, while offering
numerous important insights, does not inform concrete analysis of these issues . It is important going forward to understand how groups are
differentially racialized and relatively positioned in the US racial hierarchy: once again racism must be seen as a shifting racial project. This
has important consequences , not only with respect to emerging patterns of inequality, but also in
regard to the degree of power available to different racial actors to define, shape or
contest the existing racial landscape . Attention to such matters is largely absent in Feagin and Elias's account. In their view racially
identified groups are located in strict reference to the dominant white racial frame,
hammered into place, so to speak. As a consequence, they fail to examine how racially subordinate groups

interact and influence each others boundaries, conditions and practices. Because they
offer so little specific analysis of Asian American, Latino or Native American racial
issues, the reader finds her/himself once again in the land (real or imaginary, depending on your racial politics) of bipolar US
racial dynamics, in which whites and blacks play the leading roles, and other racially
identified groups as well as those ambiguously identified, such as Middle Eastern and South Asian Americans (MEASA) play at best supporting roles, and
are sometimes cast as extras or left out of the picture entirely.

AT: Contingency Link


The 1ac combines radical critique and reform, holding them in productive
tension to remake common understandings surrounding incarceration
this creates medium-term steps towards complete abolition
Sudbury 8, Professor of Ethnic Studies
[2008, Julia Sudbury is Metz Professor of Ethnic Studies at Mills College. She is a leading
activist scholar in the prison abolitionist movement. She was a co-founder of Critical Resistance,
a national abolitionist organization. Rethinking Global Justice: Black Women Resist the
Transnational Prison-Industrial Complex, Souls: A Critical Journal of Black Politics, Culture, and
Society, Volume 10, Issue 4]
Chronic overcrowding has led to worsening conditions for prisoners. As a result of the unprecedented growth in sentenced
populations, prison authorities have packed three or four prisoners into cells designed for two, and have taken over recreation
rooms, gyms, and rooms designed for programming and turned them into cells, housing prisoners on bunk beds or on the floor.
These new conditions have created challenges for activists, who have found themselves expending time and resources in
pressuring prison authorities to provide every prisoner a bed, or to provide access to basic education programs. As prison
populations continue to swell, anti-prison

activists are faced with the limitations of reformist


strategies. Gains temporarily won are swiftly undermined, new women-centered prison regimes are
replaced with a focus on cost-efficiency and minimal programming and even changes enforced by legal cases like Shumate vs.
Wilson are subject to backlash and resistance. 19 Of

even greater concern is the well-documented


tendency of prison regimes to co-opt reforms and respond to demands for changes in
conditions by further expanding prison budgets. The vulnerability of prison reform
efforts to cooption has led Angela Y. Davis to call for non-reformist reforms, reforms
that do not lead to bigger and better prisons . 20 Despite the limited long-term impact of
human rights advocacy and reforms, building bridges between prisoners, activists, and
family members is an important step toward challenging the racialized dehumanization
that undergirds the logic of incarceration . In this way, human rights advocacy carried out in
solidarity with prisoner activists is an important component of a radical anti-prison
agenda . Ultimately, however, anti-prison activists aim not to create more humane, culturally sensitive, womento dismantle prisons and enable formerly criminalized people to access
services and resources outside the penal system. After three decades of prison expansion, more and more
centered prisons, but

people are living with criminal convictions and histories of incarceration. In the U.S., nearly 650,000 people are released from state
and federal prisons to the community each year. 21 Organizations of formerly incarcerated people focus on creating opportunities
for former prisoners to survive after release, and on eliminating barriers to reentry, including extensive discrimination against former
felons. The wide array of post-incarceration sentences that felons are subjected to has led activists to declare a new civil rights
movement. 22 As a class, former prisoners can legally be disenfranchised and denied rights available to other citizens. While
reentry has garnered official attention, with President Bush proposing a $300 million reentry initiative in his 2004 State of the Union
address, anti-prison activists have critiqued this initiative for focusing on faith-based mentoring, job training, and housing without
addressing the endemic discrimination against former prisoners or addressing the conditions in the communities which receive
former prisoners, including racism, poverty, and gender violence. Organizations of ex-prisoners working to oppose discrimination
against former prisoners and felons include All of Us Or None, the Nu Policy Leadership Group, Sister Outsider and the National
Network for Women Prisoners in the U.S., and Justice 4 Women in Canada. All of Us Or None is described by members as a
national organizing initiative of prisoners, former prisoners and felons, to combat the many forms of discrimination that we face as
the result of felony convictions. 23 Founded by anti-imperialist and former political prisoner Linda Evans, and former prisoner and
anti-prison activist Dorsey Nunn, and sponsored by the Northern Californiabased Legal Services for Prisoners with Children, All of
Us Or None works to mobilize former prisoners nationwide and in Toronto, Canada. The organization's name, from a poem by
Marxist playwright Bertold Brecht, invokes the need for solidarity across racial, class, and gender lines in creating a unified

movement of former prisoners. Black women play a leading role in the organization, alongside other people of color. All of Us Or
None focuses its lobbying and campaign work at city, county, and state levels, calling on local authorities to end discrimination
based on felony convictions in public housing, benefits, and employment, to opt out of lifetime welfare and food stamp bans for
felons, and to ban the box requiring disclosure of past convictions on applications for public employment. In addition, the
organization calls for guaranteed housing, job training, drug and alcohol treatment, and public assistance for all newly released
prisoners. 24 In the context of the war on drugs, many people with felony convictions also struggle with addictions. The recovery
movement, which is made up of 12-step programs, treatment programs, community recovery centers, and indigenous healing
programs run by and for people in recovery from addiction, offers an alternative response to problem drug use through programs
focusing on spirituality, healing, and fellowship. However, the recovery movement's focus on individual transformation and
accountability for past acts diverges from many anti-prison activists' focus on the harms done to criminalized communities by
interlocking systems of dominance. As a result, anti-prison spaces seldom engage with the recovery movement, or tap the radical
potential of its membership. Breaking with this trend, All of Us Or None has initiated a grassroots organizing effort to reach out to
people in 12-step programs with felony convictions. This work is part of their wider organizing efforts that aim to mobilize former
prisoners as agents of social change. Building on the strengths of identity politics, these organizations suggest that those who have
experienced the prison-industrial complex first-hand may be best placed to provide leadership in dismantling it. As former prisoners
have taken on a wide range of leadership positions across the movement, there has been a shift away from leadership by white
middle-class progressives, and a move to promote the voices of those directly affected by the prison-industrial complex. Politicians
who promote punitive tough-on-crime policies rely on racialized controlling images of the criminal to inspire fear and induce
compliance among voters. Once dehumanized and depicted as dangerous and beyond rehabilitation, removing people from
communities appears the only logical means of creating safety. Activists

who pursue decarceration challenge


stereotypical images of the criminal by making visible the human stories of prisoners,
with the goal of demonstrating the inadequacy of incarceration as a response to the
complex interaction of factors that produce harmful acts. Decarceration usually involves
targeting a specific prison population that the public sees as low-risk and arguing for an
end to the use of imprisonment for this population. Decarcerative strategies often
involve the promotion of alternatives to incarceration that are less expensive and more
effective than prison and jail. For example, Proposition 36, the Substance Abuse and Crime Prevention Act, which
passed in California in 2000 and allowed first- and second-time non-violent drug offenders charged with possession to receive
substance abuse treatment instead of prison, channels approximately 35,000 people into treatment annually. 25

Drug law

reform is a key area of decarcerative work . Organizations and campaigns that promote
drug law reform include Drop the Rock, a coalition of youth, former prisoners, criminal justice reformers, artists, civil and
labor leaders working to repeal New York's Rockefeller Drug Laws. The campaign combines racial justice,
economic, and public safety arguments by demonstrating that the laws have created a
pipeline of prisoners of color

from New York City to newly built prisons in rural, mainly white areas represented

Republican senators, resulting in a transfer of funding and electoral influence from communities of color to upstate rural
communities. 26 Ultimately, the campaign calls for an end to mandatory minimum sentencing and the reinstatement of judges'
sentencing discretion, a reduction in sentence lengths for drug-related offenses and the expansion of alternatives, including drug
treatment, job training, and education. Former drug war prisoners play a leadership role in decarcerative efforts in the field of drug
policy reform. Kemba Smith, an AfricanAmerican woman who was sentenced to serve 24.5 years as a result of her relationship
with an abusive partner who was involved in the drug industry, is one potent voice in opposition to the war on drugs. While she was
incarcerated, Smith became an active advocate for herself and other victims of the war on drugs, securing interviews and feature
articles in national media. Ultimately, Smith's case came to represent the failure of mandatory minimums, and in 2000, following a
nation-wide campaign, she and fellow drug war prisoner Dorothy Gaines were granted clemency by outgoing President Clinton.
After her release, Smith founded the Justice for People of Color Project (JPCP), which aims to empower young people of color to
participate in drug policy reform and to promote a reallocation of public expenditures from incarceration to education. While women
like Kemba Smith and Dorothy Gaines have become the human face of the drug war, prison invisibilizes and renders anonymous
hundreds of thousands of drug war prisoners. The organization Families Against Mandatory Minimums (FAMM) challenges this
process of erasure and dehumanization through its Faces of FAMM project. The project invites people in federal and state prisons
serving mandatory minimum sentences to submit their cases to a database and provides online access to their stories and
photographs. 27 The Faces of FAMM project highlights cases where sentencing injustices are particularly visible in order to
galvanize public support for sentencing reform. At the same time, it dismantles

popular representations of the


war on drugs as a necessary protection against dangerous drug dealers and traffickers,
demonstrating that most drug war prisoners are serving long sentences for low-level,
non-violent drug-related activities or for being intimately connected to someone involved in these activities.
Decarcerative work is not limited to drug law reform. Free Battered Women's (FBW) campaign for the release of incarcerated

survivors is another example of decarcerative work. The organization supports women and transgender prisoners incarcerated for
killing or assaulting an abuser in challenging their convictions by demonstrating that they acted in self-defense. Most recently, FBW
secured the release of Flozelle Woodmore, an AfricanAmerican woman serving a life sentence at CCWF for shooting her violent
partner as an 18 year old. Released in August 2007, after five parole board recommendations for her release were rejected by
Governors Davis and then Schwarzenegger, Woodmore's determined pursuit of justice made visible and ultimately challenged the
racialized politics of gubernatorial parole releases. 28 While the number of women imprisoned for killing or assaulting an abuser is
smallFBW submitted 34 petitions for clemency at its inception in 1991, and continues to fight 23 casesFBW's campaign for the
release of all incarcerated survivors challenges the mass incarceration of gender-oppressed prisoners on a far larger scale. FBW
argues that experiences of intimate partner violence and abuse contribute to the criminalized activities that lead many women and
transgender people into conflict with the law, including those imprisoned on drug or property charges, and calls for the release of all
incarcerated survivors. Starting with a population generally viewed with sympathysurvivors of intimate partner violenceFBW
generates a radical critique of both state and interpersonal violence, arguing that the violence and control used by the state against
people in prison mirrors the dynamics of battering that many incarcerated survivors have experienced in their intimate relationships
and/or as children. 29 In

theorizing the intersections of racialized state violence and gendered


interpersonal violence, FBW lays the groundwork for a broader abolitionist agenda that
refutes the legitimacy of incarceration as a response to deep-rooted social inequalities
based on interlocking systems of oppression. By gradually shrinking the prison system,
Black women activists involved in decarcerative work hope to erode the public's reliance on
the idea of imprisonment as a commonsense response to a wide range of social ills.

At the

other end of anti-expansionist work are activists who take a more confrontational approach. By starving correctional budgets of
funds to continue building more prisons and jails, they hope to force politicians to embrace less expensive and more effective
alternatives to incarceration. Prison moratorium organizing aims to stop construction of new prisons and jails. Unlike campaigns
against prison privatization, which oppose prison-profiteering by private corporations, and seek to return imprisonment to the public
sector, prison moratorium work opposes all new prison construction, public or private. In New York, the Brooklyn-based Prison
Moratorium Project (PMP), co-founded by former prisoner Eddie Ellis and led by young women and gender non-conforming people
of color, does this work through popular education and mass campaigns against prison expansion. Focusing on youth as a force for
social change, New York's PMP uses compilations of progressive hip hop and rap artists to spread a critical analysis of the prisonindustrial complex and its impact on people of color. PMP's strategies have been effective; for example, in 2002 the organization, as
part of the Justice 4 Youth Coalition, succeeded in lobbying the New York Department of Juvenile Justice to redirect $53 million
designated for expansion in Brooklyn and the Bronx. 30 PMP has also worked to make visible the connections between
underfunding, policing of schools, and youth incarceration through their campaign Stop the School-to-Prison Pipeline. By
demonstrating how zero tolerance policies and increased policing and use of surveillance technology in schools, combined with
underfunded classrooms and overstretched teachers, has led to the criminalization of young people of color and the production of
adult prisoners, PMP argues for a reprioritization of public spending from the criminal justice system to schools and alternatives to
incarceration. 31 Moratorium work often involves campaigns to prevent the construction of a specific prison or jail. In Toronto, for
example, the

Prisoner Justice Action Committee formed the 81 Reasons campaign, a


multiracial collaboration of experienced anti-prison activists, youth and student
organizers , in response to proposals to build a youth superjail in Brampton, a suburb of Toronto.
32 The campaign combined popular education on injustices in the juvenile system , including
the disproportionate incarceration of Black and Aboriginal youth, with an exercise in popular democracy that
invited young people to decide themselves how they would spend the $81 million slated
for the jail. Campaigners mobilized public concerns about spending cuts in other areas,
including health care and education, to create pressure on the provincial government to look into
less expensive and less punitive alternatives to incarceration for youth. While this campaign did
not ultimately prevent the construction of the youth jail, the size of the proposed facility was reduced. More importantly, the
campaign built a grassroots multiracial antiprison youth movement and raised public
awareness of the social and economic costs of incarceration. Moratorium campaigns face tough
opposition from advocates who believe that building prisons stimulates economic development for struggling rural towns. Prisons
are sold to rural towns that have suffered economic decline in the face of global competition, closures of local factories, and
decline of small farms. In the context of economic stagnation, prisons are touted as providing stable, well-paying, unionized jobs,
providing property and sales taxes and boosting real estate markets. The California Prison Moratorium Project has worked to
challenge these assertions by documenting the actual economic, environmental, and social impact of prison construction in
California's Central Valley prison towns. According to California PMP: We consider prisons to be a form of environmental injustice.
They are normally built in economically depressed communities that eagerly anticipate economic prosperity. Like any toxic industry,
prisons affect the quality of local schools, roads, water, air, land, and natural habitats. 33 California PMP opposes prison
construction at a local level by building multiracial coalitions of local residents, farm workers, labor organizers, anti-prison activists,

and former prisoners and their families to reject the visions of prison as a panacea for economic decline. 34 In the Californian
context, where most new prisons are built in predominantly Latino/a communities and absorb land and water previously used for
agriculture, PMP facilitates communication and solidarity between Latino/a farm worker communities, and urban Black and Latino/a
prisoners in promoting alternative forms of economic development that do not rely on mass incarceration. Scholar-activist Ruth
Wilson Gilmore's research on the political economy of prisons in California has been critical in providing evidence of the detrimental
impact of prisons on local residents and the environment. 35 As an active member of CPMP, Gilmore's work is deeply rooted in antiprison activism and in turn informs the work of other activists, demonstrating the important relationship between Black women's
activist scholarship and the anti-prison movement. 36 Many

anti-prison activists view campaigns for

decarceration or moratorium as building blocks toward the ultimate goal of abolition .


These practical actions promise short and medium-term successes that are essential
markers on the road to long-term transformation . However, abolitionists believe that like slavery, the prisoncontemporary prison
abolitionist movement in the U.S. and Canada dates to the 1970s, when political
prisoners like Angela Y. Davis and Assata Shakur, in conjunction with other radical
activists and scholars in the U.S., Canada, and Europe, began to call for the dismantling
of prisons. 38 The explosion in political prisoners, fuelled by the FBI's Counter Intelligence Program (COINTELPRO) and
industrial complex is a system of racialized state violence that cannot be fixed. The

targeting of Black liberation, American Indian and Puerto Rican independence movements in the U.S. and First Nations resistance in
Canada as threats to national security, fed into an understanding of the role of the prison in perpetuating state repression against
insurgent communities. 39 The new anti-prison politics were also shaped by a decade of prisoner litigation and radical prison
uprisings, including the brutally crushed Attica Rebellion. These

common prisoners, predominantly working-class


the state's legitimacy by declaring
imprisonment a form of cruel and unusual punishment and confronting the brute force of
state power. 40 By adopting the term abolition activists drew deliberate links between the
dismantling of prisons and the abolition of slavery. Through historical excavations, the new abolitionists
people of color imprisoned for everyday acts of survival, challenged

identified the abolition of prisons as the logical completion of the unfinished liberation marked by the 13th Amendment to the United
States Constitution, which regulated, rather than ended, slavery. 41 Organizations that actively promote dialogue about what
abolition means and how it can translate into concrete action include Critical Resistance (CR), New York's Prison Moratorium
Project, Justice Now, California Coalition for Women Prisoners, Free Battered Women, and the Prison Activist Resource Center in
the U.S. and the Prisoner Justice Action Committee (Toronto), the Prisoners' Justice Day Committee (Vancouver) and Joint Action in
Canada. CR was founded in 1998 by a group of Bay Area activists including former political prisoner and scholar-activist Angela Y.
Davis. Initially, CR focused on popular education and movement building, coordinating large conferences where diverse
organizations could generate collective alternatives to the prison-industrial complex. Later work has included campaigns against
prison construction in California's Central Valley and solidarity work with imprisoned Katrina survivors. CR describes abolition as: [A]
political vision that seeks to eliminate the need for prisons, policing, and surveillance by creating sustainable alternatives to
punishment and imprisonment . An

abolitionist vision means that we must build models today


that can represent how we want to live in the future. It means developing practical
strategies for taking small steps that move us toward making our dreams real and that
lead the average person to believe that things really could be different . It means living
this vision in our daily lives . 42 In this sense, prison abolitionists are tasked with a dual
burden: first, transforming people's consciousness so that they can believe that a world
without prisons is possible, and second, taking practical steps to oppose the prisonindustrial complex . Making abolition more than a utopian vision requires practical steps
toward this long-term goal . CR describes four steps that activists can get involved in:
shrinking the system, creating alternatives, shifting public opinion and public policy, and
building leadership among those directly impacted by the prison-industrial complex. 43 Since its inception in the San Francisco Bay
Area, Critical Resistance has become a national organization with chapters in Baltimore, Chicago, Gainesville, Los Angeles, New
Orleans, New York, Tampa/St. Petersburg, and Washington, D.C. As such, CR has played a critical role in re-invigorating abolitionist
politics in the U.S. This work is rooted in the radical praxis of Black women and transgender activists.

Neolib

War Turns Structural Violence


War turns structural violence
Bulloch 8
Millennium - Journal of International Studies May 2008 vol. 36 no. 3 575-595
Douglas Bulloch, IR Department, London School of Economics and Political Science.
He is currently completing his PhD in International Relations at the London School of
Economics, during which time he spent a year editing Millennium: Journal of International
Studies
But the idea that poverty and peace are directly related presupposes that wealth
inequalities are in and of themselves unjust, and that the solution to the problem of war
is to alleviate the injustice that inspires conflict, namely poverty. However, it also suggests that
poverty is a legitimate inspiration for violence, otherwise there would be no reason to alleviate it
in the interests of peace. It has become such a commonplace to suggest that poverty and
conflict are linked that it rarely suffers any examination. To suggest that war causes poverty
is to utter an obvious truth, but to suggest the opposite is on reflection quite hard to
believe. War is an expensive business in the twenty-first century, even asymmetrically. And
just to examine Bangladesh for a moment is enough at least to raise the question concerning
the actual connection between peace and poverty. The government of Bangladesh is a threat
only to itself, and despite 30 years of the Grameen Bank, Bangladesh remains in a state of
incipient civil strife. So although Muhammad Yunus should be applauded for his work in
demonstrating the efficacy of micro-credit strategies in a context of development, it is not at all
clear that this has anything to do with resolving the social and political crisis in Bangladesh, nor
is it clear that this has anything to do with resolving the problem of peace and war in our
times. It does speak to the Western liberal mindset as Geir Lundestad acknowledges but
then perhaps this exposes the extent to which the Peace Prize itself has simply become an
award that reflects a degree of Western liberal wish-fulfilment. It is perhaps comforting to
believe that poverty causes violence, as it serves to endorse a particular kind of concern for
the developing world that in turn regards all problems as fundamentally economic rather
than deeply and potentially radically political.

Neolib Sustainable
Neolib is sustainable and the alt is a useless political abstraction
Arvidsson 13
(Adam, teaches sociology at the University of MilanoThinking beyond neo-liberalism: A
response to Detlev Zwick, ephemera: theory & politics in organization 13(2): 407-412)
We can of course still criticize the actual state of things

This makes it trickier to do critical theory.


. We can point to the
precarious relations that prevail among creative knowledge workers; show how exploitative and unjust conditions are intensified by the very forces that drive the globalization of
communicative capitalism, like the outsourcing of design work; or lament the fact that a triumphant neoliberal regime subsumes and appropriates aspects of subjectivity and
social life that we think should have been left alone. To produce such critiques remains useful intellectual work I have done it in other contexts (Arvidsson et al., 2010;
Arvidsson, 2007), as has Detlev Zwick (2008), and many others. To the extent that such critiques reach a mass audience, they can become a progressive impulse to action and

such a
critique without an alternative remains unsatisfactory for at least three reasons. First, and most superficially, since
everyone else is doing it, the marginal utility of yet another piece of critical theory rapidly
diminishes, as does the intellectual satisfaction that can be derived form producing it. Second, and more
seriously, the absence of a realistic alt ernative, or even of a historical subject in the name of
reflection as in the case of Naomi Kleins work inspiring the no global movement (to use an inadequate name coined by the mainstream press). But

which such a critique can be pronounced, risks rendering critical theory moralistic and
rather toothless . We might agree with Zwick when he suggests that the outsourcing of design work from Toronto to the
Philippines

is somehow wrong, but it is difficult to understand exactly why this would be the case.

(Why shouldnt Philippine designers be allowed to compete with Canadian designers? Can the creative class claim an exemption from the global economy? Perhaps the

in the absence of
an alternative vision, critical theory remains rather unconvincing to the people in the
name of whom it proposes to speak. I can assure you and Ive tried! that you wont become an organic
intellectual among social entrepreneurs or precarious creative workers by telling them
that they are exploited , that they sell out their subjectivity , or that the system in which
answer is yes, but I do not know of any viable alternative vision of society that is able to substantiate that yes.) Third, and most importantly,

they operate is unjust . Pure critique is simply not attractive enough to make the
multitude of new productive subjects, fragmented by neoliberalism, cohere into a
historical subject. To do that you need at least the myth of an alternative , as agitators from Sorel via Lenin
to Subcomandante Marcos could tell you. Dont get me wrong. I am not proposing that it is wrong to point to the precarious conditions of knowledge work, or that
we should not do this as academics and researchers. This is still an important task. But it is not enough . Critical
theory must do this, but it must also do more . It must also engage with the question of
what a realistic alternative to neoliberalism could be, and it must elaborate a realistic
political vision in the name of which a critique that is productive and progressive, and
not simply moralistic, can be articulated. By realistic, I mean that such an alternative must be sought
in the actual relations of production that characterize the contemporary information
economy. Zwicks suggestion that we imagine a commonism of productive consumption
as collaborative sharing in the absence of private property and combined with an inclusive model of political determination, collective sovereignty,
belonging and justice and so on is simply unproductive to my mind. We might all agree that an economy of
commons that has done away with capitalism might be more desirable, but the reality is
that hybrid forms, like the game modders that Zwick cites, where a an economy of commons co-exists with a capitalist value logic, in some form, are indeed
becoming the norm. At that point the interesting thing to do is not so much to criticize the
enduring capitalist nature of these hybrid forms, but rather to investigate the new forms

of politics that they might give rise to. This in no way implies that one does away with conflict and politics. Rather, it implies
investigating and understanding the new spaces and discourses through which such a
new type of politics can be articulated. In order to do this we must start with what the actors
involved in these processes actually think themselves. It is quite useless to simply
deploy existing philosophical perspectives, or to compare the reality of communicative capitalism to
utopian projections of the political visions of last century. Instead we must start with the empirical metaphysics, to use
Bruno Latours term, that actually prevail among people engaged in such hybrid practices. We might all want to do away with
neoliberalism and the forms of life that it has promoted. But at the same time, we all recognize
that the neoliberal project has been one of the most successful projects of
governmentality since, perhaps, the very project of disciplinary power that Foucault himself described. Rebus sic stantibus we cannot simply
wish it away . We need to recognize that people have changed, that competitive
individualism, self-branding and an entrepreneurial mentality are, by now, normal features of life. The same thing goes for the popular
political myths that prevail among advanced knowledge workers, what Zwick calls cyber-utopianism. We need to recognize that notions like peer-to-peer production, high-tech
gift economies and the like have the power to mobilize the energies of the subjects that are most likely to become the pioneers of a new political vision todays version of the
skilled workers that have taken the lead in most modern political movements. Even though the social theory that they produce might be shallow and imperfect, and even though
they might not have read Marx and Foucault as well as we have, we cannot simply dismiss this vision as a mere ideology to be replaced by our theoretically more refined
ideology. Like the relations of production that are emerging in communicative capitalism and the subjectivity of knowledge workers, these myths are part of the raw material with
which the Gramscian intellectual must engage in order to articulate new understandings of common sense that are both politically progressive and intuitively attractive to the

In other words, in order to articulate an alternative, we cannot


simply dismiss the reality of communicative capitalism and fall back on what remains of
the political utopias of last century. We need to engage with the reality of neoliberal
communicative capitalism and try to push its dialectic beyond its apolitical present state. We must investigate what the
real conditions of production and imagination are and ask ourselves where they might
lead. Critical theory needs to become an empirical , and not simply a philosophical,
enterprise.
people that they are supposed to mobilize.

Permutation Stuff

Reform Perm
Perm do the plan and all non-mutually exclusive parts of the alt --- policy
analysis key
James Ferguson 11, Professor of Anthropology at Stanford, The Uses of Neoliberalism,
Antipode, Vol. 41, No. S1, pp 166184
If we are seeking, as this special issue of Antipode aspires to do, to link our critical analyses to the world of grounded
political strugglenot only to interpret the world in various ways, but also to change it
then there is much to be said for focusing, as I have here, on mundane, real- world debates around
policy and politics, even if doing so inevitably puts us on the compromised and reformist
terrain of the possible, rather than the seductive high ground of revolutionary ideals and
utopian desires. But I would also insist that there is more at stake in the examples I have discussed here than simply a
slightly better way to ameliorate the miseries of the chronically poor, or a technically
superior method for relieving the suffering of famine victims. My point in discussing the South African BIG campaign, for instance, is not
really to argue for its implementation. There is much in the campaign that is appealing, to be sure. But one can just as easily identify a series of worries that would bring the
whole proposal into doubt. Does not, for instance, the decoupling of the question of assistance from the issue of labor, and the associated valorization of the informal, help
provide a kind of alibi for the failures of the South African regime to pursue policies that would do more to create jobs? Would not the creation of a basic income benefit tied to
national citizenship simply exacerbate the vicious xenophobia that already divides the South African poor, in a context where many of the poorest are not citizens, and would
thus not be eligible for the BIG? Perhaps even more fundamentally, is the idea of basic income really capable of commanding the mass support that alone could make it a
central pillar of a new approach to distribution? The record to date gives powerful reasons to doubt it. So far, the technocrats dreams of relieving poverty through efficient cash
transfers have attracted little support from actual poor people, who seem to find that vision a bit pale and washed out, compared with the vivid (if vague) populist promises of
jobs and personalistic social inclusion long offered by the ANC patronage machine, and lately personified by Jacob Zuma (Ferguson forthcoming). My real interest in the policy
proposals discussed here, in fact, has little to do with the narrow policy questions to which they seek to provide answers. For what is most significant, for my purposes, is not

specific governmental devices and


modes of reasoning that we have become used to associating with a very particular (and
conservative) political agenda (neoliberalism) may be in the process of being peeled away from
that agenda, and put to very different uses. Any progressive who takes seriously the
challenge I pointed to at the start of this essay, the challenge of developing new progressive arts of government,
ought to find this turn of events of considerable interest. As Steven Collier (2005) has recently pointed out, it is
important to question the assumption that there is, or must be, a neat or automatic fit between a
hegemonic neoliberal political-economic project (however that might be characterized),
on the one hand, and specific neoliberal techniques, on the other. Close attention to
particular techniques (such as the use of quantitative calculation, free choice, and price driven by supply and
demand) in particular settings (in Colliers case, fiscal and budgetary reform in post-Soviet Russia) shows that the relationship
between the technical and the political-economic is much more polymorphous and
unstable than is assumed in much critical geographical work, and that neoliberal technical mechanisms
are in fact deployed in relation to diverse political projects and social norms (2005:2). As I
suggested in referencing the role of statistics and techniques for pooling risk in the creation of social democratic welfare states, social technologies need
not have any essential or eternal loyalty to the political formations within which they
were first developed. Insurance rationality at the end of the nineteenth century had no essential vocation to provide security and solidarity to the working
class; it was turned to that purpose (in some substantial measure) because it was available, in the right place at the right time, to be appropriated for that use. Specific
ways of solving or posing governmental problems, specific institutional and intellectual
mechanisms, can be combined in an almost infinite variety of ways, to accomplish
different social ends. With social, as with any other sort of technology, it is not the
machines or the mechanisms that decide what they will be used to do. Foucault (2008:94) concluded his
whether or not these are good policies, but the way that they illustrate a process through which

discussion of socialist government- ality by insisting that the answers to the Lefts governmental problems require not yet another search through our sacred texts, but a process
of conceptual and institutional innovation. [I]f there is a really socialist governmentality, then it is not hidden within socialism and its texts. It cannot be deduced from them. It
must be invented. But invention in the domain of governmental technique is rarely something worked up out of whole cloth. More often, it involves a kind of bricolage (Le viStrauss 1966), a piecing together of something new out of scavenged parts originally intended for some other purpose. As we pursue such a process of improvisatory invention,
we might begin by making an inventory of the parts available for such tinkering, keeping all the while an open mind about how different mechanisms might be put to work, and
what kinds of purposes they might serve.

If we can go beyond seeing in neoliberalism an evil essence or an

and instead learn to see a field of specific governmental techniques, we may be


surprised to find that some of them can be repurposed, and put to work in the service of
political projects very different from those usually associated with that word. If so, we
may find that the cabinet of governmental arts available to us is a bit less bare than first
appeared, and that some rather useful little mechanisms may be nearer to hand than we
thought.
automatic unity,

2AC Perm (Barry)


Only the perm solvesproposing alternative non-capitalist economics out
of nowhere is of zero value. Environmental reform of capitalism is key.
John BARRY Reader in Politics @ Belfast 7 [Towards a model of green political economy:
from ecological modernisation to economic security Int. J. Green Economics, Vol. 1, Nos. 3/4,
2007 p. 447-448]
Economic analysis has been one of the weakest and least developed areas of broadly green/sustainable
development thinking. For example, whatever analysis there is within the green political canon is
largely utopian usually based on an argument for the complete transformation of
modern society and economy as the only way to deal with ecological catastrophe, an often
linked to a critique of the socioeconomic failings of capitalism that echoed a broadly
radical Marxist/socialist or anarchist analysis; or underdeveloped due, in part, to the need to outline and develop
other aspects of green political theory. However, this gap within green thinking has recently been filled by a number of scholars, activists, think tanks,
and environmental NGOs who have outlined various models of green political economy to underpin sustainable development political aims, principles

The aim of this article is to offer a draft of a realistic, but critical, version of
green political economy to underpin the economic dimensions of radical views about sustainable development. It is written explicitly
with a view to encouraging others to think through this aspect of sustainable development in a collaborative manner. Combined realism
and radicalism marks this article, which starts with the point that we cannot build or seek
to create a sustainable economy ab nihlo, but must begin from where we are, with the
structures, institutions, modes of production, laws and regulations that we already have.
Of course, this does not mean simply accepting these as immutable or set in stone; after all, some of the
current institutions, principles and structures underpinning the dominant economic model are the very causes of unsustainable development. We
do need to recognise, however, that we must work with (and through in the terms of the original
German Green Partys slogan of marching through the institutions) these existing structures, as well as change and reform and in
and objectives.

some cases, abandon them as either unnecessary or positively harmful to the creation and maintenance of a sustainable economy and society.

an alternative economy and society must be based in the reality


that most people (in the West) will not democratically vote for a completely different type
of society and economy. That reality must also accept that a green economy is one that
is recognisable to most people and that indeed safeguards and guarantees not just their
basic needs but also aspirations (within limits). The realistic character of the thinking
behind this article accepts that consumption and materialistic lifestyles are here to stay (so
long as they do not transgress any of the critical thresholds of the triple bottom line) and indeed there is little to be gained by
proposing alternative economic systems, which start from a complete rejection of
consumption and materialism. The appeal to realism is in part an attempt to correct the
common misperception (and self-perception) of green politics and economics requiring
an excessive degree of self-denial and a puritanical asceticism (Goodin, 1992, p.18; Allison, 1991, p.170
Equally, this article also recognises that

178). While rejecting the claim that green political theory calls for the complete disavowal of materialistic lifestyles, it is true that green politics does
require the collective reassessment of such lifestyles, and does require a degree of shared sacrifice. It does not mean, however, that we necessarily

There must be room and tolerance in a


green economy for people to live ungreen lives so long as they do not harm others,
threaten long-term ecological sustainability or create unjust levels of socioeconomic
inequalities. Thus, realism in this context is in part another name for the acceptance of a broadly
liberal or post-liberal (but certainly not anti-liberal) green perspective .1
require the complete and across-the-board rejection of materialistic lifestyles.

Perm Stuff
Only the perm solvesproposing alternative non-capitalist economics out
of nowhere is of zero value. Environmental reform of capitalism is key.
John BARRY Reader in Politics @ Belfast 7
[Towards a model of green political economy: from ecological modernisation to economic
security Int. J. Green Economics, Vol. 1, Nos. 3/4, 2007 p. 447-448]
Economic analysis has been one of the weakest and least developed areas of broadly green/sustainable
development thinking. For example, whatever

analysis there is within the green political canon is


largely utopian usually based on an argument for the complete transformation of
modern society and economy as the only way to deal with ecological catastrophe, an often
linked to a critique of the socioeconomic failings of capitalism that echoed a broadly
radical Marxist/socialist or anarchist analysis; or underdeveloped due, in part, to the need to outline and develop
other aspects of green political theory. However, this gap within green thinking has recently been filled by a number of scholars, activists, think tanks,
and environmental NGOs who have outlined various models of green political economy to underpin sustainable development political aims, principles

The aim of this article is to offer a draft of a realistic, but critical, version of
green political economy to underpin the economic dimensions of radical views about sustainable development. It is written explicitly
with a view to encouraging others to think through this aspect of sustainable development in a collaborative manner. Combined realism
and radicalism marks this article, which starts with the point that we cannot build or seek
to create a sustainable economy ab nihlo, but must begin from where we are, with the
structures, institutions, modes of production, laws and regulations that we already have.
Of course, this does not mean simply accepting these as immutable or set in stone; after all, some of the
current institutions, principles and structures underpinning the dominant economic model are the very causes of unsustainable development. We
do need to recognise, however, that we must work with (and through in the terms of the original
German Green Partys slogan of marching through the institutions) these existing structures, as well as change and reform and in
and objectives.

some cases, abandon them as either unnecessary or positively harmful to the creation and maintenance of a sustainable economy and society.

an alternative economy and society must be based in the reality


that most people (in the West) will not democratically vote for a completely different type
of society and economy. That reality must also accept that a green economy is one that
is recognisable to most people and that indeed safeguards and guarantees not just their
basic needs but also aspirations (within limits). The realistic character of the thinking
behind this article accepts that consumption and materialistic lifestyles are here to stay (so
long as they do not transgress any of the critical thresholds of the triple bottom line) and indeed there is little to be gained by
proposing alternative economic systems, which start from a complete rejection of
consumption and materialism. The appeal to realism is in part an attempt to correct the
common misperception (and self-perception) of green politics and economics requiring
an excessive degree of self-denial and a puritanical asceticism (Goodin, 1992, p.18; Allison, 1991, p.170
Equally, this article also recognises that

178). While rejecting the claim that green political theory calls for the complete disavowal of materialistic lifestyles, it is true that green politics does
require the collective reassessment of such lifestyles, and does require a degree of shared sacrifice. It does not mean, however, that we necessarily

There must be room and tolerance in a


green economy for people to live ungreen lives so long as they do not harm others,
threaten long-term ecological sustainability or create unjust levels of socioeconomic
inequalities. Thus, realism in this context is in part another name for the acceptance of a broadly
liberal or post-liberal (but certainly not anti-liberal) green perspective .1
require the complete and across-the-board rejection of materialistic lifestyles.

AT: Growth Link


Case is a pre-requisite to changes in the direction of consumption. Without
growth and minimizing conflict resources get devoted to competition.
Rasmus KARLSSON Poli Sci @ Lund 9 A global Fordian compromise?And what it would
mean for the transition to sustainability Envtl Science and Policy 12 p. 190-191
Though these caricatures may still hold true to a certain extent, I would argue that the last years have challenged this impasse. On one hand, the general public has grown

promising academic work, both in the sciences


(Hoffert et al., 2002) and in green political theory (Nordhaus and Shellenberger, 2007), has finally taken up what otherwise has been
a dormant position ever since the 1970s. I am referring to those who accept the gravity of the present
environmental crisis yet believe that the solution can never be found in the traditional
green mantra of reduction , conservation and self-denial . Instead these authors have
increasingly aware of how serious our current predicament has become. On the other hand, a string of

attempted to reconcile the politics of scarcity with technological optimism , to tap into the spirit that
once made grand projects like the Apollo program possible and, on this basis, move towards a politics of radical engagement. Nowhere
does the need for such new politics appear more urgent than on the global level. With parts of the world (mostly in Asia) rapidly
industrializing while others remaining trapped in the direst poverty, the planetary perspective goes to the heart of
the sustainable transition. Not only does it show the terrible human cost of the present status-quo but also the irreversibility in the move towards
modernization. Billions are now impatiently aspiring for the material living standard of the West,
and given the limited ecological space of the planet (Andreasson, 2005; Rist, 1997, pp. 4445), it is hard to see how these needs can be
met without radical technological innovation. However, there are reasons to doubt the feasibility of any advanced technological
paths to sustainability. Only in a climate of high and sustained economic growth would it be
possible for states to set aside the vast resources necessary to bring success to longterm projects on nuclear fusion, nanotechnology and other converging technologies (Malsch, 2008). Such benign
economic conditions are, just as the prospects of sustainable development more generally (Blinc et al., 2007), unlikely to come
about in times of international tension, unplanned mass-migration and growing resource
scarcity. This should warrant a new kind of sobering realism, an acceptance that the future of modernity is now a planetary enterprise and that we are all into this as one
common human civilization.

Perm: Movements Reinforcing


Top-down and bottom-up movements are mutually reinforcing but the alt
alone causes worse forms of trade and global wars
Brecher 2k
Jeremy Brecher is a historian and the author of numerous books on labor and social
movements, co-founded Global Labor Strategies, Tim Costello is one of Australia's most
recognised voices on social justice, leadership and ethics, Brendan Smith, social activist, cofounder of Global Labor Strategies, Globalization from Below, 2000, google books
The apparent opposition among strengthening local, national, and global institutions is
based on a false premise: that more power at one level of governance is necessarily
disempowering to people at others. But today the exact opposite is the case . The
empowerment of local and national communities and polities today requires a degree of
global regulation and governance. Far from being dichotomous, they are interdependent.
Globalization from below requires a framework that recognizes that interdependence. Global
capital has usurped powers that rightfully belong to people and to their representatives in
government. The challenge is for people to establish greater control over their economic
lives by establishing greater control over global capital. To do so requires a stronger
governmental role and increased organization in civil society at every level from local to global.
These various levels of action far from being in competition, can actually be mutually
supportive . Indeed, the programs needed at each level are unlikely to work for long
unless complemented by supportive policies and structures at the other levels. Consider
for example the issue of local empowerment. In the United States, ironically, local
empowerment has been major theme of that part of the right that is most favorable to
global capitalism. Newt Gingrich, as conservative Republican speaker of the US House of
Representatives, campaigned to eliminate federal regulations that "interfered" with states
and municipalities. Of course, the real purpose of this faux localism was to break down
regulations that establish minimum wages and labor standards, preserve the environment,
reduce poverty, and protect the rights of women, oppressed minorities, the disabled, and trade
unions. "Empowering local communities" meant putting them at the mercy of powerful
corporations and financiers. It also meant pitting them against each other in a race to the
bottom. Clearly, national regulation that protects minimum standards and limits the race to
the bottom actually strengthens, rather than undermines, the ability of people in local
communities to control their own lives. The same can be true globally . While the rules and
policies currently imposed by the IMF, World Bank, and WTO empower corporations and
disempowers people at every level, that doesn't mean that different global institutions and
policies couldn't have the opposite effect. For example, international agreements that reduce
gases that destroy the ozone layer and produce global warming can protect local communities
and countries against environmental destruction. Global economic institutions that regulate
global demand and international currency flows would strengthen the ability of national
governments to regulate their own economies. Environmental and labor conditions in

international trade agreements could put limits on the race to the bottom. So could trade union
agreements with corporations specifying minimum conditions worldwide. Enforceable
international code of conduct for corporations could reduce their ability to dictate conditions to
countries and communities. In fact, such higher-level regulation is essential if the forces of
globalization are to be tamed. This point is missed by those who see globalization simply
as a matter of greater power for international economic institutions. Globalization was
not caused (even though it was accelerated) by the WTO or NAFTA. And globalization will
continue even if such institutions are abolished. Indeed, the likely result of completely
eliminating supranational economic regulation is not genuine national self-determination
but an economic war of all-against-all . As each nation pursues policies of economic
nationalism, others will follow suit in escalating trade wars . And without means for
regulating demand, global depression is more than likely to follow.

Impact Turns

Trade good

Top-down and bottom-up movements are mutually reinforcing but the alt
alone causes worse forms of trade and global wars
Brecher 2k
Jeremy Brecher is a historian and the author of numerous books on labor and social
movements, co-founded Global Labor Strategies, Tim Costello is one of Australia's most
recognised voices on social justice, leadership and ethics, Brendan Smith, social activist, cofounder of Global Labor Strategies, Globalization from Below, 2000, google books

The apparent opposition among strengthening local, national, and global institutions is
based on a false premise: that more power at one level of governance is necessarily
disempowering to people at others. But today the exact opposite is the case . The
empowerment of local and national communities and polities today requires a degree of
global regulation and governance. Far from being dichotomous, they are interdependent.
Globalization from below requires a framework that recognizes that interdependence. Global
capital has usurped powers that rightfully belong to people and to their representatives in
government. The challenge is for people to establish greater control over their economic
lives by establishing greater control over global capital. To do so requires a stronger
governmental role and increased organization in civil society at every level from local to global.
These various levels of action far from being in competition, can actually be mutually
supportive . Indeed, the programs needed at each level are unlikely to work for long
unless complemented by supportive policies and structures at the other levels. Consider
for example the issue of local empowerment. In the United States, ironically, local
empowerment has been major theme of that part of the right that is most favorable to
global capitalism. Newt Gingrich, as conservative Republican speaker of the US House of
Representatives, campaigned to eliminate federal regulations that "interfered" with states
and municipalities. Of course, the real purpose of this faux localism was to break down
regulations that establish minimum wages and labor standards, preserve the environment,
reduce poverty, and protect the rights of women, oppressed minorities, the disabled, and trade
unions. "Empowering local communities" meant putting them at the mercy of powerful
corporations and financiers. It also meant pitting them against each other in a race to the
bottom. Clearly, national regulation that protects minimum standards and limits the race to
the bottom actually strengthens, rather than undermines, the ability of people in local
communities to control their own lives. The same can be true globally . While the rules and
policies currently imposed by the IMF, World Bank, and WTO empower corporations and
disempowers people at every level, that doesn't mean that different global institutions and
policies couldn't have the opposite effect. For example, international agreements that reduce
gases that destroy the ozone layer and produce global warming can protect local communities
and countries against environmental destruction. Global economic institutions that regulate
global demand and international currency flows would strengthen the ability of national

governments to regulate their own economies. Environmental and labor conditions in


international trade agreements could put limits on the race to the bottom. So could trade union
agreements with corporations specifying minimum conditions worldwide. Enforceable
international code of conduct for corporations could reduce their ability to dictate conditions to
countries and communities. In fact, such higher-level regulation is essential if the forces of
globalization are to be tamed. This point is missed by those who see globalization simply
as a matter of greater power for international economic institutions. Globalization was
not caused (even though it was accelerated) by the WTO or NAFTA. And globalization will
continue even if such institutions are abolished. Indeed, the likely result of completely
eliminating supranational economic regulation is not genuine national self-determination
but an economic war of all-against-all . As each nation pursues policies of economic
nationalism, others will follow suit in escalating trade wars . And without means for
regulating demand, global depression is more than likely to follow.

Liberal Norms Good


Shocks to the system are the ONLY propensity for conflictliberal norms
have eradicated warfare and structural violenceevery field study proves
Horgan 09
JOHN HORGAN is Director of the Center for Science at Stevens Institute of Technology, former
senior writer at Scientific American, B.A. from Columbia and an M.S. from Columbia,
Newsweek, The End of the Age of War, Dec 7, 2009,
http://www.newsweek.com/id/225616/page/1
The economic crisis was supposed to increase violence around the world. The truth is that we are
now living in one of the most peaceful periods since war first arose 10 or 12 millennia ago.
The relative calm of our era, say scientists who study warfare in history and even prehistory, belies the popular,
pessimistic notion that war is so deeply rooted in our nature that we can never abolish it. In fact, war
seems to be a largely cultural phenomenon, which culture is now helping us eradicate.
Some scholars now even cautiously speculate that the era of traditional warfought by two uniformed, state-sponsored armiesmight be drawing to a
close. "War

could be on the verge of ceasing to exist as a substantial phenomenon," says John Mueller, a
political scientist at Ohio State University. That might sound crazy, but consider: if war is defined as a conflict
between two or more nations resulting in at least 1,000 deaths in a year, there have been
no wars since the U.S. invasion of Iraq in 2003 and no wars between major industrialized
powers since World War II. Civil wars have also declined from their peak in the early
1990s, when fighting tore apart Rwanda, the Balkans, and other regions. Most armed conflicts now consist of low-level guerrilla campaigns,
insurgencies, and terrorismwhat Mueller calls the "remnants of war." These facts would provide little comfort if war's remnants were nonetheless

Recent studies reveal a clear downward trend. In 2008, 25,600


combatants and civilians were killed as a direct result of armed conflicts, according to the University
of Uppsala Conflict Data Program in Sweden. Two thirds of these deaths took place in just three trouble
spots: Sri Lanka (8,400), Afghanistan (4,600), and Iraq (4,000). Uppsala's figures exclude deaths from "onesided conflict," in which combatants deliberately kill unarmed civilians, and "indirect"
deaths from war-related disease and famine, but even when these casualties are
included, annual war-related deaths from 2004 to 2007 are still low by historical standards. Acts
of terrorism, like the 9/11 attacks or the 2004 bombing of Spanish trains, account for less than 1 percent of
fatalities. In contrast, car accidents kill more than 1 million people a year. The contrast between our century and
the previous one is striking. In the second half of the 20th century, war killed as many as 40 million people, both directly and
killing millions of peoplebut they're not.

indirectly, or 800,000 people a year, according to Milton Leitenberg of the University of Maryland. He estimates that 190 million people, or 3.8 million a
year, died as a result of wars and state--sponsored genocides during the cataclysmic first half of the century. Considered as a percentage of
population, the body count of the 20th century is comparable to that of blood-soaked earlier cultures, such as the Aztecs, the Romans, and the
Greeks. By far the most warlike societies are those that preceded civilization. War killed as many as 25 percent of all pre-state people, a rate 10 times

Our ancestors were not


always so bellicose, however: there is virtually no clear-cut evidence of lethal group
aggression by humans prior to 12,000 years ago. Then, "warfare appeared in the evolutionary trajectory of an
higher than that of the 20th century, estimates anthropologist Lawrence Keeley of the University of Illinois.

increasing number of societies around the world," says anthropologist Jonathan Haas of Chicago's Field Museum of Natural History. He attributes the
emergence of warfare to several factors: growing population density, environmental stresses that diminished food sources, and the separation of
people into culturally distinct groups. "It is only after the cultural foundations have been laid for distinguishing 'us' from 'them,' " he says, "that raiding,
killing, and burning appear as a complex response to the external stress of environmental problems." Early civilizations, such as those founded in
Mesopotamia and Egypt 6,000 years ago, were extremely warlike. They assembled large armies and began inventing new techniques and

nation-states also developed laws and


institutions for resolving disputes nonviolently, at least within their borders. These cultural innovations helped reduce
technologies for killing, from horse-drawn chariots and catapults to bombs. But

the endless, tit-for-tat feuding that plagued pre-state societies. A

host of other cultural factors may explain the


more recent drop-off in international war and other forms of social violence. One is a
surge in democratic rather than totalitarian governance. Over the past two centuries democracies such as the
U.S. have rarely if ever fought each other. Democracy is also associated with low levels of
violence within nations. Only 20 democratic nations existed at the end of World War II; the number has since more than quadrupled.
Yale historian Bruce Russett contends that international institutions such as the United Nations and the European Union also contribute to this
"democratic peace" phenomenon by fostering economic interdependence. Advances in civil rights for women may also be making us more peaceful.

As women's education and economic opportunities rise, birthrates fall, decreasing demands on
governmental and medical services and depletion of natural resources, which can otherwise lead to
social unrest. Better public health is another contributing factor. Over the past century, average life spans have almost
doubled, which could make us less willing to risk our lives by engaging in war and other
forms of violence, proposes Harvard psychologist Steven Pinker. At the same time, he points out,
globalization and communications have made us increasingly interdependent on, and empathetic
toward, others outside of our immediate "tribes." Of course, the world remains a dangerous place,
vulnerable to disruptive, unpredictable events like terrorist attacks. Other looming threats to peace include climate
change, which could produce droughts and endanger our food supplies; overpopulation; and the spread of violent religious extremism, as embodied by
Al Qaeda. A global financial meltdown or ecological catastrophe could plunge us back into the kind of violent, Hobbesian chaos that plagued many
pre--state societies thousands of years ago. "War is not intrinsic to human nature, but neither is peace," warns the political scientist Nils Petter
Gleditsch of the International Peace Research Institute in Oslo. So

far the trends are positive. If they continue, who


knows? World peacethe dream of countless visionaries and -beauty--pageant -contestantsor something like it may finally
come to pass.

Devp/Globalization Good
Anything else leads to system degradation a system of trade that results in
inevitable conflict
Thompson 02
[Grahame, Professor of Political Economy at the Open University of Britain, Wither the
Washington Consensus, the Developmental State, and the Seattle Protests, can Managed
Free Trade and Investment become an Alternative Development Model, Vol. 33, no. 131, x-xii,
2002 Problemas del Desarrollo]
Thus although the anti-globalization movement could mount a critique of both these
models, its main weakness in the development debate is its failure to construct a more
sensible, attractive and realistic alternative vision for development than provided by the
two models outlined so far. The anti-globalization movement remains essentially negative .
It provides a worthy and sometimes telling critique of many of the excesses of the international
economy but that is about it (Thompson 2003b). It does not have a positive alternative model
of development. In fact the anti-globalization movement is a disparate amalgam of many
mutually contradictory elements and positions. It can 'unite' the extreme Marxist left with
the far Right, along with disaffected environmentalist, human rights campaigners and debtforgivers in between. This is not a recipe for a coherent alternative position. In fact, as far
as any consistent position from this quarter can be discerned it amounts to little more
than the call for a 'new protectionism' . The desire to break up the international agencies
of economic management like the IMF, The World Bank and the WTO, would inevitably
lead to this result . Whilst these agencies have their pernicious aspects and are not to be
simply endorsed in their present form, they at least have the virtue of preventing a
devastating destruction of international trade and investment, and the likely formation of
antagonistically poised trade blocks as arose in the inter-War period.

Impact Defense

Env Up
Environment improving globally
Bailey 14
Ronald Bailey is a science correspondent at Reason magazine and author of Liberation Biology,
Reason Magazine, August 1, 2014, Predictions of a Man-Made Sixth Mass Extinction May Be
Exaggerated, http://reason.com/archives/2014/08/01/predictions-of-a-man-made-sixth-massext
countervailing trends suggest the future of animal species may not be so grim. These
trends include urbanization , slowing deforestation , the global increase in protected areas ,
Meanwhile, some

and the advent of peak farmland . Transforming hundreds of millions of hardscrabble subsistence
farmers scattered across the landscape into urban dwellers would greatly reduce the
pressure on wildlife and their habitats. The good news is that that process is happening. According to the United
Nations Population Division's World Urbanization Prospects 2014 report, 54 percent of people live in citiesup from 30
percent in 1950. The U.N. demographers forecast that proportion will rise to 66 percent by 2050. If world population
follows the medium fertility trend, rising to 9.5 billion by 2050, that means that 3.2 billion people will still live in rural areas, down slightly from 3.4 billion today. In that scenario,

population pressure will not be pushing the expansion of the human frontier into more wildlands. And
if population growth traverses along the U.N.'s low fertility trend, the year 2050 will see human numbers peaking at 8.3 billion people. If 66 percent live in cities, only 2.8 billion
people will be living on the landscape. Looking further into the future, the U.N. expects that 80 percent of people will be urbanites by 2100. Medium and low fertility population
projections suggest that would mean that only 2.2 to 1.3 billion rural dwellers. Since most species live in forests, chiefly tropical forests, we should take a look at global forest

deforestation rate is slowing . The Food and Agriculture Organization's State of the World's Forests 2012 report notes that the
global rate of deforestation slowed from 0.2 percent per year between 1990 and 2000 to 0.14 percent
between 2005 and 2010. Between 2000 and 2010, a total of 130 million hectares were cut,
but 78 million hectares returned to forests. So globally, forests declined on average by 5.2 million hectares per yearat which
rate, the report notes, "It will take 775 years to lose all of the world's forests." It adds, "This would seem to
provide enough time for actions to slow or stop global deforestation." And indeed, researchers in 2006 found that more
and more countries are passing through a "forest transition" in which their forest area starts
expanding . Roger Sedjo, a forest ecologist at Resources for the Future, predicts that by 2050 most of the world's
industrial wood will be grown on forest plantations covering only 5 to 10 percent of the extent of today's
global forests. One dark blot on forest restoration trends is biofuel mandates in rich countries, which have spurred tropical countries to chop down forests to grow
cover trends. Happily, the

palm oil to produce biodiesel. By one estimate, 87 percent of the deforestation in Malaysia and 118 percent in Indonesia occurred as result of expanding palm oil plantations.
Another beneficial trend is that

protected areas such as parks and marine preserves are expanding at a

remarkably fast pace . The World Bank notes that protected areas have nearly doubled from
8.5 percent of the world's total land area in 1990 to 14.3 percent in 2012. That's an area
twice the size of the entire U nited S tates. Marine protected areas have increased from 4.7 percent of territorial
waters in 1990 to 10 percent in 2012. Under the Convention on Biological Diversity, governments of the world
have committed to protecting 17 percent of terrestrial and inland water areas and 10 per cent of
coastal and marine areas by 2020. Considering that agriculture is the most expansive and intensive way
in which people transform natural landscapes, the really good news is that the amount of land
globally devoted to food production may be falling as population growth slows and
agricultural productivity increases. "We believe that projecting conservative values for
population, affluence, consumers, and technology shows humanity peaking in the use of

farmland ," concludes Jesse Ausubel, the director of the Human Environment Program at the
Rockefeller University. In a 2013 article titled "Peak Farmland and the Prospect for Land Sparing," Ausubel and his colleagues write: "Global arable land
and permanent crops spanned 1,371 million hectares in 1961 and 1,533 million hectares in 2009, and we project a return to 1,385 million hectares in 2060." As a
result of these trends, humanity will likely restore at least 146 million hectares of land, an area two
and a half times that of France, or ten Iowas and possibly much more . Relaxing those biofuel mandates would spare an additional 256
million hectares from the plow, the researchers estimate. That mean nearly 400 million hectaresan area nearly double the size of the United States east of the Mississippi

The late 20th century's predictions of imminent mass extinction


happily proved wrong . The positive trends cited above provide good grounds to believe that

Rivercould be restored to nature by 2060.

the

new ones will also turn out to be exaggerated .

AT: Sustainability Impacts


Sustainability impacts are bad science no basis for running into tech
barriers
Matt Ridley, visiting professor at Cold Spring Harbor Laboratory, former science editor of The
Economist, and award-winning science writer, 14
(The World's Resources Aren't Running Out, April 25, 2014,
http://online.wsj.com/news/articles/SB10001424052702304279904579517862612287156?
mg=reno64-wsj&url=http%3A%2F%2Fonline.wsj.com%2Farticle
%2FSB10001424052702304279904579517862612287156.html)
"We are using 50% more resources than the Earth can sustainably produce, and unless we change course, that number will grow fastby 2030, even
two planets will not be enough," says Jim Leape, director general of the World Wide Fund for Nature International (formerly the World Wildlife Fund).

We burst through such limits again and again . After all, as a Saudi oil
minister once said, the Stone Age didn't end for lack of stone. Ecologists call this "niche
construction"that people (and indeed some other animals) can create new opportunities for themselves by
making their habitats more productive in some way. Agriculture is the classic example of niche construction: We stopped
relying on nature's bounty and substituted an artificial and much larger bounty. Economists call the same phenomenon
innovation. What frustrates them about ecologists is the latter's tendency to think in terms of static limits.
Ecologists can't seem to see that when whale oil starts to run out, petroleum is discovered,
or that when farm yields flatten, fertilizer comes along, or that when glass fiber is invented, demand for copper falls.
That frustration is heartily reciprocated. Ecologists think that economists espouse a sort
of superstitious magic called "markets" or "prices" to avoid confronting the reality of limits to growth. The easiest way to raise a cheer in
a conference of ecologists is to make a rude joke about economists. I have lived among both tribes. I studied various
forms of ecology in an academic setting for seven years and then worked at the Economist magazine for
eight years. When I was an ecologist (in the academic sense of the word, not the political one, though I also had antinuclear stickers on my
But here's a peculiar feature of human history:

car), I very much espoused the carrying-capacity viewpointthat there were limits to growth. I nowadays lean to the view that there are no limits

This disagreement goes to the heart of many current


political issues and explains much about why people disagree about environmental policy. In the climate debate, for
example, pessimists see a limit to the atmosphere's capacity to cope with extra carbon dioxide without rapid
warming. So a continuing increase in emissions if economic growth continues will eventually accelerate warming to dangerous rates. But
optimists see economic growth leading to technological change that would result in the
use of lower-carbon energy. That would allow warming to level off long before it does
much harm. It is striking, for example, that the Intergovernmental Panel on Climate Change's
recent forecast that temperatures would rise by 3.7 to 4.8 degrees Celsius compared with preindustrial levels by
2100 was based on several assumptions: little technological change, an end to the 50-year fall in population growth
rates, a tripling (only) of per capita income and not much improvement in the energy
efficiency of the economy. Basically, that would mean a world much like today's but with lots more people burning lots more coal and
oil, leading to an increase in emissions. Most economists expect a five- or tenfold increase in income, huge
because we can invent new ways of doing more with less.

changes in technology and an end to population growth by 2100: not so many more people needing much less carbon. In 1679, Antonie van

Leeuwenhoek, the great Dutch microscopist, estimated that the planet could hold 13.4
billion people, a number that most demographers think we may never reach. Since then, estimates have bounced around between 1 billion
and 100 billion, with no sign of converging on an agreed figure. Economists point out that we keep improving the productivity
of each acre of land by applying fertilizer, mechanization, pesticides and irrigation. Further
innovation is bound to shift the ceiling upward. Jesse Ausubel at Rockefeller University calculates that the amount of land required to grow a given

Ecologists object that these innovations rely on


nonrenewable resources, such as oil and gas, or renewable ones that are being used up
faster than they are replenished, such as aquifers. So current yields cannot be maintained, let alone improved. In his recent book
quantity of food has fallen by 65% over the past 50 years, world-wide.

"The View from Lazy Point," the ecologist Carl Safina estimates that if everybody had the living standards of Americans, we would need 2.5 Earths
because the world's agricultural land just couldn't grow enough food for more than 2.5 billion people at that level of consumption. Harvard emeritus
professor E.O. Wilson, one of ecology's patriarchs, reckoned that only if we all turned vegetarian could the world's farms grow enough food to support
10 billion people. Economists respond by saying that since large parts of the world, especially in Africa, have yet to gain access to fertilizer and modern
farming techniques, there is no reason to think that the global land requirements for a given amount of food will cease shrinking any time soon. Indeed,
Mr. Ausubel, together with his colleagues Iddo Wernick and Paul Waggoner, came to the startling conclusion that, even with generous assumptions
about population growth and growing affluence leading to greater demand for meat and other luxuries, and with ungenerous assumptions about future
global yield improvements, we will need less farmland in 2050 than we needed in 2000. (So long, that is, as we don't grow more biofuels on land that
could be growing food.) But surely intensification of yields depends on inputs that may run out? Take
water, a commodity that limits the production of food in many places. Estimates made in the 1960s and 1970s of water demand by the year 2000
proved grossly overestimated: The world used half as much water as experts had projected 30 years before.

The reason was greater

economy in the use of water by new irrigation techniques. Some countries, such as Israel and Cyprus, have cut
water use for irrigation through the use of drip irrigation . Combine these improvements with solar-driven
desalination of seawater world-wide, and it is highly unlikely that fresh water will limit
human population. The best-selling book "Limits to Growth," published in 1972 by the
Club of Rome (an influential global think tank), argued that we would have bumped our heads against
all sorts of ceilings by now, running short of various metals, fuels, minerals and space.
Why did it not happen? In a word, technology: better mining techniques, more frugal use
of materials, and if scarcity causes price increases, substitution by cheaper material. We use 100 times
thinner gold plating on computer connectors than we did 40 years ago. The steel content
of cars and buildings keeps on falling. Until about 10 years ago, it was reasonable to expect that
natural gas might run out in a few short decades and oil soon thereafter. If that were to happen, agricultural yields
would plummet, and the world would be faced with a stark dilemma: Plow up all the remaining rain forest to grow food, or starve. But thanks
to fracking and the shale revolution, peak oil and gas have been postponed. They will run
out one day, but only in the sense that you will run out of Atlantic Ocean one day if you
take a rowboat west out of a harbor in Ireland . Just as you are likely to stop rowing long
before you bump into Newfoundland, so we may well find cheap substitutes for fossil
fuels long before they run out.

AT: Sustainability Impacts


Sustainability impacts are bad science no basis for running into tech
barriers
Matt Ridley, visiting professor at Cold Spring Harbor Laboratory, former science editor of The
Economist, and award-winning science writer, 14
(The World's Resources Aren't Running Out, April 25, 2014,
http://online.wsj.com/news/articles/SB10001424052702304279904579517862612287156?
mg=reno64-wsj&url=http%3A%2F%2Fonline.wsj.com%2Farticle
%2FSB10001424052702304279904579517862612287156.html)
"We are using 50% more resources than the Earth can sustainably produce, and unless we change course, that number will grow fastby 2030, even two planets will not be
enough," says Jim Leape, director general of the World Wide Fund for Nature International (formerly the World Wildlife Fund). But here's a peculiar feature of human history:

We burst through such limits again and again . After all, as a Saudi oil minister once said, the Stone Age didn't
end for lack of stone. Ecologists call this "niche construction"that people (and indeed some other animals)
can create new opportunities for themselves by making their habitats more productive in some way.
Agriculture is the classic example of niche construction: We stopped relying on nature's bounty and substituted an artificial and much larger bounty. Economists
call the same phenomenon innovation. What frustrates them about ecologists is the latter's tendency to think in terms
of static limits. Ecologists can't seem to see that when whale oil starts to run out, petroleum
is discovered, or that when farm yields flatten, fertilizer comes along, or that when glass fiber is invented, demand
for copper falls. That frustration is heartily reciprocated . Ecologists think that economists
espouse a sort of superstitious magic called "markets" or "prices" to avoid confronting the reality of limits to growth. The easiest way to
raise a cheer in a conference of ecologists is to make a rude joke about economists. I have lived among both tribes. I studied
various forms of ecology in an academic setting for seven years and then worked at the Economist magazine for
eight years. When I was an ecologist (in the academic sense of the word, not the political one, though I also had antinuclear stickers on my car), I very much
espoused the carrying-capacity viewpointthat there were limits to growth. I nowadays lean to the view that there are no limits because we can invent new ways of doing more

This disagreement goes to the heart of many current political issues and explains much about why
In the climate debate, for example, pessimists see a limit to the
atmosphere's capacity to cope with extra carbon dioxide without rapid warming. So a continuing increase in emissions if economic growth continues will
eventually accelerate warming to dangerous rates. But optimists see economic growth leading to technological change that
would result in the use of lower-carbon energy. That would allow warming to level off long before it does much harm. It is
striking, for example, that the Intergovernmental Panel on Climate Change's recent forecast that
temperatures would rise by 3.7 to 4.8 degrees Celsius compared with preindustrial levels by 2100 was based on several assumptions: little
technological change, an end to the 50-year fall in population growth rates, a tripling (only) of per
capita income and not much improvement in the energy efficiency of the economy. Basically,
that would mean a world much like today's but with lots more people burning lots more coal and oil, leading to an increase in emissions. Most economists
expect a five- or tenfold increase in income, huge changes in technology and an end to population growth by 2100: not so many more
people needing much less carbon. In 1679, Antonie van Leeuwenhoek, the great Dutch microscopist, estimated that
the planet could hold 13.4 billion people, a number that most demographers think we may never reach. Since then, estimates have
bounced around between 1 billion and 100 billion, with no sign of converging on an agreed figure. Economists point out that we keep improving the
productivity of each acre of land by applying fertilizer, mechanization, pesticides and
irrigation. Further innovation is bound to shift the ceiling upward. Jesse Ausubel at Rockefeller University calculates that the amount of land required to grow a given
quantity of food has fallen by 65% over the past 50 years, world-wide. Ecologists object that these innovations rely on
nonrenewable resources, such as oil and gas, or renewable ones that are being used up
with less.

people disagree about environmental policy.

faster than they are replenished, such as aquifers. So current yields cannot be maintained, let alone improved. In his recent book "The View from
Lazy Point," the ecologist Carl Safina estimates that if everybody had the living standards of Americans, we would need 2.5 Earths because the world's agricultural land just
couldn't grow enough food for more than 2.5 billion people at that level of consumption. Harvard emeritus professor E.O. Wilson, one of ecology's patriarchs, reckoned that only
if we all turned vegetarian could the world's farms grow enough food to support 10 billion people. Economists respond by saying that since large parts of the world, especially in
Africa, have yet to gain access to fertilizer and modern farming techniques, there is no reason to think that the global land requirements for a given amount of food will cease
shrinking any time soon. Indeed, Mr. Ausubel, together with his colleagues Iddo Wernick and Paul Waggoner, came to the startling conclusion that, even with generous
assumptions about population growth and growing affluence leading to greater demand for meat and other luxuries, and with ungenerous assumptions about future global yield

But
surely intensification of yields depends on inputs that may run out? Take water, a commodity that limits the
improvements, we will need less farmland in 2050 than we needed in 2000. (So long, that is, as we don't grow more biofuels on land that could be growing food.)

production of food in many places. Estimates made in the 1960s and 1970s of water demand by the year 2000 proved grossly overestimated: The world used half as much

The reason was greater economy in the use of water by new


irrigation techniques. Some countries, such as Israel and Cyprus, have cut water use for irrigation through the use of drip irrigation. Combine
these improvements with solar-driven desalination of seawater world-wide, and it is
highly unlikely that fresh water will limit human population. The best-selling book "Limits
to Growth," published in 1972 by the Club of Rome (an influential global think tank), argued that we would
have bumped our heads against all sorts of ceilings by now, running short of various
metals, fuels, minerals and space. Why did it not happen? In a word, technology: better
mining techniques, more frugal use of materials, and if scarcity causes price increases, substitution by
cheaper material. We use 100 times thinner gold plating on computer connectors than we
did 40 years ago. The steel content of cars and buildings keeps on falling. Until about 10 years ago, it
was reasonable to expect that natural gas might run out in a few short decades and oil soon
water as experts had projected 30 years before.

thereafter. If that were to happen, agricultural yields would plummet, and the world would be faced with a stark dilemma: Plow up all the remaining rain forest to grow food, or

But thanks to fracking and the shale revolution, peak oil and gas have been
postponed. They will run out one day, but only in the sense that you will run out of
Atlantic Ocean one day if you take a rowboat west out of a harbor in Ireland . Just as you
are likely to stop rowing long before you bump into Newfoundland, so we may well find
cheap substitutes for fossil fuels long before they run out.
starve.

Alts orientation away from WTO is impacted in the 1ac with trade wars and GATS
collapse their politics leads to apathy cant transition from theory to political
change
John Gibson, Postdoctoral Fellow, University of Kent, 08
(The Myth of the Multitude: The Endogenous Demise of Alter-globalist Politics, Global Society,
Vol. 22, No. 2, April, 2008)
Crucially, such assessments view the rhizomatic structures and reflexive capacities of alter-globalist agents as also negating the internal antagonism and factionalism often
associated with social movements. Hardt understands the World Social Forum as fulfilling the promise of anti-hierarchical politicsan infinitesimally open-ended collection of

Gill has
argued that alter-globalist practices counter long-standing differences between new
and old social movements, creating a radically fluid postmodern Prince in the process.17 Similarly,
Jackie Smith has argued that the demand for democracy, functioning as an open signifier for the
desire for greater personal control over biopolitics, is the sole commonality in otherwise
plural alter-globalist spaces.18 Such claims resemble Laclau and Mouffes model of a Logic of Equivalence, in which a vast collection of particular
struggles converging around a basic commitment to resist neoliberalism, in which ideological differences cease to produce significant schisms.16 Stephen

social movements and political demands resonate in accordance with one another through the use of a central empty signifier (in this case, democracy) as an open point for
infinite re-inscriptions, and whose ontological closure is permanently resisted.19 In such interpretations, the pronoun we in alterglobalist contexts is devoid of any fixed

Unity is structured
around a shared rejection of transnational neoliberal capitalism, facilitating cross-cultural forms of identification
and epistemic exchange, resisting any fixed meaning of the collective subject that undertakes activism. The central dilemma for activist life
becomes whether the many yeses arising in its wake can exert counter-hegemonic
subjective content and is radically democratic. The political moment is defined within such discourses by Kingsnorths one no.

power in a long-term war of position.20 However, although such accounts appear to present
sources of optimism that dominant technocratic and individualist frames of global politics can be countered successfully, there are
reasons to doubt their claims. After reaching an early apogee of 250,000 protesters at the
Genoa G8 meeting in 2001, in the wake of 11 September 2001 summit protests have declined significantly, both in
terms of frequency and attendees. The protest in Miami against the Free Trade Area of the Americas (FTAA) agreement in
November 2003, advertised within movement circles as an event that would exceed the importance of Seattle, drew a crowd of below 10,000,
and failed to generate any significant media coverage.21 The 2004 G8 summit in Georgia was met by hundreds, rather than
thousands of protesters, and planned solidarity actions elsewhere failed to materialise.22 Likewise, the 2004 London May Day gathering was cancelled after poorly attended

direct action
protests conducted around the Gleneagles G8 summit were undertaken by only a few
thousand, and controversial protest tactics, including acts of vandalism in Auchterarder, generated considerable internal schisms.24 Social Forums have also become
beset by difficulties. Intra-movement concerns reflect a growing atrophy ; the absence of a venue for the prospective fifth
European Social Forum reflects a scenario in which there is no more cooperation between our forces than there was before Florence [in 2002].25 To compound
matters, some activists have complained that such events are becoming dominated by
resource-heavy NGOs.26 The World Social Forum has become frequented by political elites. Gerhard Schroder and Jacques Chirac both
attended the 2003 event, transgressing its Charter of Principles, in which the WSF exists
in opposition to a process of globalisation commanded by the large multinational
corporations and by the governments and international institutions at the service of those corporations interests, with the complicity of national governments.27
Activists often assert that the current impasse is temporary, that as new ways of pushing
forward are imagined there will be a corresponding resurgence of antineoliberal politics.
However, rather than the current lull representing a temporary period of torpor for alter-globalist politics, a
planning meetings, and in 2005 fewer than two dozen self-declared anti-capitalists played cricket on the lawn south of the House of Commons.23 The

number of entrenched weaknesses within the dominant myths of alter-globalist


networks stymie its capacity to embody radical democracy . Caution is required against
the hubris evident in the prevailing accounts of a fluid subject, conscious of its counterhegemonic potential as a source of the dissolution of the neoliberal world order. Such mythology is ridden with
inherent deficiencies that are rarely addressed by participants. Rather than a global
multitude taking shape, in many respects we see only the phantasm of such a polis , and one
that depends upon the active repression of contradictions and flaws in order to maintain
the idealised image of the multitude presented by such prominent movement commentators as Klein and Kingsnorth.

Corporate influence is exaggerated

Jones 04
(Kent Jones is Professor of Economics at Babson College, Whos Afraid of the WTO?, 2004, pg.
174-5)
Yet domestic political channels in the MNCs
home country are still the best way to raise the issues and exert influence. In contrast, trying to
attack MNCs by attacking the WTO misses the target. For one thing, MNC influence on the WTO
For their most severe critics, such measures will hardly be enough.6

has often been exaggerated . The largest of the MNCs certainly wield a great deal of political
influence, but mainly through domestic channels. The first is through lobbying efforts in their home countries, where they
play a large role in setting national agendas for trade negotiations and also have access to unilateral trade protection through antidumping and
countervailing duty laws.7 The second channel lies in their relationships with the governments of the countries hosting MNC foreign operations. It is

many countries, and indeed localities even within the developed countries, have been courting
FDI for the economic benefits it brings. Most governments have come to view FDI as an
important to keep in mind that

overwhelmingly positive contribution to the recipient countrys economic development and growth. This view is based
on the underlying economics of international capital mobility and related activities. Such bilateral relationships always carry
the danger of abuse, as special deals may be struck to bypass taxes or regulations, for
example. In addition, many governments, especially in developing countries, want to retain as much control as possible over the terms of FDI, which is
why they also have a strong interest in TRIMs. Yet this issue is

being sorted out in the WTO as a matter of


negotiation among sovereign states, not as the result of MNCs dictating the rules to
helpless governments. Within the WTO, the MNCs can indeed exert strong influence, but it is
typically tempered through the mediating factor of governmental representation. In the years before
the conclusion of the Uruguay Round, U.S. multinationals (and other U.S. firms), for example, couldand didlobby their home government directly
for unilateral action under various forms of section 301 of U.S. trade law. Together, these provisions allowed unilateral actions by the United States to
pry open foreign markets and to retaliate against any foreign trade practices deemed unreasonable.8 By bringing investment and intellectual
property issues into the WTO through the TRIMs and TRIPs agreements, and by strengthening the dispute settlement system, trade policy issues of

Under the WTO system, only governments have


standing, and a government must therefore decide whether to bring a case before the
D ispute S ettlement B ody, based on national interests and an associated assessment of trade-offs in commercial diplomacy.
concern to MNCs were channeled into a multilateral forum.

This does not eliminate the influence of MNCs on the trading system; active lobbying; aggressive, unfair trade-law enforcement; and accommodating

the elimination of unilateral measures tends to


moderate the influence of the MNCs and of domestic business in general on trade policy.
One sign of the restraint on political influence is that many MNCs advocate WTO reform that
would allow them to file dispute cases directly, without governmental intermediaries (see
Julius 1994). Such proposals cast serious doubt on claims that MNCs somehow control the
WTO or dictate trade rules or dispute settlement decisions. In addition, MNC activities are subject to
increased public scrutiny, providing another channel of influence for NGOs. In general, stakeholders in
domestic trade officials still give them considerable clout. Yet

trade issues, whether they are NGOs, MNCs, or other interest groups, have their most direct influence on trade policy through their national political
institutions. For

those concerned abut MNC political power, the best institutional arrangement
is one in which governments can negotiate agreements and represent the broader
interests of their domestic constituents in the WTO, keeping all interest groups involved
at the domestic level of the policy debate. Other channels of influence for nontrade issues can also be developed in new international
institutions.

Impact causality is backwards---violence is significantly declining---be skeptical


of their doomsaying
Steven Pinker, Johnstone Family professor of psychology at Harvard and the author of The
Better Angels of Our Nature: Why Violence Has Declined, and Andrew Mack, fellow at the
One Earth Future Foundation and director of the Human Security Report Project at Simon
Fraser University, The World Is Not Falling Apart, 12/22/14,
http://www.slate.com/articles/news_and_politics/foreigners/2014/12/the_world_is_not_falling_ap
art_the_trend_lines_reveal_an_increasingly_peaceful.single.html

The world is not falling apart. The kinds of violence to which most people are vulnerable
homicide, rape, battering, child abusehave been in steady decline in most of the world.
Autocracy is giving way to democracy. Wars between statesby far the most destructive
of all conflictsare all but obsolete . The increase in the number and deadliness of civil
wars since 2010 is circumscribed, puny in comparison with the decline that preceded it,
and unlikely to escalate. We have been told of impending doom before: a Soviet invasion

of Western Europe, a line of dominoes in Southeast Asia, revanchism in a reunified


Germany, a rising sun in Japan, cities overrun by teenage superpredators, a coming
anarchy that would fracture the major nation-states, and weekly 9/11-scale attacks that
would pose an existential threat to civilization. Why is the world always more dangerous
than it has ever beeneven as a greater and greater majority of humanity lives in peace
and dies of old age? Too much of our impression of the world comes from a misleading
formula of journalistic narration. Reporters give lavish coverage to gun bursts, explosions,
and viral videos, oblivious to how representative they are and apparently innocent of the
fact that many were contrived as journalist bait. Then come sound bites from experts
with vested interests in maximizing the impression of mayhem: generals, politicians,
security officials, moral activists. The talking heads on cable news filibuster about the event,
desperately hoping to avoid dead air. Newspaper columnists instruct their readers on
what emotions to feel. There is a better way to understand the world. Commentators can
brush up their historynot by rummaging through Bartletts for a quote from Clausewitz, but
by recounting the events of the recent past that put the events of the present in an
intelligible context. And they could consult the analyses of quantitative datasets on violence
that are now just a few clicks away. An evidence-based mindset on the state of the world
would bring many benefits. It would calibrate our national and international responses to
the magnitude of the dangers that face us. It would limit the influence of terrorists, school
shooters, decapitation cinematographers, and other violence impresarios. It might even dispel
foreboding and embody, again, the hope of the world.

AT: Neoliberalism/Cap Impact


Reject their lens of neoliberalism. Starting with neoliberalism encourages
fake radicalism, oversimplification, and greater levels of cooptation than
positive and pragmatic politics.
-Ad hoc policies of neoliberalism also originate from Leftist movements for greater autonomy
-Sustainability politics also emerged during this time, but the neolib K ignores those and lumps
them all together
-Ignores positive action that doesnt conform to a romantic view of rebellion (i.e. the plan)
Clive BARNETT Faculty of the Social Sciences @ Open University (UK) 5 [The
Consolations of Neoliberalism Geoforum 36 (1) p. Science Direct]
The blind-spot in theories of neoliberalismwhether neo-Marxist and Foucauldian
comes with trying to account for how top-down initiatives take in everyday situations. So
perhaps the best thing to do is to stop thinking of neoliberalism as a coherent
3. There is no such thing as neoliberalism!

hegemonic project altogether . For all its apparent critical force, the vocabulary of
neoliberalism and neoliberalization in fact provides a double consolation for leftist academics: it
supplies us with plentiful opportunities for unveiling the real workings of hegemonic
ideologies in a characteristic gesture of revelation; and in so doing, it invites us to align our own professional roles
with the activities of various actors out there, who are always framed as engaging in
resistance or contestation. The conceptualization of neoliberalism as a hegemonic project does not need refining by adding a splash of Foucault. Perhaps we
should try to do without the concept of neoliberalism altogether, because it might actually compound rather than aid in the task of figuring out how the world works and how it

, between an overly economistic derivation of political economy and an


overly statist rendition of governmentality, stories about neoliberalism manage to
reduce the understanding of social relations to a residual effect of hegemonic projects
and/or governmental programmes of rule (see Clarke, 2004a). Stories about neoliberalism pay
little attention to the pro-active role of socio-cultural processes in provoking changes in
modes of governance, policy, and regulation. Consider the example of the restructuring of public services such as health care, education, and criminal justice in
changes. One reason for this is that

the UK over the last two or three decades. This can easily be thought of in terms of a hegemonic project of neoliberalization, and certainly one dimension of this process has

these ongoing changes in


the terms of public-policy debate involve a combination of different factors that add up to
a much more dispersed populist reorientation in policy, politics, and culture. These factors include
been a form of anti-statism that has rhetorically contrasted market provision against the rigidities of the state. But in fact

changing consumer expectations, involving shifts in expectations towards public entitlements which follow from the generalization of consumerism; the decline of deference,
involving shifts in conventions and hierarchies of taste, trust, access, and expertise; and the refusals of the subordinated, referring to the emergence of anti-paternalist attitudes
found in, for example, womens health movements or anti-psychiatry movements. They include also the development of the politics of difference, involving the emergence of
discourses of institutional discrimination based on gender, sexuality, race, and disability. This has disrupted the ways in which welfare agencies think about inequality, helping to
generate the emergence of contested inequalities, in which policies aimed at addressing inequalities of class and income develop an ever more expansive dynamic of
expectation that public services should address other kinds of inequality as well (see Clarke, 2004b J. Clark, Dissolving the public realm? The logics and limits of neo-liberalism,
Journal of Social Policy 33 (2004), pp. 2748.Clarke, 2004b). None of these populist tendencies is simply an expression of a singular hegemonic project of neoliberalization.

what we have
come to recognise as hegemonic neoliberalism is a muddled set of ad hoc,
opportunistic accommodations to these unstable dynamics of social change as it is to think of it as
the outcome of highly coherent political-ideological projects. Processes of privatization, market liberalization , and deThey are effects of much longer rhythms of socio-cultural change that emanate from the bottom-up. It seems just as plausible to suppose that

regulation have often followed an ironic pattern in so far as they have been triggered by
citizens movements arguing from the left of the political spectrum against the rigidities

of statist forms of social policy and welfare provision in the name of greater autonomy,
equality, and participation (e.g. Horwitz, 1989). The political re-alignments of the last three or four
decades cannot therefore be adequately understood in terms of a straightforward shift
from the left to the right, from values of collectivism to values of individualism, or as a re-imposition
of class power. The emergence and generalization of this populist ethos has much longer, deeper, and wider roots than those ascribed to hegemonic neoliberalism. And it also
points towards the extent to which easily the most widely resonant political rationality in the world today is not right-wing market liberalism at all, but is, rather, the polyvalent

Recent theories of neoliberalism have retreated from the


appreciation of the long-term rhythms of socio-cultural change, which Stuart Hall once developed in his influential
account of Thatcherism as a variant of authoritarian populism. Instead, they favour elite-focused analyses of state
bureaucracies, policy networks, and the like. One consequence of the residualization of the social is that theories of
neoliberalism have great difficulty accounting for, or indeed even in recognizing, new forms of
individualized collective-action (Marchetti, 2003) that have emerged in tandem with the apparent ascendancy of
neoliberal hegemony: environmental politics and the politics of sustainability ; new forms of
consumer activism oriented by an ethics of assistance and global solidarity; the identity politics of
sexuality related to demands for changes in modes of health care provision, and so on (see Norris, 2002). All of these might be thought of as variants of what we might
discourse of democracy (see Barnett and Low, 2004).

want to call bottom-up governmentality. This refers to the notion that non-state and non-corporate actors are also engaged in trying to govern various fields of activity, both by
acting on the conduct and contexts of ordinary everyday life, but also by acting on the conduct of state and corporate actors as well. Rose (1999, pp. 281284) hints at the

.
Instead, the trouble-free amalgamation of Foucaults ideas into the Marxist narrative of
neoliberalism sets up a simplistic image of the world divided between the forces of
hegemony and the spirits of subversion (see Sedgwick, 2003, pp. 1112). And clinging to this image only
outlines of such an analysis, at the very end of his paradigmatic account of governmentality, but investigation of this phenomenon is poorly developed at present

makes it all the more difficult to acknowledge the possibility of positive political action
that does not conform to a romanticized picture of rebellion ,//

contestation, or protest against domination (see Touraine, 2001). Theories of neoliberalism are unable to recognize the emergence of new and innovative forms of
individualized collective action because their critical imagination turns on a simple evaluative opposition between individualism and collectivism, the private and the public.

The radical academic discourse of neoliberalism frames the relationship between


collective action and individualism simplistically as an opposition between the good and
the bad. In confirming a narrow account of liberalism, understood primarily as an
economic doctrine of free markets and individual choice, there is a peculiar convergence
between the radical academic left and the right-wing interpretation of liberal thought
exemplified by Hayekian conservatism. By obliterating the political origins of modern liberalismunderstood as answering the problem of how to live freely in societies divided

the discourse of neoliberalism reiterates a longer problem


for radical academic theory of being unable to account for its own normative priorities in
a compelling way. And by denigrating the value of individualism as just an ideological ploy
by the right, the pejorative vocabulary of neoliberalism invites us to take solace in an
image of collective decision-making as a practically and normatively unproblematic
procedure. The recurrent problem for theories of neoliberalism and neoliberalization is their two-dimensional view of both political power and of geographical space.
by interminable conflicts of value, interest, and faith

They can only account for the relationship between top-down initiatives and bottom-up developments by recourse to the language of centres, peripheries, diffusion, and

The turn to an overly


systematized theory of governmentality, derived from Foucault, only compounds the
theoretical limitations of economistic conceptualizations of neoliberalism. The task for social theory
contingent realizations; and by displacing the conceptualization of social relations with a flurry of implied subject-effects.

today remains a quite classical one, namely to try to specify the recurrent causal processes that govern the intersections between abstract, centrally promoted plans and social
life on the small scale (Tilly, 2003, p. 345). Neither neoliberalism-as-hegemony nor neoliberalism-as-governmentality is really able to help in this task, not least because both
invest in a deeply embedded picture of subject-formation as a process of getting-at ordinary people in order to make them believe in things against their best interests. With
respect to the problem of accounting for how hegemonic projects of neoliberalism win wider consensual legitimacy, Foucaults ideas on governmentality seem to promise an
account of how people come to acquire what Ivison (1997) calls the freedom to be formed and normed. Over time, Foucaults own work moved steadily away from an
emphasis on the forming-and-norming end of this formulation towards an emphasis on the freedom end. This shift was itself a reflection of the realization that the circularities of
poststructuralist theories of subjectivity can only be broken by developing an account of the active receptivity of people to being directed. But, in the last instance, neither the
story of neoliberalism-as-hegemony or of neoliberalism-as-governmentality can account for the forms of receptivity, pro-activity, and generativity that might help to explain how
the rhythms of the everyday are able to produce effects on macro-scale processes, and vice versa. So, rather than finding convenient synergies between what are already

closely related theoretical traditions, perhaps it is better to keep open those tiresome debates about the degree of coherence between them, at the same time as trying to
broaden the horizons of our theoretical curiosity a little more widely.

Alt/Strategy Answers

Specify Alt Key


Failure to specify how the alternative, or event, can solve existing problems
of economic decision-makingempirically resulted in tyranny and
economic catastrophe
Andrew SAYER Reader in Political Economy @ Lancaster 95 Radical Political Economy: A
Critique p. 13-14
Yet while the 'velvet' character of the revolutions was remarkable enough, there was little else that the Left could celebrate about
them. As Habermas points out, they were also singularly depressing in that they were devoid of 'ideas that are either innovative or
orientated to the future' (1991, p. 27). Whether Habermas meant it or not, I would add that it was Western Marxists as well as people
in the former socialist states who lacked ideas about alternatives. In this context, market triumphalism could divert attention from the
continued failings of capitalism, as if the 'victory' of capitalism meant that no one had any right to criticize it. Again, as Habermas put
it, 'it is not as though the collapse of the Berlin Wall solved a single one of the problems specific to our system' (Habermas, 1991, p.
xii). While the latter statement is surely correct it could be read as implying that it was 'business-as-usual' for the Left. It is my view
that this kind of interpretation, together with those of Jameson and Callinicos, are complacent and hopelessly inad- equate. One can
agree with Jameson that Marxism is primarily a theory of capitalism, but this position is nevertheless all too smug, for it begs the
question of whether its account of capitalism is at all adequate.' Similarly, Callinicos implies that there are no

lessons to be learned from the demise of state socialism, save that it wasn't real
socialism, and there are certainly no lessons for the critique of capitalism.
This book is motivated by the view that such complacency is entirely unwarranted . The totalitarian
character of state socialism and its problems of economic motivation and coordination
are not historical aber- rations but are presaged by Marxism's lack of a sufficiently materialist
understanding of the social division of labour and its associated division and dispersion of knowledge in advanced
economies. This failing not only explains the inadequacies of state socialism's attempt to
plan such an economy centrally, but is the major unresolved flaw in Marxist theory of
capitalism. The reluctance of the Left to think through alternatives (for fear of producing
'blueprints' which might pre-empt future struggles) meant not only that radical political
movements had little idea of feasible and desirable objectives , but that the standpoints
from which capitalism and its problems were explained and criticized were unexamined
and often incoherent or undesirable. There is no way the Left can reply to market
triumphalism and the lack of alternatives without giving some consideration to the old
problems of political economy.

AT: pedagogy
Their insistence on pedagogy being first reifies extremism and
fragmentation in the academy only the perm avoids scholarship
shutdown
David Lake 11, political science prof at UC-San Diego, Why "isms" Are Evil: Theory,
Epistemology, and Academic Sects as Impediments to Understanding and Progress,
International Studies Quarterly (2011) 55, 465-480
My critique of our profession is a common one, but one worth repeating. Most generally, we
organize ourselves into academic "sects" that engage in self-affirming research and then
wage theological debates between academic religions. This occurs at both the level of
theory and epistemology. In turn, we reward those who stake out extreme positions
within each sect. Unfortunately, this academic sectarianism, a product of our own internal
political struggles, produces less understanding rather than more. Some reasonably fear
intellectual "monocultures," as McNamara (2009) has called the possible hegemony of
rationalism. But the current cacophony is not a sign of productive intellectual ferment in
the pursuit of meaningful knowledge." Rather, we have produced a clash of competing
theologies each claiming its own explanatory "miracles" and asserting its universal truth
and virtue. Instead, a large measure of intellectual humility Is in order. Theoretically, we are far
from the holy grail of a universal theory of international politicsif indeed such a grail even
exists. We should focus instead on developing contingent, mid-level theories of specific
phenomena. This analytical eclecticism is likely to be more productive (Sil and
Katzenstein 2010). But we aLso need a lexicon for translating otherwise incommensurable
theories and making them mutually intelligible. In the following section, I outline the problems
with theoretical sects and affirm the case for analytic eclecticism. I then end with one possible
"Rosetta stone" that aims to facilitate conversation across research traditions by suggesting that
all theories of international studies can be disaggregated into the basic and common concepts
of interests, interactions, and institutions. Epistemologically, there is perhaps an even deeper
divide that is, unfortunately, not so easily bridged. The nomothetic vs narrative divide cuts
through all of the social sciences and possibly beyond. This divide endures because scholars
either innately or through socializationfind one form of explanation more intellectually
satisfying than the other. Yet, in international studies, we have reified this divide and, as
with our theories, have formed mutually exclusive churches. Rather than claiming one or
the other epistemology Is always and everywhere superior, we should recognize that
both are valid and perhaps even complementary paths to understanding. The question is
not which approach is inherently superior, but which yieIds greater insights under what
circumstances. The second major section below takes up epistemology and its consequences
for professional practice and knowledge.

AT: Silly Alt


Failure to specify how the alternative can solve existing problems of
economic decision-making that create subsidies and overproduction will
produce either tyranny or ineffectiveness.
Andrew SAYER Reader in Political Economy @ Lancaster 95 Radical Political Economy: A
Critique p. 13-14
Yet while the 'velvet' character of the revolutions was remarkable enough, there was little else that the Left
could celebrate about them. As Habermas points out, they were also singularly depressing in that they
were devoid of 'ideas that are either innovative or orientated to the future' (1991, p. 27). Whether
Habermas meant it or not, I would add that it was Western Marxists as well as people in the former
socialist states who lacked ideas about alternatives. In this context, market triumphalism could divert
attention from the continued failings of capitalism, as if the 'victory' of capitalism meant that no one had
any right to criticize it. Again, as Habermas put it, 'it is not as though the collapse of the Berlin Wall solved
a single one of the problems specific to our system' (Habermas, 1991, p. xii). While the latter statement is
surely correct it could be read as implying that it was 'business-as-usual' for the Left. It is my view that this
kind of interpretation, together with those of Jameson and Callinicos, are complacent and hopelessly
inad- equate. One can agree with Jameson that Marxism is primarily a theory of capitalism, but this
position is nevertheless all too smug, for it begs the question of whether its account of capitalism is at all
adequate.' Similarly, Callinicos implies that there are no lessons to be learned from the demise of state
socialism, save that it wasn't real socialism, and there are certainly no lessons for the critique of
capitalism.
This book is motivated by the view that such complacency is entirely unwarranted. The totalitarian
character of state socialism and its problems of economic motivation and coordination are not historical
aber- rations but are presaged by Marxism's lack of a sufficiently materialist understanding of the social
division of labour and its associated division and dispersion of knowledge in advanced economies. This
failing not only explains the inadequacies of state socialism's attempt to plan such an economy centrally,
but is the major unresolved flaw in Marxist theory of capitalism. The reluctance of the Left to think
through alternatives (for fear of producing 'blueprints' which might pre-empt future struggles) meant not
only that radical political movements had little idea of feasible and desirable objectives, but that the
standpoints from which capitalism and its problems were explained and criticized were unexamined and
often incoherent or undesirable. There is no way the Left can reply to market triumphalism and the lack
of alternatives without giving some consideration to the old problems of political economy.

Neolib Alt
Reject their lens of neoliberalism. Starting with neoliberalism encourages
fake radicalism, oversimplification, and greater levels of cooptation than
positive and pragmatic politics.
-Ad hoc policies of neoliberalism also originate from Leftist movements for greater autonomy
-Sustainability politics also emerged during this time, but the neolib K ignores those and lumps
them all together
-Ignores positive action that doesnt conform to a romantic view of rebellion (i.e. the plan)
Clive BARNETT Faculty of the Social Sciences @ Open University (UK) 5
[The Consolations of Neoliberalism Geoforum 36 (1) p. Science Direct]
The blind-spot in theories of neoliberalismwhether neo-Marxist and Foucauldiancomes
with trying to account for how top-down initiatives take in everyday situations. So perhaps the
3. There is no such thing as neoliberalism!

best thing to do is to stop thinking of neoliberalism as a coherent hegemonic


project altogether . For all its apparent critical force, the vocabulary of neoliberalism and
neoliberalization in fact provides a double consolation for leftist academics : it supplies us with plentiful
opportunities for unveiling the real workings of hegemonic ideologies in a characteristic gesture of revelation;
and in so doing, it invites us to align our own professional roles with the activities of various
actors out there, who are always framed as engaging in resistance or contestation. The conceptualization of
neoliberalism as a hegemonic project does not need refining by adding a splash of Foucault. Perhaps we should try to do without the concept of neoliberalism altogether, because it might actually compound

, between an overly economistic derivation


of political economy and an overly statist rendition of governmentality, stories about
neoliberalism manage to reduce the understanding of social relations to a residual
effect of hegemonic projects and/or governmental programmes of rule (see Clarke, 2004a). Stories
about neoliberalism pay little attention to the pro-active role of socio-cultural
processes in provoking changes in modes of governance, policy, and regulation. Consider the example of the restructuring of public
rather than aid in the task of figuring out how the world works and how it changes. One reason for this is that

services such as health care, education, and criminal justice in the UK over the last two or three decades. This can easily be thought of in terms of a hegemonic project of neoliberalization, and certainly one

these ongoing changes


in the terms of public-policy debate involve a combination of different factors that add up
to a much more dispersed populist reorientation in policy, politics, and culture. These factors include
dimension of this process has been a form of anti-statism that has rhetorically contrasted market provision against the rigidities of the state. But in fact

changing consumer expectations, involving shifts in expectations towards public entitlements which follow from the generalization of consumerism; the decline of deference, involving shifts in conventions and
hierarchies of taste, trust, access, and expertise; and the refusals of the subordinated, referring to the emergence of anti-paternalist attitudes found in, for example, womens health movements or anti-psychiatry
movements. They include also the development of the politics of difference, involving the emergence of discourses of institutional discrimination based on gender, sexuality, race, and disability. This has disrupted
the ways in which welfare agencies think about inequality, helping to generate the emergence of contested inequalities, in which policies aimed at addressing inequalities of class and income develop an ever
more expansive dynamic of expectation that public services should address other kinds of inequality as well (see Clarke, 2004b J. Clark, Dissolving the public realm? The logics and limits of neo-liberalism,
Journal of Social Policy 33 (2004), pp. 2748.Clarke, 2004b). None of these populist tendencies is simply an expression of a singular hegemonic project of neoliberalization. They are effects of much longer

what we have come to recognise as


hegemonic neoliberalism is a muddled set of ad hoc, opportunistic accommodations to
these unstable dynamics of social change as it is to think of it as the outcome of highly coherent political-ideological projects. Processes
of privatization, market liberalization , and de-regulation have often followed an ironic
rhythms of socio-cultural change that emanate from the bottom-up. It seems just as plausible to suppose that

pattern in so far as they have been triggered by citizens movements arguing from the
left of the political spectrum against the rigidities of statist forms of social policy and
welfare provision in the name of greater autonomy, equality, and participation (e.g. Horwitz, 1989).

The political re-alignments of the last three or four decades cannot therefore be
adequately understood in terms of a straightforward shift from the left to the right, from
values of collectivism to values of individualism, or as a re-imposition of class power. The emergence and generalization of this populist ethos
has much longer, deeper, and wider roots than those ascribed to hegemonic neoliberalism. And it also points towards the extent to which easily the most widely resonant political rationality in the world today is

Recent theories of neoliberalism


have retreated from the appreciation of the long-term rhythms of socio-cultural change ,
which Stuart Hall once developed in his influential account of Thatcherism as a variant of authoritarian populism. Instead, they favour elite-focused
analyses of state bureaucracies, policy networks, and the like. One consequence of the residualization of the social is that theories
of neoliberalism have great difficulty accounting for, or indeed even in recognizing, new forms of
individualized collective-action (Marchetti, 2003) that have emerged in tandem with the apparent ascendancy of
neoliberal hegemony: environmental politics and the politics of sustainability ; new forms of
consumer activism oriented by an ethics of assistance and global solidarity; the identity politics of sexuality
not right-wing market liberalism at all, but is, rather, the polyvalent discourse of democracy (see Barnett and Low, 2004).

related to demands for changes in modes of health care provision, and so on (see Norris, 2002). All of these might be thought of as variants of what we might want to call bottom-up governmentality. This refers to
the notion that non-state and non-corporate actors are also engaged in trying to govern various fields of activity, both by acting on the conduct and contexts of ordinary everyday life, but also by acting on the
conduct of state and corporate actors as well. Rose (1999, pp. 281284) hints at the outlines of such an analysis, at the very end of his paradigmatic account of governmentality, but investigation of this

. Instead, the trouble-free amalgamation of Foucaults ideas into the


Marxist narrative of neoliberalism sets up a simplistic image of the world divided
between the forces of hegemony and the spirits of subversion (see Sedgwick, 2003, pp. 1112). And clinging
phenomenon is poorly developed at present

to this image only makes it all the more difficult to acknowledge the possibility of
positive political action that does not conform to a romanticized picture of rebellion ,
contestation, or protest against domination (see Touraine, 2001). Theories of neoliberalism are unable to recognize the emergence of new and innovative forms of individualized collective action because their

The radical academic discourse of


neoliberalism frames the relationship between collective action and individualism
simplistically as an opposition between the good and the bad. In confirming a narrow
account of liberalism, understood primarily as an economic doctrine of free markets and
individual choice, there is a peculiar convergence between the radical academic left and
the right-wing interpretation of liberal thought exemplified by Hayekian conservatism. By obliterating the political origins of modern liberalism
understood as answering the problem of how to live freely in societies divided by interminable conflicts of value, interest, and faith the discourse of neoliberalism
reiterates a longer problem for radical academic theory of being unable to account for its
own normative priorities in a compelling way. And by denigrating the value of individualism
as just an ideological ploy by the right, the pejorative vocabulary of neoliberalism
invites us to take solace in an image of collective decision-making as a practically and
normatively unproblematic procedure. The recurrent problem for theories of neoliberalism and neoliberalization is their two-dimensional view of both political
critical imagination turns on a simple evaluative opposition between individualism and collectivism, the private and the public.

power and of geographical space. They can only account for the relationship between top-down initiatives and bottom-up developments by recourse to the language of centres, peripheries, diffusion, and

The turn to an overly systematized


theory of governmentality, derived from Foucault, only compounds the theoretical
limitations of economistic conceptualizations of neoliberalism. The task for social theory today remains a quite classical
contingent realizations; and by displacing the conceptualization of social relations with a flurry of implied subject-effects.

one, namely to try to specify the recurrent causal processes that govern the intersections between abstract, centrally promoted plans and social life on the small scale (Tilly, 2003, p. 345). Neither neoliberalismas-hegemony nor neoliberalism-as-governmentality is really able to help in this task, not least because both invest in a deeply embedded picture of subject-formation as a process of getting-at ordinary people in
order to make them believe in things against their best interests. With respect to the problem of accounting for how hegemonic projects of neoliberalism win wider consensual legitimacy, Foucaults ideas on
governmentality seem to promise an account of how people come to acquire what Ivison (1997) calls the freedom to be formed and normed. Over time, Foucaults own work moved steadily away from an
emphasis on the forming-and-norming end of this formulation towards an emphasis on the freedom end. This shift was itself a reflection of the realization that the circularities of poststructuralist theories of
subjectivity can only be broken by developing an account of the active receptivity of people to being directed. But, in the last instance, neither the story of neoliberalism-as-hegemony or of neoliberalism-asgovernmentality can account for the forms of receptivity, pro-activity, and generativity that might help to explain how the rhythms of the everyday are able to produce effects on macro-scale processes, and vice
versa. So, rather than finding convenient synergies between what are already closely related theoretical traditions, perhaps it is better to keep open those tiresome debates about the degree of coherence
between them, at the same time as trying to broaden the horizons of our theoretical curiosity a little more widely.

AT: Bifo

Bifo Alt Fails


Bifos alternative fails at disrupting capital---a simple withdrawal from the
system evacuates politics and cedes it to the Right---turns the K.
Lear 12
(Ben is an underemployed researcher living in Manchester, UK. He recently co-authored an
article in Occupy Everything! Reflections on Why it's Kicking off Everywhere, and is a member
of Plan C., "Lifeboat Communism A Review of Franco Bifo Berardis After the Future," 5-182012, Viewpoint Magazine, http://viewpointmag.com/2012/05/18/lifeboat-communism-a-reviewof-franco-bifo-berardis-after-the-future/)
What does the end of the future mean for radical politics ? It is at this point that Bifos argument
becomes problematic. In an argument that intersects with groups such as Tiqqun, Bifo argues that we must see Communism as a necessity in the collapse of capital . Distant from the voluntarism of previous forms of Communist politics, this
post-growth Communism will be best understood as a necessary response to capitals refusal of labour. Cut adrift from the opportunity to work, with welfare systems disman-

Bifo argues that we will witness the proliferation of zones of autonomy responding to
the needs of an increasingly precarious and superfluous social body. Communist politics will emerge from an exodus, both voluntary and
tled,

compulsory, from a stagnating and increasingly predatory state-capital nexus. This exodus is both social, in the development of an alternative infrastructure, and personal, in the
withdrawal from the hyper-stimulation of the semiotic economy. Bifo abandons hope in collective contestation at the level of the political.

Bifos politics

could be described as a kind of lifeboat communism . As the crisis ripples, mutates, and deepens, Bifo
sees the role of communism as the creation of spaces of solidarity to blunt the worst effects of the crisis of social reproduction. Gone is
the demand for a better world for all , the liberation of our collective social wealth, or the
unlocking of the social potentials of technology . Rather, Bifos politics are based around
insulating a necessarily small portion of society from the dictates of capital. By withdrawing
from the political sphere , we accept the likelihood of losing the final scraps of the welfare
state and concede the terrain of the political to zombie politics and predatory capital .
Rather than seeking new forms of organization to re-enter the political stage, Bifo seems
to suggest that we seek shelter beneath it as best we can. This shying away from the
political stage is the weakness at the heart of the book. Recent eruptions of political
struggle have captured the collective-imagination because they demonstrate that political contestation
is still possible today, in spite of the obstacles Bifo has described. The Occupy movement and the uprisings in the M iddle E ast and N orth A frica have resonated with all those who still have hope in collective struggle. Although these movements have encountered varying problems, to which we must
develop solutions , they dispel the idea of an unchangeable present. The current blockages to successful organising have
been shown to be strategic and tactical , not terminal. Misdiagnosing the current inertia
of post-political public life as a terminal condition leads the left towards an evacuation
of the political , while we should instead reassert its primacy. If we abandon any hope of fighting in, against, and beyond the existing architecture of the state and capital,

and instead seek refuge in small communes , and go-slow practices, we abandon all real

hope of a generalized, or generalizable, emancipatory politics . Although Bifos analysis of the difficulties of collective action resonates

the proposal for a simple withdrawal from capitalism is


a bleak politics indeed which, at its most optimistic, calls for an orderly default by portions of the
proletariat. The horizons of communist politics appear much narrower when capitalism is no longer seen as the repository of a vast store of social wealth awaiting collective redistribution, but rather redefined as an unassailable site of universal and permanent austerity combined with widening social redundancy. It is hard to
imagine a network of self-organized projects and systems supporting the majority of the
population in the context of an increasingly predatory capitalism. Emerging from the and isolated leftist scenes,
this lifeboat communism will by its very nature have a limited carrying capacity, as the anarchist
with all of us who have attempted to organize struggles in the past few decades,

experience in post-Katrina New Orleans attests. The lifeboats that Bifo calls for will undoubtedly be too small and makeshift to harbor us all. The crisis is twofold. It is a crisis of
capitalist profitability, and of an increasingly precarious and surplus global proletariat whose reproduction (as both labour and body) is under threat. It is unlikely that the proliferation of communes, squats, food co-ops, file sharers, urban gardeners, and voluntary health services will bring forth a new, better world. But while the current seemingly postpolitical situation throws up massive obstacles to organizing, there is still a potential for collective contestation. The capitalist state, racked by its own legitimacy crisis and
weekly political scandals, is more vulnerable than it appears. We need only recall the period of unexpected hope built by students in Britain, occupiers in Oakland, and vast
swathes of North Africa and the Middle East during the past two years. These movements were mobilised through the betrayal of a vision of the future but alongside their rage,
they put forth a hope which can guide our politics. The task at hand is to unlearn old behaviour and to forge new tactical and organisational weapons for struggle. Bifos contribution is a timely and challenging one, but it ultimately leads us back towards a DIY culture and outreach politics. As our movements come to terms with these limits, we must
also hold onto the belief that luxury for all is possible. The social potential of unfilled blocks of flats, emerging technologies like 3D-printing, and the desires of the millions of
underemployed, should remind us of this. This will not be possible without a collective struggle against the state and the demands of capital, one which simultaneously defends
what we have and attempts to move beyond it.

A retreat to lifeboat politics is both premature and a self-fulfilling

prophecy . While Bifo correctly analyses the current conjuncture clearly identifying the post-political state, the weakness of the Left, the crisis of profitability and
new forms of labour, and their impact on the subject his political prescriptions lead us in the wrong direction. Just as Bifo
does, we place the struggle against work at the center; but we can also seek to liberate social wealth, rather than insulate a lucky few from the ravages of capital. Rather than
No Future, we must raise a different banner: The futures here, it just needs reorganizing.

Cap K

Specific Adv k Env


Advocacy for specific policy reform is key to environmental justice
movements---refusal of policy relevance ensures marginalization
Noonan 5
(Douglas S., Assistant Professor, School of Public Policy, Georgia Institute of Technology, 2005,
DEFINING ENVIRONMENTAL JUSTICE: POLICY DESIGN LESSONS FROM THE PRACTICE
OF EJ RESEARCH, http://www.prism.gatech.edu/~dn56/EJ.APPAM.pdf)
The negotiated nature of environmental policymaking holds some stark lessons for policymakers
and analysts alike. Even if there were no uncertainty and all of the useful scientific evidence was available the
heterogeneous interests of affected parties would persist. When policies ultimately seek to
reconcile these competing interests, essentially answering questions of social choice (for which optimal solutions
may not be available either in theory or due to practical limits to policy), only rarely or never would a policy process
be such that selfish advocacy by interest groups yields both individually and socially
optimal outcomes. In the environmental policy arena, the disconnect between the pursuit of
individual interests and the pursuit of collective goals is paramount. In this sense, the acrimony
surrounding many environmental policy debates is both undersandable and inevitable. Although this preface might apply equally

the application to
environmental justice (EJ) provides an opportune arena in which to observe the interplay
between environmental policymaking and the (allegedly) relevant research. Environmental
justice is a major theme in environmental and social policy. Its researchers are legion.
Their output is voluminous. A debate about the empirical evidence and about appropriate
policies continues among academics. In more public forums, interest groups routinely cite
environmental justice in advocating for policy reforms. As is typical in policy debates,
advocates select evidence to cite in support of their position. The influence of scholarly
well to discussions of climate change policy or species/wilderness preservation policy,

EJ research on policymakers , however, is less than straightforward . If the mounting


evidence provides only partial answers or, as is common, answers to questions only
marginally relevant to policymakers , then even hundreds of books 1 on the subject may
do little to sway public policy . Or, conversely, the evidences influence may far outstrip its limited relevance.
Regardless, like many other environmental policy topics, the role of scholarly research in policy design is inevitably contentious and
complex. The

purpose of this paper is to offer some insight about policy design from the
scholarly literature on EJ. After scaling this mountain of literature, what are the important lessons to be
learned for making EJ policy? From this vantage, this paper critiques the field of EJ research . It
also offers some suggestions for a more policy-relevant research agenda. The conclusion returns to the broad assessment of EJ
policy and suggests some future directions for designing policy and framing the discourse.

AT: Epistemology Flawed


Pragmatic, limited correction of our epistemology by testing environmental
solutions is better than trying fullscale
epistemological or ontological shift. They stop us from developing better
forms of management.
Norton 2
(Bryan, Philosophy @ Georgia Tech 2 in Pragmatist Ethics for a Technological Culture ed.
Keulartz p. 179-182)
The positivist sociologist, Otto Neurath, provided a useful simile - one that has become very popular with pragmatists today - that illustrates the pragmatist search for truth

Improving our knowledge and understanding, Ncnnth said, is most like repairing a ship while
on the high seas. Imagine a ship that is kept in service indefinitely, with no opportunity to be pulled into dry dock for repairs, As particular planks
weaken from weather and heavy use, we replace them, standing on the strongest remaining planks
while replacing the weakest ones. In principle, it would be possible to replace every plank, resulting in a "new" ship in the physical sense that
every plank is new, but the -ship" remains itself, with no need to be re-christened. The analogy captures perfectly the pragmatists' approach to epistemology. First, the
pursuit of epistemological renewal - for the pragmatists, unlike Descartes does not start by
tearing down every plank of knowledge, necessitating a completely new construction.
Rather the task of renewal begins for pragmatists by identifying the most problematic "planks" of knowledge.
Since we must keep the boat afloat as we do our repairs identifying the problematic planks is not just a matter of identifying
(discussed in Quine, 1960).

what beliefs we have the least evidence for. In the ship analogy, this might mean that we fix a moderately rotted plank below the water level before replacing a badly rotted one
in a little-used area of the deck; in pragmatist epistemology, this kind of decision involves identifying uncertainties that am particularly relevant to community survival and to other
chosen social goals The assessment is furthermore relative to appropriate values, not simply to a mechanical application of a test of physical strength of the particular "planks"
of knowledge. The analogy also illustrates the idea of piecemeal improvement of a belief system in which no belief is ultimately privileged, even though some beliefs am
accepted, in context, as unquestioned for very long periods of time. So the analogy, by noting that the entire ship is subject to the ravages of time and wear, and will eventually
be replaced with new wood. illustrates also the pragmatists idea that every belief is up for re- evaluation as necessary. If we am imagine our sailors continuing their back and
forth passages indefinitely, each and every belief will eventually be submitted to the test of experience and experiment. Its time will come when it is relevant to an important
disagreement regarding what needs to be done and how it must be done. Each plank of knowledge will thus eventually be vt-evaluated and its strength at any given point will be
the extent to which it has held up to observation and experiential tests in past cases in which it was called into question. The ship analogy also illustrates in simple tams a way
out of the quandaries of subjectivism, anti-realism, and relativism. Some readers, reasoning as follows, might think that Neurath's analogy pushes us closer to relativism. Since
the decision which 'plank to stand on and which to fix is a matter of judgment. surely affected by the - and values of the am (survival, at least), it might be argued that different
people, including different scientists, will lake different things to be "given" and unquestioned at any particular time. Accepting that our society is made up of people with different
values and different assumptions - paradigms and worldviews - it could be argued that we must expect different people with different perspectives to "Mm different goals, values,
and to put different scientific hypotheses up for t. Not only will people cite very different beliefs in support of policy choices, communication across perspectives may become
impossible, because the choice of linguistic categories and meanings depends also on individual's values and This undermines appeals to any particular linguistic vocabulary as
privileged. as valid pre-experientially (Quint, l%l) we must give up, as Dewey saw, appeals to fixed and eternal categories and to fixed and eternal maths The ways in which we
identify, characterize, individuate, and aggregate objects of the senses are deeply affected by our goals and values as well as our perspective and worldview. Through language,
we construct reality; and different languages reflect, it would seem, different wont inhabited by different tens from different perspectives. If we apply Neurath's analogy to
language, we find that a kind of linguistic relativity - perhaps better called, linguistic conventionalism- is unavoidable. Despite this form of conventionalism, which plays havoc

The - analogy avoids a


skeptical and relativist conclusion about beliefs and knowledge, because the decisions made do not
escape the test of experience. Over time, staying afloat depends on realistic assessments of
damage to the ship and realistic models of what will happen if a particular plank fails. If the
with our ability to match specific sentences with particular bits of reality, there is still an element of realism in Neurath's AM.

repair crew constantly repairs the easiest planks to access. ignoring the ones under deep water, a disaster will eventually occur. Similarly, if the crew replica the bar in the
Officers' galley every time it is scratched, while ignoring severe rot below the water line. the whole crew will be selected out of the pool of sailors and a new ship will have to be
launched. Further, while languages may suggest different and relative ontologies for individuals who speak different languages, language is not an individual matter. Just as the
repair crew on the ship must communicate if they arc to collectively decide what to do next, a seeker alter the truth will be a member of an intellectual community, Some form of
shared language or linguistic communication is presupposed in the designation of a group sa a 'community. Similarly, - as the repair crew's decisions will affect the whole crew,

the progression of the scientific endeavor will


bring larger and larger communities - and their collective and individual experiences into discourse about what to do to protect the environment. The key point here is that Neurath's analogy
shows clearly how we can adopt a position of limited realism by pushing the level at which we
judge truth and falsity from the sentential level to the action level at which we act on our total belief system in particular situations. We
giving crew members an incentive to oversee the repairmen's decisions and work,

can recognize. in the face of unexpected experiences and surprising outcomes of experiments, that our set of beliefs is inadequate; we cannot, however, on the basis of one or
a few such experiences, unambiguously identify where the problem in our belief system lies, and there is no algorithm for deciding the best way to repair problems as they arise.
Because, in any given situation, what we expect is determined not simply by one or a few beliefs our expectations arc supported by our entire system of beliefs, including

background knowledge, that is (temporarily) not doubted in the given situation. If expectations are not born out by experience, y solution requires a creative reconstruction

Direct experience, to extend Neurath's analogy, can tell us that we have a leak" in our system of
knowledge. Determining how best to fix the leak may require that we reconsider some of out background beliefs. Limited realism accepts
the fact that no substantive knowledge of the external world is knowable a priori; we must correct
(Quine. 1960).

our system of beliefs without benefit of prior principles to guide us. It also accepts the apparently unavoidable conclusion that our varied linguistic forms, designed to function in
many different situations, yield no common underlying structure for all experience. If categories we find in the world are of our own making; they we not given in reality

realism must be limited. But 11 remains realism in the one important sense. It retains a
method of selection and is thus self-correcting on the basis of broader experience and open
deliberation. One cannot believe anything one wishes and still survive -this is a necessity
imposed upon us by our environment; it is not volitional, it is imposed upon us. The crew on Neurath's ship either stays afloat or they die.
independent of us For these reasons

For the pragmatist. it is not so important to separate the hypotheses acted upon from the attitudes and values that bring them to the fore in real situations requiring inquiry; what
is important is that we observe and experiment, that there be an external check on claims and counterclaims There is, after all, an experience that will tell the repair crew they
made a mistake, given their shared goals. The experience begins with the sensation of sinking into wetness. The action contexts, and certain unavoidable experiences, correct
us when we have wrong beliefs and inappropriate goals.

Adaptive Managers can thus adopt limited reaIism as a

working epistemology. Since environmental management is clearly a cultural and social endeavor. it can be hoped that (a) many belief systems,
perspectives , and viewpoints will be proposed and tried out, but that (b) this initial relativity will be gradually reduced as proposals, the belief systems that =MM them, and even

we an learn new methods and


whereby proposals and beliefs can be tested through pilot projects, experiments, and so forth, avoiding the
necessity of having a cultural collapse to disprove every errant hypothesis . This hope is
represented, in pragmatist thought, in the emphasis pragmatists place on devising techniques and
methods that help to winnow truth from falsehood without catastrophic cultural failure. If
the perspectives taken, we subjected to the ultimate teet do they work in real situations? Hopefully, as time goes on,
techniques

indeed such methods an be devised and gradually improved, it would appear that the optimism of Adoptive Managers - who, we admitted at the outset, must have some reason
to believe that observation and experiment will provide some advantage in an open-ended pros of eavironinental management - are provided intellectual support by a pragmatist
epistemology. Our cxcursion So limited new and pragmatist epistemology suggests tint, given many diverse voices engaged in dialogue triut what we ought to do to protect the
environment a general method cmcrg from our accvunL The method an, at its best, help us to pick winning from losing strategies by use of s studies, observations, and
experiments, and a diligent recording of what works and what doesn't work in particular situations. There is thus reason to believe that Adeptive Management an provide a selfcorrective method for pursuing environmental protection.

Envt O/Ws Ontology


Prioritize environmental existence over framing and ontology.
Wapner 3
(Paul Prf. And Director of the Global Environmental Policy Program @ American 3 Leftist
Criticism of Nature, Dissent, Winter, p. 74-75)
response to eco-criticism would require critics to acknowledge the ways in which
they themselves silence nature and then to respect the sheer otherness of the nonhuman world. Postmodernism prides
itself on criticizing the urge toward mastery that characterizes modernity. But isnt mastery
exactly what postmodernism is exerting as it captures the nonhuman world within its
own conceptual domain? Doesnt postmodern cultural criticism deepen the modernist urge
toward mastery by eliminating the ontological weight of the nonhuman world? What else
could it mean to assert that there is no such thing as nature? I have already suggested the postmodernist response:
The third

yes, recognizing the social construction of nature does deny the self-expression of the nonhuman world, but how would we know what such self-expression means? Indeed,

All attempts
to listen to nature are social constructionsexcept one. Even the most radical postmodernist must acknowledge the
distinction between physical existence and nonexistence. As I have said, postmodernists accept that there is a physical
nature doesnt speak; rather, some person always speaks on natures behalf, and whatever that person says is, as we all know, a social construction.

substratum to the phenomenal world even if they argue about the different meanings we ascribe to it. This acknowledgment of physical existence is crucial. We cant ascribe
meaning to that which doesnt appear. What doesnt exist can manifest no character. Put differently, yes, the postmodernist should rightly worry about interpreting natures

all of us should be wary of those who claim to speak on natures behalf (including
But we need not doubt the simple idea that a prerequisite of expression is existence. This in turn
suggests that preserving the nonhuman worldin all its diverse embodimentsmust be seen
by eco-critics as a fundamental good. Eco-critics must be supporters, in some fashion, of environmental preservation. Postmodernists reject
expressions. And

environmentalists who do that).

the idea of a universal good. They rightly acknowledge the difficulty of identifying a common value given the multiple contexts of our value-producing activity. In fact, if there is
one thing they vehemently scorn, it is the idea that there can be a value that stands above the individual contexts of human experience. Such a value would present itself as a
metanarrative and, as Jean- Franois Lyotard has explained, postmodernism is characterized fundamentally by its incredulity toward meta-narratives. Nonetheless, I cant see
how postmodern critics can do otherwise than accept the value of preserving the nonhuman world. The nonhuman is the extreme other; it stands in contradistinction to
humans as a species. In understanding the constructed quality of human experience and the dangers of reification, postmodernism inherently advances an ethic of respecting

respect must involve ensuring that the other actually continues to exist.
In our day and age, this requires us to take responsibility for protecting the actuality of
the nonhuman. Instead, however, we are running roughshod over the earths diversity of
plants, animals, and ecosystems. Postmodern critics should find this particularly disturbing. If they dont, they deny their own intellectual insights
the other. At the very least,

and compromise their fundamental moral commitment. Now, what does this mean for politics and policy, and the future of the environmental movement? Society is constantly

As we wrestle with challenges of global


climate change, ozone depletion, loss of biological diversity, and so forth, we need to consider the economic,
being asked to address questions of environmental quality for which there are no easy answers.

political, cultural, and aesthetic values at stake. These considerations have traditionally marked the politics of environmental protection. A sensitivity to eco-criticism requires that
we go further and include an ethic of otherness in our deliberations. That is, we need to be moved by our concern to make room for the other and hence fold a commitment to
the nonhuman world into our policy discussions. I dont mean that this argument should drive all our actions or that respect for the other should always carry the day. But it
must be a central part of our reflections and calculations. For example, as we estimate the number of people that a certain area can sustain, consider what to do about climate

we must think about the lives of


other creatures on the earthand also the continued existence of the nonliving physical
world. We must do so not because we wish to maintain what is natural but because we wish to act in a morally respectable manner. I have been using postmodern
change, debate restrictions on ocean fishing, or otherwise assess the effects of a particular course of action,

cultural criticism against itself. Yes, the postmodernists are right: we can do what we want with the nonhuman world. There is nothing essential about the realm of rocks, trees,
fish, and climate that calls for a certain type of action. But postmodernists are also right that the only ethical way to act in a world that is socially constructed is to respect the
voices of the others of those with whom we share the planet but with whom we may not share a common language or outlook. There is, in other words, a limit or guiding

, One cant argue for the diversity of views of nature


without taking a stand for the diversity of nature.
principle to our actions. As political theorist Leslie Thiele puts it

Timeframe Key
Timeframe of solvency is the key issue: Administrative environmental
discourse is key to the effectiveness of short-term policy.
Jenneth PARKER Co-Director of the MSc in Environmental and Developmental Education @
South Bank Univ. 3 [Realism Discourse and Deconstruction eds. Jonathan Joseph and John
Michael Roberts p. 82]
In this way the

social action represented by 'Green Romanticism' is of a more semiotic


nature than is the social action represented by 'Administrative rationalism which has
more immediate concrete outcomes in terms of policies and material practices. 'these
discourses may actually be complementary when seen from the perspective of a diverse
and wide- ranging movement, which seeks to raise issues and effect social change in a variety
of different ways, One key consideration is the time-frame in which discourses seek to
operate. If you seek to gain short-term results, you will need a discourse that is clearly
related to dominant discourses; less so for medium-term results; and long-term results may require the subversion of
the dominant discourse itself in conjunction with changing certain key social structures and material practices. I would argue that

effective movements typically work with all these time-scales in addition to working at
different scales from micro to macro politics.

AT: Ethics O/W


Preventing death is the first ethical priority its the only impact you cant
recover from.
Bauman 95 Zygmunt Bauman, University of Leeds Professor Emeritus of Sociology, 1995,
Life In Fragments: Essays In Postmodern Morality, p. 66-71
being-for is like living towards-the-future: a being filled with anticipation, a being aware of the abyss between future foretold and future that will
eventually be; it is this gap which, like a magnet, draws the self towards the Other,as it draws life towards the future, making life into an activity of overcoming, transcending, leaving behind. The
self stretches towards the Other, as life stretches towards the future; neither can
grasp what it stretches toward, but it is in this hopeful and desperate, never
conclusive and never abandoned stretching-toward that the self is ever anew created
and life ever anew lived. In the words of M. M. Bakhtin, it is only in this not-yet accomplished world of anticipation and trial, leaning toward stubbornly an-other Other, that life
The

can be lived - not in the world of the `events that occurred'; in the latter world, `it is impossible to live, to act responsibly; in it, I am not needed, in principle I am not there at all." Art, the Other, the future:
what unites them, what makes them into three words vainly trying to grasp the same mystery, is the modality of possibility. A curious modality, at home neither in ontology nor epistemology; itself, like that
which it tries to catch in its net, `always outside', forever `otherwise than being'. The possibility we are talking about here is not the all -too-familiar unsure-of-itself, and through that uncertainty flawed, inferior
and incomplete being, disdainfully dismissed by triumphant existence as `mere possibility', `just a possibility'; possibility is instead `plus que la reahte' - both the origin and the foundation of being. The hope,
says Blanchot, proclaims the possibility of that which evades the possible; `in its limit, this is the hope of the bond recaptured where it is now lost."' The hope is always the hope of being fu filled, but what
keeps the hope alive and so keeps the being open and on the move is precisely its unfu filment. One may say that the paradox of hope (and the paradox of possibility founded in hope) is that it may pursue
its destination solely through betraying its nature; the most exuberant of energies expends itself in the urge towards rest. Possibility uses up its openness in search of closure. Its image of the better being is
its own impoverishment . . . The togetherness of the being-for is cut out of the same block; it shares in the paradoxical lot of all possibility. It lasts as long as it is unfulfilled, yet it uses itself up in never
ending effort of fulfilment, of recapturing the bond, making it tight and immune to all future temptations. In an important, perhaps decisive sense, it is selfdestructive and self -defeating: its triumph is its
death. The Other, like restless and unpredictable art, like the future itself, is a mystery. And being-for-the-Other, going towards the Other through the twisted and rocky gorge of affection, brings that mystery
into view - makes it into a challenge. That mystery is what has triggered the sentiment in the first place - but cracking that mystery is what the resulting movement is about. The mystery must be unpacked
so that the being-for may focus on the Other: one needs to know what to focus on. (The `demand' is unspoken, the responsibility undertaken is unconditional; it is up to him or her who follows the demand
and takes up the responsibility to decide what the following of that demand and carrying out of that responsibility means in practical terms.) Mystery - noted Max Frisch - (and the Other is a mystery), is an
exciting puzzle, but one tends to get tired of that excitement. `And so one creates for oneself an image. This is a loveless act, the betrayal." Creating an image of the Other leads to the substitution of the
image for the Other; the Other is now fixed - soothingly and comfortingly. There is nothing to be excited about anymore. I know what the Other needs, I know where my responsibility starts and ends.
Whatever the Other may now do will be taken down and used against him. What used to be received as an exciting surprise now looks more like perversion; what used to be adored as exhilarating
creativity now feels like wicked levity. Thanatos has taken over from Eros, and the excitement of the ungraspable turned into the dullness and tedium of the grasped. But, as Gyorgy Lukacs observed,
`everything one person may know about another is only expectation, only potentiality, only wish or fear, acquiring reality only as a result of what happens later, and this reality, too, dissolves straightaway

Only death, with its finality and irreversibility, puts an end to the musical-chairs game of the
real and the potential - it once and for all closes the embrace of togetherness which
was before invitingly open and tempted the lonely self." `Creating an image' is the dress rehearsal of that death. But creating an image is the inner urge, the constant temptation, the
into potentialities'.

must of all affection . . . It is the loneliness of being abandoned to an unresolvable ambivalence and an unanchored and formless sentiment which sets in motion the togetherness of being -for. But what
loneliness seeks in togetherness is an end to its present condition - an end to itself. Without knowing - without being capable of knowing - that the hope to replace the vexing loneliness with togetherness is
founded solely on its own unfulfilment, and that once loneliness is no more, the togetherness ( the being-for togetherness) must also collapse, as it cannot survive its own completion. What the loneliness
seeks in togetherness (suicidally for its own cravings) is the foreclosing and pre-empting of the future, cancelling the future before it comes, robbing it of mystery but also of the possibility with which it is
pregnant. Unknowingly yet necessarily, it seeks it all to its own detriment, since the success (if there is a success) may only bring it back to where it started and to the condition which prompted it to start on

The togetherness of being-for is always in the future

the journey in the first place.


, and nowhere else. It is no more once the self
proclaims: `I have arrived', `I have done it', `I fulfilled my duty.' The being-for starts from the realization of the bottomlessness of the task, and ends with the declaration that the infinity has been exhausted.

This is the tragedy of being-for - the reason why it cannot but be death-bound while
simultaneously remaining an undying attraction. In this tragedy, there are many happy moments, but no happy end. Death is
always the foreclosure of possibilities, and it comes eventually in its own time, even if not brought forward by the impatience of love. The catch is
to direct the affection to staving off the end, and to do this against the affection's nature. What follows is that, if moral relationship is grounded in the being-for togetherness (as it
is), then it can exist as a project, and guide the self's conduct only as long as its nature of a project (a not yet-completed project) is not denied. Morality, like the future itself, is forever
not-yet. (And this is why the ethical code, any ethical code, the more so the more perfect it is by its own standards, supports morality the way the rope supports the hanged man.) It is because of our
loneliness that we crave togetherness. It is because of our loneliness that we open up to the Other and allow the Other to open up to us. It is because of our loneliness (which is only belied, not overcome,

it is only through allowing the togetherness its


possibilities which only the future can disclose that we stand a chance of acting
morally, and sometimes even of being good, in the present.
by the hubbub of the being-with) that we turn into moral selves. And

Alt Answers

Alt Specificity k
Failure to specify how the alternative, or event, can solve existing problems
of economic decision-makingempirically resulted in tyranny and
economic catastrophe
Andrew SAYER Reader in Political Economy @ Lancaster 95 Radical Political Economy: A
Critique p. 13-14
Yet while the 'velvet' character of the revolutions was remarkable enough, there was little else that the Left could celebrate about
them. As Habermas points out, they were also singularly depressing in that they were devoid of 'ideas that are either innovative or
orientated to the future' (1991, p. 27). Whether Habermas meant it or not, I would add that it was Western Marxists as well as people
in the former socialist states who lacked ideas about alternatives. In this context, market triumphalism could divert attention from the
continued failings of capitalism, as if the 'victory' of capitalism meant that no one had any right to criticize it. Again, as Habermas put
it, 'it is not as though the collapse of the Berlin Wall solved a single one of the problems specific to our system' (Habermas, 1991, p.
xii). While the latter statement is surely correct it could be read as implying that it was 'business-as-usual' for the Left. It is my view
that this kind of interpretation, together with those of Jameson and Callinicos, are complacent and hopelessly inad- equate. One can
agree with Jameson that Marxism is primarily a theory of capitalism, but this position is nevertheless all too smug, for it begs the
question of whether its account of capitalism is at all adequate.' Similarly, Callinicos implies that there are no

lessons to be learned from the demise of state socialism, save that it wasn't real
socialism, and there are certainly no lessons for the critique of capitalism. This book is
motivated by the view that such complacency is entirely unwarranted . The totalitarian character of
state socialism and its problems of economic motivation and coordination are not
historical aber- rations but are presaged by Marxism's lack of a sufficiently materialist
understanding of the social division of labour and its associated division and dispersion of knowledge in advanced
economies. This failing not only explains the inadequacies of state socialism's attempt to
plan such an economy centrally, but is the major unresolved flaw in Marxist theory of
capitalism. The reluctance of the Left to think through alternatives (for fear of producing
'blueprints' which might pre-empt future struggles) meant not only that radical political
movements had little idea of feasible and desirable objectives , but that the standpoints
from which capitalism and its problems were explained and criticized were unexamined
and often incoherent or undesirable. There is no way the Left can reply to market
triumphalism and the lack of alternatives without giving some consideration to the old
problems of political economy.

Movements Fail/Perm Key


Movements/the alternative cant address immediate problemsthey are
multigenerational, educational ventures. Working within the system is
necessary to solve particular instances of the climate problemtheres no
guarantee their revolution will solve
-Working within existing political institutions is key
-Its too late to solve the whole environmental crisis, but can work to mitigate the damage
-No guarantee the alternatives regression to socialism wont have same environmental
problems
Christian PARENTI, professor of sustainable development at the School for International
Training, Graduate Institute, 13 [A Radical Approach to the Climate Crisis, Dissent, Summer
2013, http://www.dissentmagazine.org/article/a-radical-approach-to-the-climate-crisis]

Several strands of green thinking maintain that capitalism is incapable of a sustainable


relationship with non-human nature because, as an economic system, capitalism has a growth imperative while the earth
is finite. One finds versions of this argument in the literature of eco-socialism, deep ecology, eco-anarchism, and even among many
mainstream greens who, though typically declining to actually name the economic system, are fixated on the dangers of growth.
All this may be true. Capitalism, a system in which privately owned firms must continuously out-produce and out-sell their

However, that is not the same


question as whether capitalism can solve the more immediate climate crisis.
competitors, may be incapable of accommodating itself to the limits of the natural world.

Because of its magnitude, the climate crisis can appear as the sum total of all
environmental problemsdeforestation, over-fishing, freshwater depletion, soil erosion,
loss of biodiversity, chemical contamination. But halting greenhouse gas emissions is a much
more specific problem, the most pressing subset of the larger apocalyptic panorama .
And the very bad news is, time has run out. As I write this, news arrives of an ice-free arctic summer by 2050.
Scientists once assumed that would not happen for hundreds of years.

Dealing with climate change by first achieving radical social transformationbe it a


socialist or anarchist or deep-ecological/neo-primitive revolution, or a nostalgia-based localista conversion back to a
mythical small-town capitalismwould be a very long and drawn-out , maybe even
multigenerational, struggle . It would be marked by years of mass education and
organizing of a scale and intensity not seen in most core capitalist states since the 1960s or
even the 1930s.
Nor is there any guarantee that the new system would not also degrade the soil, lay waste
to the forests, despoil bodies of water, and find itself still addicted to coal and oil. Look at
the history of actually existing socialism before its collapse in 1991. To put it mildly, the economy
was not at peace with nature. Or consider the vexing complexities facing the left social
democracies of Latin America. Bolivia, and Ecuador, states run by socialists who are beholden to

very powerful, autonomous grassroots movements, are still very dependent on petroleum
revenue.
A more radical approach to the crisis of climate change begins not with a long-term vision of
an alternate society but with an honest engagement with the very compressed timeframe
that current climate science implies. In the age of climate change, these are the real parameters of
politics .

AT: Rejection (Warming)


Advocacy for specific policy reform is key to environmental justice movements--refusal of policy relevance ensures marginalization
Douglas S. Noonan 5, Assistant Professor, School of Public Policy, Georgia Institute of Technology,
2005, DEFINING ENVIRONMENTAL JUSTICE: POLICY DESIGN LESSONS FROM THE PRACTICE
OF EJ RESEARCH, http://www.prism.gatech.edu/~dn56/EJ.APPAM.pdf

The negotiated nature of environmental policymaking holds some stark lessons for policymakers and
analysts alike. Even if there were no uncertainty and all of the useful scientific evidence was available the heterogeneous
interests of affected parties would persist. When policies ultimately seek to reconcile these
competing interests, essentially answering questions of social choice (for which optimal solutions may not be available either in theory or due to practical
limits to policy), only rarely or never would a policy process be such that selfish advocacy by interest
groups yields both individually and socially optimal outcomes. In the environmental policy arena, the
disconnect between the pursuit of individual interests and the pursuit of collective goals
is paramount. In this sense, the acrimony surrounding many environmental policy debates is both undersandable and inevitable. Although this preface might
apply equally well to discussions of climate change policy or species/wilderness preservation policy, the application to environmental
justice (EJ) provides an opportune arena in which to observe the interplay between
environmental policymaking and the (allegedly) relevant research. Environmental justice is a major
theme in environmental and social policy. Its researchers are legion. Their output is
voluminous. A debate about the empirical evidence and about appropriate policies
continues among academics. In more public forums, interest groups routinely cite
environmental justice in advocating for policy reforms. As is typical in policy debates,
advocates select evidence to cite in support of their position. The influence of scholarly
EJ research on policymakers , however, is less than straightforward . If the mounting evidence
provides only partial answers or, as is common, answers to questions only marginally relevant
to policymakers , then even hundreds of books 1 on the subject may do little to sway
public policy . Or, conversely, the evidences influence may far outstrip its limited relevance. Regardless, like many other environmental policy topics, the role of
scholarly research in policy design is inevitably contentious and complex. The purpose of this paper is to offer some insight
about policy design from the scholarly literature on EJ. After scaling this mountain of literature, what are the
important lessons to be learned for making EJ policy? From this vantage, this paper critiques the
field of EJ research . It also offers some suggestions for a more policy-relevant research agenda. The conclusion returns to the broad assessment of EJ
policy and suggests some future directions for designing policy and framing the discourse.

AT: Zizek (rev/act)


Their proposal is just empty radicalism. Risking everything for revolution
creates Stalinist totalitarianism.
Julia HELL German Studies @ Michigan 6 Remnants of Totalitarianism Telos Fall Vol. 136
p. 96-103
This brings us to the present and the form of political actions that are thinkable, or unthinkable, in a condition allegedly dominated by
the opposition between totalitarianism and democracy. What is needed is a "freedom fighter with an inhuman face." In Zizek's
Revolution at the Gates, Antigone is such a model, her defiance an example of an act that "intervenes in the very

rational order of the Real, changing-restructuring its co-ordinates-an act is not irrational; rather, it
creates its own (new) rationality.""' This event "cannot be planned in advance-we have to take a risk, a step into the open, with no
Big Other to return our true message to us"-and its consequences might well be Stalinist terror , that is one of
the risks."' A freedom fighter with an inhuman face-the phrase resonates with Benjamin's early thoughts on the Angel of History as a
figure that embod- ies the creativity of destruction. iek discusses Benjamin's "Theses on the Philosophy of History" in the context of
"revolutionary violence" as "the transformation of the oppressed victim into an active agent. "6 To make the argument for the ethical
nature of the revolutionary act, 2iiek turns to Eric Santner's reading of Benjamin. "[A] present revolutionary interven- tion
repeats/redeems past failed attempts," Zizek writes."' He uses Eric Santner's notion of "symptoms" as "past traces which are
retroactively redeemed through the 'miracle' of the revolutionary intervention": they are, Santner writes, "not so much forgotten
deeds, but rather forgotten failures to act, failures to suspend the force of the social bond inhibiting acts of solidarity with society's
'others.""" Santner's political claims are more modest: these symptoms register not only past failed revolutionary attempts, but past
"failures to respond to calls for action, or even for empa- thy" on behalf of the suffering."" Santner uses Christa Wolf's reflections on
the Nazi pogroms of 1938, not on the events of 1917. But Zizek is not concerned with modest ethical acts; for
him, the excessive violence of the 1938 pogroms is a symptom that testifies to the "possibility of the authen- tic proletarian
revolution.""' This was an outburst of violence that covered "the void of the failure to intervene effectively in the social crisis.""' As

the Stalinist purges contained a redemptive kernel, so does, apparently, right-wing


violence. At stake is a contemporary politics of authentic acts that redeems these voids and creates a revolutionary future from a
revolu- tionary past. IV "A Crazy Wager on the Impossible": Eaek 's New (Post)Democratic Post-Politics If we read Zizek and Muffler
with reference to Arendt's Origins of Totali- tarianism, we discover two different, but complementary stories that express a familiar

In Zizeks writings, the entire mur- derous history of Stalinism is erased in


favor of a still unrealized future : the realization of the redemptive dimension----one that we find even at the heart of

dilemma of the left.

Stalinism. In MUller's texts, the GULAG is reified into a concept of history as catastrophe, the history of an eternal cycle of violence.
The future only exists as the repetition of that violence. Both iek and Muller draw on Benjamin's "Theses on the Philosophy of
History," which were written at the moment of the Hitler-Stalin Pact. The opposition between Muller's melancholic paralysis and iek's
revolutionary decisionism raises again a problematic that Yves de Maes- seneer discusses apropos of Benjamin's angel.
Maesseneer argues that the figure of the angel represents a "terrifying amalgam of redemption and destruction," because it implies
the "end of politics," either leading to res- ignation, or (state) terror .... If we appeal to Benjamin's angel, Maesseneer submits, we

either risk "an endorsement of the posture of a powerless witnessing of catastrophe,"


because the angel is "too immaterial to make a difference," or else we are endorsing radical destruction."'
Whether this assessment is valid for Benjamin's angel might be debatable; as a warning, it certainly applies to 2iek's and MUller's
readings of it."' I am not arguing that .iek revived Benjamin's angel with a bomb in one hand and a copy of the Koran in the other. I
do however agree with Geoff Boucher's analysis that iek's recent theorizing of the act as an "exit from the symbolic network, a
dissolution of social bonds" indicates a tension between democratic politics (as the formation of a hegeinonic project) and "quasireligious militarism,""' Boucher criticizes Zizek's notion of a foundational act as a leftover from

"Cultural-Revolution-period Maoism" and ultimately a retreat from politics, because it


seems to privi- lege individual over collective action and reduces politics and economics
to ideological struggles."' I have traced this new politics of "repeating Lenin" and the Bolsheviks' refusal of evolutionary
history to two different contexts. The first is the Eastern European context, i.e., the de-politicizing connection between petrified
(post)totalitarian conditions and the volun- tarist fantasies of Eastern Europe's dissident Marxists. The second is the context
discussed by Boucher, i.e., the politics of the 1970s. However, I propose to comprehend iek's re-invention of radical politics as a
return not to Maoism, but to the abstract radicalism of the RAE. In 1972, Ulrike Meinhof wrote a manifesto about Black September's
role in the anti-imperialist struggle. Meinhof argued that Germany was imperialism's fascist center, that Israel's conflict with the
Palestinians had turned that country into "Nazi-Faschismus," and that the bloody kidnappings in Munich constituted an "antiimperialist, anti-fascist" intervention."' Again, I am not arguing that iek is re-inventing the Angel of History as Islamic fundamentalist,
Palestinian freedom fighter, or the reincarnation of Ulrike Meinhof. But Meinhof's ghost does haunt his "freedom fighter with an

inhuman face." Anti-imperialist struggle, she wrote, aims at the "[m]aterial destruction of imperialist domination" and the "myth" of its
omnipotence.""' This sounds familiar: we could be reading a Maoist pamphlet. Meinhof's reflections on the symbolic core of militant
actions are more intriguing: "Propagandistic action as part of the material attack: the act of liberation in the act of annihilation. "`9
Libera- tion through destruction: in this statement we find remnants of Hegel's master-slave dialectic and its echoes in Fanon and
Sartre--and we find a crude foreshadowing of Zizek's conception of the authentic revolutionary act as one that changes the symbolic
itself. This raises again the question of which kinds of acts Ziek has in mind. Reading Zizek

unfortunately does not help to clarify this issue. What we do learn is that Zizek attempts to theorize politics
beyond "democracy." Discussing the challenge that Carl Schmitt's theory of the political poses to the left, Chantal Mouffe insists that
radical democracy be understood as a critique of parliamentary democracy, not as its dismissal. Radical democracy politicizes
liberal democracy by introducing Schmitt's agonistic definition of politics, which deliberative models of democracy exclude; and it
introduces agonistic pluralism into Schmitt's ineradicable 2onfiictuality by transforming antagonistic confrontations into agonistic
Dries, "enemies" into legitimate "adversaries" with whom "there exists a common ground.""' That parliamentary democracy provides
the space for the elaboration of this common symbolic ground has been the cornerstone of the post-Stalinist left and its reinvention
of democratic politics. In his essay on Schmitt's "decisionist formalism," Zizek argues that Schmitt asserts "the independence of the
abyssal act of free decision from its positive content "3' Like Mouffe, Zizek welcomes Schmitt's definition of the political as
antagonistic, but criticizes him for not properly articu- lating "the logic of political antagonism.""' Schmitt's move to limit the
friend/enemy distinction to external politics disavows the internal struggle that traverses society, while "a leftist position," hek writes,
insists on "the unconditional primacy of the inherent antagonism as constitutive of the political.""' Zizek then provides "positive
content" to Schmitt's formalism by defining the political as a struggle for democracy: "The political struggle proper is. never simply a
rational debate between mul- tiple interests, but simultaneously the struggle for one's voice to be heard and recognized as the voice
of a legitimate partner.""' The "protests of the 'excluded" always involve their right to be recognized."' Yet is Zizek's new

radical act really more than just another kind of empty, formalist decisionism? Granted, he
gives it a more material con- tent by insisting on the continuing relevance of class antagonism, i.e.,
the "notion of a radical antagonistic gap that affects the entire social body.""' In Welcome to the Desert of the Real, this gap is
exposed by the attacks on the World Trade Center, because, iek argues, these attacks represented the eruption of the real into our
symbolic order: they signaled the gap between the First and the Third Worlds, iek unequivocally distances himself from these
attacks. Nevertheless, this militant gesture does pose a problem. I see Zizek's recent involvement with theology as an attempt to
differentiate his messianic-militant politics from this kind of terrorism. And the hermeneutic pirouettes performed in the service of the
"redemptive kernel" of Stalinism serve the same function: to delineate the boundaries of what this act is and is not. The "freedom
fighter with the inhuman face" is no terrorist, Islamic or Stalinist-but is she anything more than a rev- enant from another desperate
age? To answer this question, we need to return to Ulrike Meinhof, In Welcome to the Desert of the Real, iiek compares the attacks
on the World Trade Center to those of the RAF. Meinhof's concept of the revolution- ary act, Zizek writes, is driven by the twentiethcentury "passion for the Real," a belief that violent transgression bombs people out of their numbed state."' However, this kind of act,
2iek argues, paradoxically produces only the "pure semblance of the effect of the Real.""' But does this analysis (which I read as a
kind of anticipatory rebuttal) really exhaust Meinhof's theory of the authentic act? What the RAF aimed for were three things: the
existential effect, the shock effect, and, finally, a kind of "rev- elation": the act's power to lay bare the (fascist) essence of the
(German) state. As 1 mentioned above, we find traces of Fanon's existentialism, but point two and three also hint at the legacy of
surrealism, of Debord and the Situatjonjst International, And it is here that we can locate Ziek's debt to the RAF. For we can read the
RAF's desire to "unveil" the true nature of the state in two ways: as the production of mere spectacle, a "thrill of the Real," or as a
desire to radically intervene on the level of the symbolic. '39 Like iek's authentic revolutionary act, Meinhof's theory of revolutionary
acts contained a symbolic dimension; they were aimed at a rearrangement of the very pre-conditions of politics. Zizek is thus in the
process of rethinking radical democracy through Meinhof, substituting the work of hegemonic articulation with a new strat- egy, the
authentic revolutionary act. And Zizek takes Mouffe's Gramscian rearticulation of the symbolic outside the space of liberal
parliamentary democracy. For, as Zizek points out in his response to Boucher, the time of optimism is over: "we effectively live in
dark times for democratic politics."40 Far from advocating a "crazy messianic politics of a radical violent Act," Zizek writes, in this
age of global capitalism he is concerned with finding ways to re-think radical change (which, he argues, Mouffe and Laclau
abandoned by limiting their anti-globalization strategy to "multiple local practices of resistance"). 141 Ultimately, Zizek

writes, "we cannot formulate a clear project of global change."" Zizek's angel is thus
really not much more than an intriguing, but ultimately empty, cipher-a remnant from a
bygone era. Where does this leave us? Curiously, in a position similar to that of Arendt in 1945: the conditions of both political
analysis and politics itself have fundamentally changed, Zizek argues, and therefore need to be radi- cally re-thought. While Arendt
takes recourse to the miracle of birth, Zizek conjures the miracle of the authentic act. What distinguishes Zizek from Arendt is his
willingness to take the ultimate risk: to sever the connection to liberal parliamentary democracy. In his recent writings, Zizek comes
"perilously close to an ultra-left refusal of the difference between capital- ist democracy and military dictatorship." 143 Like Arendt,
Zizek situates his recent work in the shadow of catastrophe ("dark times" is a transparent allusion to Brecht and National Socialism).
Unlike Arendt, Ziek does not escape this catastrophic imaginary but repeats its antinomies.'' Zizek's new politics thus

constitutes a curious double repetition: first, of Arendt's attempt to liberate politics from the catastrophic imagi- nary; and
second, of the RAF. iek himself analyzes 1970s terrorism as a response to the New Left's realization that the revolution will not
happen-neither in Berlin, nor Prague, nor Belgrade. '45 As the New Left disintegrated, groups like the RAP

and Red Brigades slowly slid into their suicidal politics. Muller fell for this messianic
politics at a moment when the petrified conditions of the GDR appeared to be its eternal
future. Ziek seems to fall for it now, his empty repetition of the RAP nothing but a
symptom-albeit apparently not a very enjoyable one. Zizek is certainly not the only one conceiving of a new
politics in rather empty tenns. Giorgio Agamben argues that modernity's murderous biopolitics has been accompanied by the state

of exception as a norm leading to the United States as its ultimate totalitarian instantiation. While Againben's view of (contemporary)
modernity is best described by Arendt's "law of ruin," his new politics comes down to nothing but a metaphysical desire to
experience genuine Being, a kind of Heideg- gerian great leap forward-or rather, a leap into the beyond."' Radical democracy
worked through the "shock of experience" that its theorists shared--however belatedly-with Arendt, and they heeded her advice to
think the unprecedented. Its strategies might need re-inventing (and Zizek's materialist re-centering of the social
around its basic antagonism is a productive first step). But

its basic tenets-that politics takes place within


the framework of parliamentary democracy and that it transforms the friend/enemy
antagonism into a friend/adversary agonism-still seems the adequate answer to U.S.
Republican politics and their own brand of catastrophic scenarios.

Neolib Alt Fails


Starting with neoliberalism encourages fake radicalism,
oversimplification, and greater levels of cooptation than positive and
pragmatic politics.
-Ad hoc policies of neoliberalism also originate from Leftist movements for greater autonomy
-Sustainability politics also emerged during this time, but the neolib K ignores those and lumps
them all together
-Ignores positive action that doesnt conform to a romantic view of rebellion (i.e. the plan)

BARNETT 5
(CliveFaculty of the Social Sciences @ Open University (UK) The Consolations of
Neoliberalism Geoforum 36 (1) p. Science Direct)
The blind-spot in theories of neoliberalismwhether neo-Marxist and Foucauldian
comes with trying to account for how top-down initiatives take in everyday situations. So
perhaps the best thing to do is to stop thinking of neoliberalism as a coherent
3. There is no such thing as neoliberalism!

hegemonic project altogether . For all its apparent critical force, the vocabulary of
neoliberalism and neoliberalization in fact provides a double consolation for leftist academics: it
supplies us with plentiful opportunities for unveiling the real workings of hegemonic
ideologies in a characteristic gesture of revelation; and in so doing, it invites us to align our own professional roles
with the activities of various actors out there, who are always framed as engaging in
resistance or contestation. The conceptualization of neoliberalism as a hegemonic project does not need refining by adding a splash of Foucault. Perhaps we
should try to do without the concept of neoliberalism altogether, because it might actually compound rather than aid in the task of figuring out how the world works and how it

, between an overly economistic derivation of political economy and an


overly statist rendition of governmentality, stories about neoliberalism manage to
reduce the understanding of social relations to a residual effect of hegemonic projects
and/or governmental programmes of rule (see Clarke, 2004a). Stories about neoliberalism pay
little attention to the pro-active role of socio-cultural processes in provoking changes in
modes of governance, policy, and regulation. Consider the example of the restructuring of public services such as health care, education, and criminal justice in
changes. One reason for this is that

the UK over the last two or three decades. This can easily be thought of in terms of a hegemonic project of neoliberalization, and certainly one dimension of this process has

these ongoing changes in


the terms of public-policy debate involve a combination of different factors that add up to
a much more dispersed populist reorientation in policy, politics, and culture. These factors include
been a form of anti-statism that has rhetorically contrasted market provision against the rigidities of the state. But in fact

changing consumer expectations, involving shifts in expectations towards public entitlements which follow from the generalization of consumerism; the decline of deference,
involving shifts in conventions and hierarchies of taste, trust, access, and expertise; and the refusals of the subordinated, referring to the emergence of anti-paternalist attitudes
found in, for example, womens health movements or anti-psychiatry movements. They include also the development of the politics of difference, involving the emergence of
discourses of institutional discrimination based on gender, sexuality, race, and disability. This has disrupted the ways in which welfare agencies think about inequality, helping to
generate the emergence of contested inequalities, in which policies aimed at addressing inequalities of class and income develop an ever more expansive dynamic of
expectation that public services should address other kinds of inequality as well (see Clarke, 2004b J. Clark, Dissolving the public realm? The logics and limits of neo-liberalism,
Journal of Social Policy 33 (2004), pp. 2748.Clarke, 2004b). None of these populist tendencies is simply an expression of a singular hegemonic project of neoliberalization.

what we have
come to recognise as hegemonic neoliberalism is a muddled set of ad hoc,
opportunistic accommodations to these unstable dynamics of social change as it is to think of it as
the outcome of highly coherent political-ideological projects. Processes of privatization, market liberalization , and deThey are effects of much longer rhythms of socio-cultural change that emanate from the bottom-up. It seems just as plausible to suppose that

regulation have often followed an ironic pattern in so far as they have been triggered by
citizens movements arguing from the left of the political spectrum against the rigidities

of statist forms of social policy and welfare provision in the name of greater autonomy,
equality, and participation (e.g. Horwitz, 1989). The political re-alignments of the last three or four
decades cannot therefore be adequately understood in terms of a straightforward shift
from the left to the right, from values of collectivism to values of individualism, or as a re-imposition
of class power. The emergence and generalization of this populist ethos has much longer, deeper, and wider roots than those ascribed to hegemonic neoliberalism. And it also
points towards the extent to which easily the most widely resonant political rationality in the world today is not right-wing market liberalism at all, but is, rather, the polyvalent

Recent theories of neoliberalism have retreated from the


appreciation of the long-term rhythms of socio-cultural change, which Stuart Hall once developed in his influential
account of Thatcherism as a variant of authoritarian populism. Instead, they favour elite-focused analyses of state
bureaucracies, policy networks, and the like. One consequence of the residualization of the social is that theories of
neoliberalism have great difficulty accounting for, or indeed even in recognizing, new forms of
individualized collective-action (Marchetti, 2003) that have emerged in tandem with the apparent ascendancy of
neoliberal hegemony: environmental politics and the politics of sustainability ; new forms of
consumer activism oriented by an ethics of assistance and global solidarity; the identity politics of
sexuality related to demands for changes in modes of health care provision, and so on (see Norris, 2002). All of these might be thought of as variants of what we might
discourse of democracy (see Barnett and Low, 2004).

want to call bottom-up governmentality. This refers to the notion that non-state and non-corporate actors are also engaged in trying to govern various fields of activity, both by
acting on the conduct and contexts of ordinary everyday life, but also by acting on the conduct of state and corporate actors as well. Rose (1999, pp. 281284) hints at the

.
Instead, the trouble-free amalgamation of Foucaults ideas into the Marxist narrative of
neoliberalism sets up a simplistic image of the world divided between the forces of
hegemony and the spirits of subversion (see Sedgwick, 2003, pp. 1112). And clinging to this image only
outlines of such an analysis, at the very end of his paradigmatic account of governmentality, but investigation of this phenomenon is poorly developed at present

makes it all the more difficult to acknowledge the possibility of positive political action
that does not conform to a romanticized picture of rebellion , contestation, or protest against domination (see
Touraine, 2001). Theories of neoliberalism are unable to recognize the emergence of new and innovative forms of individualized collective action because their critical

The radical academic


discourse of neoliberalism frames the relationship between collective action and
individualism simplistically as an opposition between the good and the bad. In confirming
a narrow account of liberalism, understood primarily as an economic doctrine of free
markets and individual choice, there is a peculiar convergence between the radical
academic left and the right-wing interpretation of liberal thought exemplified by Hayekian conservatism. By
imagination turns on a simple evaluative opposition between individualism and collectivism, the private and the public.

obliterating the political origins of modern liberalismunderstood as answering the problem of how to live freely in societies divided by interminable conflicts of value, interest,

the discourse of neoliberalism reiterates a longer problem for radical academic


theory of being unable to account for its own normative priorities in a compelling way. And
by denigrating the value of individualism as just an ideological ploy by the right, the
pejorative vocabulary of neoliberalism invites us to take solace in an image of
collective decision-making as a practically and normatively unproblematic procedure. The
and faith

recurrent problem for theories of neoliberalism and neoliberalization is their two-dimensional view of both political power and of geographical space. They can only account
for the relationship between top-down initiatives and bottom-up developments by recourse to the language of centres, peripheries, diffusion, and contingent realizations; and by

The turn to an overly systematized theory of


governmentality, derived from Foucault, only compounds the theoretical limitations of
economistic conceptualizations of neoliberalism. The task for social theory today remains a quite classical one, namely to try
displacing the conceptualization of social relations with a flurry of implied subject-effects.

to specify the recurrent causal processes that govern the intersections between abstract, centrally promoted plans and social life on the small scale (Tilly, 2003, p. 345).
Neither neoliberalism-as-hegemony nor neoliberalism-as-governmentality is really able to help in this task, not least because both invest in a deeply embedded picture of
subject-formation as a process of getting-at ordinary people in order to make them believe in things against their best interests. With respect to the problem of accounting for
how hegemonic projects of neoliberalism win wider consensual legitimacy, Foucaults ideas on governmentality seem to promise an account of how people come to acquire
what Ivison (1997) calls the freedom to be formed and normed. Over time, Foucaults own work moved steadily away from an emphasis on the forming-and-norming end of this
formulation towards an emphasis on the freedom end. This shift was itself a reflection of the realization that the circularities of poststructuralist theories of subjectivity can only
be broken by developing an account of the active receptivity of people to being directed. But, in the last instance, neither the story of neoliberalism-as-hegemony or of
neoliberalism-as-governmentality can account for the forms of receptivity, pro-activity, and generativity that might help to explain how the rhythms of the everyday are able to
produce effects on macro-scale processes, and vice versa. So, rather than finding convenient synergies between what are already closely related theoretical traditions, perhaps
it is better to keep open those tiresome debates about the degree of coherence between them, at the same time as trying to broaden the horizons of our theoretical curiosity a
little more widely.

Perm/Link Stuff

Envt Reform Cap k


Only the perm solvesproposing alternative non-capitalist economics out
of nowhere is of zero value. Environmental reform of capitalism is key.
Barry 7
(John BARRY Reader in Politics @ Belfast 7 [Towards a model of green political economy:
from ecological modernisation to economic security Int. J. Green Economics, Vol. 1, Nos. 3/4,
2007 p. 447-448)
Economic analysis has been one of the weakest and least developed areas of broadly green/sustainable
development thinking. For example, whatever analysis there is within the green political canon is
largely utopian usually based on an argument for the complete transformation of
modern society and economy as the only way to deal with ecological catastrophe, an often
linked to a critique of the socioeconomic failings of capitalism that echoed a broadly
radical Marxist/socialist or anarchist analysis; or underdeveloped due, in part, to the need to outline and develop other aspects of
green political theory. However, this gap within green thinking has recently been filled by a number of scholars, activists, think tanks, and environmental NGOs who have

The aim of this article


is to offer a draft of a realistic, but critical, version of green political economy to underpin the
outlined various models of green political economy to underpin sustainable development political aims, principles and objectives.

economic dimensions of radical views about sustainable development. It is written explicitly with a view to encouraging others to think through this aspect of sustainable

Combined realism and radicalism marks this article, which starts


with the point that we cannot build or seek to create a sustainable economy ab nihlo, but
must begin from where we are, with the structures, institutions, modes of production,
laws and regulations that we already have. Of course, this does not mean simply accepting
these as immutable or set in stone; after all, some of the current institutions, principles and structures underpinning the dominant economic model are the
very causes of unsustainable development. We do need to recognise, however, that we must work with (and
through in the terms of the original German Green Partys slogan of marching through the institutions) these existing structures, as well as
development in a collaborative manner.

change and reform and in some cases, abandon them as either unnecessary or positively harmful to the creation and maintenance of a sustainable economy and society.

an alternative economy and society must be based in the reality that


most people (in the West) will not democratically vote for a completely different type of
society and economy. That reality must also accept that a green economy is one that is
recognisable to most people and that indeed safeguards and guarantees not just their
basic needs but also aspirations (within limits). The realistic character of the thinking
behind this article accepts that consumption and materialistic lifestyles are here to stay (so long
as they do not transgress any of the critical thresholds of the triple bottom line) and indeed there is little to be gained by proposing
alternative economic systems, which start from a complete rejection of consumption and
materialism. The appeal to realism is in part an attempt to correct the common
misperception (and self-perception) of green politics and economics requiring an
excessive degree of self-denial and a puritanical asceticism (Goodin, 1992, p.18; Allison, 1991, p.170178). While
Equally, this article also recognises that

rejecting the claim that green political theory calls for the complete disavowal of materialistic lifestyles, it is true that green politics does require the collective reassessment of
such lifestyles, and does require a degree of shared sacrifice. It does not mean, however, that we necessarily require the complete and across-the-board rejection of

There must be room and tolerance in a green economy for people to live
ungreen lives so long as they do not harm others, threaten long-term ecological
sustainability or create unjust levels of socioeconomic inequalities. Thus, realism in this
context is in part another name for the acceptance of a broadly liberal or post-liberal (but certainly
not anti-liberal) green perspective.1
materialistic lifestyles.

Perm/ideology 1st Fails


Their insistence on ideology being first reifies extremism and
fragmentation in the academy only the perm avoids scholarship
shutdown
David Lake 11, political science prof at UC-San Diego, Why "isms" Are Evil: Theory,
Epistemology, and Academic Sects as Impediments to Understanding and Progress,
International Studies Quarterly (2011) 55, 465-480
we organize ourselves into academic
"sects" that engage in self-affirming research and then wage theological debates
between academic religions. This occurs at both the level of theory and epistemology. In
turn, we reward those who stake out extreme positions within each sect . Unfortunately, this
academic sectarianism, a product of our own internal political struggles, produces less understanding rather
than more. Some reasonably fear intellectual "monocultures," as McNamara (2009) has called the possible
hegemony of rationalism. But the current cacophony is not a sign of productive intellectual ferment
in the pursuit of meaningful knowledge." Rather, we have produced a clash of competing
theologies each claiming its own explanatory "miracles" and asserting its universal truth
and virtue. Instead, a large measure of intellectual humility Is in order. Theoretically, we are far from the holy grail of a universal theory of
international politicsif indeed such a grail even exists. We should focus instead on developing contingent,
mid-level theories of specific phenomena. This analytical eclecticism is likely to be more
productive (Sil and Katzenstein 2010). But we aLso need a lexicon for translating otherwise incommensurable theories and making them
My critique of our profession is a common one, but one worth repeating. Most generally,

mutually intelligible. In the following section, I outline the problems with theoretical sects and affirm the case for analytic eclecticism. I then end with one
possible "Rosetta stone" that aims to facilitate conversation across research traditions by suggesting that all theories of international studies can be
disaggregated into the basic and common concepts of interests, interactions, and institutions. Epistemologically, there is perhaps an even deeper
divide that is, unfortunately, not so easily bridged. The nomothetic vs narrative divide cuts through all of the social sciences and possibly beyond. This
divide endures because scholarseither innately or through socializationfind one form of explanation more intellectually satisfying than the other.
Yet, in

international studies, we have reified this divide and, as with our theories, have
formed mutually exclusive churches. Rather than claiming one or the other epistemology
Is always and everywhere superior, we should recognize that both are valid and perhaps
even complementary paths to understanding. The question is not which approach is
inherently superior, but which yieIds greater insights under what circumstances. The second
major section below takes up epistemology and its consequences for professional practice and knowledge.

AT: Vampiristic Consumption


Not vampiristic consumption
Braidotti 6
(Rosi, contemporary philosopher and feminist theoretician, Transpositions: On Nomadic Ethics,
76)
I beg to differ from Spivak's assessment. The

charge of vampiristic or consumerist consumption of others is an


ill-informed way of approaching the issue, in that it ignores the rigorous anti-humanistic,
cartographic and materialistic roots of poststructuralism. It specifically rests on a misreading
of what is involved in the poststructuralist critique of representation and on what is at
stake in the task of redefining alternative subject positions. Spivak attempts to rescue Derrida, whom she
credits with far more self-reflexivity and political integrity than she is prepared to grant to Foucault and Deleuze. The grounds for this preferential

Nomadic thinking challenges the semiotic approach that is crucial to


the 'linguistic turn' and also to deconstruction. Both Deleuze and Foucault engage in a critical dialogue with it and
work towards an alternative model of political and ethical practice. It seems paradoxical
that thinkers who are committed to an analytics of contemporary subject-positions get
accused of actually having caused the events which they account for; as if they were singlehandedly responsible for, or even profiting from, the accounts they offer as cartographies. Naming the
treatment are highly debatable.

networks of power-relations

in late postmodernity, however,

is not as simple as metaphorizing and

therefore consuming them . In my view there is no vampiristic approach towards 'otherness'


on the part of the poststructuralists. Moreover, I find that approach compatible with the emerging subjectivities of the former
'others' of Western reason. Late postmodernity has seen the proliferation of many and potentially contradictory discourses and practices of difference,

The point of coalition between


different critical voices and the poststructuralists is the process of elaborating the
spaces in-between self and other, which means the practice of the Relation . They stress
which have dislocated the classical axis of distinction between Self or Same/Other or Different.

the need to elaborate forms of social and political implementation of non-pejorative and
nondualistic notions of 'others'.

Disease

AT: Apoc Disease


Presenting the impact as the worst-case scenario is best to motivate action
these messages are unique because new antibiotics arent being
developed
-Past warnings were during a time when new antibiotics were coming out
Tom FOWLER ET AL. 14, Public Health, Epidemiology and Biostatistics @ University of
Birmingham, United Kingdom, Sally C. Davies, Chief Medical Officer for England, and David
Walker, Deputy Chief Medical Officer for England [The risk/benefit of predicting a post-antibiotic
era: Is the alarm working? Annals of the New York Academy of Sciences, Vol. 1323, p. 1-10
(September 2014)]
the main potential criticism of predicting a post-antibiotic era
can be answered thus: 1. there is a need for such predictions to help inform the response to
the issue, particularly if we are going to require societal as well as healthcare responses; 2. there is a
need for such predictions to provide an appropriate evidence base for the need for
resources to address the issue; 3. it is reasonable to make such predictions as a worstIs the alarm working? It can therefore be argued that

case scenario if action is not taken. This still leaves the question of whether this is the most effective way to lever action.
Given that Mary Barber was warning as early as 1958 of the growth of antimicrobial resistance, particularly in staphylococci, and introduced a hospital-

why are we in the current situation? It must be


warnings were made (arguably correct as they were) it was against a
backdrop of discovery of new antibiotics. Many believed that the introduction of methicillin
in 1960 marked the end of penicillin-resistant staphylococci . Ernst Chain (who shared the Noble Prize in
wide antibiotics reduction policy in Hammersmith Hospital (London),[31]
remembered that when these historical

Physiology or Medicine for penicillin with Alexander Flemming and Howard Florey) is reported to have said no more resistance problems, methicillin is
the answer.[6] It

is therefore unsurprising that these warnings did not elicit political action. At
the turn of the millennium, there was a flurry of activity around antimicrobial resistance ,[4,
32-34] including a World Health Organization Resolution, which led to the UK Antimicrobial Resistance Strategy and Action Plan.[35] Even this
was at a time where new classes of antibiotics had been discovered in the recent past .
Perhaps reflecting this in the first UK action plan, research on new antibiotics was the 16th and last objective, though more detail was then given later
in the document. This compares to the recent UK Five-Year Antimicrobial Resistance Strategy 20132018,[36] where one of the three strategic aims is

This suggests that previous warnings


around the potential for a post-antibiotic era have been ineffective. Yet, if we look at
recent activity we see a change in the high-level policy focus on the Biomedical Advanced Research and
to simulate the development of new antibiotics, diagnostics, and novel therapies.

Development Authority (BARDA). The recent Chief Medical Officer for England's Annual Report,[2] the CDC report,[13] the World Economic Forum's
Global Risks 2013 report,[7] and the Chennai Declaration[37] in India are all examples of this. In a keynote address at the conference Combating

Chan, Director-General of
the World Health Organization stated, If current trends continue unabated, the future is easy to
predict. Some experts say we are moving back to the pre-antibiotic era. No. This will be a post-antibiotic era. In terms of
new replacement antibiotics, the pipeline is virtually dry, especially for Gram-negative bacteria. The
cupboard is nearly bare. A post-antibiotic era means, in effect, an end to modern medicine as
we know it. Things as common as strep throat or a child's scratched knee could once again kill.[38]
Importantly, these calls to action are being responded to . In the United Kingdom, through the UK government scheme
Antimicrobial Resistance: Time for Action held in Copenhagen, Denmark on March 14, 2012, Margaret

Biomedical Catayst UK,[39] two small- and medium-size enterprises (SMEs), Cantab Anti-infectives and Discuva, are pioneering ways to tackle drug
resistance in Gram-negative bacteria. The National Institute for Health Research in the United Kingdom has recently announced a research funding call
in this area. In

Europe, the Innovative Medicines Initiative, a program specifically aimed at promoting public and private

has had a specific topic call around the discovery and


development of new drugs that combat Gram-negative infections. This call has the specific aim to create a
partnerships between academia and SMEs,

European Drug Discovery Centre of Excellence for antibiotic resistance.[40] The Joint Programming Initiative on Antimicrobial Resistance consists of
18 EU member states that have joined forces to address the issue of fragmented research in this area.[41] Their strategic research agenda is currently
being consulted, but the first area topic in the consultation document is the development of novel antibiotics and novel alternatives to antibiotics.

In

the United States, there is active lobbying to seek legislation based on the concept of a new
regulatory approval pathway for limited population antibacterial drugs, which would
encourage the development of antibiotics that address the greatest unmet needs of patients by making the approval
pathway more feasible.[42] The U.S. BARDA,[43] within the Office of the Assistant Secretary for Preparedness and Response, develops and procures
medical countermeasures against a broad array of public health threats (natural and intentional in origin), including vaccines, therapeutics, diagnostics,
and nonpharmaceutical countermeasures. BARDA has recently funded a collaboration with GlaxoSmithKline (GSK) to develop several antibiotics,
taking a novel approach that allows for funding to move around GSK's antibacterial portfolio and for medicines to be studied for the potential treatment
of both conventional and biothreat pathogens, that is, to develop a response to the potential threats from both antimicrobial resistance and bioterrorism.
[44] While

it remains to be seen if these initiatives do address the issue of market failure ,


(which will be the true test of whether the alarm is working), it can certainly be said that the alarm has led to
unprecedented action .

Baudrillard

AT: Baudrillard/Info Overload


Politics is possible---debate solves info overload
Bleiker 2
(Roland, professor of international relations at the University of Queensland, Politics After
Seattle: Dilemmas of the Anti-Globalisation Movement, conflits.revues.org/1057)
Any protest action that draws sufficient media attention has the potential to engender a
political process that transcends its immediate spatial environment. It competes for the attention of
global television audiences and thus interferes with the struggle over values that ultimately shapes the world we live in. "A world united by Benetton

fibre-optic cables
and shared cultural references are nonetheless laying the foundations for the first truly
international people's movement.31 But the recent wave of global protests is hardly the first
international movement of its kind. Nor is it as unproblematic as Klein suggests. For some the revolution of
speed is too random to allow for critical interference and, indeed, for human agency. Jean
Baudrillard, for instance, believes that the distinctions between reality and virtuality, political
practice and simulation are blurred to the extent that they are no longer recognisable.32 Our
slogans, Nike sweatshops and McDonald's jobs might not be anyone's utopian global village," says Naomi Klein, "but its

media culture, he says, has annihilated reality in stages, such that in the end its simulating image bears no relation to any reality whatever : it is its
own pure simulacrum. Television, the unproblematic transmission of the hyperreal, has conditioned our mind such that we have lost the ability to
penetrate beneath the manifest levels of surface.33 24

Patterns of global protest do not confirm the

pessimistic views that Baudrillard and others espouse. The blurring of reality and
virtuality has not annihilated dissent . The fact that televised images are hyperreal does
not necessarily diminish their influence. Independently of how instantaneous, distorted
and simulated images of a protest action may be, they still influence our perceptions of
issues, and thus also our political responses to them. To accept the logic of speed, then, is
not to render political influence obsolete, but to acknowledge multiple and overlapping
spatial and temporal spheres within which political practices are constantly being
shaped and reshaped.

2ac ballot
The plans method a particular response is a useful template for solving
transnational problems, without prescribing a formula for all action
outweighs co-option risks
Daniel Bray, The University of Melbourne International Politics and Political Theory Lecturer,
Ph.D., 2009, Pragmatic Cosmopolitanism: A Deweyan Approach to Democracy beyond the
Nation-State, Millennium, 37.3
In keeping with pragmatist tenets, my approach to the problem of practical institutionalisation is a
context-sensitive one that does not impose an a priori direct or representative template for the
political institutionalisation of democracy, which may require more or less degrees of
mediation and delegation depending on the particular problematic situation. As Saward points out, different democratic
mechanisms may be called for depending on whether they require permanent structures or
temporary measures and on whether they are undertaken by governments or by nongovernmental actors.103 Thus, one may have a normative preference for minimising the degree of
mediation and delegation in any democratic regime, but these features of democratic practice should not be viewed as
inherent bads. Indeed, given the scale and complexity of modern politics, transnational
democratic publics are likely to require significant levels of mediation and delegation if they are
to be effective actors in global politics. The key response of pragmatic cosmopolitanism here is to see
transnational publics as institutions of critical inquiry that are formed when associated individuals
work collectively to address problems presented by transnational consequences. As Molly
Cochran points out, in Deweyan terms these publics exist on a continuum from weak publics that are understood to involve associated activity that is only informally organised
(like a neighbourhood group) or narrowly focused on a single issue (like the control and prevention of AIDS104), to stronger publics that have political agencies invested with
public authority that are capable of issuing binding decisions for a societal group (an international regime, say, or a more densely articulated public we commonly regard as a

Today, in the absence of adequately responsive global institutions, most


transnational associations take contestation rather than popular control as their fundamental political
purpose.106 When they seek to provide alternative sites for deliberation where dominators
are not present they form what Nancy Fraser calls subaltern counterpublics that attempt to indirectly influence formal
organisations by mobilising broader public opinion.107 These oppositional networks and movements can only be
conceived as transnational public institutions, however, when their efforts are directed at shifting authority away
from states and their agents by making their own concerns authoritative in the decisionmaking
that takes place where international public authority exists in global politics international law,
regimes, the United Nations and in the broader bilateral and multilateral relations between states.108 In the view of pragmatic
cosmopolitanism, this process constitutes the core dynamic of democratic
reconstruction in international institutions. It is this perspective that highlights the key difference between the pragmatist and
deliberative approaches to the institutionalisation of transnational democracy. Instead of seeing publics as constituted by
responsible citizens who reason publicly on the basis of a distinctive form of communication, pragmatists see
responsible action as emerging from publics constituted by persons who recognise a
need for social cooperation in resolving common problematic situations. In the pragmatist view, publics
political state).105

are developing the traits of a state when they develop strong organisational and decision-making capabilities and seek to make their concerns authoritative in global politics.

Pragmatism therefore does not hold to the strict statecivil society separation that fundamentally
shapes the deliberative approach. Beyond nation-states with sharply defined constitutional structures, the desire to maintain a strict
separation between opinion-formation in the public sphere and will-formation in formal
representative institutions seems to neglect the requirement for some kind of

connective tissue between them , or at least assumes that the translation of opinions into
decision-making will occur through an underlying discursive shift that changes the context in
which formal decisions are reached. Deliberative democrats thus privilege informal procedures of
truth-seeking (that are never power-free or completely non-strategic) over political voice in formal institutions. Dryzek is
obviously concerned about the co-option of oppositional civil society which is certainly an ever-present threat
and one to be taken seriously but in many global and transnational contexts this threat tends to be overstated
and fails to acknowledge the strategic character of publics themselves. As Cochran points out,
despite the blurring of the statesociety divide, co-option is unlikely to be ever fixed or
complete.109 Additionally, not all weak publics seek permanent or even minimal levels of inclusion in existing formal institutions, preferring to focus on contestation or
developing alternative forums. Ultimately, pragmatic cosmopolitanism argues that in many contexts of
contemporary global politics the need to realise change through access to formal
decision-making outweighs the risk of co-option . One such context I discussed earlier centres on the contemporary climate
nongovernmental actors attempt to influence states in formal institutions by
channelling up the concerns of excluded publics, while at the same time monitoring the
negotiations and channelling down information to broader transnational and domestic constituencies.110
change negotiations, where a wide variety of

at: info saturation


Fawver, 8
[Kurt, Master of Arts Engilsh Cleveland State University, DESTRUCTION IN SEARCH OF
HOPE: BAUDRILLARD, SIMULATION, AND CHUCK PALAHNIUKS CHOKE, August 2008,
http://etd.ohiolink.edu/send-pdf.cgi/Fawver%20Kurt%20D.pdf?csu1219269969]
If Palahniuks Choke was merely an excellent resource for understanding Baudrillardian theory, it would still be a valuable text. As it stands, however, Choke expands on the
ideas of simulation and mediation and struggles to free itself from the snares of Baudrillards ultimate unreality. Through a regime of breakdown and disorder, the text fights to
emerge from the end or disappearance of the real, the social, history, and other key features of modernity (Best 133). It attempts to create a meaningful correspondence

While Baudrillard posits that everything can and has


been done, and all we can do is to assemble the pieces of our culture and proceed to
its extremities, Choke resists such reasoning and, in fact, runs through stages of assembly and
extremism to demonstrate how utterly futile and pointless they are (Best 137). Choke seeks to blow apart those
very reproductions that Baudrillard claims cause the implosion of meaning. Essentially, the text advocates a clean sweep of
communication, a discarding of all mediated reality. In Choke, as in many other Palahniuk novels, the flow of
true meaning can only return to society and individuals once all mediated, simulated,
reproduced meanings are razed. Thus, the text does glorify destruction, but it is destruction in search of hope, destruction that will,
between signifiers and signifieds, between images and meanings.

presumably, lead to creation. Victors eventual identity collapse, and his subsequent rebuilding, is paradigmatic of Chokes anti-Baudrillardian philosophy. Victor begins by
compiling the persona of a dysfunctional, perpetually orphaned child-cum-adult from mediated symbols of dysfunction. His sex addiction and his compulsion to simulate
choking in restaurants are symptoms of this poor attempt at constructing a workable identity. When the traumatized child-now-in-adulthood simulation fails, Victor turns to new
mediated identities: Christ and Antichrist. These personas also lack any depth or connection to Victors core being and are, subsequently, discarded. As the text progresses,
Victor drops all attempts at creating his identity from the palette of societys mass-produced conceptions. He pleads for someone to just show me one thing in this world that is
what youd think (Choke 205). But, as no inherent realness exists in contemporary society, no one can show Victor a thing or an individual with inherent meaning. Therefore, his
only option is to extricate himself from the culture of simulation by cutting himself off from his own history and other individuals mediated perceptions of his past. In a moment of

that he must reduce his identity to its simplest, most immediate terms
because Theres no way you can get the past right. You can pretend. You can delude yourself, but you cant re-create whats
clarity, Victor realizes

over (Choke 273). Thus, by the end of the novel, Victor is more a blank page than a fully fleshed character. Rather than continuing to allow his identity to be an ever-evolving
reactive simulation that forms in reference to external mediation, he becomes a clean slate on which he can write his own self-generated identity. He slakes off most of the
factors that traditionally 26 inform self; familial expectation, personal history, and even conventional emotion are all missing from his identity at the texts close. As Victor
explains, For the first time in longer than I can remember, I feel peaceful. Not happy. Not sad. Not anxious. Not horny. Just all the higher parts of my brain closing up shop.

His thoughts are of


an essentially basic order; he no longer seeks out deeper meanings or alternate
referentials. Instead, events, feelings, people, and things simply are what they appear to be,
without connection to external mediation. For Victor, the universe of multiple signified
meanings for any given signifier is no longer relevant. He has destroyed his perception of
alternate reference and, therefore, has limited his field of meaning to exclusively intrinsic values . Such perception
Im simplifying myself (Choke 282). The implication is that, in order to escape simulation, Victor must revert to a more primitive state.

comes at a price, however. Victor has to sacrifice a world of possibility, of variable signification, for concrete meaning. He can no longer ponder whether an image means one
thing or another; rather, an image will, to Victor, always be fixed to one referent. In a sense, then, he has given up the parts of his higher brain, namely a rigorous intellect and
boundless creativity, in order to gain a foothold into solid reality and flee Baudrillards infinite simulatory spiral. Whereas Baudrillard critiques representational thought which is

choosing a
path of selfimposed communicative primitivism is the only measure of prevention
against accruing a new body of simulacra. The polar opposite of Victor is Tracy, the woman to whom he
loses his virginity. She is the prime example of an individual forever lost in Baudrillardian pos t27
structuralism, representing everything that Victor, or any person, may become when nihilistic acceptance of simulation has infiltrated every aspect of self. Victor
confident that it is describing reality as it is, Victor embraces such thoughts with open arms (Best 140). Victor is intelligent enough to understand that

meets her on an airplane, in an unlocked bathroom. She takes flights, enters the restroom, leaves the door unlocked, then waits until someone walks in on her and attempts to
engage them in a sexual encounter. When Victor questions her aberrant behavior, she replies that the answer is there is no answer when you think about it, theres no good
reason to do anything. There is no point people dont want an orgasm as much as they just want to forget. Everything. (Choke 256-7). Clearly, life in the Baudrillardian void
has taken its toll on this woman. Tracy ponders Why do I do anything? Im educated enough to talk myself out of any plan. To deconstruct any fantasy. Explain away any
goal. Im so smart I can negate any dream. (Choke 257). She is the essence of Baudrillards postconstructionist theory; in her, the text introduces an embodiment of hyperintellectualism that has cut away all the joy, fulfillment, and meaning from life and reality and, subsequently, sees only a vacuum underlying all existence. Her nihilism leads into

She wants to find meaning and


absolute reality but will always be forced, due to her intelligence and her deconstructive
ability, to undermine the very goal she is trying to achieve. For Tracy, meaning is impossible
not because it has objectively disappeared, but because she cannot accept simple truths
or non-multiplicitous signifiers. She thrives on the complexity of reality and, therefore, will never be satisfied by a simplistic interpretation, even if
a quest for extrication from the ultimate emptiness and, thus, works as the catalyst for her sexual addiction.

the simplistic interpretation is that for which she yearns. Through Tracys unsatisfied, perpetually-wandering nature, the text puts forth the implication that 28

maintaining such an unflinching post-constructionist mindset has no future other than


disappointment, dysfunction, and existential despair. Indeed, Choke implicitly attacks Baudrillards blas acceptance of
simulation and attempts to show the ramifications of such acceptance. Hence, while the critical perspective from which
Baudrillards theory stems is akin to a scalpel, cutting deeper and deeper into the body of
reality to reveal unending layers of nothingness, Choke advocates a return to a
bandaged surface; it strives toward the revitalization of easily accessible signifieds, and, thus,
shuns Tracys (see also Baudrillards) system of thought that only seeks to forever prove the disappearance of meaning. Therefore, the text is ultimately
moving beyond Baudrillard by emphasizing creation over destruction and promoting
the deemphasization of post-constructionist critical inquiry as a means of understanding
reality (Kavadlo 12). To further illustrate the resurrection of meaningful signifiers and images, the text introduces Denny, Victors best friend. Denny is a recovering sex
addict who, throughout the text, earnestly seeks rehabilitation. As sex addictions in Choke seem to be symptomatic of a
fatalistic surrender to the simulatory world, Denny is the one character who consistently
seeks out a means of resistance. Strangely enough, this resistance takes the form of thousands of
rocks. As the novel progresses, Denny builds an enormous rock collection and, with those rocks, embarks on the construction of a mystery structure in an empty field. He
enlists Victors help and, when a local reporter comes to interview Victor and Denny about the construction project, Victors responses are veiled in a haze of ignorance. Victor
recalls the dialogue between himself and the reporter, saying that she asked: This structure youre building, is it a house? And I say we dont know. 29 Is it a church of some
kind? We dont know. What are you building, then? We wont know until the very last rock is set. But when will that be? We dont know. (Choke 263-4). Victors reticence
with the reporter is not due to any particular stigma or grudge against the media. Rather, his unforthcoming answers are a result of a new (or perhaps ancient) mode of
perception and, thus, communication. Instead of focusing on the possibilities of the stone structure or its eventual outcome, Denny instructs Victor to focus on the process of
building, alone. He says that the longer we can keep building, the longer we can keep creating, the more will be possible. The longer we can tolerate being incomplete, the
better (Choke 264). Initially, this statement appears to echo Baudrillards sentiments, with a perpetual process of building that leads nowhere and creation that actually creates

By compelling the rock structure to remain a work-in-progress


without a definitive end, Denny has squashed any simulatory nature the building may
possess. He and Victor are not putting stones atop one another to create any of the longmediated structures of society. The stone building is not a house or a church or any
other structure of convention and, therefore, is not founded upon any previous referent.
Dennys rock building is not trying to simulate any other structure; it is simply allowed to rise and become whatever it
eventually becomes. With the stone structure, Denny is attempting to introduce a
product that holds inherent, unmediated meaning. As soon as Denny or Victor would
conclude that the building is a house or a church, then it would, necessarily, begin to
take on aspects of those structures. It would begin to simulate a house or a church. But,
by allowing the structure to grow almost organically, Denny has set the 30 groundwork
for a signifier that may finally be connected with an inherent meaning, with a concrete
undeniable reality. The price for cultivating an unmediated, unsimulatory reality is high, however. Both Denny and Victor must discard the realm of speculation
nothing. Yet, precisely the opposite is true.

and conjecture. In order to maintain a sense of the real, all possibility outside a things readily apparent meaning must vanish. Denny and Victor do not know what the stone
building will be because they dont want to know until it is finished. They choose a path of ignorance so that realness may reassert itself within the structure without being

Denny and Victor must become simple, single-minded


individuals who have no need for multiplicitous signs and no desire for a constant
outgrowth of discourse. Theirs is a reality that requires no mediation, no simulation, and,
hence, no emptiness. Such a lifestyle choice flies in the face of our contemporary world, where formulating variable meanings for signifiers and expanding
the possible field of referentials for images is second-nature. The very fiber of critical theory, or of practically any academic discipline,
hinges on increased speculation, on infinitely sprawling discourses, and on the complication of texts,
signifiers, and reality itself. Chokes solution for escaping Baudrillards simulation is to escape that
same incisively critical manner of thinking. In doing so, Denny and Victor become primitive postpostmodern men. The duo
crushed by external mediated reality. Basically,

simultaneously evolve and devolve communication; they usher reality back into a signifier but cause the collapse of complexity. Indeed, many of the seemingly random
transgressive acts perpetrated by the characters in Palahniuks fiction, such as Denny and Victors intentional ignorance, fall within an understanding of entropy as a force for
renewal and meaning (Sartain 32). Thus, while Denny may 31 have set society on a course for a neo-stone age, his rock structure may actually be something that simply is
what it is. Victors mother, Ida, is an individual who also manages to cut ties with simulation, but in a much different, and arguably more destructive, manner. Her perspective on
reality, like the neo-primitivism of Victor and Denny, strives to attain communion with a long-lost realness. However, Ida takes a much more direct and assertive approach. She
uses drugs to simplify her state of mind. As Ida explains, Trichloroethane All my extensive testing has shown this to be the best treatment for a dangerous excess of human
knowledge (Choke 148). She is attempting to clear away the debris of contemporary societys all-consuming media (and with it mediation and simulation) by chemically altering
her consciousness, thus allowing her to ignore its multiplicity of disembodied voices and images that would, otherwise, crush her unmediated, individual perception of reality. Ida
claims that she can see things as they truly are when she is on drugs. She says that the trichloroethane makes the world appear without the framework of language. Without
the cage of associations without looking through the lens of everything she knew was true (Choke 149). Through her druginduced highs, Ida is stripping away mediation
and, therefore, making simulation impossible. Without a vast body of mediated meanings to draw upon, Ida is forced to view the world as it actually is, in its simplest terms. She
has rid herself of simulation and allowed realness to seep back into images. However, the reality is fleeting and dissipates back into the cacophony of Baudrillards simulatory

universe as soon as Ida is clean once more. Even worse, the constant drug use takes its toll on Ida; over the course of the text, she ends up with a perpetual bloody nose and,
ultimately, is reduced to a 32 feeble, emaciated skeleton. Idea proves that, while escaping Baudrillards simulation may be possible in a number of ways, the return to reality can
come at an indescribably steep price. Ida is also critical to understanding Chokes postulation on the manner in which society may be galvanized into forsaking simulation. It is
Idas ideology of adventure, her belief in the restorative power of chaos [that] serves to unbalance comfortable homogeneity. She seeks to create meaning and potential for
change through random chaotic acts (Sartain 33). Ida vandalizes merchandise in stores, kidnaps children, and causes public disturbances all in the service of disrupting
complacent adherence to mediated reality. She knows that a fire alarm is never about a fire, anymore and tries to disseminate this knowledge across society, albeit obliquely

Ida challenges simulation by creating real panic and real excitement. Her
acts of destruction are aimed squarely at bringing a sense of reality back into a populace
that, normally, experiences events and emotions in a heavily mediated environment. Ida
and illegally (Choke 161).

causes people to feel true fear, to experience events that are precisely what they appear to be: actual, unsimulated danger. However, there is no proof that Idas regime of

For a brief moment, the victims of Idas


crimes may experience a true, unmediated, unsimulated event, but as soon as the danger
has been resolved, the contemporary culture of mass media creeps back in and
continues to suffocate with its hollow signifiers. Therefore, Idas attempts to empower society
may be entirely pointless. While her personal freedom from Baudrillards simulatory
world is assured, she cannot force others to choose the same path of informed,
intelligible ignorance. 33 Indeed, Idas failure to enact social change exhibits the textual implication that release from simulation
must begin in the most intensely personal and introspective realms and radiate outward.
Perhaps meaning can be reconnected with images, but, as Choke demonstrates, such reconnection must be instituted at the
individual level long before it can solidify into an absolute reality upon which everyone
agrees. If Chokes resolution to the Baudrillardian dilemma seems somewhat perfunctory or abrupt, it would be in keeping with the theoretical concerns of the text. In a
simulatory reality, where all information is produced and mediated to individuals at a
hyperkinetic speed, it would be logical for a solution or paradigmatic rebellion to arise
just as quickly, given that this solution would still, necessarily, have a point of
emergence within a system that is unable to slow the production of information, images,
and signifiers. Thus, the texts resolution an idea that works as a competing perception of reality appears as quickly and as suddenly as any other random
philosophy-based crime alters the perception or behavior of anyone but Victor over the long term.

image or information structure; the system of mindless, endless generation has unwittingly generated its own demise. That Choke ends without much exploration of its resolution

The text fights


despair and a defeated acceptance of missing reality with unabashed romanticism. With the
to simulatory reality is also reasonable, given that such an open-ended future is antithetical to the very principle of Baudrillardian nihilism.

novel ending shortly after the characters have lain in place their newfound adherence to knowing ignorance, the future is uncertain. Anything could happen to reality following

a reunion of images and meaning is as possible as the continuation of hollow


simulation. Victor and Dennys plan for identity-formation and reality-perception may lead to the eventual destruction of all simulacra or it may be entirely useless. The
reader is left in a state of 34 unknowing, of hope for meaning-filled future. Such a conclusion is impossible in a Baudrillardian
scheme of reality. Under Baudrillards critical eye, the world has reached a point where
struggle against the forces of simulation is impossible. In Baudrillardian theory, there is
no hope for the retrieval of meaning; rather, the process of simulacra will continue , unabated. In
the close of the text;

answer to this bleak nihilistic view, Choke presents an open space, an ending that is more the beginning of a competing discourse than a summation of all that has come before

There is no definite success at the end of the text, nor is there assured defeat. The
texts concluding indeterminacy, its allowance for hope, separates it from Baudrillards
nihilism and reinforces the supposition that escape from simulation is, in fact, possible.
it.

AT: plan = fear


Attributing causality to self-loathing is the worst 19th century biologism
Michael Ure 12 date inferred, political and social theory prof at Monash University,
Resentment/Ressentiment, http://www.academia.edu/2434176/Resentment_Ressentiment
Nietzsche conceives ressentiment as symptomatic of evolutionary
degeneration, a widespread physiological disease. For the sake of the species' health, he argues, we should eliminate the
On the other hand,

physiological disorders that underpin and explain the emergence of ressentiment If ressentiment is not eliminated, he claims, the species will continue its evolutionary
degeneration. From our vantage-point Smith's Enlightenment faith in the ultimate compatibility between humanity's natural constitution and its moral and political progress may
seem to carry too many traces of a now discredited faith in nature's purposiveness or teleology.24 Nietzsche famously identified himself as the first European thinker to properly

He aimed to conceive
humanity in strictly naturalistic terms. However, his alternative naturalism is bound to an
idiosyncratic version of nineteenth century Social Darwinism. In the past decade there has been a renaissance of
scholarly interest in Nietzsche's relationship to Darwinism and other contemporary evolutionary theories.25 This research has successfully
challenged Heidegger's attempt to purge Nietzsche's philosophy of its "alleged
biologism".26 In the recent debate most scholars agree that Nietzsche is a naturalist of one stripe or another, and the
main interest lies in identifying his particular shading of nineteenth century naturalism.27 In this context Nietzsche's concept of the will to power has
been conceived as neo Darwinian or as anti-Darwinian.2* I share the latter view that Nietzsche explicitly targeted
Darwin with his own purely speculative , if not blatantly fanciful biology. Against Darwin
Nietzsche held first that healthy biological types seek to maximize or expand their power
even at the cost of self-preservation and that only sick biological types seek to preserve
themselves; and secondly that the weak and sick preserve themselves far more
effectively than the powerful largely because they emasculate or moralise the latter and
in doing so prevent them from threatening their existence. Through the morality of
ressentiment the weak become parasites sapping the vitality of the strong. If Nietzsche's view is true,
de-deify nature, eliminating from his account of nature all teleological assumptions or explanations as metaphysical hangovers.

then Darwin's notion of nature as a struggle for existence is merely a symptom of distressed life rather than a scientific account of nature.29 The point I wish to draw attention to

this anti-Darwinian conception of nature only reinforced Nietzsche's idiosyncratic


and extreme version of the social Darwinist political view we should exploit the weak,
sick and dying so that higher types have the opportunity to squander themselves or
eliminate the weak if they threaten to impinge on these higher types' capacity for
here is that

maximal self-expression . Arguably, Nietzsche's physiological theories and explanations do


not de-deify nature, but demonise it with the "born misfit[s]", "cellar-rats" and "maggot' men of
ressentiment. These two philosophical traditions confront us with the problem of how to distinguish between resentment and ressentiment Are they different
shadings and evaluations of the same basic "unsocial passion" or are they concepts deriving from incommensurable conceptual orders? The primary aim of this paper is to
clarify the nature of a now widely deployed distinction between resentment and ressentiment Since most defenders of resentment find Nietzsche's physiologically-based
aristocratic radicalism disturbing and/or untenable they usually attempt to short-circuit any possible association between resentment and its Nietzschean incarnation,
ressentiment? Often with the aid of moral sentiment theory they applaud resentment as a 'moral' emotion and condemn ressentiment as morally pernicious. By the same token,
however, even its defenders concede that the 'virtue' of resentment may have the potential to become a pathological vice that undermines justice. The shadow of Nietzschean
ressentiment still hovers over moral defences of resentment. "Resentment" as Margaret Walker acknowledges "embodies a sense of iault that can be difficult to dislodge, and
one gripped by resentment may be far too disposed to find fault in others than to question whether his or her own resentment might be misplaced".31 Its defenders often reach
for 'diabolical' metaphors to express resentment's tendency to exceed the agent's volition and remain impervious to apologies and other symbolic reparations.33 Once in
resentment's grip we are sorely tempted to groundlessly blame others for our suffering and unjustly make them the objects of resentment. Resentment also carries the danger
that agents see themselves as victims and identify the good with powerlessness, weakness and incapacity. The resentful are in danger of entrapping themselves in a victimidentity and enviously spoiling all forms of good fortune and human flourishing.34 If we can no longer rest content with Smith's natural theology to justify our confidence that
ultimately resentment is a gift of providence that naturally tends towards justice and moral order, we need to carefully examine the social and psychological basis of these
socalled 'pathological' expressions of resentment. How can we explain, for example, the compelling desire to unjustly and irrationally blame others for one's suffering? Why does

We require an account of reactive emotions that does not hinge on


fall into Nietzsche's idiosyncratic and insupportable theory
of evolutionary degeneration. We do not want our defence of resentment to rest on a
hidden or implicit teleological assumptions or our rejection of ressentiment to be based
such ressentiment continue to haunt resentment?

Smith's natural theological faith in the moral sentiments or

on insupportable physiological theories of moral judgement and action. This paper addresses these issues
by examining Nietzsche's concept of ressentiment It suggests that under the rubric of ressentiment Nietzsche identifies two
distinct phenomena: a physiological disease that finds alleviation through the mechanism of
blaming others and a form of radical envy or envious hatred that deploys moral concepts
and judgments in order to spoil others' good fortune and happiness.35 Nietzsche's
physiology of ressentiment rests on untenable speculations and assumptions he draws
from his interpretation of contemporary theories of evolution. For the purposes of this essay Nietzsche's second
concept of envious hatred is more promising for understanding of how resentment can become politically toxic.

AT: Speed K
Reject their theory of acceleration we should pursue politics in spite of
speed affirming the power of the citizen to reclaim the political
McIvor,11
(David, Kettering Foundation, The Politics of Speed: Connolly, Wolin, and the Prospects for
Democratic Citizenship in an Accelerated Polity, Polity Vol. 43 No. 1)
On
Wolins reading, national politics is little more than a spectacle, and the citizens role
within that spectacle is often only as a rooter limited to choosing sides.88 Localities, on the
other hand, remain venues that promise robust participation. As individuals slowly develop the habits related to
In order to develop these habits, Wolin wants to direct attention away from the state and towards localities with their particularities, peculiarities, and irregularities.

participation interpreting and coming to know ones environment and its other inhabitants, its multiple histories and overlapping concernstheir very being changes.

Politicalness marks our capacity to develop . . . into beings who know and value what
it means to participate in and be responsible for the care and improvement of our
common and collective life.89 By nurturing this politicalness we begin to feel a tug of
loyalty towards a common reality that had not heretofore existed. Wolin, in describing the early stages of the Free Speech Movement,

referred to this experience as the revival of a sense of shared destiny, of some common fate which can bind us into a people we have never been.90 Of course, these

Publics rise and fall; democratic moments


remain momentary. Yet those who are honed by these experiences and who are
dedicated to their recovery become what Wolin calls a multiple civic self . . . one who is
required to act the citizen in diverse settings: national, state, city or town, neighborhood,
and voluntary association.91 This is perhaps the most complex conception of citizenship ever devised yet we have no coherent conception of
assemblages are subject to the same thousand natural shocks to which all flesh is heir.

it.91 The multiple civic self is not modeled along republican or representative lines, which reduce participation to occasional ratification or refusal, and which filter popular power

The complexities
of what Wolin calls the megastate and the sheer size of the United States exceed what
an Athens-styled radical democracy could manage. The multiple civic self is one capable
of participating not simply in his/ her locality but intellectually and passionately in the
controversies surrounding the megastate in order to reclaim public space and insist
upon widened debate.92 Wolin is not (only) a localist. Rather, he thinks that the skills and habits best
acquired by consistent participation in our particular localities lay the groundwork for a
form of citizenship attuned to the plural layers of political action and struggle in latemodern America. Moreover, the multiple civic self promotes the dispersal of power between local, state, and national bodies.93 Such diffusion re-establishes a
separation of powers that forces slow-time negotiations upon the impatient megastate.94 The slowly developed habits of
participation make possible a more robust form of democratic citizenship and, perhaps,
fugitive democratic moments. These moments, in turn, can help to slow the world down. Political theorists and social
actors inspired by Wolins example and worried about the inegalitarian consequences of
social acceleration should look to start from his (so far underdeveloped) idea of the multiple civic self. Instead of refurbishing
federal institutions or romanticizing the consequences of speed, we ought to attend
primarily to what Wolin calls the recurrent aspiration of democracy : to find room in which
through elite-managed institutions. Nor, however, is it based on the radical democratic conception of citizenship as direct sharing in power.

people can join freely with others to take responsibility for solving their common
problems and thereby sharing the modest fate that is the lot of all mortals. 95 By pursuing
solutions to mutual problems through concerted action, we as citizens can hone the

craft of democratic participationbroadening our notions of self and learning to honor


the differences we encounter within a shared space. 96

at: politics is dead / depressing uq claim


Politics is still possible, they are just too stubborn to see it
Feit 12
(Mario, Assistant Professor of Political Science at Georgia State University, Wolin, Time, and
the Democratic Temperament, Theory & Event Volume 15, Issue 4, 2012)
Wolin asks an important question, namely, he invites us to think about the
temporality of democracy and political theory. However, his question emerges in response
if not in reactionto changing social facts, namely the faster pace of contemporary culture, economy, and perhaps even
theory. Thus, Wolin proceeds from a negation, and is, as shall become clearer, delimited by
this origin. Instead, I propose that the temporal orientations of democratic politics and
political theory are better understood if we pursue the following questions: Why and how is
democracy patient or impatient? Can and should political theory replicate democratic
impatience? These questions are, I think, implicit in Wolins essay on social acceleration. Wolin himself does not raise them, due to his focus
But what if we reframed Wolins problematic?4

on repudiating social acceleration and because his approach to political time does not distinguish between the worlds pacewhich may be expressed
with the language of slowness or speedand our temporal expectations about this world. This essays conceptual language of patience and
impatience is better suited to describing temporal expectations about the possibilities for political action. Indeed ,

to make patience and


impatience front and center provides an opportunity to theorize democratic temporality
in a positive fashion . Furthermore, this conceptual shift opens up Wolins overall approach to
democracy to considerations of democratic temporality. In fact, I argue that Wolins conception of the demotic
offers a democratic temporality that is fundamentally in tension with the one Wolin
contemplates in his rejection of social acceleration. For patience emerges as an implicit temporal virtue in What
Time Is It? and impatience as the corresponding vice, nurtured in part by social acceleration.5 I show that the demotic, on the other hand, would
recommend impatience as a democratic virtue, that is, the demotic entails speeding up, not slowing down, politics. Returning to Wolins problematic,

social acceleration may in fact reshape contemporary politics in part because it is


alluring to democratic citizens. Social acceleration may partially bring democracy in
temporal sync with itself, even as it may complicate democratic politics in some of the ways sketched by Wolin.6 In recuperating
democratic impatience, my argument disentangles the close temporal fit that Wolin posits between political theory and democracy. Patience may very

his
thinking about political temporality initially does not approach its subject matter with
sufficient patience. He only develops the necessary patience as a result of interlocutors,
who put critical pressure on his ideas. In other words, in part my argument illustrates that deliberative
patience is still possible for political theory in the present , and that Wolin eventually
displays such patience in his thinking about political temporality. Still, I also detect an impatience in parts
well be critical for political theoryand philosophy, overall, as Wittgenstein might add. Wolin calls for such patience. That said, I show that

of Wolins argument, which is quite distinct from his rashness in his critique of social acceleration. I argue that Wolins frustration with the Iraq war
reflects a bubbling up of valuable demotic impatience. In other words, I distinguish more closely than Wolin between necessary theoretical patience,
troubling theoretical impatience and beneficial demotic impatience that manifests itself within his political thinking.

Everything is awesome
Quality of life is skyrocketing worldwide by all measures
Ridley, visiting professor at Cold Spring Harbor Laboratory, former science editor of The
Economist, and award-winning science writer, 2010
(Matt, The Rational Optimist, pg. 13-15)
Since 1800, the population of the world has
multiplied six times, yet average life expectancy has more than doubled and real income
has risen more than nine times. Taking a shorter perspective, in 2005, compared with 1955, the
average human being on Planet Earth earned nearly three times as much money (corrected for
inflation), ate one-third more calories of food, buried one-third as many of her children and
could expect to live one-third longer. She was less likely to die as a result of war, murder,
childbirth, accidents, tornadoes, flooding, famine, whooping cough, tuberculosis, malaria,
diphtheria, typhus, typhoid, measles, smallpox, scurvy or polio. She was less likely, at any given
age, to get cancer, heart disease or stroke. She was more likely to be literate and to have
finished school. She was more likely to own a telephone, a flush toilet, a refrigerator and a
bicycle. All this during a half-century when the world population has more than doubled,
so that far from being rationed by population pressure, the goods and services available
to the people of the world have expanded. It is, by any standard, an astonishing human achievement. Averages
conceal a lot. But even if you break down the world into bits, it is hard to find any region
that was worse off in 2005 than it was in 1955. Over that half-century, real income per head ended a little lower in only
If my fictional family is not to your taste, perhaps you prefer statistics.

six countries (Afghanistan, Haiti, Congo, Liberia, Sierra Leone and Somalia), life expectancy in three (Russia, Swaziland and Zimbabwe), and infant
survival in none. In the rest they have rocketed upward. Africas rate of improvement has been distressingly slow and patchy compared with the rest of
the world, and many southern African countries saw life expectancy plunge in the 1990s as the AIDS epidemic took hold (before recovering in recent
years). There were also moments in the half-century when you could have caught countries in episodes of dreadful deterioration of living standards or
life chances China in the 1960s, Cambodia in the 1970s, Ethiopia in the 1980s, Rwanda in the 1990s, Congo in the 2000s, North Korea throughout.

overall, after fifty years, the outcome for the world is


positive. The average South Korean lives twenty-six more years and earns fifteen times as much
income each year as he did in 1955 (and earns fifteen times as much as his North Korean counter part). The average Mexican lives
longer now than the average Briton did in 1955. The average Botswanan earns more than
the average Finn did in 1955. Infant mortality is lower today in Nepal than it was in Italy in
1951. The proportion of Vietnamese living on less than $2 a day has dropped from 90 per
cent to 30 per cent in twenty years. The rich have got richer, but the poor have done even
better. The poor in the developing world grew their consumption twice as fast as the
world as a whole between 1980 and 2000. The Chinese are ten times as rich, one-third as fecund and twenty-eight years
Argentina had a disappointingly stagnant twentieth century. But
remarkably, astonishingly, dramatically

longer-lived than they were fifty years ago. Even Nigerians are twice as rich, 25 per cent less fecund and nine years longer-lived than they were in
1955. Despite

a doubling of the world population, even the raw number of people living in
absolute poverty (defined as less than a 1985 dollar a day) has fallen since the 1950s. The percentage
living in such absolute poverty has dropped by more than half to less than 18 per cent. That
number is, of course, still all too horribly high, but the trend is hardly a cause for despair: at the current rate of
decline, it would hit zero around 2035 though it probably wont. The United Nations estimates that poverty was reduced
more in the last fifty years than in the previous 500.

Death & Suffering Ks

AT: Relation to Death


they cant just wish this away modernity or how we relate to death --multiplying the ways to relate to death makes people freak out more.
Mellor, 1992
(Phillip, Leeds Professor of Religion and Social Theory Head of School, Death in high
modernity: the contemporary presence and absence of death, The Sociological Review, May,
ebsco)
The fundamentally discontinuist impulse of modernity is expressed in high modernity in
the pervasiveness of 'reflexivity', the systematic and critical examination, monitoring and
revision of all beliefs and practices in the light of changing circumstances (Giddens, 1990, 1991). Helmuth
Schelsky created the term Dauerreflexion, meaning 'permanent reflection', to characterise the continuous examination of meaning and values which he observed in the modern consciousness (Schelsky 1965).

reflexivity, is
chronic: it is a never-ending process of systematic and potentially radical reappraisals
and reassessments of all aspects of modem life, calling everything into question, and
undermining any certainty of knowledge so that it can be said that a major feature of high
modernity is the pervasiveness of 'radical doubt' (Giddens, 1991:21). Giddens argues that 'We are
Giddens's usage of the term 'reflexivity' is consistent with Schelsky's earlier conceptualisation of the systematically-questioning character of modernity. Dauerreflexion, or

abroad in a world which is thoroughly constituted through reflexively applied


knowledge, but where at the same time we can never be sure that any given element of
that knowledge will not be revised .' (Giddens, 1990:39). Gehlen has also written of the constant scrutiny of, and reflection on, motives for action which are no
longer located in transpersonal traditions, but in the isolated individual consciousness (Gehlen, 1956, 1957), signaling the same tendency towards insecurity which is at the heart of modernity. Despite
Gehlen's generally pessimistic interpretation of the implications of modem reflexivity for
the continuance of the social order as a whole, and for the psychological stability of the
individual, a more positive interpretation is possible. A person's experience of this
pervasive reflexivity, and its attendent risks, can be profoundly liberating and empowering,
since there are considerable areas of opportunity available to herlhim which would not have been so in the past. Lasch, for example, discusses the ways in which self-development programmes in the United

Lasch offers a
characteristically bleak interpretation of these activities, which he relates to the
'narcissistic' personality type (Mellor and Shilling, 1993), but it must be acknowledged
that the people experiencing such changes in their lives may often find them profoundly
empowering, and are often no doubt prepared to accept the so-called 'loss of community'
which Lasch, and others such as Sennett (1974), envisage as the price for such empowennent of the individual. Nevertheless, it is hard to see how this
reflexivity can ultimately help individuals deal with the phenomenon of death, since this
is a universal parameter within which reflexivity occurs, rather than an object to which reflexivity can be convincingly applied.
Although it could be argued that modem societies are culturally diverse, and exhibit a
degree of flexibility which allows people to draw upon a variety of cultural resources in order to deal with death , it could also be argued that
States have encouraged older people to cut their ties with the past and embark upon new marriages, new careers and new hobbies (Lasch, 1991:214).

this diversity compounds the difficulties individuals experience when death is


encountered: reflexivity may be increasingly applied to death in a multitude of ways , but
this multiplicity of particular approaches to death accentuates the reality-threatening
potential of death in general , i.e. as an unavoidable biological constraint upon various attempts at its cultural containment. The more diverse
are the approaches to death in modern societies, the more difficult it becomes to

contain it within a communally-accepted framework, and thus limit the existential


anxiety it potentially offers to the individual . The apparent cultural diversity and flexibility in modern approaches to death can therefore be explained as
This difficulty of reconciling the dominant
reflexivity of high modernity with the brute fact of human finitude is particularly apparent
if we relate them both to issues centred on questions of self-identity. These issues are of central significance in
being consistent with the sequestration of death from public space into the realm of the personal.4

contemporary society because modernity has purchased increased control over life at the expense of communally constructed values (Weber, 1948), values being sequestrated from public space into the fragile

The concern for issues of self-identity apparent in high


modernity is the result of individuals, left alone to construct meaning in their lives,
searching for meaning through the creation of a viable and stable sense of self (Sennett, 1974; Rose, 1989).
Nevertheless, self-identity is also subject to the pervasive reflexivity of high modernity,
so that it is created and maintained through the continual reflexive reordering of selfnarratives (Giddens, 1991). Berger argues that 'the individual's own biography is objectively real only insofar as it may be comprehended within the structures of the social world' (Berger, 1967: 13),
sphere of the private (Gehlen, 1956, 1957; Bauman, 1989; Giddens, 1991).

but he prioritises the idea that society assigns an identity to the individual. As Abercrombie notes, those critics who accuse Berger of neglecting social structure are wrong, since the conversation between the

In high modernity, individuals must create their own


identities, drawing upon the reflexive mechanisms and socio-cultural resources available
to them, but ultimately having to take individual responsibility for the construction of
meaning as well as the construction of identity. In this context, death is particularly
individual and social structure is entirely one-sided (Abercrombie, 1986:30).

disturbing because it signals a threatened 'irreality' of the self-projects which modernity


encourages individuals to embark upon, an ultimate absence of meaning, the presence
of death bringing home to them the existential isolation of the individual in high
modernity.

This idea of the privatisation of meaning, and the difficulties it creates for modem individuals attempting to deal with death, is borne out by a consideration of what are generally

acknowledged to be the considerable changes in how people in Europe and North America have approached the phenomenon of death (Aries, 1974, 1981; Vouvelle, 1980; Turner, 1991). Death has gradually
been removed from public space, where it was contained in communal, religious beliefs and practices (Aries, 1981), into the seclusion of the hospital (Illich, 1976; Huntington and Metcalf, 1979; Elias, 1985),
where it has become a technical matter for medical professionals (Glaser and Strauss, 1968; Giddens, 1991). Funeral rites have similarly ceased to be a concern for the community as a whole, becoming a private
matter for the family and friends of the dead person, again organised by a professional group of funeral specialists (Huntington and Metcalf, 1979). The reflexive deconstruction of communal frameworks for the
containment of death has been so extensive that even when persons find themselves at funerals they are often unsure how to act or speak, because prescribed rites of mourning are often no longer available to
them (Turner, 1991). As Elias expresses it, 'The task of finding the right word or the right gesture therefore falls back on the individual' (Elias, 1985:26). This point demonstrates how the sequestration of death
from public space makes its presence in the personal sphere potentially commanding and threatening. Individuals are likely to experience the tension between the public absence and private presence particularly
strongly when they find themselves alone with the task of not merely constructing meaning, but of even knowing how to act, when they are faced with the deaths of those they care about. Gehlen has coined the
term Hand/ungsver/ust, the decline of the capacity to act, which he associates with the retreat into subjectivism attendent upon the deconstruction of tradition. Together with this deconstruction there is an increase
in behavioural insecurity, psychological confusion, and a loss of the capacity to construct socially meaningful human reality (Gehlen, 1957; Zijderweld, 1986). The difficulties modem persons face when confronted

the aloneness individuals experience


when having to cope with the death of others, is perhaps even more intense when they
themselves begin to die. As a result of the reality threatening power of death, and the
increased vulnerability to this threat because of the reflexively-induced disorientations of
high modernity, modem persons are increasingly reluctant to come into close contact
with those who are dying (Elias, 1985). Since the maintenance of their self-identities is potentially undermined by the presence of death in others, this means that there is a
with death otTer particularly strong supports for this aspect of Gehlen's conceptualisation of modernity. Nevertheless,

tendency for all persons now to die in situations of unparalleled isolation. Death finds no easy, or generally accepted, place in the conceptions of reality generated by high modernity, and which individuals
appropriate in their reflexive constructions of self-identity. Consequently, when death becomes startlingly real in the people around them, their desires for self-preservation encourage them to shut themselves off
from those people who are dying; both spatially, sequestrating them in hospitals away from the public gaze, and emotionally, shunning physical contact and denying people the emotional interaction and support
just when they are likely to need these things most. Thus, Elias's conception of the loneliness of the dying, the way modem social conditions foster feelings of solitude in those who are dying (Elias, 1985:85), can
be extended to include a more general existential isolation encouraged by the reflexivity of modernity. The absence of death from public space makes its presence in private space an intense and potentially
threatening one.

Because meaning has been so privatised, any attempts to construct meaning

around death are now inherently fragile . Even in the case of the hospice movement, whose development has clearly been part of an attempt to counter
certain aspects of these trends, many of the features noted above are still evident. The hospice, like the hospital, remains an institutional expression of the modem desire to sequestrate death away from the
public gaze, and individuals in them are still subject to the technical expertise of the medical profession in much the same way, despite the religious patronage of many hospices. Similarly, the location of the dying

Death may be present


in the hospice in a way that it is not in the hospital, but its presence remains a
predominantly personally-located one. It is notable that the strategies for dying associated with the hospice movement, such as Kubler-Ross's (1970) various
'stages' in the psychological preparation for death, are directed towards individuals, encouraging them to construct some sort of individual awareness of the meaningfulness of their lives and deaths. T hey
therefore offer nothing to counteract the widespread privatisation of meaning in high
modernity which is the major source of many persons' contemporary difficulties in
dealing with death.
in a hospice rather than a hospital is unlikely to alter the discomfort of family and friends in the face of the reality of death which the dying temporarily embody.

Suffering kills Reflection


The aff turns the critiques impact and alternative our impacts create
intense suffering that kills reflective interpretations of death
White 12
(Richard White is associate professor of philosophy Creighton University "Levinas, the
Philosophy of Suffering, and the Ethics of Compassion" The Heythrop Journal Volume 53, Issue
1, pages 111123, Published online esept 27 2011, official publication date: January 2012,
Wiley)
Suffering includes the extremity of physical pain, as well as the emotional
anguish and spiritual despair which every individual is bound to experience at some point in her
What is suffering?

life. It has been suggested that

there is a significant difference between pain and suffering , since the first is

primarily physical while the latter is basically mental. As Eliot Deutsch comments: One has a pain or that is painful, but I am suffering. Where there is no ego there is

Against this, however, the language of physical experience is


often used to describe what Deutsch would regard as purely mental aspects of suffering: This is why we express emotional
suffering in physical terms; when we say that we are tortured by guilt, or burning with shame; or our heart aches because of something that
no suffering although there might be pain.[9]

happened. Indeed, it would not be surprising if every form of suffering, including those which are primarily spiritual or emotional had a physical correlate in the body itself

it may be possible to
understand the nature of suffering by focusing on physical pain as its most direct and
fear is both physical and mental, for example, and depression always has a somatic aspect. The upshot of all this is that

unmediated form. In suffering, we experience the limits of self-assertion , and the


most extreme form of this is physical anguish, in which the self is rendered passive and
impotent by the torment that ruins it as a subject. Herbert Fingarette puts this point succinctly when he notes that: To suffer is to be
compelled to endure , undergo, and experience the humbled will, rather than to be able to impose one's will.[10] This means that
the experience of suffering is the opposite of self-assertion and is shot through with
the will's experience of impotence and limitation. Something like this is also the starting-point for Levinas's own account of
what it is to suffer. In the course of several books and numerous articles, Emmanuel Levinas sketches the outlines of a phenomenology of suffering. Suffering is not always
a central concern of his philosophy, but it is possible to reconstruct his basic view of suffering by examining comments drawn from several different texts. In Time and the
Other, for example, Levinas announces that he will focus his remarks on the pain lightly called physical, for in it engagement in existence is without any equivocation.[11]
Once again, the point here is that physical suffering is the purest form of suffering since it completely overwhelms the sovereignty of the self and as such it is an
experience without mediation. As Steven Tudor notes in his account of compassion and remorse:

Many physical pains intrude so

forcefully into one's consciousness that they impose their own significance which

no

stoic attitude can alter that significance possibly being a pure sense of raging
chaos that obliterates all other matters of significance, so rupturing, so consuming is
the pain .[12] Levinas also notes that in spiritual suffering it is still possible to preserve an attitude of dignity and distance from whatever affects one, and in this
respect one remains independent and free. Indeed, it can be argued that spiritual suffering is itself a kind of luxury that can only exist for as long as we are not disturbed
by physical pain. As Levinas comments, from one perspective (which he refers to as socialism), solitude and its anxieties are an ostrichlike position in a world that solicits
solidarity and lucidity; they are epiphenomena phenomena of luxury or waste of a period of social transformation, the senseless dream of an eccentric individual, a
luxation in the collective body.[13] By contrast,

physical suffering in its most extreme form effaces

subjectivity and all subjective attitudes . For Levinas, physical suffering involves the irremissibility of being and the absence of all
refuge; in such pain we are backed up against being with no possibility of escape, and for this reason it provides the clearest, most unambiguous model for suffering in
general. As Levinas notes,

significant suffering corrodes all the structures of meaning that we

project into the world; it overwhelms all virility or the effort to be masters of our
own fate until finally one is reduced to a state resembling helpless infancy: Where suffering

attains its purity, where there is no longer anything between us and it, the supreme responsibility of this extreme assumption turns into supreme irresponsibility, into

Sobbing is this, and precisely through this it announces death. To die is to return
to this state of irresponsibility, to be the infantile shaking of sobbing .[14] In her own account of torture
in The Body in Pain, Elaine Scarry confirms this point, when she argues that more than resisting language, suffering and
pain actively destroy language and all other meaningful projects, so that the subject reverts to a state
anterior to language, to the sounds and cries a human being makes before language is learned.[15] In this way, suffering is worldinfancy.

destroying . Indeed, to suffer greatly is to have one's world reduced to the content of one's pain. In the passage cited above, Levinas notes a connection
between suffering and death. According to Levinas, the one announces the other: There is not only the feeling and the knowledge that suffering can end in death. Pain of
itself includes it like a paroxysm, as if there were something about to be produced even more rending than suffering, as if despite the entire absence of a dimension of
withdrawal that constitutes suffering, it still had some free space for an event, as if it must still get uneasy about something, as if we were on the verge of an event beyond
what is revealed to the end in suffering.[16] Extreme suffering involves complete passivity. In suffering we are subject to something which does not come from ourselves
and which tends to undermine all meaningful structures of subjectivity. In this respect, suffering is the anticipation of death as the encounter with something that cannot be
avoided or held at arm's length. Both suffering and death involve the end of mastery, and with each, the contents of consciousness are destroyed. In his collection of
essays, At the Mind's Limits, Jean Amery, who was tortured by the Nazis, also seeks to articulate the strong sense of a connection between acute physical suffering and
death. Speculating on the meaning of his own experience, he comments that Pain is the most extreme intensification imaginable of our bodily being. But maybe it is
even more, that is: death. No road that can be travelled by logic leads us to death , but perhaps the thought is permissible that through pain a path of feeling and
premonition can be paved to it for us. In the end, we would be faced with the equation: Body = Pain = Death, and in our case this could be reduced to the hypothesis that

In extreme
physical suffering, such as the torment that Amery describes, the individual becomes purely a body, and
nothing else besides that. For as long as it continues there is no space for reflection ; and this violent
torture, through which we are turned into body by the other, blots out the contradiction of death and allows us to experience it personally.[17]

reduction to physical being is the most intense form of negation which seems to parallel the negation of
death. Elaine Scarry agrees:

death and suffering are the purest expressions of the anti-human, of

annihilation, of total aversiveness , though one is an absence and the other a felt
presence, one occurring in the cessation of sentience, the other expressing itself in
grotesque overload.[18]

Death = Social Meaning


Death has social meaning nowthey over generalize-prefer this evidence
because she actually talked to people for 10 years unlike Baudrillard and
Bataille who never spoke to another person.
Bradbury, 1999
(Mary, London School of Economics social psychology PhD, Representations of Death : A
Social Psychological Perspective, 189-197)
It is commonly accepted that contemporary death practices are perfunctory, empty and
without old-style customary and ritual power. I disagree. It is simply that we fail to see
where our customs and death rituals are flourishing . It appears that we are blind to our
own cultural ingenuity, while remaining locked into a collective nostalgia . The following acts
are significant social customs in contemporary Loudon: certifying and registering the
cause of death, the viewing and embalming of the corpse, placing notices in papers, the
funeral cortege, the cremation ceremony (when non-religious), the use of cards and letters of
condolence and the sending of flowers. The following would appear to fit the description of ritual acts, although what makes a
ritual authentic relies on the meaning extracted by the participants, not on the objects
which are used as props: the funeral service, certain burial and cremation rites, some wakes and memorial services and personal rites centring on memorial
objects. For many people in Britain today, death rituals do have an existential dimension. Indeed, for a lucky
few the participation in ritual can afford the opportunity to transcend death altogether .
The structure and lay-out of the funeral parlour and the crematorium, the black hearse, the show-room full of coffins and the touching epithets to be found in the book of
remembrance all could be viewed as objectifications. expressed as part of the ritual process, of our representations of death as rebirth. However, as Moscovici (1984a) most
certainly did not have custom and ritual in mind when he described the processes of objectification and anchoring it is possible that another process is at work, such as
symbolization. Indeed, Joffe (1999) has noted that there is a great deal of overlap between objectification and symbohzation. Douglas (1966) states that, the modem Western
world does not utilize or display neat patterns of pollution fears as the fabric of our society is too complex. However, it is possible to identify strange systems of belief that cluster
around areas of ambiguity. These unfamiliar terrains are a source of ritual power. We have some less-than- fully rational feelings about the dead body, which is subject to strong

Precisely because of its decomposing and inert condition it is a singularly rich


source for symbols. The things that we do to the body, such as dressing it, embalming it,
en-coffining it, viewing it, cremating and/or burying it cannot be explained in purely
rationalist terms. Harnessing our fears of pollution we make the corpse express our
representations of life and death. So, contemporary mortuary practices cannot be
pollution taboos.

dismissed as some kind of ritual shadows. We all race questions of mortality, whatever
our culture. British representations of death are not only to be found in medical practice (science) or films, documentaries and newspapers (media), they also exist in
our death customs and rituals. These customary and ritual acts are replete with meaning. They express
our representations of death, serving as both vehicles of traditional representations and
the means of effecting change. Like those of other cultures, the customs and rituals of our society are almost invisible and. for many, taking part in
them can be a comfortable, almost mundane, experience. The purpose of all representations is to make something unfamiliar. or unfamiliarity itself familiar. (Moscovici 1984a:
25) The study of death would appear to provide an excellent opportunity for observing the processes whereby people make the unfamiliar familiar. What is unfamiliar about
death is relatively obvious: the inert state of the body of a relative or patient: the body's remorseless change from human corpse to rotting carcass; the strange and
unpredictable behavior of the grieving relative; the loss of family or friend or work roles. The list goes on. Almost everything about death seems unfamiliar. Probably this is true
for all cultures. However, in the industrialized West, unfamiliarity is increased by two further factors. First, as I mentioned in Chapter 6, large numbers of the urban population are
alienated from the natural world and do not often, if ever, see dying or dead animals. Second, the social organization of death is in the hands of a small group of professionals

The representations we fabricate - of a


scientific theory, a nation, an artefact, etc. - are always the result of a constant effort to
make usual and actual something which is unfamiliar or which gives us a feeling of
unfamilianty. And through them we overcome it and integrate it into our mental and
who guard their activities as jealously as the members of a medieval guild ever did.

physical world which is thus enriched and transformed. (Moscovici 1984a: 27) Moscovici also considered those things that
are 'like us, and yet not like us*, such as the mentally ill and those from another culture. Moscovici suggested that those things that fail to become familiar are pushed aside and
are likely to be endowed with imaginary, often negative- characteristics. This is typical of the 'unfamiliar'. Thus, the corpse and those who deal with it are marginalized and
subject to negative and quite erroneous characterizations. Both the death work professionals and the next of kin are contaminated by their contact with the polluting corpse.
Moscovici (1984a) dismissed three other 'traditional* explanations of why we create representations, one of which is that groups create representations to filter information from
the environment and thus control individual behavior. While he acknowledges that mere may be a certain amount of truth in this, his dismissal of this extremely significant
explanation may have been over-hasty. After all, it would appear to be one of the implicit working hypotheses of many anthropological texts. For example, Bloch and Parry
(1982) argue that the ritual celebration of death may be used to further those in power. This is doing more than just make the unfamiliar familiar. This theme of control can be
seen to operate in modern society too. It is possible, for example, that the representation of the good medical death as painless was first created by doctors involved in a power
struggle with the clergy at death beds during the nineteenth century. Medical practitioners continue to be involved in the promotion of the good medical death. This is to do with
control, rather than with familiarization. As we would expect the dominant system into which the unfamiliar corpse and the recently bereaved are both integrated is that of the
medical model. The unfamiliar corpse is anchored in medical knowledge. The dead body is treated much like a patient, the bereaved like the mentally ill. The classification and
naming which is a sign mat anchoring has taken place is centred on medical descriptions of the cause of death. This in turn feeds into a discourse about whether the death was
good or bad. There are other systems of classification. We also make use of age-old concepts of the sacred good death and the rebirth of the dying. When Moscovici first
formulated the processes of anchoring and objectification he still had in mind the split between the reified world of science and the consensual world of common sense.
Objectification, in particular, was envisaged as the process whereby intellectual ideas are made more concrete and physically accessible. Fortunately, the process of
objectification still seems to make sense if we consensualize science. Once we have anchored a death into the context of medical science then we can objectify the death by
delaying decomposition. To give another example: if we wish to construct the image of the medically controlled death it is possible to make this a physical reality by the
application of fatal doses of opiates. Thus embalming and euthanasia become objectifications of our representations of the medical good death. Moscovici noted how we also
objectify by means of language. The use of euphemisms and various labels peculiar to the funeral trade appear to fit this description. Embalming, for example, becomes

While I set out to discover our current social representations of death, at the
outset I was not sure what form these representations would assume. However, the
anthropological and historical literature had provided me with some ideas, namely, that
societies tend to shape deaths into those that are good. bad. natural and unnatural. It
appears that Bloch and Parry's (1982) prediction that ideas of regeneration and rebirth
are more weakly stressed in an industrialized, individualistic society is correct. However,
we still need to anchor unfamiliar death and in this book I have tried to show how age-old
representations of the good. bad. natural and unnatural death continue to be utilized by
professionals and bereaved alike. During the eighteenth and nineteenth century representations of dying changed along with medical
'hygienic treatment*.

innovations. Dying was increasingly viewed as a process that could be made painless through the application of powerful painkillers. Willi the development of a secular climate
of opinion, however, not only did the manner of the good death change but also its meaning. In brief for many people it became profane. The good death was no longer
concerned with the fate of the soul; it was concerned only with the character of the physical event. The medical notion of good death continues to be the dominant
representation of good deaths and the hospital environment appears to provide the optimal degree of control. We can manipulate the lime, location and expression of death. So,
with the exception of self-sacrifice or murder, medical science provides the best conditions for the human control and manipulation of the process of dying. Bloch and Parry
(1982) suggest that the key to a good death is a sense of control. As it happens, the illusion of human control over life and death can be enjoyed from within the medical model
and deaths that require some kind of medical intervention and/or hospitalization seem good candidates for becoming good deaths, as I found to be the case. The cause of death
is of crucial importance to any type of good death because the type of disease has an impact on such factors as awareness, preparation and consciousness of the dying person.
Managing to orchestrate a medical good death has become the concern of medical practitioners. However, as I mentioned earlier, this does not just influence the behaviour of
medical personnel in the hospital, home or hospice. It has also influenced the behaviour of those deathwork professionals who deal with the corpse. The coroner, registrar and
funeral director are all involved in a medical discourse about death that is partly concerned with presenting the image of the painless, sleep-like death. Deaths that do not
conform to this stereotype - the bad medical deaths and particularly those that do not produce a reposeful corpse (Prior 1989: 160) - galvanize the death-work professional into
hiding or disguising the facts of the death (Bradbury 1996). This professional good practice is justified as being a way of protecting the bereaved. The professional's image of the
bereaved, whose grief is potentially 'pathological", is engendered by the application of a medical model to grief as well as to the medical good and bad deaths. In a reaction
against a perceived excess of medical intervention in the process of dying, the term 'natural death" has recently enjoyed a comeback. While coroners" courts have been using
the expression for some time, talk about dying 'naturally* was unfashionable during the modem era. With increasing longevity and changing death trajectories our conceptions of
a good way of dying are being revolutionized. Rejecting scenarios that seem depersonalizing and over-medicalized. people are now exploring 'new* ways of dying. From the
start the natural death movement cannily anchored itself in people's consciousness by affiliating itself with a very successful representation (and movement): the natural birth
movement. There are many differences between giving birth and dying and the affinity between the two does not stand close scrutiny. Yet the use of the title worked and
everyone, particularly the media, knew exactly what the movement's leaders were saying. Moscovici described two roles of representations. The first is that they 'conventualise
the objects, persons and events we encounter" (1984a: 7). The grouping of these objects, persons and events into models helps to structure an otherwise chaotic universe. So
death is conventualized by the representations we have formed about the dead body and the grieving relative. The second is that they are prescriptive, 'that is, they impose
themselves upon us with an irresistible force" (1984a: 9). Examples of the prescriptive nature of our representations of death abound. In our moment of deepest grief we warmly
thank the doctor and are meticulously careful to be compliant, polite and composed. We find ourselves describing the deaths of our loved ones as good, although we are hard
put to find a reason. Thus calling a death good or bad may not be about believing it is either good or bad, but simply gives the survivors a socially acceptable handle on tins
most taboo of topics. As Moscovici notes: By setting a conventional sign of reality on the one hand, and, on the other, by prescribing through tradition and age-old structures,
what we perceive and imagine, these creatures of thought, which are representations, end up by constituting an actual environment. (Moscovici 1984a: 12, original italics) Thus
the anchoring of death as good or bad in sacred, medical or natural terms confines the participants to a pattern of behaviour and rules. Clearly the representation of the good or
bad death is the concern of the living, yet it constrains those same people when it is their turn to die. I do not wish to suggest that this representation of death is used cynically
by the professionals. They genuinely believe they are making life easier for the bereaved. Discussions by death-workers of their own experiences of bereavement suggest that
they use the same representations of the good death when they lose someone they love. Significantly. I found that they appear just as vulnerable to conflict with those in control
as the non-professional bereaved. We do not just occupy roles like a theatre troupe (Goffinan 1959). We also compete and argue. I agree with Billig (1986) in his call for the
merging of the theatre and game metaphors. As I hope will have become apparent in Chapter 6. The discourse about the good and bad death is 'argumentative'. As Moscovici
(1984a) notes, when one person's classification of an object or an event is imposed on another's they convey a bundle of expectations as to how the other person should
respond. There is no guarantee that the other person will agree. I found that the bereaved kin were not at all passive about classifying their partner's death as good or bad and
they certainly do not always share the same representation as the deathwork professional. We can observe a tussle for power in those instances where the bereaved's
representations of death are not in accordance with those of the professionals". We might imagine that the outcome of such negotiations or arguments would be predictable,
given that the professional is usually in a position of power over their bereaved client. Yet I found that despite their distress, families stubbornly resist the constraints imposed by
the professionals and the professionals" personal representation of the death, and still manage to win the day by reinstating their own personal representations after the
disposal of the body. We should be wary of constructing 'definitions" of the good death. Given the popularity of the good death in literature of the social sciences (Aries
1974,1933; Beier 1989; Bradbury 1993a. 1996; Kellehear 1990; Sanierel 1995; Small 1993; Walter 1994) it ts possible that the lay population will appropriate, and in the process
distort, mis 'scientific knowledge" about the good and bad ways of doing a death. Re-presenting the healthy grief Just as the medical model dominates our representations of the
good and bad death, I have argued that grief has been similarly medicalized. For decades psychologists have viewed grief as the archetypal individual experience. Counsellors,
therapists, journalists and the bereaved themselves have been looking for symptoms, stages and outcomes which they believe to be concrete phenomena, like a tumour or a
fracture, rather than a social representation of loss. The anchoring of bereavement in the grief reaction simultaneously constrains the sorrowful person and reassures the
observer about all those chaotic and powerful expressions of loss. That which was disturbing and unknown becomes the known, belonging to a category and with a familiar set
of characteristics. The psychiatric and psychological research into grief lias failed to appreciate the social nature of grief as mourning individuals battle to rebuild their damaged
selves. I found mat my respondents had a notion of grief as something mat might turn into a life-tlireatenmg disease. These women did not have to be apprenticed into knowing
about this representation, for it permeates our popular culture. So just as I tapped into the representations of good and bad deaths as salient features in the interviews

themselves, the psychiatric and psychological research into the grief reaction, discussed in the last chapter, tapped not the reality of grief as a disease but social representations
of grief as a disease. Representations, however, have a habit of becoming realities. These studies remain fascinating and important, even though the psychiatrists and
psychologists conducting them did not discover what they thought they had. The use of quantitative research tools, in which the researcher sets the agenda, may have obscured

It has been argued that the postmodern era, which has been developing
at some pace over the last few decades, represents a chance for the re-enchantment of
society, a welcome return of the 'subject* (Bauman 1992b). However, there are problems associated with this. As Bauman
notes 'the post-modem mind seems to condemn everything, propose nothing' (1992b: 9).
The wholescale demolition and deconstruction of our modem and objective view of the world can engender fear of the void. Thus we attack the
medical model and dismantle the safe, reassuring descriptions of the stages of dying,
the good medical death and healthy grief and in their place seem to leave nothing but
the process of re-presentation.

variety and choice . Incoherence seems to be postmodernism s distinctive feature. Bauman would argue that we are
tearing down the power- supported structure of the modem era, the false truths that
were constructed out of a desperate search for structure and order that followed on the
heels of the Renaissance . But this does not lessen those private fears which are
expressed in the contemporary mania for the DIY solutions poignantly illustrated by the
natural death movement, in which dying individuals are encouraged to 'take on' their own
deaths and to draw comfort from the imagined community of fellow natural death
supporters. Death is a physical event that turns the subjective self into an objective other
and for this reason represents something of a challenge to our conception of this brave
new world. My exploration of contemporary deathways revealed them to be both
traditional and innovative. With apparent ease we seem to be able to accept both continuity and change, that fine balance which enables us to draw
an embalmed corpse on a horse-drawn hearse to the local crematorium for a service with a priest. Despite the rapid rate of change over the past couple of centuries there
seems little sign that our belief in immortality has been genuinely deconstructed, thus causing an over-emphasis of the here and now which is so often described as a sign of
postmodernisni's holdover us (Bauman 1992b). There is little that is either 'ephemeral* or 'evanescent* about planting a tree or erecting a headstone (Bauman 1992b).

Many of the social representations of death and the customs and rituals which express
them do act as a balm to the survivors. These funerals are rites of passage which send
our loved ones to the other world. There is symbolism to gather by the arrnful if we only
know where to look: the floral tribute that shouts' FOREVER"; the kitsch condolence card bedecked with doves; the visit to the funeral director's chapel to see
a corpse pumped full of formaldehyde; the poem recited on a favourite hilltop as the wind catches the ashes as they are poured from their urn; the dedicated bench in the park .
The British way of death cannot be explained away in terms of mindless tradition, the
dominance of the medical profession or the seductive force of consumerism. These acts
are full of the meanings of love, life and the hereafter.

AT: Symbolic Value


No link---deaths symbolic value does not deny the value of life---the option
to continue living should always be available
Kacou 8
(Amien Kacou 8 WHY EVEN MIND?On The A Priori Value Of Life Cosmos and History: The
Journal of Natural and Social Philosophy, Vol 4, No 1-2 (2008)
cosmosandhistory.org/index.php/journal/article/view/92/184)
I. What we mean (in more detail) Regardless of whether or not we find that it is the
fundamental question of philosophy,[8] we can see that judging whether life is or is
not worth living is, in one sense at least, when understood with a general character, a
fundamentally philosophical question. The question calls on the living individual to
make a value judgment (which seems of the most serious extent) about the condition
it most basically, most generally, and in a sense most intimately, finds itself inand
in this sense, the question must be seen as a personal one.[9] But the judgment
called for only becomes especially interesting for philosophical exercise once we
attempt to make it objective, so to speak. On the one hand, from a subjective
point of view, the question whether life is or is not worth living reduces
simply to the question whether or notor howwe the living already in
fact, in our variable situations, desire or not to live. Its answer is a function of
descriptions of motivational dispositions as they may vary from individual to individual
and circumstance to circumstance. (For instance, some may find certain cases of
euthanasia justified, or find certain forms of suicide honorable.) In other words, from a
subjective point of view, the answer would simply be that life is worth

living to the extent that, while we could have a different attitude, we


just happen to want it (to be disposed towards it), perhaps in light of
circumstantial considerations. And, thus, the question could be quickly
addressed in some cases, without much need for philosophical inquiry
except perhaps on collateral issues. Similarly, typical answers referring simply to how
fun or beautiful life is, or (perhaps the contemporary scientists favorite) how
fascinating or mysteriousthese answers are, as stated, inadequate for our
purposes, to the extent that they are primitive (uncritical) generalizations expressing
our preexisting desire for (or infatuation with) life. On the other hand, from a point of
view that purports to be objective, we must have a more complex approach. We
must problematize the value of life in general. The question is not simply whether (or
how, or even why in the broad sense of an explanation) we happen to desire or not to
live, but rather whether (or why in the narrow sense of a justification) we should or
should not ever desire to live in the first place. a. The objective approach In seeking a
justification, we must look, beyond the mere freedom (or given-ness) of any primitive
desire, for something like a final authority that we could show through some fact or
logical inference[10] to make us right (i.e., as a matter of reasoning) to have such a
desire. In other words, we must try to present life as being instrumental or not to some
uncontested value (or purpose)and, thus, as either useful or futile. In this sense,
the word meaningful (pertaining to life) would be basically synonymous with the

word usefula relation between objects and moments, on the one hand, and how
what we value can be served, on the other hand. In addition, we do not look for
conditions that would sustain or increase the value of a proposed venture; we look for
a demonstration that the venture can have any value. Accordingly, in
order to find the objective position, we must avoid in our picture of life or death any
variable circumstance that could be taken to make either one of them arbitrarily more
or less attractive or pursuable. Furthermore, any description of how we desire

to live may well entail conditions under which we would not prefer to
live. But even if such conditions exist , it does not necessarily follow that
life is without value when those conditions prevail . Indeed, by analogy,
that we would prefer ten dollars to five dollars if we could choose
does not mean at all that the five dollars would then have no value.
Other example: that we would happily accept an ice cream cone if it
were free but refuse it if we had to pay for it does not mean at all that
ice cream has no value to us. The distinction can be expressed as follows: the
value of ice cream in light of its cost, we call its a posteriori value; the value of ice
cream irrespective of its cost, we call its a priori value. What we seek in this

inquiry is the a priori value of life-as-such the value that subsists in, or is
essential to, or is the initial value in, any life , irrespective of its
circumstances , including, to any extent possible, any explanation for
its existence.[11] (The objective question calls for an a priori answer.) In contrast,
we are not interested in questions such as whether seppuku is honorable or how endof-life decisions should be made. (The subjective question, the question of
circumstances and the a posteriori answer coincide.)

Life = Pre-requisite
Life is a pre-requisite to deaths symbolic value---fearing death doesnt
preclude recognizing lifes finitude and its inevitability---we can still create
provisional value in life---individuals should have the option to live
Kalnow 9
(Cara Kalnow 9 A Thesis Submitted for the Degree of MPhil at the University of St. Andrews
WHY DEATH CAN BE BAD AND IMMORTALITY IS WORSE https://research-repository.standrews.ac.uk/bitstream/10023/724/3/Cara%20Kalnow%20MPhil%20thesis.PDF)
our lives could
accumulate value through the satisfaction of our desires beyond the boundaries of the
natural termination of life. But Chapter Four determined that the finitude of life is a necessary condition for
the value of life as such and that many of our human values rely on the finite temporal structure
of life. I therefore argued that an indefinite life cannot present a desirable alternative to our finite life, because life as such would not be recognized as valuable. In this
chapter, I have argued that the finitude of life is instrumentally good as it provides the recognition that
life itself is valuable . Although I ultimately agree with the Epicureans that the finitude of life cannot be an evil,
this conclusion was not reached from the Epicurean arguments against the badness of
death, and I maintain that (HA) and (EA) are insufficient to justify changing our attitudes towards our future deaths and the finitude of life. Nonetheless, the instrumental
(PA) also provided us with good reason to reject the Epicurean claim that the finitude of life cannot be bad for us. With (PA), we saw that

good of the finitude of life that we arrived at through the consideration of immortality should make us realize that the finitude of life cannot be an evil; it is a necessary condition
for the recognition that life as such is valuable. Although my arguments pertaining to the nature of death and its moral implications have yielded several of the Epicurean
conclusions, my position still negotiates a middle ground between the Epicureans and Williams, as (PA) accounts for the intuition that

it is rational to fear

death and regard it as an evil to be avoided . I have therefore reached three of the Epicurean conclusions pertaining to the moral
worth of the nature of death: (1) that the state of being dead is nothing to us, (2) death simpliciter is nothing to us, and (3) the finitude of life is a matter for contentment. But

we can rationally fear our future deaths, as categorical desires


provide a disutility by which the prospect of death is rationally held as an evil to be
avoided. Finally, I also claimed against the Epicureans, that the prospect of death can rationally be regarded as morally good for one if one no longer desires to continue
against the Epicureans, I have argued that

living. 5.3 Conclusion I began this thesis with the suggestion that in part, the Epicureans were right: deathwhen it occursis nothing to us. I went on to defend the Epicurean
position against the objections raised by the deprivation theorists and Williams. I argued that the state of being dead, and death simpliciter, cannot be an evil of deprivation or
prevention for the person who dies because (once dead), the personand the grounds for any misfortunecease to exist. I accounted for the anti-Epicurean intuition 115 that

it is rational to fear death and to regard death as an evil to be avoided, not because death simpliciter is bad, but
rather because the prospect of our deaths may be presented to us as bad for us if our
deaths would prevent the satisfaction of our categorical desires. Though we have good
reasons to rationally regard the prospect of our own death as an evil for us, the fact that
life is finite cannot be an evil and is in fact instrumentally good, because it takes the
threat of losing life to recognize that life as such is valuable . In this chapter, I concluded that even
though death cannot be of any moral worth for us once it occurs, we can attach two
distinct values to death while we are alive : we can attach a value of disutility (or utility)
to the prospect of our own individual deaths, and we must attach an instrumentally good value to
the fact of death as such. How to decide on the balance of those values is a matter for
psychological judgment.

Maintaining the conditions of possibility for life is a prerequisite to ongoing


experiences with death
Lawtoo 5
(Nidesh Lawtoo 5, Dept of Comparative Lit, U Washington, Bataille and the Suspension of
Being, linguaromana.byu.edu/Lawtoo4.html)
Bataille's notion of communication involves a dialectic with two positives (hence a non-dialectic) where two sovereigns confront death not in view of an end but as an end in

confronting death," in fact, "puts the subjects at stake-"l'tre en eux-mmes [est] mis en jeu" (Sur Nietzsche 61). Further,
Bataille affirms that "[p]ersonne n'est-un instant-souverain qui ne se perde" (OC VIII 429). It is the Nietzschean self-forgetfulness that is here evoked; a selfforgetfulness which implies a transgression of the limits of both communicating subjects.
itself: "

Again, for Bataille "[l]a 'communication' n'a lieu qu'entre deux tres mis en jeu-dchirs, suspendus, l'un et l'autre penchs au-dessus de leur nant" (Sur Nietzsche 62).

However, if according to Nietzsche, self-forgetfulness takes place in solitude, for Bataille


it necessitates the presence of an "other ."(5) Communication in fact, asks for "deux tres mis en jeu" who participate in what he
defines as "une fte immotive" (Sur Nietzsche 31). There the sovereign loses himself (se perde) with the other, through
the other, in the other, in a process of "mutual laceration" (Essential 105) which is simultaneously tragic and ludic. The
emphasis on the other is Hegelian, but unlike dialectics, communication does not confront the subject with an object (Gegen-stand, something that stands against the subject).
As Bataille puts it (apparently echoing Baudelaire), communication takes place with "un semblable," "mon frre" (OC VIII 289). And he adds: "Cela suppose la communication

Bataille's notion of communication is not based upon a "violent


hierarchy" (Derrida's term) but rather upon egalitarianism. Moreover, transgressing the limits of the
subject implies that the two subjects already possess (in potential) the characteristics of
sovereignty. Hence, the status of sovereign is not achieved as a result of a fight to the
de sujet sujet" (OC VIII 288).

death, but requires the subject to be open to an other who is outside the limits of the
self . Derrida speaks of the "trembling" to which Bataille submits Hegelian concepts (253). This trembling, I would argue, has its source in Nietzsche (6): "The figs fall from
the trees" says Zarathustra, "they are good and sweet, and when they fall, their red skins are rent. A north wind am I unto ripe figs" (qtd. in Philosophy 135). If we apply this
passage to Bataille's philosophy, we could say that inherent in this "fall" is an explosion of Hegelian concepts, and in particular, as we have seen, the notion of Herrshaft.
Further, communication, for Bataille, involves a similar "fall" which rents (dchire) the skin of the subjects (their limits) exposing the red flesh which lies beneath the skin.
According to the French philosopher, Nietzsche's critique of the subject is more radical than Hegel's, since, as he puts it in "Hegel, la mort et le sacrifice," Hegel's philosophy,
and I would add Kojve's interpretation of it, is "une thologie, o l'homme aurait pris la place de Dieu" (OC XII 329). Hegel's "theology" preserves the identity of the subject.
Now, Bataille makes his position to this "theology" clear as he writes: "I don't believe in God-from the inability to believe in self" (Essential 10). By establishing a direct link
between the death of the subject and the death of God, Bataille extends his critique of "beings" into the larger, ontological, critique of "Being." Implicit in this theoretical move is
the articulation of the ontology of sovereignty. Bataille's philosophy is Nietzschean insofar as it is grounded in experience and in the immanence of the body. Communication, for
Bataille is first and foremost a bodily affair. Hence the interrogation of the limits of the subject starts from an interrogation of what we could call the "gates," or openings of the
body: the mouth, the vagina, the anus and the eyes are for Bataille central places for philosophical investigation because at these gates, the integrity of the subject is
questioned; its limits can be transgressed. They are spaces of transition where a "glissement hors de soi" (OC VIII 246) can take place. These bodily openings, which Bataille
also defines as "blessures," (Sur Nietzsche 64) found his conception of the sovereign subject. In fact, each "blessure" can be linked to a specific dimension of communication
which obsesses Bataille. His central themes match different bodily openings: the mouth connects to laughter; the vagina to eroticism; the eyes to tears; the anus to the
excrements which he links to death. Through these openings the subject is traversed by different fluxes and its integrity, totality and stability is challenged. They allow for the
possibility of a glissement of the subject's being. The same could be said of Bataille's corpus: it is a unitary entity, which, like a body, escapes the totalizing temptation of closure.
Despite the fact that Bataille defines sovereignty in terms of the Kojvian/Hegelian "nothingness" (Bataille's Rien), his conception of communication is built upon the
Nietzschean ontological distinction between the Dionysian and the Apollonian. In fact, the ontological movement that takes place in communication "exige que l'on glisse" (OC
VI 158) from an "insufficient" and "discontinuous" being to a reality of "continuity" that transcends binary oppositions (Erotism 13-14). To put it more simply, communication
introduces a movement from the "many" to the "One"; from a "discontinuity of being" to a "continuity of being;" from separate "beings" to a common ontological ground ("Being").
The source of Bataille's ontology is clear: it stems from Nietzsche's The Birth of Tragedy which in turn, is construed upon Schopenhauer's distinction between will and
representation. "As a sailor sits in a small boat in a boundless raging sea," writes Schopenhauer," surrounded on all sides by heaving mountainous waves, trusting to his frail

Communication, for
involves the shattering of the principium individuationis, a tearing down of the veil of Maya which
constitutes, what Bataille calls, with a blink of the eye to Schopenhauer, the "illusion of a being which is isolated" (Essential
10; my emphasis). Communication, thus, involves an opening of the subject to the larger ground
of Being. The sovereign's boat is constantly leaking. Yet, in order for communication to
take place, the boat needs to keep floating. That is to say that for transgression to take place, the
vessel; so does the individual man sit calmly in the middle of a world of torment, trusting to the principium individuationis" (Birth 21) .(7)
Bataille, as the Dionysian for Nietzsche,

limits of the subject need to be preserved (Erotism 63; Foucault 34). The being of the sovereign
subject is suspended upon the abme-what Bataille also calls "une realit plus vaste" (OC II 246)-which means that the subject neither
dwells safely within the limits of the "small, insufficient boat" of individuation, nor within
the depth of the undifferentiated "raging sea," but in the space of contact in-between the
two spheres. This precision is key in order to delineate the originality of Bataille's ontology of

sovereignty. Bataille's conception of the communicating subject (i.e., of sovereignty)


walks a thin line between its self-dissolution and its self-preservation. Hence the idea
that he is above all a thinker of limits

or borders.

The sovereign's being, in fact, is "suspended" on the

but never actually falls , except, of course, in death. Hence, for Bataille, "[i]l s'agit d'approcher la mort" [it is
approaching death] that is to say, the abme, or the continuity of being, "d'aussi prs qu'on peut l'endurer" [as close as one can
endure] (337-338). The sovereign subject confronts death while preserving his life. His being is
placed at the border between life and death. Hence, if Bataille defines philosophy as
"existence striving to reach its limits" (Essential 146), it should be specified that the being of the
subject is not found beyond its limits, as his use of "existence" seems to suggest (Ek-sistenz) since that would
imply a total dissolution of the subject. Bataille's philosophy of transgression implies the
preservation of the limits of the subject so that the sovereign can experience and endure
death in life. The tension between self-expenditure (Nietzsche's Verschwendung) and self-preservation
(linked to Hegel's Anerkennung) is analogous to the movement of a moth that is first attracted by the fire
of a candle and subsequently distances itself from the fire in order to preserve its life. (8)
This repeated back and forth movement recapitulates the movement of communication
and is responsible for the underlying tension which traverses Bataille's philosophy. It is
an inner (bodily) drive that attracts the moth to death and not, as it is the case for Hegel's master, a reasoned project in view
"bord de l'abme" (Coupable V 355)

of an end (recognition). The moth's self-sacrifice, in fact, is perfectly useless (it serves no purpose) and hence is truly sovereign. Bataille would call it "une ngativit sans
emploi." Or, as he says with respect to eroticism in his first and last interview before he died, "it is purely squandering, an expenditure of energy for itself" (in Essential 220).

This movement forwards, towards the flame of self-dissolution (which takes place in death, eroticism, laughter)
and its retreat backwards, towards life and the limits it involves, epitomizes Bataille's
notion of communication. A practice which for Bataille seems to have the characteristic of a fort-da game in which the subject is not in control of the
movement. This movement, Bataille writes in the Preface to Madame Edwarda, happens "malgr nous" (III 11). Thus conceived the sovereign accepts the place of a toy in the
hands of a child playing-a definition similar to Heraclitus' vision of life, which he defines as "a child at play, moving pieces in a game (Fragment 52, in GM 149). This view of

If Bataille
is deeply fascinated by death, decay and the dissolution of the subject in a continuity of
being, he escapes the temptation to embrace death at the expense of life . His definition
of eroticism sums up this fundamental tension: "Eroticism," he writes, "is assenting to
life up to the point of death" (Erotism11). This applies not only to eroticism but also to all communicating activities such as
laughter, play, tears, and ultimately to the ethos that sustains the totality of Bataille's philosophy. If Kojve defines dialectics as a
"negating-negativity" (5), Bataille's communication can be read as an affirmative negativity. In fact, death
is confronted and even invoked , but what is found in death is the ultimate affirmation of
communication is both tragic and joyful; violent and useless. A joyful tragedy, which challenges the limits of the subject; that puts the subject's being en jeu.

life. Negation of the integrity (the limits) of the subject leads to a radical affirmation of
life . And if in the Preface to Madame Edwarda, Bataille can affirme "l'identit de l'tre et de la mort" (OC III 10), let us also note that the identity of being
and death is realized in life . Faithful to Nietzsche, Bataille does not become a negator of the will; a
negator of life; a pessimist, a Buddhist or worse, a nihilist (some of the derogatory terms used by Nietzsche to retrospectively define
his first and last master). Bataille remains truthful to life . While the ontological premises grounding sovereignty are taken from Schopenhauer (via
Nietzsche), Bataille's conclusions are diametrically opposed to Nietzsche's first master. In fact, Bataille's philosophy can be seen as an
affirmation of the will (he operates an inversion of values) through Dionysian practices (included sexuality which Schopenhauer
condemned) that put the subject in touch with the ultimate ground of being, without dissolving
him/her in it .

Alt = Obsession
Trying to find the truth in death causes eternal obsession its not
productive criticism
Dollimore, Sussex sociology professor, 1998
(Jonathan, Death, Desire and Loss in Western Culture, pg. 221)
Occasionally one

wonders if the advocates of the denial-of-death argument are not themselves


in denial. They speak about death endlessly yet indirectly, analyzing not death so much
as our cultures attitude towards it. To that extent it is not the truth of death but the truth
of our culture that they seek. But, even as they make death signify in this indirect way, it
is still death that is compelling them to speak. And those like Baudrillard and Bauman
speak urgently, performing intellectually a desperate mimicry of the omniscience which
death denies. One sense that the entire modern enterprise of relativizing death, of
understanding it culturally and socially, may be an attempt to disavow it in the very act of
analyzing and demystifying it. Ironically then, for all its rejection of the Enlightenments arrogant belief in the power of
rationality, this analysis of death remains indebred to a fundamental Enlightenment aspiration
to mastery through knowledge. Nothing could be more Enlightenment, in the pejorative
sense that Baudrillard describes, than his own almost megalomaniac wish to penetrate
the truth of death, and the masterful controlling intellectual subject, which that attempt presupposes. And this may be true to an extent for
all of us, more or less involved in the anthropological or quasi-anthropological accounts of death which assume that, by looking at how a culture
handles death, we disclose things bout a culture which it does not know about itself. So what has been said of sex in the nineteenth century may also
be true of death in the twentieth: it has

not been repressed so much as resignified in new, complex,


and productive ways which then legitimate a never-ending analysis of it.

Survivalism First
Survivalism must be unconditionalits a prerequisite to all ethics and
values
Hgglund 8
(Martin, Fellow in Comparative Literature at Cornell University, Radical Atheism: Derrida and
the Time of Life google books)
In the preceding chapters I have demonstrated how Derrida's work offers powerful resources to think life as survival and the desire
for life as a desire for survival. I have argued that every moment of life is a matter of survival because it
depends on what Derrida calls the structure of the trace. The structure of the trace follows from the constitutive division of time.

Given that every moment of life passes away as soon as it comes to be, it must be
inscribed as a trace in order to be at all. The tracing of time enables the past to be
retained and thus to resist death in a movement of survival. However, the survival of the trace that
makes life possible must be left for a future that may erase it. The movement of survival protects life, but it
also exposes life to death, since every trace is absolutely destructible. I have argued that such
radical finitude is not a lack of being that it is desirable to overcome. Rather, the finitude of survival opens the
possibility of everything we desire and the peril of everything we fear. The affirmation of survival is
thus not a value in itself; it is rather the unconditional condition for all values. Whatever one
may posit as a value, one has to affirm the time of survival, since without the time of survival
the value could never live on and be posited as a value in the first place.
the unconditional affirmation of survival allows us to
rethink the condition of political responsibility and especially the desire that drives
political struggle. I will argue that the radical finitude of survival is not something that inhibits
In this final chapter I want to elaborate how

responsibility and political struggle; it is rather what gives rise to them . If we were not
exposed to the coming of a future that could violate and erase us there would be nothing
to take responsibility for, since nothing could happen to us. It is thus the finitude of survival and
the affirmation of such survival that raises the demand of responsibility. If I did not desire the
survival of someone or something, there would be nothing that precipitated me to take action. Even if I sacrifice my own life for
another, this act is still motivated by the desire for survival, since I would not do anything for the other if 1 did not desire the survival
of him or her or it.

AT: Ressentiment

Ressentiment not First


Ressentiment doesnt come first extinction and material impacts do
Torbjrn Tnnsj 11, the Kristian Clason Professor of Practical Philosophy at Stockholm
University, 2011, Shalt Thou Sometimes Murder? On the Ethics of Killing, online:
http://people.su.se/~jolso/HS-texter/shaltthou.pdf
if Schopenhauer is right, if life is never worth living, then according to
utilitarianism we should all commit suicide and put an end to humanity. But this does not
mean that, each of us should commit suicide. I commented on this in chapter two when I presented the idea that
utilitarianism should be applied, not only to individual actions, but to collective actions as well. It is a well-known fact that
I suppose it is correct to say that,

people rarely commit suicide. Some even claim that no one who is mentally sound commits suicide. Could that be taken as evidence for the claim that people live lives worth

It is also a
possibility that, even if people lead lives not worth living , they believe they do . And even if
some may believe that their lives, up to now, have not been worth living, their future lives
will be better . They may be mistaken about this. They may hold false expectations about the future. From the point of view of evolutionary biology, it is natural to
living? That would be rash. Many people are not utilitarians. They may avoid suicide because they believe that it is morally wrong to kill oneself.

assume that people should rarely commit suicide. If we set old age to one side, it has poor survival value (of ones genes) to kill oneself. So it should be expected that it is
difficult for ordinary people to kill themselves. But then theories about cognitive dissonance, known from psychology, should warn us that we may come to believe that we live

My strong belief is that most of us live lives worth living

better lives than we do.


. However, I do believe that our lives are
close to the point where they stop being worth living. But then it is at least not very far-fetched to think that they may be worth not living, after all. My assessment may be too

for the sake of the argument assume that our lives are not worth living, and
let us accept that, if this is so, we should all kill ourselves. As I noted above, this does not answer
the question what we should do, each one of us . My conjecture is that we should not commit
suicide. The explanation is simple. If I kill myself, many people will suffer. Here is a rough explanation of how this will happen: ...
optimistic. Let us just

suicide survivors confront a complex array of feelings. Various forms of guilt are quite common, such as that arising from (a) the belief that one contributed to the suicidal

Suicide also leads to rage,


loneliness, and awareness of vulnerability in those left behind. Indeed, the sense that suicide is an essentially
selfish act dominates many popular perceptions of suicide. The fact that all our lives lack meaning , if they do, does
not mean that others will follow my example. They will go on with their lives and their false expectations at least for a while
person's anguish, or (b) the failure to recognize that anguish, or (c) the inability to prevent the suicidal act itself.

devastated because of my suicide. But then I have an obligation, for their sake, to go on with my life. It is highly likely that, by committing suicide, I create more suffering (in their
lives) than I avoid (in my life).

Ressentiment Productive
Ressentiment is productiveinseparable for some freedom and their
crusade against it links just as much
Stefan P. Dolger 10, Brock University, "In Praise of Ressentiment: Or, How I Learned to Stop
Worrying and Love Glenn Beck", APSA 2010 Annual Meeting Paper,
papers.ssrn.com/sol3/papers.cfm?abstract_id=1642232&download=yes
After Ressentiment In closing I would suggest that

my praise of ressentiment is also in line with the more


deliberatively conceived multiculturalism of the Left than is the current puritanical disdain. As Monique
Deveaux argues, it is a failure of political imagination when we fixate on liberal principles as preconditions lo multicultural dialogue,
and in particular it is necessary to move toward a deeper level of intercultural respect rather than mere toleration (Deveaux 2000).10
But if it is appropriate to go beyond simply tolerating non- liberal peoples abroad and in immigrant communities, if we must go
beyond toleration to do justice to the rich tradition of cultural pluralism, then perhaps we can also open our hearts and minds to the
possibility that the ressentiment-suffused need to be heard out as well. Perhaps rather

than demonizing
ressentiment as a toxin to politics, as the worst of the worst for subjects whom we
purport to free, we must accept that ressentiment is for many inseparable from their
conception of their own freedom. Perhaps rather than pitying these poor fools, in ways that we would
never pity a plural wife in the global South, we should ponder whether ressentiment as a precondition
of subjectivity is as much a gift as a curse. And are we so sure, after all, we late Nietzscheans, that
our crusade against ressentiment is not itself suffused with ressentiment? Is not itself fully in the
grips of it? How would we know if it were or werent? Perhaps we are, in our own way, as spiteful,
vain, petty, weak, subjected, enraged against the past, capitalized, consumerized, unfree, as those we purport to want
to free from the chains of slave morality. Perhaps it is ourselves that we need to give a break to, that we need to get over,
when we first look to purge the other of ressentiment. Perhaps we all swim in this current, perhaps we are
all Ressentiments children, and perhaps that is OK even to the extreme of the using
ressentiment unconsciously in the effort to rid the world of ressentiment. Though just in saying
so I wouldnt expect that to do much to overturn Ressentiments reign. No, she is far too puissant for that. But we do not need
to rage against the weakness in others because we fear the dependence and weakness in
ourselves. As Vetlesen puts it, defending Amery: Against Nietzsche, who despised victims because
he saw them as weak, as losers in lifes struggles, Amery upholds the dignity of having been
forced by circumstances beyond ones control into that position, thus reminding Nietzsche
that as humans we are essentially relational beings, dependent, not self-sufficient. In hailing the strong
and despising the weak, in denying that vulnerability is a basic ineluctably given human condition, a condition from which
not only the role of victim springs but that of the morally responsible agent too, Nietzsche fails to be the
provocateur he loves to believe he is: He sides with the complacent majority and so
helps reinforce the existential and moral loneliness felt by Amery, the individual victim who speaks up
precisely in that capacity (Vetlesen 2006, 43). Perhaps we can begin to see how we have been using the
weak, the viewers of Glenn Beck and others, as the targets for our need to find blameworthy agents.
And that too is fine. The trouble comes when we think weve gone beyond Ressentiment when in
fact were just listening to her whisperings without realizing it. We think that we can well
and truly look down on the Rush Limbaughs, these destroyers of civilization, because they are
possessed by something that we are above. And far be it from me to suggest that we should not resent, should
not blame; I merely suggest we direct our blame toward more useful ends than where it is
currently located.

AT: Ressentiment = Extinction


Ressentiment doesnt cause extinction
Bernard N. Meltzer 2, and Gil Richard Musolf, profs of sociology at Central Michigan
University, Resentment and Ressentiment, Sociological Inquiry, V72 N2, Spring, 240-55
Given this negative characterizationa prevailing evaluationof ressentiment, it is not surprising to learn that Sartre (1965, p. 14) describes those who
experience the emotion as individuals who establish their human personality as a perpetual negation. Augmenting the negative view of this emotion is
a widely held view among scholars that both resentment and ressentiment tend to be base, dastardly emotions resorted to by thin-skinned individuals
and seekers after excuses for failure, that these emotions are often felt irrationally, on occasions in which one has not been morally wronged. Thus,
Solomon (1995) refers to a vindictive emotion, frequently a personal, petty, disproportionate reaction to a slight; Ortony, Clore, and Collins (1988)
write of a distasteful emotion; and Adam Smith [1759] (1969) designates a disagreeable passion.7 On the other hand, Solomon (1994), elsewhere,

ressentiment, pointing out that it often entails compassion for others in the
same situation, and its implicit sense of injustice may lead to corrective action;
thus, ressentiment can be seen as an expression of the socially responsible insistence
of the community on justice and justification (p. 124). Similarly, Haber (1991) argues
that resentment can be a form of personal protest that expresses regard for oneself, for
others, and for the normative order (p. 48). Moreover, Haber (1991, p. 82) holds that a display of resentment
may serve as an instrument of individual or social change. In fact, the
historian Hippolyte Taine (cited in Jameson 1976, p. 131) sought to explain revolutions in terms of
underlying ressentiment, and Jameson (1976) contends that this emotion is the very content of revolutions. In the same vein, various
scholars have asserted that the individual of ressentiment is a potential revolutionary (Vaneigem 1979, p. 9) and that our
revolutionaries are men and women of resentment(Solomon 1995, p. 266).8 Thus, Merton (1957, p. 155)
maintains that organized rebellion may draw upon a vast reservoir of the resentful and
discontented as institutional dislocations become acute. In the light of such characterizations, the role of the
takes a more positive view of

political agitator is readily recognized as that of raising consciousness of unjust treatment (where such consciousness is absent), inducing ressentiment
(where the emotion is absent), and organizing resistance to the recurrence or continuation of unjust treatment. Moreover, Folger (1987) claims that
revolutionary ideologies can help to create ressentiment.

That ressentiment can be used to initiate (and

sustain) revolution argues against the more passiveand contemptuousconceptions


held by Nietzsche , Scheler, and their many followers.

AT: Ressentiment Productive/K Links


Ressentiment is productiveinseparable for some freedom and their
crusade against it links just as much
Dolger 10
(Stefan P. Dolger 10, Brock University, "In Praise of Ressentiment: Or, How I Learned to Stop
Worrying and Love Glenn Beck", APSA 2010 Annual Meeting Paper,
papers.ssrn.com/sol3/papers.cfm?abstract_id=1642232&download=yes)
After Ressentiment In closing I would suggest that

my praise of ressentiment is also in line with the more

deliberatively conceived multiculturalism of the Left than is the current puritanical disdain. As Monique
Deveaux argues, it is a failure of political imagination when we fixate on liberal principles as preconditions lo multicultural dialogue,
and in particular it is necessary to move toward a deeper level of intercultural respect rather than mere toleration (Deveaux 2000).10
But if it is appropriate to go beyond simply tolerating non- liberal peoples abroad and in immigrant communities, if we must go
beyond toleration to do justice to the rich tradition of cultural pluralism, then perhaps we can also open our hearts and minds to the
possibility that the ressentiment-suffused need to be heard out as well. Perhaps rather

than demonizing
ressentiment as a toxin to politics, as the worst of the worst for subjects whom we
purport to free, we must accept that ressentiment is for many inseparable from their
conception of their own freedom. Perhaps rather than pitying these poor fools, in ways that we would
never pity a plural wife in the global South, we should ponder whether ressentiment as a precondition
of subjectivity is as much a gift as a curse. And are we so sure, after all, we late Nietzscheans, that
our crusade against ressentiment is not itself suffused with ressentiment? Is not itself fully in the
grips of it? How would we know if it were or werent? Perhaps we are, in our own way, as spiteful,
vain, petty, weak, subjected, enraged against the past, capitalized, consumerized, unfree, as those we purport to want
to free from the chains of slave morality. Perhaps it is ourselves that we need to give a break to, that we need to get over,
when we first look to purge the other of ressentiment. Perhaps we all swim in this current, perhaps we are
all Ressentiments children, and perhaps that is OK even to the extreme of the using
ressentiment unconsciously in the effort to rid the world of ressentiment. Though just in saying
so I wouldnt expect that to do much to overturn Ressentiments reign. No, she is far too puissant for that. But we do not need
to rage against the weakness in others because we fear the dependence and weakness in
ourselves. As Vetlesen puts it, defending Amery: Against Nietzsche, who despised victims because
he saw them as weak, as losers in lifes struggles, Amery upholds the dignity of having been
forced by circumstances beyond ones control into that position, thus reminding Nietzsche
that as humans we are essentially relational beings, dependent, not self-sufficient. In hailing the strong
and despising the weak, in denying that vulnerability is a basic ineluctably given human condition, a condition from which
not only the role of victim springs but that of the morally responsible agent too, Nietzsche fails to be the
provocateur he loves to believe he is: He sides with the complacent majority and so
helps reinforce the existential and moral loneliness felt by Amery, the individual victim who speaks up
precisely in that capacity (Vetlesen 2006, 43). Perhaps we can begin to see how we have been using the
weak, the viewers of Glenn Beck and others, as the targets for our need to find blameworthy agents.
And that too is fine. The trouble comes when we think weve gone beyond Ressentiment when in
fact were just listening to her whisperings without realizing it. We think that we can well
and truly look down on the Rush Limbaughs, these destroyers of civilization, because they are
possessed by something that we are above. And far be it from me to suggest that we should not resent, should

not blame; I merely suggest

currently located.

we direct our blame toward more useful ends than where it is

AT: Ressentiment Perm Solves/KT Justice


Perm solves ressentiment may often be destructive, but it is necessary
for a baseline of social justice
Soloman 94
(Robert Soloman 94, Philosophy @ UT Austin, Nietzsche, Genealogy, Morality p. 123-124)
Nietzsche separates what he calls justice from the reactive emotions, defending justice as a rare and unusually noble sentiment while attacking
such emotions as resentment for their impotence. This is no less questionable. But we

need not therefore disagree when


he objects to the abuse of justice as a faade for the defense of ones own interests,
whether in the name of rights or equality and we need not endorse the consequent
leveling effect of enforced modernity. In the name of justice, for example, one may
adopt an egalitarian standpoint, but look only in one direction. The French bourgeoisie
during the French Revolution only looked up at the aristocrats they wanted to replace,
but they never looked down at the rest of the third estate who were much worse off. The
hard question is whether there is any neutral social position (other than our position as outside observers) that would provide the proper standpoint for
making such evaluations. Justice

always begins as situated with the self and its personal passions:
but it need not therefore be selfish, and resentment need not be petty or opposed to a
noble sense of generosity and compassion .

Indeed, given

that we are not Nietzsches

fantasized Ubermenschen, wholly satisfied and in charge of our world, it is hard to even
imagine what justice and for that matter morality would be without resentment and
the modicum of selfishness that makes it possible .
in the sphere of the reactive feelings.

Perhaps Duhring was right: the home of justice is to be sought

AT: Suffering

Injustice/Suffering Reps Good


Our critique of structural injustice turns vampirism. Contextualizing
vulnerability, and the background of injustice balances emphasizing with
material suffering and avoiding sentimentality
Michalinos Zembylas 13, Education @ Open (Cyprus), The Crisis of Pity and the
Radicalization of Solidarity: Toward Critical Pedagogies of Compassion, Educational
Studies 49, p. 512-516
a politics of compassion that takes into consideration the possible dangers of
compassion fatigue, desensitization, and victimization has to begin from
acknowledging common vulnerability and its influence in inspiring meaningful
First of all,

self-

human

actions that avoid presumptuous paternalism


The recognition of ones own
vulnerability can constitute a powerful point of departure for developing compassion and
solidarity
recognition of our
own vulnerability opens up the potential for recognition of all humanity as vulnerable.
Vulnerability may
be a more appropriate term than suffering
because the focus is not merely on the alleviation of material suffering and hence a slide
(Porter 2006; Whitebrook 2002).

with the others vulnerability (Butler 2004). As Butler asserts: Each of us is constituted politically in part by virtue of the social vulnerability of our bodies. ... We cannot ... will away this vulnerability. We must attend to it (2004, p. 29).

Butlers description of the vulnerable body and self refers to the way we perform and are performed upon, and part of what we fear in the other is a projection of our own selves. Hence, Butler suggests that

, therefore,

to ground the political applications of compassion,

from compassion to benevolence and sentimentality


epistemological shift

of focus

does not imply

, however,

(Porter 2006; Whitebrook 2002).

Suggesting this

that a narrative that focuses on the alleviation

of material suffering will necessarily result in a slide into sentimentality


political
applications of compassion cannot be completely separated from questions of material
suffering
The idea of
common vulnerability enables usteachers and students
to explore how we
might move beyond dichotomies that single out the self or the other as victims, and
therefore deserving someone elses pity. That is, the idea of common vulnerability puts
in perspective the notion of all of us as vulnerable, rather than the individual-other who
needs our compassion. This addresses concerns of students
who seem to be
. Undoubtedly, the

. Thus, it needs to be acknowledged that although the move away from suffering may be theoretically useful, the shift to a narrative of common human vulnerability is not completely unproblematic. 8

in the classroom, for instance

as

notion

the

, for example,

stuck in self-victimization claims and refuse to acknowledge that others also suffer
common vulnerability
opens space to problematize moralistic
positioning

. Although the

idea of

does not guarantee any departure from such claims, it

some

. In addition, the notion of common vulnerability attacks a major emotional ideology grounded in the view that it is natural or normal to be fearful of the other, especially if it involves racial differences. This is one of the most common

and pernicious emotional ideologies underlying resistance (especially among White, middle-class students) to identifying with the other. However, if vulnerability concerns everyone and yet compassion is assigned differently (i.e., students think that some deserve

Through addressing
this issue
it is possible to respond to some of the desensitization
concerns outlined earlier, because the dichotomies between we and they will become
meaningless and unproductive.
compassion serves to reinforce a strong connection
compassion but others do not), then it is important to explore what it would take for students to begin imagining themselves as objects of lesser compassion in an unsuspected vulnerable moment.
in ways that do not reify stereotypes or promote essentialism,

Second,

between the personal and the political and accentuates the interpersonal and the
interrelational

(Whitebrook 2002).

Empathetic identification with the plight of others ,

then, is not a sentimental recognition of potential sameness you are in pain and so am
I, so we both suffer the same but a realization of our own common humanity, while

acknowledging asymmetries of suffering , inequality, and injustice. A discourse of


vulnerability neither eschews questions of material suffering nor obscures issues of
inequality
on the contrary, it highlights both the symmetries and the asymmetries of
vulnerability although the experience of vulnerability may be
universal, the
discourse of common vulnerability raises important critical questions such as
vulnerable to what? to whom? to dismiss the possibility of sliding into a sentimental
recognition of
samenesswhich is what a politics of compassion
seeks to avoid
If
properly recognized
this realization can potentially address both the concern about
the desensitization of students and that of their self-victimization, because
the distance between spectator and sufferer will not be taken for granted any more, but
and injustice;

. That is,

more or less

potential

exactly

ardently

Without this double realizationthat is, we are all vulnerable but not in the same mannerour actions run the danger of being a form of charity and condescension toward those who are systematically and institutionally oppressed (Bunch 2002).
in schools,

double

rather its multiple complexities will be acknowledged and interrogated


the kind of
compassion that is explored here requires a simultaneous identification and
disidentification with the suffering of the other. The simultaneous recognition of
symmetry and asymmetry with the other removes the arrogance of claiming that we
know and feel their
suffering. This emotional ambivalence of simultaneous
identification and disidentification is needed to focus attention on the others
suffering, but not becoming too identified with it
. In a sense then,

pain and

a point raised earlier in Nelsons (2004) reading of Arendts reporting on Eichmanns trial. Students who already endure

forms of suffering, of course, do not need a pedagogy to enlighten them how to disidentify with their own suffering. This does not imply, however, that pedagogies that interrogate pity and encourage critical compassion are not for them; on the contrary, the critical

awareness that others are vulnerable is important in the struggle for action-oriented
, too,

solidarity and the avoidance of egocentricity and cultural narcissism

. Finally, the third element of a politics of compassion is

attentiveness to how the ethics of compassion questions injustice and inequality. 9 In particular, an important component of a politics of compassion that is critical and justice-oriented is how it deals with anger at injustice (Hoggett 2006). A politics of compassion
does not intentionally seek to cause anger, however, but rather encourages students and teachers to develop a critical analysis of anger, as it is likely that they will experience such feelings, when they begin questioning long-held assumptions and beliefs about other
people and social events (Zembylas 2007). Anger may call attention to demands for recognition, but also emphasize inequalities (Holmes 2004) and injustices at the civic level (Silber 2011). Anger at injustice can be a positive and powerful source of personal and
political insight in education (Lorde 1984), because it helps to move teachers and students out of a cycle of self-pity, blame, or guilt into a mode of action that somehow responds to injustice. For example, civic anger can be promoted in the classroom as a form
of cultivating individual and collective political consciousness and social resistance to injustices in the students community, although anger is not inevitably emancipatory. However, recognizing the positive power of anger and its link to the struggle against injustice in

The pedagogical challenge for critical


pedagogues is how to encourage students to become active participants with a nuanced
understanding of the emotional complexities involved in histories of injustice and
oppression
ones own community is valuable, if teachers want to promote options for action that may change the conditions of others vulnerability.

Alt Bad Black Suffering


Alt silences black suffering
Rebecca Wanzo 9, Associate Professor, Women, Gender, and Sexuality Studies Program,
Washington U in St. Louis. The Suffering Will Not Be Televised, 180-3
Recognizing the political valence of painof speaking painis essential to black
survival . Given the poorer health status of African Americans in the United States, the fact that pain is often ignored
or borne silently before seeking care, and undertreated once care is sought, those who work with African Americans need to
emphasize the right to tell stories of pain. Obstacles to black storytelling not only come
from white institutional sources, but they also come from self-perceptions that if African
Americans can claim nothing else, they can claim strength. The strong-black-woman stereotype, John Henry, the brave and stoic
kids integrating the store, and other models of black strength ll the U.S. imaginary. Reinterpreting the naming
and speaking of pain can be an act of power, not an act of powerlessness. One person who recognized the
power in speaking about pain was Audre Lorde. In The Cancer Journals she described how she wanted people to respond to her cancer in a way that was atten- tive her identity, to the fact that she was black and
feminist and a lesbian. After her mastectomy, she journaled, I want to write about the pain.74 She wanted to write about the pain because she would willingly pay whatever price in pain was needed, to savor the
weight of completion; to be utterly lled, not with conviction nor with 181 faith, but with experienceknowledge, direct and different from all other certainties. Writing about the pain, speaking about the pain

medical
storytelling is a visceral example of how black pain has been dismissed or reframed in
relationship to various political agendas. Sentimental political storytelling, for all its faults,
imparts knowledge, a different kind of knowledge than that validated by the allegedly objective methodologies of medicine and science, but knowledge nonetheless. The example of

provides an important intervention . A Coda on Moving from Spectator/ Spectacle to Agents in Our Own Care This intervention can
be, as in the best examples of sentimental political storytelling, both public and private,
both therapeutic and a political call to arms. When I saw a call for papers for the Anarcha Symposium, The Anarcha Projects Michigan workshop, I
applied with both public and private work. I shared academic scholarship I had written about Anarcha as well as creative non ction about my surgery, nding the rare space in the academy that made space for
both. Called together in 2007, many of us engaged in scholarship who did not see ourselves as scholars, in creative performance when we did not see ourselves as performers. The group who came together to
discuss Anarcha, J. Marion Sims, and the issues the history raised were eclectic: undergraduates taking classes in disability studies, scholars and performers who focused on African American culture, dancers,
singers, those who had movement constraints, and those of us who had constraints that were less visible. Over the course of a few days we performed physical and mental exercises, bonding in both small and
large groups in order to shape a performance at the end of our time together. We were transformed from spectators into spectacle, but it was a process of constructing a spectacle that was by no means one way

If
a problem with sentimental political storytelling is the conation with other kinds of oppressed
bodies, the productively messy conations pushed us to think about a broader nexus of
institutional oppression. We were divided into small performance groups, and we 182 struggled to nd a collective response that re ected all of our readings of Anarchas,
we looked back in history and looked out to those who could engage with us. While minimalist in presentation, it contained the excesses of our emotional response to Anarchas history and our presents.

Lucys, and Betseys histories, as well as the histories of other unnamed women. On a stark stage, with our bodies, micro- phones, and lights shaping the space, my group produced a short choreo-piece after two
days of work in which the group collectively prompted individual stories with the refrain, This is Anarchas story, and . . . it was the story of all of us. One of us challenged the his- torical record that Anarcha
willingly consented in Simss narrative while also addressing the issue of her relatives lack of consent to medical experimentation during the Nazi Holocaust. Another of us without the use of her legs told the
story of being sexually molested by a medical caregiver, describing histories and futures lost . . . one black, one white, one slave, one not . . . neither touched by request, both silenced by circumstance. In
drawing a comparison between the invisible storiesAnarchas and her ownshe explored how the broader issue of nonconsent and voicelessness in medical care can be read across history and identity. One of
us discussed the lack of choices and resistance when ghting medical men; another discussed fear shaped by history. Drawing on my history, I added to the chorus with a recognition of my difference from
Anarcha as well as my sense of connection to her: I am not Anarcha, but see her story as my story, not because my issues are hers, but because I need someone to hear her pain . . . and alleviate it. And

as we moved in and out of our individual and collective refrains shaped by our specic
stories, the chorus built community, acknowledging the differences between our
histories and our similar investments at the same time. We learned, as Boal suggests in Theater of the Oppressed, to
practice theater as a language that is living and present, not as a nished product
displaying images from the past.75 I nd telling my own story dif cult; in some ways, telling the story of pain management after my surgery and telling of the
Anarcha Symposium performance are equally dif cult. Two spaces of judgment are possiblejudgment of my tolerance for pain and judgment of my creative work, both of which are linked to what it means to
make myself vulnerable. I was advised to cut my personal story from this chapter because of the danger of exposing myself. But if we take sentimental political storytelling seriously as an opportunity to treat affect
and the story of pain as essential to political progress, what example would I set if I remained continually in a space of academic distance when I believe in the political efcacy 183 of the sentimental
narrative? As a subject who has been raced and pained in the United States, I must don the mantle of articulating my affective investment in recognizing the relationship between race and pain without shame.
As I say that it is hard to talk about painbroadlyin the U.S. without talking about race, I recognize that the claim can be read as hyperbolic, and inadequately supported. The charge of hyperbole is often

When we
recognize that we can be subjects of various discourses about race and pain, and not
only subject to a specialized language, such a shift in understand- ing may empower
leveled against sentimental rhetoric. But a review of history, rhetoric, and social and medical discourse reveals that these concepts are often linked in the United States.

people , as health-care advocates encourage, to be agents in their own care. Silence, as Audre Lorde, famously wrote, will not protect you.76 Allowing stories
of pain to be silenced, dismissed, or obscured, however, will surely kill you. We must
speak pain to power.

Sentiment w/ Political Action Solves


Their sentiment K is totally wrong---stories of pain combined with political
action avoid vampiristic consumption and motivate effective harmreduction
Rebecca Wanzo 9, Associate Professor, Women, Gender, and Sexuality Studies Program,
Washington U in St. Louis. The Suffering Will Not Be Televised, 228-232
Despite my disappointment in these lms and frequent annoyance with the narrative trajectory of many of their productions, I admit that I have a bit of a
soft spot for the Lifetime network. I,

too, used to automatically criticize made-for-television movies inspired by


a true story about women at risk. I found them exploitative, as any lm can be that makes
entertainment out of a personal tragedy. Lifetime Television has been called television for victims, in a criticism of its
seemingly endless capacity to show lms about the victimization of women.5 One of the questions that this moniker raises is what kind of storylines
about people have the most dramatic impact. Popular lms with high dramatic impact depict violence, stories of surviv- ing some atypical traumatic
event, or struggling with some more powerful person or entity. One aspect of the criticisms of Lifetime is the objection to formulaic melodrama in itself,
framed within the gendered derision of womens victimization narratives or, on the other side of the political spectrum, discomfort with such narratives
as demeaning, reductive, and trite. The lms shown on the network, some produced by Lifetime but most produced elsewhere, vary in quality, but the
criticisms of Lifetime raise a question that I have explored throughout this book: What is the best way to represent a story of suffering? 229

Simply crying at a Lifetime lm clearly cannot sustain any sub- stantive political work
but what if the crying citizen is directed to, at the very least, awareness, and in the best
case scenario, action, after their emotional catharsis? Sorrow produced at the sight of a
dead or wounded woman may not accomplish anything unless the representation is framed
in relationship to some political action , but tears in relation to abolition and child
abduction did produce action. However, a major ethical problem with using sympathy and compassion as the primary mechanism
for political change is that sentimental politics depends on the cultural feelings of those in power, and the disempowered must depend on patronage.
Hannah Arendt argues that compassion cannot embrace a larger population, but pity can, and pity is a dangerous affect because it cannot exist without
misfortune, thus it has just as much vested interest in the existence of the unhappy as thirst for power has a vested interest in the existence of the
weak . . . by being a virtue of sentiment, pity can be enjoyed for its own sake, and this will almost automatically lead to a glori cation of its cause,
which is the suffering of others.6 Following Arendt, the

charge against Lifetime could be that it thus


encourages sadism because watchers could take pleasure in pity. Or, as literary critic Marianne Noble
has suggested in her study of sentimentality, the network might embrace masochism because watchers would identify with the sufferer and might begin
to take pleasure in these fantasies of subjection.7

However, these readings of the pleasures of consuming

stories of subjection are too narrow . In the case of Lifetime, casting these lms as only narratives
of victimization is too limited a reading. After watching several lms, I began to be compelled by stories I had not heard before
about women interven- ing when the state fails to protect them. The stories were clearly not only about
victimization, but also about survival. The movies negotiate a balance between structural
critique and stories of individual heroism, and I am often disappointed, as with the lms discussed above, with how much
weight is placed on the side of individual transformation. Nonetheless I later began defending the network out of political principle, as part of a broader
effort to challenge the 230 facile denunciation of the word victim. Lifetimes lms are often poor in terms of artistic merit, but the network is
contributing to a national conversation about what agency can look like. My

argument may seem as if I am looking for

politics in all the wrong places, relying on sentimentality when I should focus on politically rational arguments that eschew the
appeals of emotional response. I am not asking for radical progressivism from popular culture.
Instead, I am arguing that politics is often accomplished through the popular and
conventional work of emotional appeals, as many activists throughout history have
demonstrated. The question facing activists for African American womenor, for that matter,
advocates for any identity group outside the national imaginary of ideal citizenship is not only how to expose
discrimination, but also how to make use of existing rhetoric so that attacks on their
bodies can be read as pressing concerns for all U.S. citizens. Affect and popular culture can be
easily criticized as tools of anti-intellectual conservative machines. As Max Horkheimer and Theodor Adorno
rightly argue, popular culture focuses on producing narratives of comfort or affects that can ultimately serve the states purposes.8 Totally

escaping the political storytelling of the status quo elicited by mass-produced texts is
indeed impossible. However, the impossibility of total escape does not preclude the

possibility of making use of tools produced by ideology. Mobilizing affect demands use of
proven rhetorical tools, but this use need not forestall a criticism of the need to employ the
structures in the rst place. Negotiating the relationship between challenging the
masters tools and making use of them to garner nancial support and political power is
not an easy project, but it is a necessary one. The books title is inspired by this very tension between see- ing
popular cultural productions as inevitably politically inef cacious and recognizing the possibilities offered by making use of widely circulated genres
and media. When Gil Scott-Heron produced his famous choreo-poem, The Revolution Will Not Be Televised, in 1974, he called attention to the
disconnect between radical action and violent struggles taking place in the streets and the pleasures of oblivion offered by scripted television and
commercials.9 Television stood in for mass-produced media that would not show what was really occurring in the streets, like pigs shooting down
brothers in instant replay. Scott-Heron pointed to the need for his audience to take to the streets and participate, live, in the revolution. Indeed, a
231 rue revolution requires live political action and organizing, and television and many cultural productions neglect a multitude of issues that are
politically urgent. However, it is clearly no longer the case that pigs shooting down brothers in the street is left off of instant replay. Important events
are depicted on the news, in scripted tele- vision shows, in genre ction, in magazines, in movies, and on the Internet. You can even catch the
occasional social message in a television commercial. Rather than reject various media wholesale, we are left with a set of questions about what to do
with contemporary media realities. How and why are certain kinds of traditionally neglected issues represented? Once represented, how are they
interpreted, and can activists play a role in that interpretation? What do activists do about the complexities lost when they make use of certain kinds of
mass-marketed discourses? Octavia Butler perhaps best articulated this problem in her science- ction novel Parable of the Talents. The novel
exempli es what Lauren Berlant calls the postsentimental textone that exhibits longing for the uncon icted intimacy and political promise sentimentality offers but is skeptical of the ultimate political ef cacy of making feeling central to political change. Her heroine, Olamina, suffers from
hyperempathy syndrome, which allows her to feel the emotions of others, but Butler is careful to argue that being able to feel the pain of others is not
the means for liberationit is a delusional disorder. Thus Olamina focuses on other modes of political change, and struggles to gain followers for her
politi- cal and spiritual project for survival, Earthseed, in a United States devastated by environmental destruction and the domination of a repressive
fusion of government and a religious right organiza- tion called Christian America. Through Olaminas struggle, Butler addresses the intellectual
discomfort with consumption by having a character explicitly argue that only strategic commodi cation will result in successful dissemination of radical
ideas. Olamina struggles with the means by which she can circulate Earthseed, until someone suggests to her that she must use the marketing tools
she slightly disparages to compel people to her project. Her companion, Len, argues that Olamina must focus on what people want and tell them how
your system will help them get it. She resists the call to preach the way her Christian American enemy Jarret does, rejecting preaching, telling
folksy stories, emphasizing a pro t motive, and self-consciously using her charismatic persona to sell Earthseed. 232 Len argues that her
resistance to using the tools of commodi cation leaves the eld to people who are demagoguesto the Jarrets of the world.10 Butler ultimately
presents the moral that the project of producing populist texts for mass consumption cannot be left to those with unproductive or dangerous dreams,
abandoned by a Left that desires not only revolution but also political change resulting in real material gains. Clearly, the productions

of
mass-culture are not the only way to move people to action, but they are no doubt a tool .
The dismissiveness accompanying the label of the sentimental in contemporary culture is because
academic critics claim that it does not do anything, it is the antithesis of action. However,
this book is about how sentimentality is doing things all the time . For better or worse, it teaches people to
identify proper objects of sympathy. It teaches people how to relate to each other. It
teaches people how to make compelling arguments about their pain. The circulation of sentimental political
storytelling often depends on media to which many progressives have a schizophrenic relationship. News media and television are often tools of the
state, but citizens depend on the news for the free circulation of information and often look for progressive politics in television shows. Others disavow
the idiot box altogether and have faith only in alternative news sources. However, the dichotomy between the popular and other spaces in which

While the discourse


has many short-comings, people interested in political change are taking a perilous road
people tell stories about suffering is a false one. Sentimental political storytelling is omnipresent in U.S. culture.

if they ignore the possibilities of imperfect stories told about citizens in pain .

Perm Solves
Thats best --- sentimentality is politically effective when it is recognized as
part of a larger project
Rebecca Wanzo 9, Associate Professor, Women, Gender, and Sexuality Studies Program,
Washington U in St. Louis. The Suffering Will Not Be Televised, p. 9
When theorists criticize producers of
sentimentality for conservative politics, they sometimes attack a rhetoric that is
reactionary or designed to serve the status quo. At other times, such critics express disappointment at a texts possibly radical revolutionary or
However, sentimentality cannot easily be understood as progressive or conservative.

otherwise progressive potential having been short-circuited in favor of feel-good closure offered by the sentimental narrative. World Trade Center
provoked exactly this response from movie critic David Edelstein, who wanted the fi lm about the event of 9/11 to be more political, because the
heartwarming conclusion to the fi lm is unrepresentativeto the point where it almost seems like a denial of the deeper and more enduring
horror.22 Sentimental texts present themselves frequently as progressive about social justice issues while they eventually preserve the status quo.

the binaries of good and bad, Left and Right


are insufficient to categorize sentimentality as it does, by its nature, have a progressive
political thrust. It addresses the suffering of the politically disadvantaged but utilizes conventional
narratives and practices that will not fundamentally disrupt power. Rather than characterizing U.S. sentimentality as
good or bad politics, a more precise characterizationalbeit more of a mouthful and less
dramaticis to call it a politically effective but insufficient means of political change.
Indeed, that is an overlying tendency of most sentimental texts. However,

Agamben

AT: Becoming 1st

Flux Kills Institutions


Openness to flux and constant becoming destroys the foundations for
political institutions necessary to sustain radical democratic life---some
universal, fixed guarantees of equality are crucial to politics
Joseph Schwartz 8, Professor of Political Science at Temple University, The Future of
Democratic Equality, 56-61
Butler, Brown, and Connolly reject the essentialism of narrow identity politics as an inverted ressentiment of the Enlightenment desire for a universal, homogenized identity.
They judge identity politics to be a politics of wounding, resentment, and victimization that only can yield bad-faith moralization Wendy Brown takes to task identity politics for
essentializing conceptions of group identity. For example, she critiques the work of Catherine MacKinnon as epitomizing identity political theory, accusing MacKinnon of
denying women agency by depicting them purely as victims.38 Brown also remains wary of the patriarchal, conformist nature of traditional left conceptions of solidarity and
citizenship. Browns implicit concept of radical democratic citizenship rests upon the recognition that political identity is continually in flux and is socially constituted through
agonal political struggle. Brown celebrates an Arendtian conception of a polity in which both shared and particular identities are continually open to reconstruction. In this left

if
the human condition is a world of permanent flux, then we must postulate a human capability
of living with constant insecurity, for in this world there can be no stable political
institutions or political identities.40 An ability to calculate the probabilities of political
actions or public policies would disappear in this world of infinite liminality. By assuming
that the pre-eminent democratic value is that of leaving all issues as permanently open to
question, post-structuralist democratic theory eschews the theoretical and political
Nietzschean view of an everypersons will to power, there can be no cultural certainties or political givens, as such givens would repress difference and fluidity.39 But,

struggle over what established institutions and consensual values are needed to underpin a
democratic society . Post-structuralist analysis has contributed to a healthy suspicion of narrow and essentializing identity politics. But a self-identified
Of course, if one is
a democrat and a pluralist, one would reject the oppressive homogenization and potentially
authoritarian aspects of ethnic or racial chauvinism and of essentializing types of identity politics. The democratic political
home should be open, fluid, and self-reflective; but if participation is to be open to all, then such a
society also needs to reproduce a shared democratic culture and the institutional guarantee of
democratic rights. That is, contrary to post-structuralist analysis, not all issues can be open to agonal struggle in
a democratic society. The traditional radical democratic critique of democratic capitalism remains valid; the equal worth of the individual is
devalued by rampant social inequality within and between groups. Thus, a radical
democrat, whether post-structuralist or not, must not only be committed to institutional
protections of political and civil rights, but also to social rightsthe equal access to the
basic goods of citizenship (education, health care , housing, child care). Of course, the precise
nature and extent of these rights will be politically contested and constructed. But a democratic
society cannot leave as totally open the minimal institutional basis of democracy a
feminist, African- American, or lesbian activist is likely to value the shared historical narratives that partly constitute such group identities.

democratic society cannot be agnostic as to the value of freedom of speech, association, and universal suffrage. Social movements fighting for an expansion of civil, political,
and social rights, rarely, if ever, rest their arguments on appeals to epistemological truths whether foundational or anti-foundational. To remain democratic, their policy goals
cannot be so specific that they preclude political argument about both their worth and how best to institutionalize them. If social movements in a 58 democratic society deemed
that every policy defeat meant a betrayal of basic democratic principles, there would be no give-and-take or winners and losers within democratic politics. But if a government
were to abolish freedom of speech and competitive elections, or deny a social group basic rights, it would be reasonable for an observer to judge that democratic principles had

Democratic political movements and coalitions struggle to construct shared meanings


about those political, civil, and social rights that should be guaranteed to all citizensand they often work to
expand the types of persons to be recognized as citizens (such as excluded immigrants). Such arguments are inevitably grounded in
been violated.

normative arguments that go beyond merely asserting the import of flux, difference,
and anti-essentialism. The civil rights movement did not demand equal rights for all solely as an agonal assertion of the will of the excluded; they desired to gain for persons
of color an established set of civil and political rights that had been granted to some citizens and denied to others. The movement correctly assumed that the exclusion of
citizens from full political and civil rights violated the basic norms of a democratic society. Thus,

postmodern epistemological

commitments to flux and openness cannot in-and-of-themselves sustain the fixed moral
positions needed to sustain a radical democracy. Post-structuralist theorists openly proclaim their hostility to all philosophical
meta-narratives. They reject comprehensive conceptions of how society operates and the type of society that would best instantiate human freedom. But poststructuralists go beyond rejecting meta-narratives; they insist that only an anti-foundational epistemology can ground a politics of emancipation. For Butler,
Brown, and Connolly, not only do meta-discourses invariably fail in their efforts to ground moral positions in a theory of human nature or human reason. They also assert that

politics, they claim, can only be


sustained by a recognition of the inconstant signification of discourse and the ineluctable flux of personal and group
identity.41 Rejecting the authoritarian, celebration of the ubermensch by Nietzsche, they offer a post-Nietzschean, amoral conception of democracy as an openan agonal politics of democratic we formation can alone sustain democratic society. This agonal

ended project of defining a self and community that is constantly open to the desires of others. These theorists constantly reiterate the definitiveness (dare we say
foundational truth) of this grounding of democracy,

despite the historical reality that social movements often

contest dominant narratives in the name of a stable alternative narrative of a


democratic and pluralist community. One might well contend that the post-structuralist political stance
is guilty of a new meta-narrative of bad faith, that of anti-foundationalism. According to this antifoundational politics, a true democrat must reject any and all a priori truths allegedly grounded upon the nature of human reason or human nature. A committed democrat may
well be skeptical of such neo-Kantian or neo-Hegelian conceptions of freedom; but, many committed democrats justify their moral commitments using these philosophical
methods. A democrat might also reject (or accept) the arguments of a Jurgen Habermas or Hans Georg Gadamer that the structure of human linguistic communication contains
within it the potential for a society based on reasoned argument rather than manipulation and domination. But there are numerous other philosophically pragmatic ways to
justify democracy, even utilitarian ones. Political democrats may well disagree about the best philosophical defense of democracy. But, invariably, practicing democrats will
defend the belief (however philosophically proved or justified) that democratic regimes best fulfill the moral commitment to the equal worth of persons and to the equal
potential of human beings to freely develop and pursue their life plans. To contend that only an anti-foundationalist, anti-realist epistemology can sustain democracy is to argue

Brown and
Butler both spoke at a spring 1998 academic conference at the University of California at Santa Cruz where some attributed reactionary and left cultural
conservatism to belief in reactionary foundationalist humanism.42 Post-structuralism cannot
precisely for a foundational metaphysical grounding for the democratic project. It is to contend that ones epistemology determines ones politics. Hence,

escape its own essentialist conception of identity. For example, Butler contends in Feminist Contentions that
democratic feminists must embrace the post-structuralist nondefinability of woman as
best suited to open democratic constitution of what it is to be a woman.43 But this is itself a closed position and runs
counter to the practices of many democratic feminist activists who have tried to develop
a pluralist, yet collective identity around the shared experiences of being a woman in a
patriarchal society (of course, realizing that working-class women and women of color experience patriarchy in some ways that are distinct from the
patriarchy experienced by middle-class white women). One query that post-structuralist theorists might ask
themselves: has there ever existed a mass social movement that defined its primary
ethical values as being those of instability and flux? Certainly many sexual politics activists are cognizant of the fluid
nature of sexuality and sexual and gender identity. But only a small (disproportionately university educated) segment of the womens and gay and lesbian movement would
subscribe to (or even be aware of) the core principles of post-structuralist anti-essentialist epistemology. Nor would they be agnostic as to whether the state should protect their

Post-structuralist theorists cannot avoid justificatory arguments for why


some identities should be considered open and democratic and others exclusionary and
anti-democratic. That is, how could post-structuralist political theorists argue that Nazi or Klan ethics are antithetical to a democratic societyand that a
rights to express their sexuality.

democratic society can rightfully ban certain forms of agonal (e.g. harassing forms of behavior against minorities) struggle on the part of such anti-democratic groups.

Alt Fails
The alt fails---deriding all attempts at action as freezing becoming no way
to deal with difficult political choices---we also control terminal uniqueness
because they cant convince others to abandon liberal subjectivity
Schartz 8
(Joseph Schwartz 8, Professor of Political Science at Temple University, The Future of
Democratic Equality, 56-62)
politics of radical democratic pluralism cannot be securely grounded by a whole-hearted
epistemological critique of enlightenment rationality. For implicit to any radical
democratic project is a belief in the equal moral worth of persons; to embrace such a
position renders one at least a critical defender of enlightenment values of equality and
justice, even if one rejects enlightenment metaphysics and believes that such values are often embraced by nonWestern cultures. Of course, democratic norms are developed by political practice and 60 struggle
A

rather than by abstract philosophical argument . But this is a sociological and historical reality rather than a trumping philosophical
proof. Liberal democratic publics rarely ground their politics in coherent ontologies and
epistemologies; and even among trained philosophers there is no necessary connection
between ones metaphysics and ones politics. There have, are, and will be Kantian conservatives (Nozick), liberals (Rawls), and
radicals (Joshua Cohen; Susan Okin); teleologists, left, center, and right (Michael Sandel, Alasdair McIntyre, or Leo Strauss); anti-universalist feminists (Judith Butler, Wendy
Brown) and quasi-universalist, Habermasian feminists (Seyla Benhabib, Nancy Fraser). Post-structuralists try to read off from an epistemology or ontology a politics; such
attempts simply replace enlightenment meta-narratives with postmodern (allegedly anti) meta-narratives. Such efforts represent an idealist version of the materialist effort

A democratic political theory


must offer both a theory of social structure and of the social agents capable of building such a
society. In exchanging the gods of Weber and Marx for Nietzsche and Heidegger (or their epigones Foucault and Derrida), poststructuralist
theory has abandoned the institutional analysis of social theory for the idealism of abstract
philosophy. Connolly, Brown, and Butler reject explicit moral deliberation as a bad faith Nietzschean attempt at ressentiment. Instead, they celebrate the amoral,
which post-structuralists explicitly condemnto read social consciousness off of the structural position of the agent.

yet ethical strivings of a Machiavellian or Gramscian realist war of position.44 Sheldon Wolin, however, has written convincingly of how Machiavelli can be read as an ethical
realist, a theorist of moral utilitarianism.45 Even a Machiavellian or Gramscian political realist must depend upon moral argument to justify the social utility of hard political
choices. That is, if one reads both as ethical utilitarians who believe that, at times, one must dirty ones hands in order to act ethically in politics, then they embrace a utilitarian,
just war theory of ethical choice. According to this consequentialist moral logic, bad means are only justifiable if they are the only, unavoidable way to achieve a greater
ethical goodand if the use of such bad means are absolutely minimized. Such

hard political choices yield social policies

and political outcomes that fix identities as well as transform them . Not only in regard to epistemological
the poststructuralist analysis of the death of man and the death of the subject also radically preclude
questions has post-structuralist theory created a new political metaphysics which misconstrues the nature of democratic political practice;

meaningful political agency . As with Michel Foucault, Butler conceives of subjects as produced by powerknowledge discourses. In Butlers view,
the modernist concept of an autonomous subject is a fictive construct; and the very act of adhering to a belief in autonomous
human choice is to engage in exclusion and differentiations, perhaps a repression , that is
subsequently concealed, covered over, by the effect of autonomy.46 That is, the power of discourse, of language and the unconscious, produces subjects. If those
subjects conceive of themselves as having the capacity for conscious choice, they are
guilty of repressing the manner in which their own subjectivity is itself produced by
discursive 61 exclusion: if we agree that politics and a power exist already at the level at which the subject and its agency are articulated and made possible, then
agency can be presumed only at the cost of refusing to inquire into its construction.47 Susan Bickford pithily summarizes the post-structuralist rejection of the modernist
subject: power is not wielded by autonomous subjects; rather through power, subjectivity is crafted.48 Bickford grants that post-structuralism provides some insight into how

after post-structuralism exposes the lie


of the natural (that there are no natural human identities), socially constructed
group and individual identity is culturally constructed. But Bickford goes on to contend that

modern individuals still wish to act in consort with others and to use human
communication to influence others : people generally understand themselves as
culturally constituted and capable of agency .49 For if there is no doer behind the deed,
but only performative acts that constitute the subject, how can the theorist (or activist) assign agency or moral
responsibility to actors who are constituted by discursive practices. (Discursive practices engaged in by
whom, the observer may ask?) Butler insists that not only is the subject socially constituted by power/knowledge discourses, but so too is the ontologically reflexive self of the
enlightenment. Now if this claim is simply that all social critics are socially-situated, then this view of agency is no more radical a claim than that made by Michael Walzer in his
conception of the social critic (or agent). Walzer argues that even the most radical dissident must rely upon the critical resources embedded within his own culture (often in the

Asserting that critical


capacities are themselves socially constructed provides the reader with no means by
which to judge whether forms of resistance are democratic and which are not. That is, no
matter how hard one tries to substitute an aesthetic, ironic, amoral ethical sensibility for morality, the
almost-hidden interstices of that culture). Effective critical agency cannot depend on some abstract universal, external logic.50

social critic and

political activist cannot escape engaging in moral argument and justification

with fellow citizens . Butler astutely notes that resistance often mirrors the very powerknowledge discourses it rejectsresisting hegemonic norms without
offering alternative conceptions of a common political life. But Butler seems to affirm the possibility (by whom?) of effective rejection of such norming by performative

the resignification of performative discursive constructions provides no


criteria by which to judge whether a given resignification is emancipatory or
repressive.51 And just who (if not a relatively coherent, choosing human subject) is performing the resignification. Furthermore, if all forms of
identity and social meaning are predicated upon exclusion, then the democratic
theorist needs to distinguish among those identities which exclude in a democratic
way and those which exclude in an anti-humanist, racist, and sexist manner. Some social
identities are democratic and pluralist, such as those created by voluntary affiliations.
But other identities, such as structural, involuntary class differences and racial and
sexual hierarchies, must be transformed, even eliminated, if democracy is to be
furthered. And how we behaveor performcan subvert (or reinforce) such undemocratic social structures. But if these social structures are immutably inscribed
resignification. But

by62 performative practices, then there can be no democratic resistance. In her call for an ironic politics of performative resistance, Butler seems to imply that human beings
have the capacity to choose which performative practices to engage inand from which to abstain. If this is the case, then a modernist conception of agency and moral
responsibility has covertly snuck its way back into Butlers political strategy.52

AT: Change us = Change Institutions

The alternatives emphasis on self-artistry is anti-democratic and has no


hope of effectuating political change
Myers 13
(Ella Myers 13, Assistant Professor of Political Science and Gender Studies at the University of
Utah, 2013, Worldly Ethics: Democratic Politics and Care for the World, p. 44-45)
Unfortunately, Connolly is inconsistent in this regard, for he also

positions Foucauldian self- artistry as an essential


preliminary to, and even the necessary condition of, change at the macropolitical
level.104 That is, although Connolly claims that micropolitics and political movements work
in tandem, each producing effects on the other,105 he sometimes privileges action by the self on itself as a
starting point and necessary prelude to macropolitical change. This approach not only
avoids the question of the genesis of such reflexive action and its possible harmful
effects but also indicates that collective efforts to alter social conditions actually await
proper techniques of the self . For example, in a rich discussion of criminal punishment in the United States, Connolly
contends that today the micropolitics of desire in the domain of criminal violence has
become a condition for a macropolitics that reconfigures existing relations between
class, race, crime and punishment.106 Here and elsewhere in Connollys writing the sequencing renders
these activities primary and secondary rather than mutually inspiring and reinforcing.107

It is a mistake to grant chronological primacy to ethical self-intervention , however.


How, after all, is such intervention, credited with producing salient effects at the
macropolitical level, going to get off the ground , so to speak, or assuredly move in the
direction of democratic engagement (rather than withdrawal , for example) if it is not tethered,

from the beginning, to public claims that direct attention to a specific problem,
defined as publicly significant and changeable? How and why would an individual
take up reflexive work on the desire to punish if she were not already attuned, at least partially,
to problems afflicting current criminal punishment practices? And that attunement is

fostered, crucially, by the macropolitical efforts of democratic actors who define a public
matter of concern and elicit the attention of other citizens.108 For

reflexive self- care to be democratically

significant, it must be inspired by and continually connected to larger political


mobilizations . Connolly sometimes acknowledges that the arts of the self he celebrates are
not themselves the starting point of collaborative action but instead exist in a dynamic,
reciprocal relation with cooperative and antagonistic efforts to shape collective
arrangements. Yet the selfs relation with itself is also treated as a privileged site , the very
source of democratic spirit and action. This tendency to prioritize the selfs reflexive relationship over
other modes of relation defines the therapeutic ethics that ultimately emerges out of Foucaults and, to a lesser
degree, Connollys work. This ethics not only elides differences between caring for oneself and
caring for conditions but also celebrates the former as primary or, as Foucault says,

ontologically

prior . An ethics centered on the selfs engagement with itself may have value, but it

is not an ethics fit for democracy .

Fails
The alt fails and locks in the status quo approach to surveillance ---the
embrace of <contingency, radical democracy, alterity> discounts the role of
political institutions in modern life and reduces politics to an individualistic
project of ethical self-improvement and becoming
Lavin 6
(Chad, teaches political theory at Tulane University, Fear, Radical Democracy, and Ontological
Methadone, Polity (2006) 38, 254-275
radical democracy suggests

While liberalism exhorts us to take comfort in the promises and assurances of a fixed identity,
revealing this identity for
the reification that it is so that we can move beyond a tolerant modus vivendi and toward an ethical comportment of generosity and meaningful democracy. They prescribe, as a
cure for postmodern agency panic, an enthusiastic embrace of the contingency of everyday life, a series of practices of the self that force an examination of the existential

The problem of fear, they


suggest, resides in a dogmatic clinging to an untenable and disintegrating myth of
subjectivity with an unrealizable promise of control. Their response to the fear of homelessness runs directly counter to the
resentment felt by subjects of liberal capitalism: Don't adhere to a manufactured map! Learn to be at home in homelessness!

conspiracy theorist: with a model of contingency rather than conspiracy, there are no villains, thus no reservoir for depositing and segregating one's fear. Rather than depositing
this fear at the feet of a scapegoat, face it; overcome it. While challenging the liberal fetish for reifying a historical production and thus closing the door on possibilities for
alternative subjects to emerge, radical democrats also acknowledge the work done by liberalism to remove metaphysical obstacles to the expression of agency. As such, their
claim is not antiliberal so much as it is postliberal, interrogating the limits of the liberal subject in pursuit of a more open approach to identity and difference. For the identities and
attachments that have been forged through liberalism are anything but fetters to the unhindered exercise of democracy and production of difference. They are not only
productive of many of the freedoms that we enjoy today in the liberal world (e.g., civil rights and liberties, relative government accountability), they also offer valuable solace
from a hostile and increasingly unpredictable existence. But because the prescription for those suffering from agency panic tends to be found in the capitalist myths of merit,

we might look at liberalism as the opiate of the people, easing our pain, but
preventing us from doing the work necessary to attend to the source of our ills. Perhaps liberalism, then, for all its value and appeal, is a habit we need to kick. Radical
autonomy, and authenticity,

democrats recommend sobering up and facing our fears head-on. But where's the Betty
Ford Center for radical democracy? If the fundamentalist drive to reify assumptions and the democratic drive to challenge them are two
responses to the same panic that results from social pluralization (as Connolly convincingly claims), the question remains: why do some
subjects flee from this fear (through fundamentalism), while others draw from it strength
and forbearance (through radical democracy)? What resources allow some subjects to
respond to revealed inadequacies of their foundational beliefs with a gracious humility
while others respond by desperately clinging to and fundamentalizing the contested
position? The short answer is that the democratic move is not an easy one to make. While liberal cognitive maps
respond to the complexity of contemporary power networks with a promise of predictability and stability through the claims to fixed identity, rational control, and market

radical democracy promises precisely what subjects of liberalism fear: enduring


instability. The ethos of forbearance asks subjects not only to accept social and political instability today, but ontological instability forever after (whereas the
fundamentalizing moves promise order not only in the future, but now). By introducing contingency into the very substance
of being, this approach leaves us in indeterminacy without even a promise of finding
answers; it is "tentative, experimental," emphasizing the need for perpetual work of
"cultivation" without the promise of attaining a stable system ;41 it is "a risky and ambiguous enterprise" always
efficiency,

threatening to destroy self-confidence as soon as it is built;42 it offers "no necessary political consequences," but only possible gains in freedom.43 Going back to Hobbes,

radical democrats suggest that we should embrace rather than run from or even merely
tolerate the very conditions we fear: unknowability, instability, and discord. Further, Connolly
and Butler insinuate unmistakably that liberal authenticity has been thoroughly debunked , still adhered to only by
potential or actual fascists who (at best) have been thoroughly colonized by the culture industry and the ideological apparatuses of the state or (at worst) have no sympathy for

They thus imply that it is the


social critic who bears the mantle of democracy, and that it is not fear but a stubborn
authoritarian desire that obliges resistors to slap the table to illustrate its non-discursiveness and pound their chests to
prove their individuated, corporeal, nonporous existence. (A spectre is haunting criticismthe spectre of Leninism.) We
the contemporary movements interested in expanding the democratic web to include currently disenfranchised populations.

can almost hear radical democrats responding that liberal individualists have nothing to
lose but their chains. But if the reigning paradigms of identity and order offer solace
from an increasingly indeterminate and unpredictable world (complete with massive
layoffs, abandonment of traditional gender norms, declining national sovereignty, and
other existential disintegrations), then by what standard can radical democrats call upon
the subjects of liberal capitalism to reject the myths that make lives livable? These calls
might be the ontological counterpart of ham-fisted approaches to drug abuse, exhorting
addicts to take responsibility for themselves and quit cold turkey, with little attention to
the social and psychological dynamics that lead millions of users to abuse narcotics, hallucinogens,
opiates, and amphetaminesbe they consumers of heroin, religious fervor, Xanax, or extreme corn chips and suburban assault vehicles. The sales pitch for
radical democracy rarely accounts for the degree to which citizens are interpellated as
subjects of fear. Connolly is more than aware of the interpellations that subjects of capitalism receive; large sections of The Ethos of Pluralization are dedicated
to the relationship between economic anxiety and the drive to fundamentalism, and Why I Am Not a Secularist is largely an indictment of secularism (and much of American
liberalism) for failing to account for the habits inscribed on the body that rational argumentation cannot erase.44 His attention to what he calls "the visceral register," and his

Unfortunately,
Connolly's intellectual trajectory has shifted away from the social and political
recommendation that the left take a lesson from William Bennett on how to appeal to this register is both compelling and timely.45

apparatuses which inculcate these habits and toward the level of neurology to explain
the inward mechanisms of habits and ethics ;46 while he never denies the relevance of
social and political institutions (he merely argues that "too many cultural theorists" ignore the biological components of thinking and ethics), his
trajectory has led him to speak more about what practices of the self might lead to more
democracy and less about the institutional production of subjects who actually fear
democracy .47 As White convincingly argued well before this latest turn toward biological studies, Connolly takes attitudes to have
primacy over issues of justice, leading him to prematurely curtail pursuit of the obviously
compatible institutional dimension of criticism.48 The common scoundrel in such critiques of liberalism, the prototypical
antidemocratic citizen, is typified by Archie Bunker and his white working-class buddies. Resentful, socially and economically conservative, and easily seduced by reactionary
propagandists, this was the face of fundamentalism before 9/11. Notably, this is a population whose economic viability and (therefore) ability to live up to conventional norms of

Connolly notes the familiar


targets: civil rights, Vietnam, feminism, dismantling of the welfare state,
environmentalism, and a decline in the availability of industrial jobs.49 Along with these
transformations in established relations of power, we can also add to this list (which
Connolly may or may not have intended to be exhaustive) the supposed death of grand
narratives declared by critical intellectuals. Traditional cognitive maps are not only
becoming obsolete in the wake of shifting terrain, but they are being criticized for being
defective from the get-go. Given this, is the rise in public fear a surprise? As Jameson's, Melley's, and Furedi's attention to the paucity of concepts
suggests, isn't the declining availability of cognitive maps to those threatened by shifting networks of power a prime candidate for Connolly's list?50 If
fundamentalisms feed on the anxieties endemic to late capitalist (dis)order; if market
ideology, conspiracy theories, and aggressive nostalgia are degraded attempts to construct cognitive maps that
might provide threatened subjects with a sense of agency; what does radical democracy
offer in their stead? Traditional fundamentalisms are popular precisely because they treat the agency panic and existential angst accelerated by global
masculinity have been taking it in the gut in recent decades. Identifying the threats that have constituted this subject,

capitalism and postmodern culture. Again, Connolly recognizes the absence of viable democratic supplements to the decline of grand narratives, and he laments that the world's
most zealous moralizers have effectively filled that void. But claims that rethinking liberalism's ontological commitments cannot be an afterthought of political emancipation (but
must be coterminous with or preliminary to it), betray an inattention to how a postmodern ethos is intimately tied to the various psychological, economic, and political securities

While Connolly's diagnosis of fundamentalist epidemics clearly


highlights their social and economic roots, he presents a postmodern sensibility as a
treatment for the anxieties of identity without considering the role that this move may
have played in actually producing those anxieties by threatening the cherished existence
of those perhaps most in need of their help.51 My concern is not the diagnosis, but the
prescription, as it is for a regimen that few have any interest in adopting. Radical
democracy is premised on the idea that its ontology and ethics are necessary to an
that cognitive maps often provide.

enactment of the pluralism that liberalism purports to offer, transcending mere tolerance
toward a generous fostering of otherness and a meaningfully democratic way of life. At the
same time, however, it is insensitive to the profound threat that genealogy poses to those who
receive their only respite from a truly tenuous existence from their myths. While its pitch suggests that it
is ontologically more defensible, ethically more admirable, and practically more conducive to a generous ethical-political comportment (contentions with which I agree,

), it does not consider the sheer difficulty (and often terror) in adopting such a
position. Certainly, it does not speak to the members of society who are the most
economically, politically, and/or socially disadvantaged, whose daily survival is so
tenuous as to not provide the luxury for the practices of self-cultivation which it
advocates. I am far from the first to call attention to the dangers of and aversions to genealogy. The aristocratic baggage of Nietzscheanism, from which radical
incidentally

democrats have vigorously sought to distance themselves, stems largely from Nietzsche's acknowledgement of the difficulties of abandoning foundations. No friend of pity,
Nietzsche nevertheless prescribes revaluing our constitutive values only to a privileged class, maintaining that we cannot ask lambs to be birds of prey.52 Similarly, William
James discusses the differential capacities of modern subjects to be at home in homelessness, expressing concern for those who might not be ready or able to make such a

the "healthy" has a responsibility to attend to the "sick" soul,


and that some are not ready, willing, or able to abandon their most cherished myths : "Some
move. He argues (with unfortunate terminology) that

constitutions need them too much."53 Returning to our liberal fundamentalist, Rorty emphasizes that "most people" are not interested in facing the contingency of their identities,
and that "there is something very cruel about" revealing their contestability and groundlessness.54 While these arguably elitist arguments might sit uncomfortably with the
radically democratic impulses of Connolly and Butler, which is more democratic: Throwing everybody into the same pool? Or realizing that not everybody knows how to swim?
Or realizing that some citizens, having seen their parents devoured by sharks, might think the waters unsafe? Connolly reads Rorty's courteous capitulation to fear as an

I do not mean
to suggest that consistency would oblige radical democrats to respect discourses
hostile to pluralization (Connolly addresses this issue repeatedly). Nor do I want to imply that we should refrain
from attacking the antidemocratic presumptions of various fundamentalisms. But I am
concerned that the approach to fundamentalism in this literature is something much
closer to hostility than sympathy (to say nothing of empathy). For all of its talk of generosity, radical
democracy remains quite stingy with regard to this issue. Accepting that there remains a
compatibility between foundational thought and undemocratic or authoritarian politics,
the apparent expectation that the articulation of this compatibility might convince any
lingering fundamentalists of their error is distressing . In fact, Connolly effectively labels
those lacking in the material, philosophical, or psychological resources necessary for
the move he recommends latent brownshirts, thus betraying not only a commitment to
liberal voluntarism, but also a striking lack of generosity toward those still in need of
comforting maps.56 Radical democrats clearly recognize the material conditions that may not provide the comfort necessary to develop an ethos of
generosity. As diagnosticians, they are certainly attuned to the tactics applied by institutions
(markets, ideologies, militaries) to subjects and the constant interpellation of subjects of
fear. Nevertheless, the prescribed micropolitics of desire summons a heroic capacity to
respond generously to the myriad threats we encounter. Recommending that subjects abandon the cognitive maps
(Christianity, conspiracy theories, market fundamentalism) that allow them to find their bearings in increasingly complicated and vast networks of power, they
ultimately reduce liberal fundamentalism to a problem of individual priorities and ethical
failures rather than a medical condition requiring professional treatment or (better) a
public health issue not easily or appropriately tied to any individual's preferences. As in
abandonment of irony precisely where it is most important. Whereas Rorty coddles subjects of fear, Connolly scolds them for their cowardice.55

contemporary U.S. drug policy rhetoric, the addiction is all-too-easily reduced to an


issue of personal responsibility, taking focus away from the institutions responsible for
the daily interpellation of subjects of fear. Conclusion Despite a relatively mature Constitution,
Americans are no strangers to instability. In recent decades, the assurances of job security and the welfare state have fallen victim to a
vicious market ideology; traditional codes of masculinity and femininity have become increasingly unrealizable with Martha Stewart and J. Lo doing their best to humiliate the
ladies, Bill Gates and Vin Diesel the gentlemen; and illusions of effective agency have been ravaged by unprecedented technological and geopolitical transformations. In this

though radical democrats denounce the comforting liberal myth of the autonomous
individual as a ruse of antidemocratic power, its ideology of identity, authenticity, and
responsibility provides relief from the ontological homelessness endemic to a world with
ever fewer uncontested narratives. Liberalism allows us to personalize and segregate various anonymous hostilities, facilitating displacements
condition,

of general anxieties onto welfare moms, homosexual teachers, professional women, non-white street criminals, Zionist Occupied Governments, and Islamic fundamentalists.
While the reactionary grasp at liberal fundamentalism certainly constitutes an obstacle to a democratic politics of difference, it is also the case that subjects of late capitalism are
interpellated as subjects of fear, reared to understand every component of society with suspicion. Navigating the breathtaking and impersonal forces of bureaucratic capitalism
with categories which emphasize (indeed, almost exclusively mention) the powers of isolated individuals, we are led to an ever more hopeless situation. As history moves on
and our cognitive maps seem less and less relevant for helping us chart networks of power, we feel an overwhelming sense of powerlessness. The ideology of autonomy,
authenticity, and responsibility seems an entirely logical aid for coping with this agency panic. Indeed, liberalism both creates and then assuages the fears of late capitalism. In

radical democrats
trenchantly reveal liberal ideology as an addictive and distorting source of relief. But in
prescribing a micropolitics of desire to kickstart the stalled project of democratization,
they locate the work of politics in the contested site of the individual and discount the
degree to which these individuals actually fear the unpredictable and radically
democratic order being promised. Contesting the validity of the cognitive maps most commonly used to steer a course through institutions of
social and political power, they ask subjects to abandon the anesthetizing components of liberal
fundamentalism for a world composed of unstable identities and provisional reconciliations that are grounded in a set
of ontological commitments that are weak at best. But as all but the most steadfast purveyors of simplification in
this age of Panic, in which our surroundings appear at least comparably if not more alive and efficacious than our selves,

the so-called war on drugs realize, addiction is inscribed on the body, and requires both
desire and treatment to be overcome. Radical democrats realize why we are identity
junkies. But where, oh where, are the ontological treatment centers and methadone
clinics?

Queerness

Law Good

K Challenge Heteronorm
Even if the law isnt perfect, sexual activists have been able to strategically
use traditional political areas in queer ways to challenge heteronormativity.
Lind 10
(Amy Lind 10, Mary Ellen Heintz Endowed Chair and Professor of Womens, Gender, and
Sexuality Studies at the University of Cincinatti, Development, Sexual Rights and Global
Governance, p 17-18)
In Chapter 8, Sangeeta Budhiraja, Susana T. Fried and Alexandra Teixeira address tensions among identity-based organizing and
sexual rights advocacy. On one hand, activists around the world have addressed "lesbian and gay," and later,
"lesbian, gay and bisexual (LGB)," then "lesbian, gay, bisexual and transgender (LGBT)" or "lesbian, gay, bisexual, transgender
and questioning and or queer" (LGBTQ)" rights, the

"alphabet soup" approach, as a way to make sexual


and gender minorities visible in national and international political and development
arenas. Yet, more recently, scholars and activists have turned toward a sexual rights framework as
a way to overcome essentialisms in positing individuals as singular identities that are
often homogenized and universalized in development discourse and practice. Drawing
upon their former advocacy work at the International Gay and Lesbian Human Rights
Commission, the authors demonstrate the difficulties of naming and finding a common
ground on a global level as well as the useful- ness of utilizing a broad-based sexual rights framework for thinking about
sexual identity, gender identity, human rights, and development. They argue that a sexual rights framework
allows for greater cross-movement organizing, gives deference to local activists'
preferred ways of thinking of and expressing any gender which falls outside of social
and cultural norms, and encourages modes of organizing that do not reify gender and
sexual binaries. Yet activists must necessarily use, perhaps strategically, normative
categories of gender and sexuality in order to achieve their concrete goals for legal and
policy reform, a dilemma that they highlight through- out their chapter. In Chapter 9, Petra Doan discusses how
increasing the visibility of gender-variant individuals in the Middle East, a region often
characterized in development dis- course in orientalist terms as patriarchal and
oppressive to women, might actually "queer" the development process and stimulate
change on a broader scale. For Doan, genderqueerness does not begin or end in the West;
rather, it has always been part of Middle Eastern societies, but it has been through powerful modern dis-
courses such as that of development which have problematized these identities as
abnormal or deviant. Despite colonialist legacies, it has been through the strategic
utilization of these modern discourses that gender-variant individuals in the region have
found creative ways to organize collectively and fight for their rights .

Other

Alt Cedes Politics


The alt cedes politics to the right and reinscribes gender roles
McCluskey 8
(Professor of Law and William J. Magavern Faculty Scholar @ SUNY Buffalo Law (Martha, How
Queer Theory Makes Neoliberalism Sexy, Buffalo Legal Studies Research Paper No. 2008-15)
Queer theory's anti-moralism works together with its anti-statism to advance not simply "politics," but
a specific vision of good "politics" seemingly defined in opposition to progressive law
and morality. This anti-statist focus distinguishes queer theory from other critical legal theories that bring questions of power to bear on moral ideals of justice.
Kendall Thomas (2002), for example, articulates a critical political model that sees justice as a problem of "power,
antagonism, and interest," (p. 86) involving questions of how to constitute and support individuals as citizens with interests and actions that count as
alternative visions of the public. Thomas contrasts this political model of justice with a moral justice aimed at discovering principles of fairness or institutional processes based in
rational consensus and on personal feelings of respect and dignity. Rather than evaluating the moral costs and benefits of a particular policy by analyzing its impact in terms of
harm or pleasure, Thomas suggests that a political vision of justice would focus on analyzing how policies produce and enhance the collective power of particular "publics" and

From this political perspective of justice, neoliberal economic ideology is


distinctly moral, even though it appears to be anti-moralist and to reduce moral principles to competition between self-interested power. Free-market
economics rejects a political vision of justice, in this sense, in part because of its expressed antistatism: it turns contested normative questions of public power into objective rational
calculations of private individual sensibilities. Queer theory's similar tendency to
"counterpublics" (pp. 915).

romanticize power as the pursuit of individualistic pleasure free from public control
risks disengaging from and disdaining the collective efforts to build and advance
normative visions of the state that arguably define effective politics. Brown and Halley (2002), for
cite the Montgomery bus boycott as a classic example of the left's problematic march
into legalistic and moralistic identity politics. In contrast, Thomas (2002) analyzes the Montgomery bus boycott as a positive example of a political
instance,

effort to constitute a black civic public, even though the boycott campaign relied on moral language to advance its cause, because it also emphasized and challenged normative

By glorifying rather than deconstructing the neoliberal dichotomy


between public and private, between individual interest and group identity, and between
demands for power and demands for protection, queer theory's anti-statism and antimoralism plays into a right-wing double bind . In the current conservative political
context, the left appears weak both because its efforts to use state power get constructed
as excessively moralistic (the feminist thought police, or the naively paternalistic welfare state) and also because its efforts to resist
state power get constructed as excessively relativist (promoting elitism and materialism
instead of family values and community well-being). The right, on the other hand, has it both ways,
asserting its moralism as inherent private authority transcending human subjectivity (as
efficient market forces, the sacred family, or divine will) and defending its cultivation of
self-interested power as the ideally virtuous state and market (bringing freedom, democracy,
equality to the world by exercising economic and military authoritarianism). From Egalitarian Politics to Renewed Conservative
Identity Queer theory's anti-statism and anti-moralism risks not only reinforcing right-wing
ideology, but also infusing that ideology with energy from renewed identity politics . Susan
Fraiman (2003) analyzes how queer theory (along with other prominent developments in left academics and culture) tends to construct left
resistance as a radical individualism modeled on the male "teen rebel, defined above all by his strenuous
alienation from the maternal" (p. xii). Fraiman observes that this left vision relies on "a posture of flamboyant
unconventionality [that] coexists with highly conventional views of gender [and] is , indeed,
ideas of citizenship (p. 100, note 14).

articulated through them " (p. xiii). Fraiman links recent left contempt for feminism to a
romantic vision of "coolness ... epitomized by the modem adolescent boy in his anxious, self-conscious and
theatricalized will to separate from the mother" who is by definition uncoolcontrolling,
moralistic, sentimental and not sexy. (p. xii). Even though queer theory distinguishes itself
from feminism by repudiating dualistic ideas of gender, its anti-foundationalism covertly
promotes an essentialist "binary that puts femininity, reproduction, and normativity on
the one hand, and masculinity, sexuality, and queer resistance on the other" (p. 147). This
binary permeates queer theory's condemnation of "governance feminism." (Brown and Halley, 2002;
Wiegman, 2004) a vague category mobilizing images of the frumpy, overbearing, unexciting, unfunny, and not-so-smart
"schoolmarm" (Halley, 2002) whose authority will naturally be undermined when real "men"
appear on the scene. Suggesting the importance of gender conventions to the term's power, similar phrases do not seem to have gained comparable
academic currency as a way to deride the complex regulatory impact of other specific uses of state authority -for instance postmodernists do not seem to widely denounce
"governance anti-racism," "governance socialism," "governance populism," "governance environmentalism" or "governance masculinism" (though Brown and Halley do criticize

Queer attraction to an adolescent masculinist


idea of the "cool' dovetails smoothly with the identity politics of the right. Right-wing
politics and culture similarly condemn progressive and feminist policies with the term
"nanny state" (McCluskey, 2000; 2005a). The "nanny state" epithet enlists femaleness or femininity as shorthand to make some government
progressive law reform more generally with the term "governance legalism" (p. 11)).

authority feel bad to those comfortable with or excited by a masculinist moral order, it adds to this sentimental power by coding the maternal authority to be resisted as a "nanny"

e
"nanny state" slur tells us that a rougher and tougher neoliberal state, market, and family
will bring the grown-up pleasures, freedom, and power that are the mark and privilege of
ideal manhood. The "nanny state" is not an isolated example of the use of gender identity to disparage
progressive or even centrist policies that are not explicitly identified as feminist or
gender-related. For example, "girlie-man" gained currency in the 2004 presidential election to disparage opposition to George W. Bush's right-wing economic and
(rather than simply a "mommy"), enlisting identities of class, ageand perhaps race and nationalityto enhance uncritical suspicions of disorder and illegitimacy. Th

national security policies (Grossman and McClain, 2004), and and in 2008 critics of presidential candidate Barack Obama similarly linked him to disparaging images of femininity

These terms open a window into the connections between economic


libertarianism and moral fundamentalism. Libertarianism's anti-statism and anti-moralism requires sharp distinctions between public
(Campanile 2008; Faludi 2008).

and private, morality and power, individual freedom and social coercion. The problem, if we assume these distinctions are not self-evident facts, is that libertarianism must refer

Identity conventions have long helped to do this work

covertly to some external value system to draw its lines.


, albeit in
complex and sometimes contradictory ways. Power appears weak, deceptive, illegitimate, manipulative, controlling, undisciplined, oppressive, exceptional, or naive if it is

Conventional
political theory and culture identifies legitimate authority with an idea of a masculine
power aimed at policing supposedly weaker or subordinate others. A state that publicly depends on and
promotes such power enhances rather than usurps private freedom and security in citizenship,
market, and family, according to the traditional theory of the patriarchal household as model
for the state (see Dubber, 2005). Queer theory updates this pre-modern political ideology into
smart postmodernism and transgressive politics by re-casting its idealized masculine
feminized; but strong, self-satisfying, public-serving, protective, orderly, rational, and a normal exercise of individual freedom if it is masculinized.

power in the image of a youthful and sexy disdain for feminized concerns about social,
bodily, or material limits and support . In her challenge to this queer romanticization of "coolness," Fraiman (2003) instead urges a feminism
that will "question a masculinity overinvested in youth, fearful of the mutable flesh, and on the run from intimacy ... [to] claim, in its place, the jouissance of a body that is aging,
pulpy, no longer intact... a subject who is tender-hearted ... who is neither too hard nor too fluid for attachment; who does the banal, scarcely narratable, but helpful things that
moms' do" (p. 158). Feminist legal theory concerned with economic politics adds to this alternative vision an ideal that advances and rewards the pleasure, power, and public
value of the things done by some of those moms' nannies (McCluskey, 2005a)or by the many others engaged in the work (both paid and unpaid) that sustains and enhances
others' pleasure and power in and out of the home (McCluskey, 2003a; Young, 2001). One means toward that end would be to make the domestic work (and its play and
pleasure) conventionally treated as both banal or spiritual (see Roberts, 1997b) deserving of a greater share of state and market material rewards and resources on a more
egalitarian basis, as Fineman's (2004) vision would do.

Security

Alt Answers

Security Alt Fails


Have to transform systems from the inside out-otherwise rhetoric changes
but not policies.
McCormack 10
(Tara, PhD in International Relations from the University of Westminster, Critique, Security and
Power: The political limits to emancipatory approaches, pg 59-61)
the shift away
from the pluralist security framework and the elevation of cosmopolitan and emancipatory goals has served to enforce
international power inequalities rather than lessen them. Weak or unstable states are
subjected to greater international scrutiny and international institutions and other states have greater freedom to
intervene, but the citizens of these states have no way of controlling or influencing these
international institutions or powerful states. This shift away from the pluralist security framework
has not challenged the status quo, which may help to explain why major international institutions and states can easily
adopt a more cosmopolitan rhetoric in their security policies. As we have seen, the shift away from the pluralist security
In chapter 7 I engaged with the human security framework and some of the problematic implications of emancipatory security policy frameworks. In this chapter I argued that

framework has entailed a shift towards a more openly hierarchical international system, in which states are differentiated according to, for example, their ability to provide human security for their citizens or their
supposed democratic commitments. In this shift, the old pluralist international norms of (formal) international sovereign equality, non-intervention and blindness to the content of a state are overturned. Instead,

theorists
argue that the goal of the emancipation of the individual means that security must be
reconceptualised away from the state. As the domestic sphere is understood to be the sphere of insecurity and disorder, the international sphere represents
greater emancipatory possibilities, as Tickner argues, if security is to start with the individual, its ties to state
sovereignty must be severed (1995: 189). For critical and emancipatory theorists there must be a shift towards a cosmopolitan legal framework, for example Mary Kaldor
(2001: 10), Martin Shaw (2003: 104) and Andrew Linklater (2005). For critical theorists, one of the fundamental problems with
Realism is that it is unrealistic. Because it prioritises order and the existing status quo, Realism attempts to impose a
particular security framework onto a complex world, ignoring the myriad threats to people
emerging from their own governments and societies. Moreover, traditional international theory serves to obscure power relations and omits
international institutions and states have more freedom to intervene in weak or unstable states in order to protect and emancipate individuals globally. Critical and emancipatory security

a study of why the system is as it is: [O]mitting myriad strands of power amounts to exaggerating the simplicity of the entire political system. Todays conventional portrait of international politics thus too often ends

theorists seem
to show a marked lack of engagement with their problematic (whether the international security context, or the Yugoslav breakup and wars). Without concrete engagement and analysis, however, the critical project is
undermined and critical theory becomes nothing more than a request that people behave
in a nicer way to each other. Furthermore, whilst contemporary critical security theorists argue that they
present a more realistic image of the world, through exposing power relations, for example, their
lack of concrete analysis of the problematic considered renders them actually unable to
engage with existing power structures and the way in which power is being exercised in
the contemporary international system. For critical and emancipatory theorists the central place of the values of the theorist mean that it cannot fulfil its
up looking like a Superman comic strip, whereas it probably should resemble a Jackson Pollock. (Enloe, 2002 [1996]: 189) Yet as I have argued, contemporary critical security

promise to critically engage with contemporary power relations and emancipatory possibilities. Values must be joined with engagement with the material circumstances of the time.

Alt Fails
The alt failsthe systems too sticky to simply wish away
Georg SORENSEN, British International Studies Association, 98
[IR Theory after the cold war, p. 87-88]
What, then, are the more general problems with the extreme versions of the postpositivist position? The first problem is
that they tend to overlook, or downplay, the actual insights produced by non-post-positivists, such as, for
example, neorealism. It is entirely true that anarchy is no given, ahistorical, natural condition to which the only possible reaction is adaptation.
But the fact that anarchy is a historically specific, socially constructed product of human
practice does not make it less real. In a world of sovereign states, anarchy is in fact out
there in the real world in some form. In other words, it is not the acceptance of the real existence of
social phenomena which produces objectivist reification. Reification is produced by the
transformation of historically specific social phenomena into given, ahistorical, natural
conditions.21 Despite their shortcomings, neorealism and other positivist theories have produced
valuable insights about anarchy, including the factors in play in balance-of-power dynamics and in patterns of cooperation and
conflict. Such insights are downplayed and even sometimes dismissed in adopting the notion of 'regimes
of truth'. It is, of course, possible to appreciate the shortcomings of neorealism while also recognizing that it has merits. One way of doing so is set forth by Robert
Cox. He considers neorealism to be a 'problem-solving theory' which 'takes the world as it finds it, with the prevailing social
and power relationships . . . as the given framework for action . . . The strength of the problem-solving approach lies in its
ability to fix limits or parameters to a problem area and to reduce the statement of a
particular problem to a limited number of variables which are amenable to relatively
close and precise examination'.22 At the same time, this 'assumption of fixity' is 'also an ideological bias . . . Problem-solving theories (serve) . . .
particular national, sectional or class interests, which are comfortable within the given order'.23 In sum, objectivist theory such as neorealism contains
a bias, but that does not mean that it is without merit in analysing particular aspects of international relations from a particular point
of view. The second problem with post-positivism is the danger of extreme relativism which it contains. If there
are no neutral grounds for deciding about truth claims so that each theory will define what counts as the facts, then the
door is, at least in principle, open to anything goes . Steve Smith has confronted this problem in an exchange with yvind sterud. Smith notes that
he has never 'met a postmodernist who would accept that "the earth is flat if you say so". Nor has any postmodernist I have read argued or implied that "any narrative is as good

if we cannot find a minimum of common standards for deciding about


truth claims a post-modernist position appears unable to come up with a metatheoretically
substantiated critique of the claim that the earth is flat. In the absence of at least some common standards it appears
difficult to reject that any narrative is as good as any other.25 The final problem with extreme post-positivism I wish to address here concerns
change. We noted the post-modern critique of neorealism's difficulties with embracing change; their emphasis is on 'continuity and repetition'. But extreme post-positivists
have their own problem with change, which follows from their metatheoretical position. In short, how can post-positivist ideas and
projects of change be distinguished from pure utopianism and wishful thinking? Post-positivist
radical subjectivism leaves no common ground for choosing between different change
projects. A brief comparison with a classical Marxist idea of change will demonstrate the point I am trying to make. In Marxism, social change ( e.g. revolution) is, of
course, possible. But that possibility is tied in with the historically specific social structures (material and non-material) of the world. Revolution is possible
under certain social conditions but not under any conditions. Humans can change the
world, but they are enabled and constrained by the social structures in which they live.
There is a dialectic between social structure and human behaviour.26 The understanding of 'change' in the
as any other"'.24 But the problem remains that

Marxist tradition is thus closely related to an appreciation of the historically specific social conditions under which people live; any change project is not possible at any time.

'Critical theory allows for a normative choice in favor of


a social and political order different from the prevailing order, but it limits the range of
choice to alternative orders which are feasible transformations of the existing world . . .
Robert Cox makes a similar point in writing about critical theory:

Critical theory thus contains an element of utopianism in the sense that it can represent
a coherent picture of an alternative order, but its utopianism is constrained by its
comprehension of historical processes. It must reject improbable alternatives just as it rejects the permanency of the existing order'.27
That constraint appears to be absent in post-positivist thinking about change, because
radical post-positivism is epistemologically and ontologically cut off from evaluating the
relative merit of different change projects. Anything goes, or so it seems. That view is hard to distinguish from utopianism and wishful
thinking. If neorealism denies change in its overemphasis on continuity and repetition, then radical post-positivism is

Re-Thinking Fails
Individual rethinking fails -governments obey institutional logics that exist
independently of individuals and constrain decision making
Wight 6
(Colin, Sydney IR professor, Agents, Structures and International Relations: Politics as
Ontology, pg 48-50)
relations constitute our identity as social actors. According to
this relational model of societies, one is what one is, by virtue of the relations within which one is
embedded. A worker is only a worker by virtue of his/her relationship to his/her employer and vice versa. Our social being is constituted by relations and our social acts presuppose them. At
any particular moment in time an individual may be implicated in all manner of relations,
each exerting its own peculiar causal effects. This lattice-work of relations constitutes the
structure of particular societies and endures despite changes in the individuals occupying them.
Thus, the relations, the structures, are ontologically distinct from the individuals who enter into them . At a
minimum, the social sciences are concerned with two distinct, although mutually interdependent, strata. There is an ontological difference between people and structures: people are not
relations, societies are not conscious agents. Any attempt to explain one in terms of the other should be rejected. If there is an
ontological difference between society and people, however, we need to elaborate on the relationship between them. Bhaskar argues that
we need a system of mediating concepts, encompassing both aspects of the duality of praxis into which active subjects must fit in order to reproduce it: that is, a system of
concepts designating the point of contact between human agency and social structures.
This is known as a positioned practice system. In many respects, the idea of positioned practice is very similar to Pierre Bourdieus notion of
habitus. Bourdieu is primarily concerned with what individuals do in their daily lives. He is keen to refute the idea that social activity
can be understood solely in terms of individual decision-making, or as determined by surpa-individual objective structures. Bourdieus
notion of the habitus can be viewed as a bridge-building exercise across the explanatory gap between two extremes. Importantly, the notion of a habitus can only be
understood in relation to the concept of a social field. According to Bourdieu, a social field is a network, or a
configuration, of objective relations between positions objectively defined . A social field, then, refers
to a structured system of social positions occupied by individuals and/or institutions
the nature of which defines the situation for their occupants. This is a social field whose form is constituted in terms of the
One important aspect of this relational ontology is that these

relations which define it as a field of a certain type. A habitus (positioned practices) is a mediating link between individuals subjective worlds and the socio-cultural world into which they are born and which they

The power of the habitus derives from the thoughtlessness of habit and habituation, rather
than consciously learned rules. The habitus is imprinted and encoded in a socializing process
that commences during early childhood. It is inculcated more by experience than by
explicit teaching. Socially competent performances are produced as a matter of routine,
without explicit reference to a body of codified knowledge, and without the actors necessarily knowing what they are doing (in the
sense of being able adequately to explain what they are doing). As such, the habitus can be seen as the site of internalization of reality and the externalization of internality. Thus social
practices are produced in, and by, the encounter between: (1) the habitus and its dispositions; (2) the constraints and
demands of the socio-cultural field to which the habitus is appropriate or within ; and (3) the
share with others.

dispositions of the individual agents located within both the socio-cultural field and the habitus. When placed within Bhaskars stratified complex social ontology the model we have is as depicted in Figure 1. The

Society, as field of relations, exists prior to, and is independent of, individual
and collective understandings at any particular moment in time; that is, social action requires the conditions for action.
Likewise, given that behavior is seemingly recurrent, patterned, ordered, institutionalised, and displays a degree
of stability over time, there must be sets of relations and rules that govern it. Contrary to
individualist theory, these relations, rules and roles are not dependent upon either knowledge of
them by particular individuals, or the existence of actions by particular individuals; that is,
their explanation cannot be reduced to consciousness or to the attributes of individuals. These emergent social forms must possess
explanation of practices will require all three levels.

Society, as opposed to the individuals that


constitute it, is, as Foucault has put it, a complex and independent reality that has its own laws and mechanisms of reaction, its
regulations as well as its possibility of disturbance. This new reality is societyIt becomes necessary to reflect upon it, upon its specific
emergent powers. This leads on to arguments for the reality of society based on a causal criterion.

characteristics, its constants and its variables.

Liberal Intl Good


We have a defense of the way we view international relations---game-theory
proves that liberal internationalism is effective
Recchia and Doyle 11
(Stefano, Assistant Professor in International Relations at the University of Cambridge, and
Michael, Harold Brown Professor of International Affairs, Law and Political Science at Columbia
University, Liberalism in International Relations, In: Bertrand Badie, Dirk Berg-Schlosser, and
Leonardo Morlino, eds., International Encyclopedia of Political Science (Sage, 2011), pp. 14341439)
Relying on new insights from game theory , scholars during the 1980s and 1990s
emphasized that so-called international regimes, consisting of agreed-on international
norms, rules, and decision-making procedures, can help states effectively coordinate
their policies and collaborate in the production of international public goods, such as
free trade, arms control, and environmental protection. Especially, if embedded in formal multilateral institutions, such as
the World Trade Organization (WTO) or North American Free Trade Agreement (NAFT A), regimes crucially improve the availability of information among states in a given

As noted by Robert Keohane,


institutionalized multilateralism also reduces strategic competition over relative gains
and thus further advances international cooperation. Most international regime theorists accepted Kenneth Waltz's (1979)
neorealist assurription of states as black boxes-that is, unitary and rational actors with given interests. Little or no attention was paid to the
impact on international cooperation of domestic political processes and dynamics.
Likewise, regime scholarship largely disregarded the arguably crucial question of
whether prolonged interaction in an institutionalized international setting can
fundamentally change states' interests or preferences over outcomes (as opposed to preferences over
strategies), thus engendering positive feedback loops of increased overall cooperation. For these
issue area, thereby promoting reciprocity and enhancing the reputational costs of noncompliance.

reasons, international regime theory is not, properly speaking, liberal, and the term neoliberal institutionalism frequently used to identify it is somewhat misleading. It is only
over the past decade or so that liberal international relations theorists have begun to systematically study the relationship between domestic politics and institutionalized
international cooperation or global governance. This new scholarship seeks to explain in particular the close interna tional cooperation among liberal democracies as well as
higher-than-average levels of delegation b)' democracies to complex multilateral bodies, such as the \ Liberalism in International Relations 1437 European Union (EU), North

The reasons that


make liberal democracies particularly enthusiastic about international cooperation are
manifold: First, transnational actors such as nongovernmental organizations and private
corporations thrive in liberal democracies, and they frequently advocate increased
international cooperation; second, elected democratic officials rely on delegation to
multilateral bodies such as the WTO or the EU to commit to a stable policy line and to
internationally lock in fragile domestic policies and constitutional arrangements; and
finally, powerful liberal democracies, such as the United States and its allies, voluntarily
bind themselves into complex global governance arrangements to demonstrate strategic
restraint and create incentives for other states to cooperate, thereby reducing the costs
for maintaining international order. Recent scholarship, such as that of Charles Boehmer and colleagues, has also confirmed the classical
Atlantic Treaty Organization (NATO), NAFTA, and the WTO (see, e.g., John Ikenberry, 2001; Helen Milner & Andrew Moravcsik, 2009).

liberal intuition that formal international institutions, such as the United Nations (UN) or NATO, independently contribute to peace, especially when they are endowed with

research on global governance and


especially on the relationship between democracy and international cooperation is
thriving, and it usefully complements liberal scholarship on the democratic peace.
sophisticated administrative structures and information-gathering capacities. In short,

Scenario Planning Good


Bernstein concludes aff---scenario planning is good
Bernstein 2
(Steven, Richard Ned Lebow, Janice Gross Stein and Steven Weber, University of Toronto, The
Ohio State University, University of Toronto and University of California at Berkeley. God Gave
Physics the Easy Problems European Journal of International Relations 2000; 6; 43
One useful alternative approach is the development of scenarios, or narratives with plot
lines that map a set of causes and trends in future time. This forward reasoning strategy is based on a notion of contingent
causal mechanisms, in opposition to the standard, neo-positivist focus on efficient causes, but with no clear parallel
in evolutionary biology. It should not be confused with efforts by some to develop social scientific concepts
directly analogous to evolutionary mechanisms (such as variation or selection) in biology to explain, for example, transfor~ations in the international system or institutions, or conditions for optimum performance in

Scenarios are not predictions; rather, they start with the assumption that the
future is unpredictable and tell alternative stories of how the future may unfold. Scenarios are
generally constructed by distinguishing what we believe is relatively certain from what we
think is uncertain. The most important 'certainties' are common to all scenarios that address the same problem or trend, while the most important perceived uncertainties differentiate one
scenario from another. This approach differs significantly from a forecasting tournament or competition, where advocates of different
theoretical perspectives generate differential perspectives on a single outcome in the hope of subsequently identifYing the 'best' or most accurate performer. Rather, by constructing
scenarios, or plausible stories of paths to the future, we can identifY different driving forces (a term that we prefer to independent variable, since it implies a force
the international political economy.9

pushing in a certain direction rather than what is known on one side of an 'equals' sign) and then attempt to combine these forces in logical chains that generate a range of outcomes, rather than single futures.

Scenarios make contingent claims rather than point predictions. They reinsert a sensible
notion of contingency into theoretical arguments that would otherwise tend toward determinism. Scholars in international relations tend to
privilege arguments that reach back into the past and parse out one or two causal variables that are then posited to be the major driving forces of past and future outcomes. The field also favors variables that are

Forward reasoning undercuts structural


determinism by raising the possibility and plausibility of multiple futures . Scenarios are
impressionistic pictures that build on different combinations of causal variables that may also take on different values in different scenarios. Thus it is
possible to construct scenarios without pre-existing firm proof of theoretical claims that meet
strict positivist standards. The foundation for scenarios is made up of provisional assumptions and causal claims. These become the
subject of revision and updating more than testing. A set of scenarios often contains competing or at least contrasting assumptions. It is less important where people
start, than it is where they end up through frequent revisions, and how they got there. A good scenario is an internally consistent
hypothesis about how the future might unfold; it is a chain of logic that connects 'drivers' to outcomes (Rosell, 1999: 126).
Consider as an example one plausible scenario at the level of a 'global future' where power continues to
shift away from the state and towards international institutions , transnational actors and local communities. The state loses
structural or otherwise parametric, thus downplaying the role of both agency and accident.

its monopoly on the provision of security and basic characteristics of the Westphalian system as we have known it are fundamentally altered. In this setting, key decisions about security, economics and culture will

A detailed scenario about this


transformation would specify the range of changes that are expected to occur and how
they are connected to one another. It would also identify what kinds of evidence might
support the scenario as these or other processes unfold over the next decade, and what
kind of evidence would count against the scenario. This is simply a form of process tracing, or increasing the number of observable
implications of an argument, in future rather than past time. Eventually, as in the heuristics of evolutionary biology, future history becomes data. But
instead of thinking of data as something that can falsify any particular hypothesis, one
should think of it as something capable of distinguishing or selecting the story that was
from the stories that might have been.
be made by non-state actors. Security may become a commodity that can be bought like other commodities in the global marketplace.

One Speech =/= Link


One speech act doesnt cause securitization
Ghughunishvili 10
(Irina, Securitization of Migration in the United States after 9/11: Constructing Muslims and
Arabs as Enemies, Submitted to Central European University Department of International
Relations European Studies In partial fulfillment of the requirements for the degree of Master of
Arts Supervisor: Professor Paul Roe http://www.etd.ceu.hu/2010/ghughunishvili_irina.pdf)
As provided by the Copenhagen School securitization theory is comprised by speech act,
acceptance of the audience and facilitating conditions or other non-securitizing actors contribute to a successful securitization. The causality or a oneway relationship between the speech act, the audience and securitizing actor, where
politicians use the speech act first to justify exceptional measures, has been criticized by
scholars, such as Balzacq. According to him, the one-directional relationship between the three factors, or some of them, is not the best approach. To fully grasp
the dynamics, it will be more beneficial to rather than looking for a one-directional
relationship between some or all of the three factors highlighted, it could be profitable to
focus on the degree of congruence between them. 26 Among other aspects of the Copenhagen Schools theoretical framework,
which he criticizes, the thesis will rely on the criticism of the lack of context and the rejection of a one-way causal relationship between the audience and the actor. The
process of threat construction, according to him, can be clearer if external context,
which stands independently from use of language, can be considered. 27 Balzacq opts for more contextoriented approach when it comes down to securitization through the speech act, where a single speech does not create the
discourse , but it is created through a long process, where context is vital. 28 He indicates: In
reality, the speech act itself, i.e. literally a single security articulation at a particular point
in time, will at best only very rarely explain the entire social process that follows from it .
In most cases a security scholar will rather be confronted with a process of articulations
creating sequentially a threat text which turns sequentially into a securitization. 29 This type of
approach seems more plausible in an empirical study, as it is more likely that a single speech will not be able to
securitize an issue, but it is a lengthy process, where a the audience speaks the same
language as the securitizing actors and can relate to their speeches.

No War Impact 2AC


They have it backwardswars arent a byproduct of western desire for
controlbut ambiguous strategic calculationspreventing shocks is
better than radical critique.
Chandler 9
(David, Westminster IR professor, War Without End(s): Grounding the Discourse of `Global
War', Security Dialogue, 40.3, SAGE)
Western governments appear to portray some of the distinctive characteristics that Schmitt attributed to motorized partisans, in that the shift
from narrowly strategic concepts of security to more abstract concerns reflects the fact
that Western states have tended to fight free-floating and non-strategic wars of
aggression without real enemies at the same time as professing to have the highest
values and the absolute enmity that accompanies these. The government policy documents and critical
frameworks of global war have been so accepted that it is assumed that it is the
strategic interests of Western actors that lie behind the often irrational policy responses,
with global war thereby being understood as merely the extension of instrumental
struggles for control. This perspective seems unable to contemplate the possibility that
it is the lack of a strategic desire for control that drives and defines global war today.
Very few studies of the war on terror start from a study of the Western actors themselves rather
than from their declarations of intent with regard to the international sphere itself. This
methodological framing inevitably makes assumptions about strategic interactions and grounded interests
of domestic or international regulation and control, which are then revealed to explain the
proliferation of enemies and the abstract and metaphysical discourse of the war on
terror (Chandler, 2009a). For its radical critics, the abstract, global discourse merely reveals the
global intent of the hegemonizing designs of biopower or neoliberal empire, as critiques
of liberal projections of power are scaled up from the international to the global. Radical critics working within a broadly
Foucauldian problematic have no problem grounding global war in the needs of neoliberal or biopolitical governance or US hegemonic designs. These critics have
produced numerous frameworks, which seek to assert that global war is somehow inevitable,
based on their view of the needs of late capitalism, late modernity, neoliberalism or
biopolitical frameworks of rule or domination. From the declarations of global war and practices of military intervention, rationality,
instrumentality and strategic interests are read in a variety of ways (Chandler, 2007). Global war is taken very much on its own terms, with the declarations of Western
governments explaining and giving power to radical abstract theories of the global power and regulatory might of the new global order of domination, hegemony or empire

The alternative reading of global war rendered here seeks to clarify that the declarations
of global war are a sign of the lack of political stakes and strategic structuring of the
international sphere rather than frameworks for asserting global domination . We
increasingly see Western diplomatic and military interventions presented as justified on the basis of valuebased declarations, rather than in traditional terms of interest-based outcomes. This was
as apparent in the wars of humanitarian intervention in Bosnia, Somalia and Kosovo where there was no clarity of objectives and
therefore little possibility of strategic planning in terms of the military intervention or the post-conflict political outcomes as it is in the war on terror
campaigns, still ongoing, in Afghanistan and Iraq. There would appear to be a direct
relationship between the lack of strategic clarity shaping and structuring interventions
and the lack of political stakes involved in their outcome. In fact, the globalization of
security discourses seems to reflect the lack of political stakes rather than the urgency

of the security threat or of the intervention. Since the end of the Cold War, the central
problematic could well be grasped as one of withdrawal and the emptying of contestation
from the international sphere rather than as intervention and the contestation for control.
The disengagement of the USA and Russia from sub-Saharan Africa and the Balkans forms the backdrop to the policy debates about sharing responsibility for stability and the

It is the lack of political stakes in the international


sphere that has meant that the latter has become more open to ad hoc and arbitrary
management of failed or failing states (see, for example, Deng et al., 1996).

interventions as states and international institutions use the lack of strategic imperatives to construct
their own meaning through intervention . As Zaki Ladi (1998: 95) explains: war is not waged necessarily to achieve predefined
objectives, and it is in waging war that the motivation needed to continue it is found. In these cases of which there are very many war is no longer a
continuation of politics by other means, as in Clausewitzs classic model but sometimes the initial expression
of forms of activity or organization in search of meaning. . . . War becomes not the ultimate
means to achieve an objective, but the most efficient way of finding one. The lack of
political stakes in the international sphere would appear to be the precondition for the
globalization of security discourses and the ad hoc and often arbitrary decisions to go
to war.

In this sense,

global wars reflect the fact that the international sphere has been

reduced to little more than a vanity mirror for globalized actors who are freed from
strategic necessities and whose concerns are no longer structured in the form of
political struggles against real enemies . The mainstream critical approaches to global
wars, with their heavy reliance on recycling the work of Foucault, Schmitt and Agamben, appear to invert this reality, portraying the
use of military firepower and the implosion of international law as a product of the high
stakes involved in global struggle, rather than the lack of clear contestation involving the
strategic accommodation of diverse powers and interests .

AT: PAN K

Pan is reductionist and the alt fails


Jones 14
David Martin Jones, Professor of Politics at University of Glasgow, PhD from LSE, Australian
Journal of Political Science, February 21, 2014, 49:1, "Managing the China Dream: Communist
Party politics after the Tiananmen incident ", Taylor and Francis Online
Pan discerns an
Orientalist ideology distorting Western commentary on the party state, and especially its international
relations (6). Following Edward Said, Pan claims that such Western Orientalism reveals not something
concrete about the orient, but something about the orientalists themselves, their recurring latent desire
Notwithstanding this Western fascination with China and the positive response of former Marxists, such as Jacques, to the new China,

of fears and fantasies about the orient (16). In order to unmask the limits of Western representations of Chinas rise, Pan employs a critical methodology that draws on
constructivist and deconstructivist approaches (9). Whereas the former questions the underlying dichotomy of reality/knowledge in Western study of Chinas international

Pan
maintains that the two paradigms of China threat and China opportunity in Western
discourse shape Chinas reality for Western China watchers (3). These discourses, Pan claims, are ambivalent (65). He contends that this
relations, the latter shows how paradigmatic representations of China condition the way we give meaning to that country and are socially constitutive of it (9).

bifocal representation of China, like Western discourses of China more generally, tell us a great deal about the west itself, its self -imagination, its torn, anxious, subjectivity, as

This is a large claim. Interestingly, Pan fails to note that after the
Tiananmen incident in 1989, Chinese new left scholarship also embraced Saids critique of Orientalism
in order to reinforce both the party state and a burgeoning sense of Chinese nationalism. To counter Western
liberal discourse, academics associated with the Central Party School promoted an ideology of Occidentalism
to deflect domestic and international pressure to democratise China. In this, they drew not only upon Said,
well as its discursive effects of othering (65).

but also upon Foucault and the post-1968 school of French radical thought that, as Richard Wolin has demonstrated, was itself initiated in an appreciation of Maos cultural
revolution. In other words, the critical and deconstructive methodologies that came to influence American and European social science from the 1980s had a Maoist inspiration

Subsequently, in the changed circumstances of the 1990s, as American sinologist Fewsmith has
shown, young Chinese scholars adopted a variety of postmodernist and critical methodologies
(2008: 125). Paradoxically, these scholars, such as Wang Hui and Zhang Kuan (Wang 2011), had been educated in the
USA and were familiar with fashionable academic criticism of a postmodern and
deconstructionist hue that demythified the West (Fewsmith 2008: 12529). This approach, promulgated in the
academic journal Dushu (Readings), deconstructed, via Said and Foucault, Western narratives about China. Zhang Kuan,
in particular, rejected Enlightenment values and saw postmodern critical theory as a method to build up a national
discourse of resistance and counter Western demands regarding issues such as human
rights and intellectual property. It is through its affinity with this self-strengthening, Occidentalist lens, that
Pans critical study should perhaps be critically read. Simply put, Pan identifies a political
economy of fear and desire that informs and complicates Western foreign policy and , Pan
asserts, tells us more about the Wests self-imagination than it does about Chinese
reality. Pan attempts to sustain this claim via an analysis, in Chapter 5, of the self-fulfilling
prophecy of the China threat, followed, in Chapters 6 and 7, by exposure of the false promises and premises of the China opportunity. Pan certainly offers a
(Wolin 2010: 1218).

provocative insight into Western attitudes to China and their impact on Chinese political thinking. In particular, he demonstrates that Chinas foreign policy-makers react

Pan contends, accurately, that SinoUS


relations are mutually constitutive and the USA must take some responsibility for the rise of China threat (107). This latter
point, however, is one that Australian realists like Owen Harries, whom Pan cites approvingly, have made consistently
negatively to what they view as a hostile American strategy of containment (101). In this context,

since the late 19 90s. In other words, not all Western analysis uncritically endorses the view that

Chinas rise is threatening. Nor is all Western perception of this rise reducible to the
threat scenario advanced by recent US administrations. Pans subsequent argument that the
China opportunity thesis leads to inevitable disappointment and subtly reinforces the China threat paradigm is, also,
somewhat misleading. On the one hand, Pan notes that Western anticipation of Chinas transformation and democratization has become a burgeoning cottage
industry (111). Yet, on the other hand, Pan observes that Western commentators, such as Jacques, demonstrate a growing awareness that the democratisation thesis is a
fantasy. That is, Pan, like Jacques, argues that China will neither democratize nor collapse, but may instead remain politically authoritarian and economically stable at the same

To merge, as Pan does, the democratisation thesis into its authoritarian antithesis in
order to evoke present Western disillusionment (132) with China is somewhat reductionist. Pans
contention that we need a new paradigm shift to free ourselves from the positivist aspiration to grand theory or
transcendental scientific paradigm itself (157) might be admirable, but this will not be achieved by a
constructivism that would ultimately meet with the approval of what Brady terms Chinas thought
managers (Brady: 6).
time (132).

Identity

Debate B4 Identity
Dont take self-understanding as an incontrovertible truth we dont
perfectly know ourselves and have a lot to gain by making our beliefs
subject to external criticism
McBride 3
(Cillian, Professor of Government @ London School of Economics Self-transparency and the
possibility of deliberative politics, Journal of Political Ideologies, 8.3)
ABSTRACT I argue against the notion of self-transparency which underwrites the politics of
presence. This connects situation, identity, and perspective in such a way as to be

incompatible with deliberative politics and treats self-understand- ing as authoritative,


rendering it insensitive to the possibility that our self-under- standings may be
distorted . I propose a hermeneutic, narrative, conception of selfhood on which we relate to our
lives as authors, constructing our identities by employing the linguistic and narrative resources which
our respective situations make available to us. This admits the possibility that others may

provide us with superior interpretations of our lives , which is a precondition of


deliberative politics. Given the possibility that our self-understandings may be
distorted, deliberative citizens have a duty to challenge problematic self-understandings . Anchoring criticism to public deliberation, together with the her- meneutic
premise that a measure of self-opacity is universal, secures such challenges against
the charge of authoritarianism levelled at traditional ideol- ogy-critique. I want to focus here on
the challenge posed to dialogue-centred politics by a particular discourse connecting situation, identity
and politics, which Anne Phillips has termed the politics of presence.1 While apparently
providing a particularly firm basis for arguments for the inclusion of hitherto marginalized
groups within the democratic process, this discourse embodies highly problem- atic views
about political dialogue and the nature of the self. I hope to build on an analysis of these
flaws to clarify the ontological preconditions of deliberative politics and, furthermore, to draw some
conclusions about the nature of the obligations of parties to deliberation. The politics of presence

rests on a model of the self as transparent to itself, but not to others, with the
consequence that a persons self-understanding, at the very least, must be
acknowledged to be authoritative, or incorrigible. This model of the self cannot, however,
be made to cohere with a plausible account of communication, and consequently it
must be discarded in favour of a broadly hermeneutic model of selfhood as situated
in and constituted through a network of language and interpretative traditions. While this self is
inevitably opaque to itself in certain respects, reflection on this opacity lends a point to dialogue which
it cannot have on the assumption of self-transparency. A dialogic politics, which is sensitive to the
possibility of distorted self-understanding and aims at facilitating the transformation of perspectives
and self-interpretations, must acknowledge as a fundamental premise the provisional

character of self-under- standing. This, in turn, provides a basis for viewing deliberative

citizens as having obligations, in certain circumstances, to challenge rather than defer to


the self-understandings of others .

AT: Affirmation of Self


Their deployment of identity/social location/privilege arguments shuts
down effective collective action this reifies racism and leads to endless
squabbling about authenticity
Rob 13
(the Idealist, Carleton College, JD candidate, 10/1, Tim Wise & The Failure of Privilege
Discourse, www.orchestratedpulse.com/2013/10/tim-wise-failure-privilege-discourse/)
I dont find it meaningful to criticize Tim Wise the person and judge whether hes living up to some anti-racist bona fides. Instead, I choose to focus on the paradigm of White

Although the personal is political, not all


politics is personal; we have to attack systems . To paraphrase the urban poet and philosopher Meek Mill: there are
levels to this shit. How I Define Privilege There are power structures that shape individuals lived
experiences. Those structures provide and withhold resources to people based on factors like class, disability status,
privilege upon which his work is based, and its conceptual and practical limitations.

gender, and race. Its not a benefit to receive resources from an unjust order because ultimately, injustice is cannibalistic. Slavery binds the slave, but destroys the master. So,

the point then becomes not to assimilate the underprivileged, but to instead eradicate
the power structures that create the privileges in the first place. The conventional wisdom on privilege often says
that its benefits are unearned. However, this belief ignores the reality and history that privilege is earned and maintained through violence. Systemic advantages are

The
history and modern reality of violence is why Tim Wise comparison between whiteness and
tallness fails. White supremacy is not some natural evolution, nor did it occur by
happenstance. White folks *murdered* people for this thing that we often call White privilege; it was bought and paid for by blood and terror. White
supremacy is not some benign invisible knapsack. The same interplay between violence and advantage is true of any systemic
hierarchy (class, gender, disability, etc). Being tall, irrespective of its advantages, does not follow that pattern of violence. Privilege is Failing Us Unfortunately, I
think our use of the term privilege is no longer a productive way for us to gain a
thorough understanding of systemic injustice, nor is it helping us to develop collective
strategies to dismantle those systems. Basically , I never want to hear the word privilege
allocated and secured as a class, and simply because an individual hasnt personally committed the acts, it does not render their class dominance unearned.

again because the term is so thoroughly misused at this point that it does more harm than
good. Andrea Smith, in the essay The Problem with Privilege, outlines the pitfalls of misapplied privilege theory. Those who had little
privilege did not have to confess and were in the position to be the judge of those who did have privilege. Consequently,
people aspired to be oppressed. Inevitably, those with more privilege would develop new
heretofore unknown forms of oppression from which they suffered Consequently, the goal became not to
actually end oppression but to be as oppressed as possible. These rituals often substituted
confession for political movement-building. Andrea Smith, The Problem with Privilege Dr. Tommy Curry says it more bluntly, Its
not genius to say that in an oppressive society there are benefits to being in the superior
class instead of the inferior one. Thats true in any hierarchy, thats not an aha moment. Conceptually, privilege is best used when narrowly
focused on explaining how structures generally shape experiences. However, when we overly personalize the problem, then
privilege becomes a tit-for-tat exercise in blame, shame, and guilt. In its worst manifestations, this
dynamic becomes oppression Olympics and people tally perceived life advantages and
identities in order to invalidate one another. At best, we treat structural injustice as a personal
problem, and moralizing exercises like privilege confessions inadequately address the
nexus between systemic power and individual behavior. The undoing of privilege occurs
not by individuals confessing their privileges or trying to think themselves into a new subject position, but
through the creation of collective structures that dismantle the systems that enable these privileges. The

activist genealogies that produced this response to racism and settler colonialism were not initially focused on racism as a problem of individual prejudice. Rather, the purpose
was for individuals to recognize how they were shaped by structural forms of oppression. Andrea Smith, The Problem with Privilege Bigger than Tim Wise However, the problem
with White privilege isnt simply that Tim Wise, a white man, can build a career off of Black struggles. As Ive already said, White people need to talk to White people about the
historical and social construction of their racial identities and power, and the foundation for that conversation often comes from past Black theory and political projects. The
problem for me is that

privilege work has become a cottage industry of self-help moralizing that in

no way attacks the systemic ills that create the personal injustices in the first place. A
substantive critique of privilege requires us to get beyond identity politics . Its not about
good people and bad people; its a bad system. Its not just White people that participate in the White privilege industry, although
not everyone equally benefits/profits (see: Tim Wise). Dr. Tommy Curry takes elite Black academics to task for their role in
profiting from the White privilege industry while offering no challenge to White
supremacy. These conversations about White privilege are not conversations about race, and certainly not about racism; its a business where Blacks market
themselves as racial therapists for White people The White privilege discourse became a bourgeois distraction. Its a
tool that we use to morally condemn whites for not supporting the political goals of elite
black academics that take the vantages of white notions of virtue and reformism and
persuade departments, journals, and presses into making concessions for the benefit of a select
species of Black intellectuals in the Ivory Tower, without seeing that the white racial
vantages that these Black intellectuals claim theyre really interested in need to be
dissolved, need to be attacked all the way to the very bottom of American society. Dr. Tommy
Curry, Radio Interview The truth is that a lot of people, marginalized groups included, simply want more
access to existing systems of power. They dont want to challenge and push beyond these systems; they just want to
participate. So if we continue to play identity politics and persist with a personal privilege
view of power, then we will lose the struggle . Barack Obama is president, yet White
supremacy marches on, and often with his help (record deportations, expanded a drone war based on profiling, fought on behalf of US corporations to
repeal a Haitian law that raised the minimum wage). Adolph Reed, writing in 1996, predicted the quagmire of identity politics in the Age of Obama. In Chicago, for instance,
weve gotten a foretaste of the new breed of foundation-hatched black communitarian voices; one of them, a smooth Harvard lawyer with impeccable do-good credentials and
vacuous-to-repressive neoliberal politics, has won a state senate seat on a base mainly in the liberal foundation and development worlds. His fundamentally bootstrap line was
softened by a patina of the rhetoric of authentic community, talk about meeting in kitchens, small-scale solutions to social problems, and the predictable elevation of process
over program the point where identity politics converges with old-fashioned middle-class reform in favoring form over substance. I suspect that his ilk is the wave of the future

Obamas
election and subsequent presidency has made it starkly clear that its not just White
people that can perpetuate White supremacy. Systems of oppression condition all members of society to accept systemic injustice,
and there are (unequal) incentives for both marginalized and dominant groups to perpetuate these structures. Our approaches to injustice must
reflect this reality. This isnt a nave plea for unity, nor am I saying that talking about
identities/experiences is inherently divisive. Many of these privilege discussions use empathy to build personal and collective character, and
there certainly should be space for us to work together to improve/heal ourselves and one another. People will always make mistakes and
our spaces have to be flexible enough to allow for reconciliation. Though we dont have
to work with persistently abusive people who refuse to redirect their behavior, theres a
difference between establishing boundaries and puritanism. Fighting systemic
marginalization and exploitation requires more than good character, and we cannot fetishize personal
in U.S. black politics. Adolph Reed Jr., Class Notes: Posing As Politics and Other Thoughts on the American Scene Although it has always been the case,

morals over collective action .

Você também pode gostar